You are on page 1of 320

t

.me/
Exam_
Sakha_
Off
ici
al
,:carihant
ARIHANT PRAKASHAN (Series), MEERUT
All Rights Reserved

©AUTHOR
No part of this publication may be re-produced, stored in a retrieval system or
by any means, electronic mechanical, photocopying, recording, scanning, web or
otherwise without the written permission of the publisher. Arihant has obtained
all the information in this book from the sources believed to be reliable and true.
However, Arihant or its editors or authors or illustrators don't take any responsibility
for the absolute accuracy of any information published, and the damages or loss
suffered thereupon.
All disputes subject to Meerut (UP) jurisdiction only.

!fi Administrative & Production Offices


Regd. Office
'Ramchhaya' 4577/15, Agarwal Road, Darya Ganj, New Delhi -110002
Tele: 011- 47630600, 43518550
!fi Head Office
Kalindi, TP Nagar, Meerut (UP)- 250002 Tel: 0121-7156203, 7156204
!fi Sales & Support Offices
Agra, Ahmedabad, Bengaluru, Bareilly, Chennai, Delhi, Guwahati,
Hyderabad, Jaipur, Jhansi, Kolkata, Lucknow, Nagpur & Pune.

!fi ISBN : 978-93-25298-66-8


PO No: TXT-XX-XXXXXXX-X-XX
Published by Arihant Publications (India) Ltd.
For further information about the books published by Arihant, log on to
www.arihantbooks.com or e-mail at info@arihantbooks.com

Follow us on O [§] a(@


CHAPTER

01
Indefinite
Integral
Learning Part
Session 1
● Fundamental of Indefinite Integral

Session 2
● Methods of Integration

Session 3
● Some Special Integrals

Session 4
● Integration by Parts

Session 5
● Integration Using Partial Fractions

Session 6
● Indirect and Derived Substitutions

Session 7
● Euler’s Substitution, Reduction Formula and Integration Using Differentiation

Practice Part
● JEE Type Examples
● Chapter Exercises

Arihant on Your Mobile !


Exercises with the #L
symbol can be practised on your mobile. See inside cover page to activate for free.
2 Textbook of Integral Calculus

It was in this aspect that the process of integration was d n +1


y Example 1 If [x + c ] = (n + 1) x n , then find
treated by Leibnitz, the symbol of ∫ being regarded as the dx
∫x
n
initial letter of the word sum, in the same way as the dx .
symbol of differentiation d is the initial letter in the word d
difference. Sol. As, [ x n + 1 + C ] = ( n + 1) x n
dx
⇒ ( x n + 1 + C ) is anti-derivative or integral of (n + 1) x n .
Definition xn + 1
∴ ∫ x dx = +C
n
If f and g are functions of x such that g ′ ( x ) = f ( x ), then n +1
the function g is called a anti-derivative (or primitive
function or simply integral) of f w.r.t. x . It is written d
d y Example 2 If (sin x + c ) = cos x , then find
symbolically, ∫ f ( x ) dx = g ( x ), where, g (x ) = f (x ) dx
dx
∫ cos x dx .
Remarks
d
1. In other words, ∫ f ( x ) dx = g ( x ) iff g ′ ( x ) = f ( x ) Sol. As, (sin x + C ) = cos x
dx
2. ∫ f ( x ) dx = g ( x ) + c,wherecisconstant, ⇒ sin x + C is anti-derivative or integral of cos x.
[Q( g( x ) + C) ′= g ′( x ) = f ( x )] and Cis called constant of integration. ∴ ∫ cos x dx = (sin x ) + C

Session 1
Fundamental of Indefinite Integral
Fundamental of Indefinite Integral (v)
d
( − cos x ) = sin x ⇒ ∫ sin x dx = − cos x + C
d dx
Since, { g ( x )} = f ( x )
dx d
(vi) (sin x ) = cos x ⇒ ∫ cos x dx = sin x + C
⇔ ∫ f ( x ) dx = g ( x ) + C dx
d
(tan x ) = sec 2 x ⇒ ∫ sec x dx = tan x + C
2
Therefore, based upon this definition and various standard (vii)
differentiation formulas, we obtain the following dx
d
integration formulae (viii) ( − cot x ) = cosec 2 x ⇒∫ cosec 2 x dx = − cot x + C
d x n +1 n x n +1 dx
(i)   = x , n ≠− 1 ⇒∫ x n dx = + C, n ≠ − 1 d
dx  n + 1  n +1 (ix) (sec x ) = sec x tan x
dx
d 1 1
(ii) (log | x | ) = ⇒ ∫ dx = log | x | + C , when x ≠ 0 ⇒ ∫ sec x tan x dx = sec x + C
dx x x
d x d
(iii) (e ) = e x ⇒ ∫ e x dx = e x + C (x) ( − cosec x ) = cosec x cot x
dx dx
d  ax  ⇒ ∫ cosec x cot x dx = − cosec x + C
(iv)   = a x , a > 0, a ≠ 1
dx  log e a  d
x
(xi) (log | sin x | ) = cot x
a dx
⇒ ∫a dx = +C
x
log e a ⇒ ∫ cot x dx = log | sin x | + C
Chap 01 Indefinite Integral 3

= ∫ (x + 5x 1/ 2 − x − 1/ 2 ) dx
d 3/2
(xii) ( − log | cos x | ) = tan x
dx
 xn +1 
⇒ ∫ tan x dx = − log | cos x | + C ∫ = + C
n
 using x dx
 n +1 
d
(xiii) (log |sec x + tan x | ) = sec x x 3 / 2 + 1 5x 1/ 2 + 1 x − 1/ 2 + 1
dx = + − +C
3/2 + 1 1/2 + 1 − 1/2 + 1
⇒ ∫ sec x dx = log |sec x + tan x | + C ⇒ I =
2 5/2 2
x + ⋅ 5x 3 / 2 − 2x 1/ 2 + C
d 5 3
(xiv) (log| cosec x − cot x | ) = cosec x (ii) I = ∫ ( x + 5) dx [using (a + b) = a + 3a 2b + 3ab 2 +b 3 ]
2 3 3 3
dx
⇒ ∫ cosec x dx = log | cosec x − cot x | + C I = ∫ (x + 15x + 75x + 125) dx
6 4 2

d  −1 x  1 x7 15x 5 75x 3
(xv)  sin = 2 I= + + + 125x + C
dx  a  a − x2 7 5 3
x7
dx x I = + 3x 5 + 25x 3 + 125x + C
⇒ ∫ = sin − 1   + C 7
a2 − x 2 a

−1
y Example 4 Evaluate
d  −1 x 
(xvi)  cos = 2 dx
dx  a  ∫ tan ∫ sin 2 x cos 2 x
2
a − x2 (i) x dx (ii)
−1 x
⇒ ∫ dx = cos − 1   + C sin 6 x + cos 6 x cos x − cos 2 x
a
a2 − x 2 (iii) ∫ 2
sin x cos x 2
dx (iv) ∫ 1 − cos x
dx
d 1 −1 x  1
(xvii)  tan = 2 Sol. (i) I = ∫ tan x dx ⇒ I = ∫ (sec x − 1) dx
2 2
dx a a  a + x2
x I = ∫ sec x dx − ∫ 1 dx [using ∫ sec xdx = tan x + C ]
2 2
dx 1
⇒ ∫ 2 = tan − 1   + C
a +x 2 a a
⇒ I = tan x − x + C
d 1 −1 x  −1 1
(xviii)  cot = (ii) I = ∫ dx
dx  a a  a2 + x 2 sin x cos 2 x
2

−1 1 x sin 2 x + cos 2 x


⇒ ∫ 2 dx = cot −1   + C I = ∫ dx [Using sin 2 x + cos 2 x = 1]
a +x 2 a a sin 2 x cos 2 x
d 1 −1 x  1 sin 2 x cos 2 x
(xix)  sec
dx  a
=
a  x x 2 − a2
I = ∫ sin 2 x cos 2 x dx + ∫ sin 2 x cos 2 x dx
x I = ∫ sec x dx + ∫ cosec
2 2
dx 1 x dx
⇒ ∫ = sec −1   + C
x x −a
2 2 a a
I = tan x − cot x + C
d 1 −1 x  −1 sin 6 x + cos 6 x
(xx)  cosec
dx  a
=
a x
(iii) I = ∫ sin 2 x cos 2 x
dx
x 2 − a2
(sin 2 x )3 + (cos 2 x )3
− dx 1 x I = ∫ dx
⇒ ∫x x −a
2 2
= cosec −1   + C
a a sin 2 x cos 2 x
[ using (a + b )3 = a 3 + b 3 + 3ab (a + b )]
y Example 3 Evaluate (sin 2 x + cos 2 x )3 −3sin 2 x cos 2 x (sin 2 x + cos 2 x )
x 2 + 5x − 1
I = ∫ sin 2 x cos 2 x
dx
(i) ∫ dx (ii) ∫ (x 2 + 5) 3 dx
x 1 − 3 sin 2 x cos 2 x
x + 5x − 1
2  x2 5x 1 1 
I = ∫ sin 2 x cos 2 x
dx
Sol. (i) I = ∫ dx = ∫  1/ 2 + 1/ 2 − 1/ 2  dx
x x x x 
4 Textbook of Integral Calculus

1 y Example 6 Evaluate
I = ∫ sin 2 x cos 2 x dx − ∫ 3 dx
(i) ∫ 5 log e x
dx (ii) ∫ 2log 4 x dx
(sin x + cos x )
2 2
I = ∫ sin 2 x cos 2 x
dx − 3x + C
Sol. (i) I = ∫ 5log e x
dx = ∫ x log e 5 dx [ Using a log e b = b log c a ]

I = ∫ sec x dx + ∫ cosec 2 x dx − 3x + C x log e 5 + 1


2
= +C
I = tan x − cot x − 3x + C (loge 5 + 1)
cos x − cos 2x x log e 5 + 1
(iv) I = ∫ dx [using cos 2x = 2 cos 2 x − 1.] ∴ ∫5 e dx = +C
log x
1 − cos x loge 5 + 1
cos x − (2 cos 2 x − 1) (ii) I = ∫ 2log 4 x
dx = ∫ 2
log
22
x
dx = ∫ 21/ 2 log 2 x dx
I = ∫ 1 − cos x
dx
 1 
= ∫
− 2 cos 2 x + cos x + 1
dx using logbn x = n logb x

1 − cos x
= ∫ 2log 2 x
dx = ∫ x dx [ using a log a b = b ]
− (2 cos x + 1) (cos x − 1)
⇒I = ∫ dx
− (cos x − 1) x 3/2
= +C
[as −2 cos 2 x + cos x − 1 = −(2 cos x + 1) ⋅ (cos x − 1)] 3/2
⇒ I = ∫ (2 cos x + 1) dx 2 3/2
∴ ∫2 dx = +C
log 4 x
x
3
∴ I = 2 sin x + x + C
( x + 1)( x 2 − x )
Remark y Example 7 Evaluate ∫ x x +x+ x
dx .
In rational algebraic functions if the degree of numerator is
greater than or equal to degree of denominator, then always ( x + 1) ⋅ x ( x 3 / 2 − 1)
divide the numerator by denominator and use the result of Sol. Here, I = ∫ x ( x + x + 1)
dx
integration.
( x + 1)[( x )3 − 13 ]
y Example 5 Evaluate ∴ I = ∫ ( x + x + 1)
dx

x3 x2
(i) ∫ dx (ii) ∫ x 2 + 5 dx ( x + 1)( x − 1)( x + x + 1)
x +2 = ∫ ( x + x + 1)
dx

x3 x3 +8−8 [Using, a 3 − b 3 = (a − b )(a 2 + ab + b 2 )]


Sol. (i) I = ∫ x + 2 dx = ∫ x +2
dx
x2
 ( x 3 + 23 ) 8  = ∫ ( x − 1) dx = − x +C
I = ∫ −  dx 2
 x +2 x + 2
 ( x + 2)( x 2 − 2x + 4 )
y Example 8 Evaluate
8 
⇒ I = ∫  x +2
−  dx
x + 2 1 + 2x 2 x6 −1
(i) ∫ 2 dx (ii) ∫ (x 2 + 1) dx
 8  x (1 + x 2 )
I = ∫  x 2 − 2x + 4 −  dx
 x + 2 1 + 2x 2 1 + x2 + x2
x3
Sol. (i) Here, I = ∫ x 2 (1 + x 2 ) dx = ∫ x 2 (1 + x 2 )
dx
∴ I = − x 2 + 4 x − 8 log | x + 2 | + C
3 1 + x2 x2
x2 x2+5−5  x2 +5 5 
= ∫ x 2 (1 + x 2 ) dx + ∫ x 2 (1 + x 2 ) dx
(ii) I = ∫ 2 dx = ∫ 2 dx = ∫  2 − 2  dx
x +5 x +5  x + 5 x + 5 1 1 1
 5  dx
= ∫ x 2 dx + ∫ 1 + x 2 dx = − x + tan −1 x + C
I = ∫ 1 − 2  dx = x − 5 ∫ 2
 x + 5 x + ( 5 )2 x6 − 1 x6 + 1 − 2
5  x 
(ii) Here, I = ∫ x 2 + 1 dx = ∫ x2 + 1
dx
I =x− tan − 1   + C
5  5
( x 2 ) 3 + 13 2
I = x − 5 tan −1  x  = ∫ x +1
2
dx − ∫ x 2 + 1 dx
  +C
 5
Chap 01 Indefinite Integral 5

( x 2 + 1)( x 4 − x 2 + 1) dx  ( 1 + 2x ) ⋅ ( 4 x 2 − 2x + 1) 4 x 2 ( 2x + 1) 
I = ∫ ( x 2 + 1)
dx − 2∫
x2 + 1
= ∫
 ( 1 − 2x )

( 1 − 2x ) 
 dx

[Using, a 3 + b 3 = (a + b )(a 2 − ab + b 2 )] (2x + 1){ 4 x 2 − 2x + 1 − 4 x 2 } dx


1
=∫ 1 − 2x
= ∫ ( x 4 − x 2 + 1) dx − 2∫ 2 dx
x +1 (2x + 1)(1 − 2x )
=∫ dx
x5 x3 ( 1 − 2x )
= − + x − 2tan −1 x + C
5 3 = ∫ (2x + 1) dx = x 2 + x + C

y Example 9 Evaluate y Example 10 Evaluate


 1 − x −2 2 x −2 − x  1
(i) ∫  1/ 2 − +  dx (i) ∫ dx
x −x
−1 / 2
x 3/ 2 x 1/ 2 − x −1/ 2  sin (x − a) cos (x − b)
1
 x −6 − 64 x2 4x 2 (2 x + 1)  (ii) ∫ dx
(ii) ∫  −1
 4 + 2x + x
−2

4 − 4 x −1 + x −2

1 − 2x 
 dx cos (x − a) cos (x − b)
1
 1 − x −2 2 x −2 − x 
Sol. (i) I = ∫ sin ( x − a ) cos ( x − b ) dx
Sol. (i) Here, I = ∫  1/ 2 −1/ 2
− 3 / 2 + 1/ 2  dx
x − x x x − x −1/ 2  cos (a − b ) dx
cos (a − b ) ∫ sin ( x − a ) cos ( x − b )
I = ⋅
 ( 1 − x −2 ) + ( x −2 − x ) 2 
= ∫ − 3 / 2  dx
 x −x
1/ 2 −1/ 2
x  1 cos {( x − b ) − ( x − a )}
cos (a − b ) ∫ sin ( x − a ) cos ( x − b )
= ⋅ dx
   
 1− x 2  1 − x 2  1  cos ( x − b ) ⋅ cos ( x − a )
= ∫ − 3 / 2  dx = ∫  − 3 / 2  dx = ⋅∫ 
 x − 1 x  x −1 x  cos (a − b )  sin ( x − a ) cos ( x − b )
 x   x  sin ( x − b ) ⋅ sin ( x − a )
+  dx
= ∫ ( − x − 2x −3 / 2 ) dx sin ( x − a ) cos ( x − b ) 
1
 x 3/2
= − − 2⋅
x −1/ 2  2
 + C = − x 3/2 +
4
+C
=
cos (a − b ) ∫ {cot ( x − a ) + tan ( x − b )} dx
 3 / 2 − 1 / 2  3 x 1
= {log | sin ( x − a ) | − log| cos( x − b ) |} + C
(ii) Here cos (a − b )
 x −6 − 64 x2 4 x 2 ( 2x + 1)  1  sin ( x − a ) 
I = ∫ −1 −2
⋅ −  dx = loge  + C
 4 + 2x + x 4 − 4 x −1 + x −2 1 − 2x  cos (a − b )  cos ( x − b ) 
 1 − 64 x 6  1
 6 x 2
4 x 2
( 2 x + 1 )
 (ii) I = ∫ cos ( x − a ) cos ( x − b ) dx
= ∫ 2 x ⋅ 2 −  dx
1 sin (a − b )
 4 x + 2x + 1 4 x − 4 x + 1 ( 1 − 2x ) 
sin (a − b ) ∫ cos ( x − a ) cos ( x − b )
= dx
 
 x2 x2 
1 sin {( x − b ) − ( x − a )}
sin (a − b ) ∫ cos ( x − a ) cos ( x − b )
 1 − ( 4 x 2 )3 x6 4 x 2 ( 2x + 1)  = dx
= ∫ 6 ⋅ −  dx
 x ⋅ ( 4 x + 2x + 1) ( 4 x − 4 x + 1)
2 2
( 1 − 2x ) 
1  sin ( x − b ) cos ( x − a )
 (1 − 4 x )(1 + 4 x + 16x )
2 2
4 x ( 2x + 1) 
4 2
= ∫ 
sin (a − b )  cos ( x − a ) cos ( x − b )
= ∫ −  dx
 ( 4 x 2
+ 2 x + 1 )( 4 x 2
− 4 x + 1 ) ( 1 − 2x )  cos ( x − b ) sin ( x − a ) 
−  dx
 (1 − 4 x ) ⋅ ( 4 x + 2x + 1)( 4 x − 2x + 1) 4 x (2x + 1)
2 2 2 2 cos ( x − a ) cos ( x − b )
= ∫ −  dx 1
 ( 4 x 2 + 2x + 1)(2x − 1)2 ( 1 − 2x )  =
sin (a − b ) ∫ {tan ( x − b ) − tan ( x − a )} dx
[using, 16x 4 + 4 x 2 + 1 = 16x 4 + 8x 2 + 1 − 4 x 2 =
1
[ − log| cos( x − b )| + log | cos( x − a ) | ] + C
sin (a − b )
= ( 4 x 2 + 1) 2 − ( 2x ) 2 = ( 4 x 2 + 1 + 2x ) ]
1   cos ( x − a ) 
 (1 − 2x )(1 + 2x )( 4 x 2 − 2x + 1) 4 x 2 (2x + 1) = log cos ( x − b )  + C
= ∫ −  dx sin (a − b )
 ( 1 − 2x ) 2 ( 1 − 2x )    
6 Textbook of Integral Calculus

sin ( x + a )
y Example 11 Evaluate ∫ sin ( x + b ) dx. Important Points Related
sin ( x + a ) to Integration
Sol. Let I = ∫ sin ( x + b ) dx . Put x + b = t ⇒ dx = dt
1. ∫ k f ( x ) dx = k ∫ f ( x ) dx , where k is constant. i.e. the
sin (t − b + a )
∴ I =∫ dt integral of the product of a constant and a
sin t function = the constant × integral of the function
 sin t cos (a − b ) cos t sin (a − b )
= ∫ +  dt 2. ∫ { f 1 ( x ) ± f 2 ( x ) ±… ± f n ( x ) } dx
 sin t sin t 
= cos (a − b ) ∫ 1dt + sin (a − b ) ∫ cot (t ) dt = ∫ f 1 ( x ) dx ± ∫ f 2 ( x ) dx ± … ± ∫ f n ( x ) dx .

= t cos (a − b ) + sin (a − b )log | sin t | + C i.e. the integral of the sum or difference of a finite
= ( x + b ) cos (a − b ) + sin (a − b ) log| sin ( x + b )| +C number of functions is equal to the sum or difference
of the integrals of the various functions.
y Example 12 3. Geometrical interpretation of constant of
x 2 5
(i) If f ′ (x ) =
+ and f (1) = , then find f (x ). integration By adding C means the graph of
2 x 4 function would shift in upward or downward
dy 3 direction along y-axis as C is +ve or − ve respectively.
(ii) The gradient of the curve is given by = 2x − 2 .
dx x x2
The curve passes through (1, 2) find its equation. e.g. y = ∫ x dx = +C
2
x 2
(i) Given, f ′ ( x ) = + ,
2 x x2+C3
On integrating both sides w.r.t. x, x2+C2
 x 2 x2+C1
we get ∫ f ′ ( x ) dx = ∫  +  dx
 2 x
1 x2
⇒ f (x ) = ⋅ + 2 log | x | + c …(i)
2 2 O (x,0)
5
Now, as f (1) = (called as initial value problem Figure. 1.1
4
5
i.e. when x = 1,y = or f (1) = )
5 ∴ y = ∫ f ( x ) dx = F ( x ) + C
4 4
Putting, x = 1 in Eq. (i), ⇒ F ′ ( x ) = f ( x ); F ′ ( x 1 ) = f ( x 1 )
1
f (1) = + 2 log | 1 | + C , but f (1) =
5 Hence, y = ∫ f ( x ) dx denotes a family of curves such
4 4
5 1
that the slope of the tangent at x = x 1 on every
∴ = +C ⇒ C =1 member is same i.e. F ′ ( x 1 ) = f ( x ) [when x 1 lies in
4 4
the domain of f ( x )]
x2
⇒ f (x ) =
+ 2 log | x | + 1 Hence, anti-derivative of a function is not unique. If
4
g 1 ( x ) and g 2 ( x ) are two anti-derivatives of a function
dy 3  3
(ii) Given, = 2x − 2 or dy = 2x − 2  dx , f ( x ) on [a, b ], then they differ only by a constant.
dx x  x 
i.e. g1 (x ) − g2 (x ) = C
On integrating both sides w.r.t. x, we get
 3 Anti-derivative of a continuous function is
∫ dy = ∫ 2x − x 2  dx differentiable.
2x 2 3 4. If f ( x ) is continuous, then
⇒ y= + +C
2 x ∫ f ( x ) dx = F ( x ) + C
Since, curve passes through (1, 1).
⇒ F ′( x ) = f ( x )
⇒ 1 = 1 + 3 + C ⇒C = − 3
⇒ always exists and is continuous.
3
∴ f (x ) = x 2 + − 3 ⇒ F ′( x )
x
Chap 01 Indefinite Integral 7

5. If integral is discontinuous at x = x 1 , then its anti-derivative at x = x 1 need not be discontinuous.


3
e.g. ∫ x −1 / 3 dx . Here, x −1 / 3 is discontinuous at x = 0. But ∫ x −1 / 3 dx = x 2 / 3 + C is continuous at x = 0.
2
6. Anti-derivative of a periodic function need not be a periodic function. e.g. f ( x ) = cos x + 1 is periodic but
∫ ( cos x + 1) dx = sin x + x + C is a periodic.
Daily Life Applications
The Derivative The Integral
Function Its derivative function In symbols Function It’s Anti- In symbols
derivative Function
ds
Distance (s) Velocity (v ) v= Velocity Distance s = ∫ v (t ) dt
dt
Velocity (v ) Acceleration (a) a=
dv Acceleration Velocity v = ∫ a(t ) dt
dt
Mass (µ ) Liner Density (ρ ) dµ Linear Density Mass µ = ∫ ρ (x ) dx
ρ=
dx
Population (P ) Instantaneous growth dP Instantaneous Growth Population  dP 
ρ = ∫   dt
dt  dt 

Cost (C ) Marginal cost (MC) MC =


dC Marginal Cost Cost  dC 
C (q)= ∫   dq
dq  dq 

Revenue (R ) Marginal Revenue (MR) MR =


dR Marginal Revenue Revenue  dR 
R (q) = ∫   dq
dq  dq 

Here, q is quantity of products.

Exercise for Session 1


n
Evaluate the following integration
dx x2 + 3
1. ∫ 2. ∫ dx
x + 1− x x ( x 2 + 1)
6

(1 + x )2 4
3. ∫ dx 4. ∫ x 2 dx
x (1 + x 2 ) 1+ x
x4 + x 2 + 1 2
+ sin2 x )sec2 x
5. ∫ dx 6. ∫ ( x dx
2(1 + x 2 ) (1 + x 2 )
2
7. ∫ x 2 dx 8. ∫ 2x ⋅ e x ⋅ dx
(a + bx )
3x
+ e 5x
9. ∫ e x dx 10. ∫ (e alog x + e x log a ) dx
e + e −x

11. ∫ 1 + cos 4x dx 12. ∫ tan x tan 2x tan 3x dx


cot x − tan x

13. ∫ sin 4x dx 14. ∫ cos 3 x dx


sin x

15. ∫ sin3 x cos 3 x dx


Session 2
Methods of Integration
Methods of Integration we substitute g ( x ) = t and g ′ ( x ) dx = dt
If the integral is not a derivative of a simple function, then The substitution reduces the integral to ∫ f (t ) dt . After
the corresponding integrals cannot be found directly. In evaluating this integral we substitute back the value of t.
order to find the integral of complex problems.
sin x cos x 1 y Example 13 Prove that
e.g. ∫ dx , ∫ dx , ∫ dx
x x log x (ax + b )n +1
∫ + = + C , n ≠ 1.
n
(ax b ) dx
(n + 1)a
Sol. Putting, ax + b = t , we get
Some Integrals adx = dt or dx = dt
1

which Cannot be Found a


dt 1 t n +1
Any function continuous on an interval (a, b ) has an ∴ I = ∫ (ax + b )n dx = ∫ t n ⋅ = ⋅ +C
anti-derivative in that interval. In other words, there exists a a n +1
a function F ( x ) such that F ′ ( x ) = f ( x ). 1 (ax + b )n + 1dx
= + C.
However, not every anti-derivative F ( x ), even when it a(n + 1)
exists, is expressible in closed form in terms of elementary Remarks
functions such as polynomials, trigonometric, logarithmic, 1. If ∫ f ( x ) dx = g ( x ) + C, then ∫ f ( ax + b) dx =
1
g ( ax + b) + C
exponential functions etc. Then, we say that such a
1 1 1
anti-derivatives or integrals “cannot be found’’. 2. If ∫ dx = log| x | + C, then ∫ dx = log| ax + b| + C
x ax + b a
Some typical examples are
Thus, in any fundamental integral formulae given in article
sin x cos x
(i) ∫ dx (ii) ∫ dx fundamental integration formulae if in place of x we have
x x ( ax + b), then same formula is applicable but we must divide by
coefficient of x or derivative of (ax + b) i.e. a.
1
(iii) ∫ dx (iv) ∫ 1 − k 2 sin2 x dx
log x Here is the list of some of
∫ sin x dx (vi) ∫ sin ( x ) dx frequently used formulae
2
(v)
(ax + b ) n + 1
(vii) ∫ cos ( x 2 ) dx ∫ x tan x dx ∫ (ax + b ) dx = + C, n ≠ − 1
n
(viii) (i)
a (n + 1)
(ix) ∫ e − x dx (x) ∫ e x dx
2 2
1 1
(ii) ∫ ax + b dx = a log | ax + b | + C
x2
∫ 1 + x 5 dx ∫ 1 + x 2 dx
3
(xi) (xii) 1 ax + b
∫e
ax + b
(iii) dx = e +C
a
(xiii) ∫ 1 + x 3 dx etc. 1 a bx + c
∫ a dx =
bx + c
(iv) ⋅ +C
b log a
1
Integration by Substitution (v) ∫ sin (ax + b ) dx = − cos (ax + b ) + C
a
(or by change of the independent variable) 1
(vi) ∫ cos (ax + b ) dx = sin (ax + b ) + C
If g ( x ) is a continuously differentiable function, then to a
evaluate integrals of the form, 1
(vii) ∫ sec (ax + b ) dx = tan (ax + b ) + C
2
I= ∫ f ( g ( x )) . g ′ ( x ) dx , a
Chap 01 Indefinite Integral 9

1  2
∫ cosec (ax + b ) dx = − cot (ax + b ) + C (iii) Here, I = ∫ (7 x − 2) 3x + 2 dx = 7 ∫  x −  3x + 2 dx
2
(viii)
a  7
1 7  6
(ix) ∫ sec (ax + b ) tan (ax + b ) dx = a sec (ax + b ) + C = ∫ 3x −  3x + 2 dx
3  7
1
(x) ∫ cosec (ax + b ) cot (ax + b ) dx = − a cosec (ax + b ) +C =
7 
3∫
6
3x + 2 − 2 −  3x + 2 dx
7
1
(xi) ∫ tan (ax + b ) dx = − a log | cos (ax + b ) | + C 7 20
= ∫ (3x + 2) 3x + 2 dx − ∫ 3x + 2 dx
3 3
1
(xii) ∫ cot (ax + b ) dx = a log | sin (ax + b ) | + C 7 20
= ∫ (3x + 2) dx − ∫ 3x + 2 dx
3/2
3 3
1
(xiii) ∫ sec (ax + b )dx = a log | sec (ax + b ) + tan (ax + b )|+C    
7  (3x + 2)5 / 2  20  (3x + 2)3 / 2 
1 =   −   +C
(xiv) ∫ cosec (ax + b ) dx = a log | cosec (ax + b ) − cot(ax + b )|+C 3 5 ×3 
 2 
3  3 ×3 
 2 
y Example 14 Evaluate 14 40
= ( 3x + 2) 5 / 2 − ( 3x + 2) 3 / 2 + C
1 8x + 13 45 27
(i) ∫ dx (ii) ∫ dx . (2 + 3x 2 ) dx 2 + 2x 2 + x 2
3x + 4 − 3x + 1 4x + 7 (iv) Here, I = ∫ = ∫ dx
x 2 (1 + x 2 ) x 2 (1 + x 2 )
2 + 3x 2
(iii) ∫ (7 x − 2) 3 x + 2 dx . (iv) ∫ x 2 (1 + x 2 ) dx .  2(1 + x 2 ) x2 
= ∫ 2 +  dx
 x (1 + x ) x (1 + x )
2 2 2

dx
Sol. (i) Here, I = ∫ 3x + 4 − 3x + 1
 2
= ∫ 2 +
1 
 dx = 2∫ x dx + ∫
−2 1
dx
x 1 + x2 1 + x2
( 3 x + 4 + 3x + 1 )
= ∫( 3x + 4 − 3x + 1 ) ( 3x + 4 + 3x + 1 )
dx
= 2⋅
x −1
+ tan −1 x + C = −
−2
+ tan −1 x + C
−1 x
( 3 x + 4 + 3x + 1 )
= ∫ ( 3x + 4 ) − ( 3x + 1)
dx
y Example 15 Evaluate
1 1 sin (log x )  3 sin x + 4 cos x 
=
3 ∫ 3x + 4 dx +
3 ∫ 3x + 1 dx
(i) ∫ dx (ii) ∫   dx
x 4 sin x − 3 cos x 
1  ( 3x + 4 ) 3 / 2  1  ( 3x + 1) 3 / 2  m tan −1 x
=  +   +C e
∫ ∫ x sin (4x + 7) dx
2
3  3/2 × 3  3 3/2 × 3  (iii)
1+x2
dx (iv)
(ax + b )n +1
∫ (ax + b ) dx = +C
n sin (log x )
Using,
( n + 1) a Sol. (i) I = ∫ x
dx
2
=
[(3x + 4 )3 / 2 + (3x + 1)3 / 2 ] + C d 1
27 We know that, (log x ) =
dx x
8x + 13 8x + 14 − 1
(ii) Here, I = ∫ dx = ∫ dx Thus, let log x = t
4x + 7 4x + 7
1
⇒ dx = dt …(i)
2 ( 4 x + 7 ) −1
= ∫ 4 x + 7 dx x
∴ I = ∫ sin (t ) dt = − cos (t ) + C
4x + 7 dx
=2∫ dx − ∫ = − cos (log x ) + C [using Eq. (i)]
4x + 7 4x + 7
3 sin x + 4 cos x
(ii) I = ∫ dx
= 2 ∫ ( 4 x + 7 )dx − ∫ ( 4 x + 7 )−1/ 2dx 4 sin x − 3 cos x
 ( 4 x + 7 )3 / 2   ( 4 x + 7 )1/ 2  We know,
d
( 4 sin x − 3 cos x ) = ( 4 cos x + 3 sin x )
=2  −  +C dx
 3/2 × 4   1/2× 4 
Thus, let 4 sin x − 3 cos x = t …(i)
1 1
= ( 4 x + 7 )3 / 2 − ( 4 x + 7 )1/ 2 + C ⇒ ( 4 cos x + 3 sin x ) dx = dt
3 2
10 Textbook of Integral Calculus

dt 1
∴ I = ∫
t
= log | t | + C =
4
{2 cos x cos 5x + 2 cos 3x cos 5x}

= log | 4 sin x − 3 cos x | + C [using Eq. (i)] 1


= {(cos 4 x + cos 6x ) + (cos 2x + cos 8x )}
m tan − 1 x 4
e
(iii) I = ∫ 1 + x2
dx Let m tan − 1 x = t 1
∴ cos x cos 2x cos 5x = {cos2x + cos 4x + cos6x + cos8x}
4
m 1 1
⇒ dx = dt ⇒ dx = dt ∴ I = ∫ (cos x cos 2x cos 5x ) dx
1+ x 2
1+ x 2
m
1
4 ∫
dt 1 = (cos 2x + cos 4 x + cos 6x + cos 8x ) dx
∴ I = ∫e ⋅ ⇒ I =
m ∫
t
e t dt
m
sin 2x sin 4 x sin 6x sin 8x
1 1 −1
I = e t + C = e m tan x + C = + + + +C
m m 8 16 24 32
(iv) I = ∫ x sin ( 4 x + 7 ) dx
2
y Example 17 Evaluate
1
Let 4 x 2 + 7 = t ⇒ 8x dx = dt ⇒ x dx = dt (i) ∫ sin x cos x ⋅ cos 2 x ⋅ cos 4x dx
8
dt 1 1 − tan 2 x 1 + cos 2 x
∴ I = ∫ sin (t ) = − cos (t ) + C
8 8 (ii) ∫ 1 + tan 2 x dx (iii) ∫ 1 + cos 2x dx
1
= − cos ( 4 x 2 + 7 ) + C
cos 2 x sec 2 x − 1
∫ cos 2 x sin 2 x dx ∫ sec 2x + 1 dx
8
(iv) (v)
Remarks
While solving product of trignometric, it is expedient to use the Sol. (i) Here, I = ∫ sinx cos x ⋅ cos 2x cos 4 x dx
following trigonometric identities
1 1
2∫
1. sin mx cos nx = {sin ( m − n) x + sin ( m + n) x } = 2sin x ⋅ cos x ⋅ cos 2x ⋅ cos 4 x dx
2
1 [using, sin 2x = 2sin x ⋅ cos x ]
2. cos mx sin nx = {sin ( m + n) x − sin ( m − n) x }
1
2 × 2∫
2 = 2sin 2x ⋅ cos 2x ⋅ cos 4 x ⋅ dx
1
3. sin mx sin nx = {cos ( m − n) x − cos ( m + n) x }
2
1 1
4∫ 2× 4 ∫
1
4. cos mx cos nx = {cos ( m − n) x + cos ( m + n) x } = sin 4 x ⋅ cos 4 xdx = 2sin 4 x cos 4 x dx
2
1 − cos 8x
= ∫ sin 8x dx = +C
y Example 16 Evaluate 8 64
(i) ∫ cos 4x cos 7 x dx (ii) ∫ cos x cos 2 x cos 5 x dx 1 − tan 2 x
(ii) Here, I = ∫ 1 + tan 2 x dx
Sol. When calculating such integrals it is advisable to use the
trigonometric product formulae. 1 − tan 2 x
∴ I = ∫ cos 2x dx Using, cos 2x =
1 + tan 2 x
(i) ∫ cos 4 x cos 7 x dx sin 2x
1 = +c
Here, cos 4 x cos 7 x = (cos 3x + cos 11x ), 2
2 1 + cos 2 x
(iii) Here, I = ∫ dx ,
1
[using cos mx ⋅ cos nx = {cos(m − n )x + cos(m + n )x ] 1 + cos 2x
2
1 + cos 2 x 1 + cos 2 x
1
∴ I = ∫ cos 4 x cos 7 x dx = ∫ (cos 3x + cos 11x ) dx
= ∫ 1 + 2 cos 2 x − 1 dx = ∫2 cos 2 x
dx
2
[Using, cos 2x = 2 cos 2 x − 1]
1 1
= ∫ cos 3x dx + ∫ cos 11x dx 1 1
2
sin 3x sin 11x
2
2 ∫=(sec 2 x + 1) dx = (tan x + x ) + C
2
= + +C
6 22 cos 2x
(iv) Here, I = ∫ dx
(ii) ∫ cos x cos 2x cos 5x dx , cos 2 x ⋅ sin 2 x
1 (cos 2 x − sin 2 x ) dx
We have (cos x cos 2x )cos 5x = (cos x + cos 3x )cos 5x ,
2 I = ∫ cos 2 x ⋅ sin 2 x
1
[using cos mx .cos nx = {cos(m − n )x + cos(m + n )x ] [Using, cos 2x = cos 2 x − sin 2 x ]
2
Chap 01 Indefinite Integral 11

 1 1   cot 2 2x − 1 
= ∫ 2 −  dx Sol. Here, I = ∫  − cos 8x ⋅ cot 4 x  dx
 sin x cos 2 x   2 cot 2x 
= ∫ ( cosec 2 x − sec 2 x ) dx = − cot x − tan x + C I = ∫ (cot 4 x − cos 8x ⋅ cot 4 x ) dx
sec 2x − 1 1 − cos 2x  cot 2 A − 1
(v) Here, I = ∫ sec 2x + 1 ⋅ dx = ∫ 1 + cos 2x dx, using cot 2A = 
 2 cot A 
2sin 2 x
We get, I = ∫ 2 cos 2 x dx = ∫ cot 4 x (1 − cos 8x ) dx
[using 1 − cos 2A = 2sin 2 A ]
[using, 1 − cos 2x = 2sin 2 x and 1 + cos 2x = 2 cos 2 x]
= ∫ cot 4 x ⋅ 2sin 2 ( 4 x ) dx
= ∫ tan x ⋅ dx
2
cos 4 x
= ∫ sin 4 x ⋅ 2sin
2
4 x dx
As, tan x = sec 2 x − 1
2

∴ I = ∫ (sec 2 x − 1) dx = tan x − x + C = ∫ 2sin 4 x ⋅ cos 4 xdx,

y Example 18 Evaluate using sin 2A = 2sin A cos A


 cot 2 2x − 1  − cos 8x
= ∫ sin 8x dx = +C
I= ∫ 
 2 cot 2x
− cos 8 x ⋅ cot 4 x  dx .

8

Exercise for Session 2


n
Solve the following integration

1. ∫ dx 2. ∫ cos x − sin x ⋅ (2 + 2 sin 2x ) dx


1 + sin x cos x + sin x

3. ∫ (3 sin x cos 2 x − sin3 x ) dx 4. ∫ cos x ° dx

5. ∫ sin x + cos x ⋅ dx , here (sin x + cos x ) > 0 6. ∫ cos 2x − cos 2α


⋅ dx
1 + sin 2x cos x − cos α

7. ∫ sin 2x + cos2 x
3 3
dx 8. ∫ sec2 x ⋅ cosec2x dx
sin x ⋅ cos x

10. ∫ sin 2x + cos2 x


6 6
9. ∫ 1 − sin 2x dx dx
sin x ⋅ cos x

11. ∫ sin2  9π +


x 2  7π
 − sin 
x 
+   dx 12. ∫ cos 4x + 1 dx
 8 4  8 4 cot x − tan x

13. ∫ sin α ⋅ sin ( x x 


− α ) + sin2  − α   dx
2 
14. ∫ sin 2x + sin 5x − sin 3x
dx
 cos x + 1 − 2 sin2 2x

15. ∫ cos x + sin x


4 4
dx , here cos 2x > 0
1 + cos 4x
Session 3
Some Special Integrals
Some Special Integrals Type I
x dx dx
(i)
dx
∫ x 2 + a2 =
1
a
tan − 1   + C
a
(i) ∫ ax 2 + bx + c (ii) ∫
ax + bx + c
2

(ii)
dx
∫ x 2 − a2 =
1 x −a
log + C (iii) ∫ ax 2 + bx + c dx
2a x +a If ax 2 + bx + c can be factorised, then the integration is
dx 1 a + x
(iii) ∫ a2 − x 2 = log + C easily done by the method of partial fractions (explained
2a a − x later). If the denominator cannot be factorised, then
express it as the sum or difference of two squares by the
dx x
(iv) ∫ a −x
2 2
= sin − 1   + C
a
method of completing the square
 b c   b   c  b 2 
2
dx ax 2 + bx + c = a  x 2 + x +  = a  x +  +   − 2 
(v) ∫ a +x
2 2
= log | x + x 2 + a 2 | + C  a a   2a   a  4a 
b
dx make the substitution x + = t.
(vi) ∫ x −a
2 2
= log | x + x 2 − a 2 | + C 2a

y Example 19 Evaluate
1 1 x
∫ a − x dx = x a 2 − x 2 + a 2 sin − 1   + C
2 2 1 1
(vii)
2 2 a (i) ∫ 2 dx (ii) ∫ 2 dx
x − x +1 2x + x − 1
(viii) ∫ a 2 + x 2 dx 1
(iii) ∫ dx (iv) ∫ 2 x 2 − 3 x + 1 dx
1 1 x − 2x + 3
2
= x a 2 + x 2 + a 2 log | x + a 2 + x 2 | + C
2 2
dx
(ix) ∫ x 2 − a 2 dx Sol. (i) I = ∫ x 2 − x + 1,
1 1 completing x 2 − x + 1 into perfect square.
= x x 2 − a 2 − a 2 log | x + x 2 − a 2 | + C
2 2 dx dx
= ∫ x 2 − x + 1 / 4 − 1 / 4 + 1 = ∫ ( x − 1 / 2) 2 + 3 / 4
Some Important Substitutions
dx
Expression Substitution ∴I = ∫ ( x − 1 / 2) 2 + ( 3 / 2) 2
a2 + x 2 x = a tan θ or a cot θ
1  x − 1 / 2
a2 − x 2 x = a sin θ or a cos θ = tan − 1   +C
3 /2  3 /2 
x 2 − a2 x = a sec θ or a cosec θ
2  2x − 1
a−x a+ x ∴I = tan − 1   +C
or x = a cos 2 θ 3  3 
a+ x a−x
 1 1 x 
x −α
or (x − α ) (x − β )
x = α cos 2 θ + β sin 2 θ using ∫ 2 dx = tan − 1   + C 
 
β−x  x + a 2
a a 
1 1 1
(ii) I = ∫ 2x 2 + x − 1 dx = 2 ∫ x 2 + x /2 − 1 / 2 dx
Application of these Formulae 1 1
The above standard integrals are very important. Given =
2 ∫ x 2 + x /2 + 1 / 16 − 1 / 16 − 1 / 2 dx
below are integrals which are applications of these.
Chap 01 Indefinite Integral 13

1 dx 1 dx − t dt
2 ∫ ( x + 1 / 4 )2 − 9 / 16 2 ∫ ( x + 1 / 4 )2 − (3 / 4 )2
= = ⇒ dx = − ,
e 2x
t dt 1 t
=
1

1  x + 1/ 4 − 3/ 4
log + C dx =
t2 − 1
= ∫ t 2

t2 − 1
dt
2 2 (3 / 4 )  x + 1/ 4 + 3/ 4
dt 1  t − 1

using ∫ 2
dx
=
1
log + C
 x − a

I = ∫ t 2 − 1 = 2 ⋅ 1 log t + 1 + C
x −a  x + a
2
 2a 
1  1 − e 2x − 1
1  x − 1 / 2 = log + C
= log + C
2  1 − e + 1
2x
3  x +1 
 2x − 1  2x
∴ I =
1
log + C (ii) Here, I = ∫ 1 − x2 − x4
dx
3  2 ( x + 1) 
dx dx Let, x 2 = t , 2x dx = dt
(iii) I = ∫ x − 2x + 3
2
= ∫ x − 2x + 1 − 1 + 3
2
dt dt
dx
∴ I = ∫ 1 − t − t2
= ∫
1 1
1 + − − t − t2
= ∫ ( x − 1) 2 + ( 2 ) 2 4 4
dt dt
= log | ( x − 1) + ( x − 1)2 + ( 2 )2 | I = ∫ 5 / 4 − ( t + 1 / 2) 2
=∫
( 5 / 2) − ( t + 1 / 2) 2
2

 
− 1  2x + 1
dx  t + 1 / 2
2
using ∫ = log | x + x 2 + a 2 | + C  = sin − 1   + C = sin   +C
 x +a
2 2
  5 /2   5 
∴ I = log | ( x − 1) + x 2 − 2x + 3 | + C ax
(iii) Here, I = ∫ 1−a 2x
dx . Let, a x = t
(iv) I = ∫ ( 2x 2 − 3x + 1 ) dx = 2 ∫ ( x 2 − 3x /2 + 1 / 2 ) dx
dt
∴ a x log a dx = dt , a x dx =
= 2 ∫ ( x − 3x /2 + 9 / 16 − 9 / 16 + 1 / 2 ) dx
2 log a
1 dt 1
= 2 ∫ ( ( x − 3 / 4 )2 − 1 / 16 ) dx ∴ I = ∫ 1−t

2 log a
=
log a
⋅ sin − 1 (t ) + C

1  1
( x − 3/ 4 ) ( x − 3/ 4 )2 − 1/ 16 I = sin − 1 (a x ) + C
2  log a
= 2  +C
 − 1 log | ( x − 3/ 4 ) + ( x − 3/ 4 )2 − 1/ 16 x x 1/ 2 dx
 16 × 2  (iv) Here, I = ∫ a3 − x 3
dx = ∫ (a 3 / 2 )2 − ( x 3 / 2 )2
1 
( 4 x − 3) x 2 − 3x / 2 + 1 / 2
8  Let, x 3/2 = t,
3 1/ 2 2
x dx = dt , x 1/ 2 dx = dt
= 2  +C 2 3
− 1
log | ( x − 3 / 4 ) + x − 3x /2 + 1 / 2
2
2
 32  dt
=∫ 3
y Example 20 Evaluate (a 3 / 2 )2 − (t )2
1 2x −1 x
3/2 
(i) ∫ dx (ii) ∫ dx =
2
⋅ sin − 1
 t  2
 3 / 2  + C = sin  3 / 2  + C
1 − e 2x 1−x −x 2 4
3 a  3 a 
x
a x
(iii) ∫ 1 − a 2x
dx (iv) ∫ a −x3
3
dx y Example 21 Evaluate
cos x sin (x − α)
(i) ∫ dx . (ii) ∫ dx .
dx sin x − 2 sin x − 3
2 sin (x + α)
Sol. (i) Here, I = ∫ 1 − e 2x
2 sin 2 x − cos x
Let, 1 − e 2x = t 2 (iii) ∫ 6 − cos 2 x − 4 sin x dx .
Then, −2e 2 x dx = 2t dt
14 Textbook of Integral Calculus

cos x dx ( 4t − 1) dt
Sol. (i) Let I = ∫ sin x − 2 sin x − 3
2
∴ I = ∫ ( t 2 − 4t + 5) …(i)

Put sin x = t ∴ cos x dx = dt Now, let ( 4t − 1) = λ (2t − 4 ) + µ


dt dt Comparing coefficients of like powers of t, we get
⇒ I = ∫ t 2 − 2t − 3
= ∫ t 2 − 2t + 1 − 1 − 3 2λ = 4, − 4 λ + µ = − 1
dt ⇒ λ = 2, µ = 7 …(ii)
I = ∫ ( t − 1) 2 − ( 2) 2
2 (2t − 4 ) + 7
∴I = ∫ 2 dt [using Eqs. (i) and (ii)]
t − 4t + 5
= log | (t − 1) + (t − 1)2 − (2)2 | + C
2t − 4 dt
=2∫ dt + 7 ∫ t 2 − 4t + 5
∴ I = log | (sin x − 1) + sin 2 x − 2 sin x − 3 | + C t 2 − 4t + 5
sin ( x − α ) dt
(ii) Let I = ∫ sin ( x + α )
dx = 2 log | t 2 − 4t + 5 | +7 ∫ t 2 − 4t + 4 − 4 + 5
sin ( x − α ) sin ( x − α ) dt
I = ∫ × dx = 2 log | t 2 − 4t + 5 | + 7 ∫ ( t − 2) 2 + ( 1) 2
sin ( x + α ) sin ( x − α )
sin ( x − α ) = 2 log | t 2 − 4t + 5 | + 7 tan − 1 (t − 2) + C
= ∫ sin 2 x − sin 2 α
dx
= 2 log | sin 2 x − 4 sin x + 5 | + 7 tan −1(sin x − 2) + C

sin x cos α − cos x sin α


I = ∫ sin 2 x − sin 2 α
dx Type II
( px + q ) dx ( px + q )
sin x dx cos x dx
(i) ∫ ax 2 + bx + c (ii) ∫
ax 2 + bx + c
dx
= cos α ∫ − sin α ∫
sin 2 x − sin 2 α sin 2 x − sin 2 α
sin x dx
(iii) ∫ ( px + q ) ax 2 + bx + c dx
= cos α ∫ 1 − cos 2 x − sin 2 α The linear factor ( px + q ) is expressed in terms of the
cos x dx derivative of the quadratic factor ax 2 + bx + c together
− sin α ∫ sin 2 x − sin 2 α λd
with a constant as px + q = (ax 2 + bx + c ) + µ
sin x dx cos x dx dx
= cos α ∫ − sin α ∫
cos α − cos x
2 2
sin 2 x − sin 2 α ⇒ px + q = λ (2ax + b ) + µ
In the first part, put cos x = t , so that − sin x dx = dt
Here, we have to find λ and µ and replace ( px + q ) by
and in second part, put sin x = u , so that cos x dx = du
{λ (2ax + b ) + µ} in (i), (ii) and (iii).
dt du
∴ I = − cos α ∫ − sin α ∫
cos α − t
2 2
u − sin 2 α
2
y Example 22 Evaluate
 t  a−x a2 − x 2
= − cos α ⋅ sin − 1   − sin α ⋅ log
 cos α 
(i) ∫ a+x
dx (ii) ∫x a2 + x 2
dx
| u − (u 2 − sin 2 α )| + C
 cos x  a−x
= − cos α ⋅ sin − 1   − sin α ⋅ log
Sol. (i) Let I = ∫ a+x
dx
 cos α 
|sin x − sin 2 x − sin 2 α | + C a−x a−x a−x
I = ∫ ×
a+x a−x
dx = ∫ a2 − x 2
dx
2 sin 2 x − cos x
(iii) I = ∫ 6 − cos 2 x − 4 sin x dx a x
I = ∫ a −x
2 2
dx − ∫ a − x2
2
dx
( 4 sin x − 1) cos x
= ∫ 6 − (1 − sin 2 x ) − 4 sin x dx
x t dt
= a ⋅ sin − 1   + ∫
( 4 sin x − 1) cos x a t
= ∫ sin 2 x − 4 sin x + 5 dx
Put a 2 − x 2 = t 2 ⇒ − 2x dx = 2t dt ⇒ x dx = − t dt
Put sin x = t , so that cos x dx = dt
Chap 01 Indefinite Integral 15

x  1 1
= a ⋅ sin − 1   + t + C = ∫  − (1 − 2x ) +  1 + x − x 2 dx
a  2 2
x 1 1
= − ∫ (1 − 2x ) 1 + x − x 2 dx + ∫ 1 + x − x 2 dx
= a ⋅ sin − 1   + a 2 − x 2 + C
a 2 2

a2 − x 2 1 1 1 1
(ii) Let I = ∫x dx =−
2 ∫ t dt +
2 ∫ − (x 2 − x + − − 1) dx
4 4
a2 + x 2
[where t = 1 + x − x 2 ]
Put x 2 = t ⇒ 2x dx = dt
2
1 t 3/2  1  5   1
2

∴ I = ∫
a 2 − t dt 1
⋅ = ∫
a2 − t a2 − t
⋅ dt
=−
2
  +
 3 / 2 2
∫ 
 2
 −  x −  dx
  2
a2 + t 2 2 a2 + t a2 − t
1 a −t2 1 1
= − t 3/2 +
=
2 ∫ a −t4 2
dt 3 2
 2 2
(x − 1 / 2)  5   1 1  5 
2
1 dt 1 t dt  −1  x − 1 / 2  
  −  x −  +   sin   +C
= a2
2 ∫ (a ) − (t )
2 2 2

2 ∫ a4 − t 2  2  2   2 2  2   5 / 2  

1 2 t  1 du 1
=− (1 + x − x 2 )3 / 2
=
2
a ⋅ sin − 1  2  +
a  4 ∫ u
,
3
where a 4 − t 2 = u ⇒ − 2t dt = du 1 ( x − 1 / 2) 5  2x − 1 
+  1 + x − x 2 + sin − 1   +C
2 2 8  5  
 t  1  u 
1/ 2
1 2
= a ⋅ sin − 1  2  + ⋅   +C (ii) Let I = ∫ ( x + 1) 1 − x − x 2 dx
2  a  4  1/2 

[where t = x 2 and u = a 4 − x 4 ] d 
Put, ( x + 1) = λ ⋅  (1 − x − x 2 ) + µ
 dx 
1 2  x2  1
= a ⋅ sin − 1  2  + ⋅ a4 − x 4 + C Then, ( x + 1) = λ ( − 1 − 2x ) + µ comparing the
2  a  2 coefficients of like powers of x, we get − 2λ = 1 and
µ − λ = 1 ⇒ λ = − 1 / 2 and µ = 1 / 2.
k( x ) 1 1
Integrals of the Form ∫ dx, ∴ ( x + 1) = − ( − 1 − 2x ) +
2 2
ax + bx + c
2
 1 1
So, ∫ ( x + 1) 1 − x − x 2 dx = ∫  − ( − 1 − 2x ) + 
where k( x ) is a Polynomial of Degree  2 2

Greater than 2 1 − x − x 2 dx
1 1
2∫ ∫
To evaluate this type of integrals we divide the numerator =− ( − 1 − 2x ) 1 − x − x 2 dx + 1 − x − x 2 dx
by denominator and express the integral as 2
1
Q(x ) + 2
R (x )
, where R ( x ) is a linear function of x . = − ∫ ( − 1 − 2x ) 1 − x − x 2 dx
2
ax + bx + c 1   2 1 1
+ ∫ 1 −  x + x + −   dx
2   4 4
y Example 23 Evaluate
1 1
(i) ∫x 1 + x − x 2 dx (ii) ∫ (x + 1) 1 − x − x 2 dx =−
2
t dt +
2 ∫ ( 5 / 2)2 − ( x + 1 / 2)2 dx
[where t = 1 + x − x 2 ]
Sol. (i) Let I = ∫ x 1 + x − x 2 dx
 t 3/ 2  1 1 
1 1
d  =−   +  x +  1 − x − x
2
Put x = λ  (1 + x − x 2 ) + µ 
2 3 / 2  2  2  2 
 dx  1 5  x + 1 / 2 
+ ⋅ sin − 1   + C
Then, comparing the coefficients of like powers of x, 2 4  5 / 2 
we get 1 1
= − ( 1 − x − x 2 ) 3 / 2 + ( 2x + 1) 1 − x − x 2
1 = − 2λ and λ + µ = 0 ⇒ λ = − 1 / 2, µ = 1 / 2 3 8
∴ I = ∫ x 1 + x − x 2 dx 5  2x + 1
+ sin − 1   +C
16  5 
16 Textbook of Integral Calculus

x2 + x + 3 2x 2 + 5 x + 4
y Example 24 Evaluate ∫ x 2 − x − 2 dx. y Example 25 Evaluate ∫ x2 + x +1
dx .

x2 + x + 3 2x 2 + 5x + 4
Sol. Let I = ∫ x2 − x − 2 dx Sol. Let I = ∫ x2 + x + 1
dx

d 
 2x + 5  Put 2x 2 + 5x + 4 = λ ( x 2 + x + 1) + µ  ( x 2 + x + 1) + γ
∴ I = ∫ 1 + 2  dx  dx 
 x − x − 2
or 2x 2 + 5x + 4 = λ ( x 2 + x + 1) + µ (2x + 1) + γ
2x + 5
⇒ I = ∫ 1dx + ∫ x 2 − x − 2 dx Comparing the coefficients of like terms, we get
2 = λ , 5 = λ + 2µ , 4 = λ + µ + γ
2x + 5 ∴ λ = 2, µ = 3 / 2, γ = 1/2
I =x+ ∫ x 2 − x − 2 dx
Hence, the above integral reduces to
d  2x 2 + 5x + 4
Put, 2x + 5 = λ  ( x 2 − x − 2) + µ . Then, obtaining
 dx  I = ∫ x2 + x + 1
dx
2x + 5 = λ (2x − 1) + µ , comparing the coefficients of like
terms. We get, 2 = 2λ and 5 = µ − λ  ( x 2 + x + 1) 3 
( 2x + 1) 1 1
∴ λ = 1 and µ = 6 = ∫ 2 ⋅ + ⋅ + ⋅  dx
 x 2 + x + 1 2 x 2 + x + 1 2 x 2 + x + 1 
λ ( 2x − 1) + µ 
∴ I =x+∫ dx
x2 − x − 2 = 2 ∫ x 2 + x + 1 dx +
3 dt 1 dx
2x − 1 1 2 ∫ +
t 2 ∫ x + x +1
2
=x+ ∫ x 2 − x − 2 dx + 6 ∫ x 2 − x − 2 dx
[where t = x 2 + x + 1]
1 1
= x + ∫ dt + 6 ∫ 3 t 1/ 2
t 1 1
x2 − x + − − 2 = 2 ∫ ( x + 1 / 2)2 + ( 3 / 2)2 dx + ⋅
4 4 2 1/2
[where t = x 2 − x − 2] 1 dx
+ ∫
dx
2 ( x + 1 / 2) 2 + ( 3 / 2) 2
= x + log | t | + 6 ∫ dx
( x − 1 / 2) 2 − ( 3 / 2) 2 1  1 1 3  1 
= 2   x +  x 2 + x + 1 + ⋅ ⋅ log |  x +  + x 2 + x + 1|
 x − 1 − 3 2  2 2 4  2 
log 2 2 + C
1
= x + log | x − x − 2 | + 6 ⋅
2 1  1
2(3 /2)  1 3 + 3 x2 + x + 1 + log |  x +  + x 2 + x + 1 | + C
x− + 2  2
 2 2
 1 3  1 
 x − 2 ∴ I = x +  x 2+ x + 1 + log   x +  + x 2 + x + 1
= x + log | x 2 − x − 2 | + 2 ⋅ log + C  2 4  2 
 x + 1
1  1 
+ 3 x2 + x + 1 + log   x +  + x 2 + x + 1 + C
2   2  
Integrals of the Type  7 5  1 
ax 2 + bx + c ⇒ I =  x +  x 2 + x + 1 + log   x +  + x 2 + x + 1 + C
 2  
1. ∫ ( px 2 + qx + r ) dx 4  2 

(ax 2 + bx + c )
2. ∫ px 2 + qx + r
dx Trigonometric Integrals
(a) Integrals of the Form
3. ∫ (ax 2 + bx + c ) px 2 + qx + r dx
1 1 1
In above cases; substitute ∫ a cos 2 x + b sin 2 x dx , ∫ a + b sin 2 x dx , ∫ a + b cos 2 x dx ,
d 
ax 2 + bx + c = λ ( px 2 + qx + r ) + µ  ( px 2 + qx + r ) + γ . 1 1
 dx  ∫ (a sin x + b cos x ) 2 dx , ∫ a + b sin 2 x + c cos 2 x dx
Find λ, µ and γ. These integrations reduces to integration
of three independent functions.
Chap 01 Indefinite Integral 17

To evaluate this type of integrals, divide numerator and To evaluate this type of integrals we put
denominator both by cos 2 x , replace sec 2 x , if any, in 2 tan x / 2 1 − tan2 x / 2
sin x = and cos x = and replace
denominator by (1 + tan2 x ) and put tan x = t . So that 1 + tan2 x / 2 1 + tan2 x / 2
sec 2 x dx = dt . tan x / 2 = t , by performing these steps the integral reduces
1
y Example 26 Evaluate to the form ∫ 2 dt which can be evaluated by
at + bt + c
1 sin x
(i) ∫ dx (ii) ∫ sin 3x dx methods discussed earlier.
4 sin x + 9 cos 2 x
2

dx y Example 27 Evaluate
Sol. (i) I = ∫ 4 sin 2 x + 9 cos 2 x dx dx
(i) ∫ (ii) ∫
2 + sin x + cos x 3 sin x + cos x
Here, dividing numerator and denominator by cos 2 x .
2 tan x / 2
We get Sol. For this type we use, sin x = ,
sec 2 x 1 + tan 2 x / 2
I = ∫ 4 tan 2 x + 9 dx 1 − tan 2 x / 2
cos x = ⋅
Put tan x = t 1 + tan 2 x / 2

⇒ sec 2 x dx = dt dx
(i) Let I = ∫ 2 + sin x + cos x
dt 1 dt
∴ I = ∫ 4t 2 + 9 = 4 ∫ t 2 + ( 3 / 2) 2 dx
= ∫ 2 tan x / 2 1 − tan 2 x / 2
1 1  t  2+ +
= ⋅ tan − 1   +C 1 + tan x / 2
2
1 + tan 2 x / 2
4 3/2  3 / 2
x
1  2 tan x  dxsec 2
I = tan − 1   +C =∫ 2
6  3  x x x
2 + 2 tan 2 + 2 tan + 1 − tan 2
sin x sin x 2 2 2
(ii) Let I = ∫ sin 3x dx = ∫ 3 sin x − 4 sin 3 x dx x
sec 2 dx
dx I =∫ 2
I = ∫ 3 − 4 sin 2 x tan 2 x x
+ 2 tan + 3
2 2
Dividing numerator and denominator by cos 2 x , we get x
Put tan = t
sec 2 x dx sec 2 x dx 2
I = ∫ 3 sec 2 x − 4 tan 2 x ∫ 3 (1 + tan 2 x ) − 4 tan 2 x
= 1 x
⇒ sec 2 dx = dt = ∫ 2
2 dt
=2∫ 2
dt
2 2 t + 2t + 3 t + 2t + 1 + 2
sec 2 x dx
I = ∫ 3 − tan 2 x =2∫
dt
( t + 1) 2 + ( 2 ) 2
Let tan x = t ⇒ sec 2 x dx = dt 1  t +1 
= 2⋅ tan − 1   +C
dt 1  3 +t 2  2 
∴ I = ∫( = log + C
3 ) − (t )  3 −t  tan x / 2 + 1 
2 2
2 3
I = 2 tan − 1   +C
 2 
1  3 + tan x 
I = log + C
dx
2 3  3 − tan x  (ii) Let I = ∫ 3 sin x + cos x
dx
(b) Integrals of the Form
= ∫ 3 ⋅ 2 tan x / 2 1 − tan 2 x / 2
+
1 1 1 + tan 2 x / 2 1 + tan 2 x / 2
∫ a sin x + b cos x
dx, ∫
a + b sin x
dx,
x
sec 2 dx
1 1 = ∫ 2
∫ a + b cos x dx, ∫ a sin x + b cos x + c dx x
2 3 tan + 1 − tan 2
x
2 2
18 Textbook of Integral Calculus

x 1 x 1   π x π 
Put tan= t ⇒ sec 2 dx = dt = log tan  + −  + C
2 2 2 2   4 2 6 
2dt dt
∴ I =∫ =2∫ 1  x π
− t 2 + 2 3t + 1 − t 2 + 2 3t − 3 + 3 + 1 = log tan  +   + C
2   2 12 
dt dt
=2∫ =2∫
4 − (t − 3 ) 2
( 2) − ( t − 3 ) 2
2

1 2 + t − 3
(d) Integrals of the Form
= 2⋅ log + C
2 ( 2) 2 − t + 3 p cos x + q sin x + r p cos x + q sin x
∫ a cos x + b sin x + c dx , ∫ a cos x + b sin x dx
1  2 − 3 + tan x / 2 
∴ I = log + C
2  2 + 3 − tan x / 2  Rule for (i) In this integral express numerator as,
λ (denominator) + µ (diffn. of denominator) + γ.
Find λ, µ and γ by comparing coefficients of sin x , cos x
(c) Alternative Method to Evaluate and constant term and split the integral into sum of three
the Integrals of the Form integrals.
d.c.of (denominator) dx
1 λ ∫ dx + µ ∫ dx + γ ∫
∫ a sin x + b cos x
dx denominator a sin x + b cos x +c

To evaluate this type of integrals we substitute Rule for (ii) Express numerator as λ (denominator) + µ
a = r cos θ, b = r sin θ and so (differentiation of denominator) and find λ and µ as above.
 b  (2 + 3 cos x )
r = a 2 + b 2 , θ = tan − 1   y Example 29 Evaluate ∫ sin x + 2 cos x + 3 dx.
 a 
∴ a sin x + b cos x = r sin ( x + θ ) Sol. Write the numerator = λ (denominator) + µ (d.c. of
1 1 1
So, ∫ a sin x + b cos x dx = r ∫ sin ( x + θ ) dx denominator) + γ
⇒ 2 + 3 cos x = λ (sin x + 2 cos x + 3) + µ (cos x − 2 sin x ) + γ
1 1   x θ Comparing the coefficients of sin x , cos x and constant
= ∫ cosec ( x + θ ) dx = log tan  +   + C
r r   2 2 terms, we get
0 = λ − 2µ , 3 = 2λ + µ , 2 = 3λ + γ
1 1
∴ ∫ a sin x + b cos x dx = a + b2
2
⇒ λ = 6 / 5, µ = 3 / 5, γ = − 8 / 5
6 3 cos x − 2 sin x
Hence, I = ∫ 1 dx + ∫ dx
 x 1 b  5 5 sin x + 2 cos x + 3
log tan  + tan − 1   + C
 2 2 a  8
− ∫
dx
5 sin x + 2 cos x + 3
1

y Example 28 Evaluate 6 3 8
dx . = ⋅ x + log | sin x + 2 cos x + 3 | − I 3 …(i)
3 sin x + cos x 5 5 5
dx
Sol. Let 3 = r sin θ and 1 = r cos θ. Where, I 3 = ∫
sin x + 2 cos x + 3
3 π
Then, r = ( 3 )2 + (1)2 = 2 and tan θ = ⇒ θ= dx
1 3 = ∫ 2 tan x / 2 2 (1 − tan 2 x / 2)
1 + +3
∴ I = ∫ 3 sin x + cos x
dx 1 + tan 2 x / 2 1 + tan 2 x / 2
x
1 sec 2
∫ r sin θ sin x + r cos θ cos x dx
= dx
=∫ 2
x x x
1 dx 1 2 tan + 2 − 2 tan 2 + 3 + 3 tan 2
=
r ∫ cos ( x − θ) = r ∫ sec ( x − θ) dx 2 2 2
x
sec 2 dx
1   π x θ  =∫ 2 x
= log tan  + −   + C , let tan = t
r   4 2 2  tan 2 x x
+ 2 tan + 5 2
2 2
Chap 01 Indefinite Integral 19


1 x
sec 2 dx = dt = ∫ 2
2dt
=2∫
dt  sin( x + A ) ⋅ cos x cos ( x + A ) ⋅ sin x 
= cot A ⋅ ∫  −  dx
2 2 t + 2t + 5 ( t + 1) 2 + 22  cos x cos( x + A ) cos x ⋅ cos ( x +A ) 
 x  = cot A ⋅ {∫ tan ( x + A )dx − ∫ tan x dx }
 tan + 1
1 −1  t + 1  −1 2
I 3 = 2 ⋅ ⋅ tan   = tan   …(ii)
2  2  2 = cot A ⋅ {log | sec ( x + A ) | − log | sec x |} + C
 
  Hence, (c) is the correct answer.
From Eqs. (i) and (ii),
cos 2x
6 3 8

−1
x 
 tan + 1 
2  +C
y Example 31 The value of ∫ sin x
dx , is equal to
I = x + log | sin x + 2 cos x + 3| − tan 
5 5 5 
 2  
  (a) log | cot x + cot 2 x − 1 | + 2 log | cos x
+ cos 2 x − 1 / 2 | + C
y Example 30 The value of ∫ {1 + tan x ⋅ tan ( x + A )} dx
(b) − log | cot x + cot 2 x − 1 | + 2 log | cos x
is equal to
+ cos 2 x − 1 / 2 | + C
 sec x 
(a) cot A ⋅ log + C (c) log | cot x + cot 2 x − 1 | + 2 log | cos x
 sec (x + A ) 
+ cos 2 x − 1 / 2 | + C
(b) tan A ⋅ log | sec (x + A)| + C
(d) − log | cot x + cot 2 x − 1 | + 2 log | cos x
 sec (x + A) 
(c) cot A ⋅ log + C + cos 2 x − 1 / 2 | + C
 sec (x )  cos 2x cos 2x
(d) None of the above
Sol. Let I = ∫ sin x
dx = ∫
sin x cos 2x
dx

Sol. Let I = ∫ {1 + tan x ⋅ tan ( x + A )} dx 1 − 2 sin 2 x


= ∫ sin x cos 2x
dx

 sin x ⋅ sin ( x + A ) 
= ∫ 1+  dx 1 sin x
 cos x ⋅ cos ( x + A )  = ∫ sin x cos x − sin x
2 2
dx − 2 ∫
2 cos 2 x − 1
dx

cos x ⋅ cos ( x + A ) + sin x ⋅ sin ( x + A )


= ∫ cos x ⋅ cos ( x + A )
dx
= ∫
cosec 2 x
dx −
2

sin x
dx
cot x − 1
2 2 cos 2 x − 1 / 2
cos ( x + A − x ) dx
= ∫ cos x ⋅ cos ( x + A ) =− ∫
dt
− 2 ∫
− ds
t −1
2
s − (1 / 2 )2
2

dx
= cos A ⋅ ∫ [where t = cot x and s = cos x ]
cos x ⋅ cos ( x + A )
= − log | t + t − 1 | + 2 log | s + s 2 − 1 / 2 | + C
2
Multiplying and dividing by sin A , we get
sin A dx = − log | cot x + cot 2 x − 1 | + 2 log | cos x
= cot A ⋅ ∫
cos x ⋅ cos ( x + A )
+ cos 2 x − 1 / 2 | + C
sin ( x + A − x ) dx
= cot A ⋅ ∫ Hence, (b) is the correct answer.
cos x ⋅ cos ( x + A )
20 Textbook of Integral Calculus

Exercise for Session 3


n Evaluate the following integrals
2
1. ∫ x dx 4 2. ∫ x 6 dx
9 − 16x 9 + 16x
3
3. ∫ x 8 dx 4. ∫ x
dx
16x − 25 a3 − x 3

x4
5. ∫ dx 6. ∫ −x 1 x dx
a + x6
6
4e − 9e

2x 8x − 11
7. ∫ dx 8. ∫ dx
4 − 25
x
5 + 2x − x 2
x +2
9. ∫ dx 10. ∫ x − 3 2 dx
x 2 + 2x + 2 3 − 2x − x

11. ∫ 2 3x − 1 dx 12. ∫ x dx
4x − 4x + 17 2x + 3

a−x 1− x
13. ∫ dx 14. ∫ dx
x −b 1+ x

x 2 + 2x + 3 dx
15. ∫ dx 16. ∫
x +x +1
2 1 + sin x + cos x
2
17. ∫ dx
18. ∫ cos x sin x
dx
sin x + 3 cos x sin x − cos x

ex cos x − cos 3 x
19. ∫ dx 20. ∫ dx
5 − 4e − ex 2x (1 − cos 3 x )

3 sin x + 2 cos x
21. Evaluate ∫ dx 22. Evalute ∫ (2x − 4) 4 + 3x − x 2dx .
3 cos x + 2 sin x

23. ∫ (2x + 5x2 + 9) dx


2

( x + 1) x + x + 1
dx
24. The value of ∫ , is equal to
sec x + cosec x
 1  tan x / 2 − 1 − 2 
(a) (sin x + cos x ) + log  + C
 2  tan x / 2 − 1 + 2
 1  tan x / 2 − 1 − 2 
(b) 2 (sin x + cos x ) + log  + C
 2  tan x / 2 − 1 + 2

1 1  tan x / 2 − 1 − 2  
(c) (sin x − cos x ) + log  + C
2 2  tan x − 1 + 2  
(d) None of these
Session 4
Integration by Parts
Integration by Parts
Theorem If u and v are two functions of x , then Usually we use the following preference order for
du  selecting the first function. (Inverse, Logarithmic,
∫ uv dx = u ∫ v dx − ∫ dx ∫ v dx  dx Algebraic, Trigonometric, Exponent).
In above stated order, the function on the left is always
i.e. The integral of product of two functions = (first chosen as the first function. This rule is called as ILATE.
function) × (integral of second function) – integral of
(differential of first function × integral of second function). y Example 32 Evaluate
Proof For any two functions f ( x ) and g( x ), we have (i) ∫ sin − 1 x dx (ii) ∫ log e | x | dx
d d d
{ f ( x ) ⋅ g( x )} = f ( x ) ⋅ { g( x )} + g( x ) ⋅ { f ( x )} Sol. (i) I = ∫ sin −1 x dx = ∫ sin −1 x ⋅ 1 dx
dx dx dx
I II
 d d 
∴ ∫  f ( x ) ⋅ { g( x )} + g( x ) ⋅ { f ( x )}  dx = ∫ f ( x ) ⋅ g( x ) dx Here, we know by definition of integration by parts that
 dx dx  order of preference is taken according to ILATE. So,
 d   d  ‘sin −1 x ’ should be taken as first and ‘1’ as the second
⇒ ∫  f ( x )⋅ { g( x )}  dx + ∫  g ( x ) ⋅ { f ( x )}  dx
 dx   dx  function to apply by parts.
Applying integration by parts, we get
= ∫ f ( x ) ⋅ g( x )dx
1
I = sin − 1 x ⋅ ( x ) − ∫ ⋅ x dx
 d  1 − x2
⇒ ∫  f ( x ) ⋅ dx { g ( x )}  dx
 1 dt

= ∫ f ( x ) ⋅ g ( x ) dx − ∫  g ( x ) ⋅
d 
{ f ( x )}  dx
= x ⋅ sin − 1 x +
2 ∫ t 1/ 2
 dx 
d Let 1 − x2 = t
Let f ( x ) = u and { g ( x )} = v
dx 1
− 2x dx = dt ⇒ x dx = − dt
So that, g ( x ) = ∫ v dx 2
1 t 1/ 2
 du  = x sin − 1 x + ⋅ +C
∴ ∫ uv dx = u ⋅ ∫ v dx − ∫  dx ⋅ ∫ v dx  ⋅ dx 2 1/2
I = x sin − 1 x + 1 − x 2 + C
Remarks
∫ sin
−1
While applying the above rule, care has to be taken in the ∴ x dx = x sin − 1 x + 1 − x 2 + C
selection of first function (u) and selection of second function ( v ).
Normally we use the following methods : (ii) I = ∫ loge | x | dx = ∫ loge | x | ⋅ 1 dx
I II
1. If in the product of the two functions, one of the functions is
not directly integrable (e.g. Applying integration by parts, we get
log| x |, sin− 1 x, cos − 1 x, tan− 1 x, …, etc.) Then, we take it as the 1
first function and the remaining function is taken as the = log | x | ⋅ x − ∫ ⋅ x dx
x
second function. i.e. In the integration of ∫ x tan− 1 x dx,
tan− 1 x is taken as the first function and x as the second
= x log | x | − ∫ 1 dx
function. I = x log | x | − x + C
2. If there is no other function, then unity is taken as the second
function. e.g. In the integration of ∫ tan− 1 x dx, tan− 1 x is taken y Example 33 Evaluate
as first function and 1 as the second function. (i) ∫ x cos x dx (ii) ∫x
2
cos x dx
3. If both of the function are directly integrable, then the first
function is chosen in such a way that the derivative of the
function thus obtained under integral sign is easily integrable.
Sol. (i) ∫ x cos x dx , I = ∫ x cos x dx
I II
22 Textbook of Integral Calculus

Applying integration by parts, −1 1


= sin − 1 x (1 − 2x ) + ⋅ x 1− x …(ii)
d  2 2
I = x ( ∫ cos x dx ) − ∫  ( x ) { ∫ (cos x ) dx } dx
 dx  From Eqs. (i) and (ii), we get
I = x sin x − ∫ 1 ⋅ sin x dx = x sin x + cos x + C 4 − 1 1 
I =  (sin −1 x ) (1 − 2x ) + x 1 − x − x +C
π 2 2 
(ii) I = ∫x
2
cos x dx
I II 2
= { x − x 2 − (1 − 2x ) sin −1 x } − x + C
Applying integration by parts, π
d 
I = x 2 ( ∫ cos x dx ) − ∫  ( x 2 ) ⋅ { ∫ cos x } dx
 dx  Integral of Form ∫ e x { f ( x ) + f ′ ( x )}dx
= x sin x − ∫ 2x ⋅ (sin x ) dx
2
Theorem Prove that
= x 2 sin x − 2 ∫ x (sin x ) dx
∫e { f ( x ) + f ′ ( x ) } dx = e x f ( x ) + C
x

We again have to integrate ∫ x sin x dx using integration Proof We have, ∫ e x { f ( x ) + f ′ ( x ) } dx


by parts,
= ∫e ⋅ f ( x ) dx + ∫ e x ⋅ f ′ ( x ) dx
x

= x 2 ⋅ sin x − 2 ∫ x ⋅ sin x dx II I

= f ( x ) ⋅ e − ∫ f ′ ( x ) ⋅ e x dx + ∫ e x ⋅ f ′( x ) dx + C
I II x
  dx  
= x sin x − 2  x ( ∫ sin x dx ) − ∫   ( ∫ sin x dx ) dx 
2

  dx   = f (x ) ⋅ e x + C
= x 2 sin x − 2 { − x cos x − ∫ 1 ⋅ ( − cos x ) dx } Thus, to evaluate the integrals of the type
∫e { f ( x ) + f ′ ( x ) } dx ,
x
I = x sin x + 2x cos x − 2 sin x + C
2

sin − 1 x − cos − 1 x we first express the integral as the sum of two integrals
y Example 34 Evaluate ∫ sin − 1 x + cos − 1 x
dx.
∫ e f ( x ) dx and ∫ e f ′ ( x ) dx and then integrate the
x x

sin − 1 x − cos − 1 x
integral involving e x f ( x ) as integral by parts taking e x
Sol. Let I = ∫ sin − 1 x + cos − 1 x
dx as second function.

sin − 1 x − ( π / 2 − sin − 1 x ) Remark


= ∫ π /2
dx
The above theorem is also true, if we have ekx in place of ex
[Q sin − 1 θ + cos − 1 θ = π / 2]
∫e { f ( kx ) + f ′( kx ) } dx = ekx f ( kx ) + C
kx
i.e.
2
∫ (2 sin
−1
⇒ I = x − π / 2) dx
π
=
4
∫ sin
−1
x dx − ∫ 1 dx
General Concept
π
∫e { f ( x ) g ′ ( x ) + f ′ ( x )} dx
g (x )
4
∫ sin
−1
I = x dx − x + C …(i)
π g ( x ) f ( x ) g ′ ( x ) dx +
Proof I = ∫ e1 ∫1e442 f ′ ( x ) dx
g(x)
23 123 123
Let x = sin 2 θ, then dx = 2 sin θ cos θ dθ = sin 2θ dθ I
II II
443
as it is

∫ sin x dx = ∫ θ ⋅ sin 2θ dθ
−1
∴ Using, ∫ e g(x)
⋅ g ′ ( x ) dx = e g(x)
, we get
I II
Applying integration by parts = f ( x ) ⋅ e g ( x ) − ∫ f ′( x ) ⋅ e g ( x ) dx + ∫ e g ( x ) ⋅ f ′( x ) dx
cos 2θ 1
∫ sin xdx = − θ ⋅ 2 + ∫ 2 cos 2θ d θ
−1
= f (x ) ⋅ e g (x ) + C
−θ 1  x 2 cos 2 x − ( x sin x + cos x ) 
= ⋅ cos 2θ + sin 2θ
2 4 e.g. = ∫ e ( x sin x + cos x )   dx
 x2 
− 1⋅θ 1
= ⋅ (1 − 2 sin 2 θ ) + ⋅ sin θ ⋅ 1 − sin 2 θ
 2 x sin x + cos x 
∫e
2 2 ( x sin x + cos x )
⇒  cos x −  dx
 x2 
Chap 01 Indefinite Integral 23

( x sin x + cos x )   cos x   cos x  ′   1 + x2 2x 


⇒ ∫e  x cos x

  +
 x   x  
  dx = ∫ex 
 (1 + x )
2 2
−  dx
(1 + x 2 )2 
cos x  1 2x   d  1  2x 
⇒ e ( x sin x + cos x ) ⋅ +C = ∫ex −  dx as  2
=− 
x  1 + x 2
( 1 + x 2 )2   dx 1 + x  ( 1 + x 2 )2 
e.g. = ∫ e tan x (sin x − sec x ) dx ex
= +C
= ∫ e tan x sin x dx − ∫ e tan x sec x dx 1 + x2
ex
⇒ − e tan x ⋅ cos x + ∫ e tan x sec 2 x cos xdx − ∫ e tan x sec xdx ∴ I = +C
1 + x2
⇒ − e tan x ⋅ cos x

y Example 35 Evaluate Integrals of the Form


∫e sin bx dx, ∫ eax cos bx dx
ax
 1 + sin x cos x   1 + sin 2 x 
(i) ∫ e x   dx (ii) ∫ e 2x
  dx
 cos 2 x   1 + cos 2 x 
Let I = ∫ e ax (sin bx ) dx
 1 + sin x cos x 
Sol. (i) I = ∫ e x   dx I= ∫ sin bx ⋅ e
ax
Then, dx
 cos 2 x 
I II
 1 sin x cos x 
I = ∫ex  +  dx  e ax  e ax
 cos 2
x cos 2 x  = sin bx ⋅   − ∫ b cos bx ⋅ dx
 a  a
I = ∫ e {tan x + sec x} dx
x 2

1 b e ax e ax 
I = ∫ e x ⋅ tan x dx + ∫ e x (sec 2 x ) dx = sin bx ⋅ e ax − cos bx ⋅ − ∫ ( − b sin bx ) ⋅ dx 
II
a a a a 
I
I = tan x ⋅ e − ∫ sec 2 x ⋅ e x dx + ∫ e x ⋅ sec 2 x dx + C
x 1 b b2
=sin bx ⋅ e ax − 2 cos bx ⋅ e ax − 2 ∫ sin bx ⋅ e ax dx
a a a
I = e x tan x + C
1 b b2
 1 + sin 2x  I = sin bx ⋅ e ax − 2 cos bx ⋅ e ax − 2 I
(ii) I = ∫ e 2 x   dx a a a
 1 + cos 2x 
b2 1 ⋅ e ax
 1 + 2 sin x cos x  ∴ I+ I= ⋅ (a sin bx − b cos bx )
= ∫e 2x
  dx a2 a2
 2 cos 2 x 
 a 2 + b 2  e ax
 1 2 sin x cos x  ⇒ I = (a sin bx − b cos bx )
= ∫e 2x
 2
+  dx  a2  a2
 2 cos x 2 cos 2 x 
1  e ax
= ∫ e 2 x  sec 2 x + tan x  dx or I= (a sin bx − b cos bx ) + C
2  a2 + b2
1
= ∫ e 2 x ⋅ tan x dx + ∫ e 2 x ⋅ sec 2 x dx e ax
∫ e sin bx dx = (a sin bx − b cos bx ) + C
ax
2 Thus,
II I a2 + b2
e 2x e 2x 1 e ax
= tan x ⋅ − ∫ sec 2 x ⋅ dx + ∫ e 2 x ⋅ sec 2 x dx
2 2 2 Similarly, ∫ e ax cos bx dx = (a cos bx + b sin bx ) + C
a2 + b2
1 2x
I = e ⋅ tan x + C
2 Aliter Use Euler’s equation
2 Let P = ∫ e ax cos bx dx and Q = ∫ e ax sin bx dx
 1− x 
y Example 36 Evaluate ∫ e x   dx .
1+ x 
2
Hence, P + iQ = ∫ e ax ⋅ e ibx dx = ∫ e (a + ib ) x dx
2
x  1− x  ( 1 − 2x + x 2 ) a − ib ax
Sol. I = ∫ e  1 + x 2  dx = ∫e 1
x
(1 + x 2 )2
dx P + iQ = e (a + ib ) x = 2 e (cos bx + i sin bx )
a + ib a + b2
24 Textbook of Integral Calculus

(ae ax cos bx + be ax sin bx ) − i (ae ax sin bx −be ax cos bx ) x 2dx


=
a2 + b2
y Example 38 Evaluate ∫ ( x sin x + cos x )2 ⋅
e ax (a cos bx + b sin bx ) x2
∴ P= Sol. Let I = ∫ ( x sin x + cos x )2 dx
a2 + b2
Multiplying and dividing it by ( x cos x ), we get
e ax (a sin bx − b cos bx )
Q= ( x cos x )
a2 + b2 I = ∫ ( x sec x ) ⋅ dx
I ( x sin x + cos x )2
II
y Example 37 Evaluate x cos x
I = x sec x ⋅ ∫ dx
(i) ∫ e x cos 2 x dx ( x sin x + cos x )2
d  x cos x 
(ii) ∫ sin (log x ) dx − ∫  ( x sec x )  ∫ dx  dx
  ( x sin x + cos x )
2
 dx 
 1 + cos 2x  −1
Sol. (i) I = ∫ e x ⋅ cos 2 x dx = ∫ e x ⋅   dx = x sec x ⋅
 2  ( x sin x + cos x )
1 x 1 −1
I = ∫ e dx + ∫ cos 2x ⋅ e dx x
− ∫ ( x sec x ⋅ tan x + sec x ) ⋅ dx
2 2 ( x sin x + cos x )
1
I = e x + I1
1 − x sec x ( x sin x + cos x )
2 2
…(i) =
( x sin x + cos x )
+ ∫ cos 2 x ⋅ ( x sin x + cos x ) dx
where, I 1 = ∫ cos 2x ⋅ e x dx − x sec x
( x sin x + cos x ) ∫
= + sec 2 x dx
I 1 = ∫ cos 2x ⋅ e x dx = cos 2x ⋅ e x − ∫ − 2 sin 2x ⋅ e x dx
I II − x sec x
I = + tan x + C
= e ⋅ cos 2x + 2 ∫ sin 2x ⋅ e dx
x x ( x sin x + cos x )
I II
y Example 39 The value of
= e x ⋅ cos 2x + 2 {sin 2x ⋅ e x − ∫ 2 cos 2x ⋅ e x dx }
3− x −1  1 
= e x ⋅ cos 2x + 2 sin 2x ⋅ e x − 4 I 1 ∫ 3 + x ⋅ sin  6 3 − x  dx , is equal to
1 x
∴ I1 ={e cos 2x + 2 sin 2x e x } …(ii)  2 
5 1  − 1  x − 1  x
 − 3  cos    + 2 9 − x ⋅ cos   + 2x + C
2
(a)
From Eqs. (i) and (ii), we get 4   3  3 
1 1 1
1  
2
I = e x + ⋅ {e x cos 2x + 2 sin 2x ⋅ e x }   x  x
2 2 5 (b)  − 3  cos −1    + 2 9 − x 2 sin −1   + 2x + C
4   3  3
1 x 1
I = e + e x { cos 2x + 2 sin 2x } + C
 
2 10
1  
2
  x  x
(ii) I = ∫ sin (log x ) dx (c)  − 3 sin − 1    + 2 9 − x 2 sin −1   + 2x  + C
4   3  3 

Let log x = t
(d) None of the above
⇒ x = e t or dx = e t dt 3− x  1 
∴ I = ∫ (sin t ) ⋅ e t dt = sin t ⋅ e t − ∫ cos t ⋅ e t dt
Sol. Here, I = ∫ 3+ x
⋅ sin − 1 
 6
3 − x  dx

I II I II
Put x = 3 cos 2θ
I = sin t ⋅ e t − {cos t ⋅ e t − ∫ − sin t ⋅ e t dt } ⇒ dx = − 6 sin 2θ dθ
I = e t ⋅ sin t − e t ⋅ cos t − I 3 − 3 cos 2θ  1 
∴ I = ∫ 3 + 3 cos 2θ
⋅ sin −1 
 6
3 − 3 cos 2θ  ( −6 sin 2θ )dθ

1 t
∴ I = e (sin t − cos t ) + C
2 sin θ
∫ cos θ ⋅ sin
−1
= (sin θ ) ⋅ ( − 6 sin 2θ ) dθ
x
I = {sin (log x ) − cos (log x )} + C
2 = − 6 ∫ θ ⋅ (2 sin 2 θ ) dθ = − 6 ∫ θ (1 − cos 2θ ) dθ
Chap 01 Indefinite Integral 25

 θ2  Sol. Here, we have only one function. This can be solved


= − 6  − ∫ θ cos 2θ dθ easily by applying integration by parts taking unity as
2  second function.
 θ 2  sin 2θ  sin 2θ    If we take u = log ( 1 − x + 1 + x ) as the first function
= − 6  − θ − ∫1⋅   dθ 
2  2  2   and v = 1 as the second function.
 θ sin 2θ cos 2θ  Then,
= − 3θ 2 + 6  +  +C
 2 4  I = ∫ 1 ⋅ log ( 1 − x + 1 + x ) dx
1   
2
 x  x
=  − 3  cos −1    + 2 9 − x 2 ⋅ cos −1   + 2x  + C = {log ( 1 − x + 1 + x )} ⋅ x − ∫
1
4    3   3  1− x + 1+ x

Hence, (a) is the correct answer.  1 1 
− +  ⋅ x dx
sec x (2 + sec x )  2 1− x 2 1+ x
y Example 40 The value of ∫ (1 + 2 sec x ) 2
dx , is
= x log ( 1 − x + 1 + x )
equal to 1 1− x − 1+ x 1
sin x cos x

2 ∫ 1− x + 1+ x

1 − x2
⋅ x dx
(a) +C (b) +C
2 + cos x 2 + cos x
= x log ( 1 − x + 1 + x )
− sin x cos x
(c) +C (d) +C
2 + sin x 2 + sin x 1 (1 − x ) + (1 + x ) − 2 1 − x 2 1
sec x (2 + sec x ) 2 cos x + 1

2 ∫ (1 − x ) − (1 + x )

1 − x2
⋅ 2x dx
Sol. Let I = ∫ (1 + 2 sec x ) 2
dx = ∫ (cos x + 2)2 dx
1 1 − x2 − 1
cos x (cos x + 2) + sin x 2 = x log ( 1 − x + 1 + x ) − ∫ dx
= ∫ (2 + cos x ) 2
dx 2 1 − x2
1 1 1
2∫ ∫
= x log ( 1 − x + 1 + x ) − 1 dx + dx
cos x sin 2 x
=∫ dx + ∫ dx 2 1 − x2
2 + cos x (2 + cos x )2
1 1
1 sin 2 x = x log ( 1 − x + 1 + x ) − x + sin − 1 x + C
= ∫ cos x ⋅ dx + ∫ dx 2 2
II (2 + cos x ) (2 + cos x )2 Hence, (c) is the correct answer.
I
Applying integration by parts to first by taking cos x as  x4 +2 
sin 2 x y Example 42 The value of ∫e x   dx , is
second function, keeping ∫ dx as it is.  (1 + x )
2 5/ 2

(2 + cos x )2
1 sin x equal to
∴ I = ⋅ (sin x ) − ∫ sin x ⋅ dx e x (x + 1) e x (1 − x + x 2 )
2 + cos x (2 + cos x )2 (a) (b)
(1 + x 2 ) 3 / 2 (1 + x 2 ) 3 / 2
sin 2 x
+ ∫ (2 + cos x )2 dx (c)
e x (1 + x)
(d) None of these
sin x
∴ I = +C (1 + x 2 ) 3 / 2
2 + cos x
 x4 + 2 
Hence, (a) is the correct answer. Sol. Let I = ∫ e x   dx
 (1 + x 2 )5 / 2 
y Example 41 The value of ∫ log ( 1 − x + 1 + x ) dx , is  1 1 − 2 x2 
= ∫ex  +  dx
equal to  (1 + x 2 )1/ 2 (1 + x 2 )5 / 2 
1 1
(a) x log ( 1 − x + 1 + x ) + x − sin − 1 (x) + C  1 x x 1 − 2x 2 
2 2 = ∫ex  − + +  dx
1 1  (1 + x 2 )1/ 2 (1 + x 2 )3 / 2 (1 + x 2 )3 / 2 (1 + x 2 )5 / 2 
(b) x log ( 1 − x + 1 + x ) + x + sin − 1 (x) + C
2 2 ex xe x e x {1 + x 2 + x }
= + +C = +C
1 1 (1 + x ) 2 1/ 2
(1 + x )
2 3/2
(1 + x 2 )3 / 2
(c) x log ( 1 − x + 1 + x ) − x + sin − 1 (x) + C
2 2 Hence, (d) is the correct answer.
(d) None of the above
26 Textbook of Integral Calculus

y Example 43 If ∫ (sin 3θ + sin θ ) e sin θ cos θ dθ = e (x



sin x + cos x ) 1 
x −  +C
 x cos x 
= ( A sin 3 θ + B cos 2 θ + C sin θ + D cos θ + E ) e sin θ + F ,
Hence, (c) is the correct answer.
then
(a) A = − 4 , B = 12 (b) A = − 4 , B = − 12
 2x + 2 
y Example 45 Evaluate ∫ sin − 1   dx.
(c) A = 4 , B = 12 (d) A = 4 , B = − 12  4 x 2 + 8 x + 13 
Sol. Let I = ∫ (sin 3θ + sin θ ) e sin θ ⋅ cos θ dθ
[IIT JEE 2001]
= ∫ (3 sin θ − 4 sin θ ) ⋅ e 3 sin θ
⋅ cos θ dθ;  2x + 2 
Sol. Here, I = ∫ sin − 1   dx
Put sin θ = t ⇒ cos θ dθ = dt  4 x 2 + 8x + 13 
 
= ∫ (3t − 4t 3 ) e t dt …(i)  
2x + 2
= ∫ sin − 1   dx
As, I = ( A sin θ + B cos θ + C sin θ + D cos θ + E )e
3 2 sin θ
+F  ( 2x + 2) 2 + 32 
 
= ( A sin θ − B sin θ + C sin θ + D cos θ + B + E )e sin θ + F
3 2
Put 2x + 2 = 3 tan θ ⇒ 2 dx = 3 sec 2 θ dθ
When, sin θ = t  3 tan θ  3 3
= ∫ sin −1   sec θ dθ = ∫ θ sec θ dθ
2 2
I = ( At 3 − Bt 2 + Ct + B + E ) e t + F  3 sec θ  2 2
as by Eq. (i) D = 0 …(ii) 3
From Eqs. (i) and (ii), {θ tan θ − ∫ tan θ dθ }
=
2
∫ (3t − 4t ) e t dt = ( At 3 − Bt 2 + Ct + B + E ) e t + F ,
3
3
14444244443 = {θ tan θ − log | sec θ | } + C
f (t ) 2
differentiating both sides  2 
3  2x + 2 
 2x + 2  2x + 2   + C
(3t − 4t 3 ) e t = ( At 3 − Bt 2 + Ct + B + E )e t + (3At 2 − 2Bt + C )e t ⇒ I=  tan −1   − log  1 +  
2 3  3    3  
⇒ A = − 4 and 3A = B ⇒ B = − 12    
Hence, (b) is the correct answer. 3 2 2  
=  ( x + 1)tan −1  ( x + 1) − log 4 x 2 + 8x + 13 + C
2 3 3  
y Example 44 The value of
−1  2  3
 x 4 cos 3 x − x sin x + cos x  ⇒ I = ( x + 1) tan  ( x + 1) − log ( 4 x + 8x + 13) + C
2
3  4

( x sin x + cos x )
e ⋅   dx , is equal to
 x 2 cos 2 x 
x 2 ( x sec 2 x + tan x ) dx

(a) e ( x sin x + cos x ) ⋅  x +
1 
 +C
y Example 46 Evaluate ∫ ( x tan x + 1) 2

 x cos x 
  x sec 2 x + tan x
(b) e ( x sin x + cos x ) ⋅  x cos x +
1 Sol. Here, I = ∫ x 2 ⋅ dx
 +C ( x tan x + 1)2
 x 
 1   1 
( x tan x + 1) ∫
 1  = x 2 −  − 2x ⋅  −  dx …(i)
(c) e ( x sin x + cos x ) ⋅  x −  +C   ( x tan x + 1)
 x cos x 
 x sec 2 x + tan x dt 1 1 
(d) None of the above using ∫ dx = ∫ 2 = − = − 
 x 4 cos 3 x − x sin x + cos x   ( x tan x + 1) 2
t t ( x tan x + 1)
Sol. Let I = ∫ e (x sin x + cos x )
⋅  dx
 x 2 cos 2 x   x2  2x (cos x )
= ∫ (x ⋅ e ( x sin x + cos x )
⋅ x cos x ) dx − ∫ e ( x sin x + cos x )
⇒I = −   +
 x tan x + 1
∫ x sin x + cos x dx
 x sin x − cos x [put, x sin x + cos x = u
⋅  dx ⇒ ( x cos x + sin x − cos x ) dx = du ]
 ( x cos x )2 
x2 du
Applying integration by parts ⇒ I =− +2∫
( x tan x + 1) u
= { x ⋅e ( x sin x + cos x )
−∫e ( x sin x + cos x )
dx } 2
x
 1  =− + 2 log | u | + C
x tan x + 1
x cos x ∫
sin x + cos x ) sin x + cos x )
− e (x ⋅ − e (x dx 
  x2
=− + 2 log | x sin x + cos x | + C
x tan x + 1
Chap 01 Indefinite Integral 27

Exercise for Session 4


1. ∫ x 2e x dx 2. ∫ x 2 sin x dx

3. ∫ log x ⋅ dx 4. ∫ ( log x )2dx


−1
5. ∫ ( tan−1 x ) dx 6. ∫ (sec x ) dx

7. ∫ x tan−1 x dx 8. ∫ log2x dx
x

9. ∫ x − sin x dx 10. ∫ log (1 + x 2 ) dx


1 − cos x

11. ∫ e x (tan x + log sec x ) dx 12. ∫ e x 1 + sin x2 cos x  dx


 cos x 
   
13. ∫ log (log x ) + 1 2  dx 14. ∫ e 2x ⋅  1 + sin 2x  dx
 (log x )  1 + cos 2x 

15. ∫ e x (1 − x2) 2 dx 16. ∫ e ⋅ (2 − x )2


2 x 2
dx
(1 + x ) (1 − x ) 1 − x

17. ∫ e ax ⋅ cos (bx + c ) dx 18. ∫ sec3 x dx

19. ∫ sin x dx 20. ∫ ( sin−1 x )2dx


x
21. ∫ cot −1(1− x + x 2 ) dx 22. ∫ sin−1 dx
a+x

x 2 + 1{log ( x 2 + 1) − 2 log x }
24. ∫ cos x + sin 2x 2 dx
2
23. ∫ dx
x 4
(2 cos x − sin x )

 x cos 2 x − sin x 
25. ∫ e sin x   dx
 cos 2 x 
Session 5
Integration Using Partial Fraction
This section deals with the integration of general algebraic From now on, we consider only proper rational functions.
f (x ) f (x )  r(x ) 
rational functions, of the form , where f ( x ) and g( x ) If is not proper, we make it proper   by the
g( x ) g( x )  g( x ) 
are both polynomials. We already have seen some procedure described in (1) above. Let us consider a few
examples of this form. For example, we know how to examples.
1 L( x ) P(x )
integrate functions of the form or or Let g( x ) be a product of non-repeated, linear factors :
Q(x ) Q(x ) Q(x ) g( x ) = L1 ( x ) L2 ( x )K Ln ( x )
where L( x ) is a linear factor, Q ( x ) is a quadratic factor and f (x )
P ( x ) is a polynomial of degree n ≥ 2. We intend to Then, we can expand in terms of partial fractions as
g( x )
generalise that previous discussion in this section.
f (x ) A1 A2 An
We are assuming the scanario where g( x ) (the = + +K +
denominator) is decomposible into linear or quadratic g ( x ) L1 ( x ) L2 ( x ) Ln ( x )
factors. These are the only cases relevant to us right now. where the Ai′s are all constants that need to be determined.
Any linear or quadratic factor in g( x ) might also occur Suppose f ( x ) = x + 1 and g( x ) = ( x − 1) ( x − 2 ) ( x − 3 ). Let
repeatedly. f (x )
Thus, g( x ) could be of the following general forms. us write down the partial fraction expansion of :
g( x )
l g( x ) = L1 ( x ) L2 ( x )K Ln ( x ) (n linear factors) f (x ) x +1 A B C
 n linear factors; the rth  = = + +
l g( x ) = L1 ( x )K Lkr ( x )... Ln ( x )   g( x ) ( x − 1) ( x − 2 ) ( x − 3 ) x − 1 x − 2 x − 3
 factor is repeated k times 
We need to determine A, B and C. Cross multiplying in the
 n linear factors, the ith  expression above, we obtain :
l g( x ) = Lk11 ( x ) Lk22 ( x )... Lknn (x )  
 factor is repeated k i times  ( x + 1) = A ( x − 2 ) ( x − 3 ) + B( x − 1) ( x − 3 )
l g( x ) = L1 ( x ) L2 ( x )... Ln ( x )Q ( x )Q 2 ( x )... Q m ( x ) + C ( x − 1) ( x − 2 )
 n linear factors and  A, B, C can now be determined by comparing coefficients
 
m quadratic factors  on both sides. More simply since this relation that we
have obtained should held true for all x ,we substitute
 a particular quadratic factor 
l g( x ) =K Q rk ( x )...   those values of x that would straight way give us the
 repeats more than once  required values of A, B and C. These values are obviously
l A combination of any of the above the roots of g( x ).
Suppose that the degree of g( x ) is n and that of f ( x ) is m. x =1 ⇒ 2 = A ( −1) ( −2 ) + B(0 ) + C (0 )
If m ≥ n,we can always divide f ( x ) by g( x ) to obtain a ⇒ A =1
quotient q( x ) and a remainder r ( x ) whose degree would x =2 ⇒ 3 = A (0 ) + B (1) ( − 1) + C (0 )
be less than n. ⇒ B = −3
f (x ) r(x ) x =3 ⇒ 4 = A (0 ) + B (0 ) + C (2 ) (1)
= q( x ) + …(i)
g( x ) g( x ) ⇒ C =2
f (x )
If m < n, is termed a proper rational function. Thus, A = 1, B = − 3 and C = 2.
g( x )
f (x )
The partial fraction expansion technique says that a proper We can therefore write as a sum of partial fractions.
g( x )
rational function can be expressed as a sum of simpler
f (x ) 1 3 2
rational functions each possessing one of the factors of = − +
g( x ). The simpler rational functions are called partial g( x ) x − 1 x − 2 x − 3
fractions.
Chap 01 Indefinite Integral 29

f (x ) Again, let x = − 1
Integrating is now a simple matter of integrating the
g( x ) ⇒ 2( − 1) + 1 = A ( − 1 − 2) + B(0)
partial fractions. This was our sole motive in writing such 1
∴ A= ⋅
an expansion, so that integration could be carried out 3
easily. In the example above : 2x + 1 1/3 5/3
∴ = +
f (x ) ( x + 1) ( x − 2) x + 1 x − 2
∫ g( x ) dx = ln ( x − 1) − 3 ln ( x − 2) + 2 ln ( x − 3) + C
1
Now, suppose that g( x ) contains all linear factors, but a y Example 48 Resolve into
( x − 1) ( x + 2) (2x + 3)
particular factor, say L1 ( x ), is repeated k times.
partial fractions.
Thus, g( x ) = Lk1 ( x ) L2 ( x ) K Ln ( x ) 1 A B C
Sol. Let = + + ,
f (x ) ( x − 1) ( x + 2) ( 2x + 3) x − 1 x + 2 2x + 3
can now be expanded into partial fractions as follows
g( x ) where A , B, C are constants.
f (x ) A1 A2 A3 A B2 B 1 = A ( x + 2)(2x + 3) + B( x − 1) (2x + 3) + C ( x − 1)( x + 2) …(i)
= + + +... k k + +...+ n
g ( x ) L 1 ( x ) L 21 ( x ) L 31 ( x ) L1 ( x ) L 2 ( x ) Ln ( x ) For finding A, let x − 1 = 0 or x = 1 in Eq. (i), we get
14444442444444 3
k partial fractions corresponding to L1 (x )
1 = A ( 1 + 2) ( 2 + 3) + B ( 0) + C ( 0)
This means that we will havek terms corresponding to L1 ( x ). 1
The rest of the linear factors will have single corresponding ∴ A=
15
terms in the expansion. Here are some examples.
Similarly, for getting B, let x + 2 = 0 or x = − 2 in Eq. (i), we
1
⇒ get
( x − 1) ( x − 2 )
2
1 = A ( 0) + B ( − 2 − 1) ( − 4 + 3) + C ( 0)
A B C 1
can be expanded as + + ⇒ B=
x − 1 ( x − 1) 2 x −2 3

1 For getting C, let 2x + 3 = 0 or x = − 3 / 2 in Eq. (i), we get



( x − 1) 3 ( x − 2 ) ( x − 3 )  3   3 
1 = A (0) + B (0) + C  − − 1  − + 2
 2   2 
can be expanded as
4
A B C D C ⇒ C =−
⇒ + + + + 5
x − 1 ( x − 1) 2
( x − 1) 3 ( x − 2) ( x − 3)
1 1 1 4
Hence, = + −
1 ( x −1)( x +2) (2x + 3) 15( x − 1) 3( x + 2) 5(2x + 3)

( x − 1) ( x + 5 ) 3
2
3x 3 + 2x 2 + x + 1
can be expanded as y Example 49 Resolve into partial
A B C D E ( x + 1) ( x + 2)
+ + + + fractions.
x − 1 ( x − 1) 2 ( x + 5 ) ( x + 5 ) 2 ( x + 5 ) 3
Sol. This is not a proper fraction. Hence, by division process it
2x + 1 is to be expressed as the sum of an integral polynomial
y Example 47 into partial fractions and a fraction.
( x + 1) ( x − 2)
Now, 3x 3 + 2x 2 + x + 1 = 3x ( x 2 + 3x + 2)
2x + 1
Sol. Here, has Q ( x ) = ( x + 1) ( x − 2) i.e linear − 7 ( x 2 + 3x + 2) + (16x + 15)
( x + 1) ( x − 2)
and non-repeated roots. So, the given polynomial
2x + 1 A B 3x 3 + 2x 2 + x + 1 (16x + 15)
∴ = + = ( 3x − 7 ) + …(i)
( x + 1) ( x − 2) x + 1 x − 2 ( x + 1) ( x + 2) ( x + 1) ( x + 2)
⇒ (2x + 1) = A ( x − 2) + B( x + 1) Now, the second term is proper fraction hence it can be
expressed as a sum of partial fractions.
On putting, x = 2 we get
16x + 15 A B
5 = +
5 = A (0) + B(3) ⇒ B = ( x + 1) ( x + 2) x + 1 x + 2
3
30 Textbook of Integral Calculus

To find A put x + 1 = 0, ie, x = − 1 in the fraction except in 5 8 13


0= A +C + D ⇒ A= − =
the factor ( x + 1). 4 9 36
16 ( − 1) + 15 3x − 2
∴ = A ⇒ A = −1 …(ii) ∴ =
13
+
1
( − 1 + 2) 36 ( x − 1) 6 ( x − 1)2
( x − 1) 2 ( x + 1) ( x + 2)
To find B put x + 2 = 0, ie, x = − 2 in the fraction except in 5 8
the factor ( x + 2). − +
4 ( x + 1) 9 ( x + 2)
16 ( − 2) + 15
∴ = B ⇒ B = 17 …(iii)
( − 2 + 1) Case III When some of the factors in denominator are
1 17 quadratic but non-repeating. Corresponding to each
⇒ The given expression = (3x − 7 ) − + quadratic factor ax 2 + bx + c , we assume the partial
x +1 x +2
Ax + B
[using Eqs. (i), (ii) and (iii)] fraction of the type 2 , where A and B are
Case II When the denominator g ( x ) is expressible as the ax + bx + c
product of the linear factors such that some of them are constants to be determined by comparing coefficients of
repeating. (Linear and Repeated) similar powers of x in numerator of both the sides.
Let Q ( x ) = ( x − a ) k ( x − a 1 ) ( x − a 2 ) … ( x − a r ). Then, we 2x + 7
assume that y Example 52 Resolve into partial
( x + 1) ( x 2 + 4 )
P (x) A1 A2 Ak fractions.
= + + …+
Q ( x ) ( x − a) ( x − a) 2
( x − a) k 2x + 7 A Bx + C
Sol. Let = + 2
B1 B2 Br ( x + 1) ( x + 4 )
2
x +1 x + 4
+ + + …+
( x − a1 ) ( x − a 2 ) (x − ar ) ∴ 2x + 7 = A ( x 2 + 4 ) + ( Bx + C ) ( x + 1)
Put x = −1
x+5 ∴ 5 = 5A or A = 1
y Example 50 Expression has repeated
( x − 2) 2 Comparing the terms, 0 = A + B ⇒ B= −1
(twice) linear factors in denominator, so find partial 7 = 4A + C ⇒ C =3
fractions. 2x + 7 1 ( − x + 3)
∴ = + 2
x +5 A B ( x + 1) ( x 2 + 4 ) x + 1 x +4
Sol. Let = +
( x − 2) 2
( x − 2) ( x − 2) 2 Aliter To obtain values of A , B and C from
∴ ( x + 5) = A ( x − 2) + B 2x + 7 = A ( x 2 + 4 ) + ( Bx + C ) ( x + 1)
Comparing the like terms, A = 1, − 2A + B = 5 or B = 7 i.e., 2x + 7 = ( A + B ) x 2 + ( B + C ) x + 4 A + C
x +5 1 7
∴ = + Equating the coefficients of identical powers of x, we get
( x − 2) 2 ( x − 2) ( x − 2) 2 A + B = 0, B + C = 2 and 4 A + C = 7.
3x − 2 Solving, we get A = 1, B = − 1, C = 3
y Example 51 Resolve into
( x − 1) ( x + 1) ( x + 2)
2
y Example 53 Find the partial fraction
partial fractions. 2x + 1
3x − 2 ⋅
Sol. Let ( 3x + 2) (4 x 2 + 5x + 6 )
( x − 1) 2 ( x + 1) ( x + 2)
2x + 1 A Bx + C
A B C D Sol. Let = + ,
= + + + ( 3x + 2) ( 4 x 2 + 5x + 6) ( 3x + 2) ( 4 x 2 + 5x + 6)
( x − 1) ( x − 1) 2
( x + 1) ( x + 2)
then 2x + 1 = A ( 4 x 2 + 5x + 6) + ( Bx + C ) (3x + 2)
∴ 3x − 2 = A ( x − 1) ( x + 1) ( x + 2) + B ( x + 1) ( x + 2)
where A , B, C are constants.
+ C ( x − 1) 2 ( x + 2) + D ( x − 1) 2 ( x + 1)
For A, let 3x + 2 = 0,
Putting ( x − 1) = 0, we get B = 1 / 6,
i.e., x = − 2/3
Putting ( x + 1) = 0, we get C = − 5 / 4
 2  4 10    2 
Putting ( x + 2) = 0, we get D = 8 / 9 2  −  + 1 = A  4⋅ − + 6  +  B  −  + C  (0)
 3  9 3    3  
Now, equating the coefficient of x 3 on both the sides, we get
1  40  3
− =A  ⇒ A=−
3  9  40
Chap 01 Indefinite Integral 31

Comparing coefficients of x 2 and constant term on both the 1 3 log | 1 − 2x |


= x + log | x | + +C
sides for B and C, we get 2 2 −2
4 A + 3B = 0,
1 3
4 1 = x + log | x | − log |1 − 2x | + C .
∴ B=− A ⇒ B= and 6A + 2C = 1, 2 4
3 10
1 − 6A 3x −1 A B
∴ C = ⇒C =
29 (ii) Let, = +
2 40 ( x − 2) 2 ( x − 2) ( x − 2) 2
 29 
x +  ⇒ 3x − 1 = A ( x − 2) + B, …(i)
2x + 1 −3  4
∴ = + On putting x = 2 in Eq. (i), we get B = 5.
(3x + 2) ( 4 x + 5x + 6) 40 (3x + 2) 10 ( 4 x + 5x + 6)
2 2
On equating coefficients of x on both sides of (i), we
Case IV When some of the factors of the denominator get A = 3,
are quadratic and repeating. For every quadratic repeating 3x − 1  3 5 
factor of the type (ax 2 + bx + c ) k , we assume : ∴ ∫ ( x −2)2 dx = ∫  ( x − 2) + ( x −2)2  dx
A1 x + A2 A3 x + A4 A2k − 1 x + A2k 5
+ +… + = 3 log | x − 2 | − +C
ax + bx + c (ax + bx + c )
2 2 2
(ax 2 + bx + c ) k x −2
x2 + x + 1 A B C
(iii) Let, = + +
2x + 2x + x + 1
4 2
x 2 ( x + 2) x x2 x + 2
y Example 54 Resolve into partial
x ( x 2 + 1) 2 ⇒ x 2 + x + 1 = Ax ( x + 2) + B( x + 2) + C ( x 2 ) …(i)
fractions. On putting, x = − 2 and x = 0 in Eq. (i), we get
2x 4 + 2x 2 + x + 1 A Bx + C Dx + E C =3 / 4 and B =1 / 2
Sol. Let = + 2 + 2
x ( x 2 + 1) 2 x x + 1 ( x + 1) 2 On equation Coefficient of x 2 on both sides of (i),
or 2x 4 + 2x 2 + x + 1 = A ( x 2 + 1)2 + ( Bx + C ) x ( x 2 + 1) we get 1 = A + C ⇒ A = 1 / 4.
+ ( Dx + E ) x x2 + x + 1 1 / 4 1 / 2 3/ 4 
∴ ∫ x 2 ( x + 2) dx = ∫  x
+ 2
x
+
x + 2
 dx
Comparing coefficients of x 4 , x 3 , x 2 , x and constant term
1 1 3
∴ A + B = 2, C = 0, 2A + D + B = 2, E = 1, A = 1 = log | x | − + log | x + 2| + C
4 2x 4
∴ we get A = 1, B = 1, C = 0, D = − 1, E = 1 8 A Bx +C
(iv) Let, = +
Hence, the partial fraction, ( x + 2) ( x 2 + 4 ) x + 2 x 2 +4
2x 4 + 2x 2 + x + 1 1 x 1− x
= + + ⇒ 8 = A ( x + 4 ) + ( Bx + C ) ( x + 2)
2
…(i)
x ( x 2 + 1) 2 x 1 + x 2 (1 + x 2 )2
On putting x = −2 in Eq. (i), we get A = 1.
y Example 55 Evaluate the following integrals: On equating coefficient of x 2 on both sides we get,
(1 − x 2 ) dx 3x − 1 0= A + B ⇒B = −1
(i) ∫ (ii) ∫ dx On equating constant term on both sides, we get,
x (1 − 2 x ) (x −2) 2
8 = 4 A + 2C ⇒ C = 2
x2 + x +1 8dx  1 ( − x + 2) 
(iii) ∫ x 2 (x + 2) dx (iv) ∫ (x + 2) (x 2 + 4) ∴
8
∫ ( x + 2) ( x 2 + 4 ) dx = ∫  x + 2 +  dx
x2 + 4 
1 − x2 1 A B 1 1 2x dx dx
Sol. (i) Let, = + +
x ( 1 − 2x ) 2 x ( 1 − 2x )
= ∫ x + 2 dx − 2 ∫ x 2 + 4 + 2 ∫ x 2 + 4
1 1 1 x
⇒ (1 − x 2 ) = x (1 − 2x ) + A (1 − 2x ) + B( x ) = log | x + 2| − log | x 2 + 4| + 2 . tan −1   + C.
2 2 2  2
1
On putting x = 0 and x = , we get
1
1
2
1 3
y Example 56 Evaluate ∫ sin x − sin 2x dx.
1 = A and 1 − = B . ⇒ A = 1, B =
4 2 2 1 1
1 − x2 1 1 3 
Sol. Let I = ∫ sin x − sin 2x dx = ∫ sin x − 2 sin x cos x dx
∴ ∫ x (1 − 2x ) dx = ∫  2 + x + 2(1 − 2x ) dx 1 sin x
= ∫ sin x (1 − 2 cos x ) dx = ∫ sin 2 x (1 − 2 cos x ) dx
32 Textbook of Integral Calculus

y Example 58 Evaluate ∫ sin 4 x ⋅e tan


2
sin x x
= ∫ (1 − cos 2 x ) (1 − 2 cos x ) dx dx .
Sol. The given integral could be written as,
(put cos x = t ⇒ − sin x dx = dt )
I = ∫ 4 sin x ⋅ cos x ⋅ cos 2x ⋅ e tan x dx
2
− dt −1
∴ I = ∫ (1 − t 2 ) (1 − 2t )
= ∫
(1 − t ) (1 + t ) (1 − 2t )
dt …(i)
= 4 ∫ tan x ⋅ cos 2 x (cos 2 x − sin 2 x ) ⋅ e tan x dx
2

Here, in Eq. (i) we have linear and non-repeated factors thus


= 4 ∫ tan x ⋅ cos 4 x (1 − tan 2 x ) ⋅ e tan
2
x
we use partial fractions for; dx
−1 A B C
= + + tan x
∫ (sec 2 x )2 ⋅ (1 − tan
2

(1 − t ) (1 + t ) (1 − 2t ) (1 − t ) (1 + t ) (1 − 2t ) =4 2
x ) ⋅ e tan x
dx
or − 1 = A (1 + t ) (1 − 2t ) + B(1 − t ) (1 − 2t ) + C (1 − t ) (1 + t )
Put tan 2 x = t
...(ii)
Putting (t + 1) = 0 or t = − 1, we get
1 ⇒ 2 tan x ⋅ sec 2 x dx = dt
− 1 = B ( 2) ( 1 + 2) ⇒ B = −
6 (1 − t ) e t dt (1 − t ) e t
Putting (1 − t ) = 0 or t = 1, we get
1
⇒ I =4 ∫ (1 + t )3 2 ∫ (1 + t )3 dt
⋅ = 2
− 1 = A ( 2) ( − 1) ⇒ A =
2  2e t − (1 + t ) e t 
Putting (1 − 2t ) = 0 or t = 1 / 2, we get = 2 ∫ dt 
 (1 + t )3 
 1  1 4
− 1 = C 1 −  1 +  ⇒ C = −  2 1 
 2  2 3 = 2 ∫ et −  dt
−1 1 1 4  (1 + t ) 3
(1 + t )2 
∴ = − −
(1 − t ) (1 + t ) (1 − 2t ) 2 (1 − t ) 6 (1 + t ) 3 (1 − 2t ) 1
= −2 et + C
So, Eq. (i) reduces to (1 + t )2
1 1 1 1 4 1 [ using ∫ e x ( f ( x )) + f ′ ( x ))dx = e x . f ( x ) + c ]
I = ∫ dt − ∫ dt − ∫ dt
2 1−t 6 1+t 3 1 − 2t 2
2e tan x
1 1 4 1
= − log | 1 − t | − log | 1 + t | − × − log | 1 − 2t | + C =− +C
2 6 3 2 (1 + tan 2 x )2
1 1 2
= − log | 1 − cos x | − log | 1 + cos x | + log
2
I = − 2 cos 4 x ⋅ e tan x
+C
2 6 3
| 1 − 2 cos x | + C
1 + x cos x
(1 − x sin x ) dx
y Example 59 Solve ∫ x (1 − x 2e 2 sin x ) dx.
y Example 57 Evaluate ∫ x (1 − x 3e 3 cos x ) ⋅
1 + x cos x
(1 − x sin x ) dx
Sol. Let I = ∫ x (1 − x 2e 2 sin x ) dx
Sol. Here, I = ∫ x (1 − ( xe cos x )3 )
Put ( x e sin x ) = t
Put xe cos x
=t
Differentiating both the sides, we get
⇒ ( xe cos x ⋅ ( − sin x ) + e cos x ) dx = dt
( x e sin x ⋅ cos x + e sin x ) dx = dt
dt dt
∴ I =∫ =∫
t (1 − t 3 ) t (1 − t ) (1 + t + t 2 ) ⇒ e sin x ( x cos x + 1) dx = dt
A Ct + D  dt
= ∫ +
B
+  dt
⇒ I = ∫ t (1 − t 2 )
t 1 − t 1 + t + t2
dt
Comparing coefficients, we get
1 2 1
= ∫ t (1 − t ) (1 + t ) [using partial fraction]
A = 1, B = , C = − , D = −
3 3 3 1 1 1 
 2 1 = ∫ + −  dt
 − t −   t 2 (1 − t ) 2 (1 + t ) 
dt 1 dt  3 3
∴ I =∫ + ∫ +∫ dt 1 1
t 3 1−t 1 + t + t2 = log | t | − log | 1 − t | − log | 1 + t | + C
2 2
1 1
= log | t | − log | 1 − t | − log | 1 + t + t 2 | 1
3 3 = log | x e sin x | − log | 1 − x 2e 2 sin x | + C
[where, t = xe cos x ] 2
Chap 01 Indefinite Integral 33

y Example 60 Evaluate; dx
= ∫ x {1 + log e x } {2 + log e x } {3 + log e x }
1
∫ x {log e .log e .log e } dx
ex e 2x e 3x
1
Put loge x = t ⇒ dx = dt
Sol. We have, x
dt 1 1 1 1 
1
I = ∫ {log e ex . log e e x .loge e 3 x } dx I = ∫ (1 + t )(2 + t )(3 + t ) = ∫  2 . (1 + t ) − (2 + t ) + (3 + t ) dt
2

x
1  1 1 1  [using partial fraction]
=∫  . . 3 
dx
x  log e ex e 2
x
log e e x  1
log e = log |1 + t | − log |2 + t | + log |3 + t | + C .
2
dx
=∫ 1
e x e2 2 3
x {log e + log e } {log e + log e x } {log e e + log x 2 } = log |1 + log e x | − log |2 + log e x | + log |3 + log e x | + C .
2

Exercise for Session 5


n Evaluate the following Integrals :

x2
1. ∫ dx 2. ∫ dx 3
(x − 1) ( x − 2) ( x −3) 1+ x

3. ∫ dx
4. ∫ 2 2x 2
x ( x n + 1) ( x + 1) ( x + 3)

cos x dx
5. ∫ dx 6. ∫ dx
(1 + sin x ) (2 + sin x ) sin x (3 + 2 cos x )

tan x + tan3 x
7. ∫ sec x dx 8. ∫ dx
1 + cosec x 1 + tan3 x

dx tan−1
9. ∫ 10. ∫ . dx
x | 6 (log x ) + 7 log x + 2 |
2
x2
Session 6
Indirect and Derived Substitutions
Indirect and Derived Substitutions =−
2 2  1 
log | 1 + y | + C = log + C
5 5 1 +y
(i) Indirect Substitution 2  x 5/2 
= log 5 / 2 + C …(i)
If the integral is of the form f ( x ) ⋅ g ( x ), where g ( x ) is a 5 x + 1
function of the integral of f ( x ), then put integral of
 xk 
f (x ) = t. where, I = a log   +C (given) …(ii)
 1 + xk 
d ( x 2 + 1) ∴ From Eqs. (i) and (ii), we get
y Example 61 The value of ∫ x2 +2
, is
 xk  2  x 5/2 
a log   + C = log   +C
 1 + xk  5  1 + x 5/2 
(a) 2 x 2 + 2 + C (b) x 2 + 2 + C
⇒ a = 2 / 5 and k = 5 / 2
(c) x x 2 + 2 + C (d) None of these
Hence, (a) is the correct answer.
d ( x 2 + 1)
Sol. Here, I = ∫ 5x 4 + 4 x 5
x2 + 2 y Example 63 Evaluate ∫ ( x 5 + x + 1)2 dx .
We know, d ( x 2 + 1) = 2x dx
5x 4 + 4 x 5 x 4 (5 + 4 x )
∴ I = ∫
2x dx Sol. Here, I = ∫ ( x 5 + x + 1) 2 dx = ∫ 2
dx
x2 + 2  1 1
1 + 4 + 5 
10
x
 x x 
Put, x2 + 2 = t2
5/ x6 + 4 / x5
∴ 2x dx = 2t dt ⇒ I = ∫
2t dt
= 2t + C = ∫ 2
dx
t  1 1
1 + 4 + 5 
 x x 
⇒ I = 2 x2 + 2 + C
1 1
Hence, (a) is the correct answer. Put 1+ 4 + 5 =t
x x
( x )5  xk   4 5
y Example 62 If ∫ ( x )7 + x 6 dx = a log  1 + x k  + c ,

⇒  − 5 − 6  dx = dt
 x x 
dt 1 1
then a and k are I = ∫− 2 = +C = +C
t t 1 1
(a) 2 / 5, 5 / 2 (b) 1 / 5, 2 / 5 1+ 4 + 5
x x
(c) 5 / 2, 1 / 2 (d) 2 / 5, 1 / 2
x5
( x )5 dx = 5 +C
Sol. Here, I = ∫(
x) + x 7 6
dx = ∫( x ) + ( x )7
2
x + x +1

=∫
dx
,
y Example 64 For any natural number m, evaluate
7/2  1 
∫ (x + x 2m + x m ) (2x 2m + 3x m + 6)1/m dx , x > 0
3m
x 1 + 
 x 5/2  [IIT JEE 2002]
1 5
Put 5 / 2 = y ⇒ − 7 / 2 dx = dy Sol. Here, I = ∫ ( x 3m
+x 2m
+ x ) ( 2x
m 2m
+ 3x m
+ 6)1/m dx
x 2x
(2x 3m + 3x 2m + 6x m )1/m
I =− ∫
2 dy = ∫ ( x 3m + x 2m + x m ) dx
5 1+y x
= ∫ ( x 3m −1+ x 2m − 1+ x m − 1 ) (2x 3m + 3x 2m + 6x m )1/m dx ...(i)
Chap 01 Indefinite Integral 35

Put 2x 3m + 3x 2m + 6x m = t y Example 67 The evaluation of


⇒ 6m ( x 3m − 1 + x 2m − 1 + x m − 1 ) dx = dt p x p + 2q − 1 − qx q − 1
∴ Eq. (i) becomes, ∫ x 2p + 2q + 2x p + q + 1 dx is
dt 1 t (1/m ) + 1
I = ∫ t 1/m = ⋅ +C xp xq
6m 6m (1 / m ) + 1 (a) − p+q
+C (b) p+q
+C
m +1 x +1 x +1
1
I = {2x 3m + 3x 2m + 6 x m} m +C xq xp
6 ( m + 1) (c) − +C (d) +C
xp + q + 1 xp + q + 1
x dx px p + 2q − 1 − qx q − 1
y Example 65 ∫ is equal to Sol. Here, I = ∫ x 2 p + 2q + 2x p + q + 1
1 + x + (1 + x )
2 2 3

1 px p + 2q − 1 − qx q − 1 px p − 1 − qx − q − 1
(a) ln (1 + 1 + x 2 ) + C (b) 2 1 + 1 + x 2 + C ∫ ( x p + q + 1) 2
dx = ∫ ( x p + x −q ) 2
dx
2
(c) 2 (1 + 1 + x 2 ) + C (d) None of these Taking x q as x 2q common from denominator and take it in
numerator.
x dx
Sol. ∫ Put x p + x −q = t ⇒ ( px p − 1 − qx −q − 1 ) dx = dt
(1 + x ) 1 + 1 + x 2
2
 
dt 1 xq
2x
∴ I = ∫ t2 =−
t
+ C = −  p +q
x
 +C
+ 1
Put 1 + 1 + x2 = t2 ⇒ dx = 2t dt
2 1 + x2 Hence, (c) is the correct answer.


x dx
= 2t dt x 2 (1 − ln x )
1+ x 2 y Example 68 ∫ ln 4 x − x 4
dx is equal to

2t dt 1  x  1

t
I = ∫
= 2t + C = 2 1 + 1 + x 2 + C (a) ln   − ln (ln 2 x − x 2 ) + C
2  ln x  4
Hence, (b) is the correct answer.
1  ln x − x  1 −1  ln x 
(b) ln   − tan   + C
(2x + 1) 4  ln x + x  2  x 
y Example 66 ∫ ( x 2 + 4 x + 1)3/ 2 dx 1  ln x + x  1 −1  ln x 
(c) ln   + tan   + C
x 3
x 4  ln x − x  2  x 
(a) +C (b) +C
(x + 4 x + 1)
2 1/ 2
(x + 4 x + 1)1 / 2
2
1   ln x − x  −1  ln x  
(d) ln   + tan    + C
x 2
1 4   ln x + x   x 
(c) +C (d) +C
(x 2 + 4 x + 1)1 / 2 (x 2 + 4 x + 1)1 / 2 x 2 (1 − ln x )
2x + 1 2x + 1
Sol. Here, I = ∫ ln x 4 x − x 4
Sol. ∫ ( x 2 + 4 x + 1)3 / 2 dx = ∫ 3/2
dx
 4 1 x 2 (1 − ln x ) 1 − ln x
x 1 + + 2 
3
 x x 
I = ∫  4 
dx = ∫   ln x  4 
dx
4   ln x 
 − 1 x    − 1
2
2x −2 + x −3 x 
= 3/2 ∫
dx  x    x  
 4 1
1 + + 2  ln x 1 − ln x
 x x  Put =t ⇒ = dt
x x2
1 4  2 4
Now, put 2 + + 1 = t 2 ⇒  − 3 − 2  dx = 2t dt dt dt
x x  x x  I = ∫ ( t 4 − 1) = ∫ ( t 2 + 1) ( t 2 − 1)
−t dt 1
∴ I = ∫ 3 = +C
t t 1 ( t 2 + 1) − ( t 2 − 1)
2 ∫ ( t 2 + 1) ( t 2 − 1)
= dt
x
= +C
x 2 + 4x + 1
1  dt dt  1  1 t − 1 
2  ∫ t 2 − 1 ∫ t 2 + 1 2  2 t + 1
Hence, (b) is the correct answer. I =  −  =  ln − tan −1 t 

36 Textbook of Integral Calculus

1  ln x − x  1 −1  ln x  xn − 1
∫ x g ( x ) dx = ∫ (1 + nx n )1/n dx
n−2
= ln   − tan   +C ∴
4  ln x + x  2  x 
1
= ∫n ⋅ x n − 1 ⋅ (1 + nx n )−1/n dx
Hence, (b) is the correct answer. 2
n2
x2 −1
∫ x3
1
y Example 69 1−
dx is equal to 1 (1 + nx n ) n
2x − 2x + 1
4 2
= 2⋅
1
+C
[IIT JEE 2006] n 1−
2x 4 − 2x 2 + 1 2x 4 − 2x 2 + 1 n
(a) +C (b) +C 1−
1

x2 x3 (1 + nx n ) n
= +C
2x 4 − 2x 2 + 1 2x 4 − 2x 2 + 1 n ( n − 1)
(c) +C (d) +C
x 2x 2 Hence, (a) is the correct answer.
x2 − 1
Sol. Let I = ∫ x3 2x 4 − 2x 2 + 1
dx
Derived Substitutions
x −1
2
dx Some times it is useful to write the integral as a sum of
= ∫ x5

2 1 two related integrals which can be evaluated by making
2− + suitable substitutions.
x2 x4
4 4 Examples of such integrals are

1 3
x5 1 2 1
=
4 ∫ x
2 1
dx =
4
⋅2 2 − 2 + 4
x x
2− +
x2 x4
 f ′( x ) 
Type I
Q ∫ dx = 2 f (x ) + C
 f (x )  (a) Algebraic Twins
2x 4 − 2x 2 + 1 2x 2 x2 +1 x2 −1
= +C
2x 2
∫ x 4 +1 dx = ∫
x 4 +1
dx + ∫
x 4 +1
dx
Hence, (d) is the correct answer.
2 x2 +1 x2 −1
y Example 70 Let f ( x ) =
x
for n ≥ 2 and ∫ x 4 +1 dx = ∫
x 4 +1
dx − ∫
x 4 +1
dx
(1 + x ) n 1/n

∫ x g ( x ) dx equals to 2x 2
n−2
g ( x ) = fofo K of ( x ) , then 2
14243 ∫ x 4 + 1 + kx 2 dx , ∫ ( x 4 + 1 + kx 2 ) dx
n times
[IIT JEE 2007]
1 1
(a)
1 1−
(1 + nx n ) n + C (b)
1 1−
(1 + nx n ) n + C (b) Trigonometric Twins
n (n − 1) n−1
1+
1
1+
1 ∫ tan x dx , ∫ cot x dx ,
1 1
(c) (1 + nx n ) n + C (d) (1 + nx n ) n + C 1 1
n(n + 1) n+1 ∫ (sin 4 x + cos 4 x ) dx , ∫ sin 6 x + cos 6 x dx ,
x
Sol. Q f (x ) = ± sin x ± cos x
(1 + x n )1/n ∫ a + b sin x cos x dx
y
∴ f ( f ( x )) = ,
(1 + x n )1/n Method of evaluating these integral are illustrated by
x x mean of the following examples :
Where, y= =
(1 + x ) n 1/n
(1 + 2x n )1/n

Similarly, f ( f ( f ( x ))) =
x Integral of the Form
(1 + 3x n )1/n  1  1 
x
1. ∫f  x +   1 − 2  dx
 x   x 
and ( fofofo ... of ) ( x ) = g ( x ) =
1442443 (1 + nx n )1/n 1  1 
n times Put x + = t ⇒  1 − 2  dx = dt
x  x 
Chap 01 Indefinite Integral 37

 1  1  1 1 + x2 1 1 − x2
2. ∫ f  x −   1 + 2  dx .
 x  x 
⇒ I =
2 ∫ x 4 + 5x 2 + 1 dx +
2 ∫ x 4 + 5x 2 + 1 dx
1  1  1 1 + 1/ x2 1 1 − 1/ x2
Put x − = t ⇒  1 + 2  dx = dt

= ∫ x 2 + 5 + 1 / x 2 dx − 2 ∫ x 2 + 5 + 1 / x 2 dx
x x  2

x2 +1 [(dividing Numerator and Denominator by x 2 )]


3. ∫ x 4 + kx 2 + 1 dx . 1 (1 + 1 / x 2 ) 1 (1 − 1 / x 2 )
= ∫
2 ( x − 1 / x )2 + 7
dx − ∫
2 ( x + 1 / x )2 + 3
dx
Divide numerator and denominator by x 2 .
1 dt 1 du
x2 −1 = ∫ − ∫ 2
4. ∫ x 4 + kx 2 + 1 dx 2 t + ( 7)
2 2
2 u + ( 3 )2
1 1
2 where t = x −and u = x +
Divide numerator and denominator by x . x x
1 1  −1 t  1 1  −1 u 
5 ∴ I = ⋅  tan  − ⋅  tan  +C
y Example 71 Evaluate ∫ 1 + x 4 dx. 2 7  7 2 3  3
1 1 x − 1/ x 1  x + 1 / x 
=  tan −1   − tan −1   +C
Sol. Let I = ∫
55
dx = ∫
2
dx 2 7  7  3  3  
1 + x4 2 1 + x4

=
5  1 + x2
∫ dx +
1 − x2 
∫ 1 + x 4 dx 
y Example 73 Evaluate ∫ tan x dx .
2  1 + x4  Sol. Here I = ∫ tan x dx
5  x2 + 1 x2 − 1 
= ∫ 4 dx − ∫ x 4 + 1 dx  Put tan x = t 2
2  x +1  2t dt
⇒ sec 2 x dx = 2t dt ⇒ dx =
1 + t4
Remark
2t 2t 2
Here, dividing Numerator and Denominator by x 2 and ∴ I = ∫t ⋅ dt = ∫ t 4 + 1 dt
converting Denominator into perfect square so as to get 1+t 4

differential in Numerator
t +1
2
t2 − 1
5  1 + 1/ x2 1 − 1/ x2 
= ∫ t4 + 1 dt + ∫ t 4 + 1 dt
2 ∫ x 2 + 1 / x 2 ∫ x 2 + 1 / x 2 dx 
i.e. I =  dx −
 1 + 1/t2 1 − 1/t2
5 1 + 1/ x2 1 − 1/ x2 
= ∫ t 2 + 1 / t 2 dt + ∫ t 2 + 1 / t 2 dt
= ∫ dx − ∫ dx 
2  (x − 1 / x ) + 2
2
( x + 1 / x )2 − 2  1 + 1/t2 1 − 1/t2
= ∫ (t − 1 / t )2 + 2 dt + ∫ (t + 1 / t )2 − 2 dt
5 dt du 
2 ∫ t 2 + ( 2 ) 2 ∫ u 2 − ( 2 ) 2 
=  − , ds dr  1 1
I = ∫ s 2+ ( 2) 2
+ ∫ r 2− ( , s = t − and r = t + 
2
2)  t t
1 1
[where t = x −and u = x + ]
x x 1  s  1 r − 2
= tan − 1   + log + C
51  t  1 u − 2  2  2  2 2 r + 2
=  tan − 1   − log  + C
2 2  2 2 2  u + 2    1 
1  − 1 t − 1 / t  1 t + − 2
=  + log  t  + C
  x + 1 − 2  tan 
2  2  2 t + 1 + 2
51  −   + C
log
x 1 / x 1  
∴ I =  tan −1  − x
  t
2 2  2  2 2  1 
x+ + 2  [(where t = tan x )]
  x 
4
y Example 72 Evaluate
1
∫ x 4 + 5x 2 + 1 dx. y Example 74 Evaluate ∫ sin 4 x + cos 4 x dx.
dx
Sol. Let I =
1 2
∫ x 4 + 5x 2 + 1 dx Sol. Let I =4 ∫ sin 4 x + cos 4 x
2
38 Textbook of Integral Calculus

Dividing numerator and denominator by cos 4 x, we get 1+


1
1+
1
y2 + 1 y2 y2
∫ y 4 + 1 dy = ∫ ∫
4
sec x I = dy =
∫ tan 4 x + 1 dx
I =4 2
dy
1  1
y2 +
y2 y −  + 2
 y
sec 2 x (1 + tan 2 x )
I =4 ∫ 1 + tan 4 x
dx
1  1
Put, y − = t ⇒ 1 + 2  dy = dt
y  y 
Put tan x = t ⇒ sec 2 x dx = dt
dt 1  −1 t 
∴ I =4
t2 + 1
∫ t4 + 1 dt
I = ∫ t2 + 2 = 2
 tan

 +C
2
 1  x 1 
1 + 1/t2 1 + 1/t2 y −  x − x 
=4 ∫ t2 + 1/t2 dt = 4 ∫ (1 − 1 / t )2 + 2 dt
=
1
tan −1 
y
+C=
1
tan −1  x  +C
2  2  2  2 
   
Again, put z =t −
1  
t
dz 4  z 
∴ I =4 ∫ z2 + 2 = 2
tan − 1   + C
 2
y Example 77 Evaluate
( x 2 − 1) dx
 tan x − 1 / tan x 
I = 2 2 tan − 1   +C ∫ −1  1 
.
 2  ( x + 3x + 1) tan
4 2
 x+ 
 x
(ax 2 − b ) dx ( x 2 − 1)
y Example 75 The value of ∫x c x − (ax + b )
2 2 2 2
, is Sol. Here I =

∫ ( x 4 + x 3 + 1) tan −1  x + x 
1

equal to The given integral can be written as


1  b  b (1 − 1 / x 2 ) dx
(a) sin −1 ax  + k (b) c sin −1 ax +  + k I = ∫
c  x  x  1 
( x 2 + 3 + 1 / x 2 ) tan − 1  x + 
 x 
ax + b / x 
(c) sin −1   +k (d) None of these
 c  (dividing numerator and denominator by x 2 )
 b (1 − 1 / x 2 ) dx
a − 2  dx I = ∫
(ax 2 − b ) dx  x   1 
{( x + 1 / x )2 + 1} tan − 1  x + 
Sol. Here, I = ∫x c 2 x 2 − (ax 2 + b )2
= ∫ 2  x 
 b
c 2 − ax +  1  1 
 x Put, x + = t ⇒  1 − 2  dx = dt
x  x 
b
Put ax + =t dt
x ∴ I = ∫ 2 …(i)
 b (t + 1) ⋅ tan − 1 (t )
∴ a − 2  dx = dt
 x  Now, make one more substitution
dt
dt t  tan −1 t = u . Then, 2 = du
I = ∫ c −t2 2
= sin −1   + k
c  t +1
du
 ax + b / x  ∴ Eq. (i) becomes, I = ∫ = log | u | + C
⇒ sin −1   +k u
 
c ⇒ I = log | tan − 1 t | + C = log | tan − 1 ( x + 1 / x ) | + C
Hence (c) is the correct answer.
( x − 7 / 6 − x 5/ 6 ) dx
x x ( x 2x + 1) (ln x + 1) y Example 78 ∫ x 1/ 3 ( x 2 + x + 1)1/ 2 − x 1/ 2 ( x 2 + x + 1)1/ 3
y Example 76 ∫ x 4x
+1
dx

x 2x
+ 1) (ln x + 1) x 7 / 6 ( x − 7 / 6 − x 5 / 6 ) dx
∫ x 7 / 6 ⋅ x 1/ 3 ( x 2 + x + 1)1/ 2 − x 1/ 2 ⋅ x 7 / 6 ( x 2 + x + 1)1/ 3
x (x Sol. I =
Sol. I = ∫ x 4x + 1
dx

Put x x = y ⇒ x x (ln x + 1) dx = dy (1 − x 2 ) dx
= ∫ x 3 / 2 ( x 2 + x + 1)1/ 2 − x 5 / 3 ( x 2 + x + 1)1/ 3
Chap 01 Indefinite Integral 39

 1   1  The following examples illustrate the procedure :


−  1 − 2  dx
 x  Putting x + x = t 
= ∫ 1/ 2 1/ 3
 
⇒ 1 − 1  dx = dt  y Example 80 Evaluate ∫ ( x + 1)
1
dx .
 1   1 
 x + + 1 −  x + + 1   x −2
 x   x  x2 
1
=−
dt
∫ (t + 1)1/ 2 − (t + 1)1/ 3
Sol. Let I = ∫ ( x + 1) x −2
dx

Here, P and Q both are linear, so we put Q = t 2


Substitute, (t + 1) = u 6
i.e. x − 2 = t2
6u 5 du u3
=−∫ 3 = − 6 ∫ u − 1 du, So that dx = 2t dt
u − u2
1 dt
Put u −1=z ∴ I = ∫ ( t 2 + 2 + 1) t 2
⋅ 2t dt = 2 ∫
t2 + 3
( z + 1) 3
dz = − 6 ∫ dz
z 1  t 
=2× tan − 1   + C
z 3 + 3z 2 + 3z + 1 3  3
= −6∫ dz
z 2  x − 2
 1 ∴ I = tan − 1   +C
= − 6 ∫  z 2 + 3z + 3 +  dz 3  3 
 z 
 z 3 3z 2  φ (x )
= −6  + + 3z + log | z |  + C (b) Integrals of the Form ∫ dx, where P is a
 3 2  P Q
1/ 6
 1  quadratic expression and Q is a linear expression
where, z =  x + +1 −1
 x  To evaluate this type of integrals we put Q = t 2 i.e. to
evaluate the integrals of the form
y Example 79 The value of ∫ {{[ x ]}} dx , where {.} and
1
[.] denotes fractional part of x and greatest integer ∫ (ax 2 + bx + c ) px + q dx
function, is equal to put px + q = t 2 .
(a) 0 (b) 1 (c) 2 (d) –1
x+2
Sol. Let I = ∫ {{[ x ]}} dx y Example 81 Evaluate ∫ ( x 2 + 3x + 3) x+1
dx .
where, [ x ] = Integer and we know {n} = 0;n ∈ Integer.
I = ∫ 0 dx = 0 x +2
∴ Sol. Let I = ∫ ( x 2 + 3x + 3) x +1
dx
Hence, (a) is the correct answer.
Put x + 1 = t 2 ⇒ dx = 2t dt
( t 2 − 1) + 2
Type II ∴ I = ∫ {(t 2 − 1)2 + 3 (t 2 − 1) + 3} t2
⋅ (2t ) dt

t2 + 1 1 + 1/t2
Integration of Some Special = 2∫ dt = 2 ∫ dt
t4 + t2 + 1 t2 + 1 + 1/t2
Irrational Algebraic Functions 1 + 1/t2 du
In this case we shall discuss four integrals of the form =2∫ dt = 2 ∫
(t − 1 / t ) + ( 3 )
2 2
u + ( 3 )2
2
φ (x )
∫ P Q dx , where P and Q are polynomial functions of x  1
where u = t − t 
and φ ( x ) is polynomial in x . =
2 u 
tan − 1   + C
φ (x ) 3  3
(a) Integrals of the Form ∫ dx, where P and Q
P Q 2  t 2 − 1
are both linear of x ∴ I = tan −1   +C
3  3t 
To evaluate this type of integrals we put Q = t 2 , i.e. to  
2 x
1 = tan −1   +C
evaluate integrals of the form ∫ dx , put 3  3 ( x + 1) 
(ax + b ) cx + d
cx + d = t 2 .
40 Textbook of Integral Calculus

1 dx 1 2z dz dz
2 ∫ ( z 2 + 1 + 1) z 2
(c) Integral of the Form ∫ ⋅ , ∴ I =− =−∫ 2
(ax + b ) px + qx + r
2 ( z + 2)
dx
where in ∫ P is linear and Q is a quadratic put, 1  z 
P Q I =− tan − 1   + C
1 2  2
ax + b = ⋅
t 1  u − 1
dx I =− tan − 1   +C
y Example 82 Evaluate ∫ ( x − 1) x2 + x +1
⋅ 2  2 

1  t 2 − 1
=− tan − 1   +C
dx  2 
Sol. Let I = ∫ ( x − 1) x2 + x + 1
2 

1  1 − x2 
1 1 =− tan − 1   +C
Put x − 1 = ⇒ dx = − 2 dt 2  2x 
t t 
− 1 / t 2 dt Aliter Put x = cos θ, dx = − sin θ dθ
∴ I =∫
2 sin θ dθ dθ
1 
1 / t  + 1 +  + 1 + 1
1  ∴ I =−∫ =−∫
t  t  (1 + cos 2 θ ) sin θ 1 + cos 2 θ
dt 1 dt sec 2 θ dθ sec 2 θ dθ
=−∫ =− ∫ =− ∫ sec 2 θ + 1 =− ∫ tan 2 θ + 2
3t + 3t + 1
2 3  1
2
1
t +  +
 2 12 Put tan θ = t . ⇒ sec 2 θ dθ = dt
1 dt 1  t 
=− log | (t + 1 / 2) + (t + 1 / 2)2 + 1 / 12 | + C
3
∴ I =− ∫ t2 + 2 = − 2
tan − 1   + C
 2
  1 1
2 
 12  +  + 1 1  tan θ 
1  1 1  x − 1 2 =− tan − 1   +C
=− log  +  + + C 2  2 
 
3   x − 1 2 12  where, cos θ = x 
 
 1 − x2   
  =−
1
tan − 1   +C  sin θ = 1 − x 2 
dx  2x 
(d) Integrals of the Form ∫ , where P and Q both
2    
 1 − x2 
P Q
∴ tan θ = 
are pure quadratic expression in x , i.e.P = ax 2 + b x
dx
and Q = cx 2 + d , i. e. ∫ ⋅ y Example 84 Evaluate
(ax 2 + b ) cx 2 + d
( x − 1) x 4 + 2x 3 − x 2 + 2x + 1
To evaluate this type of integrals of the form we put x =
1 I= ∫ x 2 ( x + 1)
dx .
t
dx ( x 2 − 1) x 4 + 2x 3 − x 2 + 2x + 1
y Example 83 Evaluate ∫ . Sol. Here, I = ∫ dx
(1 + x 2 ) 1 − x 2 x 2 ( x + 1) 2
dx  1  2 1
Sol. Let I = ∫ (1 + x 2 ) 1 − 2  x 2  x 2 + 2x − 1 + + 2 
 x   x x 
1 − x2 = ∫ x 2 ( x 2 + 2x + 1)
dx
1 1
Put x = , so that dx = − 2 dt
t t x2
− 1 / t 2 dt  1  2 1  1
∴ I =∫ =−∫
t dt 1 − 2  x + 2  + 2 x +  − 1
 x   x   x
(1 + 1 / t ) 1 − 1 / t ( t + 1) t 2 − 1

2 2 2
= dx
 1 
Again, t = u ⇒ 2t dt = du .
2
 x + + 2
 x 
1 du dx 1  1
=− ∫
2 ( u + 1) u − 1
which reduces to the form ∫P Q
Put x +
x
= t , i.e. 1 − 2  dx = dt
 x 
where both P and Q are linear so that we put u − 1 = z 2 so (t 2 − 2) + 2t − 1 t 2 + 2t − 3
that du = 2z dz = ∫ ( t + 2)
dt = ∫ ( t + 2)
dt
Chap 01 Indefinite Integral 41

t 2 + 2t − 3 t 2 dt dt
= ∫ ( t + 2) t + 2t − 3
2
dt =− ∫ 1+t 2
= ∫ 1 + t2
− ∫ 1 + t 2 dt

t ( t + 2) dt t 1
= ∫ ( t + 2) t + 2t − 3
2
dt − 3∫
(t + 2) t + 2t − 3
2
= log | t + 1 + t 2 | −
2
1 + t 2 − log | t + 1 + t 2 | + C
2
1 t
I = I 1 − 3I 2 …(i) = log | t + 1 + t 2 | − 1 + t2 + C
t dt dt 2 2
Where, I 1 = ∫ and I 2 = ∫
  1 + x 2 − 6x + 10  x 2 − 6x + 10 
t 2 + 2t − 3 (t + 2) t 2 + 2t − 3 1
= log −  +C
t dt 2   | x − 3|  | x − 3 |2 
∴ I1 = ∫ ( t + 1) 2 − 4
ax 2 + bx + c
(z − 1) dz (f) Integrals of the Form ∫ dx
Put, t + 1 = z = ∫ z −2
2 2 (dx + e ) fx 2 + gx + h
Here, we write
zdz dz ax 2 + bx + c = A1 (dx + e ) (2 fx + g ) + B 1 (dx + e ) + C 1
= ∫ z 2 − 22
− ∫ z 2 − 22
Where A1 , B 1 and C 1 are constants which can be obtained
= z 2 − 22 − log | z + z 2 − 4 | by comparing the coefficients of like terms on both the
= t 2 + 2t − 3 − log | (t + 1) + t 2 + 2t + 3 | …(ii) sides.
2x 2 + 5 x + 9
∫ ( x + 1)
dy
Also, I 2 = ∫ y Example 86 Evaluate dx .
x2 + x +1
2
1 1  1 
y ⋅2
 − 2 + 2  − 2 − 3
y y  y  Sol. Let 2x 2 + 5x + 9 = A ( x + 1) (2x + 1) + B ( x + 1) + C
1 dy 1 dy
Put t + 2 =
y
= ∫ 1 − 2y − 3y 2
=
3 ∫ 2 2
or 2x 2 + 5x + 9 = x 2 (2A ) + x (3A + B ) + ( A + B + C )
 2  1 ⇒ A = 1, B = 2, C = 6
  − y + 
 3  3 2x 2 + 5x + 9
 1 Thus, ∫ dx
y +  ( x + 1) x 2 + x + 1
=
1 −1
sin  3  = 1 sin − 1  5 + t 
  …(iii)
( x + 1) ( 2x + 1) x +1
2 2 + t 
3 
 3 
 3 = ∫ ( x + 1) x + x +1
2
dx + 2 ∫
( x + 1) x 2 + x + 1
dx

∴ I = t 2 + 2t − 3 − log (t + 1 + t 2 + 2t − 3 ) dx
+ 6∫
 t + 5 ( x + 1) x 2 + x + 1
− 3 sin − 1  
 t + 2 2x + 1 dx dx
where, t = x +
1
x
= ∫x + x +12
dx + 2∫
x + x +1 2
+ 6∫
( x + 1) x 2 + x + 1
du dx − dt
dx =∫ +2∫ +6∫
(e) Integrals of the Form∫ , u ( x + 1 / 2) + ( 3 / 4 )
2
t −t +1
2
(x − k ) r
ax 2 + bx + c
where r ≥ 2 and r ∈ I [where u = x 2 + x + 1 and = x + 1]
1
1 t
Here, we substitute, x − k =
t = 2 x 2 + x + 1 + 2 ⋅ 1 log | ( x + 1 / 2) + x 2 + x + 1 |
dt
dx −6 ∫
y Example 85 Evaluate ∫ ( x − 3) 3 x 2 − 6 x + 10
⋅ ( t − 1 / 2) 2 + 3 / 4
 1 
= 2 x 2 + x + 1 + 2 log  x +  + x 2 + x + 1 
1 1  2 
Sol. Substitute ( x − 3) = ⇒ dx = − 2 dt   1  
t t −6 logt −  + t 2 − t + 1 + C
dx   2  
We get, ∫ ( x − 3) 3 x 2 − 6x + 10 
 1 

= 2 x + x + 1 + 2 log  x +  + x + x + 1 − 6 log
2 2
 2 
− 1 / t 2 dt
= ∫ 1/t3 (1 / t + 3)2 − 6 (1 / t + 3) + 10
1− x + x2 + x + 1
 + C
 2 ( x + 1) 
42 Textbook of Integral Calculus

dx
Type III y Example 89 Evaluate ∫ 2 sin x + sec x ⋅
Integration of Type ∫ (sinm x ⋅ cos n x ) dx Sol. Let I =
dx cos x dx 1 2 cos x dx
∫ 2 sin x + sec x = ∫ sin 2x + 1 = 2 ∫ 1 + sin 2x
(i) Where m, n belongs to natural number. 1 (cos x + sin x ) + (cos x − sin x )
2 ∫ (sin 2 + cos 2 x + 2 sin x cos x )
= dx
(ii) If one of them is odd, then substitute for term of even
power. 1 cos x + sin x 1 (cos x − sin x )
2 ∫ (sin x + cos x )2
= dx + ∫ dx
(iii) If both are odd, substitute either of them. 2 (sin x + cos x )2
(iv) If both are even, use trigonometric identities only. 1 dx 1 dv
2 ∫ sin x + cos x 2 ∫ v 2
= + , where, v = sin x + cos x
m + n − 2 
(v) If m and n are rational numbers and   is a
  1 dx 1
2 2 ∫ 1 sin x + 1 cos x 2v
2 = − +C
negative integer, then substitute cot x = p or tan x = p
which so ever is found suitable. 2 2
1 dx 1
2 2 ∫
= − +C
y Example 87 Evaluate ∫ sin 3 x ⋅cos 5 x dx .  π  2 (sin x + cos x )
sin  x + 
 4
Sol. I = ∫ sin 3 x ⋅ cos 5 x dx
1  π  π 1
= log| cosec  x +  − cot  x +  | − +C
Let cos x = t ⇒ − sin x dx = dt 2 2  4  4  2 (sin x + cos x )
I = − ∫ (1 − t 2 ) ⋅ t 5dt
t8 t6
I = ∫ t 7 dt − ∫ t 5 dt = − +C
8 6 Type IV
cos 8 x cos 6 x
I = − +C
8 6 Integrals of the Form∫ x m( a + bx n )P dx
Aliter I = ∫ R 3 (1 − R 2 )2 dR,
Case I If P ∈ N . We expand using binomial and integrate.
if sin x = R, cos x dx = dR −
Case II If P ∈ I (ie, negative integer), write x = t k ,
I = ∫ R 3 dR − ∫ 2R 5 dR + ∫ R 7dR
where k is the LCM of m and n.
I =
sin 4 x 2 sin 6 x
− +
sin 8 x
+C
m +1
Case III If is an integer and P ↔ fraction, put
4 6 8 n
Remark (a + b x n ) = t k , where k is denominator of the fraction P.
This problem can also be handled by successive reduction or by
m + 1 
trigonometrical identities. Answers will be in different form but Case IV If  + P  is an integer and P ∈fraction.
identical with modified constant of integration.  n 
y Example 88 Evaluate ∫ sin − 11/ 3 x ⋅cos − 1/ 3 x dx . We put (a + b x n ) = t k x n , where k is denominator of the
11 1 fraction P.
− − −2
Sol. Here, ∫ sin − 11/ 3
x ⋅ cos − 1/ 3
x dx i.e. 3 3 = −3 y Example 90 Evaluate ∫ x 1/ 3 (2 + x 1/ 2 ) 2 dx .
2
− 1/ 3
cos x
∫ sin − 1/ 3 x ⋅ sin 4 x dx = ∫ (cot
− 1/ 3
∴ I = x ) (cosec 2 x )2 dx Sol. I = ∫ x 1/ 3 (2 + x 1/ 2 )2 dx
Since, P is natural number.
I = ∫ (cot − 1/ 3 x ) (1 + cot 2 x ) cosec 2 x dx
∴ I = ∫ x 1/ 3 ( 4 + x + 4 x 1/ 2 ) dx
= − ∫ t − 1/ 3 (1 + t 2 ) dt = − ∫ (t − 1/ 3 + t 5 / 3 ) dt
= ∫ ( 4 x 1/ 3 + x 4 / 3 + 4 x 5 / 6 ) dx
[Put cot x = t , ⇒ − cosec 2 x dx = dt ]
4 x 4 / 3 x 7 / 3 4 x 11/ 6
3 3  = + + +C
= −  t 2/3 + t 8/3 + C 4 /3 7 /3 11 / 6
2 8 
3 24 11/ 6
 3 3  = 3x 4 / 3 + x 7 / 3 + x +C
= −  (cot 2 / 3 x ) + (cot 8 / 3 x ) + C 7 11
2 8 
Chap 01 Indefinite Integral 43

y Example 91 Evaluate ∫ x − 2/ 3 (1 + x 2/ 3 ) − 1 dx . =
2  3 16 / 3 3 10 / 3 
 t − t  +C
3  16 10 
Sol. If we substitute x = t 3 (as we know P ∈ negative integer) 1 1
= (1 + x ) − (1 + x 3 )5 / 3 + C
3 8/3
∴ Let x = t k , where k is the LCM of m and n. 8 5
∴ x = t 3 ⇒ dx = 3t 2 dt
y Example 95 Evaluate ∫ x − 11 (1 + x 4 ) − 1/ 2 dx .
3t 2 dt
∫ t 2 (1 + t 2 ) dt = 3 ∫ t 2 + 1 = 3 tan
−1
or I = (t ) + C
m + 1   − 11 + 1 1
Sol. Here,  + p =  −  = −3
 n   4 2
⇒ I = 3 tan − 1 ( x 1/ 3 ) + C
If we substitute (1 + x ) = t x ,
4 2 4

∫x
− 2/ 3
y Example 92 Evaluate (1 + x 1/ 3 )1/ 2 dx . then 1 +
1
= t 2 and
−4
dx = 2t dt
x4 x5
1
Sol. If we substitute 1 + x 1/ 3 = t 2 , then dx = 2t dt dx dx
3x 2 / 3 ∴ I = ∫ x 11 (1 + x 4 ) 1/ 2 = ∫ x 11 ⋅ x 2 (1 + 1 / x 4 )1/ 2
t ⋅ 6t dt
∴ I = ∫ = 6 ∫ t 2 dt = 2t 3 + C dx 1 2t dt
1 = ∫ x 13 (1 + 1 / x 4 )1/ 2 =−
4 ∫ x8 t
or I = 2 (1 + x 1/ 3 )3 / 2 + C
1 1
=− ∫ (t 2 − 1)2 dt = − ∫ (t 4 − 2t 2 + 1) dt
∫ x (1 + x 1/ 3 4
y Example 93 Evaluate ) dx . 2 2
1 t 5 2t 3 
Sol. Here, m =
1
and n =
1 =−  − + t + C
2 3 2 5 3 
Put x = t 6 ⇒ dx = 6t 5 dt 1
Wheret = 1 +
x4
⇒ I = ∫ t 3 (1 + t 2 )4 6t 5 dt
1
⇒ I = 6 ∫t 8 (1 + 4t 2 + 6t 4 + 4t 6 + t 8 ) dt y Example 96 Evaluate ∫ 3 x + 4 x dx.
= 6 ∫ (t 8 + 4t 10 + 6t 12 + 4t 14 + t 16 ) dt 1
Sol. Let I = ∫3x +4x
dx
 t 9 4t 11 6t 13 4t 15 t 17 
=6 + + + +  +C Put x 1/12 = t , ⇒ x = t 12 and dx = 12t 11 dt
 9 11 13 15 17 
1 t8
∴ I = ∫ t 4 + t 3 ⋅ 12 t dt = 12 ∫
11
dt
 4 6 4 1 17 / 6  t +1
I = 6 x 2 / 3 + x 11/ 6 + x 13 / 6 + x 5 / 2 + x  +C
 11 13 15 17  Again put (t + 1) = y
( y − 1) 8
y Example 94 Evaluate ∫ x 5 (1 + x 3 ) 2/ 3 dx . ∴ dt = dy = 12 ∫ dy
y
2 (y 8 − 8y 7 + 28y 6 − 56y 5 + 70y 4 − 56y 3 + 28y 2 − 8y + 1)
∫x (1 + x 3 )2 / 3 dx have m = 5, n = 3 and p =
5
Sol. Here,
3 = 12 ∫ dy
y
m +1 6 [using binomial]
∴ = =2 [an integer]
n 3 = 12 ∫ (y − 8y + 28y − 56y + 70y − 56y + 28y − 8 + 1 /y )dy
7 6 5 4 3 2

So, we substitute 1 + x 3 = t 2 and 3x 2 dx = 2t dt


 y 8 8y 7 28y 6 56y 5 70y 4 
− + − +
∴ ∫ x (1 + x ) dx = ∫ x (1 + x ) x dx  
5 3 2/3 3 3 2/3 2

= 12  8 7 6 5 4  +C
1
2
= ∫ ( t 2 − 1) ( t 2 ) 2 / 3
t dt  56y 3 28y 2 
− + − 8y + log | y | 
3  3 2 
2 2
= ∫ (t 2 − 1) t 7 / 3dt = ∫ (t 13 / 3 − t 7 / 3 ) dt Where y = x 1/12 + 1
3 3
44 Textbook of Integral Calculus

Exercise for Session 6


n Evaluate the following integrals
x4 − 1
1. ∫ dx 2. ∫ 2 ( x + 2) dx
x x + x 2 + 1)1/ 2
2( 4
( x + 3x + 3) x +1

dx dx
3. ∫ 4. ∫
(x + 1)1/ 3 + ( x + 1)1/ 2 (x + a )8 / 7 ( x − b )6 / 7
sec x . dx
5. ∫
sin ( x + 2A) + sin A

6. The value of ∫ [{x }] dx ; ( where [. ] and {.} denotes greatest integer and fractional part of x) is equal to
(a) 0 (b) 1
(c) 2 (d) − 1
1 2
7. If ∫ f ( x ) cos x dx = f ( x ) + C, then f ( x ) can be
2
(a) x (b) 1
(c) cos x (d) sin x
sin x + cos x
8. The value of, ∫ dx is
9 + 16 sin 2x
1 5 + 4 (sin x − cos x ) 5 + 4 (sin x − cos x )
(a) log +C (b) log +C
40 5 − 4 (sin x − cos x ) 5 − 4 (sin x − cos x )
1 5 + 4 (sin x + cos x )
(c) log +C (d) None of these
10 5 − 4 (sin x + cos x )

cos 7x − cos 8x
9. The value of ∫ dx , is
1 + 2 cos 5x
sin 2x cos 3x
(a) + +C (b) sin x − cos x + C
2 3
sin 2x cos 3x
(c) − +C (d) None of these
2 3
cos 5x + cos 4x
10. The value of ∫ dx , is
1 − 2 cos 3x
sin 2x
(a) sin x + sin 2x + C (b) sin x − +C
2
sin 2x
(c) − sin x − +C (d) None of these
2
Session 7
Euler’s Substitution, Reduction Formula
and Integration Using Diffrentiation

Euler’s Substitution, Reduction and  5 + 2t 2 


 ⋅
− 6t
dt
Integration Using Diffrentiation Hence, I = ∫(
x dx
= ∫
 1 + t  (1 + t 2 )2
2

3
7 x − 10 − x 2 )3  3t 
Integration Using  
1 + t 2 
Euler’s Substitutions − 6 5 + 2t 2
27 ∫ t 2
= dt
Integrals of the form ∫ f ( x ), ax + bx + c dx are
2

−2  5  −2 − 5 
calculated with the aid of one of the three Euler’s = ∫
9 t
 2 + 2 dt =
 9  t + 2t  + C
substitutions
x dx −2 − 5 
(i) ax 2 + bx + c = t ± x a , if a > 0. ∴ ∫( 7 x − 10 − x ) 2 3
=
9

 t
+ 2t  + C ,

(ii) ax + bx + c = tx + c , if c > 0.
2
7 x − 10 − x 2
where, t =
(iii) ax 2 + bx + c = ( x − α ) t , if x −2
ax 2 + bx + c = a ( x − α ) ( x − β), i.e. If α is real root of dx
(ax 2 + bx + c ).
y Example 98 Evaluate ∫x+ x2 − x +1

Remark Sol. Since, here c = 1, we can apply the second Euler’s Substi-
The Euler’s substitutions often lead to rather cumbersome tution.
calculations, therefore they should be applied only when it is x 2 − x + 1 = tx − 1
difficult to find another method for calculating a given integral. 2t − 1
Therefore, (2t − 1) x = (t 2 − 1) x 2 ⇒ x =
x dx t2 − 1
y Example 97 Evaluate I = ∫ ⋅ 2 (t 2 − t + 1) dt t
∴ dx = − and x + x 2 − x + 1 =
( 7 x − 10 − x ) 2 3
( t 2 − 1) 2 t −1
Sol. In this case a < 0 and c < 0. Therefore, neither (I) nor (II) dx − 2t 2 + 2t − 2
Euler’s Substitution is applicable. But the quadratic
∴ I = ∫x + x − x +1
2
= ∫ t (t − 1) (t + 1)2 dt
7 x − 10 − x 2 has real roots α = 2, β = 5.
Using partial fractions, we have
∴ We use the substitution (III) − 2t 2 + 2t − 2 A B C D
= + + +
i.e. 7 x − 10 − x 2 = ( x − 2) (5 − x ) = ( x − 2) t t ( t − 1) ( t + 1) 2
t t − 1 ( t + 1 ) ( t + 1) 2
Where ( 5 − x ) = ( x − 2) t 2 or ( − 2t 2 + 2t − 2) = A (t − 1) (t + 1)2 + Bt (t + 1)2
or 5 + 2t 2 = x (1 + t 2 ) + C (t − 1) (t + 1) t + Dt
we get A = 2, B = − 1 / 2, C = − 3 / 2, D = − 3
5 + 2t 2
∴ x= dt 1 dt 3 dt dt
Hence, I = 2 ∫ − ∫
2 t − 1 2 ∫ ( t + 1)
1 + t2 − −3∫
t ( t + 1) 2
 5 + 2t 2  3t
( x − 2) t =  − 2 t = 1 3
= 2 loge | t | − loge | t − 1 | − loge | t + 1 | +
3
+C
1+t 2
 1 + t2 2 2 ( t + 1)
− 6t 
∴ dx = dt x 2 − x + 1 + 1
(1 + t 2 )2 where t = 
 x 
46 Textbook of Integral Calculus

where, tan x = t ⇒ sec 2 x dx = dt


Introduction of Reduction tn −1
Formulae (a recursive relation) In =
n −1
− In −2

Over Indefinite Integrals tann − 1 x


∴ In = − In −2
Reduction formulae makes it possible to reduce an integral n −1
depending on the index n > 0, called the order of the
tann − 1 x
⇒ ∫ tan x dx = − ∫ tann − 2 x dx
integral, to an integral of the same type with smaller n

index. (i.e. To reduce the integrals into similar integrals of n −1


order less than or greater than given integral).
Application of reduction formula is given with the help of Reduction Formula for ∫ cosec n x dx
some examples.
Let I n = ∫ cosec n x dx = ∫ cosec n − 2 x cosec 2 x dx
Reduction Formula for ∫ sin n x dx I II
= cosec n − 2 x ( − cot x ) − ∫ (n − 2 ) cosec n − 2 x (cosec 2 x − 1)dx
Let I n = ∫ sin x dx = ∫ sin
n n −1
x sin x dx
= − cosec n − 2 x cot x − (n − 2 ) ∫ (cosec n x − cosec n − 2 x ) dx
I II
= − cosec n − 2 x cot x − (n − 2 ) I n + (n − 2 ) I n − 2
= − sinn − 1 x cos x + ∫ (n − 1) sinn − 2 x cos 2 x dx
∴ (n − 1) I n = − cosec n − 2 x cot x + (n − 2 ) I n − 2
= − sinn − 1 x cos x + (n − 1) ∫ sinn − 2 x (1 − sin2 x ) dx
cosec n − 2 x cot x n − 2
= − sinn − 1 x cos x + (n − 1) ∫ (sinn − 2 x − sinn x ) dx or In = − + In −2
n −1 n −1
= − sinn − 1 x cos x + (n − 1) I n − 2 − (n − 1) I n cosec n − 2 x cot x n − 2
∴ ∫ cosec n xdx = −
n −1 ∫
+ cosec n −2 x dx
∴n I n = − sinn − 1 x cos x + (n − 1) I n − 2 n −1
sinn − 1 x cos x n − 1
⇒ In = −
n
+
n
In −2
Reduction Formula for ∫ sec n x dx
− sinn − 1 x cos x n − 1 Let I n = ∫ sec n x dx = ∫ sec n − 2 x sec 2 x dx
Thus, ∫ sinn x dx =
n ∫
+ sinn − 2 x dx
n I II
= sec n − 2 x tan x − ∫ (n − 2 ) sec n − 3
Reduction Formula for ∫ cos x dx n
x sec x tan x ⋅ tan x dx
= sec n − 2 x tan x − (n − 2 ) ∫ sec n − 2 x (sec 2 x − 1) dx
Let I n = ∫ cos n x dx = ∫ cos n − 1 x cos x dx
I II = sec n − 2 x tan x − (n − 2 ) I n + (n − 2 ) I n − 2
= cos n − 1 x sin x + ∫ (n − 1) cos n − 2 x sin2 x dx ⇒ (n − 1) I n = sec n − 2 x tan x + (n − 2 ) I n − 2
= cos n − 1 x sin x + (n − 1) ∫ cos n − 2 x (1 − cos 2 x ) dx sec n − 2 x tan x (n − 2 )
or In = + In −2
( n − 1) (n − 1)
= cos n − 1 x sin x + (n − 1) I n − 2 − (n − 1) I n
sec n − 2 x tan x (n − 2 )
∴ ∫ sec n x dx =
(n − 1) ∫
∴ nI n = cos n − 1 x sin x + (n − 1) I n − 2 + sec n − 2 x dx
(n − 1)
cos n − 1 x sin x n − 1
∫ cos x dx = + ∫ cos n − 2 x dx
n
or
n n Reduction Formula for ∫ cot n x dx
Reduction Formula for ∫ tan n x dx Let I n = ∫ cot n x dx = ∫ cot n − 2 x cot 2 x dx

Let I n = ∫ tann x dx = ∫ cot n − 2 x (cosec 2 x − 1) dx

⇒ I n = ∫ tann − 2 x tan2 x dx = ∫ tann − 2 x (sec 2 x − 1) dx = ∫ cot n − 2 x (cosec 2 x − 1) dx = ∫ cot n − 2 x dx

= ∫ tann − 2 x sec 2 x − I n − 2 = ∫ t n − 2 dt − I n − 2 = ∫ t n − 2 dt − I n − 2 , where t = cot x


Chap 01 Indefinite Integral 47

In = −
cot n − 1 x
− In −2 Reduction Formula for∫ cos m x sin nx dx
n −1
cot n − 1 x Let I m , n = ∫ cos m x sin nx dx
∫ cot x dx = − n − 1 − ∫ cot x dx
n −2
∴ n
I II
cos m x cos nx m
=− − ∫ cos m − 1 x sin x cos nx dx
Reduction Formula for ∫ sin m x cos n x dx m
n
cos x cos nx m
n

=− − ∫ cos m − 1 x
Let A = sinm − 1 x cos n + 1 x n n
dA {sin nx cos x − sin (n − 1) x } dx
∴ = (m − 1) sinm − 2 x cos n + 2 x − (n + 1) sinm x cos n x
dx [using sin (n − 1) x = sin nx cos x − cos nx sin x
= (m − 1) sinm − 2 x cos n x (1 − sin2 x ) ⇒ sin x cos nx = sin nx cos x − sin (n − 1) x ]
− (n + 1) sinm x cos n x cos m x cos nx m
=− − ∫ cos m x sin nx dx
= (m − 1) sinm − 2 x cos n x − (m − 1 + n + 1) n n
m
sinm x cos n x + ∫ cos m − 1 x sin (n − 1) x dx
dA n
⇒ = (m − 1) sinm − 2 x cos n x − (m + n ) sinm x cos n x cos m x cos nx m m
dx Im, n = − − I m , n + I m − 1, n − 1
n n n
Integrating with respect to x on both the sides, we get
m +n cos m x cos nx m
A = (m − 1) ∫ sinm − 2 x cos n x dx − (m + n ) ⇒ Im, n = − + I m − 1, n − 1
n n n
∫ sin
m
x cos n x dx cos m x cos nx m
or Im, n = − + I m − 1, n − 1
⇒ (m + n ) ∫ sinm x cos n x dx = (m − 1) m +n m +n
∫ sin
m −2
x cos n x dx − P
Remarks
(m − 1) Similarly, we can show
⇒ ∫ sin x cos x dx = ∫ sinm − 2 x cos n x dx
m n
(m + n ) cos m x sin nx m
1. ∫ cos m x cos nx dx = +
sinm − 1 x cos n + 1 x m+ n m+ n
− m−1
m +n ∫ cos x cos ( n − 1) x dx
m −1 n +1 m m−1
(m − 1) sin x cos x 2. ∫ sinm x sin nx dx =
n sin x cos nx

m sin x cos x cos nx
or Im, n = I m − 2, n − m2 − n2 m2 − n2
(m + n ) (m + n )
m ( m − 1) m− 2
+
m2 − n2 ∫ sin x sin nx dx
Remarks
n sinm x sin nx m sinm − 1 x cos x cos nx
Similarly, we can show 3. ∫ sinm x cos nx dx = −
sinm+ 1 x cos n + 1 x n−1 m −n2 2
m2 − n2
1. ∫ sinm x cos n x dx = sinmx cos n − 2 x dx
m+ n ∫
+ m ( m − 1) m− 2
m+ n
m+ 1 n+1
+
m2 − n2 ∫ sin x cos nx dx
sin x cos x m+ n+ 2
2. ∫ sinm x cos n x dx = +
m+1 m+1 dx
∫ sin
m+ 2
x cos n x dx
y Example 99 Evaluate I n = ∫ ( x 2 + a 2 )n ⋅
sinm + 1 x cos n + 1 x m+ n+ 2
3. ∫ sinm x cos n x dx = + dx 1
n+ 1 n+ 1 Sol. Here, I n = ∫ ( x 2 + a 2 )n = ∫ ( x 2 + a 2 )n ⋅ 1 dx
n+
∫ sin x cos
m 2
x dx
Applying Integration by parts, we get
m−1 n+1
sin x cos x m−1 1 ( 2x )
4. ∫ sinm x cos n x dx = − + = 2 ⋅x − ∫ 2 ⋅ ( − n ) ⋅ ( x ) dx
n+ 1 n+ 1
m− 2
( x + a 2 )n ( x + a 2 )n + 1
∫ sin x cos n + 2
x dx
x x2
5. ∫ sin x cos x dx =
m n sinm+ 1
x cos n −1
x
+
n−1 =
( x 2 + a 2 )n
+ 2n ∫ ( x 2 + a 2 )n + 1 dx
m+1 m+1
m+ 2 n−2 x x 2 + a2 − a2
∫ sin x cos x dx = + 2n ∫ ( x 2 + a 2 )n + 1 dx
(x + a )
2 2 n
48 Textbook of Integral Calculus

x 1 dx
∴ In = + 2n ∫ dx − 2a 2n ∫
( x 2 + a 2 )n ( x 2 + a 2 )n ( x 2 + a 2 )n +1 Integration Using
In =
(x + a )
2
x
2 n
+ 2n I n − 2n a 2 I n + 1 Differentiation
dx dx dx
∴ 2n a 2 I n + 1 =
x
+ (2n − 1) I n In ∫ , ∫ (a + b sin x ) 2 , ∫ (sin x + a sec x ) 2 ,
(x + a )
2 2 n
(a + b cos x ) 2

1 (2n − 1) 1
x a + b sin x
= ⋅ 2 + ⋅ 2 In
or In + 1
2n a ( x + a )
2 2 n
2n a ∫ (b + a sin x ) 2 dx , K we follow the following method.
sin x cos x
Remark 1. Let A = or A = according to the
Above obtained formula reduces the calculations of the integral a + b cos x a + b sin x
In + 1 to the calculations of the integral In and consequently, allows integral to evaluated is of the form
us to calculate completely an integral with natural index, as
dx dx
I1 =
dx
∫ x 2 + a2 = a tan
1 −1  x  + C
  ∫ (a + b cos x ) 2 or ∫ (a + b sin x ) 2
 a 
∴ From above formula dA 1
2. Find and express it in terms of or
Let n = 1 dx a + b cos x
dx 1 x 1 1
I2 = ∫ 2 = ⋅ + ⋅ I1 as the case may be.
( x + a2 ) 2 2a2 x 2 + a2 2a2 a + b sin x
=
1

x
+
1 1
⋅ ⋅ tan− 1 x + C
  3. Integrate both the sides of the expression obtained in
2a2 x 2 + a2 2a2 a  a
step 2 to obtain the value of the required integral.
⋅ tan− 1   + C
1 x 1 x
= ⋅ +
 a dx
x + a
∫ ( 5 + 4 cos x )2 ⋅
2 2 2 3
2a 2a y Example 101 Evaluate
Let n = 2
dx 1 x 3 sin x
I3 = ∫ ( x 2 + a2 )3 = 4 a2 ⋅ ( x 2 + a2 )2 + 4 a2 I2 Sol. Here, A =
5 + 4 cos x
, then

tan− 1   + C
1 x 3 x 3 x dA (5 + 4 cos x ) (cos x ) − sin x ( − 4 sin x )
= ⋅ + ⋅ +
4 a2 ( x 2 + a2 ) 2 8 a4 x 2 + a2 8 a5  a =
dx (5 + 4 cos x )2
…and so on.
5 25
( 4 cos x + 5) + 4 −
y Example 100 Derive reduction formula for dA 5 cos x + 4
⇒ = = 4 4
sin n x dx (5 + 4 cos x )2 (5 + 4 cos x )2
I (n, m ) = ∫ dx . dA 5 1 9 1
cos m x ⇒ = ⋅ − ⋅
dx 4 (5 + 4 cos x ) 4 (5 + 4 cos x )2
Sol. Using Integration by parts for I (n , m ), we get
Integrating both the sides w.r.t. ‘ x ’, we get
sin x
I (n , m ) = ∫ sinn − 1 x dx 5 dx 9 dx
I
cosm x
II
A=
4 ∫ 5 + 4 cos x −
4 ∫ (5 + 4 cos x )2
(cos x )− m + 1 9 dx 5 dx
= sinn − 1 x ⋅
( m − 1)
− ∫ (n − 1) sin
n−2 ⇒
4 ∫ (5 + 4 cos x )2 =
4 ∫ 5 + 4 cos x −A

(cos x )− m + 1
x ⋅ cos x ⋅ 5 dx sin x
( m − 1)
dx =
4 ∫ (1 − tan 2 x / 2)

(5 + 4 cos x )
5+ 4
1 sinn − 1 x ( n − 1) sinn − 2 x (1 + tan 2 x / 2)
= ⋅ − ⋅ ∫
m − 1 cosm − 1 x (m − 1) cosm − 2 x
dx
dx 5 1 + tan 2 x / 2 4 sin x
1 sin x ( n − 1) n −1 ⇒ ∫ (5 + 4 cos x )2 =
9 ∫ 9 + tan 2 x / 2
dx − ⋅
9 5 + 4 cos x
In , m = ⋅ − ⋅ I (n − 2 , m − 2 )
(m − 1) cosm − 1 x (m − 1) dx 5 2 dt 4 sin x
is required reduction formula.
⇒ ∫ (5 + 4 cos x )2 =
9 ∫ 9 + t 2 9 5 + 4 cos x
− ⋅

(where tan x / 2 = t )
Chap 01 Indefinite Integral 49

dx 10 1 t  4 sin x cos 2 x dx
⇒ ∫ (5 + 4 cos x )2 = . tan − 1   − ⋅
9 3  3 9 5 + 4 cos x
= ∫ 2 1
a + a sin 2x + sin 2 2x
dx 10  tan x / 2 4  sin x  4
⇒∫ = tan −1  −   +C
(5 + 4 cos x )2   9  5 + 4 cos x  4 cos 2 x dx (1 + cos 2x ) dx
27 3 = ∫ ( 4a 2 + 4a sin 2x + sin 2 2x ) = 2 ∫ (2a + sin 2x )2
dx
y Example 102 Evaluate ∫ (16 + 9 sin x )2 ⋅ =2∫
dx
(2a + sin 2x ) 2
+2∫
cos 2x dx
(2a + sin 2x )2
cos x dt
Sol. Let A=
16 + 9 sin x
...(i) ⇒ I = 2I 1 + ∫ t 2 [where (2a + sin 2x ) = t , (2 cos 2x ) dx = dt ]
dA (16 + 9 sin x ) ( − sin x ) − cos x (9 cos x ) 1
⇒ = ⇒ I = 2I 1 − +C
dx (16 + 9 sin x ) 2 (2a + sin 2x )
dA − 16 sin x − 9 dx
⇒ =
dx (16 + 9 sin x )2
where I1 = ∫ (2a + sin 2x )2 ...(i)

16 256 cos 2x
− (9 sin x + 16) + −9 Put A=
dA 9 9 2a + sin 2x
⇒ =
dx (16 + 9 sin x )2 dA (2a + sin 2x ) ( − 2 sin 2x ) − cos 2x (2 cos 2x )
⇒ =
dA 16 1 175 dx (2a + sin 2x )2
⇒ =− ⋅ + ...(ii)
dx 9 (16 + 9 sin x ) 9 (16 + 9 sin x )2 dA − 4a sin 2x − 2
⇒ =
Integrating both the sides of Eq. (ii) w.r.t. ‘ x ’, we get dx (2a + sin 2x )2
16 dx 175 dx dA − 4a (sin 2x + 2a ) − 2 + 8a 2
9 ∫ 16 + 9 sin x 9 ∫ (16 + 9 sin x )2
A=− + ⇒ =
dx (2a + sin 2x )2
175 dx 16 (1 + tan 2 x / 2) dx (8a 2 − 2)
9 ∫ (16 + 9 sin x )2 9 ∫ 16 +16 tan 2 x /2 + 18 tan x /2
⇒ = A + ⇒
dA
=−
4a
+
dx (2a + sin 2x ) (2a + sin 2x )2
175 dx 16 2 dt
9 ∫ (16 + 9 sin x )2 9 ∫ 16t 2 + 18t + 16
⇒ =A+ Integrating both the sides w.r.t. ‘x’, we get
dx
⇒ A = − 4a ∫ + (8a 2 − 2) I 1
[where tan x / 2 = t ] (2a + sin 2x )
175 dx 2 dt
⇒ ∫
9 (16 + 9 sin x ) 2
=A+ ∫
9 t2 + 9 t + 1 ⇒ (8a 2 − 2) I 1 = A + 4a
sec 2 x dx
∫ 2a + 2 tan x + 2a tan 2 x
8
2 4a dt

dt =A+
=A+ ∫ t
9  2
 175 
2 2a t + +1
2
9
t +  +   a
 16  16  dt
= A +2∫
2 16  16t + 9   1 
2
1 
× =A+tan − 1  
9  175  t +  + 1 − 2 
175  2a   4a 
dx 9 cos x
⇒ ∫ = ⋅ (2a )  (2at + 1) 
(16 + 9 sin x )2 175 (16 + 9 sin x ) = A +2 tan − 1  
4a 2 − 1  4a 2 − 1 
 
2  16 tan x / 2 + 9 
+ tan − 1   +C cos 2x 4a
(175) 3/2  175  ⇒ (8a 2 − 2) I 1 = +
2a + sin 2x 4a 2 − 1
dx
y Example 103 Evaluate ∫ (sin x + a sec x )2  (2a tan x + 1)
tan − 1   ...(ii)
 4a 2 − 1 

when | a | > 1 / 2 . From Eqs. (i) and (ii)
2 1 cos 2x 4a
dx cos x dx I = ⋅ +
Sol. Here, I = ∫ (sin x + a sec x )2 or I = ∫ (sin x cos x + a )2 ( 4a − 1) (2a + sin 2x ) ( 4a − 1)3 / 2
2 2

cos 2 x dx  2a tan x + 1 1
= ∫ a 2 + 2a sin x cos x + sin 2 x cos 2 x tan − 1  −
 4a 2 − 1  (2a + sin 2x )
+C
 
50 Textbook of Integral Calculus

JEE Type Solved Examples :


Single Option Correct Type Questions

Ex. 1 The value of ∫


dx ex e −x
l , is equal to l Ex. 3 Let I = ∫ dx , J = ∫ dx .
cos 6 x + sin 6 x e 4x + e 2 x + 1 e −4 x + e −2 x + 1
(a) tan− 1 ( 2 cot 2x ) + C (b) tan− 1 (cot 2x ) + C Then, for an arbitrary constant c, the value of J − I equals to
[IIT JEE 2008]
−1 1  −1  e 4 x − e 2 x + 1  e 2 x + e x + 1
(c) tan  cot 2x  + C (d) tan ( − 2 cot 2x ) + C 1
(a) log  4 x
1
 + C (b) log  2 x  +C
2 
2  e + e + 12x
2  e − e x + 1
dx sec 6 x dx
Sol. Let I = ∫ =∫ 1  e 2 x − e x + 1 1  e 4 x + e 2 x + 1
cos x + sin x
6 6
1 + tan 6 x (c) log  2 x  +C (d) log  4 x  +C
2  e + e x + 1 2  e − e 2 x + 1
(1 + tan 2 x ) 2 ⋅ sec 2 x
=∫ dx e 3x
1 + tan 6 x Sol. J = ∫ dx
1 + e 2x + e 4 x
(1 + t 2 ) 2
Put tan x = t ⇒ sec 2 x dx = dt = ∫ dt (e 3x − e x ) (u 2 − 1 )
1 + t6 J −I =∫ dx = ∫ du (u = e x )
(1 + t ) 2 2 1+e +e2x 4x
1 + u2 + u4
=∫ dt
(1 + t ) (1 − t 2 + t 4 )
2
 1  1
 1  1 − 2  du 1 − 2  du
 1 + 2  dt  u   u  dt  1
1 + t2 (1 + 1 / t 2 ) dt  t  =∫ =∫ =∫ t = u + 
=∫ dt = ∫
(1 / t 2 − 1 + t 2 ) ∫ (t − 1 / t ) 2 + 1
= , 1  1
2
t −1
2  u
1 − t2 + t4 1 + 2 + u2 u +  − 1
u  u
1
Put t − = z 1 t −1 1 u2 − u + 1
t = log + C = log 2 +C
 1 dz 2 t+1 2 u +u+1
∴ 1 + 2  dt = dz = ∫ 2 = tan − 1 (z ) + C
 t  z +1 1 e 2x − e x + 1
= log 2x +C
 t 2 − 1  tan 2 x − 1 2 e + ex + 1
= tan − 1   + C = tan − 1   +C
 t   tan x  Hence, (c) is the correct answer.
= tan − 1 ( − 2 cot 2x ) + C
l Ex. 4 Integral of 1 + 2 cot x (cot x + cosec x ) w.r.t. x, is
Hence, (d) is the correct answer.
x x
tan −1 x
(a) 2 ln cos+C (b) 2 ln sin +C
e  1 − x   2 2 2
l Ex. 2 ∫ ( sec
−1
1 + x 2 ) 2 + cos −1    dx, 1 x
(1 + x )  2
 1 + x 2   (c) ln cos + C (d) ln sin x − ln (cosec x − cot x ) + C
2 2
(x > 0) is equal to
−1 Sol. I = ∫ 1 + 2 cosec x cot x + 2 cot 2 x dx
(a) e tan x
⋅ tan−1 x + C
−1 = ∫ cosec 2 x + 2 cosec x cot x + cot 2 x dx
e tan x
⋅ (tan−1 x )2
(b) +C
2 = ∫ ( cosec x + cot x ) dx
tan−1 x −1 2
(c) e ⋅ ( sec ( 1 + x )) + C 2
1 + cos x  x x
=∫ dx = ∫ cot   dx = 2 log sin +C
−1 sin x  2 2
(d) e tan x
⋅ ( cosec −1 ( 1 + x 2 ))2 + C
Hence, (b) is the correct answer.
−1 −1 −1  1 − x2
Sol. Note that sec 1 + x = tan x; cos
2
  = 2 tan −1 x,
1 + x2 l Ex. 5 If I n = ∫ cotn x dx , then I 0 + I 1 + 2
For x > 0 −1 (I 2 + I 3 + ....+ I 8 ) + I 9 + I 10 equals to (where u = cot x )
e tan x
⇒ I =∫ {(tan −1 x ) 2 + 2 tan −1 x } dx, u2 u9  u2 u9
1 + x2 (a) u + + ....+ (b) − u + + ...+ 
Put tan −1 x = t 2 9  2 9

= ∫ et (t 2 + 2t ) dt = et ⋅ t 2 = e tan
−1
x
(tan −1 x ) 2 + C  u2 u9 u 2u 2 9u 9
(c) − u + + ....+  (d) + + ....+
 2! 9!  2 3 10
Hence, (c) is the correct answer.
Chap 01 Indefinite Integral 51

Sol. I n = ∫ cotn x dx = ∫ cotn − 2 x ⋅ (cosec 2x − 1 ) dx sin θ cos θ dθ


= 2 (b − a ) ∫
(b sin 2 θ − a sin 2 θ ) (b cos2 θ − a cos2 θ )
un −1 un −1
In = − − I n − 2 or I n + I n − 2 = − [put n = 2, 3, 4, ..., 10] sin θ cos θ dθ
n −1 n −1 = 2 (b − a ) ∫ = 2 ∫ 1 dθ
u (b − a ) sin θ cos θ
I2 + I0 = −
1 x −a
= 2θ + C = 2 sin − 1 +C
u2 b −a
I 3 + I1 = −
2 Hence, (a) is the correct answer.
u3
I4 + I2 = − ( x − 1)
3 l Ex. 8 The value of ∫ dx , is
………………… ( x + 1) x 3 + x 2 + x
…………………
u9 x +1 x2 + x +1
I 10 + I 9 = − (a) 2 tan−1 +C (b) tan−1 +C
9 x x
Adding, I 0 + I 1 + 2 ( I 2 + I 3 + ....+ I 8 ) + I 9 + I 10 x2 + x +1
 (c) 2 tan−1 +C (d) None of these
u2 u9  x
= − u + + .....+ 
 2 9 (x − 1)
Sol. Let I =∫ dx
Hence, (b) is the correct answer. (x + 1) x 3 + x 2 + x
(x 2 − 1)
l Ex. 6 Let f ( x ) = x + sin x . Suppose g denotes the inverse =∫ dx
(x + 1)2 x3 + x2 + x
π 1 
function of f . The value of g ′  +  has the value equal x 2 (1 − 1 / x 2 )
4 2 =∫ dx
to (x + 2x + 1) x 3 + x 2 + x
2
2 +1
(a) 2 − 1 (b) x 2 (1 − 1 / x 2 )
2 =∫ dx
x (x + 2 + 1 / x ) ⋅ x x + 1 + 1 / x
(c) 2 − 2 (d) 2 + 1
1
Sol. f ( x ) = y = x + sin x Put x+ = t,
x

dy
= 1 + cos x ⇒(1 − 1 / x ) dx = dt
2

dx dt dx
=∫ , which reduces to ∫ ⋅
g′ (y ) =
dx
=
1 (t + 2 ) t + 1 P Q
dy 1 + cos x Let t + 1 = z 2
π 1 π 2z dz
where y = + = x + sin x ⇒ x = ∴ dt = 2zdz = ∫
4 2 4 (z + 1 ) z 2
2

 π 1  1
∴ g′  +  = =2 ∫
dz
= 2 tan − 1 (z ) + C
4 2  1 + (1 / 2 )
z +1
2

2 x2 + x + 1
= = 2 ( 2 − 1) = 2 − 2 = 2 tan − 1 ( t + 1 ) + C = 2 tan − 1 +C
2+1 x
Hence, (c) is the correct answer.
Hence, (c) is the correct answer.

dx (1 + x 2 ) dx
l Ex. 7 The value of ∫
( x − a ) (b − x )
, is l Ex. 9 The value of ∫ (1 − x 2 ) 1+ x2 + x4
, is

x −a x −b x 4 + x 2 + 1 − 3x
(a) 2 sin−1 +C (b) 2 sin−1 +C (a) −
1
log +C
b −a b −a 2 3 x 4 + x 2 + 1 + 3x
x −a
(c) sin−1 +C (d) None of these
b −a 1 x 4 + x 2 + 1 + 2x
(b) log +C
Sol. Let x = a cos2 θ + b sin 2 θ in the given integral. 2 3 x 4 + x 2 + 1 − 2x
So that, dx = a (2 cos θ ) ( − sin θ ) + b (2 sin θ ) (cos θ ) dθ 1 x 4 − x 2 + 1 − 3x
dx = 2 (b − a ) sin θ cos θ dθ (c) log +C
2 (b − a ) sin θ cos θ dθ
2 3 x 4 + x 2 + 1 + 3x
∴ I =∫
(a cos2 θ + b sin 2 θ − a) (b − a cos2 θ − b sin 2 θ) (d) None of the above
52 Textbook of Integral Calculus

(1 + x 2 ) dx cos θ dθ
Sol. Let I =∫ = a ∫ (a 2 + b 2 sin2 θ ) ⋅ cos θ
(1 − x ) 1 + x + x
2 2 4


 1
x 2 1 + 2  dx
= a ∫ a 2 + b 2 sin2 θ ,
 x 
=∫
1  1 dividing numerator and denominator by cos2 θ, we get
x  − x x + 1 + x2
x  x2 sec 2 θ dθ
(1 + 1 / x ) dx2
= a ∫ a 2 sec 2 θ + b 2 tan2 θ , put tan θ = t
=−∫
(1 − 1 / x ) ( x − 1 / x ) 2 + 3 dt dt
= a ∫ a 2 (1 + t 2 ) + b 2 t 2 = a ∫ (a 2 + b 2 ) t 2 + a 2
1  1 dt
Put x − = t = 1 + 2  dx = dt ⇒ − ∫
x  x  t t2 + 3 a dt
=
a2 + b2 ∫ a2
Again, put t + 3 = s
2 2
t +
2
a + b2
2
s ds ds
⇒ 2t dt = 2s ds = − ∫ =−∫ 2  a2 + b2  t a2 + b2 
s (s 2 − 3 ) s − ( 3 )2 a
= ⋅  tan − 1   +C
a2 + b2    
1 s − 3   a   a 
=− log  + C
2 3 s + 3  1  x a2 + b2   x 
= ⋅ tan − 1   +C
 Q t = tanθ = 
1  t +3− 3 2
a(a 2 + b 2 )  a b − ax 2   b −ax 2 
=− log  + C
 t +3 + 3
2 3 2
Hence (a) is the correct answer.
1  (x − 1 / x )2 + 3 − 3 
=− log  + C dx
l Ex. 11 The value of I = ∫
 (x − 1 / x ) + 3 + 3 
2 3 2
2x 1 − x (2 − x ) + 1 − x
 1 
x2 + 2 + 1 − 3 1  3  1 1
  =− log  z + + z + 3 z +3   + log s − + s − s +1 + C
1 x 2 2
=− log  + C 2  2  2 2
2 3  x2 + 1 + 1 + 3 
 x2  k
and s − z = , then value of k, is
1 x 4 + x 2 + 1 − 3x x
=− log +C
2 3 x 4 + x 2 + 1 + 3x (a) 1 (b) 2 (c) 3 (d) 4
dx
Hence (a) is the correct answer. Sol. Here, I = ∫ ,
2x 1 − x (2 − x ) + 1 − x
dx put (1 − x ) = t 2 − dx = 2t dt
l Ex. 10 The value of I = ∫ , is
(a + bx 2 ) b − ax 2 =−∫
2t dt
2 (1 − t ) ⋅ t 1 + t 2 + t
2

1  x a2 + b 2 
(a) tan−1   +C dt
 =−∫
a(a 2 + b 2 )  a b − ax 2  (1 − t ) t 2 + t + 1
2

1  x a2 + b 2  dt 1  1 1  dt
(b) tan−1   +C
 =∫ = ∫  t − 1 − t + 1 ⋅
(a 2 + b 2 )  a b − ax 2  (t − 1 ) (t + 1 ) t + t + 1 2 2 t +t+1
2

 x a2 + b 2  1 1 1 1 
1 Q =  − 
(c) tan−1   +C
 (t − 1 ) (t + 1 ) 2  t − 1 t + 1
a(a 2 + b 2 )  a  1 1 1 1
(d) None of the above
=
2 ∫ (t − 1) t2 + t + 1
dt −
2 ∫ (t + 1) t2 + t + 1
dt

Sol. Substituting ax 2 = b sin 2 θ 1 1


b Let I = I1 − I 2 …(i)
⇒ dx = cos θ dθ 2 2
a dt
where, I1 = ∫
b
cos θ dθ (t − 1 ) t 2 + t + 1
∴ I =∫ a dt
 b 2  and I2 = ∫
a+ sin 2 θ  b − b sin 2 θ (t + 1 ) t 2 + t + 1
 a 
1 1
For I 1, put (t − 1 ) = ⇒ dt = − 2 dz
z z
Chap 01 Indefinite Integral 53

− 1 / z 2 dz dz dx x m
I1 = ∫ = −∫ l Ex. 13 If ∫ = +
1  1 
2
1  3  3
2 2
(x + a )
2
4a ( x + a ) na 2
2 3 2 2 2
1 +  + 1 +  + 1 z +  +  
z  z  z    2   
−1  x 
2 x 1
 2 2 2 + 3 tan    + C . Then m − n is equal to
 3 
= − log   z +  + z 2 + 3z + 3  …(ii)  2a ( x + a ) 2a  a 
 2 
(a) 4 (b) 3 (c) 2 (d) 1
1
For I 2, put (t + 1 ) = dx
S Sol. Let I = ∫ 2 ...(i)
1 (x + a 2 )3
⇒ dt = − 2 ds 1
s and I 1 = ∫ 2 dx ...(ii)
ds (x + a 2 )2
I2 = − ∫
2 1 1 − 2 (2 x )
 1 3 =∫ 2 ⋅ 1 dx = 2 ⋅x −∫ 2 x dx
s −  + (x + a 2 )2 (x + a 2 )2 (x + a 2 )3
 2 4
I II
 1 
= − log  s −  + s 2 − s + 1  …(iii) = 2
x
+4∫
x2 + a2 − a2
 2  (x + a 2 )2 (x 2 + a 2 )3
dx
1  3  x 1 dx
∴ I =−
log z + + z + 3z + 3 
2
= 2 +4∫ 2 dx − 4a 2 ∫ 2
2 2  (x + a ) 2 2
(x + a ) 2 2
(x + a 2 )3
1  1 

+ log s −  + s 2 − s + 1  + C x
2  2  ⇒ I1 = 2 + 4 I 1 − 4a 2 ⋅ I [using Eqs. (i) and (ii)]
(x + a 2 )2
1 1
where, z = and s = x
1 − x −1 1−x +1 ⇒ 4a 2I = 2 + 3I 1
(x + a 2 )2
1 1 2
∴ s −z = − = ⇒k = 2 x 3
1−x +1 1 − x −1 x ⇒ I = 2 2 + I1 ...(iii)
4a ( x + a 2 ) 2 4a 2
Hence (b) is the correct answer.
[using previous example,
dx 
l Ex. 12 If ∫ I1 = ∫ 2
dx
=
x
+
1  x
tan −1   + C 
(x 2 + a 2 ) 2 ( x + a 2 ) 2 2a 2 ( x 2 + a 2 ) 2a 3 a 
1  x 1 x 3 
= 2  2 + tan −1  + C . Then the value of k, is x x 1  x 
⇒ I = + 2  2 2 2 + 3 tan −1    + C
ka  x + a 2 a a 4a ( x + a )
2 2 2 2
4a 2a ( x + a ) 2a a 
(a) 1 (b) 2 (c) 3 (d) 4
…(iv)
Sol. Here, we know m = 3 and n = 4
dx 1 −1 x
∫ x 2 + a 2 = a tan a + C ...(i) |m −n| =|3 − 4| =| −1| =1
Hence, (d) is the correct answer.
1 1 − 2x
Also, ∫ 2 ⋅ 1 dx = 2 x−∫ 2 x dx
x +a 2
x +a 2
(x + a 2 )2 l Ex. 14 If y ( x − y ) 2 = x , then
x x + a −a 2 2 2
dx m
= +2∫ dx
∫ ( x − 3y ) = n ln [( x − y ) − 1].Then (m + 2n ) is equal to
2
x2 + a2 (x 2 + a 2 )2
I II
dx x dx dx (a) 1 (b) 3 (c) 5 (d) 7
∫ x +a
2 2
= 2
x +a 2
+2∫ 2
x +a 2
− 2a 2 ∫ 2
(x + a 2 )2
...(ii)
dx 1
Sol. Let P =∫ = ln [( x − y ) 2 − 1 ]
( x − 3y ) 2
From Eqs. (i) and (ii), we get
 dy 
1 x x 1 x dx ( x − y ) 1 − 
tan − 1 = 2 + 2 tan − 1 − 2a 2 ∫ 2 dP 1  dx 
a a x + a2 a a (x + a 2 )2 ∴ = = …(i)
dx x − 3y {( x − y ) − 1 }
2
dx x 1 x
⇒ 2a 2 ∫ 2 = + tan − 1
(x + a 2 )2 x 2 + a 2 a a Given, y ( x − y ) 2 = x, on differentiating both the sides, we get
dx 1  x 1 −1 x  dy 1 − 2y ( x − y )
or ∫ (x 2 + a 2 )2 = 2a 2  x 2 + a 2 + a tan a  + C =
dx ( x − y ) ( x − 3y )
…(ii)

1  x 1 x
= 2 2 + tan −1  + C  1 − 2y ( x − y ) 
ka  x + a 2 a a (x − y )  1 − 
dP  ( x − y ) ( x − 3y ) 
∴ k =2 ∴ =
dx {( x − y ) 2 − 1 }
Hence, (b) is the correct answer.
54 Textbook of Integral Calculus

( x − y ) ( x − 3y ) − 1 + 2y ( x − y ) ( x − y ) 2− 1 f (x )
= = l Ex. 16 If ∫ dx , where f ( x ) is a polynomial of
( x − 3y ) {( x − y ) 2 − 1 } ( x − 3y ) {( x − y ) 2− 1 } x3 −1
dP 1 degree 2 in x such that f (0 ) = f (1) = 3 f ( 2 ) = − 3 and
∴ = …(iii)
dx ( x − 3y ) f (x )
∫ x 3 − 1 dx = − log x − 1 + log x + x + 1
2
which is true as given
dx 1 m  2 x + 1
∴ ∫ (x − 3y ) = 2 log {(x − y ) − 1}, tan −1 
2
+  + C . Then ( 2m + n ) is
n  3 
∴ m = 1, n = 2 (a) 3 (b) 5 (c) 7 (d) 9
⇒ m + 2n = 5
Sol. Let f ( x ) = ax 2 + bx + c
Hence, (c) is the correct answer.
Given, f ( 0 ) = f (1 ) = 3 f (2 ) = − 3
∴ f (0) = c = − 3
∫ (x + 1 + x ) dx .
2 n
l Ex. 15 If
f (1 ) = a + b + c = − 3
1 1 3 f (2 ) = 3 ( 4a + 2b + c ) = − 3
= { x + 1 + x 2 } n +1 + { x + 1 + x 2 } n −1 + C
a (n + 1) −b(n − 1) On solving, we get a = 1, b = − 1, c = − 3
Then (a + b ) is equal to ∴ f (x ) = x 2 − x − 3
(a) 2 (b) 3 (c) 4 (d) 5 f (x ) x2 − x − 3
⇒ I =∫ dx = ∫ dx
Sol. Let I = ∫ ( x + 1 + x ) dx 2 n x −1
3
(x − 1) (x 2 + x + 1)
Using partial fractions, we get
Put x + 1 + x2 = t …(i)
(x 2 − x − 3) A Bx + C
  = +
⇒ 1+ 1
⋅ 2x  dx = dt (x − 1) (x 2 + x + 1) (x − 1) (x 2 + x + 1)
 + x2 
 2 1  we get, A = − 1, B = 2, C = 2
 1 (2 x + 2 )
1+x +x 2
∴I =∫ − dx + ∫ 2 dx
⇒   dx = dt …(ii) x −1 (x + x + 1)
 1 + x 2 

(2 x + 1 ) dx 1 dx
x − 1 + x2
= − log | x − 1 | + ∫ x2 + x + 1
+ ∫
x +x+1 2
We know, t = x + 1 + x 2 = x + 1 + x 2 ×
x − 1 + x2 dx
= − log| x − 1 | + log | x + x + 1 | + ∫
2

−1 (x + 1 / 2)2 + ( 3 / 2)2
t= ⇒ t = x + 1 + x2
x − 1 + x2 2  2x + 1
= − log | x − 1 | + log | x 2 + x + 1 | + tan −1   +C
3  3 
1 ∴ On comparing m = 2 ,n = 3 ⇒ 2m + n = 7.

= x − 1 + x2
t Hence, (c) is the correct answer.
Subtracting, we get
(1 + x )
Ex. 17 The value of ∫
1 1 2t
2 1 + x2 = t + or = …(iii) l dx , is equal to
t 1+x 2 t +12
x (1 + xe x ) 2
 x  1
From Eqs. (i), (ii) and (iii), we get (a) log + +C
 1 + xe  (1 + xe )
x x
t2 + 1
dx = dt
2t 2  xe x  1
(b) log + +C
t2 + 1 1 + x
+ xe x
∴ I = ∫ tn ⋅ dt = ∫ (t
n
+ tn − 2 ) dt  1 xe  1
(2t 2 ) 2
 xe x  1
1  tn + 1 tn − 1  (c) log + +C
 1 + e  1 + xe
x x
= + +C
2  n + 1 n − 1 

(d) None of the above
1
⇒ I = [ x + (1 + x 2 ) ]n + 1 (1 + x ) (1 + x ) e x
2 (n + 1 ) Sol. Let I = ∫ dx = ∫ dx,
x (1 + xe )x 2
( xe x ) (1 + xe x ) 2
1
+ ( x + (1 + x 2 ) )n − 1 + C …(iv) put 1 + xe x = t
2 (n − 1 )
dt
Then comparing the values of a and b by Eq. (iv) a = 2, b = 2 ∴ (1 + x ) e x dx = dt = ∫ , applying partial fraction,
(t − 1 ) ⋅ t 2
∴ (a + b ) = (2 + 2 ) = 4
1 A B C
Hence, (c) is the correct answer. we get = + + 2
(t − 1 ) t 2 t − 1 t t
Chap 01 Indefinite Integral 55

⇒ 1 = A (t 2 ) + Bt (t − 1 ) + C (t − 1 ) 1  x 1 x x2 
= sin − 1   + log  + 1 − 2 + C
For t =1 ⇒ A =1 2 a 2
a a 
For t = 0 ⇒ C = − 1 and B = − 1
1  x 1 1
 1 1 1 1 = sin − 1   + {log | x + a 2 − x 2 |} − log a + C
∴ I =∫ − − 2  dt = log | t − 1 | − log | t | + + C 2 a 2 2
t − 1 t t  t
1  x 1
= log | xe | − log | 1 + xe | +
x x 1
+C = sin − 1   + log | x + a 2 − x 2 | + C1
2 a 2
1 + xe x  1 
 xe x  1 where C1 = C − log a
= log  + +C  2 
 1 + xe  1 + xe
x x Hence, (a) is the correct answer.
Hence, (b) is the correct answer.
x 2 −1
dx
l Ex. 19 The value of ∫ dx , is equal to
l Ex. 18 The value of ∫ , is equal to ( x 2 + 1) x 4 + 1
x + a2 − x2
1  x 2 + 1  x 2 + 1
1 x 1 (a) sec −1   +C (b) 2 sec −1   +C
(a) sin− 1   + log | x + a − x | + C1
2 2
2  2x   2x 
2 a 2
1 x 1 1  x 2 + 1  x 2 + 1
(b) sin− 1   − log | x + a − x | + C1
2 2
(c) cosec −1   +C (d) 2 cosec −1   +C
2 a 2 2  2x   2x 
1 x (x 2 − 1) x 2 (1 − 1 / x 2 ) dx
(c) sin− 1   − log | x + a 2 − x 2 | + C1 Sol. Let I = ∫ dx = ∫
2 a
(x + 1) x + 1
2 4  1 1
x2 x +  x2 + 2
1 x 1  x
(d) cos − 1   + log | x + a 2 − x 2 | + C1 x
2 a 2 (1 − 1 / x 2 ) dx
=∫
dx 2
Sol. Let I = ∫ , Put x = a sin θ  1  1
x+ a −x 2 2 x +  x +  − 2
 x  x
a cos θ dθ  1
∴ dx = a cos θ dθ = ∫ Put x +
1
=t ⇒ 1 − 2  dx = dt
a sin θ + a 2 − a 2 sin 2 θ x  x 
cos θ dθ 1 cos θ + sin θ + cos θ − sin θ dt 1  t 
=∫
sin θ + cos θ 2 ∫
= dθ =∫ = sec −1   + C
sin θ + cos θ t t2 − 2 2  2
1 1 cos θ − sin θ
= ∫ 1 dθ + ∫ dθ 1  x 2 + 1
2 2 sin θ + cos θ = sec −1   +C
2  2x 
1 1
= ⋅ θ + log (sin θ + cos θ ) + C Hence, (a) is the correct answer.
2 2

JEE Type Solved Examples :


More than One Correct Option Type Questions

4 +x2 A ( 4 + x 2 ) 3 / 2 (Bx 2 − 6 ) t= 1+
4 4
⇒ t2 = 1 + 2

Put
l Ex. 20 dx = + C, x2 x
x6 x5 8
then ∴ 2t dt = − dx
1 x3
(a) A = (b) B = 1 1 1 t 3 t 5 
120 ⇒ I =
16 ∫ (t 2 − t 4 ) dx =  − +C
16  3 5
1
(c) A = − (d) B = − 1 1 (4 + x ) 2 3/ 2
120 = ⋅ (x 2 − 6) + C
4 4 120 x5
1+ 1+ 2 1
4 − x2 x 2 dx = x dx A= , B =1
Sol. Here, I = ∫ dx = ∫ ∫ x2 ⋅ x3 120
x6 x5
Hence, (a) and (b) are the correct answers.
56 Textbook of Integral Calculus

l Ex. 21 The value of the integral l Ex. 22 If I = ∫ ( tan x + cot x ) dx = f ( x ) + c , then



sin 2 x
e (cos x + cos 3 x ) sin x dx is f ( x ) is equal to
1 2
(a) e sin x (3 − sin2 x ) + C π
(a) 2 sin−1(sin x − cos x ) (b) − 2 cos −1(sin x − cos x )
2 2
2  1   tan x − 1 
(b) e sin x 1 + cos 2 x  + C (c) 2 tan−1   (d) None of these
 2   2 tan x 
2
(c) e sin x
(3 cos 2 x + 2 sin2 x ) + C sin x + cos x
Sol. I = ∫ ( tan x + cot x ) dx = ∫ 2 ⋅ dx
2 2 sin x cos x
(d) e sin x
( 2 cos 2 x + 3 sin2 x ) + C
If sin x − cos x = p, then (cos x + sin x ) dx = dp
Sol. Put t = sin 2 x dp
I = 2∫ = 2 sin −1 p + c = 2 sin −1 (sin x − cos x ) + c
1 t 3 tet 1 − p2
The integral reduces to I =
2 ∫ e (2 − t ) dt = et −
2 2
+C
π sin x − cos x
1 sin 2 x = − 2 cos−1 (sin x − cos x ) = 2 tan −1
= e (3 − sin 2 x ) + C [option (a)] 2 1 − (sin x − cos x ) 2
2
 1  sin x − cos x  tan x − 1
= 2 tan −1 = 2 tan −1 
2
= e sin x 1 + cos2 x + C [option (b)] 
 2  2 sin x cos x  2 tan x 
Hence, (a) and (b) are the correct answers. Hence, (a), (b) and (c) are the correct answers.

JEE Type Solved Examples :


Passage Based Questions
Passage 2  2
Put x 2 + + 1 = t ⇒ 2  x − 3  dx = dt
(Q. Nos. 23 to 25) x 2  x 
1 dt 1 t1/2
 a  a  a = ∫ = ⋅ +C = t +C
For integral ∫ f  x − x  ⋅ 1 + x 2  dx, put x − x = t 2 t 2 1 /2
2
 a  a  a = x2 + +1+C
For integral ∫ f  x +  ⋅ 1 − 2  dx , put x + = t x2
 x  x  x Hence, (b) is the correct answer.
 a   a  a
For integral ∫ f  x 2 − 2  ⋅  x + 3  dx , put x 2 − 2 = t ( x − 1)
 x   x  x l Ex. 24 ∫ ( x + 1) x3 + x 2 + x
dx
 2 a   a  a
For integral ∫ f  x + 2  ⋅  x − 3  dx , put x 2 + 2 = t
 x   x  x  1  1
(a) tan−1  x + + 1 + C (b) tan−1 x + +1+C
many integrands can be brought into above forms by  x  x
suitable reductions or transformations. −1 1
(c) 2 tan x + +1+C (d) None of these
x
 1
x4 −2 1 − 2 
 x 
l Ex. 23 ∫ x2 x4 +x2 +2
dx Sol. ∫ (x + 1)2
x2 − 1
dx = ∫
 
dx
x3 + x2 + x +
1
+ +
1
+
 x 2 x 1
 x  x
1 2
(a) x 2 + 1 + +C (b) x 2 + 1 + +C 1  1
x2 x2 Put x + + 1 = t2 ⇒ 1 − 2  dx = 2t dt
x  x 
1 2
(c) x 2 + +C (d) x 2 + +C =∫ 2
2t dt
=2 ∫ 2
1
dt
x2 x2 (t + 1 ) t (t + 1 )
2
x−  1 
Sol. Here, I = ∫ x3 dx = 2 ⋅ tan −1 (t ) + C = 2 tan −1  x + + 1 + C
2  x 
x2 + 1 +
x2 Hence, (c) is the correct answer.
Chap 01 Indefinite Integral 57

5x 4 + 4x 5 Divide numerator and denominator by x10, we get


l Ex. 25 ∫ ( x 5 + x + 1) 2 dx
I =∫
5 x −6 + 4 x −5
dx
(1 + x −4 + x −5 ) 2
x5
(a) x 5 + x + 1 + C (b) +C Put 1 + x −4 + x −5 = t ⇒( −4 x −5 − 5 x −6 ) dx = dt
x5 + x + 1
dt 1 1
x5 ∴ I =−∫ = +C= +C
(c) x −4 + x − 5 + C (d) +C t2 t 1 + x −4 + x −5
x5 + x + 1
x5
= +C
5x + 4x
4 5
x +x+1
5
Sol. Here, I =
(x 5 + x + 1)2
Hence, (d) is the correct answer.

JEE Type Solved Examples :


Matching Type Questions
l Ex. 26 If x ∈(0, 1) then match the entries of Column I 4
=
4
tan 5 θ + C = ( x 5/4 ) + C
with Column II considering ‘c’ as an arbitrary constant of 5 5
 1 + x − 4 x 
integration. (B) I = ∫ cot 2 tan −1 dx
 1 + x + 4 x 
Column I Column II 
 1+ x − 4 x   sec θ − tan θ 
 x − 1 
(A) ∫ tan  2 tan −1
1+ 4 3/ 4
x +C ∴ cot 2 tan −1 = cot 2 tan −1 
dx (p)  1+ x + x
4   sec θ + tan θ 
 1+ x + 1 3 

= cot (2 tan −1 (sec θ − tan θ ) 2 )
 1+ x − x 4
(B) ∫ cot  2 tan −1 dx (q)
4 5/ 4
x +C  1 − sin θ 
 1+ x + 4 x  5 = cot 2 tan −1 
  cos θ 
 1  1 − x   If θ ∈( 0, π / 4 ), then sec θ − tan θ > 0
 1 − tan  sin −1   
 2 1 + x    π θ  π 
(C) ∫  2 3/ 4
x +C = cot 2 tan −1 tan  −   = cot  − θ = tan θ
 
dx (r)   4 2  2 
 1 + tan  1 sin −1  1 − x   
3
 2  1 + x    ∴ I = ∫ tan θ ⋅ 4 tan 3 θ sec 2 θ dθ

 4 4
 1+ x + 1 − 1+ x − 1   =tan 5 θ + C = ( x 5/4 ) + C
x tan  2 tan −1
2 5/ 4
(D) ∫   
dx (s) x +C 5 5
1+ x +1+ 1 + x − 1   5
−1  1 − tan θ 
  1 1 − x  1 2
(C) sin −1   = sin  
2 1 + x  2  1 + tan 2 θ 
Sol. Let x = tan 2 θ
1 1 π  π
x = tan 4 θ ⇒ tan θ = x1/4 = sin −1 (cos 2θ ) = sin −1 sin  − 2θ = − θ
2 2  2  4
Q x ∈ ( 0, 1 ) 
∴ dx = 4 tan 3 θ sec 2 θ dθ ∴ θ ∈ ( 0, π / 4 )   1 1 − x  
   1 − tan  sin −1   
 2 1 + x  
∴ 1+ x = sec θ ∴ ∫  1  dx
x  
  1 + tan  sin −1  1 −  
1+ x − 1  
 2 1 + x  
(A) I = ∫ tan 2 tan −1 dx
 1+ x + 1  π 
 1 − tan  − θ
4 
 x − 1   =∫ 4 tan 3 θ sec 2 θ dθ
1+ sec θ − 1  π 
⇒ tan 2 tan −1 = tan 2 tan −1  1 + tan  − θ
 1+ x + 1   sec θ + 1  4 

  θ  π π 
= tan 2 tan −1  tan   = tan θ = ∫ tan  −  − θ  4 tan 3 θ sec 2 θ dθ
  2  4 4 

∴ I = ∫ tan θ ⋅ 4 tan 3 θ sec 2 θ dθ = ∫ 4 tan 4 θ sec 2 θ dθ


58 Textbook of Integral Calculus

=
4 4
tan 5 θ + C = ( x 5/4 ) + C   sec θ + 1 − sec θ − 1  
= tan 2 θ ⋅ tan 2 tan −1  
5 5
  sec θ + 1 + sec θ − 1  
(D) Let x = tan 2 θ ⇒ x = tan 4 θ
  θ θ 
Q x ∈ ( 0, 1 )    cos − sin  
∴ dx = 4 tan 3 θ sec 2 θ dθ ∴ θ ∈ ( 0, π / 4 )  = tan θ ⋅ tan 2 tan −1
2
 2 2 
    cos θ + sin θ  
  2 2 
∴ 1+ x = sec θ
  π θ 
  1+ x +1− 1+ x − 1   = tan 2 θ ⋅ tan 2 tan −1 tan  −  
  4 2 
I =∫ x tan 2 tan −1  dx
  
  1+ x +1+ 1+ x − 1   π 
= tan θ ⋅ tan  − θ = tan 2 θ ⋅ cot θ = tan θ
2
2 
  1+ x +1− 1+ x − 1  
∴ x tan 2 tan −1 

4 tan 5 θ
I = ∫ tan θ ⋅ 4 tan 3 θ sec 2 θ dθ =
4
+ C = ( x 5/ 4 ) + C
  
  1+ x +1+ 1+ x − 1   5 5
(A) → (q); (B) → (q); (C) → (q); (D) → (q)

JEE Type Solved Examples :


Single Integer Answer Type Questions
l Ex. 27 If the primitive of the function f (x )
m Integrating, +C …(i)
x 2009 1  x2  f ′ (x )
f (x ) = w.r.t. x is equal to   + C, f (0) 1
(1 + x 2 ) 1006 n 1 + x 2  Put x = 0, =C ⇒ C = .
f ′ (0) 2
n
then is equal to …… f (x ) 1
m Hence, = …(ii)
2009 f ′ (x ) 2
x
Sol. f (x ) = ∫ dx f ′ (x )
(1 + x 2 )1006 From Eq. (i), 2 f ( x ) = f ′ ( x ) ∴ =2
f (x )
Put 1 + x2 = t ⇒ 2x dx = dt
1004
Again, integrating, ln [ f ( x )] = 2 x + k
1 (t − 1 )1004 dt 1  1 1 Put x = 0 to get, k = 0
∴ I = ∫ = ∫ 1 −  ⋅ 2 dt
2 t 1006
2  t t f ( x ) = e 2x ⇒ λ + k = 2 + 0 = 2
1 1
Put 1 − = y ⇒ 2 dt = dy sin (100 x ) (sin x ) λ
t t l Ex. 29 ∫ {sin(101x ) ⋅ sin 99 x } dx = ,
1 1 y 1005 µ
∴ I = ∫ y 1004dy = ⋅ +C λ
2 2 1005 then is equal to ……
1005 µ
1  x2 
1005
1  t − 1
= ⋅  +C = ⋅  +C Sol. (1) I = ∫ {sin (100 x + x ) ⋅ (sin x ) 99 } dx
2010  t  2010  1 + x 2 

⇒ m = 1005, n = 2010 ⇒
n 2010
= =2 = ∫ {sin (100 x ) cos x + cos 100 x sin x } (sin x ) 99 dx
m 1005
= ∫ sin (100 x ) 1
cos x ⋅ (sin x 3
) 99 dx + ∫ cos(100x ) ⋅ (sin x )
100
dx
f ′ (x ) f (x ) 1424 3 44244
l Ex. 28 Suppose = 0 where f ( x ) is I II
f ′ ′ (x ) f ′ (x )
sin (100 x ) (sin x )100
continuous differentiable function with f ′ ( x ) ≠ 0 and satis- =
100
fies f (0 ) = 1 and f ′ (0 ) = 2, then f ( x ) = e λx + k , then λ + k is
100
− ∫ cos (100x ) (sin x ) dx + ∫ cos (100x ) (sin x )
100 100
equal to …………… dx
100
[ f ′ ( x )] − f ( x ) f ′′ ( x )
2
Sol. f ′ ( x ) ⋅ f ′ ( x ) − f ( x ) ⋅ f ′′ ( x ) = 0 or =0 sin (100 x ) (sin x )100
[ f ′ ( x )]2 = +C
100
d  f (x ) 
=0 λ 100
dx  f ′ ( x )  ⇒ λ = 100, µ = 100 ⇒ = =1
µ 100
Chap 01 Indefinite Integral 59

Subjective Type Questions


l Ex. 30 If I n denotes ∫ z n e 1/ z dz , then show that  n+2 xn − 1 (a 2 − x 2 ) 3/2 (n − 1 ) a 2
  In = − + In − 2
 3  3 3
(n + 1) ! I n = I 0 + e 1/ z (1 ! z 2 + 2 ! z 3 +… + n ! z n + 1 ).
xn − 1 (a 2 − x 2 ) 3/2 (n − 1 ) a 2
Sol. I n = ∫ z n e1/z dz , applying integration by parts taking e1/z as In = − + In − 2
(n + 2 ) (n + 2 )
first function and z n as second function. We get, Hence Proved.
e 1/ z ⋅ z n + 1  1  zn + 1
In = − ∫ e 1/ z  − 2  ⋅ dz l Ex. 32 If I m = ∫ (sin x + cos x ) dx , then show that
m
(n + 1 )  z  n+1
e 1/ z ⋅ z n + 1 1 n −1
m I m = (sin x + cos x ) m − 1 ⋅ (sin x − cos x ) + 2 (m − 1) I m − 2
= + ∫e ⋅z
1/ z
dz
(n + 1 ) (n + 1 ) Sol. Q I m = ∫ (sin x + cos x )m dx
n+1
e 1/ z
⋅z In − 1
In = + = ∫ (sin x + cos x )m − 1 ⋅ (sin x + cos x ) dx,
(n + 1 ) (n + 1 )
Applying integration by parts
e 1/ z ⋅ z n + 1 1  e 1/ z ⋅ z n 1 
= + + In − 2  = (sin x + cos x )m − 1(cos x + sin x ) − ∫ (m − 1 ) (sin x + cos x )m − 2
(n + 1 ) (n + 1 )  n n 
⋅ (cos x − sin x ) ⋅ (sin x − cos x ) dx
e1/z (z )n + 1 e1/z ⋅ (z )n 1
= + + In − 2 = (sin x + cos x )m − 1(sin x − cos x ) + (m − 1 ) ∫ (sin x + cos x )m − 2
(n + 1 ) (n + 1 ) n (n + 1 ) n
(sin x − cos x ) 2 dx
e 1/ z ⋅ (z )n + 1 e 1/ z ⋅ (z )n e 1/ z ⋅ (z )n −1 1 As we know, (sin x + cos x ) + (sin x − cos x ) 2 = 2,
2
= + + + In − 3
(n + 1) (n + 1) n (n + 1)n ⋅ (n −1) (n + 1) n (n − 1)
∴I m = (sin x + cos x )m − 1 (sin x − cos x ) + (m − 1 )
……………………………………………………… m−2
……………………………………………………… ∫ (sin x + cos x ) ⋅ {2 − (sin x + cos x ) 2 } dx
e1/z ⋅ (z )n + 1 e1/z ⋅ (z )n e1/z ⋅ (z )1 = (sin x + cos x )m − 1 (sin x − cos x ) + (m − 1 )
= + +…+
n+1 (n + 1 ) n (n + 1 ) n … 3 ⋅2 m−2
1
∫ 2 (sin x + cos x ) dx − (m − 1 ) ∫ (sin x + cos x )m dx
+ I0 I m = (sin x + cos x )m − 1 (sin x − cos x ) + 2 (m − 1 )
(n + 1 ) n (n − 1 ) … 3 ⋅2 I m − 2 − (m − 1 ) I m
Multiplying both the sides by (n + 1 ) !. We get, m −1
or (m − 1 ) I m + I m = (sin x + cos x ) (sin x − cos x ) + 2 (m − 1 ) I m − 2
(n + 1 ) ! I n = (e1/z ⋅ z n + 1 ⋅ n ! + e1/z ⋅ z n (n − 1 ) ! + … or m I m = (sin x + cos x )m − 1 (sin x − cos x ) + 2 (m − 1 ) I m − 2
+…+ e 1/ z
⋅ z ⋅ (2 ) ! + e
3 1/ z
⋅ z ⋅ 1 !) + I 0
2
Hence Proved.
⇒ I n (n + 1 ) ! = I 0 + e 1/ z
(1 ! z + 2 ! z + … + n ! z n + 1 )
2 3
l Ex. 33 If I m , n = ∫ cos x ⋅ cos nx dx , show that
m
Hence Proved.
(m + n ) I m , n = cos m x ⋅ sin nx + m I (m − 1, n − 1)
l Ex. 31 If I n = ∫ x n a 2 − x 2 dx , prove that Sol. We have,
I m , n = ∫ cosm x ⋅ cos n x dx
x n − 1 (a 2 − x 2 ) 3 / 2 (n − 1) 2 I II
In = − + a In − 2 .
(n + 2 ) (n + 2 ) sin nx  sin nx
= (cosm x ) − ∫ m cosm − 1 x ( − sin x ) ⋅ dx
Sol. I n = ∫ x n
a − x dx = ∫ x
2 2 n −1
⋅ {x a − x } dx 2 2  n  n
II I 1 m
cosm − 1 x {sin x ⋅ sin nx} dx
n ∫
= cosm x ⋅ sin nx +
Applying integration by parts, we get n
 (a 2 − x 2 ) 3/2  As we have, cos (n − 1 ) x = cos nx cos x + sin nx sin x
n − 2  (a − x )
2 2 3/ 2 
= x n − 1 ⋅ −  + ∫ (n − 1 ) x ⋅ −  dx
 3   3  1 m m −1
∴ I m, n =
n
cosm x ⋅ sin x +
n ∫ cos x {cos (n − 1 ) x
xn − 1 (a 2 − x 2 ) 3/2 (n − 1 ) − cos nx ⋅ cos x} dx
xn − 2 ⋅ (a 2 − x 2 ) a 2 − x 2 dx
3 ∫
=− +
3 1 m m −1 m
xn − 1 (a 2 − x 2 ) 3/2 (n − 1 ) a 2 (n − 1 )
= cosm x ⋅ sin x +
n n ∫ cos x ⋅ cos (n − 1 ) x dx −
n
⇒ In = − + In − 2 − In
∫ ⋅
m
3 3 3 cos x cos n x dx
(n − 1 ) xn − 1 (a 2 − x 2 ) 3/2 (n − 1 ) a 2 1 m m
∴ In + In = − + In − 2 = cosm x ⋅ sin nx + I m − 1, n − 1 − I m , n
3 3 3 n n n
60 Textbook of Integral Calculus

m 1 ( x sec x )
( x sin x + n cos x ) ∫
I m, n + I m , n = [cosm x ⋅ sin nx + m I m − 1, n − 1 ] =− + sec 2 x dx
n n
 m + n 1 ( x sec x )
  I = [cosm x ⋅ sin nx + m I m − 1, n − 1 ] =− + tan x + C Hence Proved.
 n  m, n n ( x sin x + n cos x )
(m + n ) I m , n = cosm x ⋅ sin nx + m I m − 1, n − 1
l Ex. 36 If cos θ > sin θ > 0, then evaluate
 π 
tan  − x   cos 2 θ 
 4    1 + sin 2 θ   cos 2 θ  
l Ex. 34 Evaluate ∫ cos 2 x dx . ∫  log  
 1 − sin 2 θ 
+ log    dθ
 1 + sin 2 θ  
tan 3 x + tan 2 x + tan x  
 π   cos 2 θ 
tan  − x  dx   1 + sin 2 θ   cos 2 θ  
 4  Sol. Here, I = ∫ log   + log    dθ
Sol. I = ∫   1 − sin 2 θ   1 + sin 2 θ  
cos x 2
tan x + tan x + tan x
3 2 

(1 − tan 2 x ) dx   cos θ + sin θ   cos θ + sin θ  


=∫ = ∫ 2 cos2 θ log   − log    dθ
(1 + tan x ) 2 cos2 x tan 3 x + tan 2 x + tan x   cos θ − sin θ   cos θ − sin θ  

 1   cos θ + sin θ 
− 1 −  sec 2 x dx = ∫ (2 cos2 θ − 1 ) log   dθ
 tan 2 x   cos θ − sin θ 
I =∫
 1  1  cos θ + sin θ 
 tan x + 2 +  tan x + 1 + = ∫ cos 2 θ ⋅ log   dθ, applying integration by parts
 tan x  tan x  cos θ − sin θ 
II
1 I
let y = tan x + 1 +
tan x  cos θ + sin θ  sin 2 θ 2 sin 2 θ
= log  ⋅ −∫ ⋅ dθ
 1   cos θ − sin θ  2 cos 2 θ 2
⇒ 2y dy =  sec 2 x − ⋅ sec 2 x  dx
 tan 2 x  sin 2 θ  cos θ + sin θ  1
= log   + log | cos 2 θ | + C
− 2y dy dy 2  cos θ − sin θ  2
∴ I =∫ = −2 ∫ = − 2 tan − 1 y + C
(y 2 + 1 ) ⋅ y 1 + y2
 1 
tan − 1 x
= − 2 tan −1
 tan x + 1 +  +C
l Ex. 37 Evaluate ∫ x4
dx .
 tan x 
tan − 1 x 1
x 2 + n (n − 1) Sol. I = ∫ dx = ∫ tan − 1 x ⋅ 4 dx
l Ex. 35 Evaluate ∫ ( x sin x + n cos x ) 2 dx. x4
I
x
II
 1  1 1
x 2 + n (n − 1 ) = (tan − 1 x )  − 3  − ∫ ⋅ dx
Sol. Here, I =∫ dx  3x  1 + x 2 (− 3x 3 )
( x sin x + n cos x ) 2
tan − 1 x 1 dx
Multiplying and dividing by x 2n − 2, we get =− + ∫ x 3 (1 + x 2 ) ,
3x 3 3
{ x 2 + n (n − 1 )} x 2n − 2
I =∫ n dx Put 1 + x 2 = t,
( x sin x + n xn − 1 cos x ) 2
tan − 1 x 1 dt
We know xn sin x + n xn − 1 cos x = t 2x dx = dt = −
3x 3
+
6 ∫ (t − 1)2 ⋅ t
⇒ {(n xn − 1 sin x ) + ( xn cos x ) + n (n − 1 ) xn − 2 cos x
tan − 1 x 1
− (n xn − 1 sin x ) dx = dt } I =− + I1 …(i)
3x 3 6
n−2
⇒ x cos x ⋅ { x + n (n − 1 )} dx = dt
2

1  A B C 
Keeping this in mind, we put Where, I 1 = ∫ dt = ∫  + +  dt
(t − 1 ) ⋅ t
2
 t − 1 (t − 1 ) 2
t 
{ x 2 + n (n − 1 )} ⋅ xn − 2 ⋅ cos x n
I =∫ ⋅ x ⋅ sec x dx
( xn sin x + n xn − 1 cos x ) 2 Comparing coefficients, we get
II I A = − 1, B = 1, C = 1
Applying integration by parts, we get
 1 1 1 
 1  ∴ I1 = ∫  − + +  dt
= xn sec x ⋅  − n n −1
  (t − 1 ) (t − 1 ) 2
t 
 ( x sin x + n x cos x )
( xn sec x tan x + n xn − 1 ⋅ sec x ) = − log | t − 1 | −
1
+ log | t |
+∫ dx …(ii)
( xn sin x + n xn − 1 cos x ) (t − 1 )
Chap 01 Indefinite Integral 61

∴ From Eqs. (i) and (ii), we get a + b sin x


−1
l Ex. 39 Evaluate ∫ dx
I =−
tan x 1
+
 1 2  (b + a sin x ) 2
− log | x | − 2 + log | 1 + x |  + C
2
3x 3 6  x  a2
− b + (b + a sin x )
−1
x + 1 1 2 a + b sin x b
=−
tan x 1
− log  − +C Sol. Here, I = ∫ dx = ∫ b dx
3x 3 6 2
 x  6x
2 (b + a sin x ) 2
a (b + a sin x ) 2
a2 − b2 dx b dx
l Ex. 38 Evaluate ∫ x log (1 − x ) dx , and hence prove
2 2 I =
a ∫ (b + a sin x )2 + a ∫ (b + a sin x ) ...(i)

1 1 1 2 8 cos x dA − b sin x − a
that + + +… = log 2 − . Now, let A = ⇒ =
1⋅5 2 ⋅7 3 ⋅ 9 3 9 b + a sin x dx (b + a sin x ) 2
 x2 x3 x4   a2 
Sol. We know, log (1 − x ) = −  x + + + +…∞  a sin x + b + −b 
2 3 4 dA b  b 
  ⇒ =−  
2 dx a  (b + a sin x ) 2

Put x instead of x in the above identity,  
 x4 x6 x8 
⇒ log (1 − x 2 ) = −  x 2 + + + +…∞  dA b  1 a − b2
2 
 2 3 4  ⇒ =−  + 2 
dx a  b + a sin x b (b + a sin x ) 
 x 6
x 8
x 10 
⇒ x 2 log (1 − x 2 ) = −  x 4 + + + +…∞  Integrating both the sides w.r.t. ‘x’, we get
 2 3 4 
b dx (a 2 − b 2 ) dx
Integrating both the sides, we get A=−
a ∫ b + a sin x − a ∫ (b + a sin x )2
 x5 x7 x9 
∫ − = − + + +…∞ + C
2 2
x log (1 x ) dx  a2 − b2 dx b dx
⋅ ⋅ ⋅
 1 5 2 7 3 9  ⇒
a ∫ (b + a sin x )2 = − a ∫ b + a sin x − A ...(ii)
Now, to find constant of integration, put x = 0
⇒ 0=0+C From Eqs. (i) and (ii), we get
⇒ C =0 I =− ∫
b dx b dx
 x5 x7 x9  a (b + a sin x )
−A+
a ∫ (b + a sin x )
∴ ∫ x 2 log (1 − x 2 ) dx = −  + + +…∞ 
 1 ⋅5 2 ⋅7 3 ⋅9  ⇒

I =−A+C ⇒ I =−
cos x 
II I  +C
 b + a sin x 
Applying integration by parts, and taking limits 0 to 1 for LHS
1
 x3  1 x 3 1 (− 2x ) dx
⇒  log (1 − x 2 )  − ∫ ⋅ dx l Ex. 40 Evaluate ∫ ⋅
 3 0 0 3 1 − x2
( x − 1) 3/ 4
( x + 2 ) 5/ 4
1 1
 x3  2  x3 1 1 + x  
⇒  log (1 − x 2 )  +  − − x + log   Sol. Let I = ∫
dx dx
( x − 1 ) 3/ 4 ( x + 2 ) 5/ 4 ∫
=
 3 0 3  3 2  1 − x  0  (x − 1) 
3/ 4
(x + 2)2  
Taking log (1 − x 2 ) = log (1 + x ) + log (1 − x )  (x + 2) 
1 + x x −1
and log   = log (1 + x ) − log (1 − x ) Let =t
1 − x x+2
1 1 2 2  x3 1  3
⇒ log 2 + log 2 − − + lim  −  log (1 − x ) So that, dx = dt
3 3 3 9 x→1  3 3  (x + 2)2
 
Q lim ( x 3 − 1 ) log (1 − x ) = 0 I =∫
dt 1 − 3/ 4
3 t 3/ 4 3 ∫
 x → 1  ∴ = t dt
2 8
⇒ log 2 − = RHS 1/ 4
3 9 1 t 1/ 4 4  x − 1
= ⋅ +C=   +C
1 1 1 2 8 3 1/4 3  x + 2
∴ + + + … = log (2 ) −
1 ⋅5 2 ⋅7 3 ⋅9 3 9
62 Textbook of Integral Calculus

#L Indefinite Integral Exercise 1 :


Single Option Correct Type Questions
x2 x2 −1
1. Let f ( x ) = ∫ dx and f (0) = 0. Then 8. ∫ x3 dx is equal to
(1 + x 2 ) (1 + 1 + x 2 ) 2x 4 − 2x 2 + 1
f (1) is equal to 2x 4 − 2x 5 + 1 2x 4 − 2x 2 + 1
π (a) +C (b) +C
(a) loge (1 + 2 ) (b) loge (1 + 2 ) − x 2
x3
4
π 2x 4 + 2x 2 + 1 2x 4 − 2x 2 + 1
(c) loge (1 + 2 ) + (d) None of these (c) +C (d) +C
4 x 2x 2

2. If ∫ f ( x ) dx = f ( x ), then ∫ { f ( x )} 2 dx is equal to 9. Let f ( x ) be a polynomial satisfying f (0) = 2, f ′ (0) = 3 and


1
f ′′( x ) = f ( x ). Then f ( 4 ) is equal to
(a) { f ( x )} 2 (b) { f ( x )} 3 5(e 8 + 1 ) 5 (e 8 − 1 )
2 (a) (b)
{ f ( x )} 3 2e 4 2e 4
(c) (d) { f ( x )} 2 2e 4 2e 4
3 (c) (d)
5 (e 8 − 1 ) 5 (e 8 + 1 )
3. If ∫ f ( x ) dx = F ( x ), then ∫ x 3 f ( x 2 ) dx is equal to
2 2
+ 4 ln x )
e (x − x3 ex

1
(a) [ x 2 { F ( x )} 2 − ∫ { F ( x )} 2 dx ] 10. dx is equal to
2 x −1
1
(b) [ x 2F ( x 2 ) − ∫ F ( x 2 ) d ( x 2 )]
2
 e 3 ln x − e ln x  x 2 ( x − 1 ) xe x
2 (a)  e +C (b) +C
1 2 1  2x  2
(c) [ x F ( x ) − ∫ { F ( x )} 2 dx ]
2 2 (x 2 − 1) 2
(c) − ex + C (d) None of these
(d) None of the above 2x
4. If n is an odd positive integer, then ∫ | x n | dx is 11. ∫ tan 4 x dx = A tan 3 x + B tan x + f ( x ), then
equal to 1
(a) A = , B = − 1, f ( x ) = x + C
xn + 1 xn + 1 3
(a) +C (b) +C
n+1 n+1 2
(b) A = , B = − 1, f ( x ) = x + C
3
|x |n x
(c) +C (d) None of these 1
n+1 (c) A = , B = 1, f ( x ) = x + C
3
3x + 2 2
5. Let F ( x ) be the primitive of w.r.t. x. If F(10) = 60, (d) A = , B = 1, f ( x ) = − x + C
x −9 3
then the value of F(13) is sin 4 x
12. If the anti-derivative of ∫ dx is f ( x ), then
(a) 66 (b) 132 x
(c) 248 (d) 264 sin 4 {( p + q )x }
6. ∫ ( x x ) x (2x loge x + x ) dx is equal to ∫ x
dx in terms of f ( x ) is
f {( p + q ) x }
(a) x ( x
x
)
+C (b) ( x x ) x + C (a) f {( p + q ) x } (b)
p+q
(c) x 2 ⋅ loge x + C (d) None of these (c) f {( p + q ) x } ( p + q ) (d) None of these
7. The value of ∫ x log x (log x − 1) dx is equal to  sin θ sin 3θ sin 9θ 
13. ∫  + +  dθ is equal to
(a) 2 ( x log x − x ) + C 2
 cos 3θ cos 9θ cos 27θ
1
(b) ( x log x − x ) 2 + C 1 sec 27 θ 1 sec θ
2 (a) log +C (b) log +C
2 sec θ 2 sec 27 θ
(c) ( x log x ) 2 + C
1 1 27 sec 27θ
(d) ( x log x ) 3 + C (c) log +C (d) None of these
2 2 sec θ
Chap 01 Indefinite Integral 63

14. Let x 2 ≠ nπ − 1, n ∈ N . Then, the value of 18. The primitive of the function f ( x ) = x | cos x |, when
2 sin ( x + 1) − sin 2 ( x + 1)
2 2 π
< x < π is given by
∫x 2 sin ( x 2 + 1) + sin 2 ( x 2 + 1)
dx is equal to 2
(a) cos x + x sin x + C
1  x 2 + 1 (b) − cos x − x sin x + C
(a) log sec ( x 2 + 1 ) + C (b) log sec   +C
2  2  (c) x sin x − cos x + C
1 (d) None of the above
(c) log | sec ( x 2 + 1 )| + C (d) None of these
2
19. The primitive of the function f ( x ) = (2x + 1)| sin x |,
dx
15. ∫ cos (2x ) cos ( 4 x )
is equal to when π < x < 2π is
(a) − (2 x + 1 ) cos x + 2 sin x + C
1 1 + 2 sin 2 x 1 (b) (2 x + 1 ) cos x − 2 sin x + C
(a) log − (log | sec 2 x − tan 2 x | ) + C
2 2 1 − 2 sin 2 x 2 (c) ( x 2 + x ) cos x + C
1 1 + 2 sin 2 x 1 (d) None of the above
(b) log − (log | sec 2 x − tan 2 x | ) + C
2 2 1 + 2 sin x 2
0 x 2 − sin x cos x − 2
1 1 + 2 sin 2 x 1
(c) log − (log | sec 2 x − tan 2 x | ) + C 20. Given, f ( x ) = sin x − x 2 0 1 − 2x , then
2 1 − 2 sin 2 x 2
2 − cos x 2x − 1 0
(d) None of the above
1 − 7 cos 2 x f (x ) ∫ f ( x ) dx is equal to
16. ∫ sin 7 x cos 2 x dx = (sin x ) 7 + C, then f ( x ) is equal to
x3
(a) − x 2 sin x + sin 2 x + C
(a) sin x (b) cos x (c) tan x (d) cot x 3
3
sin x x3
17. ∫ (cos 4 x + 3 cos 2 x + 1) tan −1 (sec x + cos x ) dx is equal to (b)
3
− x 2 sin x − cos2 x + C

(a) tan −1 (sec x + cos x ) + C (b) loge | tan −1 ( sec x + cos x ) | + C x3


(c) − x 2 cos x − cos2 x + C
1 3
(c) +C (d) None of these (d) None of the above
( sec x + cos x ) 2

#L Indefinite Integral Exercise 2 :


More than One Option Correct Type Questions
dx 4e x + 6e − x
21. ∫ ( x + 1) ( x − 2) = A log ( x + 1) + B log ( x − 2) + C , where 24. If ∫ −x
dx = Ax + B loge (9e 2 x − 4 ) + C , then
9e x
− 4e
(a) A + B = 0 (b) AB = 0 3 35
(c) A / B = − 1 (d) None of these (a) A = (b) B =
2 36
dx x 19
22. If ∫ = k tan −1 x + l tan −1 + C , then (c) C is indefinite (d) A + B = −
( x + 1)( x + 4 )
2 2 2 36

(a) k =
1
(b) l =
2
(c) k = −
1
(d) l = −
1 25. If ∫ tan 5 x dx = A tan 4 x + B tan 2 x + g ( x ) + C , then
3 3 3 6 1 1
(a) A = , B = −
23. If ∫ x log(1 + x 2 ) dx = φ ( x ) log(1 + x 2 ) + x ( ψ ) + C , then 4 2
1 + x2 1 + x2 (b) g( x ) = ln | sec x |
(a) φ( x ) = (b) ψ ( x ) =
2 2 (c) g( x ) = ln | cos x |
1 + x2 1 + x2 1
(d) A = − , B =
1
(c) ψ( x ) = − (d) φ( x ) = −
2 2 4 3
64 Textbook of Integral Calculus

#L Indefinite Integral Exercise 3 :


Statement I and II Type Questions
n Directions (Q. Nos. 26 to 30) For the following 28. Statement I If a > 0 and b 2 − 4ac < 0, then the value of
questions, choose the correct answers from the codes (a), dx
(b), (c) and (d) defined as follows : the integral ∫ will be of the type
(a) Statement I is true, Statement II is also true; Statement II ax + bx + c
2

x+A
is the correct explanation of Statement I. µ tan −1 + C, where A, B, C , µ are constants.
(b) Statement I is true, Statement II is also true; Statement II B
is not the correct explanation of Statement I. Statement II If a > 0, b 2 − 4ac < 0, then ax 2 + bx + C
(c) Statement I is true, Statement II is false. can be written as sum of two squares.
(d) Statement I is false, Statement II is true.  
1 −1
26. Statement I If y is a function of x such that 29. Statement I ∫  1 + x 4  dx = tan (x 2 ) + C
dx 1
y ( x − y ) 2 = x , then ∫ = [log ( x − y ) 2 − 1] 1 −1
x − 3y 2 Statement II ∫ 1 + x 2 dx = tan x+C
dx
Statement II ∫ x − 3y = log ( x − 3y ) + C
tan −1 x −1
−1
2 tan x
27. Statement I Integral of an even function is not always
30. Statement I ∫2 d (cot x) =
ln 2
+C

an odd function. d x
Statement II (a + C ) = a x ln a
dx
Statement II Integral of an odd function is an even
function.

#L Indefinite Integral Exercise 4 :


Passage Based Questions
Passage I ( x + 1 + x 2 )15
(Q. Nos. 31 to 33) 32. ∫ 1+ x2
dx is equal to
Let us consider the integral of the following forms
( x + 1 + x 2 )16
f ( x1 , mx 2 + nx + p )1/ 2 (a) +C
10
1
Case I If m > 0, then put mx 2 + nx + C = u ± x m (b) +C
15 ( 1 + x 2 + x )
Case II If p > 0, then put mx 2 + nx + C = ux ± p 15
(c) +C
Case III If quadratic equation mx + nx + p = 0 has real roots
2 ( 1 + x2 − x)

α and β there put mx 2 + nx + p = ( x − α ) u or ( x − β ) u ( x + 1 + x 2 )15


(d) +C
15
dx
31. If I = ∫ to evaluate I, one of the most 33. To evaluate ∫
dx
one of the most
x − 9x 2 + 4x + 6 ( x − 1) − x 2 + 3x − 2
proper substitution could be suitable substitution could be
(a) 9 x 2 + 4 x + 6 = u ± 3 x
(a) − x 2 + 3 x − 2 = u
(b) 9 x + 4 x + 6 = 3u ± x
2
(b) − x 2 + 3 x − 2 = (ux 2 )
1
(c) x = (c) − x 2 + 3 x − 2 = u (1 − x )
t
1
(d) 9 x + 4 x + 6 =
2
(d) − x 2 + 3 x − 2 = u ( x + 2 )
t
Chap 01 Indefinite Integral 65

1
Passage II (b) I 4 , 2 = ( − sin 5 x cos3 x + 8 I 4 , 4 )
3
(Q. Nos. 34 to 36) 1
(c) I 4 , 2 = (sin 5 x cos3 x − 8 I 4 , 4 )
Let I n, m = ∫ sin x cos m x dx. Then, we can relate I n, m with
n
3
1
each of the following : (d) I 4 , 2 = (sin 5 x cos3 x + 6 I 4 , 4 )
3
(i) I n − 2, m (ii) I n + 2, m
(iii) I n, m − 2 (iv) I n, m + 2 Passage III
(v) I n − 2, m + 2 (vi) I n + 2, m − 2 (Q. Nos. 37 to 38)
Suppose we want to establish a relation between I n, m and If f : R → ( 0, ∞ ) be a differentiable function f ( x ) satisfying
I n, m − 2 , then we get f ( x + y ) − f ( x − y ) = f ( x ) ⋅ { f ( y ) − f ( y ) − y}, ∀ x, y ∈ R,
( f ( y ) ≠ f ( − y ) for all y ∈ R) and f ′ ( 0) = 2010.
P ( x ) = sin n + 1 x cos m − 1 x …(i)
Now, answer the following questions.
In I n, m and I n, m − 2 the exponent of cos x is m and m − 2
respectively, the minimum of the two is m − 2, adding 1 to the
37. Which of the following is true for f ( x )
minimum we get m − 2 + 1 = m − 1. Now, choose the exponent (a) f ( x ) is one-one and into
m − 1of cos x in P ( x ). Similarly, choose the exponent of sin x for (b) { f ( x )} is non-periodic, where {⋅} denotes fractional part
of x.
P ( x ) = ( nH ) sin n x cos m x − ( m − 1) sin n + 2 x cos m − 2 x.
(c) f ( x ) = 4 has only two solutions.
Now, differentiating both the sides of Eq. (i), we get (d) f ( x ) = f −1( x ) has only one solution.
= ( n + 1) sin n x cos m x − ( m − 1) sin n x (1 − cos 2 x ) cos m − 2 x
38. let g ( x ) = loge (sin x ), and ∫ f ( g ( x )) cos x dx = h( x ) + c ,
= ( n + 1) sin n x cos m x − ( m − 1) sin n x cos m − 2 x
 π
+ ( m − 1) sin n x cos n x (where c is constant of integration), then h   is equal
 2
= ( n + m ) sin n x cos m x − ( m − 1) sin n x cos m − 2 x to
1
(a) 0 (b)
Now, integrating both the sides, we get 2010
1
sin n + 1 x cos m − 1 x = ( n + m ) I n, m − ( m − 1) I n, m − 2 (c) 1 (d)
2011
Similarly, we can establish the other relations.
Passage IV
34. The relation between I 4 , 2 and I 2 , 2 is
(Q. Nos. 39 to 41)
1
(a) I 4 , 2 = ( − sin 3 x cos3 x + 3 I 2, 2 ) Let f : R → R be a function as
6
1 f ( x ) = ( x − 1)( x + 2)( x − 3)( x − 6) − 100. If g ( x ) is a
(b) I 4 , 2 = (sin 3 x cos3 x + 3 I 2, 2 ) g (x )
6 polynomial of degree ≤ 3 such that ∫ dx does not contain
1 f (x )
(c) I 4 , 2 = (sin 3 x cos3 x − 3 I 2, 2 )
6 any logarithm function and g ( − 2) = 10. Then
1
(d) I 4 , 2 = ( − sin 3 x cos3 x + 2 I 2, 2 ) 39. The equation f ( x ) = 0 has
4
(a) all four distinct roots
35. The relation between I 4 , 2 and I 6 , 2 is (b) three distinct real roots
1 (c) two real and two imaginary
(a) I 4 , 2 = (sin 5 x cos3 x + 8 I 6, 2 )
5 (d) all four imaginary roots
1 40. The minimum value of f ( x ) is
(b) I 4 , 2 = ( − sin 5 x cos3 x + 8 I 6, 2 )
5
1 (a) − 136 (b) − 100
(c) I 4 , 2 = (sin 5 x cos3 x − 8 I 6, 2 ) (c) − 84 (d) − 68
5
g( x )
∫ f ( x ) dx , equals
1 41.
(d) I 4 , 2 = (sin 5 x cos3 x + 8 I 6, 2 )
6
36. The relation between I 4 , 2 and I 4 , 4 is  x − 2  x − 1
(a) tan −1   +c (b) tan −1   +c
 2   1 
1
(a) I 4 , 2 = (sin 5 x cos3 x + 8 I 4 , 4 ) (c) tan −1 ( x ) + c (d) None of these
3
66 Textbook of Integral Calculus

#L Indefinite Integral Exercise 5 :


Matching Type Questions
42. Match the following :
Column I Column II
sin x − cos x π 3π sin x
(A) If I = ∫ dx , where < x < , then I is equal to (p)
| sin x − cos x | 4 8

x2 1  x 3 + 1 (q) x+C
(B) If ∫ dx = f  3  + C , then f (x ) is equal to
(x + 1) (x + 2)
3 3
3  x + 2

π  (r) ln | x |
(C) If ∫ sin −1 x ⋅ cos−1x dx = f −1 (x ) x − xf −1 (x ) − 2 1 − x 2 + 2x + C , then f (x ) is equal to
 2 

(D) If ∫
dx
= f ( f (x )) + C , then f (x ) is equal to (s) sin −1 x
xf (x )

43. Match the following :


Column I Column II
 x + cos x 
2 2
cosec x (p) A =1
(A) If ∫   cosec2x dx = A cot −1 x + B , then
 1+ x 2
sec x

(B) If ∫ x + x 2 + 2 dx =
A
(x + x 2 + 2 )3/ 2 −
B
, then (q) B = −1
3 (x + x + 2)
2

2 − x − x2 2 − x − x2  4 − x + 4 2 − x − x2  (r) B=2
(C) If ∫ dx = A +
B
log   − sin −1  2x + 1 , then
2    3 
x x 4 2  x 

(D) If ∫
sin 2x
dx = B cot −1 (tan 2 x ) , then (s) A = −1
sin 4 x + cos4 x

#L Indefinite Integral Exercise 6 :


Single Integer Answer Type Questions
(2x + 3) dx 1
44. If ∫ =C − , where f ( x ) is of the form of ax 2 + bx + c , then (a + b + c ) equals to …… .
x ( x + 1) ( x + 2) ( x + 3) + 1 f (x )
3x + 2
45. Let F ( x ) be the primitive of w.r.t. x. If F(10) = 60, then the sum of digits of the value of F(13), is ……… .
x −9
u( x ) u′ (x )  u( x ) ′ p +q
46. Let u( x ) and v ( x ) are differentiable function such that = 7. If = p and   = q , then has the value
v( x ) v ′ (x )  v ( x ) p −q
equal to ……… .
2
1  x − 1   x − 1 
47. If ∫ 2 ln   dx = 6A ln    + C , then find 24 A.
( x − 1)  x + 1   x + 1 
λ
e x (2 − x 2 ) 1 + x 
48. If ∫ dx = µe x
  + C , then 2( λ + µ ) is equal to ……… .
(1 − x ) 1 − x 2 1 − x 
Chap 01 Indefinite Integral 67

cos x − sin x + 1 − x tan x 1


49. If ∫ dx = ln { f ( x )} + g ( x ) + C , where 52. The value of ∫ dx = x − tan −1
e + sin x + x
x
1 + tan x + tan x 2
A
C is the constant of integrating and f ( x ) is positive, then  2 tan x + 1
f ( x ) + g( x )   + C , then the value of A is……… .
is equal to ……… .  A 
e x + sin x
53. ∫ sin 5 / 2 x cos 3 x dx = 2 sin A / 2 x  − sin 2
 1 1
x + C,
dx dx B C 
50. Suppose A = ∫ and B = ∫ ⋅
x 2 + 6x + 25 x 2 − 6x − 27 then the value of ( A + B ) − C is equal to ……… .
 x + 3 x −9 54. If∫ (x 2010 + x 804 + x 402 )(2x 1608 + 5x 402 + 10)1/ 402 dx
If 12( A + B ) = λ ⋅ tan −1   + µ ⋅ ln + C , then 1
 4  x +3 = (2x 2010 + 5x 804 + 10x 402 )a / 402 . Then (a − 400) is
10a
the value of ( λ + µ ) is … .
equal to ...........
cos 6x + cos 9 x sin 4 x
51. If ∫
3
dx = − − sin x + C , then the + x2 −1
55. If ∫ e x (3x 4 + 2x 3 + 2x ) dx = h( x ) + c Then the
1 − 2 cos 5x k
value of k is ……… . value of h (1) ⋅ h ( − 1), is ………

Indefinite Integral Exercise 7 :


Subjective Type Questions
(x sin x + cos x )  x 4 cos 3 x − x sin x + cos x  64. Evaluate ∫
dx
,a ∈N.
56. Evaluate e dx  
 x 2 cos 2 x  (sin x + a sec x ) 2
dx
57. Evaluate ∫ x + x 2 + 2 dx . 65. Evaluate ∫ ⋅
x − x + 2x + 4
2

dx
58. Evaluate ∫ .
66. Evaluate ∫
dx

( ( x − α ) − β 2 ) (ax + b )
2
1 + x + 2x + 2
2

1+ 3
x x4 +1
59. Evaluate ∫ dx. 67. Evaluate ∫ dx .
3
x2 x6 +1
sin 3 (θ / 2) dθ dx
60. Evaluate ∫ ⋅ 68. Evaluate ∫ dx .
cos θ / 2 cos θ + cos θ + cos θ
3 2 (1 − x 3 )1/ 3
(2 sin θ + sin 2θ) dθ ( x + 1 + x 2 )15
61. Evaluate ∫ ⋅ 69. Evaluate ∫ dx .
(cos θ − 1) cos θ + cos 2 θ + cos 3 θ 1+ x2
62. Connect ∫ x m − 1 (a + bx n ) p dx with xn
70. If y 2 = ax 2 + 2bx + c , and u n = ∫ dx , prove that
x 8 dx y
∫ x m − n − 1 (a + bx n ) p dx and evaluate ∫ ⋅
(1 − x 3 )1/ 3 (n + 1) a u n + 1 + (2n + 1) bu n + n c y n − 1 = x n y and deduce

63. Evaluate ∫ cosec 2 x ln (cos x + cos 2x ) dx . that au 1 = y − b u 0 ; 2a 2 u 2 = y (ax − 3b ) − (ac − 3b 2 ) u 0 .


68 Textbook of Integral Calculus

#L Indefinite Integrals Exercise 8 :


Questions Asked in Previous 10 Years’ Exams

(i) JEE Advanced & IIT-JEE


sec 2 x ex e −x
71. ∫ (sec x + tan x ) 9 / 2
dx equals to (for some arbitrary 72. If I = ∫
e 4x + e 2x + 1
dx , J = ∫
e − 4 x + e −2 x + 1
dx . Then,

constant K) [Only One Correct Option 2012]


for an arbitrary constant c, the value of J − I equals
−1 1 1 2
(a)  − (sec x + tan x )  + K [Only One Correct Option 2008]
(sec x + tan x )11/2 11 7 
1 1 1 2 1 e 4 x − e 2x + 1 1 e 2x + e x + 1
(b)  − (sec x + tan x )  + K (a) log 4 x + C (b) log 2x +C
(sec x + tan x )11/2 11 7  2 e +e +1 2x
2 e − ex + 1
−1 1 1 
(c) 11/ 2 
+ (sec x + tan x ) 2  + K
(sec x + tan x ) 11 7  1 e 2x − e x + 1 1 e 4 x + e 2x + 1
(c) log 2x +C (d) log 4 x +C
1 1 1 2 2 e + ex + 1 2 e − e 2x + 1
(d)  + (sec x + tan x )  + K
(sec x + tan x )11/2 11 7 

(ii) JEE Main & AIEEE


2x 12 + 5x 9 76. If ∫ f ( x ) dx = ψ( x ), then ∫ x 5 f ( x 3 ) dx is equal to
73. The integral ∫ dx is equal to
( x 5 + x 3 + 1) 3 [2016 JEE Main] 1 3
[2013 JEE Main]
(a) [ x ψ ( x 3 ) − ∫ x 2ψ ( x 3 )dx ] + C
−x 5
x 10
3
(a) +C (b) +C 1
(x + x 3 + 1)2
5
2( x + x 3 + 1 ) 2
5
(b) x 3ψ ( x 3 ) − 3 ∫ x 3ψ ( x 3 ) dx + C
3
x5 − x10
(c) +C (d) +C 1 3
2( x 5 + x 3 + 1 ) 2 2( x 5 + x 3 + 1 ) 2 (c) x ψ ( x 3 ) − ∫ x 2ψ ( x 3 ) dx + C
3
where, C is an arbitrary constant. 1
(d) [ x 3ψ ( x 3 ) − ∫ x 3ψ ( x 3 ) dx ] + C
dx 3
74. The integral ∫ 3
equals
5 tan x
x 2 ( x 4 + 1) 4 77. If the integral ∫ dx = x + a log
[2015 JEE Main] tan x − 2
1
 x 4 + 1 4
1 | sin x − 2 cos x | + k , then a is equal to [2012 AIEEE]
(a)   +C (b) ( x 4 + 1 ) 4 + C (a) –1 (b) –2
 x4  (c) 1 (d) 2
1
1
 x 4 + 1 4 sin x dx
(c) −( x + 4
1) 4 +C (d) −   +C 78. The value of 2 ∫ is
 x4   π
sin  x − 
1  4 [2012 AIEEE]
1 x+
75. The integral ∫ 1 + x −  e x dx is equal to π π
 x [2014 JEE Main]  
(a) x + log cos  x −  + C (b) x + log sin  x −  + C
1 1  4  4
x+ x+
(a) ( x − 1) e x +C (b) xe x +C
 π  π
x+
1
x+
1 (c) x − log sin  x −  + C (d) x − log cos  x −  + C
(c) ( x + 1 ) e x +C (d) − x e x +C  4  4
Answers
Exercise for Session 1 12.
1
2x2 + 3x −
3
log x +
3 2 3
x + x +C
2 2 2 2 3 2 4 2 4 2
1. (x + 1)3/ 2 + x3/ 2 + c 2. − + − − 2 tan −1 x + c
3 3 x 3x3 5x5 −1 a − x
13. a − x(x − b) − (a − b) tan +C
x3 x−b
3. log x + 2 tan −1 x + c 4. − x + tan −1 x + c
3 14. sin −1 x + 1 − x2 + C
1  x3 
5.  + tan −1 x + c 6. tan x − tan −1
x+ c (2 x + 5 ) 2
log  x +  + x2 + x + 1 + C
15 1
2 3  15. x + x+1+
4 8  2
1  a2  π
17. log tan  +  + C
bx − 2a log | bx + a| − x 1 x
7.
3 + c 16. log tan + 1 + C
b  a + bx  2 2 2 6
2x ex e4x 1 1 1
8. + c 9. + c 18. log |sin x − cos x| + cos 2x + sin 2x + C
1 + loge 2 4 4 8 8
−1  e + 2 
x
2
xa + 1 ax 1 19. sin   +C 20. − sin −1 (cos3/ 2 x) + C
10. + + c 11. − cos 4x + C  3  3
a + 1 log a 8
12 5
12.
1 1
ln | sec 3x | − ln | sec 2 x | − ln | sec x | + e 21. (3 sin x − 2 cos x) − log (3 cos x + 2 sin x) + C
3 2 13 13
2 3 1 22.
13. sin 3x + 2 sin x + C 14. sin x + sin 3x + C 1  2x − 3  
sin −1
2 3 25
3 4 12 − (4 + 3 x − x2 ) 3/ 2 −  x −  4 + 3x − x +
2
 +C
3 1 3 2   2 4  5  
15. − cos 2x + cos 6x + C
23. 2 x2 + x + 1 + 2 log  x +
64 192 1
 + x2 + x + 1
 2
Exercise for Session 2 1− x+ x2 + x + 1
− 6 log +C
1. tan x − sec x + C 2. sin 2x + C 2 (x + 1)
24. (c)
cos 3x 180
3. − +C 4. sin x° + C
3 π Exercise for Session 4
5. x + C 6. 2(sin x + x cos α ) + C
1. x2ex − 2(xex − ex ) + C 2. − x2 cos x + 2(x sin x + cos x) + C
7. sec x − cosec x + C 8. tan x − cot x + C
9. (sin x + cos x) sin (cos x − sin x) + C 3. x(log x) − x + C 4. x(log x)2 − 2(x log x − x) + C

11. − 2 cos   + C
1
10. tan x − cot x − 3x + C
x 5. x tan −1 x − log |1 + x2 | + C 6. x(sec−1 x) − log | x + x2 − 1| + C
 2 2
− cos 4x 1 x2 1 1
12. +C 13. (x − sin x) + C 7. tan −1 − (x − tan −1 x) + C 8. − (1 + log x) + C
8 2 2 2 x
x
14. − 2 cos x + C 15.
x
+C 9. − x cot + C 10. x log (x2 + 1) − 2x + 2 tan −1 x + C
2 2
11. ex ⋅ log (sec x) + C 12. ex tan x + C
Exercise for Session 3 13. x log (log x) −
x
+C
1
14. e2x tan x + C
1  3x4  1  4x3  log x 2
1. tan −1   + C 2. tan −1   + C ex 1+ x
48  4  36  3  15. +C 16. ex ⋅ +C
x +1
2
1− x
1 4x − 5
4
2  x3/ 2 
4. sin −1 
ax
3. log 4 +C  +C
3/ 2  17.
e
{a cos (bx + c) + b sin (bx + c)} + C
160 4x + 5 3 a  a + b2
2

1 1 2 + 3e x 1 1
sec x tan x + log | sec x + tan x| + C
5. log | x3 + a6 + x6 | + C 6. log +C 18.
3 12 2 − 3ex 2 2
1 19. − 2(− x cos x + sin x ) + C
7. log |2x + 4x − 25 | + C
loge 2 20. x(sin −1 x)2 − 2(− sin −1 x ⋅ 1 − x2 + x) + C

8. − 8 5 + 2x − x2 − 3 sin −1 
 x − 1 + C 21.
1 1
x tan −1 x − log (1 + x2 ) − (1 − x) tan −1 (1 − x) + log{1 + (1 + x2 )} + C

 6  2 2
x x x x
1
9. log | x2 + 2x + 2| + tan −1 (x + 1) + C 22. a  tan −1 − + tan −1  + C
2 a a a a
3/ 2 3/ 2
 x − 1 + C
−  1 + 2  ⋅ log  1 + 2  +  1 + 2 
1 1 1 2 1
10. − 3 − 2x − x2 − 4 sin −1   23. +C
 6  3 x   x  9 x 
−1  2 x − 1
cos x x 2
11.   − − log | 2 cos x − sin x | + C
3 1
log |4x − 4x + 17| + tan   + C
2 24.
8 6  4   2 cos x − sin x 5 5
25. esin x (x − sec x) + C
70 Textbook of Integral Calculus

Exercise for Session 5 31. (a) 32. (d) 33. (c) 34. (a) 35. (a) 36. (b)
1 9 37. (b) 38. (d) 39. (c) 40. (c) 41. (a)
1. log | x − 1| − 4 log | x − 2| + log | x − 3| + C 42. A → q; B → r; C → p; D → r
2 2
2 x − 1 43. A → p, q; B → p, r; C → r; D → q 44. (5) 45. (6)
tan −1 
1 1 1
2. log |1 + x| − log |1 − x + x2 | +  + C. 46. (1) 47. (1) 48. (3) 49. (1) 50. (4) 51. (4)
3 6 3  3 
52. (3) 53. (3) 54. (3) 55. (1)
1 xn 1 x2 + 1  1 
3. log n +C 4. log 2 +C 56. e(x sin x + cos x) ⋅   x −  + C
n x +1 2 x + 3  cos x 
 x
1 + sin x 1 1 θ
5. log +C 57. (x + x2 + 2 )3/ 2 − 2 +C 58. tan =t
2 + sin x 3 x + x2 + 2 2
1 1 2
6. log |1 − cos x | − log |1 + cos x| + log | 3 + 2 cos x | + C 59. 2 (1 + x1/ 3 )3/ 2 + C 60. tan −1 (cos θ + sec θ + 1)1/ 2 + C
10 2 5
2 cos θ + sec θ + 1 − 3
1 1 + sin x 1 61. − log +C
7. log + +C 3 cos θ + sec θ + 1 + 3
4 1 − sin x 2(1 + sin x)
1 3 3 9
8.
1 1
− log |1 + tan x| + log |tan 2 x − tan x + 1| 62. − x6 (1 − x3 )2/ 3 − x (1 − x3 )2/ 3 − (1 − x3 )2/ 3 + C
3 6 8 20 40
2 tan x − 1
tan −1  63. − cot x log (cos x + cos 2x ) − cot x − x + cot 2 x − 1 + C
1
+  +C
3  3 
1  −1  2a sin 2x + 1  2a sin 2x + 1 
2 log x + 1 64. 2a sin   + 1−  
9. log +C (4a2 − 1)3/ 2   2a + sin 2x   2a + sin 2x  
3 log x + 2
1
tan −1 x 1 − +C
10. − + log | x| − log |1 + x2 | + C (2a + sin 2x)
x 2 3 log
65. 2 ln | x2 + 2x + 4 − x | −
Exercise for Session 6 2( x2 + 2x + 4 − (x + 1))
3
x4 + x2 + 1 2  x  − ln x2 + 2x + 4 − x − 1 + C
1. + C, 2. tan −1   + C. 2
x2
3  3 x+ 1  2
66. log (x + 1 + x + 2x + 2 ) +
2
+C
3. 2t 3 − 3t 2 + 6t − 6 log |1 + t | + C , where, t = (x + 1)1/ 6. (x + 2) + (x2 + 2x + 2)
1/ 7
7  x − b 67. tan −1  x − 
1 2
− tan −1 (x3 ) + C
4.   + C.
(a + b )  x + a   x 3
5. 2 sec A ( tan x sin A + cos A ) + C 6. (a) 7. (d) 1 (1 − x3 )1/ 3 + x 1 (1 − x3 )2/ 3 − x (1 − x3 )1/ 3 + x2
68. log − log
8. (c) 9. (d) 10. (c) 3 x 6 x2
1/ 3
1  2(1 − x3 ) 
Chapter Exercises − tan −1  − x +C
3  3x 
1. (b) 2. (a) 3. (b) 4. (c) 5. (b) 6. (b)
7. (b) 8. (d) 9. (b) 10. (d) 11. (a) 12. (a) (x + 1 + x2 )15
69. +C 71. (c) 72. (c) 73. (b)
13. (c) 14. (b) 15. (a) 16. (c) 17. (b) 18. (b) 15
19. (b) 20. (d) 21. (a,c) 22. (a, d) 23. (a, c) 24. (b,c,d) 74. (d) 75. (b) 76. (c) 77. (d) 78. (b)
25. (a, b) 26. (c) 27. (c) 28. (a) 29. (d) 30. (d)
3x + 2
5. F ( x ) = ∫ dx. Let x − 9 = t 2
x −9

Solutions ⇒


dx = 2t dt
 3 (t 2 + 9 ) + 2 
F (x ) = ∫ 
 t
⋅ 2t dt

= 2 ∫ (29 + 3t 2 ) dt = 2 [29t + t 3 ]
x2 F ( x ) = 2 [29 x − 9 + ( x − 9 ) 3/2 ] + C
1. Let I = dx
(1 + x )1 + (1 + x 2 )
2
Given, F (10 ) = 60 = 2 [29 + 1 ] + C ⇒ C = 0
Put x = tan θ ⇒ dx = sec 2 dθ ∴ F ( x ) = 2 [29 x − 9 + ( x − 9 ) 3/2 ]
tan 2 θ F (13 ) = 2 [29 × 2 + 4 × 2 ] = 132
Putting, x = tan θ, I = ∫ dθ
1 + sec θ
∫ (x ) (2 x loge x + x ) dx
x x
6. We have,
= ∫ (sec θ − 1 ) dθ 2
= ∫ x x (2 x loge x + x ) dx
= log (sec θ + tan θ ) − θ + C 2 2

−1
= ∫ 1 ⋅ d (x x ) = x x + C = (x x )x + C
f ( x ) = loge ( x + x + 1 ) − tan x + C
2

f (0) = 0 ⇒ C = 0
7. We have, ∫ x log x (log x − 1) dx
⇒ f (1 ) = loge (1 + 2 ) − tan −1 1 = ∫ log x ( x log x − x ) dx
π
= loge (1 + 2 ) −
4 = ∫ ( x log x − x ) d ( x log x − x )
2. We have, ∫ f ( x ) dx = f ( x )
( x log x − x ) 2

d
{ f ( x )} = f ( x ) ⇒
1
d { f ( x )} = dx = +C
dx f (x ) 2

⇒ log { f ( x )} = x + log C ⇒ f ( x ) = Ce x  1 1
 3 − 5  dx
x x 
⇒ { f ( x )} 2 = C 2e 2x 8. ∫ 2 1
2 2x 2− 2 + 4
Ce 1
∫ { f (x )} dx = ∫ C 2e 2x dx =
⇒ 2
= { f ( x )} 2 x x
2 2 2 1 1 dz
3. We have, ∫ f ( x ) dx = F ( x )
Let 2−
x 2
+ 4 =z ⇒
x 4 ∫ z
1 1 1 2 1
⇒ z +C ⇒ 2− 2 + 4 +C
∴ ∫x f ( x 2 ) dx = ∫ x{ 1
3 2 2 2
f (x ) d (x )
2 4243 2 2 x x
I II
1 2 2x 4 − 2x 2 + 1
= [ x F ( x ) − ∫ F ( x 2 ) d ( x 2 )]
2 or +C
2 2x 2
4. We have the following cases : 9. We have, f ′′ ( x ) = f ( x )
Case I When x ≥ 0 ⇒ 2 f ′ ( x ) f ′′ ( x ) = 2 f ( x ) f ′ ( x )
In this case, we have
d d
⇒ { f ′ ( x )} 2 = { f ( x )} 2
∫ |x | dx = ∫ | x |n dx = ∫ xn dx [Q | x | = x ]
n
dx dx
xn + 1 | x |n x ⇒ { f ′ ( x )} 2 = { f ( x )} 2 + C …(i)
= +C = +C [Q x ≥ 0 ⇒ | x | = x]
n+1 n+1 Now, f ( 0 ) = 2 and f ′ ( 0 ) = 3. Therefore, from Eq. (i), we get
Case II When x ≤ 0 { f ′ ( 0 )} 2 = { f ( 0 )} 2 + C
In this case, we have | x | = − x ⇒ 9 = 4 + C ⇒ C =5
∴ ∫ |x | dx = ∫ | x |n dx
n
∴ { f ′ ( x )} 2 = { f ( x )} 2 + 5
= ∫ ( − x )n dx = − ∫ xn dx [Qn in odd] ⇒ f ′ ( x ) = 5 + { f ( x )} 2
n+1
x (−x ) x n
|x | x n
1
=− +C = +C = +C
n+1 n+1 n+1
⇒ ∫ ( 5 ) 2 + { f ( x )} 2
d { f ( x )} = ∫ dx

| x |n x
∫ |x | dx = +C
n
Hence, ⇒ log f ( x ) + 5 + { f ( x )} 2 = x + C1
n+1
72 Textbook of Integral Calculus

∴ f ( 0 ) = 2 ⇒ log | 2 + 3 | = C1 1 − cos ( x 2 + 1 )  x 2 + 1
=∫x dx = ∫ x tan   dx
⇒ C1 = log 5 1 + cos ( x + 1 )
2
 2 
∴ log | f ( x ) + 5 + { f ( x )} 2 | = x + log 5  x 2 + 1  x 2 + 1  x 2 + 1
= ∫ tan  d   = log sec   +C
 f ( x ) + 5 + { f ( x )} 2   2   2   2 
⇒ log  =x
 5  sin ( 4 x − 2 x ) dx sin ( 4 x ) dx
15. ∫ sin (2x ) cos (2x ) cos (4x ) = ∫ sin (2x ) cos (4x ) − ∫ sec 2x dx
⇒ f ( x ) + 5 + { f ( x )} 2 = 5e x
cos 2 x dx 1
⇒ 5 + { f ( x )} 2 + f ( x ) = 5e x and 5 + { f ( x )} 2 − f ( x ) = 5e − x =2 ∫ − (log | sec 2 x − tan 2 x | )
cos 4 x 2
⇒ 2 f ( x ) = 5 (e x − e − x )
cos 2 x 1
5 =2 ∫ dx − (log | sec 2 x − tan 2 x | )
⇒ f ( x ) = (e x − e − x ) (1 − 2 sin 2 x )
2
2
2
5 4 5(e 8 − 1 ) 2  1 1 + 2 sin 2 x  1
⇒ f (4) = (e − e −4 ) ⇒ f ( 4 ) = =  log  − log | sec 2 x − tan 2 x | + C
2 2e 4 2 2 2 × 1 1 − 2 sin 2 x  2
2 2 2 2
+ 4 ln x
ex − x3 ex e x ⋅ x 4 − x 3e x 1  1 + 2 sin 2 x  1
10. We have, ∫ x −1
dx = ∫
x −1
dx = log
2 2 
 − log | sec 2 x − tan 2 x | + C
1 − 2 sin 2 x  2
1  sec 2 x 7 
2
= ∫ x 3 e x dx =
2∫
tet dt, [where t = x 2] 1 − 7 cos2 x
16. ∫ sin7 x cos2 x dx = ∫  sin7 x −  dx
sin 7 x 
1 1 2
= (t − 1 ) et + C = ( x 2 − 1 ) e x + C sec 2 x 7
2 2 =∫ dx − ∫ 7 dx = I 1 + I 2
sin 7 x sin x
∫ tan x dx = ∫ ( tan 2 x sec 2x − sec 2x + 1 ) dx
4
11.
sec 2x tan x tan x ⋅ cos x
3 Now, I1 = ∫ dx = + 7∫ dx
=
tan x
− tan x + x + C sin 7 x sin 7 x sin 8 x
3 tan x
= + I2
1 sin 7 x
⇒ A = , B = − 1 and f ( x ) = x + C
3 tan x
∴ I1 + I 2 = +C ⇒ f ( x ) = tan x
sin 4 x sin 4 ( p + q ) x f {( p + q ) x } sin 2 x
12. ∫ x dx = f (x ) ⇒ ∫ (p + q )x dx = p + q sin 3 x
17. We have, ∫ dx
sin ( p + q ) x
4 (cos x + 3 cos x + 1 ) tan −1 (sec x + cos x )
4 2

∫ x
dx = f {( p + q ) x }
sin 3 x
sin θ 1 cos2 x
13. On solving, = [tan 3θ − tan θ] =∫ dx
cos 3 θ 2 (cos2 x + 3 + sec 2x ) tan −1 (sec x + tan x )
sin 3 θ 1 sin x (1 − cos2 x )
= [tan 9 θ − tan 3 θ ] =∫
1
×
cos 9 θ 2 1 + (sec x + cos x ) 2
cos2 x
sin 9 θ 1
= [tan 27 θ − tan 9 θ ] ×
1
dx
cos 27 θ 2 tan −1 (sec x + tan x )
 sin θ sin 3 θ sin 9 θ  1
∴∫ + +  d θ = ∫ ( tan 27 θ − tan θ ) dθ =∫
1
×
1
 cos3 θ cos 9 θ cos 27 θ  2 tan −1
(sec x + cos x ) 1 + (sec x + cos x ) 2
1 1  (tan x sec x − sin x ) dx
=  log (sec 127θ ) − log (sec θ )  + C
2 27  1
=∫ d | tan −1(sec x + cos x )|
1 27 sec27 θ tan −1(sec x + cos x )
= log +C
2 sec θ = loge | tan −1 (sec x + cos x ) | + C
2 sin ( x 2 + 1 ) − sin 2 ( x 2 + 1 ) π
14. We have, ∫ x dx 18. We have, f ( x ) = x | cos x |, <x<π
2 sin ( x 2 + 1 ) + sin 2 ( x 2 + 1 ) 2
⇒ f ( x ) = − x cos x [Q cos x < 0 for x ∈( π / 2, π )]
2 sin ( x 2 + 1 ) − 2 sin ( x 2 + 1 ) cos ( x 2 + 1 )
=∫x dx Hence, required primitive is given by
2 sin ( x 2 + 1 ) + 2 sin ( x 2 + 1 ) cos ( x 2 + 1 )
∫ f (x ) dx = − ∫ x cos x dx + C = − x sin x − cos x + C
Chap 01 Indefinite Integral 73

19. We have, f ( x ) = (2x + 1) |sin x |, π < x < 2 π  z+4


1  1 dz
Then, I = ∫ 4 ⋅ + 6
⇒ f ( x ) = − (2 x + 1 ) sin x 
z 9  18 z + 4
9
Hence, required primitive is given by 1 2z + 8 
=∫  + 3  dz
− ∫ (2 x + 1 ) sin x dx = − [ − (2 x + 1 ) cos x + 2 sin x ] + C z (z + 4 )  9 
I II
= (2 x + 1 ) cos x − 2 sin x + C 1 2z + 35 1 2 (z + 4 ) + 27
=
9 ∫ z (z + 4) dz = 9 ∫ z (z + 4 )
dz
0 x 2 − sin x cos x − 2
20. We have, f ( x ) = sin x − x 2
1 − 2x 2dz 3  1 1 

+ ∫ −
0 =  dz
2 − cos x 2x − 1 0 9 z 4  z z + 4
 2 3 3
sin x − x 2 2 − cos x =  +  log z − log (z + 4 ) + C
0  9 4 4
⇒ f ( x ) = x − sin x
2
0 2x − 1 35 3
cos x − 2 1 − 2x 0 = log (9e 2x − 4 ) = log (e 2x ) + C
36 4
[Interchanging rows and columns] 3 35 3
=− x+ log (9e 2x − 4 ) − log 3 + C
0 x 2 − sin x cos x − 2 2 36 2
⇒ f ( x ) = ( − 1 ) sin x − x 2 1 − 2x
∫ tan x (sec 2x − 1 ) dx = ∫ tan 3 x sec 2 x dx − ∫ tan 3x dx
3 3
0 25.
2 − cos x 2x − 1 0
tan 4 x
[Taking ( − 1 ) common from each column] I 1 = ∫ tan 3 x sec 2x dx = + C1
4
⇒ f (x ) = − f (x )
I 2 = ∫ tan x (sec 2 x − 1 ) dx
⇒ f (x ) = 0
tan 2 x
⇒ ∫ f (x ) dx = 0 =
2
− ln | sec x | + C 2

dx  1 1 
21. ∫ (x + 1) (x − 2) = ∫  − 3(x + 1) + 3(x − 2) dx 26. The Statement II is false since while writing
dx
1 1 ∫ x − 3y = log ( x − 3y ) + C ,
= − log ( x + 1 ) + log( x − 2 ) + C
3 3 we are assuming that y is a constant. We will know prove the
1 1 x
∴ A=− ,B= Statement I. From the given relation ( x − y ) 2 = and
3 3 1 y

A 2 log ( x − y ) = log x − log y .
∴ A+B=0 ⇒ = 3 = −1
B 1
dy  y  x + y
3 Also, = −  ⋅ ⋅ To prove the integral relation it is
1  1 1  dx  x  x − 3y
22. ∫  2 − 2  dx
3  x + 1 x + 4 d 1
sufficient to show that RHS = ⋅
1 1 x x dx x − 3y
⇒ tan −1 x − tan −1 = k tan −1 x + l tan −1
3 6 2 2 1 x   x
1 1 Now, RHS = log  − 1  Q (x − y )2 = 
∴ k= and l = − 2 y   y
3 6
2
2 x 1 x2 1
= [log( x − y ) − log y ]
23. I = ∫ {x log (1 + x ) dx = log (1 + x ) ⋅
2
−∫ ⋅ 2 x ⋅ dx
14243 2 1+x 2
2 2
I II x2 x3 1  log x − log y
= log (1 + x ) − ∫ 2
2  1
dx = − log y = [log x − 3 log y ]
2 x +1 2  2  4
 x 2 + 1  x 2 + 1 d 1  1 3 dy 
I =  log (1 + x 2 ) −   +C ⇒ RHS =  −
 2   2  dx 4  x y dx 
x2 + 1 1 + x2
∴ φ( x ) = , ψ (x ) = −   1 1 3  y  x + y  1
2  2   =
− −   =
−x 4 x y  
x x − 3y  x − 3y
4e + 6
2
24. I = ∫ x dx
9e − 4e − x Thus, Statement I is true. Hence, choice (c) is correct.
4e 2x + 6 27. Let g ( x ) = f ( x ) + f ( − x )
I =∫ dx, put 9e 2x − 4 = z ⇒ 18e 2xdx = dz
9e 2x − 4 Assuming, ∫ f (x ) dx = F (x ) + C
74 Textbook of Integral Calculus

∫ g (x ) dx = ∫ { f (x ) + f (− x )} dx ⇒ I =∫
u15 1
du = ∫ u14du = u15 + C
u 15
= ∫ f ( x ) dx + ∫ f ( − x ) dx
1
= ( x + 1 + x 2 )15 + C
= F (x ) + C + { − F (− x ) + C ′ } 15
= F (x ) − F (−x ) + C + C ′ 33. Here, m = −1 < 0
which may be an odd function, if C + C ′ = 0. p = −2 < 0
Similarly, integral of an odd function is not always an even Also, − x + 3 x − 2 = − ( x − 1 ) ( x − 2 )
2
function.
We can use case III
Hence, Statement I is true and Statement II is false.
28. If a > 0 and b 2 − 4ac < 0, then ⇒ Putting, − x 2 + 3 x − 2 = u ( x − 2 )

 b 4ac − b 2
2 or ( x − 1 ) u or u (1 − x )
ax 2 + bx + c = a  x +  +
 2a  4a 34. Let P = sin 3 x cos3 x
dx dx dP
⇒ ∫ ax 2 + bx + c = ∫  b  2 dx
= 3 sin 2 x cos4 x − 3 sin 4 x cos2 x
a x +  + k2
 2a  = 3 sin 2 x (1 − sin 2 x ) cos2 x − 3 sin 4 x cos2 x
4ac − b 2
= 3 sin 2 x cos2 x − 6 sin 4 x cos2 x
where k 2 = > 0. which will have an answer of the type
4a
b ∴ P = 3 I 2, 2 − 6 I 4 , 2
x+
1

1
tan −1 2a + C or µ tan −1 x + A + C ∴
1
I 4 , 2 = ( − P + 3 I 2, 2 )
a k/ a k/ a B 6
Thus, choice (a) is correct. 35. Let P = sin 5 x cos3 x
2 dP
dx 1 ∴ = 5 sin 4 x cos4 x − 3 sin 6 x cos2 x
29. I = ∫ x2
x4 + 1
⇒ I =
2 ∫ 1
dx dx
x2 + = 5 sin 4 x (1 − sin 2 x ) cos2 x − 3 sin 6 x cos2 x
x2
= 5 sin 4 x cos2 x − 8 sin 6 x cos2 x
 1  1
1 + 2  1 − 2  ∴ P = 5 I 4 , 2 − 8 I 6, 2
1  x  1  x  dx
=
2 ∫  1
2
dx −
2 ∫  1
2 1
∴ I 4 , 2 = ( P + 8 I 6, 2 )
x −  + 2 x +  − 2 5
 x  x
36. Let P = sin 5 x cos3 x
 1  1 
1 1 x −  1 1  x + − 1 dP
I = ⋅ tan −1  x  + ⋅ log  x  +C ∴ = 5 sin 4 x cos4 x − 3 sin 6 x cos2 x
2 2  2  2 2  x + 1 + 1 dx
   x  = 5 sin 4 x cos4 x − 3 sin 4 x (1 − cos2 x ) cos2 x
∴ Statement I is false. = 8 sin 4 x cos4 x − 3 sin 4 x cos2 x
π
30. Since, cot −1 x = − tan −1 x, ∴ P = 8I 4 , 4 − 3I 4 , 2
2
−1 −1 1
∴ d (cot x ) = − d (tan x ) ∴ I 4 , 2 = (− P + 8I 4 , 4 )
−1
3
tan x −1
Thus, ∫ 2 d (cot −1 x ) = − ∫ 2 tan x
d (tan −1 x )  f (x + h ) − f (x ) f (x x − h ) − f (x )
37. Here, 2 f ′ ( x ) = lim  + 
h→ 0  h −h 
−1
2 tan x
=− +C  f (x + h ) − f (x − h )
ln 2 = lim   …(i)
h→ 0  h 
∴ Statement I is false. Statement II is true.
 f (h ) − f ( 0 ) f ( − h ) − f ( 0 )
31. As m = 9 > 0, hence, we can substitute ∴ 2 f ′ ( 0 ) lim  + 
h→ 0  h −h 
9x 2 + 4x + 6 = u ± 3x f (h ) − f ( −h )
= lim …(ii)
32. Here, as per notations given, we can substitute h→ 0 h
1 + x 2 = (u − x ) Now by given relation, we have
f (x + h ) − f (x − h )
As m = 1 > 0 and p = 1 > 0 f (h ) − f ( − h ) = and f ( 0 ) = 1
−h
Chap 01 Indefinite Integral 75

f ′ (x ) π 3π
From Eqs. (i) and (ii), we have = 2010 42. (A) If <x< , then sin x > cos x
f (x ) 4 8
⇒ f ( x ) = e 2010e , f ( 0 ) = 1 sin x − cos x
∴ ∫ | sin x − cos x | dx = ∫ 1dx = x + C
∴ { f ( x )} is non-periodic.
38. Here, ∫ f ( g( x )) cos x dx = ∫ f (log (sin x )) . cos x dx x 2dx 1  1 1 
(B) ∫ = ∫ 3x 2  3 − 3  dx
(x + 1) (x + 2) 3
3 3
 x + 1 x + 2
= ∫ e 2010 log (sin x ) . cos x dx
1 x3 + 1
= ∫ (sin x ) 2010
. cos x dx = ln 3 +C
3 x +2
= ∫ (sin x ) 2010 . cos x dx π 
(C) ∫ sin −1 x cos−1 x dx = ∫ sin −1 x − (sin −1 x ) 2 dx
2011
 2 
(sin x )
= +C π
2011 ⇒ ( x sin −1 x + 1 − x 2 ) − { x (sin −1 x ) 2
2
(sin x ) 2011
∴ h( x ) = + sin −1 x 1 − x 2 − x } + C (by parts)
2011
 π 1 π  π
⇒ h  = ⇒ sin −1 x x − x sin −1 x − 2 1 − x 2 + 1 − x 2 + 2x + C
 2  2011  2  2
−1 −1
∴ f ( x ) = sin x, f ( x ) = sin x
Sol. (Q.Nos. 39 to 41)
Here, f ( x ) = ( x − 1 ) ( x + 2 ) ( x − 3 ) ( x − 6 ) − 100 dx
(D) ∫ = ln | ln | x || + C
x ln | x |
= ( x 2 − 4 x + 3 ) ( x 2 − 4 x − 12 ) − 100
∴ f ( x ) = ln | x |
= ( x 2 − 4 x ) 2 − 9 ( x 2 − 4 x ) − 136
 x 2 + cos2 x 
= ( x 2 − 4 x + 8 ) ( x 2 − 4 x − 17 ) 43. (A) I = ∫   ⋅ cosec 2 x dx
 1 + x2 
39. ∴ f ( x ) = 0 ⇒ ( x 2 − 4x + 8) ( x 2 − 4x + 17 = 0
14
42443 14
4244 3  x 2 + 1 − sin 2 x 
D>0 D<0 =∫  ⋅ cosec 2x dx
 1 + x2 
∴ Equation has two distinct and two imaginary roots.
 1 
40. f ( x ) = ( x 2 − 4x − 17) ( x 2 − 4x + 8) = ∫  cosec 2x −  dx = − cot x + cot −1 x + k
 1 + x2
= {(( x − 2 ) 2 − 21 } {( x − 2 ) 2 + 4 }
⇒ A = 1, B = − 1
Q ( f ( x )) min = ( −21 ) ( 4 ) = − 84
(B) I = ∫ x + x 2 + 2 dx
which occurs at x = 2
g( x ) g( x ) 2
x2 + 2 + x = t ⇒ x2 + 2 − x =
41. Q ∫ f (x ) = (x 2 − 4x − 17) (x 2 − 4x + 8) Put
t
2  2
Ax + B Cx + D ∴ 2x = t − ⇒ 2 dx = 1 + 2  dt
= + t  t 
x 2 − 4 x − 17 x 2 − 4 x + 8
1  2 
Clearly, A, B and C must be zero. =
2 ∫  t +  dt
t 3/ 2 
g( x ) D
∴ = 2 1 t 3/ 2 1
( x 2 − 4 x − 17 ) ( x 2 − 4 x + 8 ) x − 4x + 8 ⇒ I = ⋅ + +k
2 3 1
− t 1/ 2
∴ g( x ) = D( x 2 − 4 x − 17 ) 2 2
1 2
g( −2 ) = D( 4 + 8 − 17 ) = − 10 [given] = ( x + x 2 + 2 ) 3/ 2 − +k
3 x + x2 + 2
g( x ) 2( x − 4 x − 17 )
2
2
⇒ = = ⇒ A =1 and B = 2
f ( x ) ( x 2 − 4 x − 17 ) ( x 2 − 4 x + 8 ) x 2 − 4 x + 8
2 − x − x2
g( x ) 2 dx (C) I = ∫ dx
∴ ∫ f (x ) dx = ∫ x 2 − 4x + 8 dx = 2∫ (x − 2)2 + (2)2 x2 2 − x − x2
dx dx dx
1  x − 2 −1  x − 2  =−∫ +2∫ −∫
= 2 ⋅ tan −1   + C = tan   +C 2 − x − x2 x2 2 − x − x2 x 2 − x − x2
2  2   2 
76 Textbook of Integral Calculus

1
dt 46. u ( x ) = 7v( x ) ⇒ u′ ( x ) = 7v′ ( x )
dx dt
=−∫ + 2∫ − + ∫ t
⇒ p =7 (given)
2 1 1 1 1
9  1 2− − 2 2− − 2
− x +  u( x )  u( x ) ′
4  2 t t t t Again, =7 ⇒   =0
 1 v( x )  v(x )
 put x = 
 t ⇒ q=0
 2x + 1 1 ( 4t − 1 )dt 1 dt p+q 7+0
= − sin −1   − ∫ + ∫ = =1
 3  2 2t 2 − t − 1 2 2t 2 − t − 1
Now,
p −q 7 − 0

 2x + 1 1 2t 2 − t − 1  x − 1 dt 2
= − sin −1   − 47. Let t = ln   ⇒ =
 3  2 1  x + 1 dx x 2 − 1
2 1 1 1
dt I = ∫ t dt = t 2 + C
2 2∫
+ 2 4
t 1 1 1
t2 − − + −   x − 1 
2
2 2 16 16 1
I = ln  x + 1  + C
 2x + 1 2 − x − x2 4  
= − sin −1   −
 3  x 1
1  1 ⇒ 6A = ⇒ 24 A = 1
+ log t −  + 2t 2 − t − 1 + K 4
2 2  4
 1 1 + x 
2 − x − x2 (4 − x ) + 2 − x − x 2 48. I = ∫ e x  + dx
1  1 − x 
 (1 − x ) 1 − x
2
=− + log
x 2 2 4x
 2x + 1 d  1 + x 1
− sin −1   +K As   =
 3  dx  1 − x  (1 − x ) 1 − x 2
sin 2 x
(D) ∫ 4 r r
dx, dividing N and D by cos x 4
1/ 2
sin x + cos4 x 1+x 1 + x
∴ I = ex + C ⇒ I = ex   +C
2 tan x ⋅ sec 2x 1−x 1 − x
I =∫ dx, put tan 2 x = t
tan 4 x + 1 1
⇒ µ = 1, λ =
dt 2
⇒ 2 tan x ⋅ sec 2x dx = dt = ∫ 2 = − cot −1(t ) + C
t +1  1 3
⇒ 2(µ + λ ) = 2 1 +  = 2 × = 3
 2 2
⇒ − cot −1(tan 2 x ) + C ∴ B = − 1
(e x + cos x + 1 ) − (e x + sin x + x )
44. ∫ 2
2x + 3 49. I = ∫ dx
dx e x + sin x + x
(x + 3x ) (x 2 + 3x + 2) + 1
= ln (e x + sin x + x ) − x + C
Put x 2 + 3 x = t ⇒ (2 x + 3 ) dx = dt
dt dt 1 1 ∴ f ( x ) = e x + sin x + x and g( x ) = − x
∫ t(t + 2) + 1 = ∫ (t + 1)2 = C − t + 1 = C − x 2 + 3x + 1 ⇒ f ( x ) + g( x ) = e x + sin x
⇒ a = 1, b = 3, c = 1 ⇒ a + b + c = 5 f ( x ) + g( x )
⇒ =1
3x + 2 e x + sin x
45. F ( x ) = ∫ dx.
x −9 1 x +3 1 x −9   x + 3 x −9
Let x − 9 = t 2 ⇒dx = 2t dt
50. 12  tan −1 + ln −1
 = 3 tan  4  + ln x + 3
 4 4 2 ⋅ 6 x + 3 
 3(t 2 + 9 ) + 2  ⇒ λ = 3, µ = 1
∴ F (x ) = ∫  ⋅ 2t dt
 t  ⇒ λ+µ=4
= 2 ∫ (29 + 3t ) dt = 2[29t + t ]
2 3 15 3  5x 5x  3x
2 cos x cos x 2  4 cos3 − 3 cos  cos
2 2  2 2  2
51. =
F ( x ) = 2[29 x − 9 + ( x − 9 ) 3/2 ] + C  2 5x  2 5x
1 − 2 2 cos − 1 3 − 4 cos
Given, F (10 ) = 60 = 2[29 + 1 ] + C  2  2
⇒ C =0 = − 2 cos
5x
cos
3x
∴ F ( x ) = 2[29 x − 9 + ( x − 9 ) 3/ 2
] 2 2
= − (cos 4x + cos x )
F (13 ) = 2[29 × 2 + 4 × 2 ]
sin 4 x
= 4 × 33 = 132 I =− − sin x + C
4
Hence, sum of digits = 1 + 3 + 2 = 6
Chap 01 Indefinite Integral 77

3
sin x + x2 − 1
= x 2 ⋅ex + C = h( x ) + C
tan x cos x
52. I = ∫ dx = ∫ dx ∴ h( x ) = x ⋅ e
2 x3 + x2 − 1
1 + tan x + tan 2 x 1
+
sin x
cos2 x cos x ⇒ h(1 ) ⋅ h( − 1 ) = e1 ⋅ e −1 = 1
sin 2 x dx
=∫ dx = ∫ dx − 2 ∫  x 4 cos3 x − x sin x + cos x 
2 + sin 2 x 2 + sin 2 x 56. I = ∫ e ( x sin x + cos x )   dx
 x 2 cos2 x 
sec 2x
= x − 2∫ dx = ∫ e ( x sin x + cos x ) ⋅ ( x 2 cos x ) dx
2 sec x + 2 tan x
2

d  1 
Let t = tan x, dt = sec 2 x dx − ∫ e ( x sin x + cos x ) ⋅   dx
dx  x cos x 
2 dx dt  1 
=x− ∫ t2 + t + 1 = x − ∫ = e ( x sin x + cos x ) ⋅  x −  +C
2  1  3 2 2
 x cos x 
t +  +  
 2   2 
57. I = ∫ x+ x 2 + 2 dx
1  2 tan x + 1
I =x− tan −1   +C
3  3 
Let x + x 2 + 2 = p or x 2 + 2 = p 2 + x 2 − 2 px

∫ sin x cos x dx = ∫ sin x sin


5/ 2 3 2 1/ 2 2
53. x cos x cos x dx p2 − 2 ( p 2 + 2 ) dp
⇒ x= or dx =
= ∫ sin 2x sin1/2 x (1 − sin 2 x ) cos x dx
2p 2p 2
p1/ 2 ( p 2 + 2 ) 1 − 3/ 2
= ∫ t 2t1/2(1 − t 2 ) dt = ∫ [t 5/2 − t 9/2 ]dt I =∫ 2
dp = ∫p
1/ 2
dp + ∫p dp
2p 2
7/ 2 11/ 2
t t
= ∫ t 5/2dt − ∫ t 9/2dt = − +C =
1
(x + x 2 + 2 ) 3/ 2 − 2
1
+C
7 / 2 11 / 2 3 x+ x2 + 2
2 2 2 2
= t 7/2 − t11/2 + C = sin 7/2 x − sin11/2 x + C dx
7 11 7 11 58. I = ∫
(ax + b ) ( x − α ) 2 − β 2
1 1 
= 2 sin 7/2 x − sin 2 x + C
7 11  Put ( x − α ) = β sec θ ⇒ dx = β sec θ tan θ dθ
⇒ A = 7, B = 7, C = 11 dθ
∴ I =∫
a (α cos θ + β ) + b cos θ
⇒ ( A + B ) − C = (7 + 7 ) − 11 = 3

54. Let I = ∫ ( x 2010 + x 804 + x 402 ) (2x1608 + 5x 402 + 10)1/402dx =∫
(aα + b ) cos θ + aβ
1 dθ
= ∫ x( x 2009 + x 508 + x 401 ) ⋅ (2 x1608 + 5 x 402 + 10 )1/402dx = ∫
(aα + b )  aβ 
cos θ +  
= ∫ ( x 2009 + x 803 + x 401 ) ⋅ (2 x 2010 + 5 x 804 + 10 402 )1/402dx  aα + b 
 aβ 
402 if < 1 …(i)
Put 2 x 2010 + 5 x 804 + 10 x =t  aα + b 
 α1 
⇒ 4020( x 2009 + x 803 + x 401 ) dx = dt  cot +1 
1 2
Then, I = ⋅ cosec α 1 log  + C
t1/420 + 1 (aα + b )  cot α − 1
1
1 1 
∴ I =∫ ⋅ (t )1/402dt = ⋅
4020 4020 1 / 402 + 1  2 
t 403/402 aβ θ
=
1
⋅ where = cos α 1, tan = t
4020 403 / 402 aα + b 2
 aβ 
=
1
(2 x 2010 + 5 x 804 + 10 402 ) 403/402 Again, if  > 1
4030  aα + b 
1  α1 
∴ a − 400 = 3 I = cot α 1 tan − 1 t tan 
aα + b  2 
3
+ x2 − 1
55. Let e x (3 x 4 + 2 x 3 + 2 x ) dx aβ θ
where sec α 1 = ⇒ tan = t
= ∫ x2 ⋅ex
3
+ x2 − 1 3
+ x2 − 1 aα + b 2
⋅ (3 x 2 + 2 x ) dx + ∫ ex ⋅ (2 x ) dx
1+ 3
x
I II 59. I = ∫ dx = ∫ x − 2/3 (1 + x1/3 )1/2 dx,
3 2
Applying by parts in first internal, we get x
2 1 1
⋅ x3 + x2 − 1 x3 + x2 − 1 x3 − x2 − 1
m=− ,n = , p =
I = xe − ∫ 2x ⋅ e dx + ∫ e (2 x ) dx 3 3 2
78 Textbook of Integral Calculus

m+1 2 u − 3 
∴ = 1 ie, integer =− log  + C
n 3 u + 3 
∴ Let us make the substitution,
1 − 2/ 3 2  t2 + 3 − 3 
1 + x 1/ 3 = t 2 ∴ x dx = 2t dt =− log  + C
 t +3 + 3
3 3 2

Hence, I = 6 ∫ t 2 dt = 2t 3 + C = 2 (1 + x1/3 ) 3/2 + C


2  x2 + 1 / x2 + 1 − 3 
=− log  + C
sin (θ / 2 )
3
 x + 1/x + 1 + 3 
2 2
60. I = ∫ dθ 3
cos θ / 2 cos3 θ + cos2 θ + cos θ
2  cos θ + sec θ + 1 − 3 
1 2 sin θ / 2 ⋅ cos θ / 2 ⋅ 2 sin 2 θ / 2 =− log  + C
= ∫ 2 cos2 θ / 2 dθ 3  cos θ + sec θ + 1 + 3 
2 cos3 θ + cos2 θ + cos θ
62. Let I m − 1 = ∫ xm − 1 (a + bxn ) p dx
1 sin θ (1 − cos θ )
=
2 ∫ (1 + cos θ ) cos3 θ + cos2 θ + cos θ

and I m − n − 1 = ∫ xm − n − 1 (a + bxn ) p dx
Put cos θ = t ⇒ − sin θ dθ = dt Let p = x λ + 1 (a + bxn ) µ + 1
1 (t − 1 )
∴ I = ∫ dt where λ and µ are the smaller indices of x and (a + bxn ).
2 (t + 1 ) t 3 + t 2 + t
Here, λ = m − n − 1, µ = p
1 t2 − 1
= ∫ (t + 1)2 dt ∴ p = xm − n (a + bxn ) p + 1
2 t3 + t2 + t
Differentiating w.r.t. x, we have
1 (1 − 1 / t 2 )
=
2 1∫ 1
dt dp
dx
= xm − n ( p + 1 ) (a + bxn ) p (n bxn − 1 )
(t +
+ 2) t + 1 +
t t + (a + bxn ) p + 1 (m − n ) xm − n − 1
1 1
Put t + 1 + = u ⇒ (1 − 2 ) dt = 2udu
2
= nb ( p + 1 ) xm − 1 (a + bxn ) p
t t
1 2u du du + (m − n ) xm − n − 1 (a + bxn ) p {a + bxn}
I = ∫ 2 = tan − 1 (u ) + C
2 (u + 1 ) ⋅ u ∫ u 2 + 1
=
= nb ( p + 1 ) xm − 1 (a + bxn ) p
+ a (m − n ) xm − n − 1 (a + bxn ) p + b (m − n ) xm − 1 (a + bxn ) p
1/ 2
 1
= tan − 1 t + 1 +  + C
 t = b (np + m ) xm − 1 (a + bxn ) p + a (m − n ) xm − n − 1 (a + bxn ) p Int
−1
= tan (cos θ + sec θ + 1 )1/2 + C egrating both the sides w.r.t. x, we get
(2 sin θ + sin 2θ ) dθ p = b (np + m ) I m − 1 + a (m − n ) I m − n − 1
61. I = ∫
(cos θ − 1 ) cos θ + cos2 θ + cos3 θ ∴ xm − n (a + bxn ) p + 1 = b (np + m ) I m − 1 + a (m − n ) I m − n − 1
Put cos θ = x 2 xm − n (a + bxn ) p + 1 a (m − n )
or Im − 1 = − ⋅ Im − n − 1
⇒ − sin θ dθ = 2 x dx b (np + m ) b (np + m )
(1 + x 2 ) 2 x dx Hence Proved.
=2 ∫ ⋅
(1 − x 2 ) x +x +x
2 4 6
x8
Again, ∫ dx = ∫ x 9 − 1 (1 − x 3 ) − 1/3 dx
(1 − x 3 )1/3
(1 + 1 / x 2 ) dx
=4∫ ,
(1 / x − x ) (1 / x − x ) 2 + 3 Here, m = 9, b = − 1, n = 3, p = − 1 / 3, a = 1
1 ( x 6 (1 − x 3 ) 2/3 ) 6
Put − x =t ∴ I8 = + I5 …(i)
x −8 8
 1 
⇒  − 2 − 1 dx = dt x 3 (1 − x 3 ) 2/3 3
 x  ⇒ I5 = + I2 (here m = 6)
−5 5
dt
=−4∫ (1 − x 3 ) 2/3
t t2 + 3 ⇒ I2 =
2
Again, put t 2 + 3 = u 2 1 3 9
Hence, I 8 =− x 6(1 − x 3 ) 2/3 − x 3(1 − x 3 ) 2/3 − (1 − x 3 ) 2/3+ C
⇒ 2t dt = 2u du 8 20 40
− u du du 63. Let I = ∫ cosec 2 x ln (cos x + cos 2x ) dx
∴ I =4∫ =−4∫ 2
u (u − 3 )
2
u −3
= ∫ cosec 2 x ⋅ ln {sin x (cot x + cot 2 x − 1 )} dx
Chap 01 Indefinite Integral 79

= ∫ cosec 2 x ln (sin x ) dx 65. I = ∫


dx
x − x 2 + 2x + 4
+ ∫ cosec x ⋅ ln (cot x + cot 2 x − 1 ) dx
2

In second integral put cot x = 1 Put x 2 + 2 x + 4 = t + x

∴ cosec 2 x dx = dt ⇒ x 2 + 2 x + 4 = t 2 + x 2 + 2tx

∴ I = ∫ cosec 2 x ⋅ ln (sin x ) dx − ∫ ln(t + t 2 − 1 ) dt ⇒ 2 x − 2tx = t 2 − 4


t 2 − 4 1 (t 2 − 4 )
In first integral (integrating by parts taking cosec 2 x as second ⇒ x= =
2 − 2t 2 (1 − t )
integral) and in second integral (integration by parts taking
unity as second function). 1 t 2 − 2t + 4 
⇒ dx = −
We have, (ln sin x ) ( − cot x ) − ∫ cot x ( − cot x ) dx 2  (1 − t ) 2 
t dt 1 t 2 − 2t + 4
− ln (t + t 2 − 1 ) t + ∫
2 ∫ − t (1 − t ) 2
∴ I =− dt
t −1
2

= − cot x (ln sin x ) − cot x − x − t ln (t + t − 1 ) + t − 1 + C 2 2


1 4 3 3 
= − cot x (ln sin x ) − cot x − x − cot x {ln (cot x + cot x − 1 ) } 2
= ∫  + +
2  t (1 − t ) (1 − t ) 2 
dt

+ cot 2 x − 1 + C  1 3 
=4 log | t | − 3 log | 1 − t | + (1 − t ) 
= − cot x ⋅ ln (cos x + cos 2 x ) − cot x − x + cot 2 x − 1 + C  2 
3
dx cos2 x dx = 2 log | x + 2 x + 4 − x | − log
2
64. I = ∫ a ∈N = ∫ 2
(sin x + a sec x ) 2
(a + sin x cos x ) 2 3
| 1 − x 2 + 2x + 4 + x | +
cos2 x dx 2 (1 − x + 2 x + 4 + x )
2
=∫
(a + sin x ⋅ cos2 x + 2a sin x ⋅ cos x )
2 2
3
= 2 log | x 2 + 2 x + 4 − x | − log
4 cos2 x dx 1 + cos 2 x 2
=∫ =2 ∫ dx 3
4a + sin 2 2 x + 4a sin 2 x
2
(2a + sin 2 x ) 2 | x 2 + 2x + 4 − 1 − x | −
1 cos 2 x 2 ( x 2 + 2x + 4 − x − 1)
=2 ∫ dx + 2 ∫ dx dx
(2a + sin 2 x ) 2 (2a + sin 2 x ) 2 66. I = ∫
1 1+ x + 2x + 2
2

= 2I 1 − …(i)
(2a + sin 2 x ) x 2 + 2 x + 2 = t − x, squaring both the sides, we get
dx
I1 = ∫ , x 2 + 2 x + 2 = t 2 + x 2 − 2tx
(2a + sin 2 x ) 2
2 x + 2tx = t 2 − 2
we know
t2 − 2
∴ I1 = ∫
du x=
2 (1 + t )
4a 2 − 1 1 − u 2
( 4a 2 − 1 )
(2a − u ) t 2 + 2t + 2
⇒ dx = dt
sin 2 x + 1 1 (2a − u ) 2 (1 + t ) 2
u = 2a =
2a + sin 2 x ( 4a 2 − 1 ) 3/2 ∫ 1 − u2
du
t2 − 2 t 2 + 4t + 4
∴ 1+ x 2 + 2x + 2 = 1 + t − =
2 (1 + t ) 2 (1 + t )
du ( 4a 2 − 1 ) cos 2 x
⇒ =
(2a + sin 2 x ) 2 2 (1 + t ) (t 2 + 2t + 2 ) (t 2 + 2t + 2 )
dx ⇒ I =∫ dt = ∫ (1 + t ) (t + 2)2 dt
1 2au − 1 (t 2 + 4t + 4 ) ⋅ 2 ⋅ (1 + t ) 2
⇒ [2a sin − 1 u + 1 − u 2 ] = I 1 and sin 2 x =
( 4a 2 − 1 ) 3/2 2a − u Using partial fractions, we get
dt dt
1   2a sin 2 x + 1  I =∫ −2 ∫
∴ I = 3/ 2 
2a sin − 1   t+1 (t + 2 ) 2
( 4a − 1 ) 
2
 2a + sin 2 x 
2
= ln | t + 1 | + +C
 2a sin 2 x + 1   1 (t + 2 )
+ 1−  − +C
 2a + sin 2 x   (2a + sin 2 x ) 2
I = ln ( x + 1 + x 2 + 2x + 2 ) + +C
(x + 2) + x 2 + 2x + 2
80 Textbook of Integral Calculus

x4 + 1 (x 2 + 1)2 − 2x 2  
67. I =∫ dx = ∫ dx ∴ 1+
x  dx = dt or tdx
= dt
x6 + 1 (x 2 + 1) (x 4 − x 2 + 1)  1 + x 2  1 + x2

(1 + 1 / x 2 ) dx x 2 dx
=∫ −2 ∫ 3 2 dt t15 ( x + 1 + x 2 )15
(x + 1 / x − 1)
2 2
(x ) + 1 ∴ I = ∫ t15 ⋅ = ∫ t14 dt = +C= +C
t 15 15
(1 + 1 / x 2 ) dx x 2 dx xn + 1 xn + 1
=∫ − 2 ∫ 70. Qun + 1 = ∫ dx = ∫ dx
(x − 1 / x )2 + 1 (x 3 )2 + 1 y ax 2 + 2bx + c
In first integral put x − 1 / x = t and in second integral put
x3 = u 1 xn (2ax + 2b ) − 2bxn
=
2a ∫ ax 2 + 2bx + c
dx
dt 2 du
=∫ − ∫ u2 + 1
t2 + 1 3 1 (2ax + 2b ) dx b xn
2
=
2a ∫ xn
ax 2 + 2bx + c

a ∫ ax 2 + 2bx + c
dx
= tan − 1 (t ) − tan − 1 (u ) + C
3 1 (2ax + 2b ) b
un + 1 = ∫x −
n
2 un
= tan − 1 ( x − 1 / x ) − tan − 1 ( x 3 ) + C 2a ax + 2bx + c
2 a
3
I II
dx 1 n
68. I = ∫ un + 1 = [ x ⋅ 2 ax 2 + 2bx + c
(1 − x 3 )1/3 2a
b
Put x = 1 / t, dx = − 1 / t 2 dt − ∫ nxn − 1 ⋅ 2 (ax 2 + bx + c ) dx ] − un
a
dt dt
∴ I =−∫ =−∫ 1 n n n −1 b
t 2 (1 − 1 / t 3 )1/3 t (t 3 − 1 )1/3
=
a
x y −
a ∫x ⋅ ax 2 + 2bx + c dx −
a
un

Again, put t 3 − 1 = u 3 ⇒ 3t 2 dt = 3u 2 du xn − 1 (ax 2 + 2bx + c ) b


2
aun + 1 = xn y − n ∫ ax + 2bx + c
2
dx −
a
un
u du u du
=−∫ =−∫
(1 + u ) ⋅ u
3
(1 + u ) (1 − u + u 2 ) aun + 1 = x y − n [aun + 1 + 2bun + cun − 1 ] − bun
n

1 du 1 u+1
=
3 ∫ −
1+u 3 ∫ u −u + 1
2
du ⇒ (n + 1 ) a un + 1 + (2n + 1 ) bun + nc ⋅ un − 1 = xn y …(i)
Now, putting, n = 0 in both the sides, we get
(using partial fractions)
1 du 1 1 / 2 (2u − 1 ) + 3 / 2 au1 + bu0 = x 0y
=
3 ∫ −
u+1 3 ∫ (u 2 − u + 1 )
du
au1 = y − bu0 …(ii)
1 du 1 2u − 1 1 du Putting n = 1 in Eq. (i), we get
=
3 ∫ u + 1 − 6 ∫ u 2 − u + 1 du − 2 ∫ (u − 1 / 2)2 + 3 / 4 2au2 + 3bu1 + cu0 = xy
 y − bu0 
1 1 1 2au2 + 3b   + cu0 = xy [from Eq. (ii)]
= log | u + 1 | − log | u 2 − u + 1 | −  a 
3 6 2
1  2u − 1  ⇒ 2a 2u2 + 3by − 3b 2u0 + acu0 = axy
⋅ tan − 1  +C
3 /2  3  ⇒ 2a 2u2 = y (ax − 3b ) + (3b 2 − ac ) u0
1 1
= log | (t 3 − 1 )1/3 + 1 | − log | (t 3 − 1 ) 2/3 − (t 3 − 1 )1/3 + 1 | 71. Plan Integration by Substitution
3 6
 2 (t 3 − 1 )1/3 − 1  i.e. I = ∫ f { g( x )} ⋅ g′ ( x ) dx
1
− tan − 1  +C
3  3  Put g ( x ) = t ⇒ g′ ( x ) dx = dt
 (1 − x 3 )1/3 + x  1 ∴ I = ∫ f (t ) dt
 − log (1 − x ) − (1 − x ) + x
3 2/ 3 3 1/ 3 2
1
= log  2
3  x  6 x Description of Situation Generally, students gets confused
 2 (1 − x 3 )1/3 − x  after substitution, i.e. sec x + tan x = t.
1
− tan − 1  +C Now, for sec x, we should use
3  3x 
sec 2 x − tan 2 x = 1
( x + 1 + x 2 )15
69. I = ∫ dx ⇒ (sec x − tan x ) (sec x + tan x ) = 1
1 + x2 1
⇒ sec x − tan x =
Put (x + 1 + x 2 ) = t t
Chap 01 Indefinite Integral 81

sec 2 dx ⇒ ( − 2 x − 3 − 5 x − 6 ) dx = dt
Here, I =∫
(sec x + tan x ) 9/2
⇒ (2 x − 3 + 5 x − 6 ) dx = − dt
Put sec x + tan x = t
dt
⇒ (sec x tan x + sec 2 x ) dx = dt ∴ I =−∫ = − ∫ t − 3 dt
t3
⇒ sec x ⋅ t dx = dt
t− 3 + 1 1 x10
dt =− +C= 2 +C= +C
⇒ sec x dx = −3 + 1 2t 2(x 5 + x 3 + 1) 2
t
1 1  1 dx dx
∴ sec x − tan x =
t
⇒ sec x = t + 
2  t
74. ∫ 3
=∫ 3
x 2( x 4 + 1) 4  1 4
sec x ⋅ sec x dx x 5 1 + 4 
∴ I =∫  x 
(sec x + tan x ) 9/2
1 4
1  1 dt Put 1 + = t 4 ⇒ − 5 dx = 4 t 3dt
t +  ⋅ x4 x
2  t t 1  1 1 
⇒ I =∫ = ∫  9/2 + 13/2  dt ⇒
dx
= − t 3dt
t 9/ 2
2  t t  x5
1 2 2  1
= −  7/2 + 11/2  + K −t 3dt  1 4
2 7 t 11 t  ∴ I =∫ = − ∫ dt = − t + C = − 1 + 4  + C
t 3  x 
 1 1 
= − + 11/ 2 
+K  1 x + x+
1 1
 1 x+
1
7 (sec x + tan x ) 11 (sec x + tan x ) 
7/ 2
75. ∫ 1 + x − x  e x dx = ∫ e x dx + ∫ x 1 − x 2  e x dx
−1 1 1 2
=  + (sec x + tan x )  + K x+
1
x+
1
x+
1
(sec x + tan x )11/2 11 7  = ∫e x dx + xe x −∫
d
( x )e x dx
dx
ex e 3x
72. Since, I = ∫ dx and J = ∫ dx x+
1
x+
1
x+
1
e 4x
+ e +1
2x
1 + e 2x + e 4 x = ∫e x dx + xe x − ∫e x dx
(e − e )
3x x
 
∴ J −I =∫
1 1
dx  1 x+ x+
1 + e 2x + e 4 x Q ∫ 1 − 2  e x dx = e x

  x  
Put e x = u ⇒ e xdx = du
1 1 1
x+ x+ x+
 1 =∫ e x dx + xe x − ∫ ex x dx
1 − 2 
(u 2 − 1 )  u 
∴ J −I = ∫ du = ∫ du x+
1
1 + u2 + u4 1
1 + 2 + u2 = xe x +C
u
76. Given, ∫ f ( x ) dx = ψ( x )
 1 
1 − 2 
 u  I = ∫ x 5 f ( x 3 ) dx
=∫ 2
du Let
 1
 u +  − 1 Put x3 = t
 u
dt
1  1 ⇒ x 2dx = …(i)
Put u + = t ⇒ 1 − 2  du = dt 3
u  u 
1
dt 1 t −1 ∴ I = ∫ t f (t ) dt
=∫ = log +C 3
t −1 2
2
t+1
1  d  
u2 − u + 1 e 2x − e x + 1
= t ∫ f (t ) dt _ ∫  (t ) dt  dt 
dtt  
1
= log 2
1
+ C = log 2x +C
3 
2 u +u+1 2 e + ex + 1 [Integration by parts]
2x + 5x12
2x + 5x
9 12 9 1
73. Let I = ∫ dx = ∫ 15 dx = [t ψ (t ) − ∫ ψ (t ) dt ]
(x + x + 1)
5 3 3
x (1 + x − 2 + x − 5 ) 3 3
1
2x − 3 + 5x − 6 = [ x 3ψ ( x 3 ) – 3 ∫ x 2ψ ( x 3 ) dx ] + C [from Eq. (i)]
=∫ dx 3
(1 + x − 2 + x − 5 ) 3 1
= x 3ψ ( x 3 ) − ∫ x 2ψ ( x 3 ) dx + C
Now, put 1 + x − 2 + x − 5 = t 3
82 Textbook of Integral Calculus

5 tan x (sin x − 2 cos x ) + 2 (cos x + 2 sin x )


77. Given Integral is ∫ dx =∫ dx
tan x − 2 (sin x − 2 cos x )
To find The value of a, if sin x − 2 cos x (cos x + 2 sin x )
⇒ I =∫ dx + 2 ∫ dx
5 tan x sin x − 2 cos x (sin x − 2 cos x )
∫ tan x − 2 dx = x + a log | sin x − 2 cos x | + k…(i) d (sin x − 2 cos x )
⇒ I = ∫ 1 dx + 2 ∫
5 tan x (sin x − 2 cos x )
Now, let us assume that I = ∫ dx
tan x − 2 ⇒ I = x + 2 log | (sin x − 2 cos x ) | + k …(ii)
Multiplying by cos x in numerator and denominator, we get where, k is the constant of integration.
5 sin x Now, by comparing the value of I in Eqs. (i) and (ii), we get
I =∫ dx
sin x − 2 cos x a = 2.
sin x
This special integration requires special substitution of type 78. Let I = 2 ∫ dx
 π
 dDr  sin  x − 
N r = A( Dr ) + B    4
 dx 
π
⇒ Let 5 sin x = A (sin x − 2 cos x ) + B (cos x + 2 sin x ) Put x − =t ⇒ dx = dt
4
⇒ 0 cos x + 5 sin x = ( A + 2 B ) sin x + ( B − 2 A ) cos x π 
sin  + t dt
Comparing the coefficients of sin x and cos x, we get 4 
∴ I = 2∫
A + 2 B = 5 and B − 2 A = 0 sin t
Solving the above two equations in A and B, we get  1 1 
= 2∫ cot t + dt
 2 2 
A = 1 and B = 2
⇒ 5 sin x = (sin x − 2 cos x ) + 2 (cos x + 2 sin x ) = 1 + log | sin t | + C
5 sin x π
⇒ I =∫ dx 
= x + log sin  x −  + C
sin x − 2 cos x  4
CHAPTER

02
Definite Integral
Learning Part
Session 1
● Integration Basics

● Geometrical Interpretation of Definite Integral

● Evaluation of Definite Integrals by Substitution

Session 2
● Properties of Definite Integral

Session 3
● Applications of Piecewise Function Property

Session 4
● Applications of Even-Odd Property and Half the Integral Limit Property

Session 5
● Applications of Periodic Functions and Newton-Leibnitz’s Formula

Session 6
● Integration as Limit of a Sum

● Applications of Inequality

● Gamma Function

● Beta Function

● Walli’s Formula

Practice Part
● JEE Type Examples
● Chapter Exercise

Arihant on Your Mobile !


Exercises with the #L
symbol can be practised on your mobile. See inside cover page to activate for free.
Session 1
Integration Basics, Geometrical Interpretation
of Definite Integral, Evaluation of Definite
Integrals by Substitution

Integration Basics 1 p/ 2
4 ò0
= (1 - 2 cos 2x + cos 2 2x ) dx
What is Definite Integral ? 1 p/ 2 æ 1 + cos 4 x ö
Let f be a function of x defined in the closed interval [a, b ] = ò ç1 - 2 cos 2x + ÷ dx
4 0 è 2 ø
and f be another function, such that f¢ ( x ) = f ( x ) for all x
1 p/ 2 æ 3 - 4 cos 2x + cos 4 x ö
in the domain of f , then =
4 ò0 ç
è 2
÷ dx
ø
b
òa f ( x ) dx = [ f( x ) + c ]ba = f(b ) - f (a )
1 é 4 sin 4 x ù
p/ 2
= êë3x - 2 sin 2x + 4 úû
is called the definite integral of the function f ( x ) over the 8 0
interval [a, b ], a and b are called the limits of integration, a é æ 3p
1 1 ö ù
being the lower limit and b be the upper limit. = ê çè 2 - 2 sin p + 4 sin 2p ÷ø - 0ú
ë8 û
Remark 1 æ 3p ö 3p
= ç - 0 + 0÷ =
In definite integrals constant of integration is never present. 8è 2 ø 16

Working Rules 1 éd æ -1 1 öù
b
y Example 2 The value of ò-1 êë dx çè tan ÷ dx is
x ø úû
To evaluate definite integral ò f ( x ) dx .
a
(a) p / 2 (b) p / 4 (c) -p / 2 (d) None of these
1. First evaluate the indefinite integral ò f ( x ) dx and
1 éd æ -1 1 öù
suppose the result is g( x ). Sol. Let I = ò-1 êë dx çè tan ÷ dx
x ø úû
2. Next find g (b ) and g (a ). d æ -1 1 ö d -1
Here, ç tan ÷= (cot -1 x ) =
3. Finally, the value of the definite integral is obtained dx è x ø dx 1+ x2
by subtracting g(a ) from g(b ). 1 1 1 1
b
\ I =ò - dx = - ò dx
-1 1 + x 2 -1 1 + x 2
Thus, òa f ( x ) dx = [ g( x )]ba = g(b ) - g(a )
= -(tan -1 x )-1 = -[tan -1(1) - tan -1( -1)]
1

y Example 1 Evaluate æp pö p
= -ç + ÷ = -
1 1 p /2 è4 4ø 2
(i) ò dx (ii) ò0 sin 4 x dx
0 3 + 4x Hence, (c) is the correct answer.
1
1 1 é ln (3 + 4 x ) ù Remark
Sol. (i) Here, I = ò0 3 + 4x
dx = ê
ë 4 úû 1
0 1 æd 1ö -1 æ -1 1 ö = tan-1( 1) - tan-1( -1)
Note that ò-1 çè dx tan ÷ dx = ç tan
xø è
÷
x ø -1
1 1 æ 7 ö
= [ln 7 - ln 3] = ln ç ÷ p p p
4 4 è 3 ø = - æç - ö÷ =
4 è 4ø 2
p/ 2
(ii) Let I = ò0 sin 4 x dx
is incorrect, because tan-1 æç ö÷ is not a anti-derivative (primitive)
1
èx ø
1 p/ 2 1 p/ 2
=
4 ò0 (2 sin 2 x )2 dx =
4 ò0 (1 - cos 2x )2 dx of
d æ -1 1 ö
ç tan
dx è xø
÷ on the interval [ -1, 1].
Chap 02 Definite Integral 85

e b
y Example 3 If I n = ò (log x )n dx , then I n + nI n -1 is equal to i.e. òa f ( x ) dx = Area (OLA ) - Area ( AQM ) - Area ( MRB )
1
1 + Area ( BSCD )
(a) (b) e (c) e - 1 (d) None of these
e Remark
e e
Sol. We have, I n = ò1 (log x )n dx = ò1 (log x )n × 1 dx b
123 {
II
òa f ( x ) dx, represents algebraic sum of areas means that area of
I
function y = f ( x ) is asked between a to b.
e n -1 1
\ I n = [x × (log x )n ]e1 - ò n × (log x ) × × xdx b
e
1 x Þ Area bounded = ò | f ( x ) | dx and not been represented
a
= (e - 0) - n ò (loge x )n -1dx = e - n × I n -1 b
by ò f ( x ) dx . e.g. If someone asks for the area of y = x 3
1
\ I n + n × I n -1 = e a
Hence, (b) is the correct answer. between - 1 to 1, then y = x 3 could be plotted as

y Example 4 All the values of ‘a’ for which


2 y = x3
ò1 {a
2
+ (4 - 4a )x + 4 x 3 }dx £ 12 are given by 1

(a) a = 3 (b) a £ 4
–1
(c) 0 £ a < 3 (d) None of these O 1
2
Sol. We have, ò1 {a 2 + ( 4 - 4a )x + 4 x 3 }dx £ 12
Þ [a 2 x + (2 - 2a )x 2 + x 4 ] 12 £ 12 –1

Þ a 2 (2 - 1) + (2 - 2a )( 4 - 1) + (24 - 14 ) £ 12
Figure 2.2
Þ a 2 + 3(2 - 2a ) + 15 £ 12
Þ a 2 - 6a + 9 £ 0 0 1 1
\ Area = ò - x 3 dx + ò x 3 dx =
Þ ( a - 3) 2 £ 0 -1 0 2
\ a=3 1 1
or using above definition, area = ò | x 3 | dx = 2 ò x 3 dx
Hence, (a) is the correct answer. -1 0
1
é x4 ù 1
=2 ê ú =
Geometrical Interpretation ë 4 û0 2

of Definite Integral But, if we integrate x 3 between - 1 to 1.


1
ò-1 x
3
b Þ dx = 0 which does not represent the area.
If f ( x ) > 0 for all x Î[a, b ], then ò f ( x ) dx is numerically
a
equal to the area bounded by the curve y = f ( x ), the Thus, students are adviced to make difference between
b area and definite integral.
X-axis and the straight lines x = a and x = b i.e. ò f ( x ) dx
a
3
In general,
b
òa f ( x ) dx represents the algebraic sum of the y Example 5 Evaluate ò0 | ( x - 1) ( x - 2)| dx .
3
areas of the figures bounded by the curve y = f ( x ), the Sol. Let I = ò0 | ( x - 1) ( x - 2) | dx
X-axis and the straight lines x = a and x = b. The areas
We know,
above X-axis are taken with plus sign and the areas below
( x - 1) ( x - 2), x < 1 or x > 2
X-axis are taken with minus sign, | ( x - 1) ( x - 2) | = ìí
î - ( x - 1) ( x - 2), 1 < x < 2
x=a f (x) x=b
S + +
L C –
+ + 1 2

O A – M – B D Using number line rule,


3
Q R I = ò0 | ( x - 1) ( x - 2) | dx
Figure 2.1
86 Textbook of Integral Calculus

1 2 2 dx
= ò0 ( x - 1) ( x - 2) dx - ò1 ( x - 1) ( x - 2) dx
3
Sol. Let I = ò- 2 4 + x2
+ ò2 ( x - 1) ( x - 2) dx 2
é1 æ x ö ù 1 é tan - 1 (1) - tan -1 ( -1)ù
1 2
= ê tan - 1 ç ÷ =
ò0 ò1
2 2
= ( x - 3x + 2) dx - ( x - 3x + 2) dx è 2 ø úû êë úû
ë 2 -2
2
3
+ ò2 ( x 2 - 3x + 2) dx
1 ép æ p ö ù p
1 2 3
= ê 4 - çè - 4 ÷ø ú = 4
éx 3
3x ù éx2
3x ù éx 3x 3 ù 2 3 2 2 ë û
=ê - + 2x ú - ê - + 2x ú + ê - + 2x ú
p
ë 3 2 û0 ë 3 2 û1 ë 3 2 û2 Þ I =
4
æ1 3 ö æ 8 12 1 3 ö
= ç - + 2÷ - ç - + 4 - + - 2÷ On the other hand; if x = 1 / t , then
è3 2 ø è3 2 3 2 ø
2 dx 1/ 2 dt 1/ 2 dt
I =ò =-ò =- ò - 1/ 2
æ 27 27 8 12 ö 11 -2 4 + x 2 - 1/ 2 t 2 ( 4 + 1 / t 2 ) 4t 2 + 1
+ç - +6- + - 4÷ =
è3 2 3 2 ø 6 1/ 2
é1 ù
= - ê tan - 1 (2 t ) ú
ë2 û - 1/ 2

æ 1 ö
Evaluation of Definite 1
= - tan - 1 (1) - ç - tan - 1 ( - 1) ÷
2 è 2 ø
Integrals by Substitution p p
=- - =-
p
Sometimes, the indefinite integral may need substitution, 8 8 4
say x = f (t ). Then, in that case don’t forget to change the p 1
\ I = - , when x =
limits of integration a and b corresponding to the new 4 t
variable t. The substitution x = f (t ) is not valid, if it is not In above two results, I = - p / 4 is wrong. Since, the
1
continuous in the interval [a, b ]. integrand > 0 and therefore the definite integral of
4 + x2
y Example 6 Show that this function cannot be negative.
p /2 dx p Since, x = 1 / t is discontinuous at t = 0, then substitution is
ò0 2
a cos x + b sin x2 2 2
=
2ab
; a, b > 0. not valid. (Q I = p / 4 )

x = p/ 2 dx Remark
Sol. Let I = òx = 0 2 2
a cos x + b sin x 2 2 It is important that the substitution must be continuous in the
2 interval of integration.
x = p/ 2 sec x dx
= òx = 0 a 2 + b 2 tan 2 x 1/ 2 sin -1 x
(divide numerator and denominator by cos 2 x )
y Example 8 Evaluate ò0 x×
1- x 2
dx .

Put tan x = t Þ sec 2 x dx = dt 1/ 2 sin -1 x


t=¥ dt
Sol. Let I = ò0 x×
1- x2
dx
\ I = òt = 0 a 2 + b 2t 2 Put sin -1 x = q, then x = sinq Þ dx = cosqdq
We find the new limits of integration t = tan x Þ t = 0 1
when x = 0 and t = ¥ when x = p / 2. Also when x = 0, then q = 0 and when x = ,
2
¥
1 ¥ dt 1 1 é bt ù æ 1 ö p
Þ I = ò0 = × tan - 1 then q = sin -1 ç ÷ =
b2 æ a ö 2
2
b2 a / b êë a úû 0
è2ø 6
ç ÷ +t
è b ø p/ 6 q p/ 6
\ I = ò sin q × × cos qdq = ò q × sin qdq
0 2 0
1 é p ù p 1 - sin q
= ê -0 ú = p/ 6
ab ë 2 û 2ab = ( -q × cos q )p0 / 6 + ò cos q dq ,
0
2 dx using integeration by parts.
y Example 7 Evaluate ò directly as well as by = ( - q cos q )p0 / 6 + (sin q )0p/ 6
4+ x2 -2
p p p - 3p 1
the substitution x = 1 / t. Examine as to why the answer =- cos + 0 + sin - 0 = +
don’t valid? 6 6 6 12 2
Chap 02 Definite Integral 87

y Example 9 For any n > 1, evaluate the integral dx


Now, g ¢ ( x ) = = 1+ x4
¥ 1 dy
ò0 ( x + x 2 + 1 )n dx . x dt
When y = 0 i.e. ò2 1+t4
= 0, then x = 2
¥ 1
Sol. Let I = ò0 ( x + 1 + x 2 )n
dx Therefore, g ¢ (0) = 1 + 16 = 17
Hence, (c) is the correct answer.
Put x + 1+ x2 = t Þ 1+ x2 = t - x
p /2
t2 -1 1 æ 1ö y Example 12 Let an = ò (1 – sint )n sin 2t,
Þ 1 + x 2 = (t - x )2 Þ x= or x = çt - ÷ 0
2t 2è t ø n
1æ 1ö a
\ dx = ç1 + 2 ÷dt
2è t ø
then lim
n ®¥
å nn is equal to
n =1
¥ 1 1æ 1ö 1 ¥
\ I = òt =1 t n × 2 çè1 + t 2 ÷ødt = 2 ò1 (t -n + t -n - 2 )dt (a) 1/2 (b) 1
(c) 4/3 (d) 3/2
¥
1 é t 1-n t -n -1 ù 1é æ 1 1 öù p/ 2
= ê + ú = ê0 - çè
2 ë 1 - n -( n + 1 ) û 1 2 ë
- ÷
1 - n n + 1 ø úû
Sol. We have, an = ò0 (1 –sin t )n sin 2t dt
Let 1–sin t = u Þ –cos t dt = du
1 é -2n ù n
an = 2ò u n (1 – u )du = 2æç ò u n du – ò u n + 1 du ö÷
1 1 1
= ê 2ú
= 2 \
2 ë1 - n û n - 1 0 è 0 0 ø
æ 1 1 ö
y Example 10 The value of =2ç – ÷
2 èn + 1 n + 2ø
x + 2x - 1
x2 - 2
e -1 e 2 e an æ 1 1 ö
ò0 ( x + 1)
dx + ò x log x × e
1
2 dx is equal to Therefore,
n
=2 ç – ÷
è n ( n + 1) n ( n + 2) ø
( e2 + 1) 2
-1 2
-2 n
é æ1 1 ö 1 æ1 1 öù
(b) ( e ) e (d) ( e ) e a
(a) ( e ) (c) 0 \ lim
n ®¥
å nn =2 ê å çè n – n + 1 ÷ø – 2 å çè n – n + 2 ÷ø ú
2
x + 2x - 1 n =1 ë û
2 x2 - 2
e -1 e n
1 ö n æ1
e
æ1 1 ö
Sol. Let I = ò0 ( x + 1)
dx + ò1 x log x × e 2 dx =2å ç –
è n n + 1
÷– å ç –
ø è n n
÷
+ 2ø
n =1 n =1
Put x + 1 = t in first integral
éæ 1ö æ1 1 ö æ1 1ö ù 3 1
t2 - 2 = 2(1)– ê ç1 – ÷ + ç – ÷ + ç – ÷ + ...ú = 2 – =
e e 2 e
x2 - 2 ëè 3ø è2 4 ø è3 5ø û 2 2
\ I = ò1 t
dt + ò1 x log x ×e 2 dx Hence, (a) is the correct answer.
e
t2 - 2 æ t2 - 2 ö
y Example 13 The value of x > 1 satisfying the equation
e ì1 ü ç ÷
= ò1 e 2 í + t × log t ý dt = ç log t × e 2 ÷ x 1
2
ît þ è ø1 ò1 t lnt dt = 4 is
= ( e )e - 2
Hence, (d) is the correct answer. (a) e (b) e (c) e 2 (d) e – 1
x
x é t2 ù
dt
y Example 11 Let f ( x ) = ò and g be the
x Sol. Let I = ò1 t ln t dt = ê ln t × ú
2 û1
2 ë
1+ t 4 x
inverse of f. Then, the value of g ¢(0) is 1 x 1 2 x2 1 ét 2 ù
(a) 1 (b) 17 (c) 17 (d) None of these
– ò
2 1 t
× t dt =
2
ln x – ê ú
2 ë 2 û1

1 dy x 2 ln x 1 2 1
Sol. Here, f ¢( x ) = = = – [ x – 1] =
1+ x4 dx 2 4 4
x 2 ln x 1 2
f \ – x = 0 Þ [2 ln x – 1] = 0 (as x > 1 )
2 4
1
x f(x)=0 Þ ln x = Þ x = e
2
Hence, (a) is the correct answer.
g
88 Textbook of Integral Calculus

1 ¥ x 2 + ax + 1 æ1ö y Example 15 If the value of definite integral


ò 1+ x 4 × tan
–1
y Example 14 If lim ç ÷ dx is a
èxø
ò x × a a dx, where a > 1 and [x] denotes the
a ®¥ a 0 –[log x ]
1
p2 e–1
equal to , where k Î N , then k equals to greatest integer, is , then the value of ‘a’ equals to
k 2
(a) 4 (b) 8 (c) 16 (d) 32 (a) e (b) e (c) e + 1 (d) e – 1
¥ x 2 + ax + 1 æ1ö a
Sol. Let I = ò0 1+ x4
× tan –1 ç ÷ dx
èx ø
Sol. Let I = ò1 x × a –[loga x ] dx

Put x = 1/ t and adding, we get Put loga x = t Þ at = x


[ using tan –1(1 / x ) + cot –1 x = p / 2] \
1 1
I = ln a × ò (at × a –[t ] × at ) dt = ln a × ò (at –[t ] × at )dt
0 0
p ¥ ( x 2 + 1) + ax
ò0
1 1
I = dx = ln a × ò (a {t } × at )dt = ln a × ò a 2t dt
4 1+ x4 0 0

p é ¥ ( x 2 + 1) x dx ù
1
¥ é ln a a 2t ù 1 2
4 ë ò0 1 + x 4 ò0
= ê dx + a ú =ê ×
1+ x4 û ú = ( a – 1) [ as {t } = t , if t Î(0,1)]
ë 2 ln a û0 2
pé p ap ù é p 2 p 2a ù 1 2 e –1
= + =ê + ú \ ( a – 1) = Þ a= e
4 êë 2 2 4 úû ë 8 2 16 û 2 2
1 é p 2 p 2a ù é p2 p2 ù p2 Aliter x Î(1, a )
\ l = lim ê + ú = lim ê + ú= Þ loga x Î (0, 1) Þ [loga x ] = 0
a ®¥ a 8 2
ë 16 û a ® ¥ ë (8 2)a 16 û 16
a 1 e –1
\ I = ò x dx = (a 2 – 1) = Þ a= e
Þ k = 16 1 2 2
Hence, (c) is the correct answer. Hence, (a) is the correct answer.

Exercise for Session 1


p/ 4 p/ 2 dx
1. ò0 cos 2 x dx 2. ò0
1+ cos x
p/ 2 p/ 6
3. ò0 1+ cos x dx 4. ò0sin2x × cos x dx
2 dx 1
5. ò1 6. ò0 log x dx
x - x -1
p / 4 (sin x + cos x ) b 1
7. ò0 dx 8. òa dx , b > a
9 + 16sin2x ( x - a )(b - x )
b x -a p/ 4
9. òa dx 10. ò0 tan x dx
b -x
p p / 4 sin x æ x cos 3 x - sin x ö
11. ò0 cos 2x .log (sin x )dx 12. ò0 e ç ÷ dx
è cos 2 x ø

æ 1ö cosec q
13. If f ( x ) is a function satisfying f ç ÷ + x 2f ( x ) = 0 for all non-zero x, then ò f ( x )dx is equal to
èxø sin q

1 æ n ö æ n 1 ö
14. The value of ò çç Õ (n + r )÷÷ çå
ç
÷dx equals to
÷
0
èr =1 ø è k = 1x + k ø
(a) n (b) n ! (c) (n + 1) ! (d) n × n !
0
15. The true set of values of ‘a’ for which the inequality ò (3-2x - 2 × 3- x ) dx ³ 0 is true, is
x
(a) [0, 1] (b) [-¥, - 1] (c) [0, ¥] (d) [-¥, - 1] È [1, ¥]
Session 2
Properties of Definite Integral

Properties of Definite Integrals This property says that when integrating from 0 to a, we
b b will get the same result whether we use the function f ( x )
Property I. òa f ( x ) dx = ò
a
f (t ) dt or f (a - x ). The justification for this property will become
clear from the figures below :
i.e. The integration is independent of the change of
variable. y

Proof Let f( x ) be a primitive of f ( x ), then f(x)

d d
[ f( x )] = f ( x ) Þ [ f(t )] = f (t )
dx dt
b
Therefore, òa f ( x ) dx = [ f( x )] ba = f(b ) - f(a ) …(i)
x a–x
b x
and òa f (t ) dt = [ f(t )]ba = f(b ) - f(a ) …(ii) 0 a

As x progresses from 0 to a, the variable a - x progresses


From Eqs. (i) and (ii), we have
b b
from a to 0. Thus, whether we use x or a - x , the entire
òa f ( x ) dx = ò
a
f (t ) dt interval [0, a ] is still covered.
b a y
Property II. òa f ( x ) dx = - ò
b
f ( x ) dx f(x)

i.e. if the limits of definite integral are interchanged, then


its value changes by minus sign only.
Proof Let f( x ) be a primitive of f ( x ), then
b
òa f ( x ) dx = f(b ) - f(a ) f(a – x)
x
0
a
and - ò f ( x ) dx = - [ f(a ) - f(b )] = f(b ) - f(a ) The function f (a - x ) can be obtained from the function
b
f ( x ) by first flipping f ( x ) along the y-axis and then
b a shifting it right by a units. Notice that in the interval [0, a ],
\ òa f ( x ) dx = - ò f ( x ) dx
b f ( x ) and f (a - x ) describe precisely the same area.
a a
Property III. ò0 f ( x ) dx = ò0 f (a - x ) dx (King’s property) There are two ways to look at the justification of this
property, as described in the figures on the left and right
Proof On RHS put (a - x ) = t , so that dx = - dt respectively.
Also, when x = 0, then t = a and when x = a, then t = 0
a 0 a a y Example 16 Show that
\ ò0 f (a - x ) dx = - ò f (t ) dt = ò f (t ) dt = ò f ( x ) dx p /2 p /2
ò0 f (sin x ) dx = ò
a 0 0
(i) f (cos x ) dx
a a 0
\ ò0 f (a - x ) dx = ò f ( x ) dx p /2 p /2
0 (ii) ò0 f (tan x ) dx = ò
0
f (cot x ) dx

Remark p /2 p /2

This property is useful to evaluate a definite integral without first


(iii) ò0 f (sin 2 x )sin x dx = ò
0
2 f (cos 2 x ) × cos x dx
finding the corresponding indefinite integrals which may be p/ 2
difficult or sometimes impossible to find.
=ò f (sin 2x) × cos x dx
0
p p p
ò0 x f (sin x ) dx = ò0
a a
(iv) f (sin x ) dx
Geometrically ò0 f ( x ) dx = ò f (a - x ) dx
0 2 [IIT JEE 1982]
90 Textbook of Integral Calculus

Sol. (i) We know, y Example 17 If f and g are continuous functions


p/ 2 p/ 2 é æp öù
ò0 f (sin x ) dx = ò0 f êsin ç - x ÷ ú dx satisfying f ( x ) = f (a - x ) and g ( x ) + g (a - x ) = 2, then
ë è2 øû a a
show that ò f ( x ) g ( x ) dx = ò f ( x ) dx .
é using a f ( x ) dx = a
f (a - x ) dx ù
ëê ò0 ò0 ûú
0
a
0
a
p/ 2
Sol. Let I = ò0 f ( x ) g ( x ) dx = ò0 f (a - x ) g (a - x ) dx
= ò0 f (cos x ) dx

a
f ( x ) × [2 - g ( x )] dx
0
p/ 2 p/ 2
\ ò0 f (sin x ) dx = ò0 f (cos x ) dx é using a f ( x ) dx = a f (a - x ) dx ù
ò0 ò0
êë úû
p/ 2 p/ 2 æ æp öö
(ii) ò0 f (tan x ) dx = ò0 f ç tan
è
ç - x ÷ ÷ dx
è2 øø
Q f ( x ) = f (a - x ) and g (a - x ) + g ( x ) = 2 (given)
a a a
é using a f ( x ) dx =
ò0 ò0
a
f (a - x ) dx ù
\ ò0 f ( x ) × g ( x ) dx = 2 ò
0
f ( x ) dx - ò0 f ( x ) × g ( x ) dx
ëê ûú a a
p/ 2 or 2 ò f ( x ) × g ( x ) dx = 2 ò f ( x ) dx
= ò0 f (cot x ) dx 0
a a
0


p/ 2
f (tan x ) dx = ò0
p/ 2
f (cot x ) dx
Þ ò0 f ( x ) g ( x ) dx = ò0 f ( x ) dx
0
p/ 2
y Example 18 Evaluate
(iii) We know, I = ò0 f (sin 2x ) sin x dx …(i)
p /2 dx p /2
p/ 2 é æp öù æp ö (i) ò (ii) ò0 log (tan x ) dx
= ò0 f êsin 2 ç - x ÷ ú × sin ç - x ÷ dx
ë è 2 ø û è 2 ø
0 1 + tan x
p /4 p /2 sin x - cos x
é using a f ( x ) dx =
êë ò0
a
ò0 f (a - x ) dx ù
úû
(iii) ò0 log (1 + tan x ) dx (iv) ò0 1 + sin x cos x
dx

p/ 2
= ò0 f (sin ( p - 2x )) cos x dx
Sol. (i) Let I = ò0
p/ 2 dx
= ò0
p/ 2 cos x
dx …(i)
p/ 2 1 + tan x cos x + sin x
I = ò0 f (sin 2x ) cos x dx …(ii)
p/ 2 cos ( p / 2 - x )
Adding Eqs. (i) and (ii), we get Then, I = ò0 cos ( p / 2 - x ) + sin ( p / 2 - x )
dx
p/ 2
2I = ò0 f (sin 2x ) (sin x + cos x ) dx p/ 2 sin x
p/ 2 æ pö
= ò0 sin x + cos x
dx …(ii)
= 2 ò0 f (sin 2x ) sin ç x + ÷ dx
è 4ø Adding Eqs. (i) and (ii), we get
p æp ö æ p ö p/ 2 sin x p/ 2 cos x
Put x + = ç - q ÷ ç i.e. x = - q ÷ 2I = ò
4 è2 ø è 4 ø 0 sin x + cos x
dx + ò0 sin x + cos x
dx
- p/ 4
Þ 2I = - 2 òp/ 4 f (cos 2q ) cos q dq p/ 2 sin x + cos x p/ 2
p/ 4
= ò0 sin x + cos x
dx = ò0 1 dx
= 2 ò- p/ 4 f (cos 2q ) cos q dq
p p p
= 2 2ò
p/ 4
f (cos 2q )cos q dq (since it is even) = [ x ] p0 / 2 = - 0 Þ 2I = Þ I =
0 2 2 4
p/ 2
\ I = 2 ò0
p/ 4
f (cos 2q ) cos q dq (ii) Let I = ò0 log (tan x ) dx …(i)

p p/ 2 ì æp öü
(iv) Let I = ò0 x f (sin x ) dx …(i) Then, I = ò0 log í tan ç - x ÷ý dx
î è 2 øþ
Replacing x by ( p - x ), we get p/ 2
p
Þ I = ò0 log (cot x ) dx …(ii)
I = ò0 ( p - x ) f (sin ( p - x )) dx
Adding Eqs. (i) and (ii), we get
p p/ 2 p/ 2
Þ I = ò0 ( p - x ) f (sin x ) dx …(ii) 2I = ò0 log (tan x ) dx + ò0 log (cot x ) dx
Adding Eqs. (i) and (ii), we get p/ 2
=ò ( log tan x + log cot x ) dx
p p p 0
2I = ò0 p f (sin x ) dx Þ I = ò0 f (sin x ) dx p/ 2 p/ 2

p p p
2 = ò0 log (tan x × cot x ) dx = ò0 log (1) dx
\ ò0 x f (sin x ) dx =
2 ò0 f (sin x ) dx Þ 2I = 0 Þ I = 0
Chap 02 Definite Integral 91

p/ 4 b b
(iii) Let I = ò0 log (1 + tan x ) dx …(i) Property IV. òa f ( x ) dx = ò
f (a + b - x ) dx
a
p/ 4 (King’s property)
= ò0 log [1 + tan ( p / 4 - x )] dx Proof Put x = a + b - t Þ dx = - dt
p/ 4 æ tan p / 4 - tan x ö Also, when x = a, then t = b, and when x = b
= ò0 log ç1 +
è
÷ dx
1 + tan ( p / 4 ) × tan x ø b a a
\ò f ( x ) dx = ò f (a + b - t ) ( - dt ) = - ò f (a + b - t ) dt
p/ 4 æ 1 + tan x + 1 - tan x ö a b b
= ò0 log ç
è 1 + tan x
÷ dx
ø b b
=ò f (a + b - t ) dt = ò f (a + b - x ) dx
p/ 4 æ 2 ö a a
= ò0 log ç ÷ dx
è 1 + tan x ø b b
\ò f ( x ) dx = ò f (a + b - x ) dx
p/ 4 p/ 4 a a
= ò0 log (2) dx - ò0 log (1 + tan x ) dx a ab

Þ I = (log 2) ( x )p0/ 4 - I [using Eq. (i)]


Geometrically ò0 f ( x )dx = ò f (a + b - x ) dx
y
p p
Þ 2I = log 2 Þ I = log 2
4 8
p/ 2 sin x - cos x
(iv) Let I =ò dx …(i)
0 1 + sin x cos x
æ p ö æp ö
sin ç - x ÷ - cos ç - x ÷ x a+b–x
p/ 2 è 2 ø è2 ø
Then, I = ò0 æ p ö æ p ö
dx
a b
x
1 + sin ç - x ÷ cos ç - x ÷
è2 ø è2 ø
p/ 2 cos x - sin x As the variable x varies from a to b, the variable a + b - x
Þ I = ò0 1 + cos x × sin x
dx …(ii)
varies from b to a. Thus, whether we use x or a + b - x ,
the entire interval [a, b ] is covered in both the cases and
Adding Eqs. (i) and (ii), we get
the areas will be the same.
y
sin x - cos x
p/ 2 p/ 2 cos x - sin x
2I = ò0 1 + sin x cos x dx + ò0 1 + sin x cos x
dx Þ

2I = 0 Þ I = 0
a
y Example 19 The value of ò0 log (cot a + tan x ) dx ,
x a+b–x
where a Î(0, p / 2) is equal to x
(a) a log (sin a) (b) - a log (sin a) a b

(c) - a log (cos a) (d) None of these


a The graph of f (a + b - x ) can be obtained from the graph
Sol. Let I = ò0 log (cot a + tan x ) dx
of f ( x ) by first flipping the graph of f ( x ) along the y-axis
a æ cos a sin x ö and then shifting it (a + b ) units towards the right; the
= ò0 log ç + ÷ dx
è sin a cos x ø areas described by f ( x ) and f (a + b - x ) in the interval
a æ cos (a - x ) ö [a, b ] are precisely the same.
= ò0 log ç
è sin a cos x ø
÷ dx
p /3 dx
a a
y Example 20 Evaluate òp / 6 1 + tan x
×
= ò0 log [cos (a - x )] dx - ò0 log (sin a ) dx
a
- ò0 log (cos x ) dx
Sol. Let I =
p/ 3
òp/ 6
dx
=
p/ 3
òp/ 6
cos x × dx
... (i)
a a a 1 + tan x cos x + sin x
= ò0 log cos( x )dx - ò log(sin a )dx - ò log (cos x ) dx
0 0 p/ 3 cos ( p / 2 - x )
é using f ( x ) dx = a a
f (a - x ) dx to first integral ù
then, I = òp/ 6 dx
êë ò0 ò0 úû
cos ( p / 2 - x ) + sin ( p / 2 - x )
a (Qa + b = p / 2)
= - log (sin a ) ò dx = - a log (sin a ) p/ 3 sin x
0
Hence, (b) is the correct answer.
Þ I = òp/ 6 sin x + cos x
dx …(ii)
92 Textbook of Integral Calculus

Adding Eqs. (i) and (ii), we get Integrating both sides, we get
p/ 3 p p p p ln f ( x ) = – f (–x ) + C
2I = ò 1 dx = - = Þ I =
p/ 6 3 6 6 12 ln[ f ( x )× f (–x )] = C
y Example 21 Prove that f ( x ) × f (–x ) = C
But f ( 0) = 3
b f (x ) b- a
òa f ( x ) + f (a + b - x ) dx = 2 × Þ f 2 ( 0) = C \ C = 9
\ f ( x ) × f (–x ) = 9
b f (x )
Sol. Let I = òa f ( x ) + f (a + b - x )
dx …(i) Aliter f ( x ) × f ¢ ( x )– f (–x ) × f ¢ ( x ) = 0
d
b f (a + b - x ) Þ [ f (–x ) × f ( x )] = 0
then, I = òa f (a + b - x ) + f (a + b - (a + b - x ))
dx dx
Integrating both sides, we get f ( x ) × f (–x ) = Constant
b f (a + b - x ) Hence, (b) is the correct answer.
Þ I = òa f (a + b - x ) + f ( x )
dx …(ii)
51 dx
Adding Eqs. (i) and (ii), we get y Example 24 ò–51 3 + f ( x ) has the value equal to
b f (a + b - x ) + f ( x )
2I = ò dx (a) 17 (b) 34
a f (a + b - x ) + f ( x )
(c) 102 (d) 0
b b-a
Þ 2I = òa 1 dx = (b - a ) Þ I =
2 Sol. Let I =
51 dx 51 dx
ò–51 3 + f ( x ) = ò–51 3 + f (–x )
y Example 22 Solve é using b f ( x )dx = b f (a + b – x )dx ù
- sin t 4 f (z ) dz ëê òa òa ûú
I=ò ×
cos 4 t f (cos 2 t - z ) + f (z ) 51 6 + f ( x ) + f (–x )
2I = ò–51 [3 + f ( x )] [3 + f (–x )]dx
- sin 4 t f (z ) dz
Sol. We have, I = òcos 4
f (cos 2 t - z ) +
...(i) 51 6 + f (x ) + f (-x )
= ò-51
t f (z ) dx
9 + 3[ f ( x ) + f ( - x )] + f ( x ) × f ( - x )
- sin 4 t f (cos 2t - z ) dz
\ I = òcos 4
t f (cos 2 t - z ) + f (z )
..(ii)
=
51 6 + f (x ) + f (-x )
ò-51 18 + 3 [ f ( x ) + f ( - x )] dx
é using b f ( x ) dx = b
f (a + b - x ) dx ù
êë òa òa úû =
1 51
ò dx =
2 × 51
3 – 51 3
Adding Eqs. (i) and (ii), we get
51
- sin 4 t 4 Þ I = = 17
2I = òcos 4 dz Þ 2I = (z )- sin4 t
3
t cos t

1 1 Hence, (a) is the correct answer.


\ I =- (sin 4 t + cos 4 t ) = - (1 - 2 sin 2 t cos 2 t )
2 2
y Example 25 Number of roots of f ( x ) = 0 in [ -2, 2] is
1æ 1 ö 1 1
= - ç1 - sin 2 2t ÷ = - + sin 2 2t (a) 0 (b) 1
2è 2 ø 2 4
(c) 2 (d) 4
Directions (Ex 23-25) Let the function f satisfies
Sol. Let x = a be the root of f ( x ) = 0.
f ( x ) × f ¢ (– x ) = f (– x ) × f ¢ ( x ) for all x and f ( 0) = 3.
\ f (a ) = 0
y Example 23 The value of f ( x ) × f (– x ) for all x is f ( x ) × f (–x ) = 9
(a) 4 (b) 9 Put x = a, then 0=9 (impossible)
(c) 12 (d) 16 Therefore, f ( x ) has no root but f (0) = 3.
Sol. Given, f ( x ) × f ¢ (–x ) = f (–x ) × f ¢ ( x ) \ f ( x ) > 0, " x Î R as f is continuous possible function
f ( x ) = 3e –x .
f ¢ ( x ) f ¢ (–x )
Þ = Hence, (a) is the correct answer.
f (x ) f (–x )
Chap 02 Definite Integral 93

Exercise for Session 2


p/ 4
1. The value of ò log (1 + tan q) dq is equal to
0
p p p
(a) log 2 (b) - log 2 (c) log 2 (d) None of these
2 4 8
p 2
2. For any integer n, the value of ò e cos x
× cos 3 (2n + 1) x × dx is equal to
0
(a) 0 (b) 1 (c) –1 (d) None of these
3 x
3. The value of ò dx is equal to
2 5-x + x
(a) 1/2 (b) 1/3 (c) 1/4 (d) None of these
2 dx
4. The value of ò is equal to
0 (17 + 8x - 4x 2 ) (e 6 ( 1 - x ) + 1)
1 ½2- 21 ½ 1 ½ 2 + 21½
(a) - log½ ½ (b) - log½ ½
8 21 ½ 2 + 21½ 8 21 ½ 21 - 2 ½
1 ïì ½ 2 - 21 ½ ½ 2 + 21½ ïü
(c) - ílog½ ½ - log½ ½ý (d) None of these
8 21 ïî ½ 2 + 21½ ½ 21 - 2 ½ ïþ
a f (sin x )
5. If f is an odd function, then the value of ò dx is equal to
-a f (cos x ) + f (sin2 x )
(a) 0 (b) f (cos x ) + f (sin x ) (c) 1 (d) None of these
2
10 [ x ] dx
6. If [ x ] stands for the greatest integar function, then ò is
4 [ x 2 - 28x + 196] + [ x 2 ]
(a) 1 (b) 2 (c) 3 (d) 4
xdx p
7. The value of ò (0 < a < p ) is
0 1 + cos a × sin a

p pa a sin a
(a) (b) (c) (d)
sin a sin a sin a a

8. If f , g, h be continuous functions on [0, a ] such that f (a - x ) = f ( x ), g(a - x ) = - g( x ) and 3h( x ) = 5 + 4h(a - x ),


a
then the value of ò f ( x ) × g( x ) × h( x )dx is
0
(a) 0 (b) 1 (c) a (d) 2a
1 æ 1ö e
9. If 2f ( x ) + f ( - x ) = sin ç x - ÷ , then the value of ò f ( x )dx is
x è xø 1/ e

(a) 0 (b) e (c) 1/ e (d) e + 1/ e


p p p
10. Prove that ò xf (sin x )dx = ò f (sin x )dx .
0 2 0
æp ö
x 2 sin 2x × sin ç cos x ÷ dx
p è2 ø
11. Evaluate ò .
0 (2x - p )
1 1 1
12. Number of positive continuous function f ( x ) defined on [0, 1] for which ò f ( x )dx = 1, ò xf ( x )dx = 2 andò x 2f ( x )dx = 4.
0 0 0
1 ex 1 x2 I
13. Let I1 = ò dx and I 2 = ò dx . Then 1 is equal to
0 1+ x 0 x 3
3 I
e (2 - x ) 2
3 3 1
(a) (b) (c) 3e (d)
e e 3e
ex f( x ) f( a ) I
14. If f ( x ) = ,I = ò
x 1
x × g {x (1- x )}dx and I 2 = ò g {x (1- x )}dx , then the value of 2 is
1+ e f ( - a ) f ( -a ) I1
(a) 1 (b) -3 (c) -1 (d) 2
Session 3
Applications of Piecewise Function Property

Applications of Piecewise Proof Let us consider the function f ( x ) on [a, b ] when


f (a + x ) = f (b - x ), then f is even symmetric about the
Function Property mid-point x =
a +b
on the interval [a, b ] when
b c b
2
Property V (a). òa f ( x ) dx = ò f ( x ) dx + ò
a c
f ( x ) dx ,
f (a + x ) = - f (b - x ), then f is odd symmetric about the
where c « R
a +b
Proof Let f( x ) be primitive of f ( x ), then mid-point x = of the interval [a, b ].
2
b
òa f ( x ) dx = f(b ) - f(a ) …(i)
\ Using ò
a
f ( x ) dx = ìí
0, if f is an odd function
-a î 2 f ( x )dx , if f is an even function
c b
and òa f ( x )dx + ò f ( x )dx = [ f(c ) - f(a )] + [ f(b ) - f(c )]
c ì 0, if f (a + x ) = - f (b - x )
b ï
= f(b ) - f(a ) …(ii) Þ òa f ( x )dx = í a +b

From Eqs. (i) and (ii), we get ïî 2 òa 2 f ( x ) dx , if f (a + x ) = f (b - x )


b c b
òa f ( x ) dx = ò f ( x ) dx + ò
a c
f ( x ) dx ì x 2 , for 0 £ x < 1
ï
y Example 26 Given function, f ( x ) = í ×
Generalisation Property V(a) can be generalised into the
ïî x , for 1 £ x £ 2
following form 2
b c1 c2 b Evaluate ò f ( x ) dx .
òa f ( x )dx = ò
a
f ( x )dx + ò
c1
f ( x ) dx +¼ + ò f ( x )dx
cn
0
2 1 2

where, a < c 1 < c 2 <¼ < c n - 1 < c n < b


Sol. Here, ò0 f ( x ) dx = ò0 f ( x ) dx + ò1 f ( x ) dx

Property V (b). 2 1 2

a a /2 a /2
\ ò0 f ( x ) dx = ò0 x 2 dx + ò1 x dx
ò0 f ( x ) dx = ò
0
f ( x ) dx + ò
0
f (a - x ) dx
éx3 ù
1
é x 3/2 ù
2

Proof As we know, =ê ú +ê ú
a a /2 a ë 3 û 0 ë 3 / 2 û1
ò0 f ( x ) dx = ò
0
f ( x ) dx + ò
a /2
f ( x ) dx
é x3 ù
1
é 2 ù
2
=ê ú +ê x x ú
Put x = a - t Þ dx = - dt in the second integral also, ë 3 û ë 3 û1
0
when x = a / 2, then t = a / 2 and when x = a, then t = 0.
a a /2 0 é 1 ù 2
= ê - 0 ú + [2 2 - 1]
\ ò0 f ( x ) dx = ò
0
f ( x ) dx + ò
a /2
f (a - t ) ( - dt )
ë 3 û 3
a /2 a /2
=ò f ( x ) dx + ò f (a - t ) dt =
1 4 2 2 4 2 1 1
+ - = - = ( 4 2 - 1)
0 0
3 3 3 3 3 3
a a /2 a /2
ò0 f ( x ) dx = ò
0
f ( x ) dx + ò
0
f (a - x ) dx
y Example 27 Evaluate the integral I = ò | 1 - x | dx .
2
0
Property V (c).
b - a, if a < b
ì 0, if f (a + x ) = - f (b - x ) Sol. By definition, | a - b | = ìí
b ï î a - b, if a > b
òa f ( x ) dx = í a +b
| 1 - x | = ìí
(1 - x ), 0 £ x £ 1
ïî 2 òa 2 f ( x ) dx , if f (a + x ) = f (b - x ) Then,
î ( x - 1), 1 £ x £ 2
Chap 02 Definite Integral 95

2 1 2 2
\ ò0 | 1 - x | dx = ò0 (1 - x ) dx + ò1 ( x - 1) dx y Example 30 Evaluate ò0 {x } dx, where {x } denotes
1 2
é x2 ù éx2 ù the fractional part of x.
= êx - ú +ê - xú 2 2 2 2
ë 2 û0 ë 2 û1 Sol. ò { x } dx = ò0 ( x - [ x ]) dx = ò0 x dx - ò0 [ x ] dx
0
2
ìæ 1 ö ü ìæ 4 ö æ 1 öü æx2 ö
= ç ÷ - æç ò [ x ] dx + [ x ] dx ö÷
1 2
= í ç1 - ÷ - (0 - 0)ý + í ç - 2÷ - ç - 1÷ý è ò1 ø
îè 2 ø þ îè 2 ø è 2 øþ è ø0
2 0

1
( 4 - 0) - æç ò 0 dx + 1 dx ö÷
1 2
1 ì 1ü
= + í0 + ý = 1
2 î 2þ
=
2 è 0 ò1 ø
= 2 - ( x )12 = 2 - (1) = 1
y Example 28 Evaluate
p 2 Remark
(i) ò0 |cos x | dx (ii) ò0 | x 2 + 2 x - 3| dx In above example, for greatest integer less than or equal to x, it is
compulsory to break it at integral limits.
p p/ 2 p
Sol. (i) ò0 | cos x | dx = ò0 | cos x | dx + òp/ 2 | cos x | dx 9
p/ 2 p
y Example 31 Evaluate ò0 { x } dx , where {x} denotes
= ò0 (cos x ) dx - òp/ 2 (cos x ) dx
the fractional part of x.
= [sin x ] 0p/ 2 - [sin x ] pp/ 2 9 9 æ ö
Sol. ò { x } dx = ò0 ç x - [ x ]÷ dx
0 è ø
= ( 1 - 0) - ( 0 - 1) = 2
9 9
(ii) ò0
2 2
| x + 2x - 3 | dx
= ò0 ( x 1/ 2 ) dx - ò0 [ x ] dx
2 3/2 9 9
1 2
= ( x ) 0 - ò [ x ] dx
= ò0 | x 2 + 2x - 3 | dx + ò1 | x 2 + 2x - 3 | dx …(i)
3 0
2 2
We have, x + 2x - 3 = ( x + 3) ( x - 1) 9
= [27 ] - ò [ x ] dx
3 0
\ x 2 + 2x - 3 > 0 for x < - 3 or x > 1
9
and x 2 + 2x - 3 < 0 for -3< x <1 As ò0 [ x ] dx Þ 0 £ x £ 9 and 0 £ x £ 3

So, | x 2 + 2x - 3 | Thus, it should be divided into three parts


ì ( x 2 + 2x - 3), for x < - 3 or x > 1 0 £ x £ 1, 1 £ x £ 2, 2 £ x £ 3
=í 2 9
- + - - < x <1
î ( x 2 x 3 ), for 3 i.e. I = 2 (9 ) - ò0 [ x ] dx
\ Eq. (i) becomes
= 18 - é ò [ x ] dx + [ x ] dx ù
1 4 9
1 2
ëê 0 ò1 [ x ] dx + ò4 ûú
ò0 ò1
2 2
I = - ( x + 2x - 3) dx + ( x + 2x - 3) dx
= 18 - æç 2 dx ö÷
1 4 9
é x3 ù
1
é x3 ù
2
è ò0 0 dx + ò1 1 dx + ò4 ø
=-ê + x 2 - 3x ú +ê + x 2 - 3x ú
ë 3 û 0 ë 3 û1 = 18 - æç0 + ( x )41 + (2x )94 ö÷ = 18 - (3 + 10) = 5
è ø
=4
1 y Example 32 If for a real number y, [ y ] is the
y Example 29 Evaluate ò- 1 ( x - [ x ]) dx , where [.] greatest integer less than or equal to y , then find the
denotes the greatest integral part of x . 3p / 2
value of the integral ò [2 sin x ] dx.
1 1 1 p /2
Sol. Let I = ò- 1 ( x - [x ]) dx = ò- 1 x × dx - ò- 1 [x ] dx Sol. We know, - 1 £ sin x £ 1 as x Î[ p / 2, 3p / 2]
1
æx2 ö Þ - 2 £ 2 sin x £ 2
= ç ÷ - æç ò [ x ] dx + [ x ] dx ö÷
0 1

è 2 ø- 1 è -1 ò0 ø \ 2 sin x must be divided (or broken) at x = 5p / 6, p, 7 p / 6.


As 2 sin x = + 1, 0, - 1 at these points.
1
(1 - 1) - æç ò - 1 dx + 0 dx ö÷
0 1
=
2 è -1 ò0 ø \ò
3 p/ 2
p/ 2
[2 sin x ] dx =
5 p/ 6
òp/ 2 [2 sin x ] dx +
p
ò5 p/ 6 [2 sin x ] dx

= 0 + ( x ) 1- 1 - 0 = 1 7 p/ 6 3 p/ 2
+ òp [2 sin x ] dx + ò7 p/ 6 [2 sin x ] dx
96 Textbook of Integral Calculus

5 p/ 6 p 7 p/ 6 3 p/ 2 Y
= òp/ 2 1 dx + ò5 p/ 6 0 dx + òp ( - 1) dx + ò7 p/ 6 ( - 2) dx

æ 5p p ö æ7p ö æ 3p 7 p ö p 1 {–x} {x}


=ç - ÷ +0- ç - p÷ - 2 ç - ÷=-
è 6 2ø è 6 ø è 2 6 ø 2
100 X′ X
y Example 33 The value of ò0 [tan - 1 x ] dx is equal –2 –1 O 1 2

to (where [.] denotes the greatest integer function)


(a) tan 1 - 100 (b) p / 2 - tan 1
Y′
(c) 100 - tan 1 (d) None of these
2
100 From the above graph, we need ò min ({ x }, {- x } ) shown as
Sol. Let I = ò0 [tan - 1 x ] dx -2

where [tan - 1 x ] is shown as


2 1/2

π/2
1 –2 –1 O 1 2

–tan 1 tan 1

–1 2 1 1 1
–π/2 \ ò- 2 f ( x ) dx = 4 ò0 f ( x ) dx = 4 ´
2
´1´ =1
2
–2
Hence, (b) is the correct answer.
2 2
y Example 35 The value of ò ( x [ x ]
+ [ x 2 ] x ) dx is
1
1
equal to where [.] denotes the greatest integer function
–tan 1 O tan 1 5 3 2 1 3
(a) + 3 + (2 -2 )+ (9 - 3 )
–1 4 log 3
–2 5 2 1 3 2 1 3
(b) + 3+ + (2 -2 )+ (9 - 3 )
4 3 log 2 log 3
100
\ò [tan - 1 x ] dx is shown as
0 5 2 1 3 2 1 3
(c) + + (2 -2 )+ (9 - 3 )
Y 4 3 log 2 log 3

1
(d) None of the above
2 2
ò1 ( x[x ]
Sol. Let I = + [ x 2 ]x ) dx
O tan 1 100 2 3 2
= ò1 ( x + 1) dx + ò2 ( x 2 + 2x ) dx + ò3 ( x 3 + 3x ) dx

2 3 2
éx2 ù éx3 2x ù éx4 3x ù
= 1 ´ (100 - tan 1) =ê + xú +ê + ú +ê + ú
ë 2 û1 ë 3 log 2 û 2 ë 4 log 3 û 3
Hence, (c) is the correct answer.
5 2 1 3 2 1
y Example 34 The value of = + 3+ + (2 -2 )+ ( 32 - 3 3
)
2 4 3 log 2 log 3
ò min { x - [ x ], - x - [ - x ]} dx is equal to (where [.]
-2
Hence, (b) is the correct answer.
denotes the greatest integer function)
2p
(a) 1/2 (b) 1 y Example 36 The value of ò0 [| sin x | + | cos x |] dx is
(c) 3/2 (d) 2
equal to
Sol. Let f ( x ) = min ( x - [ x ], - x - [ - x ]) = min ({ x }, { - x }) p 3p
(a) (b) p (c) (d) 2p
Graphically, { x } and { -x } could be plotted as; 2 2
Chap 02 Definite Integral 97

Sol. Let f ( x ) = [| sin x | + | cos x | ] \ f (t ) = e t – 1, for t > 0 (corresponding to x > 1)


2 2
As, | sin x | ³ sin x and | cos x | ³ cos x Therefore, f ( x ) = e x – 1, for x > 0 ...(i)
\ | sin x | + | cos x | ³ 1 Again, for 0 < x £ 1,
and | sin x | + | cos x | £ 1 + 1 2 2
f ¢ (ln x ) = 1 (Q x = e t )
Þ 1 £ | sin x | + | cos x | £ 2 f ¢ (t ) = 1, for t £ 0
f (t ) = t + C
Thus, [| sin x | + | cos x | ] = 1
2p 2p
f (0) = 0 + C Þ C = 0 Þ f (t ) = t , for t £ 0
\ ò0 [| sin x | + | cos x | ] dx = ò0 1 dx = 2p Þ f ( x ) = x , for x £ 0
Hence, (d) is the correct answer. Hence, (d) is the correct answer.

y Example 37 The value of the definite integral y Example 39 The integral


p/2 5p / 4
ò sin |2x – a | dx , where a Î[0, p ] , is
0
ò (| cos t | sin t + | sin t | cost ) dt has the value
p /4

1 + cosa 1 – cosa
equal to
(a) 1 (b) cosa (c) (d) (a) 0 (b) 1/2
2 2
(p - a) dt
(c) 1/ 2 (d) 1
Sol. Let I = ò- a sin| t | dt , where 2x –a = t Þ dx =
2 Sol. Let
p/ 2 p
1 0 1 p- a I = ò 2 sin t cos t dt + ò [(–sin t cos t ) + (sin t cos t )]dt
=
2 ò- a –sin t dt + 2 ò0 sin t dt p/ 4 1/44444
p 2 42444444 3
zero
p–a 5 p/ 4
òp
0
é1 ù é1 ù + (–2 sin t cos t ) dt
= ê cos t ú - ê cos t ú p/ 2 5 p/ 4
ë 2 û –a ë 2 û0 = òp/ 4 sin 2t dt – ò
p
sin 2t dt
1 1
= [1 – cos a ]– [–cos a – 1] These two integrals cancels
2 2
Þ Zero.
1 1
= (1 – cos a ) + (1 + cos a ) = 1 Hence, (a) is the correct answer.
2 2
1 1 2
= (1 – cos a ) + (1 + cos a ) = 1
2 2
y Example 40 The value of ò0 f ( x ) dx , where
Hence, (a) is the correct answer. ì n
ï0 , when x = , n = 1, 2, 3 ...
f (x ) = í n+1 3 is equal to
y Example 38 Let f be a continuous functions
ïî1 , elsewhere
satisfying
ì 1 for 0 < x £ 1 (a) 1 (b) 2
f ¢(ln x ) = í x (c) 3 (d) None of these
îe – 1 for x >1
2 1/ 2 2/3 3/4
and f (0) = 0, then f ( x ) can be defined as Sol. Here, ò0 f ( x ) dx = ò0 1 dx + ò1/ 2 1 dx + ò2 / 3 1 dx + ¼
ì 1 , if x £ 0 ì 1 , if x £ 0 n
(a) f (x) = í (b) f (x) = í x 2

î 1 – e x
, if x > 0 îe – 1, if x > 0
+ òn +1
n -1
1 dx +¼+ ò1 1 dx
n
ì x , if x < 0 ì x , if x £ 0 æ1ö æ2 1ö æ 3 2ö æ n n - 1ö
(c) f (x) = í x (d) f (x) = í x = ç ÷ + ç - ÷ + ç - ÷ +¼+ ç -
îe , if x > 0 îe – 1, if x > 0 ÷ +¼+1
è2ø è3 2ø è 4 3ø èn + 1 n ø
1, for 0 < x £ 1
Sol. f ¢ (ln x ) = ìí =
n
+ ... + 1 , as n ® ¥
î , for x > 1
x n +1
Put ln x = t Þ x = e t We take, limit n ® ¥
t 2
For x > 1; f ¢ (t ) = e for t > 0 We have, ò0 f ( x ) dx = 1 + 1 = 2
t 0
Integrating f (t ) = e + C ; f (0) = e + C Þ C = –1 Hence, (b) is the correct answer.
[ given, f (0) = 0]
98 Textbook of Integral Calculus

Exercise for Session 3


3
1. The value of ò {| x - 2 | + [ x ]} dx , where [.] denotes the greatest integer function, is equal to
-1

(a) 5 (b) 6 (c) 7 (d) None of these


3
2. The value of ò (| x | + | x - 1|) dx is equal to
-1

(a) 9 (b) 6 (c) 3 (d) None of these


1
3. Let f ( x ) = x - [ x ], for every real number x (where, [ x ] is integral part of x). Then, the value of ò f ( x ) dx is equal
-1
to
(a) 0 (b) 1 (c) 2 (d) None of these
2
4. The value of ò [ x + [ x + [ x ]]] dx (where, [.] denotes the greatest integer function) is equal to
0
(a) 2 (b) 3 (c) –3 (d) None of these
x
[x ] 2
5. The value of ò dx is equal to (where, [.] denotes the greatest integer function)
0 2[ x ]
[x ] [x ] [x ]
(a) (b) (c) (d) None of these
log 2 2 log 2 4 log 2
4
6. The value of ò {x} dx (where, {.} denotes fractional part of x) is equal to
0
4 5 7
(a) (b) (c) (d) None of these
3 3 3
4
7. The value of ò {x}[ x ] dx (where, [.] and {.} denote the greatest integer and fractional part of x) is equal to
1
11 13
(a) (b)
12 12
7 19
(c) (d)
12 12
x
8. The value of ò [t + 1]3 dt (where, [.] denotes the greatest integer function of x) is equal to
0
2 3
[x ] ([x ] + 1) ö [x ] ([x ] + 1) ö
(a) æç 3
÷ + ([x ] + 1) {x} (b) æç 3
÷ + ([x ] + 1) {x}
è 2 ø è 2 ø
3
[x ] ([x ] + 1) ö
(c) æç 2
÷ + ([x ] + 1) {x} (d) None of these
è 2 ø
10 p
9. The value of ò [tan-1 x ]dx (where, [×] denotes the greatest integer function of x ) is equal to
0
(a) tan1 (b) 10p
(c) 10p - tan 1 (d) None of these
1
10. If f ( x ) = min {| x - 1|,| x |,| x + 1|, then the value of ò f ( x )dx is equal to
-1
1
(a) 1 (b)
2
1 1
(c) (d)
4 8
¥
11. The value of ò [2e - x ]dx (where, [×] denotes the greatest integer function of x ) is equal to
0
(a) 1 (b) loge 2
1
(c) 0 (d)
e
Chap 02 Definite Integral 99

10 p
12. The value of ò ([sec-1 x ] + [cot -1 x ])dx (where, [×] denotes the greatest integer function) is equal to
1
(a) (sec 1) - 10p (b) 10p - sec 1
(c) p - sec 1 (d) None of these
p/ 2
13. The value of ò [cot -1 x ]dx (where, [×] denotes greatest integer function) is equal to
-p/ 2
(a) p + cot1 (b) p + cot2
(c) p + cot 1+ cot 2 (d) cot 1+ cot 2
p/ 4
14. The value of ò (tann( x - [ x ]) + tann- 2( x - [ x ]))dx (where, [×] denotes greatest integer function) is equal to
0

1 1 1 1
(a) (b) (c) (d)
n n-1 n (n - 1) n (n + 1)
2
15. The value of ò [ x 2 - x + 1]dx (where, [×] denotes the greatest integer function) is equal to
0

5+ 5 1+ 5 1- 5 5- 5
(a) b) (c) (d)
2 2 2 2
a
16. Evaluate ò [ x n ]dx , (where, [×] denotes the greatest integer function).
0
x 1
17. Prove that ò [ x ]dx = x [ x ] - [ x ]([ x ] + 1), where [×] denotes the greatest integer function.
0 2
n

18. If f (n ) = ò0n
[ x ]dx
(where, [×] and {}× denotes greatest integer and fractional part of x and n Î N). Then, the value
ò0 {x }dx
of f (4) is ...
x æ pö
19. If f (n ) = ò [cos t ]dt , where x Î ç2np , 2np + ÷; n Î N and [×] denotes the greatest integer function. Then, the value
0 è 2ø
æ 1ö
of f ç ÷ is ...
èpø
x [x ]
20. If ò [ x ]dx = ò xdx , x Ïinteger (where, [×] and {}× denotes the greatest integer and fractional parts respectively,
0 0
then the value of 4{x } is equal to ...
Session 4
Applications of Even-Odd Property and
Half the Integral Limit Property

Applications of Even-Odd y Example 42 Evaluate


p /4
ò- p / 4 x
3
sin 4 x dx.
Property and Half the Integral Sol. Let f ( x ) = x 3 sin 4 x , then
Limit Property f ( - x ) = ( - x )3 sin 4 ( - x ) = - x 3 [sin ( - x )]4
Property VI. = - x 3 ( - sin x )4 = - x 3 sin 4 x = - f ( x )
ìï 2 a
f ( x ) dx = í ò0
a f ( x ) dx , if f ( x ) is an even function So, f ( x ) is an odd function.
ò- a Hence,
p/ 4
ò - p/ 4 f ( x ) dx = 0
îï 0, if f ( x ) is an odd function
p/ 4
Proof We know,
b c b
i.e. ò - p/ 4 x 3 sin 4 x dx = 0

òa f ( x ) dx = ò f ( x ) dx + ò
a c
f ( x ) dx , if a < c < b
p /2
ò- p / 2 sin
2
y Example 43 Evaluate x dx.
a 0 a
\ ò- a f ( x ) dx = ò
-a
f ( x ) dx + ò
0
f ( x ) dx …(i)
Sol. Let f ( x ) = sin 2 x , then
0 0
Now, ò f ( x ) dx = ò f ( - t ) ( - dt ), where t = - x f ( - x ) = sin 2 ( - x ) = [sin ( - x )]2 = ( - sin x )2
-a a
= sin 2 x = f ( x )
0 a
=-ò f ( - t ) dt = ò f ( - t ) dt So, f ( x ) is an even function, hence
a 0
p/ 2 p/ 2


a
f ( - x ) dx (using properties I and II)
ò- p/ 2 (sin 2 x ) dx = 2 ò
0
(sin 2 x ) dx
0 p/ 2 1 - cos 2x
=2ò dx
ì a 0 2
ï ò
f ( x ) dx , if f ( x ) is even p/ 2
= í a0 …(ii) æ sin 2x ö p
= çx - ÷ =
ï- ò f ( x ) dx , if f ( x ) is odd è 2 ø 2
î 0 0
p/ 2 p
ò- p/ 2
2
From Eqs. (i) and (ii), we get \ sin x dx =
2
ìï 2 a
f ( x ) dx = í ò0
a f ( x ) dx , f ( x ) is even
ò- a ïî y Example 44 The value of ò
1 æ2 - x ö
log ç ÷ dx is equal to
0, if f ( x ) is odd -1 è2+ x ø
1
1 (a) (b) 1 (c) –1 (d) 0
ò- 1 ( x
3
y Example 41 Evaluate + 5x + sin x )dx . 2
æ 2- x ö
Sol. Let, f ( x ) = x 3 + 5x + sin x Sol. Let f ( x ) = log ç ÷
è 2+ x ø
\ f ( - x ) = - x 3 - 5x - sin x = - f ( x ) -1
æ 2+ x ö æ 2- x ö
So, f ( x ) is an odd function). Now, f ( - x ) = log ç ÷ = log ç ÷
1
è 2- x ø è 2+ x ø
Hence, ò ( x 3 + 5x + sin x )dx = 0
-1 æ 2- x ö
= - log ç ÷
é ìï2 f ( x )dx , f ( x ) is even ù
a è 2+ x ø
ê using ò f ( x ) dx = í ò0
a
ú
êë -a
ïî0, f ( x ) is odd úû æ 2- x ö
\ f ( - x ) = - log ç ÷ = - f (x )
è 2+ x ø
Chap 02 Definite Integral 101

i.e. f ( x ) is an odd function. ½cos x e x 2 2x cos 2 x / 2½


æ 2- x ö
y Example 46 If f ( x ) =½ x 2 sec x sin x + x 3 ½,
1 1
So, ò- 1 f ( x ) dx = ò- 1 log ç ÷ dx = 0
è 2+ x ø ½ ½
1 2 x + tan x
é
½ ½
if f ( x ) is odd ù
ìï 0, p /2
a
then value of ò ( x 2+ 1) [ f ( x ) + f ¢¢( x )] dx is equal
êQ ò- a f ( x ) dx = í a ú - p /2
îï ò0
êë 2 f ( x ) dx , if f ( x ) is even ú
û to
(a) 1 (b) –1 (c) 2 (     
Hence, (d) is the correct answer.
½ cos x e x 2 2x cos 2 x / 2½
y Example 45 The value of Sol. As, f ( x ) =½ x 2 sec x sin x + x 3 ½
½ ½
1 2 x + tan x
æ p ö ½ ½
x sin (2x ) × sin ç cos x ÷
p è 2 ø Þ f (- x ) = - f (x )
ò0 (2x - p )
dx is equal to
Þ f ( x ) is odd. Þ f ¢( x ) is even.
Þ f ¢¢( x ) is odd.
8 p 8 p2
(a) (b) (c) (d) Thus, f ( x ) + f ¢¢( x ) is odd function, let
p 8 p2 8
f ( x ) = ( x 2 + 1) × { f ( x ) + f ¢¢( x )}
æ p ö Þ f (- x ) = - f (x )
x sin (2x ) × sin ç cos x ÷
p è 2 ø i.e. f ( x ) is odd.
Sol. Let I = ò0 ( 2x - p )
dx …(i) p/ 2
\ ò- p/ 2 f ( x ) dx = 0
æp ö
( p - x ) × sin 2 ( p - x ) × sin ç cos ( p - x )÷ Hence, (d) is the correct answer.
p è2 ø
I = ò0 2 (p - x ) - p
dx
y Example 47 The value of
1 x -1
æ p
ò- 1 1 - x 2 × sin (2x 1 - x ) dx is equal to
ö 2
( p - x ) × sin (2x ) × sin ç cos x ÷
p è 2 ø
I = ò0 - ( 2x - p )
dx …(ii)
(a) 4 2 (b) 4 ( 2 - 1) (c) 4 ( 2 + 1) (d) None of these
Adding Eqs. (i) and (ii), we get 1 x
Sol. Let I = ò- 1 × sin - 1 (2x 1 - x 2 ) dx
æ p ö 1- x 2
(2x - p ) sin 2x × sin ç cos x ÷
p è 2 ø
2I = ò0 ( 2x - p )
dx =2ò
0
1 x
× sin - 1 (2x 1 - x 2 ) dx
1 - x2
1 p æ p ö
\ I = ò0 2 sin x cos x × sin ç cos x ÷ dx é using a f ( x ) dx = 2 a f ( x ) dx , if f ( - x ) = f ( x ) ù
2 è 2 ø
êë ò- a ò0 úû
(put cos x = t , then - sin x dx = dt)
Put x = sin q Þ dx = cos q dq
-1 æ p ö
=- ò1 t sin ç t ÷ dt
è 2 ø \ I =2ò
p/ 2 sin q
× sin - 1 (2 sin q cos q ) × cos q dq
0
1 - sin 2 q
1 æ p ö
= 2 ò t × sin ç t ÷ dt (using by parts) p/ 4 p/ 2
0 è 2 ø =2ò sin q × 2q dq + 2 ò ( p - 2q ) sin q dq
0 p/ 4
ìæ æp öö
1
æp ö ü é -1 ì 2q , 0<q < p/4 ù
ï ç cos ç t ÷ ÷ cos ç t ÷ ï êë using sin (sin 2q ) = íî p - 2q , p / 4 < q < p / 2ûú
ï è2 ø 1 è2 ø ï
= 2 í çt × ÷ -
ò0 1 × dt ý p/ 4 p/ 2 p/ 2
p ÷ p = 4ò
ï çç - ÷ - ï 0
q × sin q dq + 2p òp/ 4 sin q dq - 4 òp / 4 q × sin q dq
ïè 2 ø0 2 ï
î þ p/ 4
= 4 {q ( - cos q )} p0 / 4 - 4 ò0 1 × ( - cos q )dq + 2p ( - cos q )pp// 24
ì æ æp ö ö

p/ 2
ï ç sin ç t ÷ ÷ ï - 4 {q ( - cos q )} pp// 24 + 4
ï 2 ç è2 ø ï
÷ ý= 8
òp/ 4 ( - cos q ) dq
= 2 í0 + × p p
p
ï p çç ÷ ï p2
÷
=- + 2 2 + 2p - - 4 +2 2 = 4 2 - 4
ï è 2 ø0ï 2 2
î þ
= 4 ( 2 - 1)
Hence, (c) is the correct answer.
Hence, (b) is the correct answer.
102 Textbook of Integral Calculus

y Example 48 Suppose the function p ( p - x ) dx p p dx p x dx


= ò0 = ò0 - ò0
g n ( x ) = x 2n +1 + an x + b n (n Î N ) satisfies the equation 1 + cos 2 ( p - x ) 1 + cos 2 x 1 + cos 2 x
1 p dx
ò-1 (px + q )g n ( x )dx = 0 for all linear functions (px + q ), \ I =p ò0 1 + cos 2 x
-I

then p dx p/ 2 dx
(a) a n = b n = 0 (b) b n = 0 ; a n = –
3 Þ 2I = p ò0 1 + cos x 2
= 2p ò0 1 + cos 2 x
2n + 3
é 2a ìï 0, f (2a - x ) = - f ( x ) ù
3 3 3 ê using ò f ( x ) dx = í ú
(c) a n = 0 ; b n = – (d) a n = ;b n = – a
2n + 3 2n + 3 2n + 3 êë 0
ïî 2 ò0 f ( x ) dx , f (2a - x ) = f ( x )úû
1
Sol. We have, ò-1 ( px + q )( x 2n + 1 + an x + bn )dx = 0 p/ 2 sec 2 x
Þ I =p ò0 sec 2 x + 1
dx
Equating the odd component to be zero and integrating,
we get (dividing numerator and denominator by cos 2 x)
2p 2a p
+ n + 2bn q = 0 for all p, q p/ 2 sec 2 x
2n + 3 3 I =p ò0 2 + tan 2 x
dx
3
Therefore, bn = 0 and an = –
2n + 3 Put tan x = t Þ sec 2 x dx = dt
Hence, (b) is the correct answer. Also, when x = 0,then t = 0 and when x = p / 2, then t = ¥
¥
Property VII (a). ¥ dt p æ -1 t ö
ìï a üï
Hence, I =p ò0 2 + t2
= ç tan
è
÷
2 ø
f ( x ) dx = í2 ò0 f ( x ) dx , if f (2a - x ) = f ( x ) ý
2a 2 0
ò0 ïî 0, if f (2a - x ) = - f ( x )ïþ p æ p ö p 2
= ç -0 ÷ =
2 è 2 ø 2 2
Proof We know,
2a a 2a y Example 50 Prove that
ò0 f ( x ) dx = ò f ( x ) dx + ò f ( x ) dx …(i) p /2 p /2 p
2a
0 a
ò0 log (sin x ) dx = ò
0
log (cos x ) dx = -
2
log 2.
Consider the integral ò f ( x ) dx ; putting x = 2a - t , so p/ 2
0 Sol. Let I = ò0 log (sin x ) dx …(i)
that dx = - dt p/ 2

Also, when x = a,then t = a and when x = 2a, then t = 0.


Then, I = ò0 log sin ( p / 2 - x ) dx
p/ 2
2a 0 0 = ò0 log (cos x ) dx …(ii)
\ òa f ( x ) dx = ò
a
f (2a - t ) ( - dt ) = - ò
a
f (2a - t ) dt
Adding eqs. (i) and (ii), we get
a a
=ò f (2a - t ) dt = ò f (2a - x ) dx p/ 2 p/ 2
0 0 2I = ò0 log sin x dx + ò0 log cos x dx

ì a p/ 2
ï ò0 f ( x ) dx , if f (2a - x ) = f ( x ) = ò0 (log sin x + log cos x ) dx
=í a
…(ii)
ï ò
- f ( x ) dx , if f (2a - x ) = - f ( x ) p/ 2 p/ 2 æ 2 sin x cos x ö
î 0 = ò0 log (sin x cos x ) dx = ò0 log ç
è 2
÷dx
ø
From Eqs. (i) and (ii), we have p/ 2 æ sin 2x ö
= ò0 log ç
è 2 ø
÷ dx
ïì üï
a
f ( x ) dx = í2 ò0 f ( x ) dx , if f (2a - x ) = f ( x ) ý
2a
ò0 = ò0
p/ 2
log (sin 2x ) dx - ò0
p/ 2
(log 2) dx
îï 0 , if f (2a - x ) = - f ( x )þï
p/ 2
= ò0 log sin 2x dx - (log 2) ( x )0p/ 2
p x
y Example 49 Evaluate ò0 1 + cos 2 x
dx .
Þ 2I = ò0
p/ 2
log (sin 2x ) dx -
p
log 2 …(iii)
2
p x p/ 2
Sol. Let I = ò0 1 + cos x 2
dx Let I1 = ò0 log (sin 2x ) dx
Chap 02 Definite Integral 103

p dt 1 p æ p x ö æ p x ö
I1 = ò0 log sin t =
2 2 ò0 log sin t dt (putting 2x = t ) \ 2 f ç + ÷ - 2 f ç - ÷ = x (log 2)
è 4 2 ø è 4 2 ø
x /2
1 p/ 2 -2ò log (cos 2z ) dz
×2 ò = log (sin t ) dt 0
2 0
æ p x ö æ p x ö
ì 0, f (2a - x ) = - f ( x ) Þ 2 f ç + ÷ - 2 f ç - ÷ = x (log 2) + f ( x )
é using 2a f ( x ) dx = ï a è 4 2 ø è 4 2 ø
ëê ò0
í2
ïî ò 0 f ( x ) dx , f (2a - x ) = f ( x ), æ p x ö æ p x ö
p/ 2 or f ( x ) - 2 f ç + ÷ + 2 f ç - ÷ = - x log 2
=ò log (sin x ) dx è 4 2 ø è 4 2 ø
0
p Hence, (a) is the correct answer.
\ Eq. (iii) becomes 2I = I - log 2
2 2
æ x ö p
Hence, ò
p/ 2 p
log sin x dx = - log 2. y Example 52 If ò ç ÷ dx = A, then the value
0 0 è 1 + sin x ø
2
2 2
Remark p 2x × cos x / 2
for ò dx is equal to
Students are advised to learn 0 (1 + sin x ) 2
p/ 2 p/ 2 p
ò0 log (sin x ) dx = ò
0
log (cos x ) dx = -
2
log 2. (a) A + 2p - p 2 (b) A - 2p + p 2
(c) 2p - A - p 2 (d) None of these
x
y Example 51 If f ( x ) = - ò log (cos t ) dt, then the p 2x 2 cos 2 x / 2
æp xö
0
æp xö
Sol. Let B= ò0 (1 + sin x )2
dx
value of f ( x ) - 2 f ç + ÷ + 2f ç - ÷ is equal to
è4 2ø è4 2ø p x 2 (2 cos 2 x / 2 - 1)
x \ B-A= ò0 (1 + sin x )2
dx
(a) - x log 2 (b) log 2
2
x p x 2 × cos x
(c) log 2
3
(d) None of these = ò0 (1 + sin x )2
dx

æ p x ö p/ 4 + x / 2
Sol. Here, f ç + ÷ = -
è 4 2 ø ò0 log (cos t ) dt Using by parts,
p
ì x2 ü p x
p/ 4 p/ 4 + x / 2 B- A =í - ý + 2 ò0 dx
=- ò0 log (cos t ) dt - òp/ 4 log (cos t ) dt …(i) î 1 + sin x þ0 ( 1 + sin x)

æ p x ö p/ 4 - x / 2 B - A = - p 2 + 2K …(i)
fç - ÷=-
è 4 2 ø ò0 log (cos t ) dt
p x dx p ( p - x ) dx
p/ 4 p/ 4 - x / 2
where, K= ò0 1 + sin x
= ò0 1 + sin x
=- ò0 log (cos t ) dt - òp/ 4 log (cos t ) dt …(ii)
p dx
æ p x ö æ p x ö
\ 2f ç + ÷ - 2f ç - ÷
\ K=p ò0 1 + sin x
- K,
è 4 2 ø è 4 2 ø
ì 0, f (2a - x ) = - f ( x )
p/ 4 - x / 2 p/ 4 + x / 2 é using 2a f ( x ) dx = ï a
=2ò log (cos t ) dt - 2 ò log (cos t ) dt ò í2
îï ò 0
p/ 4 p/ 4 ëê 0 f ( x ) dx , f (2a - x ) = f ( x ),
p p
Put t = - z in first integral and t = + z in second p/ 2 dx p/ 2 dx
4 4 Þ 2K = 2p ò0 1 + sin x
= 2p ò0 1 + cos x
integral, we get
x /2 æp ö x /2 æ pö p/ 2
= -2ò log cos ç - z ÷ dz - 2ò log cos çz + ÷ dz p/ 2 1 x ì xü
0 è4 ø 0 è 4ø = 2p ò0 2
sec 2 dx = 2p í tan ý
2 î 2 þ0
= 2p
x /2 æ 1 ö
= -2ò log ç (cos 2 z - sin 2 z ) ÷ dz \ K=p
0 è 2 ø
x /2 x /2 Thus, B - A = - p 2 + 2K
=2ò (log 2) dx - 2 ò log (cos 2z ) dz
0 0 Þ B = A - p 2 + 2p
x /2
= x log 2 - 2 ò log (cos 2z ) dz Hence, (a) is the correct answer.
0
104 Textbook of Integral Calculus

y Example 53 The value of As described in the figure above, the new variable t is
a given by,
ò- a (cos - 1 x - sin - 1 1 - x 2 ) dx (a > 0) (where,
æ b¢ - a¢ ö
a t = a¢ + ç ÷ ( x - a ).
cos - 1 x dx = A) is è b -a ø
ò0
b¢ - a¢
(a) pa - A (b) pa + 2A Thus, dt = dx
b -a
(c) pa - 2A (d) pa + A
b
a
(cos - 1 x - sin - 1 1 - x 2 ) dx Þ I = ò f ( x ) dx
Sol. Let I = ò- a (as a > 0) 0
0 -1 -1 æ b¢ æ b -a ö ö æ b -a ö
= ò- a (cos x - sin 2
1 - x ) dx = ò f ça + ç ÷ (t - a ¢ ) ÷ ç ÷ dt
a¢ è è b¢ - a¢ ø ø è b¢ - a¢ ø
a
+ ò0 (cos - 1 x - sin - 1 1 - x 2 ) dx The modified integral has the limits (a¢ to b¢). A particular
0 a case of this property is modifying the arbitrary integration
= ò- a cos - 1 x dx + A - 2 ò sin - 1 1 - x 2 dx
0 limits (a to b) to (0 to 1) i.e. a¢ = 0 and b¢ = 1. For this case,
0 b 1
=- òa ( p - cos - 1 x ) dx + A - 2A I = ò f ( x ) dx = (b - a ) ò f (a + (b - a ) t ) dt
a 0
a -1
= pa - ò0 cos x dx - A
y Example 54 Evaluate
= pa - A - A = pa - 2A æ 2ö2
-5 2 2/ 3 9 çx - ÷
è 3ø
Hence, (c) is the correct answer.
ò- 4 e ( x + 5) dx + 3 ò e dx .
1/ 3
Property VII (b) 2
Sol. Here, we know ò e x dx cannot be evaluated by indefinite
b 1
òa f ( x ) dx = (b - a ) ò f [(b - a ) x + a ] dx
0
integral.
-5 + 5 )2 1 - 4 + 5 ]2
Sometimes, it is convenient to change the limits of Let I 1 = ò- 4 e (x dx = ( - 5 + 4 ) ò e [(- 5 + 4 ) x dx
0
integration into some other limits. For example, suppose
1 - 1) 2
we have to add two definite integrals I 1 and I 2 ; the limits \ I1 = - ò0 e (x dx …(i)
of integration of these integrals are different. if we could
2/3 - 2 / 3 )2
somehow change the limits of I 2 into those of I 1 or Again, let I2 = ò1/ 3 e 9 (x dx
vice-versa, or infact change the limits of both I 1 and I 2
2
into a third (common) set of limits, the addition could be é æ 2 1 ö 1 2 ù
9ê ç - ÷ x + - ú
æ 2 1 ö 1
ë è 3 2 ø
accomplished easily. =ç - ÷ò e 3 3 û
dx
è 3 3 ø 0
b
Suppose that I = ò f ( x ) dx . We need to change the limits 1 1 2 1
a = ò e (x - 1) dx = ( - I 1 ) …(ii)
(a to b) to (a¢ to b¢). As x varies from a to b, we need a new 3 0 3
variable t (in terms of x) which varies from a¢ to b¢. æ I1 ö
where, I = I 1 + 3I 2 = I 1 + 3 ç - ÷ = I1 - I1 = 0
è 3 ø
t
-5 + 5 )2 2/3 - 2 / 3 )2
b′ \ ò-4 e (x dx + 3 ò
1/ 3
e 9 (x dx = 0

Property VII (c) If f (t ) is an odd function, then


x
f( x ) = ò f (t ) dt is an even function.
a
a′
-x
Proof We have, f( - x ) = ò f (t ) dt
a
x -a -x
a b
Þ f( - x ) = ò f (t ) dt + ò f (t ) dt
a -a
As x varies from a to b, t varies from a to b, t varies from a¢
-x
to b¢. Þ f( - x ) = 0 + ò f (t ) dt
-a
t - a¢ b¢ - a¢
Thus, = éQ f (t ) is an odd function, then a f (t ) dt = 0 ù
x -a b -a êë ò- a úû
Chap 02 Definite Integral 105

x x
Þ f (- x ) = - ò f ( - y ) dy , where t = - y Þ f( - x ) = - ò f (y ) dy
a 0
x [Q f is even function Þ f ( - y ) = f (y )]
Þ f( - x ) = ò f (y ) dy
a x
Þ f( - x ) = - ò f (t ) dt
[Q f is an odd function, then f ( - y ) = - f (y )] 0
x
Þ f( - x ) = ò f (t ) dt Þ f( - x ) = - f( x )
a
Hence, f ( x ) is an odd function.
Þ f( - x ) = f( x )
x
Hence, f( x ) = ò f (t ) dt is an even function, if f (t ) is odd. Remark
a x
If f ( t ) is an even function, then for non-zero ‘ a’, ò f ( t ) dt is not
a
x æ1 -t ö necessarily an odd function. It will be an odd function, if
y Example 55 If f ( x ) = ò log ç ÷ dt, then discuss a x
0 è 1+t ø ò f (t ) dt = 0, because, if f( x ) = ò f (t ) dt, it is an odd function.
0 a

whether even or odd? Þ f( - x ) = - f( x )


-x
æ 1-t ö x
Sol. Let f (t ) = log ç ÷
Þ òa f ( t ) dt = - ò
a
f ( t ) dt
è 1+t ø 0 -x 0 x

\
æ
f ( - t ) = log ç
1+t ö æ 1-t ö Þ òa f ( t ) dt + ò0 f ( t ) dt = - ò f ( t ) dt - ò
0
f ( t ) dt
÷ = - log ç ÷ = - f (t )
a

è 1-t ø è 1+t ø 0 x a x
Þ òa f ( t ) dt - ò
0
f ( - y ) dy = - ò
0
f ( t ) dt - ò
0
f ( t ) dt
Þ f ( - t ) = - f (t ), i.e. f(t ) is odd function
[ where, y = - t in second integral of LHS Þ f ( - y ) = f ( y)]
x æ1 - t ö
\ f ( x ) = ò log ç ÷ dt is an even function. Þ 2ò
0
f ( t ) dt = ò
x
f ( y ) dy - ò
x
f ( t ) dt
0 è1 + t ø a 0 0
0
Property VII (d) If f (t ) is an even function, then Þ 2ò f ( t ) dt = 0
a
x
f( x ) = ò f (t ) dt is an odd function. -ò
a a
0
Þ
0
f ( t ) dt = 0 Þ ò0 f ( t ) dt = 0
-x x x
Proof We have, f( - x ) = ò f (t ) dt = - ò f ( - y ) dy , or f(x) = ò
a
f ( t ) dt is an odd function, [when f ( t ) is even.]
0 0
a
where t = - y Only, if ò f ( t ) dt = 0
0
106 Textbook of Integral Calculus

Exercise for Session 4


p/ 2
1. Let f : R ® R and g : R ® R be continous functions, then the value of ò [f ( x ) + f ( - x )][ g( x ) - g( - x )] dx is
- p/ 2
equal to
(a) - 1 (b) 0 (c) 1 (d) None of these
1
2. The value of ò ( x x ) dx is equal to
-1
1
(a) 1 (b) (c) 0 (d) None of these
2
1 æ x 2 + sin x ö
3. The value of ò ç ÷ dx is equal to
-1 è 1+ x 2 ø
p
(a) 2 - p (b) p - 2 (c) 2 - (d) None of these
2
a f (sin x )
4. If f ( x ) is an odd function, then the value of ò dx is equal to
-a f (cos x ) + f (sin2 x )
(a) 0 (b) f (cos x ) + f (sin x ) (c) 1 (d) None of these
æ 2x ö æ 2x ö
cos - 1ç 2
÷ + tan- 1ç ÷ dx
1/ 3 è1+ x ø è1- x 2 ø
5. The value of ò is equal to
-1/ 3 1+ ex
p p p p
(a) (b) (c) (d)
2 3 2 3 3 3
p cos 2 x
6. The value of ò dx , where a > 0, is
-p 1+ ax [IITJEE 2001]
p
(a) p (b) ap (c) 2p (d)
2
1/ 2 ì æ1+ x ö ü
7. The integral ò í[ x ] + loge ç ÷ ýdx is equal to (where, [×] denotes greatest integer function)
- 1/ 2
î è 1- x ø þ

(d) 2 log æç ö÷
1 1
(a) - (b) 0 (c) 1
2 è 2ø
p/2 1
8. The value of ò dx is equal to
-p/2 e sinx + 1
p p
(a) 0 (b) 1 (c) (d) -
2 2
p/2 æé p ù ö
9. If [×] denotes greatest integer function, then the value of ò ç ê ú + 0.5÷ dx is
-p/2 èë 2 û ø
p p
(a) p (b) (c) 0 (d) -
2 2
p /4 ì b sin x ü
10. The equation ò ía sin x + 2
+ cý dx = 0, where a, b, c are constants gives a relation between
- p /4
î 1 + cos x þ
(a) a, b and c (b) a and c (c) a and b (d) b and c
2
2 sin x
11. The value of ò dx , where [×] denotes greatest integer function, is
-2 éxù 1
+
êë p úû 2
(a) 1 (b) 0 (c) 4 - sin 4 (d) None of these
n+1 3
12. Let f ( x ) be a continuous function such that ò f ( x )dx = n ,n Î Z. Then, the value of ò
3
f ( x )dx is
m -3

(a) 9 (b) - 27 (c) - 9 (d) 27


Chap 02 Definite Integral 107

ex + 1 1 x
3 e + 1 1
13. Let f ( x ) = x
e -1
and ò 0
x × x
e -1
dx = a. Then, ò t 3 f (t ) dt is equal to
- 1

(a) 0 (b) a (c) 2a (d) None of these


2
14. Let f : R ® R be a continuous function given by f ( x + y ) = f ( x ) + f ( y ) for all x , y Î R. if ò f ( x ) dx = a, then
0
2
ò - 2 f ( x ) dx is equal to
(a) 2a (b) a (c) 0 (d) None of these
2
15. The value of ò [ x ] dx is equal to
-2

(a) 1 (b) 2 (c) 3 (d) 4

n Directions (Q. Nos. 16 to 17)


ïì1 - x ,|x| £ 1
Let f ( x ) = í and g( x ) = f ( x + 1) + f ( x - 1) for all x Î R .
îï0, x > 1

16. The graph for g( x ) is given by

Y Y
(0, 1) (0, 1)

(a) X (b) X
–3 –1 O 1 3 –3 –2 –1 O 1 2 3

Y Y

1
–3 –2 –1 O 1 2 3
(c) X (d) X
–3 –2 –1 O 1 2 3
1

3
17. The value of ò g( x )dx is
-3

(a) 2 (b) 3 (c) 4 (d) 5

n
Direction (Q. Nos. 18 to 20)
p sin x
n
Let In = ò dx; n = 0, 1, 2, ...
-p (1 + p x ) sin x

18. The value of I n + 2 - I n is equal to


(a) np (b) p (c) - p (d) 0
10
19. The value of å I 2m + 1 is equal to
m =1

(a) 0 (b) 5p
(c) 10p (d) None of these
10
20. The value of å I 2m is equal to
m =1

(a) 0 (b) 5p
(c) 10p (d) None of these
Session 5
Applications of Periodic Functions and
Newton- Leibnitz’s Formula
Applications of Periodic Functions y Example 56 Evaluate ò0
4p
| cos x | dx .
and Newton-Leibnitz’s Formula Sol. Note that | cos x | is a periodic with period p.
If f ( x ) is a periodic function with period T , then the area p
under f ( x ) for n periods would be n times the area under Let I =4 ò0 | cos x | dx
f ( x ) for one period, i.e. é using property, f ( x ) dx ù
nT T

nT T ëê ò0 f ( x ) dx = n ò
0 ûú
ò0 f ( x ) dx = n ò f ( x ) dx
0
= 4 ìí
p/ 2 p
cos x dx üý
î ò0 cos x dx - òp/ 2 þ
Now, consider the periodic function f ( x ) = sin x as an
ìï æ p/ 2 p üï
example. The period of sin x is 2p. ö æ ö
= 4 í çsin x ÷ - çsin x ÷ ý = 4 {1 + 1} = 8
y ïî è ø0 è ø p/ 2 ïþ
25
2π y Example 57 Prove that ò0 e x - [ x ] dx = 25 (e - 1).

0 A
Sol. Since, x - [ x ] is a periodic function with period one.
x
a π 2π a+2π Therefore, ex - [x ]
has period one
25 ´ 1 - [x ] 1 - [x ]

\ I = ò0 ex dx = 25 ò e x
0
dx
é using property, nT T
f ( x ) dx ù
Figure 2.6 êë ò0 f ( x ) dx = n ò
0 úû
1 -0
a +2p = 25 ò e x dx = 25 (e x ) 10 = 25 (e 1 - e 0 )
Suppose we intend to calculate ò sin x dx as depicted 0
a = 25 (e - 1)
above. Notice that the darkly shaded area in the interval 2np
[2 p, a + 2 p ] can precisely cover the area marked as y Example 58 The value of ò0 [sin x + cos x ] dx is
a +2p 2p equal to
Thus, òa sin x dx = ò sin x dx
0 (a) - np (b) np (c) - 2np (d) None of these
This will hold true for every periodic function, i.e. (where [.] denotes the greatest integer function)
2np

òa
a +T T
f ( x ) dx = ò f ( x ) dx
Sol. Let I = ò0 [sin x + cos x ] dx
0 æ p ö
We know, sin x + cos x = 2 sin ç x + ÷ …(i)
(where T is the period of f ( x )) è 4 ø
This also implies that ì 1, 0 £ x £ p /2
a + nT nT T
ï 0, p / 2 < x £ 3p / 4
ï
òa f ( x ) dx = ò
0
f ( x )dx = n ò f ( x ) dx
0 ï - 1, 3p / 4 < x £ p
\ sin x + cos x = í
b + nT b
ï - 2, p < x < 3p / 2
and òa + nT f ( x ) dx = òa f ( x ) dx ï - 1, 3p / 2 £ x < 7 p / 4
ï
b +nT b T î 0, 7 p / 4 £ x £ 2p
and òa f ( x ) dx = ò f ( x )dx + n ò f ( x ) dx
a 0 2p p/ 2 3 p/ 4
\ò [sin x + cos x ]dx = ò (1) dx + òp/ 2 (0) dx
0 0
Chap 02 Definite Integral 109

p 3 p/ 2 7 p/ 4 2p 5p
+ ò3 p/ 4( - 1)dx + òp ( -2) dx + ò3 p/ 2 ( - 1) dx + ò7 p/ 4 (0) dx y Example 61 The value of ò- 2p cot - 1 (tan x ) dx is
æpö æ 3p ö æ 3p ö æ 7 p 3p ö equal to
= ç ÷ + ( 0) - ç p - ÷ -2ç - p÷ - 1 ç - ÷ +0
è2ø è 4 ø è 2 ø è 4 2 ø 7p 7p 2
(a) (b)
=-p 2 2
3p
Since, sin x + cos x has the period 2p. (c) (d) p 2
2np 2p
2
So, I = ò0 [sin x + cos x ] dx = n ò0 [sin x + cos x ] dx
Sol. Let I =
5p
ò- 2 p cot - 1 (tan x ) dx
= - np
p æ æ p ö ö
Hence, (a) is the correct answer. Þ I =7 ò0 cot - 1 ç cot ç - x ÷ ÷ dx …(i)
è è 2 ø ø
5
éQ f ( x ) dx ù
ò- 5
mT T
y Example 59 The value of f ( x ) dx ; where ònT f ( x ) dx = (m - n ) ò
ëê 0 ûú
f ( x ) = minimum ({ x + 1}, { x - 1}), " x ÎR and {.} denotes ì x , 0 < x < p /2
fractional part of x, is equal to As we know, cot - 1 (cot x ) = í
î p + x , p /2 < x < p
(a) 3 (b) 4 (c) 5 (d) 6
ì p/ 2 æ p ö p æ p ö ü
Sol. We know, { x + 1} = { x - 1} = { x } \ I = 7 íò ç - x ÷ dx + òp/ 2 ç p + - x ÷ dx ý
î 0 è 2 ø è 2 ø þ
Thus, f ( x ) = minimum ({ x + 1}, { x - 1}) = { x }
ì æp p/ 2 p
5 5 1
ï x2 ö æ 3p x 2 ö üï
Þ ò- 5 f ( x ) dx = ò- 5 { x } dx = (5 - ( - 5)) ò
0
{ x } dx =7í ç x - ÷ +ç x- ÷ ý
ïî è 2 2 ø0 è 2 2 ø p/ 2 ï
þ
[as { x } is periodic with period ‘1’]
1
= 10 ò ( x - [ x ]) dx = 10 ò x dx
1 ïì æ p 2 p 2 ö æ 3p 2 p 2 3p 2 p 2 öïü 7 p 2
= 7 íç - ÷+ç - - + ÷ý =
0 0
ïî è 4 8 ø è 2 2 4 8 øïþ 2
1
æ x ö 2
Hence, (b) is the correct answer.
= 10 ç ÷ =5
è 2 ø0
y Example 62 Let g ( x ) be a continuous and
Hence, (c) is the correct answer.
differentiable function such that
np + V
y Example 60 Show ò0 | sin x | dx = (2n + 1) - cos V , 2 ì 5/ 2 ü
ò0 íî ò 2 [2x - 3] dx ýþ × g ( x ) dx = 0, then g ( x ) = 0
2

where n is a positive integer and 0 £ V < p.


[IIT JEE 1994, 2004] when x Î(0, 2) has (where, [.] denotes the greatest inte-
np + V V np + V ger function)
Sol. ò | sin x | dx = ò0 | sin x | dx + òV | sin x | dx
0 (a) exactly one real root (b) atleast one real root
V p
= ò0 sin x dx + n ò0 | sin x | dx (c) no real root (d) None of these
Sol. As, 1 < 2x 2 - 3 < 2, " x Î ( 2, 5 / 2 )
é a +nT T
ê using property, òa f ( x )dx = n ò f ( x )dx i.e. 5/2
ë
npV
0
p ù
Þ ò2 [2x 2 - 3] dx > 0, " x Î (0, 2)
òV |sin x |dx = n òV |sin x |dx úû Þ g ( x ) = 0 should have atleast one root in (0, 2).
æ ö
V
p
[Q g ¢ ( x ) ¹ 0]
= ç - cos x ÷ + n ò sin x dx Hence, (b) is the correct answer.
è ø0 0
p
æ ö y Example 63 The value of x satisfying
= ( - cos V + 1) + n ç - cos x ÷
è ø0 2 [ x + 14 ] ì x ü {x}

= - (cos V ) + 1 + n (1 + 1) = (2n + 1) - cos V ò0 í ý dx = ò0 [ x + 14 ] dx is equal to


î 2 þ
np + V
\ò | sin x | dx = (2n + 1) - cos V , where n is a positive (where, [.] and {.} denotes the greatest integer and
0
fractional part of x)
integer and 0 £ V < p.
(a) [ - 14 , - 13) (b) (0, 1)
(c) (- 15, - 14 ] (d) None of these
110 Textbook of Integral Calculus

2 [ x + 14 ] ì x ü {x } d ì x
cos t dt üý = cos x
ò0 dx î ò0
Sol. Given, í ý dx = ò0 [ x + 14 ] dx Þ í
î þ
2 þ
28 + 2 [ x ] ì x ü {x } x2
Þ ò0 í ý dx = ò0 (14 + [ x ]) dx
î 2 þ
(ii) Let f ( x ) = ò0 cos t 2 dt

d ìd ü ìd ü
28 ì x ü 28 + 2 [ x ] ì x ü \ ( f ( x )) = cos ( x 2 )2 × í ( x 2 )ý - cos (0)2 í (0) ý
Þ ò0 í ý dx +
î 2 þ
ò28 í ý dx = (14 + [ x ]) { x }
î 2 þ
dx î dx þ î dx þ
= 2x × cos x 4
2 ì x ü 2 [x ] ì x ü
Þ 14 ò0 í ý dx + ò0 í ý dx = (14 + [ x ]) { x } d æ x2 ö
î 2 þ î 2 þ Þ ç
dx è ò0 (cos t 2 ) dt ÷ = 2x cos x 4
ø
é using nT f ( x ) dx = n T f ( x ) dx and ù
ê ò0 ò0 ú
ê a + nT ú d æ x ö
dx è ò1/x
nT y Example 65 Evaluate ç cos t 2 dt ÷ .
êò f ( x ) dx = ò f ( x ) dx ; where T is period of f ( x )ú ø
ëa 0 û
Þ 14 + [ x ] = (14 + [ x ]) { x } x
Sol. Let f ( x ) = ò1/ x cos t 2 dt
Þ (14 + [ x ]) (1 - { x }) = 0
2
Þ [ x ] = - 14 d ìd ü æ 1 ö ì d æ 1 öü
\ ( f ( x )) = cos ( x )2 × í ( x )ý - cos ç ÷ í ç ÷ý
Þ x Î [ - 14, - 13) dx î dx þ è x ø î dx è x øþ
Hence, (a) is the correct answer. 1 1 æ 1 ö
= cos x + × cos ç 2 ÷
Property IX. Leibnitz’s Rule for the Differentiation 2 x x 2 èx ø
under the Integral Sign
d æ x ö 1 1 æ 1 ö
(i) If the functions f( x ) and y( x ) are defined on [a, b ] Þ ç
dx è ò1/ x cos t 2 dt ÷ =
ø 2 x
cos x + 2 cos ç 2 ÷
x èx ø
and are differentiable at a point x Î(a, b ) and f ( x , t )
is continuous, then d æ y - t2 x2 ö
d é y(x)
y Example 66 If ç ò0 e dt + ò sin 2 t dt ÷ = 0,
f ( x , t ) dt ù dx è ø
0
dx êë òf ( x ) úû dy
find ×
y(x) ¶ ìd y ( x ) ü ( x , y ( x )) dx
=ò f ( x , t ) dt + í ýf d æ y - t2 x2 ö
f ( x ) ¶x
dx è ò0 ò0
î dx þ Sol. We know, ç e dt + sin 2 t dt ÷ = 0
ìd f ( x ) ü ( x , f ( x )) ø
-í ýf
î dx þ 2 ì d ü ìd ü ìd ü
Þ e - y × í (y )ý - e - 0 í (0)ý + sin 2 ( x 2 ) í ( x 2 )ý
(ii) If the functions f( x ) and y( x ) are defined on [a, b ] î dx þ î dx þ î dx þ
and differentiable at a point x Î(a, b ) and f (t ) is ìd ü
- sin 2 0 í (0)ý = 0
continuous on [ f (a ), f (b )], then î dx þ
d æ y(x)
f (t ) dt ö÷ =
d
dx è òf ( x )
ç { y( x )} f ( y( x )) 2 dy dy 2
ø dx Þ e- y + 2x sin 2 x 2 = 0 Þ = - 2x e y sin 2 x 2
d dx dx
- { f ( x )} f ( f ( x ))
dx y Example 67 Find the points of maxima/minima of
2
x 2 t - 5t + 4
y Example 64 Find the derivative of the following with
respect to x.
ò0 2 + e t dt.
x x2 x2 t 2 - 5t + 4
(i) ò0 cost dt (ii) ò0 cost 2dt Sol. Let f ( x ) = ò0 2 + et
dt
x
Sol. (i) Let f (x ) = ò0 cos t dt
\ f ¢( x ) =
x 4 - 5x 2 + 4
× 2x - 0
2
d ì d ü ì d ü 2 + ex
\ ( f ( x )) = cos ( x ) í ( x ) ý - cos 0 × í (0) ý = cos x
dx î dx þ î dx þ ( x - 1) ( x + 1) ( x - 2) ( x + 2) × 2x
=
é d 2
ê using Leibnitiz’s rule, dx 2 + ex
ë
y (x ) d d ù – + – + – +
òf(x ) f ( t )dt =
dx
{y ( x )} f ( y ( x )) -
dx
{ f( x )} + f ( f( x )) ú
û –2 –1 0 1 2
Chap 02 Definite Integral 111

From the wavy curve, it is clear that f ¢( x ) changes its d p d æpö dy


sign at x = ± 2, ± 1, 0 and hence the points of maxima are Þ 3 - sin 2 x × ( x ) - 3 - sin 2 × ç ÷ + cos y ×
dx 3 dx è 3 ø dx
- 1, 1 (as sign changes from + ve to - ve) and of the
minima are –2, 0, 2 (as sign changes from - ve to + ve). - cos (0)
d
( 0) = 0
dx
d æ x3 1 ö
y Example 68 Find ç
dx è òx 2 dt ÷ .
log t ø
Þ 3 - sin 2 x + cos y
dy
dx
=0

d æ x3 1 ö 1 d 1 d dy 3 - sin 2 x
Sol. ç
dx è òx 2
log t
dt ÷ = ×
ø log x 3 dx
(x 3 ) - ×
log x 2 dx
(x 2 ) Þ
dx
=-
cos y
3x 2 2x
= - d e sin x
3 log x 2 log x y Example 71 Let (F ( x )) = , x > 0. If
dx x
d æ x3 1 ö 1 2
\ ç òx dt ÷ = (x 2 - x ) 4 2e sin x
ò1 dx = F (K ) - F (1), then the possible value of
2
dx è log t ø log x
x
x K is
y Example 69 If y = ò f (t ) sin {K ( x - t )} dt, then
0 (a) 10 (b) 14 (c) 16 (d) 18
d 2y d e sin x
prove that + K 2 y = Kf ( x ). Sol. We have, ( F ( x )) =
dx 2 dx x
x
e sin x
Sol. We have, y = ò0 f (t ) sin { K ( x - t )} dt Þ ò x
dx = F ( x ) …(i)
Differentiating w.r.t. x, we get 2 2


4 2e sin x 4 e sin (x )
16 e sin t
dy x d Now, ò dx = ò1 d( x 2 ) = ò1 dt
dx ò0 ¶x
= { f (t ) sin K ( x - t )} dt + (x ) 1 x x 2
t
dx
d (Q x 2 = t )
× { f ( x ) sin K ( x - x )} - (0) { f (0) sin K ( x - 0)}
dx = [ F (t )]16
1 = F (16) - F (1)
x
=K ò0 f (t ) cos K ( x - t ) dt + 0 - 0 Þ K = 16
dy x Hence, (c) is the correct answer.
Þ = K ò f (t ) cos K ( x - t ) dt
dx 0
y Example 72 The function
Again, differentiating both the sides w.r.t. x, we get x æ 1ö
f ( x ) = ò log | sin t | ç sin t + ÷ dt, where x Î(0, 2p ), then
d 2y ì x ¶ 0 è 2ø
dx
=Kí
2
î
ò0 ¶x
{ f (t ) cos K ( x - t )} dt
f ( x ) strictly increases in the interval
ì d ü d ü æ p 5p ö æ 5p ö
+í f ( x ) × cos K ( x - x ) × ( x )ý - { f (0)cos K ( x - 0) × (0)}ý (a) ç , ÷ (b) ç , 2p ÷
î dx þ dx þ è 6 6 ø è 6 ø
æ p 7p ö æ 5p 7 p ö
= K é- K f (t ) sin K ( x - t ) dt + f ( x ) - 0 ù
x

ëê ò0 ûú
(c) ç
è 6
,
6
÷
ø
(d) ç
è 6 6
, ÷
ø
x
æ 1ö
= - K2 ò0 f (t ) sin K ( x - t ) dt + K f ( x ) Sol. Here, f ¢ ( x ) = log| sin x | çsin x + ÷ > 0
è 2ø
d 2y d 2y
Þ = - K 2y + K f ( x ) Þ + K 2y = Kf ( x ) 1 æ 1ö
dx 2
dx 2 Þ sin x + < 1 and çsin x + ÷ > 0
2 è 2ø
x y 1
y Example 70 If òp / 3 3 - sin 2 t dt + ò
0
cos t dt = 0, Þ 0 < sin x +
2
<1
dy Þ - 1 / 2 < sin x < 1 / 2
then evaluate .
dx æ 5p 7 p ö
Þ x Îç , ÷ as x Î(0, 2p )
Sol. Differentiating w.r.t. x, we have è 6 6 ø
d æ x
3 - sin 2 t dt ö÷ +
d y

dx è òp/ 3 ò0
ç (cos t ) dt = 0 Hence, (d) is the correct answer.
ø dx
112 Textbook of Integral Calculus

x F ¢¢( x ) = f ¢ ( x ) Þ F ¢¢(2) = f ¢ (2)


y Example 73 Let f :(0, ¥ ) ® R and F ( x ) = ò t f (t ) dt.
0 From Eq. (i), f ¢ (2) = 256 + 1 = 257
12
2 4 5 2
If F ( x ) = x + x , then S f (r ) is equal to Hence, (c) is the correct answer.
r =1

(a) 216 (b) 219 (c) 221 (d) 223 Property X. Let a function f ( x , a ) be continuous for
x2 a £ x £ b and c £ a £ d , then for any a Î[c , d ],
ò0
2 4 5
Sol. Here, F(x ) = x + x = t f (t ) dt
b dI (a ) b ¶f ( x , a )
Differentiating w.r.t. x, we get if I (a ) = ò f ( x , a ) dx , then =ò dx
a da a ¶a
x 2 f ( x 2 ) × 2x = 4 x 3 + 5x 4
1 xb -1
Þ
5 5
f ( x ) = 2 + x Þ f (r 2 ) = 2 + r
2 y Example 76 Evaluate I(b ) = ò dx ;b ³ 0.
2 2 0 ln x
n
5 n (5n + 13) xb - 1
\ å f (r 2 ) = 2n +
2
n ( n + 1) =
4
Sol. We have, I (b ) = ò0
1
dx
r =1 ln x
12
12 (60 + 13) d 1 ¶ æ xb - 1 ö
Þ å f (r 2 ) =
4
= 219 Þ
db
( I (b )) = ò0 ç
¶b è ln x ø
÷ dx + 0 - 0
r =1

Hence, (b) is the correct answer. 1


1 x b ln x 1 æ xb + 1 ö
= ò0 ln x
dx = ò0 ( x b ) dx = ç ÷
è b + 1 ø0
y Example 74 A function f ( x ) satisfies
x 1 1
f ( x ) = sin x + ò f ¢ (t )(2 sin t – sin 2 t ) dt, then f ( x ) is = [1 - 0] =
0 b+1 b+1
x sin x 1– cos x tan x d 1
(a) (b) (c) (d) \ ( I (b )) =
1– sin x 1 – sin x cos x 1 – sin x dx b+1
Sol. Differentiating both the sides w.r.t. x, we get Integrating both the sides w.r.t. b, we get
f ¢ ( x ) = cos x + f ¢ ( x )(2sin x –sin x ) 2 I (b ) = log (b + 1) + C …(i)
b
2
Þ (1 + sin x – 2sin x ) f ¢ ( x ) = cos x 1 x -1
Given, I (b ) = ò0 ln x
dx
cos x cos x
Þ f ¢( x ) = =
2
1 + sin x – 2sin x (1 –sin x )2 \ I ( 0) = 0 ( when b = 0) …(ii)
cos x Also, from Eq. (i), I (0) = log (1) + C
Integrating, we get f ( x ) = ò (1 –sin x )2 dx \ I ( 0) = C …(iii)
From Eqs. (ii) and (iii),C = 0
dt 1 1
Put 1–sin x = t , f ( x ) = –ò 2
= = +C Þ I (b ) = log (b + 1)
t t 1–sin x
Also, f (0) = 0, hence C = –1 y Example 77 Prove that
1 1 – 1 + sin x sin x p /2 dx p (a 2 + b 2 )
f (x ) = –1 = =
1 –sin x 1 –sin x 1 –sin x ò0 (a 2 sin 2 x + b 2 cos 2 x ) 2
=
4a 3 b 3
.
Hence, (b) is the correct answer.
p/ 2 1 dx
y Example 75 If F ( x ) = ò
x
f (t ) dt, where
Sol. Let I = ò0 a sin x + b 2 cos 2 x
2 2
1
p/ 2 sec 2 x ¥ dt
t2 1+ u 4 Then, I = ò0 dx = ò0
f (t ) = ò du, then the value of F ¢¢(2) equals to a tan 2 x + b 2
2
a2 t 2 + b2
1 u
(where, t = tan x )
7 15 15 17
(a) (b) (c) 257 (d) ¥
4 17 17 68 1 é æ at ö ù 1 æ p ö p
Þ I = tan - 1 ç ÷ = ç -0 ÷ =
ab êë è b ø úû 0 ab è 2 ø 2ab
1 + t 8 × 2t 2 1+t8
Sol. Here, f ¢ (t ) = 2
= ...(i) p/ 2 1 p
x
t t Thus, ò0 2 2
a sin x + b cos x 2 2
dx =
2ab
= f (a, b ) …(i)
Now, F(x ) = ò1 f (t )dt Þ F ¢ ( x ) = f ( x )
Chap 02 Definite Integral 113

Differentiating both the sides w.r.t. ‘a’, we get p


\ f ¢( x ) =
p/ 2 - 2a sin x 2
p 2 1+ x
ò0 (a 2 sin 2 x + b 2 cos 2 x )2
dx = -
2a 2b Þ f (x ) = p 1 + x + C
2
p/ 2 sin x p where, f (0) = 0 Þ C = - p
Þ ò0 (a 2 sin 2 x + b 2 cos 2 x )2
dx =
4a 3b
…(ii)
Þ f ( x ) = p 1 + x - p = p ( 1 + x - 1)
Similarly, by differentiating Eq. (i) w.r.t. ‘b’, we get Hence, (a) is the correct answer.
p/ 2 cos 2 x p
ò0 (a 2 sin 2 x + b 2 cos 2 x )2
dx =
4ab 3
…(iii) y Example 79 Let f ( x ) be a continuous functions for
x 2 sec 2 t
all x, such that ( f ( x )) = ò f (t ) ×
2
dt and
Adding Eqs. (ii) and (iii), we get 0 4 + tan t
p/ 2 (sin 2 x + cos 2 x ) dx p p f (0) = 0, then
ò0 2 2
(a sin x + b cos x ) 2 2 2
=
4a b 3
+
4ab 3 æpö 5 æpö 3
(a) f ç ÷ = log (b) f ç ÷ =
è4ø 4 è4ø 4
p/ 2 dx p
Þ ò0 2 2
(a sin x + b cos x ) 2 2 2
=
4a 3b 3
(a 2 + b 2 ) æpö
(c) f ç ÷ = 2 (d) None of these
è 2ø
y Example 78 The value of x 2 sec 2 t
2
p / 2 log (1 + x sin q )
Sol. Here, ( f ( x ))2 = ò0 f (t ) ×
4 + tan t
dt ,

ò0 sin 2 q
dq ; x ³ 0 is equal to On differentiating both the sides w.r.t. x, we get
2 sec 2 x
(a) p ( 1 + x - 1) (b) p ( 1 + x - 2) 2f ( x ) . f ¢( x ) = f ( x ) ×
4 + tan x
(c) p ( 1 + x - 1) (d) None of these
sec 2 x
p/ 2 log (1 + x sin 2 q ) Þ f ¢( x ) =
4 + tan x
Sol. Given, f ( x ) = ò0 sin 2 q
dq ; x ³ 0
Integrating both the sides, we get
As above integral is function of x. Thus, differentiating both sec 2 x
the sides w.r.t. ‘ x ’, we get f (x ) = ò 4 + tan x
dx = log ( 4 + tan x ) + C
p/ 2 1 sin 2 q
f ¢( x ) = ò0 1 + x sin 2 q sin 2 q
× dq + 0 - 0 Since, f ( 0) = 0
Þ 0 = log ( 4 ) + C
(using Newton-Leibnitz’s formula) Þ C = - log 4
p/ 2 dq p/ 2 cosec 2 q dq \ f ( x ) = log ( 4 + tan x ) - log ( 4 )
= ò0 1 + x sin q 2
= ò0 cot 2 q + (1 + x ) æpö 5
Þ f ç ÷ = log ( 4 + 1) - log ( 4 ) = log
p/ 2 è4ø 4
æ 1 cot q ö p
= ç- × tan - 1 ÷ = Hence, (a) is the correct answer.
è 1+ x 1 + x ø0 2 1+ x
114 Textbook of Integral Calculus

Exercise for Session 5


10
1. The value of ò sgn ( x - [ x ])dx is equal to (where, [×] denotes the greatest integer function)
-1
(a) 9 (b) 10 (c) 11 (d) 12
[x ]
2. The value of ò ( x - [ x ])dx (where, [×] denotes the greatest integer function)
0
[x ]
(a) [x ] (b) (c) x [x ] (d) None of these
2
np - p / 4
3. The value of ò |sin x + cos x | dx is equal to
-p/ 4
1
(a) 2 2n (b) 2n (c) n (d) None of these
2 2
1
4. Let f ( x ) = x - [ x ] for every real number x (where, [×] denotes greatest integer function), then ò f ( x )dx is equal to
-1
1
(a) 1 (b) 2 (c) 0 (d)
2
x 1
5. If ò f (t )dt = x + f ò tf (t ) dt , then the value of f (1) is
0 x
1 1
(a) (b) 0 (c) 1 (d) -
2 2
x é 5p 4p ù
6. The least value of the function f( x ) = ò (3 sin t + 4 cos t )dt on the interval ê , is
5p / 4 ë 4 3 úû
3 3 1
(a) 3 + (b) - 2 3 + +
2 2 2
3 1
(c) + (d) None of these
2 2
x 2
7. The points of extremum of f( x ) = ò e -t /2
(1- t 2 ) dt are
1
(a) x = 1, - 1 (b) x = -1, 2 (c) x = 2, 1 (d) x = - 2, 1

8. If f ( x ) is periodic function, with period T, then


b b+T b b+T
(a) ò f (x )dx = òa f (x )dx (b) ò f (x )dx = òa + T f (x )dx
a a
b b b b + 2T
(c) ò f (x )dx = òa + T f (x )dx (d) ò f (x )dx = òa + T f (x )dx
a a
2
d e sinx 4 2e sin x
9. Let (F ( x )) = , x > 0. If ò dx = F (k ) - F (1), then one of the possible value of k is
dx x 1 x
(a) 4 (b) 8 (c) 16 (d) 32
x
10. Let f :(0, ¥ ) ® R and F ( x ) = ò f (t )dt . If F ( x 2 ) = x 2(1+ x ), then f (4) is equal to
0
5
(a) (b) 7 (c) 4 (d) 2
4

11. Let T > 0 be a fixed real number. Suppose f is continuous function such that f ( x + T ) = f ( x ) for all x Î R. If
T 3 + 3T
I = ò f ( x ) dx , then the value of ò f (2x ) dx is
0 3
3
(a) I (b) 2I (c) 3I (d) 6I
2
x
12. Let f ( x ) = ò 2 - t 2dt , then the real roots of the equation x 2 - f ¢ ( x ) = 0 are
1
1 1
(a) ±1 (b) ± (c) ± (d) ± 2
2 2
Chap 02 Definite Integral 115

x
13. Let f ( x ) be an odd continuous function which is periodic with period 2. If g( x ) = ò f (t ) dt , then
0
(a) g (x ) is an odd function (b) g (n ) = 0 for all n ÎN
(c) g (2n ) = 0 for all n ÎN (d) g (x ) is non-periodic
x
14. Let f ( x ) be a function defined by f ( x ) = ò t (t 2 - 3t + 2)dt , 1 £ x £ 3. Then, the range of f ( x ) is
1

(b) é - , 4ù (c) é - , 2ù
1 1
(a) [0, 2] (d) None of these
êë 4 úû êë 4 úû
cos x
2ò cos -1(t )
15. The value of lim 0
dt is
x ®0 2x - sin 2x
1 1 2
(a) 0 (b) (c) - (d)
2 2 3
x bt cos 4t - a sin 4t a sin 4x
16. If ò dt = for all x ¹ 0, then a and b are given by
0 t2 x
1
(a) a = ,b = 1 (b) a = 2,b = 2
4
(c) a = -1,b = 4 (d) a = 2,b = 4
x 1
17. If f ( x ) = ò {f (t )} -1dt and ò {f (t )} -1 = 2, then
0 0

(a) f (x ) = 2x (b) f (x ) = 2 loge x


(c) f (x ) = 3x - 1 (d) None of these
x
18. Let f be a real valued function defined on the interval ( -1, 1) such that e - x f ( x ) = 2 + ò t 4 + 1dt , for all x Î ( -1, 1)
0
and let f -1 be the inverse of f . Then, (f -1)¢ (2) is equal to [IIT JEE 2010]
(a) 1 (b) 1/3
(c) 1/2 (d) 1/ e

n
Directions (Q. Nos. 19 to 20) Consider the function defined on [0, 1] ® R
sin x – x cos x
f (x) = , if x ¹ 0 and f (0) = 0.
x2 [IIT JEE 2012]
1
19. ò0 f ( x ) dx is equal to
(a) 1– sin (1) (b) sin (1) – 1 (c) sin (1) (d) - sin (1)
1 t
20. lim
t®0 t2 ò0 f ( x ) dx is equal to
(a) 1/3 (b) 1/6 (c) 1/ 12 (d) 1/24
Session 6
Integration as Limit of a Sum, Applications of Inequality
Gamma Function, Beta Function, Walli’s Formula

Integration as Limit of a Sum Some Important Results


to Remember
Applications of Inequality n

and Gamma Integrals (i) S r = n (n + 1)


r =1 2
b
An alternative way of describing ò f ( x ) dx is that the n
n (n + 1) (2n + 1)
b
a (ii) å r2 =
6
definite integral ò f ( x ) dx is a limiting case of r =1
a
summation of an infinite series, provided f ( x ) is
n
n 2 (n + 1) 2
n -1 (iii) å r3 =
S 4
b
continuous on [a, b ], i.e. ò f ( x ) dx = lim h r =1 ì a (r n - 1)
a n® ¥ r =1 ï ,|r | >1
b -a
f (a + rh ), where h = × The converse is also true, ie, if ï (r - 1)
ï
n (iv) In GP, sum of n terms, S n = í an, r =1
we have an infinite series of the above form, it can be ï a (1 - r n )
expressed as definite integral. ï ,|r | <1
ïî (1 - r )
Method to Express the Infinite (v) sin a + sin(a + b) + sin(a + 2 b) +¼+ sin[a + (n - 1) b]
Series as Definite Integral sin n b / 2
= × sin [a + (n - 1) b / 2 ]
1 æ r ö sin b / 2
(i) Express the given series in the form S f ç ÷×
n è n ø (vi) cos a + cos(a + b) + cos(a + 2 b) +¼+ cos [a + (n - 1)b]
(ii) Then, the limit is its sum when n ® ¥,
1 ær ö sin n b / 2
i.e. lim S f ç ÷ × = × cos [a + (n - 1) b/ 2 ]
n® ¥ n ènø sin b / 2
r 1 p2
(iii) Replace by x and by (dx ) and lim S by the sign of ò . (vii) 1 -
1
+
1
-
1
+
1
-
1
+¼ ¥ =
n n n ®¥
22 32 42 52 62 12
(iv) The lower and the upper limit of integration are
r 1 1 1 1 1 p2
limiting values of for the first and the last term of r (viii) 1 + + + + + +¼ ¥ =
n 22 32
52 62 42 6
respectively.
1 p21
Some particular cases of the above are (ix) 1 + + +¼ ¥ =
n 32 52 8
1 ær ö
(a) lim å f ç ÷ 1 1 1 p2
n® ¥
r =1 n
ènø (x) + + +¼ ¥ =
n -1 22 42 62 24
1 ær ö 1
or lim
n® ¥
å f ç ÷ = ò f ( x ) dx
n ènø 0 (xi) cos q =
e iq + e - iq
, and sin q =
e iq - e - iq
r =1
Pn
2 2i
1 ær ö b
(b) lim
n® ¥
å f ç ÷ = ò f ( x ) dx
è ø a (xii) cos h q =
eq +e-q
and sin h q =
eq -e-q
r =1 n n
r 2 2
where, a = lim = 0 ( as r = 1)
n ®¥ n Remark
r The method of evaluate the integral, as limit of the sum of an
and b = lim = p ( as r = pn ) infinite series is known as integration by first principles.
n®¥ n
Chap 02 Definite Integral 117

n -1
y Example 80 Evaluate the following : 1
æ 1 2 3 n -1 ö
y Example 81 Evaluate S = å as n ® ¥.
(i) lim ç 2 + 2 + 2 +¼ + 2 ÷ r=0 4n - r 2
2
n® ¥ è n n n n ø n -1 n -1
1 1
æ 1 1 1 ö
Sol. Let Sn = å 2 2
= å
(ii) lim ç + +¼ + ÷ r =0 4n - r r =0 n 4 - (r / n )2
n® ¥ è n +1 n +2 2n ø n -1
1 1
æ
= å
n n n ö n r =0 4 - (r / n )2
(iii) lim ç 2 + +¼ + ÷
n ® ¥ è n + 12 n 2 + 22 2n 2 ø n -1
1 1
P P P
Hence, S = lim Sn = lim
n ®¥ n ®¥ n
å
(1 + 2 +¼ + n ) r =0 4 - (r / n )2
(iv) lim ,P >0
nP +1
1
n® ¥ 1 dx é x ù
= ò0 = ê sin - 1 ú
Sol. Let 4 - x2 ë 2 û 0
1 2 3 n -1 æ1ö p p
(i) Sn = + + +¼+ Þ S = sin -1 ç ÷ = \ S=
n2 n2 n2 n2 è2ø 6 6
n
r -1 n
1 æ r -1 ö
= å n2
= å ç
n è n ø
÷
y Example 82 Evaluate
r =2 r =2
n
1 ær ö æ 1 ö æ 1 1 1 ö
Þ S = lim
n®¥
å n
ç ÷
èn ø
ç as n ® ¥ Þ ® 0÷
è n ø
lim ç +
n ® ¥ è 2n + 1 2n + 2
+¼+ ÷ ×
6n ø
r =2

1 1 æ 1 1 1ö
= ò0 x dx =
2
Sol. Let Sn = ç +
è 2n + 1 2n + 2
+¼+ ÷
6n ø
n
1 1 1 1
= å
4n 4n
(ii) Let Sn = + +¼+ 1 1 1
n +1 n +2 2n r = 1 n + r = å 2n + r
= å ×
n 2 + (r / n )
r =1 r =1
n
1
= å æ rö Þ S = lim Sn = ò0
4 dx
= [ln | 2 + x | ] 40
r =1 n ç1 + ÷ n ®¥ 2+ x
è nø
1 n
1 = log 6 - log 2 = log 3
Hence, S = lim Sn = lim
n®¥ n®¥ n
å r \ S = ln 3
r =1 1+
n 4
dx
=
1
ò0 1 + x = [log (1 + x )]0
1
= log 2 y Example 83 Evaluate ò1 (ax 2+ bx + c ) dx from the

n n n first principle.
(iii) Let Sn = 2 + +¼+ 2 4 -1 3
n + 12 n 2 + 22 2n Sol. Let x = 1 + rh , where h = =
n n
n n
n 1 As x ® 1, r ® 0 and x ® 4, r ® n
= å n +r2 2
= å n (1 + r 2 / n 2 ) 4 n
r =1 r =1
n
\ ò1 (ax 2 + bx + c ) dx = lim
n ®¥
åhf (1 + rh )
1 1 n r =0
Hence, S = lim Sn = lim
n ®¥ n ®¥ n
å r2
= lim å h [a (1 + rh )2 + b (1 + rh ) + c ]
n®¥
r =1
1+ r =0
n2
3 é æ ù
n 2
3r ö æ 3r ö
å
1
1 dx é ù p = lim êa ç1 + ÷ + b ç1 + ÷ + c ú
= ò0 = ê tan - 1 xú = n®¥
r =0 n ë ê è n ø è n ø úû
û0 4
2
1+ x ë
n
1 é æ 9r 2
6r ö æ 3r ö ù
= 3 lim å
P
1P + 2P + ¼ + n P rP êa ç1 + 2 + ÷ + b ç1 + ÷ + c ú
n n
1 ær ö
(iv) Let Sn =
nP + 1
= å nP + 1
= å n çè n ÷ø n ®¥
r =0 n ë ê è n nø è nø úû
r =1 r =1
1 é æ 9n (n + 1) (2n + 1) 6n (n + 1) ö
n P = 3 lim a çn + + ÷
ær ö n ®¥ n ê
1
\ S = lim Sn = lim å çè n ÷ø ë è 6n 2 2n ø
n®¥ n®¥ n r =1 æ 3n (n + 1) ö ù
+ b çn + ÷ + cn ú
1 è 2n ø û
1 éxP +1ù 1
= ò0 x P dx = ê
P + 1
ú =
P +1
15
= 3 [a (1 + 3 + 3) + b (1 + 3 / 2) + c ] = 21a + b + 3c
ë û0 2
118 Textbook of Integral Calculus

y Example 84 The value of 1 2n r


1/n y Example 86 The value of lim ×å is
æ p 2p 3p (n - 1) p ö n ®¥ n 2
r =1 n + r
2
lim ç sin × sin × sin ¼ sin ÷ is equal to
n®¥ è 2n 2n 2n n ø equal to
1 1 1 (a) 1 + 5 (b) -1 + 5 (c) -1 + 2 (d) 1 + 2
(a) (b) (c) (d) None of these
2 3 4
1 2n r
ì p 2p 3p 2 ( n - 1) p ü
1/n
Sol. We have, lim ×å
Sol. Let A = í lim sin × sin × sin ¼ sin ý n ®¥ n 2
r =1 n + r
2
în ® ¥ 2n 2n 2n 2n þ
r
1 æ p 2p 2 ( n - 1) p ö 1 2n
\ log A = lim log çsin × sin ¼ sin ÷ = lim × å n =ò
2 x
dx = ( x 2 + 1 )20 = 5 - 1
n ®¥ n è 2n 2n 2n ø n ®¥ n 2 0 2
2 (n - 1) r =1 r 1+ x
1 rp 1+ 2
= lim
n ®¥ n
å log sin
2n
n
r =1
Hence, (b) is the correct answer.
2 æ px ö
= ò0 log sin ç ÷ dx
è 2 ø
é 1 ær ö b ù
ê using nlim
ë ®¥
S f ç ÷ = ò f ( x ) dx ú
n èn ø a
û Applications of Inequality
p 2 é px ù Sometimes you are asked to prove inequalities involving
= ò0 log (sin t ) × dt
p êë putting 2 = t úû definite integrals or to estimate the upper and lower
2 ×2 p/ 2 boundary values of definite integral, where the exact value
=
p ò0 log (sin t ) dt
of the definite integral is difficult to find. Under these
é 2a a ù circumstances, we use the following types
ê using ò0 f ( x ) dx = 2 ò0 f ( x ) dx , if f (2a - x ) = f ( x ) ú Type I. If f ( x ) is defined on [a, b ], then
ë û
4ì p üé p/ 2 p ù ½ b f ( x ) dx ½ £ b | f ( x ) | dx .
½ òa ½ òa
= í - log 2 ý ê using ò log (sin x )dx = - log 2ú
pî 2 þë 0 2 û
= - 2 log 2 Equality sign holds, where f ( x ) is entirely of the same
\ log A = log (1 / 4 ) Þ A = 1 / 4 sign on [a, b ].
Hence, (c) is the correct answer.

y Example 85 The interval [0, 4] is divided into n y Example 87 Estimate the absolute value of the
19 sin x
equal sub-intervals by the points x 0 , x 1 , x 2 , ..., x n –1 , x n integral ò dx.
10 1 + x 8
where 0 = x 0 < x 1 < x 2 < x 3 < ... < x n = 4 .
½ 19 sin x ½ ½ sin x ½
½ 19
n Sol. To find, I =½ ò dx ½ £ ò10 ½dx …(i)
If dx = x i – x i –1 for i = 1, 2, 3, ..., n, then lim
dx ® 0
å x i dx is ½ 10 1 + x 8
½
8
½1 + x ½
i =1 (using type I)
equal to
32 Since, | sin x | £ 1 for x ³ 10
(a) 4 (b) 8 (c) (d) 16
3 ½ sin x ½ 1
The inequality ½ ½£ …(ii)
Sol. lim dx ( x 1 + x 2 + x 3 +...+ x n ) 8 8
½1 + x ½ |1 + x |
dx ® 0

4 é 4 8 12 nù æ 4ö Also, 10 £ x £ 19 Þ 1 + x 8 > 108


= lim + + +......+ 4 × ú çQ dx = ÷
n ®¥ n ê
ë n n n nû è nø 1 1 1
Þ < or < 10- 8 …(iii)
1 + x8 108 |1 + x8 |
½ sin x ½ ½ < 10- 8 is fulfilled.
From Eqs. (ii) and (iii), ½ 8
½ 1 + x ½
½ 19 sin x ½ 19
h ½
ò dx ½ < ò10 10- 8 dx
10 1 + x 8
O x1 x2 x3 xn–1 xn=4 ½ ½
δx δx ½ 19 sin x ½
16 16 n (n + 1) \ ½
ò dx ½ < (19 - 10) . 10- 8 < 10- 7
10 1 + x 8
= lim 2 (1 + 2 + 3+...+ n ) = lim 2 × =8 ½ ½
n ®¥ n n ®¥ n 2 (Q the true value of integral » 10- 8 )
Hence, (b) is the correct answer.
Chap 02 Definite Integral 119

8
y Example 88 The minimum odd value of ‘ a ’ (a > 1) for ò4 f ( x ) dx = 0, which shows f ( x ) can’t have any value
19 sin x 1 greater than zero in the sub-interval [4, 8].
which ò a
dx < , is equal to
10 1 + x 9 Þ f ( x ) is constant in the sub-interval [4, 8] and has to be
zero at all points, x Î[ 4, 8] .
(a) 1 (b) 3 (c) 5 (d) 9
Þ f ( x ) = 0, " x Î [ 4, 8]
19 sin x dx 19 dx Þ f ( 6) = 0
Sol. Let I = ò10 1+ x a
< ò10 1 + xa
æ sin x 1 ö Type IV. For a given function f ( x ) continuous on[a, b ], if
çQsin x < 1 Þ a
< ÷ you are able to find two continuous functions f 1 ( x ) and f 2 ( x )
è 1+ x 1 + xa ø
19 dx 19 dx on[a, b ]such that f 1 ( x ) £ f ( x ) £ f 2 ( x ), " x Î[a, b ], then
\ I < ò10 1 + xa
< ò10 1 + 10a b b b

(Q 10 < x < 19 Þ 10a + 1 < 1 + x a < 19a + 1)


òa f 1 ( x ) dx £ ò
a
f ( x ) dx £ ò
a
f 2 ( x ) dx .

9 p 1 dx p
Þ I < a
y Example 91 Prove that £ò £ ×
1 + 10 6 0
4-x -x 2 3 4 2
9 1
\ a
< Þ 1 + 10a > 81 or 10a > 80 i.e. a = 2, 3, 4, 5, ¼ Sol. Since, 4 - x 2 ô4 - x 2 - x 3 ô4 - 2x 2 > 1, " x Î [0, 1]
1 + 10 9
\ Minimum odd value of ‘a’ is 3. 4 - x 2 ô 4 - x 2 - x 3 ô 4 - 2x 2 > 1, " x Î [0, 1]
Hence, (b) is the correct answer. 1 1 1
Þ £ £ , " x Î [0, 1]
Type II. 4-x 2
4-x -x 2 3
4 - 2x 2
½ b f ( x ) g( x ) dx ½ £ æç b
f 2 ( x ) dx ö÷ æç
b
g 2 ( x ) dx ö÷ ,
½ òa òa òa 1 dx 1 dx
½ è øè ø Þ ò0 4-x 2
£ ò0 4 - x2 - x3
2 2
where f ( x ) and g ( x ) are integrable on [a, b ]. 1 1
1
£ ò0 4 - 2x 2
dx , " x Î [0, 1]
y Example 89 Prove that ò0 (1 + x ) (1 + x 3 ) dx cannot 1
1 æ
1
æ -1 x ö 1 dx -1 x ö
exceed 15 / 8 . Þ ç sin
è
÷ £
2 ø0 ò0 4 - x2 - x3
£ ç sin
2 è
÷
2 ø0
(1 + x )(1 + x 3 )dx £ æç ò (1 + x )dx ö÷ æç ò (1 + x 3 ) dx ö÷
1 1 1
Sol. ò p 1 dx p
0 è 0 øè 0 ø Þ
6
£ ò0 4-x -x 2 3
£
4 2
1 1
æ x2 ö æ x4 ö Type V. If m and M be global minimum and global
£ çx + ÷ çx + ÷
è 2 ø0 è 4 ø0 maximum of f ( x ) respectively in[a, b ] , then
b
3 5 15 m (b - a ) £ ò f ( x ) dx £ M (b - a ).
£ × £ a
2 4 8
Proof We have, m £ f ( x ) £ M for all x Î[a, b ]
Type III. If f ( x )ôg( x ) on [a, b ], then b b b

òa
b
f ( x ) dx ³ ò
b
g( x ) dx . In particular, if f ( x ) ³ 0,
Þ òa m dx £ ò
a
f ( x ) dx £ ò
a
M dx
a
b
b Þ m (b - a ) £ ò f ( x ) dx £ M (b - a )
then ò f ( x ) dx ô0. a
a
3
y Example 90 If f ( x ) is a continuous function such y Example 92 Prove that 4 £ ò 3 + x 3 dx £ 2 30.
1
8
that f ( x )ô0, " x Î [2, 10] and ò4 f ( x ) dx = 0, then find Sol. Since, the function f ( x ) = 3 + x 3 increases
f (6 ). monotonically on the interval [1, 3].
Sol. f ( x ) is above the X -axis or on the X -axis for all x Î[2, 10]. \ M = Maximum value of 3 + x 3 = 3 + 33 = 30
If f ( x ) is greater than zero at any sub-interval of [4, 8],
8
then ò f ( x ) dx must be greater than zero. But and m = Minimum value of 3 + x 3 = 3 + 13 = 2
4
120 Textbook of Integral Calculus

\ m = 2, M = 30, b - a = 2 (ii) If n Î N , then G(n + 1) = (n ) !


3 (iii) G(1 / 2 ) = p
Hence, 2×2 £ ò1 3 + x 3 dx £ 2 30 æm + 1ö æ n + 1ö
Gç ÷Gç ÷
3 p /2 è 2 ø è 2 ø
Þ 4£ ò1 3 + x 3 dx £ 2 30 (iv) ò sinm x × cos n x dx =
0 æm + n + 2 ö
2G ç ÷
1 2 è 2 ø
y Example 93 Prove that 1 £ ò e x dx £ e. p
0 (v) Gn G (1 - n ) = ,0 <n <1
0 x2 sin np
Sol. For 0 £ x £ 1, we have e £ e £ e1
1 2 æ 1ö p
\ e 0 ( 1 - 0) £ ò0 e x dx £ e 1 (1 - 0) (vi) G m G çm + ÷ = G 2m
è 2ø 2 -1
2m
1 2
Þ 1£ ò0 e x dx £ e n -1
1 æ 2 ö æ n -1 ö (2 p ) 2
(vii) G Gç ÷ KG ç ÷=
n è n ø è n ø n 1 /2
Gamma Function y Example 95 Evaluate ò0
¥
e - x x 3 dx .
If n is a positive number, then the improper integral
¥ Sol. By definition of Gamma function,
ò0 e - x x n - 1 dx is defined as Gamma function and is ¥ ¥
ò0 e - x x 3 dx = ò0 e - x x 4 - 1 dx = G 4
denoted by Gn.
¥ ¥
i.e. Gn = ò e - x x n - 1 dx , where x Î Q + . \ ò0 e - x x 3 dx = 6
0
n -1
y Example 94 Evaluate 1 æ 1 ö
(i) G1 (ii) G2
y Example 96 Evaluate ò0 ç log ÷
è x ø
dx .

¥ b n -1
e - x . x 1 - 1 dx = lim e - x dx æ 1 ö
ò0 ò0
1
Sol. (i) G1 =
b® ¥
Sol. Let I = ò0 ç log ÷
è x ø
dx
b
é ù 1
= lim ê - e - x ú = lim ( - e - b + e 0 ) Put log = t Þ dx = - e -t dt
b® ¥ ë û b® ¥ x
0
0 ¥
=0+1=1 \ I = ò¥ t n - 1 ( - e - t ) dt = ò0 e - t t n - 1 dt
¥ b
(ii) G2 = ò0 e - ¥ x 2 - 1 dx = lim ò0 e - x × x dx n -1
b® ¥ 1 æ 1 ö
b
\ ò0 ç log ÷
è x ø
dx = Gn
é ù
= lim ê - x e - x - e - x ú
b® ¥ ë û0
= lim [( - b e - b - e - b ) - (0 - 1)]
b® ¥
Beta Function
é -b 1 ù - ( b + 1)
= lim ê b - b + 1 úû = blim +1 The beta function is
b® ¥ ë e e ®¥ eb 1
B (m, n ) = ò x m - 1 (1 - x ) n - 1 dx ,
æ 1 ö 0
= ç lim - b ÷ + 1 (using L’Hospital’s rule)
è b® ¥ e ø where m, n > 0
=1
Properties of Beta Function
Properties of Gamma Function (i) B (m, n ) = B (n, m ), where m, n > 0
Gamma function has following properties : GmG n
(ii) B (m, n ) = , where m, n > 0
(i) G 1 = 1, G 0 = ¥ and G (n + 1) = n Gn G (m + n )
e.g. G 5 = 4 G 4 = 4 ´ 3 G 3 = 4 ´ 3 ´ 2 G 2 ¥ x m -1
(iii) B (m, n ) = ò dx
= 4 ´ 3 ´ 2 ´ 1 G1 = 4 ´ 3 ´ 2 ´ 1 0
(1 + x ) m + n
Chap 02 Definite Integral 121

1 p/ 2 p/ 2

ò0 x ò0 sinn x dx = ò0 cos n x dx
6
y Example 97 Evaluate (1 - x 2 ) dx .
n-1 n- 3 n-5 3 1 p
= × × ¼ × × , n is even.
1 n n-2 n-4 4 2 2
ò0
6 2
Sol. Let, I = x (1 - x ) dx
n-1 n- 3 n-5 4 2
= × × ¼ × × 1, n is odd.
Put let x 2 = t n n-2 n-4 5 3
Þ 2x dx = dt p /2
1 1
y Example 98 Evaluate ò0 sin 4 x × cos 6 x dx.
ò0
5/2
\ I = t (1 - t ) dt
2 Sol. Using Gamma function, we have
1 æ 7 3 ö 1 G 7 /2G 3/2 p/ 4 G ( 5 / 2) G ( 7 / 2) G ( 5 / 2) G ( 7 / 2)
ò0 sin x × cos x dx = æ 4 + 6 + 2 ö =
4 6
I = Bç , ÷= ×
2 è 2 2 ø 2 æ3 7 ö 2 G6
Gç + ÷ 2 Gç ÷
è2 2 ø è 2 ø
5 3 1 1 æ 3 1 öæ 5 3 1 ö
× × × p× × p ç ´ ´ G ( 1 / 2) ÷ ç ´ ´ ´ G ( 1 / 2) ÷
1 2 2 2 5p è 2 2 øè 2 2 2 ø 3p
= 2 = = =
2 4 ×3×2×1 256 2 ´ 5! 512
¥ 2 2
y Example 99 The value of ò0 e -a x
dx is equal to
Walli’s Formula p p
p /2 (a) (b)
An easy way to evaluate ò sin m n
x × cos x dx, where 2a 2a
0
p
m, n Î I + . (c) (d) None of these
p /2 p /2 2a
We have, ò sinm x × cos n x dx = ò sinn x × cos m x dx
0 0 ¥ 2 2
Sol. Let I = ò0 e-a x
dx
(m - 1) (m - 3) ¼ (1 or 2) . (n - 1) (n - 3) ¼ (1 or 2) p
= × , ¥ 1 1 ¥
(m + n ) (m + n - 2) ¼ (1 or 2) e-t × e - t × t - 1/ 2 dt
2 \ I = ò0 2a t
dt =
2a ò0
when both m and n Îeven integer.
(m - 1) (m - 3) ¼ (1 or 2) × (n - 1) (n - 3) ¼ (1 or 2) (put a 2 x 2 = t Þ 2a 2 x dx = dt )
= ,
(m + n ) (m + n - 2) ¼ (1 or 2) 1 ¥ 1/ 2 -1 - t ¥
ò= t × e × dt (using ò e - x x n - 1dx = Gn )
when either of m or n Îodd integer. 2a 0 0

1 1 1 1
Remark = G = p (using G = p)
2a 2 2a 2
If n be a positive integer, then
Hence, (a) is the correct answer.
122 Textbook of Integral Calculus

Exercise for Session 6


p/ 2 log (1 + x sin2 q)
1. The value of f ( x ) = ò dq; x ³ 0 is equal to
0 sin2 q
1
(a) ( 1 + x - 1) (b) p ( 1+ x - 1) (c) p ( 1+ x - 1) (d) None of these
p
1 n
æ r ö
2. The value of lim
n®¥n
å ç ÷ is equal to
èn + r ø
r -1
(a) 1 - log 2 (b) log 4 - 1 (c) log 2 (d) None of these
1
3. The value of lim { (n + 1) (n + 2) (n + 3) ¼ (n + n ) }1/ n is equal to
n®¥ n
e 4
(a) 4e (b) (c) (d) None of these
4 e
b
4. If m , n Î N, then the value of ò ( x - a )m (b - x )n dx is equal to
a

(b - a )m + n × m ! n ! (b - a )m + n + 1 × m ! n !
(a) (b)
(m + n ) ! (m + n + 1) !
(b - a )m × m !
(c) (d) None of these
m!
2 n4 + 1
æ n ! ö 5n5 +1
5. The value of lim ç n ÷
è n ø
is equal to
n®¥
2

(c) æç ö÷
2 1 5
(a) e (b) (d) None of these
e èe ø
ì 1 1 ü
6. The value of lim n í 2 + 2
+ ¼ + to n termsý is equal to
n®¥
î 3n + 8 n + 4 3n + 16 n + 16 þ
log æç ö log æç ö÷
1 9 1 9
(a) ÷ (b)
2 è 5 ø 3 è 5ø
1
(c) log çæ 9 ö
÷ (d) None of these
4 è 5 ø
1 ì 3 p 2p np ü
7. The value of lim ísin + 2 sin3 + ¼ + n sin3 ý is equal to
n®¥ n2 î 4n 4n 4n þ
2 2
(a) (52 - 15n ) (b) (52 - 15n )
9p 2 9p 2
1
(c) (15n - 15) (d) None of these
9p 2
p/ 2
8. The value of f (k ) = ò log (sin2 q + k 2 cos 2 q) dq is equal to
0
(a) p log (1 + K) - p log 2 (b) p log 2 - log (1 + K) (c) log (1 + K) - p log 2 (d) None of these
1
9. If m , n Î N, then I m, n = ò x (1 - x ) dx is equal to
m n
0
m !n ! 2m ! n ! m !n !
(a) (b) (c) (d) None of these
(m + n + 2) ! (m + n + 1) ! (m + n + 1) !
p sin2 nq
10. The value of I (n ) = ò dq is (" n Î N )
0 sin2 q
np np
(a) np (b) (c) (d) None of these
2 4
JEE Type Solved Examples :
Single Option Correct Type Questions
¥ p l Ex. 3 The true set of values of ‘a’ for which the inequal-
l Ex. 1 If ò f ( x )dx = and f ( x ) is an even function, 0
0 2 ity ò (3 – 2 x – 2 × 3 –x ) dx ³ 0 is true, is
¥ æ 1ö a
then ò f ç x - ÷ dx is equal to (b) (–¥,–1] (c) [0, ¥ ) (d) (–¥, –1] È [0, ¥ )
0 è xø (a) [0, 1]
0

(a)
p
(b)
p
(c) p (d) None of these
Sol. We have, òa 3 – x (3 – x – 2 ) dx ³ 0
4 2 Put 3 – x = t Þ 3 – x ln 3 dx = –dt
¥ p 3–a
Sol. Here, ò0 f (x )dx = 2 Þ
3 -a ét2
ln 3 ò (t – 2 ) dt ³ 0 = ê – 2t ú
ù
³0
1 1
ë 2 û1
Put x = t -
t æ 3 –2a ö æ 1 ö
Þ ç – 2 × 3 –a ÷ – ç – 2 ÷ ³ 0
Þ
æ 1ö
dx = ç1 + 2 ÷dt è 2 ø è2 ø
è t ø
¥ ¥ æ 1ö æ 1ö
\ ò0 f (x )dx = ò1 f çèt - t ÷ø × çè1 + t 2 ÷ødt 3
Þ 3 –2a – 4 ´ 3 –a + 3 > 0 Þ (3 –a – 3 )(3 –a – 1 ) > 0
¥ æ 1ö ¥ æ 1ö 1
= ò f çt - ÷dt + ò f çt - ÷ × 2 dt Þ 3 –a > 31 Þ a < 1 or 3 –a < 3 0 Þ a > 0
1 è t ø 1 è tø t
0 æ1 ö æ Thus, a Î(– ¥,– 1 ] È [ 0, ¥ )
¥ æ 1ö 1ö
= ò f çt - ÷dt + ò f ç - y ÷ × ( -dy ) ç put t = ÷ Hence, (d) is the correct answer.
1 è tø 1 èy ø è yø
¥ æ 1ö 0 æ 1ö l Ex. 4 The value of the definite integral
= ò f çt - ÷dt - ò f çy - ÷dy as f ( x ) is even
1 è tø 1 è yø 2np

¥ æ 1ö 1 æ 1ö ¥ æ 1ö
ò0 max(sin x , sin –1 (sin x ))dx equals to (where, n Î I )
=ò f çt - ÷dt + ò f çt - ÷dt = ò f çt - ÷dt
1 è tø 0 è t ø 0 è tø n( p 2 – 4 ) n( p 2 – 4 )
(a) (b)
¥ æ 1ö ¥ p 2 4
\ ò0 f çè x - x ÷ø dx = ò0 f (x )dx = 2 n( p 2 – 8 ) n( p 2 – 2)
(c) (d)
4 4
l Ex. 2 The value of 2np
Sol. Let I = ò max (sin x, sin –1(sin x )) dx
1 æ n ö æ n 1 ö 0

ò0 è Õ
ç
r =1
( x + r ) ÷ çç å ÷ dx equals to
ø è k = 1 x + k ÷ø
Y

π/2
(a) n (b) n! (c) (n +1)! (d) n × n !
n 1 x π–x
Sol. The given integrand is perfect coefficient of Õ (x + r ) π–x
r =1 O
1 X
é n ù –π/2 π/2 π 3π/2 2π
\ I = ê Õ ( x + r ) ú = (n + 1 )!– n ! = n × n !
ë r =1 û0 –1
Aliter ( x + 1 )( x + 2 )( x + 3 )...( x + n ) = et
–π/2
So that, when x = 0, then t = ln n! when x = 1, then t = ln (n + 1 )!
[ln( x + 1 ) + ln( x + 2 ) + ....... + ln( x + n )] = t p /2 p 2p
=n éò x dx + ò ( p – x ) dx + ò (sin x ) dx ù
æ 1 1 1 ö ëê 0 p /2 p ûú
\ ç + + ... + ÷ dx = dt
è (x + 1) (x + 2) (x + n ) ø é p2 p2 1 æ 2 p2 ö ù
=n ê + – ç p – ÷ – 2ú
ln (n + 1)! (n + 1)! êë 8 2 2è 4 ø úû
Therefore, I =ò et dt = [et ]ln
ln n !
ln n !
ép 2 2
p 3p 2 ù n( p 2 – 8 )
=n ê + – – 2ú =
= e ln (n + 1)! – e ln n ! = (n + 1 )! – n ! = n × n !
ë 8 2 8 û 4
Hence, (d) is the correct answer. Hence, (c) is the correct answer.
124 Textbook of Integral Calculus

1 æ 1ö 1 Applying Leibnitz theorem, we get


– ç 1+ ÷
2è nø 2
l Ex. 5 lim n × (11 × 2 2 × 3 3 .....n n ) n is equal to 1 – ( f ¢( x )) 2 = f ( x )
n ®¥
1 1 Þ 1 – ( f ¢( x )) 2 = f 2( x )
(a) e (b) (c) 4
(d) 4 e
e e Þ ( f ¢( x )) 2 = 1 – f 2( x ) Þ f ¢( x ) = ± 1 - f 2( x )
1æ 1ö 1 dy
– ç1+ ÷
2è n ø 1 2 3 n n2 Þ = ± 1 – y 2 , where y = f ( x )
Sol. Let L = lim n × (1 × 2 × 3 ...n ) dx
n ®¥
dy
Þ = ± dx
1 æn + 1ö 1 n
\ ln L = lim– ç ÷ ln n + 2 å k ln k 1–y 2
n ®¥ 2 è n ø n k =1
Integrating both the sides, we get sin –1(y ) = ± x + C
1 æn + 1ö 1 n \ f (0) = 0 Þ C = 0
= lim – ç ÷ ln n + 2 å ( k ln k – k ln n + k ln n )
n ®¥ 2 è n ø n k =1 \ y = ± sin x
Þ y = sin x = f ( x ), given f ( x ) ³ 0 for x Î[ 0, 1 ]
1 æn + 1ö 1 n k ln n n
= lim– ç ÷ ln n + 2 å k ln + 2 å k Its known that, sin x < x, " x Î R +
n ®¥ 2 è n ø n k =1 n n k =1
æ1ö 1 æ1ö 1 æ1ö 1
1 æn + 1ö 1 n k k ln n n (n + 1 ) sin ç ÷ < Þ f ç ÷ < and sin ç ÷ <
= lim– ç ÷ ln n + å ln + 2 × è2ø 2 è2ø 2 è3ø 3
n ®¥ 2 è n ø n k =1 n n n 2
æ1ö 1
Þ fç ÷<
1 æn + 1ö 1 1 æn + 1ö è3ø 3
=– ç ÷ ln n + ò x ln x dx + ç ÷ ln n
è
2 n ø 0 2è n ø Hence, (c) is the correct answer.
1
1
=ò {
1 –
1 x t ln(1 + t )
0 {x dx = – 4 Þ L
x ln =e 4 l Ex. 8 The value of lim 3 ò0 dt
II I
x® 0 x t4 +4 [IIT JEE 2010]
Hence, (c) is the correct answer. 1 1 1
(a) 0 (b) (c) (d)
12 24 64
sec 2 x 1 x t log(1 + t )
l Ex. 6
ò
lim 2
f (t ) dt
is equal to
Sol. lim
x® 0x 3 ò0 4 + t4
dt


p p2
2 Using L’Hospital’s rule,
4 x –
16 [IIT JEE 2007] x log(1 + x )
4 log (1 + x ) 1 1
8 2 2 æ 1ö lim 4 + x2 = lim × =
(a) f ( 2) (b) f ( 2) (c) fç ÷ (d) 4 f ( 2) x ®0 3x x ®0 3x 4 + x 4 12
p p p è 2ø é log (1 + x ) ù
êë using xlim = 1ú
sec 2 x ®0 x û
ò
Sol. lim 2
f (t ) dt
= lim
f (sec 2x ) × 2 sec 2 x tan x - 0
Hence, (b) is the correct answer.
2

p p x®
p 2x
4 x2 – 4
16 (applying L’Hospital rule) 1 x 4 (1 – x ) 4
f (2 ) × 4 8 f (2 )
l Ex. 9 The value(s) of ò dx is (are)
= =
0
1+ x 2 [IIT JEE 2010]
p /2 p
22 2 71 3 p
Hence, (a) is the correct answer. (a) –p (b) (c) 0 (d) –
7 105 15 2
1 x 4 (1 – x ) 4
l Ex. 7 Let f be a non-negative function defined on the Sol. Let I = ò dx
x x 0 1 + x2
interval [0,1]. If ò 1 – ( f ¢ (t )) dx = ò 2
f (t ) dt ,0 £ x £ 1 and 1 ( x 4 – 1 )(1 – x ) 4 + (1 – x ) 4
0 0
=ò dx
f (0 ) = 0, then [IIT JEE 2009] 0 (1 + x 2 )
æ 1ö 1 æ 1ö 1 æ 1ö 1 æ 1ö 1
1 1 (1 + x 2 – 2x )2
(a) f ç ÷ < and f ç ÷ > (b) f ç ÷ > and f ç ÷ > = ò ( x 2 – 1 )(1 – x ) 4dx + ò dx
è 2ø 2 è3 ø 3 è 2ø 2 è3 ø 3 0 0 (1 + x 2 )
1ì 4x 2 ü
æ 1ö 1 æ 1ö 1 æ 1ö 1 æ 1ö 1 = ò í( x 2 – 1 )(1 – x ) 4 + (1 + x 2 ) – 4 x + ý dx
(c) f ç ÷ < and f ç ÷ < (d) f ç ÷ > and f ç ÷ < 0
î (1 + x 2 )þ
è 2ø 2 è3 ø 3 è 2ø 2 è3 ø 3
1æ 4 ö
x x = ò ç( x 2 – 1 )(1 – x ) 4 + (1 + x 2 ) – 4 x + 4 – ÷ dx
Sol. Given, ò0 1 – ( f ¢(t )) 2dt = ò f (t ) dt, 0 £ x £ 1
0
0è 1– x2 ø
Chap 02 Definite Integral 125

1æ 4 ö tan q
= ò ç x 6 – 4x 5 + 5x 4 + 4 –

÷ dx
1 + x2 ø
(c) sinq ò1 f ( x cos q )dx
1 sin q tan q
æp
tanq òsin q
1 4 5 4 ö 22 (d) f ( x )dx
= – + – + 4 – 4ç – 0÷ = – p
7 6 5 3 è 4 ø 7
cos q
Hence, (a) is the correct answer. Sol. Let, I = ò f ( x tan q)dx. Put x tan q = z sin q
sin q
1
cos q æ np ö \ I =ò f (z sin q) cos q dz
l Ex. 10 If òsin q f ( x tan q )dx ç q ¹ , n Î I ÷ is equal to
è 2 ø
tan q

Þ dx = cos q dz
tan q tan q tan q
(a) - cos q ò1 f ( x sinq )dx = - cos qò
1
f (z sin q) dz = - cos qò
1
f ( x sin q) dx
sin q
(b) - tanq òcos q f ( x )dx Hence, (a) is correct option.

JEE Type Solved Examples :


More than One Correct Option Type Questions
l Ex. 11 Let f ( x ) be an even function which is mapped Sol. Here, min {| x - n|, | x - (n + 1 )| } can be shown as
p
from ( -p, p ). Then, the value of ò æç ò f (t ) dt + [ f ( x )]ö÷ dx
x
y
-p è 0 ø
can be (where, [× ] denotes greatest integer function)
(a) 0 (b) p
(c) 2p (d) 4p
x ½
Sol. As, f ( x ) is an even function, then ò0 f (x ) dx is an odd function. x
O x x+1
Also, [ f ( x )] = 1, 2 [as 1 < f ( x ) < 3]
p æç x f (t ) dt ö÷dx + p [ f ( x )] dx n+1
g( x ) = ò
è ò0 ò-p
\ \ A1 = ò (min {| x - n|, | x - (n + 1 )| }) dx
-p ø n
p 1 1 1
= 0 + ò [ f ( x )] dx, [Q since [ f ( x )] = 1 or 2] = ´1 ´ =
-p 2 2 4
p n+ 2
= ò 1 dx Now, A2 = ò (| x - n| - | x - (n + 1 )| ) dx
-p n+1
p
or ò-p 2 dx = 2p or 4p 2
= ò (| t | - | t - 1 | )dt
1
[put x = n + t Þ dx = dt]
Hence, options (c) and (d) are correct. 2 2
= ò (t - (t - 1 ))dt = ò 1dt = (t )12 = 1
1 1
n +1
l Ex. 12 Let, A1 = ò (min {| x - n |, | x - (n + 1)|}) dx , and A3 = ò
n+ 3
(| x - (n + 4 )| - | x - (n + 3 )| )dx,
n n+ 2
n+2 3
A2 = ò (| x - n| -| x - (n + 1)|) dx , = ò (| t - 4 | - | t - 3 | )dt [put x = n + t Þ dx = dt]
n +1 2
3 3
n+3 = ò (( 4 - t ) - (3 - t ))dt = ò 1 dt = 1
A3 = ò (| x - (n + 4)| -| x - (n + 3)|) dx 2 2
n+2
1 9
Also, g( x ) = A1 + A2 + A3 = +1+1=
and g (x ) = A1 + A 2 + A 3 , then 4 4
100
9
(a) A1 + A 2 + A 3 = 9 (b) A1 + A 2 + A 3 =
4
\ å g( x ) = g(1 ) + g(2 ) + g(3 ) + ¼+ g(100 )
n =1
100 100
900
(c) å g(x ) =
4
(d) å g ( x ) = 300 =
9 9 9 900
+ + ¼+ =
4 4 4 4
n =1 n =1
126 Textbook of Integral Calculus

JEE Type Solved Examples :


Statement I and II Type Questions
n Directions (Q. Nos. 13 to 15) For the following questions, f (– x ) = –sin x + x cos x + C
choose the correct answers from the codes (a), (b), (c) and On adding, f ( x ) + f (– x ) = constant which need not to be zero.
(d) defined as follows. x
(a) Statement I is true, Statement II is also true; Statement II For Statement I f ( x ) = ò 1 + t 2dt; g( x ) = 1 + x 2
0
is the correct explanation of Statement I. -x
f (– x ) = ò 1 + t 2dt; t = –y
(b) Statement I is true, Statement II is also true; Statement II 0
is not the correct explanation of Statement I. x
f (– x ) = – ò 1 + y 2dy
0
(c) Statement I is true, Statement II is false.
\ f ( x ) + f (– x ) = 0
(d) Statement I is false, Statement II is true.
Þ f is odd and g is obviously even.
1 Hence, (c) is the correct answer.
l Ex. 13 Statement I If f ( x ) = ò ( x f (t ) +1) dt , then
0
3
Ex. 15 Given, f ( x ) = sin 3 x and P ( x ) is a quadratic
ò0 f ( x ) dx = 12
l

Statement II f ( x ) = 3 x +1 polynomial with leading coefficient unity.


1 2p
Sol. Let ò0 f (t )dt = k, so f (x ) = xk + 1 Statement I ò P ( x ) × f ¢¢( x ) dx vanishes.
0
1 k 2p
Now, ò0(kt + 1)dt = k Þ 2 + 1 = k, so k = 2 Statement II ò0 f ( x ) dx vanishes.
\ f (x ) = 2x + 1 Sol. P ( x ) = ax 2 + bx + c; f ( x ) = sin 3 x
3 2p
Also ò0 f (x )dx = 12 I =ò
0
( x ) × f ¢¢( x ) dx
P{
123
I II
Hence, (c) is the correct answer.
2p
Using I.B.P. P ( x ) × f ¢( x )| 20p – ò0 ¢(3
P12 x ) × f ¢( x ) dx
x 14 4244 3 123
l Ex. 14 Statement I The function f ( x ) = ò 2
1 + t dt is zero I II
0
2p
an odd function and g ( x ) = f ¢ ( x ) is an even function. = – é[ P ¢( x ) × f ( x )]20p – ò P ¢¢( x ) × f ( x ) dx ù
ëê 0 ûú
Statement II For a differentiable function f ( x ) if f ¢( x ) is
2p 2p
an even function, then f ( x ) is an odd function. = ò P ¢¢( x ) × f ( x ) dx = 2 ò sin 3 x dx = 0
0 0
Sol. If f ( x ) is of odd, then f ¢( x ) is even but converse is not true. Hence, (a) is the correct answer.
e.g. If f ¢( x ) = x sin x, then f ( x ) = sin x – x cos x + C

JEE Type Solved Examples :


Passage Based Questions
Passage I
(Q. Nos. 16 to 18)
b b –a
Suppose we define the definite integral using the following formula òa f ( x ) dx =
2
( f (a ) + f (b )), for more accurate result
c –a b –c
for c Î (a ,b ), F (c ) =( f (a ) + f (c ))+ ( f (b ) + f (c )).
2 2
a +b b b –a
When c = , then ò f ( x ) dx = ( f (a ) + f (b ) + 2 f (c )).
2 a 4
[IIT JEE 2007]
Chap 02 Definite Integral 127

p /2 Sol. F ¢(c ) = (b - a ) ¢(c ) + f (a ) - f (b )


l Ex. 16 ò0 sin x dx is equal to
F ¢¢(c ) = f ¢¢(c ) (b - a ) < 0
p p
(a) (1 + 2 ) (b) (1 + 2 ) Þ F ¢(c ) = 0
8 4
p p f (b ) - f (a )
(c) (d) Þ f ¢(c ) =
8 2 4 2 b -a
p æ æ p öö Hence, (a) is the correct answer.
p /2
–0 ç
æpö ç 0 + ÷÷ p
Sol. ò sin x dx = 2 çsin( 0 ) + sin ç ÷ + 2 sin ç 2 ÷ ÷ = (1 + 2 )
0 4 ç è2ø ç 2 ÷÷ 8
Passage II
è è øø (Q. Nos. 19 to 20)
Hence, (a) is the correct answer.
cos (a + b) - sin (a + b) cos 2b
Let f (a, b) = sin a cos a sin b .
l Ex. 17 If f ( x ) is a polynomial and if
-cos a sin a cosb
t (t – a )
òa f ( x ) dx – 2 ( f (t ) + f (a ))
lim = 0 for all a, then the l Ex. 19 The value of
t ®a (t – a ) 3
p /2 b æ æp ö æ 3p p öö
degree of f ( x ) can atmost be I =ò e ç f (0, 0 ) + f ç , b÷ + f ç , - b÷ ÷ db is
0 è è2 ø è 2 2 øø
(a) 1 (b) 2
(c) 3 (d) 4 (a) e p/ 2 (b) 0
Sol. Applying L’ Hospital’s rule, (c) 2( 2e p/ 2 - 1) (d) None of these
1 (t - a ) Sol. Here, f ( a, b ) = 2 cosb i.e. independent of a
f (t ) – ( f (t ) + f (a )) - f ¢(t )
lim 2 2 =0 p /2 b æ æp ö æ 3p p öö
t ®a 3(t – a ) 2 \ I =ò e ç f ( 0, 0 ) + f ç , p ÷ + f ç , - b ÷ ÷db
0 è è2 ø è 2 2 øø
1 (t - a ) p /2 b
f (t ) – ( f (t ) + f (a )) - f ¢(t ) =ò e (2 + 2 cosb + 2 sin b )db
Þ lim 2 2 =0 0
t ®a 3(t – a ) 2
Hence, = [eb (2 + 2 sin b )]0p /2
1 (t - a )
f ¢(t ) – ( f (t ) + f (a )) - f ¢(t ) = 4e p /2 - 2 = 2(2e p /2 - 1 )
Þ lim 2 2 =0
t ®a 12(t – a ) Hence, (c) is the correct answer.

f ¢¢(t ) æ
Þ lim =0 p /2 æ p öö
t®a 12 l Ex. 20 If I = ò cos 2 bç f (0, b) + f ç 0, - b÷ ÷ db, then
-p / 2 è è 2 øø
1 (t – a )
f ¢(t ) – ( f (t ) + f (a )) – f ¢(t ) [I ] is
Þ lim 2 2 =0
t®a 12(t – a ) (a) e p/ 2 (b) 2
f ¢¢(t ) (c) 2 ( 2e p/ 2 - 1) (d) None of these
Þ lim = 0 f ¢¢(a ) = 0 for any a.
t ® a 12 p /2 æ æ p öö
Sol. Again, I = ò cos2 b ç f ( 0, b ) + f ç 0, - b ÷ ÷ db
-p /2 è è 2 øø
Þ f (a ) is atmost of degree 1.
p /2
Hence, (a) is the correct answer. =ò cos2 b(cosb + sin b) db
-p /2
p /2 p /2
l Ex. 18 If f ¢¢( x ) < 0," x Î (a , b ) and c is a point such =ò cos3 b db + ò cos2 b sin b db
-p /2 -p /2
that a < c < b and (c , f (c )) is the point on the curve for which p /2
F (c ) is maximum, then f ¢(c ) is equal to = 2ò cos3 b db + 0
0
f (b )– f (a ) 2 ( f (b )– f (a )) q
(a) (b) Þ I= Þ [I ] = 2
b –a b –a 3
2 f (b )– f (a ) Hence, (b) is the correct answer.
(c) (d) 0
2b – a
128 Textbook of Integral Calculus

JEE Type Solved Examples :


Matching Type Questions
l Ex. 21 Match the following. 1
Put nx = t Þ dx = dt
n
Column I Column II 1 1 -1
– 1– x e x cos x
(p) –1 n òn /n + 1 tan (t ) dt æ0 ö
(A) The function f (x ) = is not defined at \ lim ç form ÷
sin x 2 n®¥ 1 1 è0 ø
sin -1 (t ) dt
x = 0. The value of f (0), so that f is continuous at n òn /n + 1
x = 0, is æ n ö p
1 dx tan -1 ç ÷
(q) 0 èn + 1ø 4 1
(B) The value of the definite integral ò0 is Use L’Hospital’s rule, lim = =
x+3x n®¥ æ n ö p 2
equal to a + b ln 2, where a and b are integers, then sin -1 ç ÷ 2
èn + 1ø
(a + b) is equal to
2
n n sec q - tan q (r) 1/ 2
(C) Given, e ò q
dq = 1, then the value of l Ex. 22 Match the following [IIT JEE 2006]
0 e
tan n is equal to
Column I Column II
1
(s) 1
(D) Let an = ò n1 tan -1 (nx ) dx and 1 dx 1 æ 2ö
n+ 1
(A) ò–1 1 + x 2 (p)
2
log ç ÷
è 3ø
1
an dx æ 3ö
bn = ò n1 sin -1 (nx ) dx, then lim
1 1
n ® ¥ bn
has the value (B) ò0 1– x 2
(q)
2
log ç ÷
è 2ø
n+ 1
equal to dx
3 p
Sol. (A) ® (r), (B) ® (p), (C) ® (s), (D) ® (r)
(C) ò2 1 – x 2 (r)
3
e x cos x - 1 - x e x cos x - 1 - x
2 dx p
(A) lim 2
= lim (D) ò1 x 2
(s)
2
x®0 sin x x®0 x2 x –1
x2 × 2
x
(applying L’Hospital’s twice) Sol. (A) ® (s), (B) ® (s), (C) ® (q), (D) ® (r)
x cos x
e × ( - x sin x + cos x ) - 1 1 dx
= lim
x®0 2x
(A) ò-1 1 + x2
e x cos x ( - x cos x - sin x - sin x ) + e x cos x ( - x sin x + cos x ) 1
= lim Q f (x ) = is an reven function.
x®0 2 1 + x2
1
= 1 dx p
2 \ I =2 ò 2
= [2 (tan -1 x )]10 =
0 1+ x 2
(B) Put x = u 6 Þdx = 6u 5 du
1 dx p
1 6u 5 du 1 u3 + 1 - 1 (B) ò0 = [sin -1 x ]10 =
\ I =ò =6 ò du = 5 - 6 ln 2 1 - x2 2
0 3
u +u 2 0 u+1
3
Þa + b = 5 - 6 = - 1 3 dx 3 dx é 1 ½ x - 1 ½ù
n -q
(C) ò2 1-x 2
=-ò 2
2 x -1
= - ê ln½ ½ú
ë 2 ½ x + 1 ½û 2
(C) en ò0 e (sec 2 q - tan q) dq = 1
1 2 1 æ3ö
Put - q = t Þ dq = - dt = - ln = ln ç ÷
-n t 2 3 2 è2ø
- en ò0 e (sec 2 t + tan t ) dt = 1 2 dx
[use ò e x ( f ( x ) + f ¢( x )) = e x f ( x )] (D) Let I = ò [put x = sec q Þ dx = sec q tan q dq]
1
x x2 -1
Þ -en [et tan t ]-0n = 1 Þ -en [ -e -n tan n ] = 1 Þ tan n = + 1 p /3 sec q tan q dq
1/n =ò
-1 sec q tan q
ò1/n + 1 tan (nx ) dx 0

(D) lim p /3 p
n®¥ 1/n -1 =ò 1 dq =
ò1/n + 1 sin (nx ) dx 0 3
Chap 02 Definite Integral 129

JEE Type Solved Examples :


Single Integer Answer Type Questions
l Ex. 23 Let f :R ® R be a continuous function which l Ex. 25 Let f ( x ) be a differentiable function satisfying
æ 1ö x
x
satisfies f ( x ) = ò f (t ) dt . Then, the value of f (ln 5 ) is____. f ( x ) + f ç x + ÷ = 1,"x Î R and g ( x ) = ò f (t )dt . If g(1) = 1,
0 è 2 ø 0

[IIT JEE 2009] æ ö


¥
ç ÷
Sol. (0) From given integral equation, f ( 0 ) = 0
then the value of å ç 8 ÷ is.
Also, differentiating the given integral equation w.r.t. x, we get ç n ÷
n =2
ç å ( g( x + k 2 ) - g( x + k ) ÷
f ¢( x ) = f ( x ) è k =1 ø
f ¢( x ) æ 1ö
If f ( x ) ¹ 0 , then = 1 Þ f ( x ) = ece x Sol. (6) Here, f ( x ) + f ç x + ÷ = 1
f (x ) è 2ø
Q f (0) = 0 Þ ec = 0, a contradiction æ 1ö æ 1ö
Replace x by ç x + ÷ , we get f ç x + ÷ + f ( x + 1 ) = 1
\ f ( x ) = 0, " x Î R Þ f (ln 5 ) = 0 è 2 ø è 2ø
On subtracting, f ( x ) = f ( x + 1 ) …(i)
l Ex. 24 For any real number x, let [ x ] denotes the largest x + 12 x x + 12
Also, g( x + 1 2 ) = ò f (t ) dt = ò f (t ) dt + ò f (t ) dt
integer less than or equal to x. Let f be a real valued func- 0 0 n

tion defined on the interval [–10, 10 ] by Since, f ( x + 1 ) = f ( x )


x 1
g( x + 1 2 ) = ò f (t ) dt + 1 2 × ò f (t )dt
f ( x ) = ìí
x –[ x ] , if [ x ] is odd \
. 0 0
î1 + [ x ]– x , if [ x ] is even x 1
g( x + 2 2 ) = ò f (t ) dt + 2 2 × ò f (t )dt
p 2 10 0 0

10 ò–10
Then, the value of f (x )cos p x dx is. x 1
[IIT JEE 2010] g( x + k 2 ) = ò f (t ) dt + k 2 × ò f (t )dt …(ii)
0 0
ì x - [ x ], if [ x ] is odd
Sol. (4) We have, f ( x ) = í x 1
î 1 + [ x ] - x , if [ x ] is even and g( x + k ) = ò f (t ) dt + k × ò f (t )dt …(iii)
0 0
f ( x ) and cos px are both periodic with period 2 and are both n n 1
even. Thus, å ( g( x + k 2 ) - g( x + k )) = å (k 2 - k ) × ò f (t ) dt
0
k =1 k =1
10 10 n
\ ò-10 f (x ) cos px dx = 2 ò0 f (x ) cos px dx = å (k 2 - k ) × g(1 ), given g( x ) = ò f (t ) dt
0
x

k =1
2
= 10 ò f ( x ) cos px dx æ n(n + 1 )(2n + 1 ) n(n + 1 ) ö
0 =ç - ÷ ´1
è 6 2 ø
1 1 1
ò0 f (x ) cos px dx = ò0 (1 - x ) cos px dx = - ò0 u cos pu du =
n(n - 1 )(n + 1 )
3
¥ ¥
8 8 ´3
\ å n

(n - 1 )n (n + 1 )
n =2
å 2
( g( x + k ) - g( x + k )) n =2

k =2
¥
–10 –9 –2 –1 0 1 2 9 10 æ (n + 1 ) - (n - 1 ) 1 1 ö
= 12 å ç = - ÷
2 2 1
n =2 è (n - 1 ) × n(n + 1 ) (n - 1 )n n(n + 1) ø
ò1 f (x ) cos px dx = ò1 (x - 1) cos px dx = - ò0 u cos pu du éæ 1 1 ö æ 1 1 ö
10 1 40 = 12 ê ç - ÷+ç - ÷+
\ ò-10 f ( x ) cos px dx = - 20 ò u cos pu du = 2
0 p ë è 1 ´ 2 2 ´ 3 ø è 2 ´ 3 3 ´ 4ø
æ 1 1 öù
K+ ç - ÷ú
p2 10 è (n - 1 ) n n (n + 1 ) ø û
Þ
10 ò-10 f (x ) cos px dx = 4 æ1
= 12 ç -
1 ö
÷ =6
è 2 n(n + 1 ) ø n ®¥
130 Textbook of Integral Calculus

Subjective Type Questions


l Ex. 26 Find the error in steps to evaluate the following Remark
integral. Students are advised to check continuity of anti-derivatives
p p 2 p 2 before substitution of integral limits.
dx sec x dx sec x dx
ò0 1 + 2 sin 2 x ò0 sec 2 x + 2 tan 2 x ò0 1 + 3 tan 2 x
= =
b a +b
l Ex. 27 If ò | sin x | dx = 8 and ò | cos x | dx = 9,
1 a 0
= [tan - 1 ( 3 tan x )] p0 = 0 b
3 then find the value of ò x sin x dx .
a
Sol. Here, the Newton-Leibnitz’s formula for evaluating the defi- b
nite integrals is not applicable because the anti-derivative, Sol. We know, òa | sin x | dx represents the area under the curve
1 from x = a to x = b.
f (x ) = [tan - 1 ( 3 tan x )]
3 We also know, area from x = a to x = a + p is 2.
p
has a discontinuity at the point x = which lies in the interval b

[ 0, p ].
2 \ òa | sin x | dx = 8
1 é æp öù p 8p
LHL = lim tan - 1 3 ê tan ç - h ÷ ú at x = Þ b -a = ...(i)
h®0 3 ë è 2 ø û 2 2
a +b
= lim
h®0
1
tan - 1 ( 3 cot h ) Similarly, ò0 | cos x | dx = 9
3
9p
1 p Þ a +b -0= ...(ii)
= lim tan - 1 ( ¥ ) = …(i) 2
h®0 3 2 3
From Eqs. (i) and (ii), a = p / 4, b = 17 p / 4
1 é æp öù p b 17p / 4
RHL = lim tan - 1 3 ê tan ç + h ÷ ú at x =
Also,
h®0 3 ë è 2 ø û 2
Hence, òa x sin x dx = ò
p/4
x sin x dx

p /4 17p / 4
p /4 + ò
= [ - x cos x ]17
1 cos x dx
= lim tan - 1 ( - 3 cot h ) p /4
h®0 3
17 p 17 p p p p /4
1 p =- cos + cos + [sin x ]17
= lim tan - 1 ( - ¥ ) = - …(ii) 4 4 4 4
p /4
h®0 3 2 3
4p
From Eqs. (i) and (ii), LHL ¹ RHL at x = p / 2 =-
2
Þ Anti-derivative f ( x ) is discontinuous at x = p / 2 . b
So, the correct solution for above integral; \ òa x sin x dx = - 2 2p
p p sec x dx 2
dx
I =ò =ò …(iii)
sin (sin - 1 b ) ½ cos (cos - 1 x)½
2
0 1 + 2 sin x 0 1 + 3 tan 2 x
l Ex. 28 Evaluate ò ½ ½dx.
ìï 0, f (2a - x ) = - f ( x ) cos (cos - 1 a ) -1
Using, ò
2a
f ( x ) dx = í a
½ sin (sin x ) ½
0
ïî 2 ò0 f ( x ) dx, f (2a - x ) = f ( x ) Sol. We know, cos (cos- 1 x ) and sin (sin - 1 x ) could be plotted as
2
We know, if f ( x ) =
sec x
f (p - x ) = f (x ) \ sin (sin - 1 x ) and cos (cos- 1 x ) are identical functions.
1 + 3 tan 2 x
sin (sin - 1 b ) cos (cos- 1 x ) sin (sin - 1 b )

\ Eq. (iii) reduces to I = 2 ò


p/ 2 2
sec x dx \ òcos (cos -1
a) -1
sin (sin x )
dx = ò
cos (cos - 1 a )
1 dx = (b - a )
0 1 + 3 tan 2 x
1 ¥ dt y=sin(sin–1x)
=2×
3 ò0 1 + t2
(put 3 tan x = t Þ 3sec 2x dx = dt) y=cos(cos–1x) 1
1
2
= [tan - 1 (t )]¥0 –1 –1
3 1 x x
O 1
2 2 æp ö p
= (tan - 1 ¥ - tan - 1 0 ) = ç - 0÷ = –1
3 3 è2 ø 3 –1
p dx p
\ ò0 1 + 2 sin2 x = 3
Chap 02 Definite Integral 131

b x -a (a 2 + b 2 ) / 2 x
l Ex. 29 Evaluate òa b-x
dx. l Ex. 30 Evaluate ò 2
(3a + b ) / 42
dx .
( x 2 - a 2 ) (b 2 - x 2 )
b x -a
Sol. Let I =ò dx …(i) (a 2 + b 2 )/ 2
x
a b-x Sol. Let I = ò dx …(i)
( 3a 2 + b 2 )/ 4
( x - a ) (b 2 - x 2 )
2 2
Put x = a cos2 t + b sin 2 t
Put x 2 = a 2 cos2 t + b 2 sin 2 t
\ x - a = a cos2 t + b sin 2 t - a = b [sin 2(t ) + a(cos2 t - 1 )]
Þ 2 x dx = [2a 2 cos t ( - sin t ) + 2b 2 sin t (cos t )] dt
= b sin 2 t - a sin 2 t Þ ( x - a ) = (b - a ) sin 2 t
1
Similarly, b - x = (b - a ) cos2 t Þ x dx = (b 2 - a 2 ) sin 2 t dt
2
\ dx = 2 (b - a ) sin t cos t dt
a2 + b2
where, x = a Þ sin 2 t = 0 Þ t = 0 (Q a < b ) For x2 = = a 2 cos2 t + b 2 sin 2 t,
2
and x = b Þ cos2 t = 0 Þ t = p / 2 a 2 + b 2 = 2 (1 - sin 2 t ) a 2 + 2b 2 sin 2 t
\ Eq. (i) reduces to or (a 2 + b 2 ) = 2a 2 + 2 (b 2 - a 2 ) sin 2 t
p /2 (b - a ) sin 2 t 1 p
I =ò × 2 (b - a ) sin t cos t dt Þ sin 2 t = Þ cos 2t = 0 Þ t =
0 (b - a ) cos2 t 2 4
p /2 sin t 3a 2 + b 2
=ò × 2 (b - a ) sin t cos t dt For x2 = = a 2 cos2 t + b 2 sin 2 t,
0 cos t 4
é sin 2 t sin t ù 3a 2 + b 2 = 4a 2 + 4 (b 2 - a 2 ) sin 2 t
êQ =+ as t Î[ 0, p / 2 ]ú
êë
2
cos t cos t úû 1 1 p
Þ sin 2 t =
Þ cos 2 t = Þ t =
p /2 p /2 4 2 6
= (b - a ) ò 2 sin 2 t dt = (b - a ) ò (1 - cos 2t ) dt \ Eq. (i) reduces to
0 0
p /2
æ sin 2 t ö p /4 1 (b 2 - a 2 ) sin 2 t
= (b - a ) çt - ÷ I =ò × dt
è 2 ø0 p /6 2 (b 2 - a 2 ) sin 2 t (b 2 - a 2 ) cos2 t
é p 1 ù épù p /4
= (b - a) ê æç - sin p ö÷ - æç 0 - sin 0 ö÷ ú = (b - a ) ê ú
1 p /4 æ ö æ p p ö p
è ø è ø =ò dt = ç t ÷ =ç - ÷ =
ë 2 2 2 û ë2û p /6 è ø p /6 è 4 6 ø 12
p
\ I = (b - a )
2 é æ p öù
é put b - x = t ù
e sec x ê sin ç x + ÷ ú
p /4 ë è 4 øû
I =ò
b x -a ê 2ú l Ex. 31 Evaluate ò dx .
Aliter Let
a b-x
dx ê or b - x = t ú 0 cos x (1 - sin x )
ê Þ - dx = 2 t dt ú
ë û é æ p öù
e sec x êsin ç x + ÷ ú
When x = a, then b - a = t 2 Þ b - a =t p /4 ë è 4 øû
Sol. Let I = ò dx
When x = b, then 0 = t 2 Þ 0 = t 0 cos x (1 - sin x )

(b - t 2 ) - a 1 p/ 4 e sec x (sin x + cos x ) (1 + sin x )


ò0
0
\ I =ò ( - 2t ) dt = dx
b -a t 2 cos x (1 - sin 2 x )
0
= -2 ò (b - a ) - t 2 dt 1 p/ 4 e sec x (sin x + cos x ) (1 + sin x )
b -a =
2 ò0 cos2 x × cos x
dx
b -a
=2 ò ( b - a ) - t dt 2 2
1 p/ 4
0
b -a
=
2 ò0 e sec x × (sec x tan x + sec x ) (sec x + tan x ) dx
ìï t (b - a ) - 1 æ t öüï
=2í (b - a ) - t 2 + sin ç ÷ý 1 p/ 4 sec x
ïî 2 2 è b - a øïþ 0 =
2 ò0 e × [sec x tan x (sec x + tan x ) + sec 2x
+ sec x tan x ] dx
éì (b - a ) - 1 ü ì (b - a ) - 1 ü ù
= 2 êí 0 + sin (1 )ý - í 0 + sin ( 0 )ý ú 1 p/4 sec x
2 ò0
ëî 2 þ î 2 þû = e × [sec x tan x (sec x + tan x + 1 ) + sec 2 x ] dx
2 (b - a ) p p
= × = (b - a ) 1 p/4 sec x
2 ò0
2 2 2 = e × sec x tan x (sec x + tan x + 1 )
1 p/4 sec x
2 ò0
+ e × sec 2x dx
132 Textbook of Integral Calculus

p /2
Applying integration by parts, p ìï æ t ö üï p 2
1 = í ( p - 0 ) + 4 çè log cos ÷ø ý- {1 - 0 }
I = {(sec x + tan x + 1 ) e sec x } 0p /4 8 ïî 2 0 ïþ 8
2
1 p/4 1 p/4 sec x pì 1 ü p2 p2 1 p2
= í p + 4 log ý- = + p log -
-
2 ò0
(sec x tan x + sec 2x ) × e sec xdx +
2 ò0
e × sec 2 x dx 8î 2þ 8 8 2 8
1 1 sec x p /4 p2 1 p2 p
= {( 2 + 1 + 1 ) e 2 - (1 + 1 ) e } - [e ]0 = - p log 2 = - log 2
2 2 8 2 16 4
1 1 1 e
= ( 2 + 2 ) e 2 - 2e - (e 2 - e ) = (1 + 2 ) e 2 - p x 2 cos x
2 2 2 2 l Ex. 33 Evaluate ò dx .
0
(1 + sin x ) 2
p /4 x 2 (sin 2 x - cos 2 x ) dx
l Ex. 32 Evaluate ò 2
× Sol. Let I = ò
p x 2 cos x p
dx = ò x 2 {(1 + sin x ) - 2 cos x } dx
0
(1 + sin 2 x ) cos x 0 (1 + sin x ) 2 0

2
p/ 4 x (sin 2 x - cos 2 x ) dx Applying integration by parts,
Sol. Let I = ò , p
0 (1 + sin 2 x ) cos2 x æ (1 + sin x ) - 1 ö p (1 + sin x ) - 1
I = çx2 ÷ - ò 2x × dx
1 p/2 t 2 (sin t - cos t ) è -1 ø0 0 -1
8 ò0 (1 + sin t ) (cos2 t / 2 )
= dt (put 2x = t)
p x
= (- p 2 + 0) + 2 ò dx …(i)
1 p/2 t 2 (sin t - cos t ) 0 1 + sin x
I= ò dt …(i)
4 0 (1 + sin t ) (1 + cos t ) é using, a f ( x ) dx = a f (a - x ) dx ù
é using, a a ù êë ò0 ò0 úû
êë ò0 f (x ) dx = ò0 f (a - x ) dx úû p (p - x )
2 I = - p2 + 2 ò dx …(ii)
æ p ö 0 1 + sin x
ç - t ÷ × (cos t - sin t )
1 p /2 è 2 ø
Adding Eqs. (i) and (ii), we get
4 ò0
I= dt …(ii)
(1 + cos t ) (1 + sin t ) p p
2I = - 2p 2 + 2 ò dx
Adding Eqs. (i) and (ii), we get 0 1 + sin x

æ p2 ö p 1 - sin x
ç pt - ÷ (sin t - cos t ) = - 2p 2 + 2p ò dx
0 cos2 x
1 p /2 è 4 ø
2I = ò dt …(iii)
4 0 (1 + cos t ) (1 + sin t ) = - 2 p 2 + 2 p [tan x - sec x ]0p
p/ 2 sin t - cos t = - 2 p 2 + 2 p [ 0 - ( - 1 - 1 )] = - 2 p 2 + 4 p
where, ò dt = 0
0 (1 + cos t ) (1 + sin t ) \ I = - p 2 + 2p
é as 2a f ( x ) dx = 0, if f (2a - x ) = - f ( x ) ù
ëê ò0 ûú l Ex. 34 Compute the following integrals.
\ Eq. (iii) becomes ¥ dx
1 p /2 pt (sin t - cos t ) dt (i) ò f (x n + x - n )ln x =0
I= ò +0 0 x
8 0 (1 + cos t ) (1 + sin t )
¥ dx
p p /2 t {(1 + sin t ) - (1 + cos t )} dt (ii) ò f (x n + x - n )ln x =0
=
8 ò0 (1 + cos t ) (1 + sin t )
0 1+x 2
p p /2 t dt p p /2 t dt Sol. (i) Let t = ln x Þ x = et
=
8 ò0 -
1 + cos t 8 ò0 1 + sin t dx
dx = et dt or = dt
p p /2 t dt p p /2 ( p / 2 - t ) dt x
=
8 ò0 -
1 + cos t 8 ò0 1 + cos t Also, x =0 Þ t =-¥
2p p /2 t dt p 2 p /2 dt and x =¥ Þ t =¥
= ò0 -
1 + cos t 16 ò0 1 + cos t
¥
f ( xn + x - n ) ln x
dx ¥
f (ent + e - nt ) × t dt = 0
8 \ ò0 x ò- ¥
=
p p /2 p2 p /2
=
8 ò0 t (sec 2 t / 2 ) dt -
16 ò0 (sec 2 t / 2 ) dt (Q integrand is an odd function of t)
¥ dx ¥ et
ò0 f ( xn + x - n ) ln x × - nt
1 + x 2 ò- ¥
nt
p ìï æ tö
p /2
p /2 t üï p 2 æ tö
p /2 (ii) = f (e + e ) × t × × dt
= í ç2t × tan ÷ - ò 1 × 2 tan dtý - ç2 tan ÷ 1 + e 2t
8 ïî è 2ø 0 0 2 ïþ 16 è 2ø0
Chap 02 Definite Integral 133

2
t et é x g(t ) ù
Now, if h (t ) = f (ent + e - nt ) ×
1 + e 2t 1 ëê ò0 ûú = 1 ( f ( x ))
2
Þ
- nt nt 1 æ et 1 ö 2 x 2 x
Then, h ( - t ) = f (e + e ) × (- t ) × t çQ = -t ÷
e +e -t
è 1+e 2t
e + et ø 1 1é x 2 ù 1 ( f ( x )) 2
Þ × ò g (t ) dt =
h( - t ) = - h(t ) ê
2 2ë 0 ú
û 2 x
Thus, integrand is an odd function and hence 1 x 1 ( f ( x )) 2
¥ -n dx
Þ
2 ò0
( f ¢(t )) 2 dt =
2 x
[using Eq. (i)]
ò0 f (x + x ) ln x × 1 + x 2 = 0
n
Differentiating both the sides w.r.t. x, we get
1 x 2 f ( x ) f ¢( x ) - ( f ( x )) 2
[ f ¢( x )]2 =
l Ex. 35 Show that 2 x2
¥ ¥ 1
(a) ò sin x dx = 1 (b) ò cos x dx = 0 Þ [ g( x )]2 x 2 = 2 x f ( x ) ( g( x )) - ( f ( x )) 2
0 0
2
Sol. Let us first evaluate; 2
æ x g( x ) ö æ x g( x ) ö
- sx - sx Þ ç ÷ =4ç ÷ -2
I =ò e sin x dx and J = ò e cos x dx è f (x ) ø è f (x ) ø
Using integer by parts, we get x g( x )
Þ t 2 - 4t + 2 = 0, where t =
f (x )
I = - e - sx cos x - sJ …(i)
4± 8 x f ¢( x )
and J =e - sx
sin x + sI …(ii) Þ t= = 2 ± 2 or =2 ± 2
2 f (x )
Subtracting Eqs. (i) and (ii), we get Þ ln f ( x ) = 2 ± 2 ln x + constant
½ cos x + s sin x ½ Þ f ( x ) = cx 2 + 2
or cx 2 - 2
I = - e - sx ½ ½
½ 1 + s2 ½
Þ g( x ) = f ¢( x ) = c1 x1 + 2
or c 2 x1 - 2

½ s s ½
2
Þ J = e - sx ½sin x - 2 sin x - 2
cos x ½ where, c1 and c 2 are constants of integration.
½ s +1 1+s ½ But g is continuous on [ 0, ¥ ), then c 2 x1 - 2
is ruled out.
½ sin x - s cos x ½ g( x ) = c1 x1 + 2
= e - sx ½ ½ Hence,
½ 1 + s2 ½
Also, g( 0 ) = c 1 = 1 Þ g( x ) = x1 + 2
¥ 1
Thus, ò0 e - sx sin x dx = 2
s +1 p /4
l Ex. 37 Let I n = ò tan n x dx (n > 1 and is an integer).
¥ s 0
and ò0 e - sx cos x dx = 2 Show that
s +1 1 1 1
¥ ¥ 1
(i) In + In - 2 = (ii) < In <
Now, ò0 sin x dx = lim ò0 e - sx
sin x dx = lim 2 =1 n -1 2 (n + 1) 2 (n - 1)
s® 0 s®0 s + 1
p /4 p/ 4
¥ ¥ s Sol. (i) Given, I n = ò tann x dx = ò tann - 2 x × tan 2 x dx
- sx
ò0 cos x dx = lim ò0 cos x dx = lim 2 =0 0 0
and e
s® 0 s® 0 s +1 p /4 p /4
=ò sec 2 x × tann - 2 x dx - ò tann - 2 x dx
0 0
1 n-2
l Ex. 36 Find a function g : R ® R, continuous in [0, ¥) =ò t dt - I n - 2, where t = tan x
0
and positive in (0, ¥) satisfying g(0 ) = 1 and æ tn -1 ö
1
1
1 x 2 1æ x 2
ö÷ . In + In - 2 = ç ÷ \ In + In - 2 =
2 ò0 x è ò0
g (t ) dt = ç g (t ) dt è n -1 ø 0 n -1
ø
x (ii) For 0 < x < p / 4, we have 0 < tann x < tann - 2 x
Sol. Let f (x ) = ò g(t ) dt
0 So that; 0 < In < In - 2
Þ f ¢( x ) = g ( x ) \ I n + I n + 2 < 2I n < I n + I n - 2
1 1
1 x 1 x Þ < 2I n <
Þ
2 ò0
( f ¢(t )) 2 dt = ò g 2(t ) dt
2 0
…(i) n+1 n -1
1 1
1é x 2 Þ < In <
g(t ) ù = ( f ( x )) 2
1
Þ
ê ò
2ë 0 ú
û 2
2 (n + 1 ) 2 (n - 1 )
134 Textbook of Integral Calculus

1 - cos n xp Sol. To prove; sin 2 Kx = 2 sin x [cos x + cos 3 x + cos 5 x +


l Ex. 38 If U n = ò dx , where n is positive ¼+ cos (2 K - 1 ) x ]
1 - cos x 0
Taking RHS, [2 sin x cos x + 2 sin x cos 3 x + 2 sin x cos 5 x +
integer or zero, then show that U n + 2 + U n = 2 U n + 1 .
¼+ 2 sin x cos (2 K - 1 ) x ]
p / 2 sin 2 nq 1
Hence, deduce that ò = np. = (sin 2 x ) + (sin 4 x - sin 2 x ) + [sin 6 x - sin 4 x ) +
2
0
sin q 2 ¼+ (sin 2 Kx - sin (2 K - 2 ) x ]
p [1 - cos (n + 2 ) x ] - [1 - cos (n + 1 ) x ] = sin 2Kx
Sol. U n + 2 - U n + 1 = ò dx
0 1 - cos x p/ 2 p/ 2 æ sin 2 Kx ö
p cos (n + 1 ) x - cos (n + 2 ) x
Now, ò0 sin 2 K x × cot x dx = ò
0
ç
è sin x ø
÷ × cos x dx
=ò dx
0 1 - cos x p/ 2
=ò 2 cos x [cos x + cos 3 x + ¼+ cos (2 K - 1 ) x ] dx
0
æ 3ö x
2 sin ç n + ÷ x × sin p /2 p /2
p è 2ø 2 =ò (1 + cos 2 x ) dx + ò (cos 4 x + cos 2 x ) dx
=ò dx 0 0
0 x 2 p/ 2
2 sin
2 + ¼+ ò [cos 2 K x + cos (2 K - 2 ) x ] dx
0
æ 3 ö But we know that,
sin çn + ÷ x
p è 2ø p/ 2
Un + 2 - Un + 1 = ò dx …(i)
0
sin
x ò0 (cos 2nx ) dx = 0, " n Î I and n ¹ 0
2 p /2 p /2 p
æ 1ö
Þ ò0 sin 2 Kx × cot x dx = ò
0
1 dx + 0 =
2
sin çn + ÷ x
p è 2ø
ÞU n + 1 - U n = ò dx …(ii)
0 x 3 1 æ 2x ö
sin
2
l Ex. 40 Evaluate ò × sin - 1 ç ÷ dx .
2
0
1+ x è 1+ x2 ø
From Eqs. (i) and (ii), we get
æ 3ö æ 1ö 3 1 æ 2x ö
sin çn + ÷ x - sin çn + ÷ x Sol. Let I =ò sin - 1 ç ÷ dx
è
p 2ø è 2ø 0 1 + x2 è 1 + x2 ø
(U n + 2 - U n + 1 ) - (U n + 1 - U n ) = ò dx
0 x
sin æ 2 x ö ì 2 tan - 1 x, if - 1 £ x £ 1
2 Now, sin - 1 ç ÷ =í
p 2 cos (n + 1 ) x × sin x / 2
è 1 + x 2 ø î p - 2 tan - 1 x, if x > 1
Un + 2 + Un - 2 Un + 1 = ò dx
0 sin x / 2 1 1 æ 2x ö 3 1 æ 2x ö
p \ I =ò 2
sin - 1 ç ÷ dx + ò × sin - 1 ç ÷ dx
p æ sin (n + 1 ) x ö 0 1+ x è1 + x2 ø 1 1+ x 2
è1 + x2 ø
= 2 ò cos (n + 1 ) x dx = 2 ç ÷ =0
0 è n + 1 ø0 1 2 tan - 1 x 3 p - 2 tan - 1 x
Þ U n + 2 + U n = 2U n + 1 =ò dx + ò dx
0 1 + x2 1 1 + x2
Þ U n , U n + 1, U n + 2 are in AP.
1 -1
1 tan - 1 x 3 1 3 tan - 1 x
Now, U0 = ò
p
dx = 0 =2 ò dx + p ò dx - 2 ò dx
0 1 - cos x
0 1 + x2 1 1 + x2 1 1 + x2
p /4 p /3
p 1 - cos x =2 ò t dt + p (tan - 1 x )1 3
-2 ò t dt (put tan - 1 x = t)
U1 = ò dx = p 0 p /4
0 1 - cos x p /4 p /3
æ t2 ö æ t2 ö
U1 - U 0 = p (common difference) =2 ç ÷ + p {tan - 1 3 - tan - 1 1 } - 2 ç ÷
U n = U 0 + np = np è 2 ø0 è 2 ø p /4
U n = np p2 ì p pü ì p
2
p2 ü 7 2
2 = + p í - ý -í - ý= p
p /2 sin nq p / 2 1 - cos 2 nq 16 î 3 4 þ î 9 16 þ 72
Now, In = ò dq = ò dq
0 2
sin q 0 1 - cos 2 q
x 2 2 x 2
1 p 1 - cos n x Ex. 41 Prove that ò e xt × e - t dt = e x e-t
1
ò0
/4 /4
dt .
ò
l
= dx = np 0
2 0 1 - cos x 2
x 2 x +z 1
Sol. Let I = ò e xt × e - t dt. Put t = Þ dt = dz
l Ex. 39 Prove that for any positive integer K , 0 2 2
2
æ x+z ö æ x+z ö
sin 2Kx x -t 2 1 x ç ÷x -ç ÷
= 2 [cos x + cos 3 x +¼+ cos(2 K - 1) x ] \ I = ò e ×e tx
dt = ò e è 2 ø
×e è 2 ø
dz
sin x 0 2 -x
p /2 p æ x+ z öæ x+z ö æ x + z öæ x -z ö
Hence, prove that ò sin 2Kx × cot x dx = × 1 x çè 2 øè
֍ x-
2 ø
÷ 1 x çè ÷ç
2 øè 2 ø
÷

2 ò- x 2 ò- x
0 2 = e dz = e dz
Chap 02 Definite Integral 135

x2 z2 ¥ 2dt ¥ 2dt
1 x - 1 2 x 2 =ò =ò
= ò e 4 4 dz = e x /4 × ò e - z /4 dz 0 2 2
(1 + t ) + a (1 - t ) 0 (1 - a ) t 2 + (1 + a )
2 -x 2 -x
2 ¥ dt
1 2 1 2 = ò0
x 2 x 2
= e x /4 ò e - t /4 dt = e x /4 × 2 ò e - t /4 dt 1 -a 2
2 - x 2 0 æ 1+a ö
t2 + ç ÷
é using, a f ( x ) dx = 2 a f ( x ) dx, when f ( - x ) = f ( x ) ù è 1 -a ø
êë ò-a ò0 úû ¥
2 1 æ 1 -a ö 2 p p
x 2 /4 x - t 2 /4 = × × ç tan - 1 t ÷ = × =
=e ò0 e dt 1 - a (1 + a ) /(1 - a ) è 1 + a ø0 1 -a 2 2
1 -a 2
x 2 2 x 2
Þ ò0 e xt × x - t dt = e x /4
ò0 e -t /4
dt (integrating both the sides w.r.t. ‘a’)
Þ f (a ) = p sin - 1 a + c
1
Ex. 42 If f ( x ) = e x + ò (e x + te - x ) f (t ) dt , find f ( x ). p log (1 + 0 cos x ) dx
l
0
But f (0) = ò =0
0 cos x
Sol. We can write f ( x ) = Ae x + Be - x , where Þ c =0
A = 1 + ò f (t ) dt
1
and
1
B = ò tf (t ) dt \ f (a ) = p sin - 1 a
0 0
1 p /2
\ A = 1 + ò ( Ae + Be ) dt = 1 + ( Aet - Be - t ) 0t
t -t
l Ex. 44 Evaluate ò cosec q tan - 1 (c sin q ) dq.
0 0
A = 1 + A (e1 - 1 ) - B (e - 1 - 1 ) p /2
Sol. Let f (c ) = ò cosec q × tan - 1 (c sin q) dq
-1 0
Þ (2 - e ) A + (e - 1) B = 1 …(i)
p /2 (sin q)
1 \ f ¢(c ) = ò cosec q × dq + 0 - 0
B = ò t ( Aet + Be - t ) dt 0 1 + c 2 sin 2 q
0
p /2 dq
= A (tet - et ) 0t + B ( - te - t - e - t ) 01 =ò
0 1 + c 2 sin 2 q
B = A + B (1 - 2e - 1 )
p /2 cosec 2 q dq p /2 cosec 2 q dq
Þ A - 2e - 1 B = 0 …(ii) =ò =ò
0 2 2 2
c + cosec q 0 (c + 1 ) + cot 2 q
From Eqs. (i) and (ii), we get p /2
æ ö
A=
2 (e - 1 )
,B =
e -1 =-
1 ç tan - 1 cot q ÷
4e - 2e 2
- 2e ç c + 1 ÷ø 0
2
4 c2 +1 è
2 (e - 1 ) x e - 1 - x p
Hence, f (x ) = ×e + ×e f ¢(c ) =
4e - 2e 2 4 - 2e 2 c2 + 1
Integrating both the sides, we get
l Ex. 43 If | a | <1, show that p dc p
f (c ) = ò = log (c + c 2 + 1 ) + A
p log (1 + a cos x ) 2 c +1 2
2
ò0 dx = p sin - 1 a.
cos x where, f ( 0 ) = A
p /2
Sol. Given, | a | < 1 But f (0) = ò cosec q tan - 1 ( 0 ) dq = 0
0
p log (1 + a cos x )
Let f (a ) = ò dx p
0 cos x Þ f (c ) = log (c + c 2 + 1 )
2
p cos x p dx
\ f ¢(a ) = ò dx = ò
0 cos x × (1 + a cos x ) 0 1 + a cos x p /2
l Ex. 45 Evaluate ò sec q × tan - 1 (a cos q ) dq.
0
(differentiating w.r.t. ‘a’ using Leibnitz rule)
æ x dx 2 ö Sol. Given definite integral is a function of ‘a’. Let its value be I (a ).
ç put tan = t Þ = 2
, when x = 0, t = 0 ÷ p /2
ç 2 dt 1 + t ÷ Then, I (a ) = ò sec q × tan - 1 (a cos q) dq
è and when x = p , t ® ¥ ø 0
p /2 1
2dt Þ I ¢(a ) = ò sec q × × cos q dq + 0 - 0
0 1 + a cos2 q
2
¥ 1 + t2
Þ f ¢(a ) = ò (using Leibnitz rule)
0 æ 1 -t2 ö
1+a ç ÷ p /2 1 p /2 sec 2 q
è 1 + t2 ø =ò 2
dq = ò
2
dq
0 1 + a cos q 0 sec 2 q + a 2
136 Textbook of Integral Calculus

p /2 sec 2 q From Eqs. (ii) and (iii),


=ò dq (put tan q = t)
0 1 + tan 2 q + a 2 180 80
a= and b =
¥ 119 119
¥ dt 1 æ t ö
=ò = ç tan - 1 ÷ \ Eq. (i) reduces to
0 t 2 + (a 2 + 1 ) 2
a +1 ç a 2 + 1 ÷ø 0
è 80 x 2 + 180 x
f (x ) =
1æp ö p 119
= ç - 0 ÷ Þ I ¢(a ) =
a +12è 2 ø 2 a2 + 1 l Ex. 48 Prove that
Integrating both the sides w.r.t. ‘a’, we get x u x
p
I (a ) = log | a + a 2 + 1 | + C
ò0 { ò0 f (t ) dt } du = ò
0
( x - u ) f (u ) du.
2 Sol. Here, applying integration by parts to
Since, I (0) = 0 + C x u
p ò0{ 1 × {1ò044
f (t ) dt } du
Þ I (a ) = log | a + 1 + a 2 | II
244 3
I
2
u
i.e. taking ‘1’ as second function and ò f (t ) dt as first function,
0
l Ex. 46 Let f be a continuous function on [a , b ]. Prove we have
x
that there exists a number x Î[a , b ] such that x ì u ü ì u ü x
ò0 1 × íî ò0 f (t ) dtýþ du = íî ò0 f (t ) dtýþ 0 × (u )0 - ò0 f (u ) × u du
x
x b
òa f (t ) dt = ò
x
f (t ) dt . x
= æçu ò f (t ) dt ö÷ - ò uf (u ) du
u x
x b è 0 ø0 0
Sol. Let g( x ) = ò f (t ) dt - ò f (t ) dt, x Î[a, b ]
a x x x
b b =x ò f (t ) dt - ò uf (u ) du
We have, g(a ) = - ò f (t ) dt and g(b ) = ò f (t ) dt 0 0
a a x
2
=ò ( x - u ) f (u ) du
0
Þ g(a ) × g(b ) = - æç ò f (t ) dt ö÷ £ 0
b
èa ø 3p / 2
l Ex. 49 Evaluate ò (log | sin x |) (cos ( 2nx )) dx , n Î N .
Clearly, g( x ) is continuous in [a, b ] and g(a ) × g(b ) £ 0. 0
It implies that g( x ) will become zero at least once in [a, b ]. 3p / 2
Sol. Let I (n ) = ò (log | sin x | ) (cos 2nx ) dx
x b 0
Hence, ò f (t ) dt = ò f (t ) dt for atleast one value of x Î[a, b ]. I II
a x
3p / 2
æ sin 2nx ö 3p / 2 sin 2nx
I (n ) = ç log | sin x | × ÷ -ò cot x × dx
l
1
Ex. 47 If f ( x ) = x + ò ( xy + x y ) ( f (y )) dy , find f ( x ). 2 2 è 2n ø 0 0 2n
0
1 1 (using integration by parts)
Sol. Given, f ( x ) = x + x ò y 2 f (y ) dy + x 2 ò y f (y ) dy 1 3p / 2 sin 2nx × cos x 1
0 0 I (n ) = 0 -
2n ò0 sin x
dx = 0 - I 1(n )
2
…(i)
= x æç1 + ò y 2 f (y ) dy ö÷ + x 2 æç y f (y ) dy ö÷
1 1
è 0 ø è ò0 ø 3p / 2 sin (2nx ) cos x
Let I 1(n ) = ò dx
Þ f ( x ) is a quadratic expression. 0 sin x
Þ f ( x ) = ax + bx 2 or f (y ) = ay + by 2
3p / 2 sin (2n + 2 ) x × cos x
…(i) I 1(n + 1 ) = ò dx
0 sin x
1 1
where, a = 1 + ò y 2 f (y ) dy = 1 + ò y 2 (ay + by 2 ) dy 3p / 2 (sin (2n + 2 ) x - sin 2nx ) × cos x
0 0 \ I 1(n + 1 ) - I 1(n ) = ò dx
1
0 sin x
æ ay 4 by 5 ö
÷ a = 1 + æç + ö÷
a b 3p/ 2 2 cos (2n + 1 ) x × sin x × cos x
= 1 + çç + ÷ =ò dx
è 4 5 ø0 è 4 5 ø 0 sin x
3p/ 2
Þ 20a = 20 + 5a + 4b or 15a - 4b = 20 …(ii) =ò (cos (2n + 2 ) x + cos 2nx ) dx
0
1 1
and b = ò y f (y ) dy = ò y (ay + by ) dy 2
æ sin (2n + 2 ) x sin (2n ) x ö
3p / 2
0 0
=ç + ÷ =0+ 0
1 è (2n + 2 ) 2n ø0
æ ay 3 by 4 ö a b
= çç + ÷ Þ b= +
÷ Þ I 1(n + 1 ) = I 1(n ) = ¼= I 1(1 )
è 3 4 ø0 3 4
3p / 2 sin 2 x × cos x 3p/ 2 2 sin x cos x × cos x
Þ 12 b = 4a + 3b or 9b - 4a = 0 …(iii) \ I 1(1 ) = ò dx = ò dx
0 sin x 0 sin x
Chap 02 Definite Integral 137

3p/ 2 3p/ 2 1 1
=ò 2 cos2 x dx = ò (1 + cos 2 x ) dx Again, I = ò (tx + 1 - x )n dx = ò {(1 - x ) + tx} n dx
0 0 0 0
3p / 2 1 n
æ sin 2 x ö = ò { C 0 (1 - x )n + nC1 (1 - x )n - 1 (tx ) + nC 2 (1 - x )n - 2
= çx + ÷ 0
è 2 ø0
(tx ) 2 + K+ nCr (1 - x )n - r (tx )r + K + nCn (tx )n } dx
3p 1
Þ I 1(1 ) = = I 1(n ) Þ I (n ) = - I 1(n ) [using Eq. (i)] 1 ì n ü
2 2n =ò í å Cr (1 - x )n - r (tx )r ý dx
n
0
3p î r =0 þ
\ I (n ) = - n 1
4n =å n
Cr tr ò0 {(1 - x )n - r xr } dx ...(ii)
r =0
¥
l Ex. 50 Evaluate ò e - x sin n x dx , if n is an even integer. From Eqs. (i) and (ii),
0 n 1 1
Sol. Here, I n = ò
¥
e -x
sin x dxn å nCr tr ò0 (1 - x )n - r xr dx =
n+1
{tn + tn - 1 + tn - 2 + K + t + 1}
0 r =0
¥
= ( - sinn x e - x ) 0¥ + n ò sinn - 1 x cos x e - x dx Equating coefficient of tk on both the sides, we get
0 1 1
¥
n
Ck ò (1 - x )n - k xk dx =
= n ò (sinn - 1 x × cos x )(e - x ) dx [where ( - sinn x e - x ) 0¥ = 0 ] 0 n+1
0
I II 1 1
n -1
Þ ò0 (1 - x )n - k xk dx = n
Þ I n = n [( - sin x × cos x e - x ) 0¥ Ck (n + 1 )
¥ ¥
+ (n - 1 ) ò sinn - 2 x cos2 x e - xdx - ò sinn x e - x dx ]
0 0 l Ex. 52 Given a real valued function f ( x ) which is
¥ -x n-2 ¥ -x
I n = n (n - 1 ) ò e sin x dx - n 2
ò0 e n
sin x dx monotonic and differentiable, prove that for any real
0
numbers a and b;
= n (n - 1 ) I n - 2 - n 2I n b f (b) -1
òa (a)} dx = ò
2 2
2 {f (x ) - f 2 x {b - f (x )} dx
Þ (n + 1 ) I n = n (n - 1 ) I n - 2 f (a )

n (n - 1 ) Sol. As, f ( x ) is differentiable and monotonic.


\ In = In - 2
n2 + 1 \f -1
( x ) exists.
Replacing n by (n - 2 ), (n - 4 ), ..., 2, we get Let f -1
( x ) = t Þ x = f (t ) or dx = f ¢(t ) dt
(n - 2 ) (n - 3 ) (n - 1 ) (n - 3 ) (n - 4 ) (n - 5 ) -1
In - 2 = 2
In - 4 = × In - 6 As, x = f (a ) Þ f { f (a )} = t Þ t = a
(n - 2 ) + 1 (n - 2 ) 2 + 1 (n - 4 ) 2 + 1
-1
and x = f (b ) Þ f { f (b )} = t Þ t = b
... and so on.
f (b ) b
2 (2 - 1 ) \ òf (a ) 2 x {b - f - 1( x )} dx = ò 2 f (t ) (b - t ) f ¢(t ) dt
I2 = I0 a
22 + 1 b
n (n - 1 ) (n - 2 ) (n - 3 ) (n - 4 ) (n - 5 ) 2 (2 - 1 ) = ò (b - t ) {2 f (t ) f ¢(t )} dt
\ In = 2 × × K 2 I0 a
I II
n + 1 (n - 2 ) 2 + 1 (n - 3 ) 2 + 1 2 +1 b
= ((b - t ) { f (t )} 2 )ba + ò { f (t )} 2 dt
n! n! æçQ I = ¥e - x dx = 1 ö÷
è 0 ò0
= n /2 × I 0 = n /2 a
ø b
P {1 + (2r ) 2 } P 1 + 4r 2 = - (b - a ) { f (a )} 2 + ò { f (t )} 2 dt
r =1 r =1 a
b
= ò ({ f (t )} 2 - { f (a )} 2 ) dt
1 a
l Ex. 51 Evaluate ò (tx + 1 - x ) n dx , n Î N and t is b
0 =ò { f 2 ( x ) - f 2 (a )} dx
a
independent of x. Hence, show
1 k 1
ò0 x (1 - x ) n -k dx = . l Ex. 53 Evaluate
n
C k (n +1) 1 sin q (cos 2 q - cos 2 p / 5 ) (cos 2 q - cos 2 2 p / 5 )
1
Sol. Here, I = ò (tx + 1 - x )n dx = ò ((t - 1 ) x + 1 )n dx
1 ò0 sin 5 q
dq.
0 0

ì ((t - 1 ) x + 1 )n + 1 ü
1 Sol. We know,
=í ý é æpö ùé æ 2p ö ù
z 10 - 1 = (z 2 - 1 ) êz 2 - 2 cos ç ÷ z + 1 ú êz 2 - 2 ç cos ÷ z + 1 ú
î (n + 1 ) (t - 1 ) þ 0 ë è 5 ø ûë è 5 ø û
1 æ 2 4p öæ 2 6p ö
= {tn + tn - 1 + tn - 2 + K + t + 1 } ...(i) ´ ç z - 2 cos z + 1 ÷ ç z - 2 cos z +1 ÷
n+1 è 5 øè 5 ø
138 Textbook of Integral Calculus

1 æ 1 öæ p 1 öæ 2p 1 ö 1 é ap æb öù
or z5 - = ç z - ÷ ç z - 2 cos + ÷ ç z - 2 cos + ÷ I= + b ln ç ÷ ú
z5 è z øè 5 z øè 5 z ø a 2 + b 2 êë 2 èa øû
æ 4p 1 ö æ 6p 1 ö bp
´ ç z - 2 cos + ÷ ç z - 2 cos + ÷ Again, abI + b 2I =
è 5 z øè 5 z ø 2
Put z = cos q + i sin q and abI – a 2J = a ln(b / a )
Þ 2 i sin 5 q = (2 i sin q) (2 cos q - 2 cos p / 5 ) (2 cos q - 2 cos 2 p / 5 )
´ (2 cos q - 2 cos 4 p / 5 ) (2 cos q - 2 cos 6 p / 5 ) 1 é bp æb öù
On subtracting, we get J = 2ê
– a ln ç ÷ ú
2
a +b ë 2 èa øû
Þ sin 5 q = 16 sin q (cos2 q - cos2 p / 5 ) (cos2 q - cos2 2 p / 5 )

sin q (cos2 q - cos2 p / 5 ) (cos2 q - cos2 2 p / 5 ) 1 Aliter Convert a cos x + b sin x into a single cosine say
Þ = cos ( x + f ) and put ( x + f ) = t.
sin 5 q 16
¥ ln x dx
sin q (cos2 q - cos2 p / 5 ) (cos2 q - cos2 2 p / 5 )
ò0
1 l Ex. 56 Evaluate .
\ ò0 sin 5 q
dq 2
x + 2x + 4
1 1 1
=ò dq = ¥ ln x dx
0 16 16 Sol. I = ò (put x = 2t Þ dx = 2dt to make coefficient of
n
0x 2 + 2x + 4
n CK x 2 and constant term same)
l Ex. 54 Show that lim S =e - 2.
K
n® ¥ k =0 n (K + 3 ) ¥ ln 2 + ln t ln 2 ¥ dt 1 ¥ ln t dt
=2 ò 2
dt = ò 2
+ ò 2
n n
CK n
0 4(t + t + 1 ) 2 140 t2 +4t +31 2 1
0 t +t +1
4243
1 n 1
Sol. lim
n® ¥
å K
n (K + 3)
= lim
n®¥
å K +3
CK × K
n
I1 I 2 = zero

K =0 K =0 ¥ ln t dt æ 1 1 ö
I2 = ò ç put t = Þ dt = – 2 dy ÷
n
1 1 æ 1 1 ö 2
t +t +1è ø
å y
0
ò0 x K + 2 dx çQ = x K + 2 dx ÷ y
è K + 3 ò0
n
= lim CK × K
n®¥ n ø
K =0 0 ln y ×( +1 )
I2 = ò dy
1 æ 2 n
æ x ö ö
K ¥ æ 1 1 ö 2

0
çç x lim
è n®¥
å C K × ç ÷ ÷÷ dx
n
è n ø ø
ç 2
èy
+
y
+ 1 ÷
ø
y
K =0
ln y dy ln y dy
ïì x ö ïü
n 0 0
1 æ I2 = ò = -ò = –I 2
= ò x 2 í lim ç 1 + ÷ ý dx ¥ y2 +y +1 ¥ y2 +y +1
n®¥ è n ø ïþ
îï
0
¥
é n ù ¥ dt é 2 –1 [t + (1 / 2 )] ù
2
1 æ x ö Now, I 1 = ò = tan
= ò x × e dx 2 x
êQ lim ç1 + ÷ = e ú
x
2 ê ú
êë n ® ¥ è n ø 0 2
0
úû æ 1ö æ 3ö ë 3 3 û0
çt + ÷ + ç ÷
1 ì 1 ü è 2ø è 2 ø
= ( x × e ) 0 - ò 2 x × e dx = e - 2 í ( x e ) 0 - ò e dxý
2 x 1 x x 1 x
0 î 0 þ 2 é p p ù 2p ln2 2 p p ln 2
= – = \I= × =
= e - 2 {e - e + 1 } = e - 2 3 êë 2 6 úû 3 3 2 3 3 3 3
¥ ln x dx
p /2 cos x Note For a > 0, I = ò =0
l Ex. 55 Let I = ò dx 0 ax 2 + bx + a
0 a cos x + b sin x
p /2
[Hint By putting x = 1 / t, we get I = – I , so I = 0]
sin x
and J = ò dx , where a > 0 and b > 0.
0 a cos x + b sin x l Ex. 57 Find a function f , continuous for all x (and not
Compute the values of I and J. x f (t ) sin t
p zero everywhere) such that f 2 ( x ) = ò dt .
Sol. Q aI + bJ = ...(i) 0 2 + cos t
2
p /2 b cos x – a sin x f (t )sin t x
and bI – aJ = ò dx Sol. We have, f 2( x ) = ò dt
0 a cos x + b sin x 2 + cost 0

\ bI – aJ = ln[a cos x + b sin x ]p0 /2 (Note that f 2( x ) being an integral function of a continuous
function is continuous and differentiable)
æ bö
Þ bI – aJ = ln ç ÷ ...(ii) f ( x ) × sin x
èaø 2 f ( x ) f ¢( x ) =
2 + cos x
From Eqs. (i) and (ii),
Integrating, we get 2 f ( x ) = C – ln(2 + cos x )
ap
a 2I + abJ = Q x = 0 Þ f ( 0 ) = 0 Þ C = ln 3
2
1 3
b 2I – abJ = b ln(b / a ) \ f ( x ) = ln
2 2 + cos x
Chap 02 Definite Integral 139

¥ tan –1 ax – tan –1 x 2a p × 2a pa
l Ex. 58 Evaluate ò dx, where a is a =– [tan –1 t 1 – a 2 ]¥0 = – =–
2 2
0 x 1–a 2 1–a 1–a2
parameter. –1
tan ax – tan –1 x
¥ or I = p 1 – a 2 + C
Sol. Let I =ò dx
0 x
If a = 0 and I = 0, then C = –p
dI ¥ 1× x ¥ dx
da ò0 (1 + a 2x 2 ) x
\ = dx = ò
0 1 + a 2x 2 \ I = p æè 1 – a 2 – 1 öø
1 ¥ dx 1 p
= [a tan –1 x ]¥0 =
a 2 ò0 x 2 + 1 a 2
=
2a p /2 æ 1 + a sin x ö dx
a2
l Ex. 60 Evaluate ò ln ç ÷ (|a | <1).
0 è 1 – a sin x ø sin x
p da p
ò dI = 2 ò a Þ I = 2 lna + C p/ 2 æ 1 + a sin x ö dx
Sol. Let I =ò ln ç ÷ (| a | < 1 )
When a = 1 and I = 0 then C = 0
0 è 1 – a sin x ø sin x
p dI p/ 2 2 sin x dx
I = ln a
da ò0 (1 – a 2 sin 2 x ) sin x
Hence, \ = ×
2

ln (1 – a 2 x 2 ) p/ 2 2 sec 2 x
1 =ò dx
l Ex. 59 Evalute ò0 dx (a 2 < 1). 0 1 + tan 2 x – a 2 tan 2 x
x 2 (1 – x 2 )
p/ 2 2 sec 2 x
1 ln (1 – a 2x 2 ) =ò dx (put tan x = t )
Sol. Let I =ò dx, | a| < 1 0 (1 – a 2 ) tan 2 x + 1
0 2 2
x (1 – x )
¥ 2 dt 2 ¥ dt

(1 – a 2 ) t 2 + 1 (1 – a 2 ) ò0
=
dI 1 – 2ax 2 dx 0
æ 1 ö
2
\ =ò × (put x = sin q)
da 0 (1 – a 2x 2 ) x 2 1 – x 2 t2 + ç ÷
è 1–a2 ø
p /2 2adq p /2 2a sec 2 q dq 2 p
=ò =ò Þ = [tan –1(t 1 – a 2 )]¥0 =
0 2 2
a sin q – 1 0 a tan 2 q – (1 + tan 2 q)
2
1–a2 1–a2
p da
p /2 2a sec 2 q dq Þ dI =
=-ò (put tan q = 1) 1–a2
0 (1 – a 2 ) tan 2 q + 1
When a = 0 and I = 0 , then C = 0
¥ 2adt 2a ¥ dt
=ò 2 2
or – 2 ò0 2 \ I = p sin –1(a )
0 (1 – a ) t + 1 (1 – a ) æ 1 ö
2
t +ç ÷
è 1–a2 ø Hence, I = p sin –1a + C
#L Definite Integral Exercise 1 :
Single Option Correct Type Questions
4 (y 2 − 4y + 5) sin (y − 2)dy  dt  2
1. The value of ∫
1
is 8. If x satisfies the equation  ∫ x
0
(2y 2 − 8y + 1)  0
t + 2t cos α + 1
2

(a) 0 (b) 2  3 t sin 2t 


2

(c) –2 (d) None of these − ∫ dt  x − 2 = 0(0 < α < π ), then the value of
 −3 t 2 + 1 
2. Let f ( x ) = x 2 + ax + b and the only solution of the x is
equation f ( x ) = minimum f ( x ) is x = 0 and f ( x ) = 0 has α 2 sin α
β (a) ± (b) ±
root α and β, then ∫ x 3 dx is equal to 2 sin α α
α
1 1 α sin α
(a) (β 4 + α 4 ) (b) (a 2 − b 2 ) (c) ± (d) ± 2
4 4 sin α α
(c) 0 (d) None of these
x t dt
x y  dy  9. If f ( x ) = e g ( x ) and g ( x ) = ∫ , then f ′ (2) is equal to
∫ (3 − 2 sin t ) dt + ∫ cos t dt = 0, then   at x = π
2
3. 2
1+t4
π /2 0  dx 
(a) 2/17 (b) 0
and y = π is
(c) 1 (d) Cannot be determined
(a) 3 (b) − 2
(c) − 3 (d) None of these 10. If a, b and c are real numbers, then the value of
1 t 
sin −1 (sin x ) + cos −1 (cos x )
4  (1 + x 2 )  lim ln  ∫ (1 + a sin bx )c / x dx  equals
4. ∫ dx = log  t→ 0 t 0 
–4
  x 2   a  ab
(1 + x ) 1 +  
2
(a) abc (b)
  17  c
bc ca
c − π  (c) (d)
+bπ tan −1   (where, [⋅] denotes greatest integer a b
 1 + cπ 
r = 4n
n
function), then number of ways in which a − (2b + c )
a −5
11. The value of lim
n→ ∞

r =1 r (3 r + 4 n ) 2
is equal to
distinct object can distributed among persons
c 1 1
(a) (b)
equally, is 35 14
9! 12 ! 15 ! 10 ! 1 1
(a) (b) (c) (d) × 3! (c) (d)
(3 !) 3 ( 4 !) 3 (5 !) 3 (6 !) 3 10 5
5. The value of the definite integral x
12. Let f ( x ) = ∫ e t dt and h( x ) = f (1 + g ( x )) where g ( x ) is
2

∞ dx −1
∫0
(1 + x )(1 + x 2 )
a
(a > 0) is
defined for all x , g ′ ( x ) exists for all x, and g ( x ) < 0 for
π π x > 0. If h ′ (1) = e and g ′ (1) = 1, then the possible values
(a) (b)
4 2 which g(1) can take
(c) π (d) Some function of a (a) 0 (b) −1
(c) −2 (d) −4
6. The value of the definite integral
3π /4
∫0
[(1 + x ) sin x + (1 − x ) cos x ] dx is 13. Let f ( x ) be a function satisfying f ′ ( x ) = f ( x ) with
3π π f (0) = 1 and g be the function satisfying
(a) 2 tan (b) 2 tan
8 4 f ( x ) + g( x ) = x 2 .
π 1
(c) 2 tan (d) 0 The value of the integral ∫ f ( x ) g ( x ) dx is
8 0
1 /n tan −1 (nx ) 1 5
7. Let C n = ∫ dx , then lim n 2 ⋅ C n is equal (a) e − e 2 − (b) e − e 2 − 3
1 /n + 1
sin −1 (nx ) n→ ∞ 2 2
to 1 1 2 3
(c) (e − 3 ) (d) e − e −
(a) 1 (b) 0 2 2 2
1
(c) −1 (d)
2
Chap 02 Definite Integral 141

g (x ) dt
14. Let f ( x ) = ∫ , 21. The value of π  ∫
2008
x | sin πx | dx  is equal to
1+t2
0
0 
cos x
where g ( x ) = ∫ (1 + sin t 2 ) dt . Also, h( x ) = e − | x | and (a) 2008 (b) π 2008 (c) 1004 (d) 2008
0
1  π n
n
f ( x ) = x 2 sin , if x ≠ 0and f (0) = 0, then f ′   is equal to
x  2 22. lim
n→ ∞

k =1 n + k 2x 2
2
, x > 0 is equal to
(a) l′( 0 ) (b) h′ ( 0 − ) (a) x tan −1 ( x ) (b) tan −1 ( x )
1 − cos x
(c) h′ ( 0 ) +
(d) lim tan −1 ( x ) tan −1 ( x )
x → 0 x sin x (c) (d)
x x2
π
15. For f ( x ) = x 4 + | x |, let I 1 = ∫ f (cos x ) dx and 23. Let a > 0 and f ( x ) is monotonic increasing such that
0
a b

I2 = ∫
π /2
f (sin x ) dx , then
I1
has the value equal to f (0) = 0 and f (a ) = b, then ∫ f ( x ) dx + ∫ f −1
( x ) dx is
0 0
0 I2 equal to
(a) 1 (b) 1/2 (a) a + b (b) ab + b (c) ab + a (d) ab
(c) 2 (d) 4 4
1/ 3x 2x 1/ 3 x4
24. If ∫ cos −1 dx = k ∫ , then
16. Let f be a positive function. Let −1 / 3
1− x4 1+ x2 0
1− x4
k k
I1 = ∫ ( x ) f ( x (1 − x )) dx ; I 2 = ∫ f ( x (1 − x )) dx , ‘k’ is equal to
1 −k 1 −k
I (a) π (b) 2π (c) 2 (d) 1
where 2k − 1 > 0. Then, 2 is
I1 ∞  1  ln x
(a) k (b) 1/2
25. ∫0
f x +  ⋅
 x x
dx is equal to

(c) 1 (d) 2 (a) 0 (b) 1


1
17. Suppose that the quadratic function f ( x ) = ax + bx + c 2
(c) (d) Cannot be evaluated
2
is non-negative on the interval [−1, 1]. Then, the area 1/λ
26. lim  ∫ (1 + x ) λ dx 
1
under the graph of f over the interval [−1, 1] and the λ→ 0 0
is equal to
X -axis is given by the formula
4 4
(a) A = f ( −1 ) + f (1 ) (a) 2 ln 2 (b) (c) ln (d) 4
e e
 1  1 27. If g ( x ) is the inverse of f ( x ) and f ( x ) has domain
(b) A = f  −  + f  
 2  2
x ∈[1, 5], where f (1) = 2 and f (5) = 10, then the values of
1
(c) A = [ f ( −1 ) + 2 f ( 0 ) + f (1 )] 5 10
2 ∫ f ( x ) dx + ∫ g (y ) dy is equal to
1 2
1 (a) 48 (b) 64 (c) 71 (d) 52
(d) A = [ f ( −1 ) + 4 f ( 0 ) + f (1 )]
3
2
28. The value of the definite integral
π x  π /2
18. Let I (a ) = ∫
0
 + a sin x  dx , where ‘a’ is positive real.
a  ∫0
sin x sin 2x sin 3x dx is equal to
1 2 1 1
The value of ‘a’ for which I (a ) attains its minimum value, (a) (b) − (c) − (d)
3 3 3 6
is x
29. If f ( x ) = ∫ ( f (t )) 2 dt , f : R → R be differentiable function
2 3 π π 0
(a) π (b) π (c) (d) and f ( g ( x )) is differentiable function at x = a, then
3 2 16 13
(a) g( x ) must be differentiable at x = a
19. The set of values of ‘a’ which satisfy the equation (b) g( x ) may be non-differentiable at x = a
2 4 (c) g( x ) may be discontinuous at x = a
∫0
(t − log 2 a ) dt = log 2  2  , is
a  (d) None of the above
(a) a ∈ R (b) a ∈ R + 30. The number of integral solutions of the equation
(c) a < 2 (d) a > 2 ∞ ln t dt
4∫ − π ln 2 = 0 ; x > 0, is
 1/x ln (1 + t ) 
2 0
x2 +t2
20. lim  x 3
x→ ∞  ∫
−1 / x
1 + et
dt  is equal to
 (a) 0 (b) 1 (c) 2 (d) 3
1 2 16n 2 / π π  xπ 
(a)
3
(b)
3
31. ∫0
cos
2  n 
dx is equal to
(c) 1 (d) 0 (a) 0 (b) 1 (c) 2 (d) 3
142 Textbook of Integral Calculus

−1 2
32. If ∫ (ax 2 − 5) dx = 0 and 5 + ∫ (bx + c ) dx = 0, then
n
−2 1
34. If I n = ∫ ({x + 1}{x 2 + 2} + {x 2 + 3} {x 3 + 4 }) dx , (where,
−n
(a) ax 2 − bx + c = 0 has atleast one root in (1, 2) {} denotes the fractional part), then I 1 is equal to
(b) ax 2 − bx + c = 0 has atleast one root in ( −2, − 1 ) 1 2 1
(a) − (b) − (c) (d) None of these
(c) ax 2 + bx + c = 0 has atleast one root in ( −2, − 1 ) 3 3 3
(d) None of the above π /2 sin (2n − 1) x 2
π / 2 sin nx
6 35. Let I n = ∫ dx , J n = ∫ ,n ∈N,
33. The value of ∫ ( x + 12x − 36 + x − 12x − 36 ) dx 0 sin x 0
sin 2 x
0
is equal to then
(a) 6 3 (b) 4 3 (a) Jn + 1 − Jn = I n (b) Jn + 1 − Jn = I n + 1
(c) 12 3 (d) None of these (c) Jn + 1 + Jn = Jn (d) Jn + 1 − Jn + 1 = Jn

#L Definite Integral Exercise 2 :


More than One Option Correct Type Questions
36. If f ( x ) = [sin −1 (sin 2x )] (where, [] denotes the greatest  1  3
(a) f   + f   = 1
 4  4
integer function), then
π /2 π (b) the value of J equals to 1/2
(a) ∫ − sin −1 (sin 1 )
f ( x ) dx =
0 2  1  2
(c) f   ⋅ f   = 1
(b) f ( x ) is periodic with period π  3  3
(c) lim f ( x ) = − 1 π /2 sin x dx
x→
π
(d) ∫ has the same value as J
2 0 (sin x + cos x ) 3
(d) None of the above
37. Which of the following definite integral(s) vanishes? 42. Let f ( x ) is a real valued function defined by
1 1

(a) ∫
π /2

0
ln (cot x ) dx (b) ∫

0
sin 3 x dx f (x ) = x 2 + x 2 ∫−1
tf (t ) dt + x 3 ∫ −1
f (t ) dt ,
then which of the following hold(s) good?
e dx π 1 + cos 2 x
(c) ∫ (d) ∫ dx 1 10 30
1 /e x (ln x )1 / 3 0 2 (a) ∫ t f (t ) dt = (b) f (1 ) + f ( −1 ) =
−1 11 11
38. The equation 10x 4 − 3x 2 − 1 = 0 has 1 1 20
(c) ∫ t f (t ) dt > ∫ f (t ) dt (d) f (1 ) − f ( −1 ) =
−1 −1
(a) atleast one root in ( −1, 0 ) 11
(b) atleast one root in (0, 1) 43. Let f ( x ) and g ( x ) be differentiable functions such that
(c) atleast two roots in ( −1, 1 ) x
f ( x ) + ∫ g (t ) dt = sin x (cos x − sin x ) and
(d) no root in (−1, 1) 0

π /2 ( f ′ ( x )) 2 + ( g ( x )) 2 = 1, then f ( x ) and g ( x ) respectively,


39. Suppose I 1 = ∫ cos ( π sin 2 x ) dx ; can be
0
π /2 π /2 1 cos 2 x
I2 = ∫ cos (2 π sin 2 x ) dx and I 3 = ∫ cos ( π sin x ) dx, (a) sin 2 x, sin 2 x (b) , cos 2 x
0 0 2 2
then 1
(c) sin 2 x, − sin 2 x (d) − sin 2 x, cos 2 x
(a) I 1 = 0 (b) I 2 + I 3 = 0 2
(c) I 1 + I 2 + I 3 = 0 (d) I 2 = I 3 x
44. Let f ( x ) = ∫ (t sin at + bt + c )dt , where a, b, c are
1 −x
40. Let f ( x ) = ∫ (1 − | t | ) cos ( xt ) dt , then which of the f (x )
−1
non-zero real numbers, then lim is
following holds true? x→ 0 x
(a) f ( 0 ) is not defined (a) independent of a
(b) lim f ( x ) exists and is equal to 2 (b) independent of a and b, and has the value equals to c
x→0

(c) lim f ( x ) exists and is equal to 1 (c) independent a, b and c


x→0
(d) dependent only on c
(d) f ( x ) is continuous at x = 0
∞ n dx
41. The function f is continuous and has the property 45. Let L = lim
n→ ∞ ∫ a
1 + n 2x 2
, where a ∈ R, then L can be
1
f ( f ( x )) = 1 − x for all x ∈[0, 1] and J = ∫ f ( x ) dx , then (a) π (b) π /2 (c) 0 (d) 1
0
Chap 02 Definite Integral 143

#L Definite Integral Exercise 3 :


Passage Based Questions
Passage I 52. The number of solutions f ( x ) = 2ex is equal to
(Q. Nos. 46 to 48) (a) 0 (b) 1
t dt2 (c) 2 (d) None of these

x

( a + t r ) 1/ p
0 53. lim ( f ( x )) f ( − x ) is
Suppose lim = l, where x→ ∞
x→ 0 bx − sin x (a) 3 (b) 6
p ∈ N , p ≥ 2, a > 0, r > 0 and b ≠ 0. (c) 1 (d) None of these
46. If l exists and is non-zero, then 54. The function f ( x ) is
(a) b > 1 (b) 0 < b < 1 (a) increasing for all x (b) non-monotonic
(c) b < 0 (d) b = 1 (c) decreasing for all x (d) None of these
47. If p = 3 and l = 1, then the value of ‘a’ is equal to Passage IV
(a) 8 (b) 3 (Q. Nos. 55 to 57)
(c) 6 (d) 3/2
f ( x ) = ∫ ( 4t 4 − at 3 ) dt and g ( x ) is quadratic satisfying
x

48. If p = 2 and a = 9 and l exists, then the value of l is equal 0

to g ( 0) + 6 = g ′ ( 0) − c = g ′′ ( c ) + 2b = 0. y = h( x ) and y = g ( x )
3 2 1 7 intersect in 4 distinct points with abscissae x i ; i = 1, 2, 3, 4 such
(a) (b) (c) (d)
2 3 3 9 i
that ∑ = 8, a, b, c ∈ R + and h( x ) = f ′ ( x ) .
xi
Passage II
(Q. Nos. 49 to 51) 55. Abscissae of point of intersection are in
Suppose f ( x ) and g ( x ) are two continuous functions defined (a) AP (b) GP
1
for 0 ≤ x ≤ 1 . Given, f ( x ) = ∫ e x + 1 ⋅ f ( t ) dt and (c) HP (d) None of these
0

g (x ) = ∫ e
1
x+ t
⋅ g ( t ) dt + x. 56. ‘a’ is equal to
0 (a) 6 (b) 8 (c) 20 (d) 12
49. The value of f (1) equals 57. ‘c’ is equal to
(a) 0 (b) 1 (a) 25 (b) 25/2 (c) 25/4 (d) 25/8
(c) e −1 (d) e
Passage V
50. The value of g (0) − f (0) equals (Q. Nos. 58 to 60)
2 3 1
(a) (b) (c) (d) 0 v ( x) dy dy
3 − e2 e2 − 2 e2 − 1 Let y = ∫ f ( t ) dt , let us define as
u ( x) dx dx
g (0) = v′ ( x ) f 2 ( v( x )) − u′ ( x ) f 2 ( u( x )) and the equation of the
51. The value of equals
g (2)  dy
tangent at ( a, b ) and y − b =   ( x − a ).
(a) 0 (b)
1
(c)
1
(d)
2  dx ( a , b)
3 e2 e2 x2
58. If y = ∫ t 2 dt , then the equation of tangent at x = 1 is
x
Passage III (a) x + y = 1 (b) y = x − 1
(Q. Nos. 52 to 54) (c) y = x (d) y = x + 1
We are given the curves y = ∫
x
f ( t )dt through the point x4 dy
−∞ 59. If y = ∫ (ln t )dt , then lim is equal to
x2 x → 0+ dx
 1
 0,  any y = f ( x ), where f ( x ) > 0 and f ( x ) is differentiable, (a) 0 (b) 1 (c) 2 (d) −1
 2
∀ x ∈ R through ( 0, 1) . Tangents drawn to both the curves at
x d
60. If f ( x ) = ∫ e t
2
/2
(1 − t 2 ) dt , then f ( x ) at x = 1 is
the points with equal abscissae intersect on the same point on 1 dx
the X-axis. (a) 0 (b) 1
(c) 2 (d) −1
144 Textbook of Integral Calculus

Passage VI (Q. Nos. 61 to 62)


 π π  log e x 
Consider f :( 0, ∞ ) →  − ,  , defined as f ( x ) = tan −1  .
 2 2  (log e x ) 2 + 1

61. The about function can be classified as
(a) Injective but nor surjective
62. The value of ∫ 0
[tan]dx is equal to (where, [⋅] denotes

(b) Surjective but not bijective the greatest integer function)


π π
(c) Neither injective nor surjective (a) − (b)
(d) Both injective and surjective 2 2
(c) −1 (d) 1

#L Definite Integral Exercise 4 :


Matching Type Questions
1 T
63. Let lim
T→ ∞ T ∫ 0
(sin x + sin ax ) 2 dx = L, then
Column I Column II

Column I Column II (B) The value of the definite integral (q) e−1/ 4
1 1/ e

∫ e− x dx + ∫ − ln x dx is equal to
2

(A) For a = 0, the value of L is (p) 0 0 1

(B) For a = 1, the value of L is (q) 1/2 1


(r) e1/ 2
(C)  11 ⋅ 22 ⋅ 33 K (n − 1)n − 1 ⋅ nn  n 2
(C) For a = − 1, the value of L is (r) 1 lim   equals
n→ ∞  n1 + 2 + 3 + K + n 
(D) For a ∈ R − {− 1, 0, 1}, the value of L is (s) 2
(s) e
1 1
64. Let f (θ = ∫ ( x + sin θ) dx and g (θ) = ∫ ( x + cos θ) dx
2 2
0 0
66. Match the following.
where θ ∈[0, 2π ]. The quantity f (θ) − g (θ), ∀ θ in the
interval given in column I, is Column I Column II

Column I Column II (A) If f (x ) = ∫


g( x ) dt
, where (p) −2
0
1 + t3
(A)  π 3π  (p) negative
 ,  g (x ) = ∫
cos x
(1 + sin t 2 ) dt, then value of
4 4  0

 π
(B)  3π  (q) positive f ′   is
 ,π  2
 4 
(B) (q) 2
(C)  3π 7π  (r) non-negative If f (x ) is a non-zero differentiable
, 
 2 4 
x
function such that ∫ f (t ) dt = { f (x )}2 ,
0

(D)  π   7π  (s) non-positive ∀x ∈ R, then f (2) is equal to


 0,  ∪  , 2π 
 4  4 
b
(C) If ∫ (2 + x − x 2 ) dx is maximum, then (r) 1
a
65. Match the following. a + b is equal to
Column I Column II  sin 2x b (s) −1
(D) If lim  3 + a + 2  = 0, then 3a + b
1 −1 x→ 0  x x 
(A)
∫ (1 + 2008 x 2008 ) ex (p) e
2008
dx equals
0 has the value
Chap 02 Definite Integral 145

#L Definite Integral Exercise 5 :


Single Integer Answer Type Questions
sin nx π  m
67. If f ( x ) = ∑ and ∫ f ( x ) dx = log   , then the value of (m + n ) is ...
n =1 4 n 0 n
π /2 cos x dx
68. The value of I = ∫ (where, [⋅] denotes greatest integer function) is ...
−π / 2
1 + 2 [sin −1 (sin x )]
1 π / 2 sin 2 nθ f (15) + f (13)
69. If f ( x ) =
π ∫0
sin θ
2
dθ (n ∈ N ), then the value of
f (15) − f (a )
...

x
70. Let f ( x ) = ∫ e (1 + t ) 2
dt and g ( x ) = (h / x ), where h( x ) is defined for all x ∈ h. If g 1 / 2 = e 4 and h′ (2) = 1. Then, absolute value
−2

of sun for all possible value of h(2), is ...

71. If = ∫ sin x ⋅ log (sin x )dx = log   . Then, the value of K is …


π /2 K
0 e

#L Definite Integrals Exercise 6 :


Questions Asked in Previous 10 Years’ Exams
(i) JEE Advanced & IIT-JEE 76. The option(s) with the values of a and L that satisfy the

72. The value of ∫


x cos x π /2
2
dx is equal to equation
∫ 0
e t (sin 6 at + cos 4 at )dt
= L, is/are
−π / 2
1 + e x [Only One Correct Option 2016] π

π2 π2
∫ 0
e t (sin 6 at + cos 4 at )dt
(a) −2 (b) +2 [More than One Correct Option 2015]
4 4
(c) π − e − π / 2
2
(d) π + e π / 2
2 e 4π − 1 e 4π + 1
(a) a = 2, L = π (b) a = 2, L =
e −1 eπ + 1
73. The total number of distincts x ∈[0, 1] for which e 4π − 1 e 4π + 1
t2 (c) a = 4, L = (d) a = 4, L =
eπ − 1 eπ + 1

x
dt = 2x − 1 is
0 1+t4 [Integer Type 2016]
n
Directions (Q. Nos. 77 to 78) Let F : R → R be a thrice
74. Let f ( x ) =7 tan x + 7 tan x –3 tan x – 3 tan 2 x for all
8 6 4
differentiable function. Suppose that F(1) = 0, F(3 ) = − 4
 π π and F′ ( x ) < 0 for all x ∈(1, 3 ). Let f ( x ) = xF( x ) for all x ∈ R.
x ∈ – ,  . Then, the correct expression(s) is/are
 2 2
77. The correct statement(s) is/are
[More than One Correct Option 2015] [Passage Based Questions 2015]
π /4 1 π /4
(a) ∫ x f ( x ) dx = (b) ∫ f ( x ) dx = 0 (a) f ′ (1 ) < 0
0 12 0

π /4 π /4
(b) f (2 ) < 0
1
(c) ∫ x f ( x ) dx = (d) ∫ f ( x ) dx = 1 (c) f ′ ( x ) =/ 0 for any x ∈(1, 3 )
0 6 0
(d) f ′ ( x ) = 0 for some x ∈(1, 3 )
192x 3  1
75. Let f ′ ( x ) = for all x ∈ R with f   = 0. If 3
78. If ∫ x 2 F ′ ( x ) dx = − 12 and
3
2 + sin 4 πx  2 1 ∫1
x 3 F ′′( x ) dx = 40, then the
1
m ≤ ∫ f ( x ) dx ≤ M, then the possible values of m and M correct expression(s) is/are
t /2 3
(a) 9 f ′ (3 ) + f ′ (1 ) − 32 = 0 (b) ∫ f ( x ) dx = 12
are [More than One Correct Option 2015] 1
3
1 1 (c) 9 f ′ (3 ) – f ′ (1 ) + 32 = 0 (d) ∫ f ( x ) dx = − 12
(a) m = 13, M = 24 (b) m = , M = 1
4 2
(c) m = − 11, M = 0 (d) m = 1, M = 12
146 Textbook of Integral Calculus

[x ], x ≤ 2 List I List II


79. Let f : R → R be a function defined by f ( x ) =  ,
0, x > 2 C. 2 3x 2
r. 4
where [x ] denotes the greatest integer less than or equal ∫ −2 1 + ex
dx equals
2 xf ( x 2 )
to x. If I = ∫ dx , then the value of ( 4 I − 1) is  1/ 2  1 + x  s. 0
−1 2 + f ( x + 1)
[Integer Answer Type 2015]  ∫ cos 2x log   dx
 − 1/ 2  1 − x 
1  12 + 9 x 2  D.
 1/ 2  1 + x 
equals
80. If α = ∫ (e 9 x + 3 tan
−1
−1
)  dx , where tan x takes
x
 ∫ cos 2x log   dx
0
 1+ x2   0  1 − x 
 3π 
only principal values, then value of  loge |1 + α | −  is
 4 Codes
[Integer Answer Type 2015] A B C D A B C D
π /2 (a) r q s p (b) q r s p
81. The integral ∫ (2 cosec x )17 dx is equal to (c) r q p s (d) q r p s
π /4
[Only One Correct Option 2014]
1  d2 
log(1 + 2 ) log(1 + 2 ) 85. The value of ∫ 4 x 3  (1 − x 2 ) 5  dx is
∫ ∫
−u 16 −u 17
(a) 2(e + e ) du
u
(b) (e + e ) du
u
2
 dx 
0
0 0
log(1 + 2 ) log(1 + 2 ) [Integer Answer Type 2014]
∫ ∫
−u 17
(c) (e − e ) du
u
(d) 2(eu − e −u )16 du
0 0 π /2  2 π − x
86. The value of the integral ∫  x + log  cos x dx
82. Let f :[0, 2] → R be a function which is continuous on − π /2  π + x
is
[0, 2] and is differentiable on (0, 2) with f (0) = 1. [Only One Correct Option 2012]
x2 π2
Let F ( x ) = ∫ f ( t ) dt , for x ∈[0, 2]. If F ′ ( x ) = f ′ ( x ), ∀ (a) 0 (b) −4
0 2
x ∈(0, 2), then F(2) equals [Only One Correct Option 2014] π2 π 2
(c) +4 (d)
(a) e 2 − 1 (b) e 4 − 1 2 2
(c) e − 1 (d) e 4 log 3 x sin x 2
83. Match the conditions/expressions in Column I with
87. The value of ∫ log 2
sin x 2 + sin (log 6 − x 2 )
dx is
statement in Column II. [Matching Type 2014]
[Integer Answer Type 2011]
1 3 1 3
Column I Column II (a) log (b) log
4 2 2 2
A. 1 dx p. 1  2 3 1 3
∫ −1 1 + x2 2
log  
 3
(c) log
2
(d) log
6 2

B. 1 dx q.  2 88. Let f : [1, ∞ ] → [2, ∞ ] be differentiable function such that


∫ 2 log  
 3 x
1− x f (1) = 2. If 6∫ f ( x ) = dt = 3x f ( x ) − x 3 , ∀ x ≥ 1 then the
0 2

C. 3 dx r. π
∫ 2 1 − x2 3
value of f (2) is .... [Integer Answer Type 2011]
1
x (1 − x )
4 4

dx π 89. The value(s) of ∫ dx is (are)


1+ x 2
2
D. s.
∫ 1
x x2 − 1 2
0
[Only One Correct Option 2010]
22 2
(a) −π (b)
84. Match List I with List II and select the correct answer 7 105
using codes given below the lists. 71 3 π
[Matching Type 2014] (c) 0 (d) −
15 2
List I List II
90. For a ∈ R (the set of all real numbers), a ≠ − 1,
A. The number of polynomials f (x ) with non-negative p. 8
integer coefficients of degree ≤ 2, satisfying (1a + 2a + K + n a ) 1
1 lim = .
f (0) = 0 and ∫ f (x ) dx = 1, is n→ ∞ (n + 1)a −1 [(na + 1) + (na + 2) + K + (na + n )] 60
0

B. The number of points in the interval [ − 13 , 13 ] at q. 2 Then, a is equal to [More than One Correct Option 2010]
which f (x ) = sin(x 2 ) + cos(x 2 ) attains its maximum (a) 5 (b) 7
−15 −17
value, is (c) (d)
2 2
Chap 02 Definite Integral 147

n Directions (Q Nos. 91 to 92) Let f ( x ) = (1 − x )2 sin 2 x + x 2 , 94. Let f be a non-negative function defined on the interval
x  2 ( t − 1) 
∀ x ∈ R and g ( x ) = ∫  − ln t f ( t ) dt ∀ x ∈ (1, ∞).
x x
1  t+1  [0, 1]. If ∫ 1 − ( f ′(t )) 2 dt = ∫ f (t ) dt , 0 ≤ x ≤ 1 and
0 0
[Passage Based Questions 2010] f (0) = 0, then [Only One Option Correct 2009]
91. Consider the statements  1 1  1 1  1 1  1 1
P : There exists some x ∈ R such that, (a) f   < and f   > (b) f   > and f   >
 2 2  3 3  2 2  3 3
f ( x ) + 2x = 2 (1 + x 2 ).
 1 1  1 1  1 1  1 1
Q : There exists some x ∈ R such that 2 f ( x ) + 1 = 2x (1 + x ). (c) f   < and f   < (d) f   > and f   <
 2 2  3 3  2 2  3 3
Then,
π sin nx
(a) both P and Q are true (b) P is true and Q is false 95. If I n = ∫ dx , n = 0, 1, 2,..., then
(c) P is false and Q is true (d) both P and Q are false −π
(1 + π x ) sin x
[More than One Correct 2009]
92. Which of the following is true? 10
(a) g is increasing on (1, ∞ ) (a) I n = I n + 2 (b) ∑I 2m + 1 = 10 π
(b) g is decreasing on (1, ∞ ) 10
m =1

g is increasing on (1, 2) and decreasing on (2, ∞ )


(c)
(d) g is decreasing on (1, 2) and increasing on (2, ∞ )
(c) ∑I
m =1
2m =0 (d) I n = I n + 1

93. For any real number x, let [x ] denotes the largest n


n n −1
n
integer less than or equal to x. Let f be a real valued
96. Let S n = ∑ and T n = ∑ 2 , for
k =0 n + kn + k 2 2
k =0 n + kn +k2
function defined on the interval [− 10, 10] by
n = 1, 2, 3, ..., then [More than One Correct Option 2008]
 x − [x [, if f ( x ) is odd π π
f(x) =  (a) Sn < (b) Sn >
1 + [x [− x, if f ( x ) is even 3 3 3 3
π2 10 π π
Then, the value of
10 ∫
− 10
f ( x ) cos πx dx is……
[Integer Answer Type 2010]
(c) Tn <
3 3
(d) Tn >
3 3

(ii) JEE Main & AIEEE


3π /4 dx
97. The integral ∫ is equal to 102. Statement I The value of the integral [2013 JEE Main]
π /4 1 + cos x [2017 JEE Main] π /3 dx
(a) − 1 (b) − 2 ∫π /6
1 + tan x
is equal to π/6 .
(c) 2 (d) 4
b b

98. Let I n = ∫ tan n x dx , (n > 1). I 4 + I 6 = a tan 5 x + bx 5 + C, Statement II ∫


a
f ( x ) dx = ∫ f (a + b − x ) dx
a

where C is a constant of integration, then the ordered (a) Statement I is true; Statement II is true; Statement II is a true
explanation for Statement I
pair (a, b) is equal to [2017 JEE Main]
(b) Statement I is true; Statement II is true; Statement II is not a
 1   1  1  1 
(a)  − , 0 (b)  − , 1 (c)  , 0 (d)  , − 1 true explanation for Statement I
 5   5  5  5 
(c) Statement I is true; Statement II is false
1 /n (d) Statement I is false; Statement II is true
(n + 1)(n + 2) K 3n 
99. lim   is equal to
n→ ∞
 n 2n  [2016 JEE Main] 103. The intercepts on X-axis made by tangents to the curve,
x
18
(a) 4
27
(b) 2
9
(c) 2 (d) 3 log 3 − 2 y = ∫ | t | dt , x ∈ R, which are parallel to the line y = 2x ,
0
e e e
are equal to [2013 JEE Main]
log x 2
100. The integral ∫ (a) ± 1 (b) ± 2
4
dx is equal to
2 log x 2 + log(36 − 12x + x 2 ) (c) ± 3 (d) ± 4
x
[2015 JEE Main]
104. If g ( x ) = ∫ cos 4t dt , then g ( x + π ) equals
(a) 2 (b) 4 (c) 1 (d) 6 0 [2012 AIEEE]
g( x )
π
x x (a) (b) g( x ) + g( π ) (c) g( x ) − g( π ) (d) g( x ) ⋅ g( π )
101. The integral ∫ 1 + 4 sin 2 − 4 sin dx is equal to g( π )
2 2 [2014 JEE Main] 1 8 log (1 + x )
105. The value of ∫
0
2π dx is
(a) π − 4 (b) −4−4 3 0
1+ x2 [2011 AIEEE]
3 π π
(c) 4 3 − 4 (d) 4 3 − 4 − π / 3 (a) log 2 (b) log 2 (c) log 2 (d) π log 2
8 2
148 Textbook of Integral Calculus

106. For x ∈ 0,


5π  x
 , define f ( x ) = ∫ 0 t sin t dt . Then, f has
108. ∫ 0
[cot x ] dx ,[ ] denotes the greatest integer function, is
 2 equal to [2009 AIEEE]
[2011 AIEEE]
(a) local minimum at π and 2π π
(a) (b) 1
(b) local minimum at π and local maximum at 2π 2
(c) local maximum at π and local minimum at 2π π
(c) − 1 (d) −
(d) local maximum at π and 2π 2
1 1
sin x cos x
107. Let p ( x ) be a function defined on R such that 109. Let I = ∫ dx and J = ∫ dx .
f (3x ) 0 x 0 x
lim = 1, p ′ ( x ) = p ′ (1 − x ), for all x ∈[0, 1], p (0) = 1
x→ ∞ f (x ) Then, which one of the following is true? [2008 AIEEE]
1 2 2
and p (1) = 41 . Then, ∫ p ( x ) dx equals (a) I > and J > 2 (b) I < and J < 2
0 [2010 AIEEE] 3 3
(a) 41 (b) 21 2 2
(c) I < and J > 2 (d) I > and J < 2
(c) 41 (d) 42 3 3

Answers
Exercise for Session 1 13. (c) 14. (c) 15. (c) 16. (a) 17. (a) 18. (b)
π+ 2 2 3 4 2 19. (a) 20. (b)
1. 2. 1 3. 2 4. − 5.
8 3 4 3
1 π Exercise for Session 6
6. − 1 7. loge 3 8. π 9. (b − a)
20 2 1. (c) 2. (a) 3. (c) 4. (b) 5. (c) 6. (c)
π −π π
12. 1 +  − 2  e1/
1 7. (a) 8. (a) 9. (c) 10. (a)
10. + log ( 2 − 1) 11. 2

2 2 2 2 4 
13. 10 ! 14. (d) 15. (d) Chapter Exercises
1. (a) 2. (c) 3. (a) 4. (a) 5. (a) 6. (a)
Exercise for Session 2 7. (d) 8. (d) 9. (a) 10. (a) 11. (c) 12. (c)
1. (c) 2. (a) 3. (a) 4. (c) 5. (a) 6. (c) 13. (d) 14. (c) 15. (c) 16. (d) 17. (d) 18. (a)
16 19. (b) 20. (a) 21. (d) 22. (c) 23. (d) 24. (a)
7. (b) 8. (a) 9. (a) 11. 12. 0 13. (c)
π 25. (a) 26. (b) 27. (a) 28. (d) 29. (a) 30. (c)
14. (d) 31. (a) 32. (b) 33. (a) 34. (b) 35. (b) 36. (a,b,c)
37. (c,d) 38. (a) 39. (a,b,c) 40. (c,d) 41. (a,b,d)
Exercise for Session 3 42. (b,d) 43. (c) 44. (d) 45. (a,b,c) 46. (d) 47. (a)
1. (c) 2. (a) 3. (b) 4. (b) 5. (a) 6. (d) 48. (b) 49. (a) 50. (a) 51. (b) 52. (b) 53. (c)
7. (b) 8. (d) 9. (c) 10. (b) 11. (b)
54. (a) 55. (a) 56. (c) 57. (a) 58. (b) 59. (a)
12. (b) 13. (c) 14. (b) 15. (d)
k 60. (a) 61. (c) 62. (c)
∑ (r − 1) ⋅ (r − (r − 1)1/ n ) + k (9 − k1/ n )
1/ n
16. 63. (A) → (q), (B) → (s), (C) → (p), (D) → (r)
r =1
64. (A) → (q), (B) → (r), (C) → (s), (D) → (p)
18. 3 19. 1 20. 2
65. (A) → (s), (B) → (p), (C) → (q)
66. (A) → (s), (B) → (r), (C) → (r), (D) → (q)
Exercise for Session 4
67. (8) 68. (0) 69. (3) 70. (2) 71. (2) 72. (a)
1. (b) 2. (c) 3. (c) 4. (a) 5. (c) 6. (a)
73. (1) 74. (a,b) 75. (b) 76. (a,c) 77. (a,b,c) 78. (c,d)
7. (a) 8. (c) 9. (c) 10. (b) 11. (b) 12. (b)
13. (c) 14. (c) 15. (d) 16. (c) 17. (a) 18. (d) 79. (0) 80. (9) 81. (a) 82. (b)
19. (c) 20. (a) 83. A→s, B→s, C→p, D→r 84. (d) 85. (2) 86. (b)
87. (a) 88. (8/3) 89. (a) 90. (b,d) 91. (c) 92. (b)
Exercise for Session 5 93. (4) 94. (c) 95. (a,b,c) 96. (d) 97. (c) 98. (c)
1. (c) 2. (b) 3. (a) 4. (a) 5. (a) 6. (b) 99. (b) 100. (c) 101. (d) 102. (d) 103. (a) 104. (c)
7. (a) 8. (a) 9. (c) 10. (c) 11. (c) 12. (a) 105. (d) 106. (c) 107. (b) 108. (d) 109. (b)
= log (1 + π 2 ) + 2 π (tan −1 x ) π4 − [log (1 + 16 ) − log (1 + π 2 )]
17
= log(1 + π 2 ) + 2 π (tan −1 4 − tan −1 π ) − log
1 + π2

Solutions = log
(1 + π 2 ) ⋅ (1 + π 2 )

= log 
 17 
17
 (1 + π 2 ) 2 
 + 2 π tan 


−1  4 − π 

 1 + 4π 
 4 −π 
+ 2 π tan −1 tan −1  
 1 + 4 π  

 (1 + π 2 ) 2  −1  c − π 
4 (y 2 − 4y + 5 )sin (y − 2 )dy On comparing with log   + bπ tan   , we get
1. Let I = ∫  a   1 + cπ 
0 (2y 2 − 8y + 1 )
Put y −2 =z a = 17, b = 2 and c = 4
a −5
⇒ dy = dz ∴ a − (2b + c ) = 17 − 8 = 9 and =3
c
2 (z + 1 )sin z
2
∴ I =∫ dz Thus, the number of ways to distribute 9 distinct bijective into
z = −2 (2z 2 − 7 ) 9!
3 persons equally is .
a
(3 !) 3
⇒ I = 0, as ∫ –a
f ( x ) dx = 0, when f (– x ) = − f ( x )
5. Put x = tan θ
(z + 1 )sin z
2
and f (z ) = dz is an odd function. π /2 dθ π /2 (cos θ )a π
(2z 2 − 7 ) ∴ I =∫ =∫ dθ ⇒ I =
0 1 + (tan θ )a 0 (sin θ )a + (cos θ )a 4
2. Hence, f ( x ) = x 2 + ax − b 3π /4 3π /4
6. I = ∫ (sin x + cos x ) dx + ∫ x (sin x − cos x ) dx
{
−a 0 0 144244 3
The solution of f ( x ) = minium f ( x ) is x = .
I
II
3π /4
2 =∫ (sin x + cos x ) dx
0
Given, f ( x ) minimum is at x = 0. 3π /4


−a
= 0 or a = 0
+ [ x ( − cos x − sin x )]30 π / 4 + ∫ 0
(sin x + cos x ) dx
2 3π /4
=2 ∫ (sin x + cos x ) dx + 0 = 2 [ − cos x + sin x ]30 π / 4
∴ f ( x ) = x 2 − b = 0 has roots α and β. 0

 1 1  3π
⇒ α = b and β = − b =2 + + + 1 = 2 ( 2 + 1 ) = 2 tan
 2 2  8
β − b
⇒ ∫ α
x 3dx = ∫ x 3dx = 0 1
tan −1 (nx )
7. We have, Cn = ∫ n 1
b
dx
 a
f ( x )dx = 0, when f ( − x ) = f ( x )  sin −1 (nx )
 ∫ −a
n +1

1 1 tan −1 (t )
3. Here, ∫
x
(3 − 2 sin t ) dt + ∫
2
y
cost dt = 0
Cn =
n ∫ n
n +1 sin −1 (t )
dt (put nx = t)
π /2 0

Differentiating both the sides, we get 1 tan −1 t


 dy 
Now, L = lim n 2 ⋅ Cn = lim n
n→ ∞ n→ ∞ ∫ n
sin −1 t
dt ( ∞ × 0 )
( 3 − 2 sin 2 x ) ⋅ 1 + (cosy )   = 0 n +1
 dx  tan t 1 −1

dy − 3 − 2 sin x
2 ∫
sin −1 t
dtn
n +1 0 
⇒ = L=  form
dx cosy 1 0 
− 3 Applying Leibnitz rule, n
 dy 
∴   = = 3 n
 dx  ( π , π ) −1 tan −1

n+1  1 
x2 0−  
4. Here, −4 ≤ x ≤ 4 ⇒ 0 ≤ <1 n  (n + 1 ) 2 
sin −1
17 n+1 π 2 1
L = lim = ⋅ =
x2  n→ ∞ 1 4 π 2
∴   = 0 and sin −1 (sin x ) is an odd function. − 2
 17  n
3 t 2 sin 2t
4 sin −1 (sin x ) cos−1 (cos x )
Let, I = ∫ dx + ∫
4
dx 8. ∫ dt = 0 as the integrand is an odd function.
−4 (1 + x 2 ) × 1 −4 (1 + x 2 ) × 1
−3 t2 + 1
cos−1 (cos x )
1 dt
= 0+2 ∫
π
dx Also, ∫0 t 2 + 2t cos α + 1
0 (1 + x 2 )
1
 π x 2π  1 t + cos α α
= tan −1 =
4
I = 2 ∫ dx + ∫ dx 
0 1 + x2 π (1 + x 2 ) sin α sin α 2 sin α
  0
150 Textbook of Integral Calculus

Thus, the given equation reduces to 1


14. Here, f ′ ( x ) = g′ ( x );
α sin α 1 + g 2 (x )
x2 ⋅ −2 = 0 ⇒ x = ± 2
2 sin α α  π g′ ( π / 2 )  π  π
Now, f ′   = ; g   = 0 = g′  
x  2 1 + g 2 (π / 2)  2  2
9. f ′ ( x ) = e g (x )
. g′ ( x ) and g′ ( x ) =
1 + x4
But g′ ( x ) = [1 + sin (cos2 x )] ( − sin x )
x  π
∴ f ′ (x ) = e g ( x ) ⋅ e g (2) = e 0 = 1 g′   = 1 ( −1 ) = − 1
1 + x4  2
f ′ (2 ) = e g ( 2 ) ⋅ g′ (2 ) = e 0 ⋅
2
=
2  π
Hence, Hence, f ′   = − 1 as h′ ( 0 + ) = − 1
17 17  2
1 t 
10. lim ln 
t→0 t ∫ 0
(1 + a sin bx )c / x dx

15. Clearly, f is an even function, hence
π π
t I 1 = ∫ f [cos ( π − x )] dx = ∫ f ( − cos x ) dx
∫ (1 + a sin b x ) c /x
dx 0 0

= ln lim 0 π
t→0 t = ∫ f (cos x ) dx
0
(1 + a sin bt )c /t lim a sin bt π /2 π /2
= ln lim = ln e t → 0 = ln eabc = abc ∴ I1 = 2 ∫ f (cos x ) dx ⋅ 2 ∫ f (sin x ) dx = 2 I 2
t→0 1 t /c 0 0

1 I1
11. Tr = ⇒ =2
2 I2
r  r 
⋅ n 3 + 4 Aliter Let u = cos x ⇒du = − sin x dx
n  n 
1 f (u )
1 4n
1 4 dx ∴ I1 = ∫ du
S=
n
∑  r 
2
=∫
0 x (3 x + 4 ) 2
−1
1 − u2
1 r
3 + 4 ⋅ ⇒ I1 = 2 ∫
1 f (u )
du ...(i)
 n  n 0
1 − u2
3 1
Put 3 x + 4 = t ⇒ ⋅ dx = dt Similarly, with sin t = t,
2 x 1 f (t )
4 I2 = ∫ dt ...(ii)
2 10 dt 2 1  2 1 1  2 6 1 1 − t2

0
= = = − = ⋅ =
3 4 t 2 3  t 10 3  4 10  3 40 10 I1
x From Eqs. (i) and (ii), =2
12. Given, f ( x ) = ∫ e dt; h( x ) = f (1 + g( x )); g( x ) < 0 for x > 0
t2
I2
−1
k
1 + g (x ) 16. Given, I 1 = ∫ x f ( x (1 − x )) dx and
Now, h( x ) = ∫
2
et dt = f (1 + g( x )) (given) 1 −k
−1
k
Differentiating, we get h′ ( x ) = e (1 + g ( x )) ⋅ g′ ( x )
2
I2 = ∫ f ( x (1 − x )) dx
1 −k

Now, h′ (1 ) = e (given)
Using King’s property I 1 = ∫
k
(1 − x ) f ( x (1 − x )) dx
(1 + g (1 )) 2 1 −k
∴ e ⋅ g′ (1 ) = e
k
⇒ (1 + g(1 )) 2 = 1 2I 1 = ∫ f ( x (1 − x )) dx = I 2
1 −k
⇒ 1 + g(1 ) = ± 1 I2
⇒ g(1 ) = 0 (not possible) ∴ =2
I1
or g(1 ) = − 2 1 1

13. Given, f ′ ( x ) = f ( x ) ⇒ f ( x ) = Ce x 17. A = ∫ (ax 2 + bx + c ) dx = 2 ∫ (ax 2 + c ) dx


−1 0

Since, f ( 0 ) = 1 ∴ 1 = f (0) = C Y
∴ f ( x ) = e x and hence g( x ) = x 2 − e x
1 1
Thus, ∫ f ( x ) g( x ) dx = ∫ ( x 2e x − e 2 x ) dx
0 0
1
1e 2 x 
= [ x e ] − 2 ∫ xe dx −  
2 x 1
0
x
X
0
 2 0 –1 O 1
1
= (e − 0 ) − 2 [( xe x )10 − (e x )10 ] − (e 2 − 1 ) a  1
2 =2 + c = [2a + 6c ]
1  3  3
= (e − 0 ) − 2 [(e − 0 ) − (e − 1 )] − (e 2 − 1 ) 1
2 ∴ A= [ f ( −1 ) + 4 f ( 0 ) + f (1 )]
1 2 3 3
=e − e −
2 2
Chap 02 Definite Integral 151

π  x2  Q π x sin x dx = π  ⇒ I = (2008 ) 2 ⇒ I = 2008


18. I (a ) = ∫  + a 2 sin 2 x + 2 x sin x dx  ∫0 
0  a2  n 1
22. Tn = 2 = ;
π3 πa 2  π2 a2  n +k x
2 2
n[1 + (k / n ) 2 x 2 ]
∴ I (a ) = 2
+ + 2π = π
3a 2 + 2  + 2 π n
3a 2   1 1 1 dt
S= ∑ k / n ) 2 x 2 ∫0 1 + t 2 x 2
1 + (123
=
 π a 
2
2π  n k =1
= π  −  +  + 2π
 3a 
t
 2 6
tan −1 ( x )
1
1 1 dt 1 
π a 2
= 2
x ∫ 0 t + (1 / x )
2 2
=
 x
tan −1 (tx ) =
 0 x
I (a ) is minimum when = ⇒ a2 = π
3a 2 3 23. Let y = f ( x ) ⇒ x = g(y ) and dy = f ′( x ) dx
2
⇒ a= π f
-1
f /g
3
2
Also, [ I (a ) ]min = 2 π + π 2 a f(a) = b f(a) = b
3
2 b f(0) = 0 f(0) = 0
t 2 
 2 − log 2 a ⋅ t  = 2 − log 2 (a ) (Qa > 0)
2
19.
 0 a b
I = ∫ f ( x ) dx + ∫ g(y ) dy ;y = f ( x )
⇒ (2 − 2 log 2 a ) = 2 − 2 log 2 a 0 0

⇒ 2 log 2 a = 2 log 2 a ⇒ a ∈ R +
a a
⇒ x = f −1 (y ) = g(y ) = ∫ f ( x )dx + ∫ xf ′ ( x ) dx
0 0
1/x ln (1 + t 2 )
20. Consider I = ∫ dt ...(i) a
= ∫ ( f ( x ) + xf ′ ( x )) dx = [ xf ( x )] a0 = a f (a ) = ab
−1 / x 1 + et 0

1/x ln (1 + t 2 ) x4 2x
I =∫
1/ 3
⇒ dt (using King’ property) 24. Let I = ∫ cos−1 dx ...(i)
−1 / x 1 + e −t −1 / 3 1 − x4 1 + x2
1/x ln (1 + t 2 ) et x4  −2 x 
I =∫
1/ 3
dt ...(ii) ⇒I = ∫ cos−1   dx (using King’s property)
−1 / x 1 + et −1 / 3 1 − x4 1 − x4
On adding Eqs. (i) and (ii), we get 1/ 3 x4  2x 
1/x 1/x ⇒ I =∫ 4 
π − cos−1  dx ...(ii)
2I = ∫ ln (1 + t 2 ) dt = 2 ∫ ln (1 + t 2 ) dt −1 / 3 1−x  1 − x4
−1 / x 0
1/x On adding Eqs. (i) and (ii), we get
⇒ I =∫ ln (1 + t 2 ) dt
0 1/ 3 x4 1/ 3 x4
1/x ∴ 2I = π ∫ dx ⇒ 2 I = 2 π ∫ dx
l = lim x ∫ ln (1 + t ) dt −1 / 3 1 − x 4 1 − x4
3 2
Hence, 0
x→∞ 0
1/x ∴ k=π
= lim
∫ 0
ln (1 + t 2 ) dt 0 
 form 25. Put ln x = t or x = et ⇒dx = et dt
x→∞ x −3 0  ∞ t t ∞
∴ I =∫ f (et + e −t ) e dt = ∫ f (et + e −t ) t dt = 0
Using L’ Hospital’s rule, −∞ et −∞

 1  1 (as the function is odd)


x 4 ln 1 + 2  ⋅ − 2
 x   x  1  1 Aliter I Put x = tan θ
l = lim = lim x 2 ln 1 + 2 
x→0 −3 3 x→∞  x  π /2  1  ln tan θ
∫0 f  tan θ + tan θ  tan θ ⋅ sec θ dθ
2
x2
1  1
= lim ln 1 + 2  (1 ∞ form)
3 x → ∞  x  π /2  1  ln tan θ
=∫ f  tan θ +  dθ
1  1  1 0  tan θ  sin θ cos θ
= lim x 2 1 + 2 − 1 =
x→∞ 3  x  3 Aliter II Put x = 1 / t ⇒ I = − 1 ⇒ 2 I = 0 ⇒ I = 0
1/λ
21. Put πx = t ⇒ dx =
dt 1/λ  (1 + x ) λ + 1 1 
 1

26. lim  ∫ (1 + x ) dx = lim 
λ 
π 
1 π 2008 π 1 2008 π
λ→ 0 0 λ→ 0
 λ + 1 0
∴ I = ⋅ ∫ t | sin t | dt = ∫ t | sin t | dt ...(i) 1/λ
π π 0 π 0  2λ + 1 − 1
= lim   (1 ∞ form)
1 2008 π λ→ 0  λ + 1 
⇒ I = ∫ (2008 π − t ) | sin t | dt ...(ii)
π 0 1  2λ

+ 1
−1 − λ −1

 2λ + 1 − 2 − λ 
lim lim  
λ → 0 λ λ +1  λ → 0  λ (λ + 1) 
On adding Eqs. (i) and (ii), we get =e =e
2008 π 2008 π π  2(2λ − 1) 

π ∫0
2I = | sin t | dt = (2008 ) 2 ⋅ ∫ | sin t | dt lim 
λ → 0 λ
− 1

ln  
4
e  4
0
=e = e 2 ln 2 − 1 = e =
e
152 Textbook of Integral Calculus

−1
27. Let y = f ( x ) ⇒ x = f −1 (y ) = g(y )
2
32. ∫ −2
(ax 2 − 5 ) dx + ∫1
(bx + c ) dx + 5
dy = f ′( x ) dx −1
5 5 =∫ (ax 2 − 5 − bx + c + 5 ) dx = 0
∴ I = ∫ f ( x ) dx + ∫ xf ′ ( x ) dx −2
1 1
Hence, ax 2 − bx + c = 0 has atleast one root in ( −2, − 1 ).
When y = 2, then x = 1 6

and when y = 1, then x = 5 33. I = ∫ (( x − 3 + 3 ) + ( 3 − x − 3 )) dx = 6 3


3
1
x y 34. I 1 = ∫ ({ x } + { x 3 }) { x 2 } dx
−1

1 f(1) = 2 1
1  x3  2
= − 2 ∫ { x 2 } dx = − 2 ×  = −
5 f(5) = 10
0
 3 0 3
sin 2 nx − sin 2 (n − 1 ) x
π /2
35. Jn − Jn − 1 = ∫ dx
5
0 sin 2 x
∴ I = ∫ ( f ( x ) + x f ′ ( x )) dx = | x f ( x ) |15 π / 2 sin (2n − 1 ) x ⋅ sin x
1
=∫ dx = I n
= 5 f (5 ) − f (1 ) = 5 ⋅ 10 − 2 = 48
0 sin 2 x
π /2
i.e. Jn − Jn − 1 = I n
28. I = ∫ sin x sin 2x sin 3x dx …(i) ⇒ Jn + 1 − Jn = I n + 1
0

π /2 π  π  π 
=∫ sin  − x sin 2  − x sin 3  − x dx
0 2  2  2  36.
Y
π /2
⇒ I =∫ − cos x sin 2 x cos 3 x dx …(ii) π
0 –
π /2
2
∴ 2I = ∫ − sin 2 x (cos x cos 3 x − sin 3 x sin x ) dx
0 1
π /2
=∫ − sin 2 x cos ( 4 x ) dx –π –π π
– 3–π

– –
0 2 4 O 2 4 π
X′ X
–π
π /2
=−∫ sin 2 x (2 cos 2 x − 1 ) dx
2
4
0

Put cos 2x = t ⇒ − sin 2 x × 2 dx = dt –1


−1
1 2t  –π
−1 3
1 2 –
∴ 2I = ∫ (2t − 1 ) dt =  − t 2
1 2 23 1
1 2 2 
=  ( − 1 ) 3 − ( −1 ) −  (1 ) 3 − 1  Y′
2 3  3  π /2 π /2
37. (a) I = ∫ ln (cot x ) dx ⇒ I = ∫ ln (tan x ) dx
1 2 2  1 2  1 0 0
= − +1− +1 = ⇒ I =
2  3  2 3  π /2
3 6 I =−∫ ln (cot x ) dx ⇒ I = − I ⇒ I = 0
0
d 2π 2π
29. Here, f ′ ( x ) = ( f ( x )) > 0; 2
f ( g( x ))| x = a (b) I = ∫ sin 3 x dx = − ∫ sin 3 x dx ⇒ I = 0
dx 0 0
g( x ) − g(a ) 1 − (1 / t 2 ) dt dt
= f ′ ( g( x )) lim
1 /e e
(c) At x = , I = ∫ =−∫
x →a x −a t e − 1 / t (ln t )1 / 3 1 /e t (ln t )1 / 3

It implies that g( x ) must be differentiable at x = a. I = − 1 or I = 0


x2
30. On putting t = and then solving, 1 + cos 2 x π 1 + cos 2 x
z (d)
2
>0 ⇒ ∫ 0 2
dx > 0
∞ ln x 2 π ln x
∫0 x 2 + t 2 dt = x 1
38. I = ∫ (10x 4 − 3x 2 − 1) dx = [2x 5 − x 3 − x ] 10 = 0
0
ln x ln 2 Since, f ( x ) is even, hence must have a root in ( −1, 0 ).
⇒ =
x x π /2
39. We have, I 1 = ∫ cos ( π sin 2 x ) dx
⇒ x = 2 and 4 i.e. two solutions. 0
π /2
xπ I1 = ∫ cos ( π cos2 x ) dx
31. Put =t 0
n π /2
16n 2 On adding, 2 I 1 = ∫ cos ( π sin 2 x ) + cos ( π cos2 x ) dx
π  πx  n 16n π 0
∴∫ π
cos dx = ∫ cos [t ] dt
0 2  n  π 0 2 =∫
π /2  π π 
2 cos   ⋅ cos  cos 2 x dx = 0
4n 2 4 π
0  2 2 
π ∫0
= cos [t ] dt = 0
2 ⇒ I1 = 0 ...(i)
Chap 02 Definite Integral 153

π /2 π /2 π /2
I2 = ∫ cos { π (1 − cos 2 x )} dx 1 1 dx
Now, ∴ 2I = ∫ dx = ∫
0 0 (sin x + cos x ) 2
2 0
 1 1 
2
π /2
 sin x + cos x
=−∫ cos ( π cos 2 x ) dx  2 2 
0
π 1 π /2 dx
1
2
=− ∫ cos ( π cos t ) dt (put 2x = t ) =
2 ∫ 0  π 
sin 2  + x
0

2 π /2 4 
=−
2 ∫ 0
cos ( π cos t ) dt = I 3
1 π /2 π  1  π 
π /2

2 ∫0
= cosec 2  + x dx = − cot  4 + x 
⇒ I2 + I3 = 0 4  2  0
π /2
I3 = − ∫ cos ( π sin t ) dt 1 1
0 =− [ −1 − 1 ] = 1 ⇒I =
∴ I2 + I3 = 0 ...(ii) 2 2
Hence, I 1 + I 2 + I 3 = 0 42. We have, f ( x ) = x 2 + ax 2 + bx 3
1 1
a=∫ t f (t ) dt and b = ∫
1
40. f ( x ) = 2 ∫ (1 − t ) cos ( xt ) dt where,
−1 −1
f (t ) dt
0 123 123
I II 1

 t
  1
 Now, a=∫ t [(a + 1 ) t 2 + bt 3 ] dt
sin xt 1 1 1 −1
= 2 (1 − t ) + ∫ sin xt dt  = 2  0 − 2 cos xt 
 1 2b

 x 0 x 0
  x 0 ⇒ a = 2b t 4dt = . ..(i)
0 5
1 − cos x 
f (x ) = 2 (x ≠ 0)
 x 2 
1 1
Again, b=∫ f (t ) dt = ∫ ((a + 1 ) t 2 + bt 3 ) dt
−1 −1
1 1
If x = 0, then f ( x ) = ∫ (1 − | t | ) dt = 2 ∫ (1 − t ) dt = 1 1
−1 0 ⇒ b = 2 ∫ (a + 1 ) t 2 dt
0
∴ option (c) is correct.
  1 − cos x  2(a + 1 )
⇒ b= ...(ii)
f ( x ) =  
2  , if x ≠ 0 3
Hence, x2 
 , if x = 0 5a 2(a + 1 )
 1
From Eqs. (i) and (ii), =
∴ f is continuous at x = 0. 2 3
∴ option (d) is correct. 5 2 2 11 2
⇒  − a= ⇒ a=
2 3  3 6 3
41. Given, f ( f ( x )) = − x + 1
Replacing x by f ( x ), we get 4 10
⇒ a= and b =
f ( f ( f ( x ))) = − f ( x ) + 1 11 11
f (1 − x ) = − f ( x ) + 1 1 4 1 10
f ( x ) + f (1 − x ) = 1 ...(i)
Hence, ∫ −1
t f (t ) dt =
11
and ∫ −1
f (t ) dt =
11
∴ f ( x ) = (a + 1 ) x 2 + bx 3
1 1
Now, J = ∫ f ( x ) dx = ∫ f (1 − x ) dx (using King’s property)
0 0
1 f (1 ) = (a + 1 ) + b
⇒ 2 J = ∫ ( f ( x ) + f (1 − x )) dx f ( −1 ) = (a + 1 ) − b
0
1 1 30
⇒ 2J = ∫ dx = 1 ⇒ J = ⇒ f (1 ) + f ( −1 ) = 2(a + 1 ) =
0 2 11
1 20
Put x = in Eq. (i), and f (1 ) − f ( −1 ) = 2b =
4 11
 1  1  1  3 43. Given, ( f ′ ( x )) 2 + ( g( x )) 2 = 1
F   + F 1 −  = 1 ⇒ F   + F   = 1
 4  4  4  4 x

Now I =∫
π /2 sin x
dx
f (x ) + ∫ 0
g(t ) dt = sin x (cos x − sin x )
0 (sin x + cos x ) 3 Differentiating both the sides, we get
π  f ′ ( x ) + g( x ) = cos 2 x − sin 2 x ...(i)
sin  − x
π /2 2  Squaring both the sides of Eq. (i), we get
I =∫ dx
0
 π  π 
3
( f ′ ( x )) 2 + ( g( x )) 2 = 2 f ′ ( x ) ⋅ g( x ) = 1 − sin 4 x
sin  − x + cos  − x 
 2 2  ⇒ 1 + 2 f ′ ( x ) ⋅ g( x ) = 1 − sin 4 x
π /2 cos x ∴ 2 f ′ ( x ) g( x ) = − sin 4 x
⇒ I =∫ dx
0 (cos x + sin x ) 3 Now, substituting g( x ) = −
sin 4 x
in Eq. (i), we get
2 f ′ (x )
154 Textbook of Integral Calculus

sin 4 x x2 x2 1
f ′ (x ) − = cos 2 x − sin 2 x 47. l = lim ⋅ ⋅
2 f ′ (x ) x→0 (a + x )
r 1/p
(1 − cos x ) x 2
Put f ′ (x ) = t 1 2
= 2 lim = 1/p
⇒ 2t 2 − 2 (cos 2 x − sin 2 x ) t − sin 4 x = 0 x→0 (a + x )
r 1/p
a
2 (cos 2 x − sin 2 x ) ± 4(1 − sin 4 x ) + 8 sin 4 x If p = 3 and l = 1, then 1 =
2
⇒ a =8
⇒ t=
4 a1 / 3
2 2
∴ 4t = 2 (cos 2 x − sin 2 x ) ± 4(1 − sin 4 x ) + 8 sin 4 x 48. If, p = 2 and a = 9, then l = 1 / 2 =
9 3
1
⇒ 2t = (cos 2 x − sin 2 x ) ± 1 + sin 4 x 49. Here, f ( x ) = e x ∫ et ⋅ f (t ) dt
0
1 4
4244 3
Taking + ve sign, 2t = cos 2 x − sin 2 x + cos 2 x + sin 2 x f ( x ) = Ae x
A ( say)
...(i)
⇒ t = cos 2 x ⇒ f (t ) = Aet
Taking − ve sign, t = − sin 2 x 1
where, A = ∫ et ⋅ f (t ) dt
Since, f ′ ( x ) = cos 2 x or f ′ ( x ) = − sin 2 x 0
1 1
1 cos 2 x ⇒ A = ∫ et ⋅ Aet dt; A = A ∫ e 2t dt
f ( x ) = sin 2 x + C1 or f (x ) = + C2 0 0
2 2
Now, A ∫ e 2t dt − 1  = 0 ⇒ A = 0, as ∫ e 2t dt ≠ 0
1 1

f (0) = 0  0  0

Hence, f ( x ) = 0 ⇒ f (1 ) = 0
⇒ C1 = 0 and C 2 = − 1 / 2 1


1
f ( x ) = sin 2 x or f (x ) =
cos 2 x − 1 50. Again, g( x ) = e x ∫ 0
et g(t ) dt + x
2 2 ⇒ g( x ) = Be x + x ...(ii)
If f ′ ( x ) = cos 2 x, then g( x ) = − sin 2 x ⇒ g(t ) = Bet + t
If f ′ ( x ) = − sin 2 x, then g( x ) = cos 2 x
1 t
where, B = ∫ et g(t ) dt ⇒ B = ∫ et ( Bet + t ) dt
0 0
1
i.e. f ( x ) = sin 2 x and g( x ) = − sin 2 x 1 1
2 ⇒ B=B ∫ 0
e 2t dt + ∫ 0
et ⋅ t dt
cos 2 x − 1
⇒ f (x ) = and g( x ) = cos 2 x 1 1 2 1
2 But ∫ 0
e 2t dt =
2
(e − 1 ) and ∫ 0
tet dt = 1
x
44. Consider f ( x ) = ∫ (t1
sin at + {
bt + {
c )dt B
−x 23
even odd even
∴ B = (e 2 − 1 ) + 1
2
x
= 2 ∫ (t sin at + c ) dt ⇒ 2 B = B(e 2 − 1 ) + 2
0
2
 cos at x
x cos at  ⇒ 3 B = Be 2 + 2 ⇒ B=
= 2  −t +∫ dt + | ct| x0  (using I.B.P.) 3 − e2
 a 0 0 a   2  x 2
 − x cos ax 1  From Eq. (ii), g( x ) =   e + x ⇒ g( 0 ) =
=2 + 2 sin ax + cx 3 − e2  3 − e2
 a a 
Also, f (0) = 0
f (x )  cos ax sin ax  2 2
∴ lim = lim 2 − + + c ∴ g( 0 ) − f ( 0 ) = − 0=
x→0 x x→0
 a a ⋅ ax  3 − e2 3 − e2
 1 1  2e 2 6
= 2 − + + c = 2c 51. g(2) = +2=
 a a 
3 − e2 3 − e2
π
= ⋅ n (tan −1 nx ) 0∞ =  − tan −1 an 
∞ n dx 1
45. Consider I = ∫ g( 0 ) 2 3 − e2 1
a  1 n 2  ∴ = ⋅ =
n x + 2
2 2
g(2 ) 3 − e 2
6 3
 n 
 π, if a < 0 Solutions (Q. Nos. 52 to 54)
π −1  
∴ L = lim  − tan an = π / 2 , if a = 0 We have the equations of the tangents to the curve
n→ ∞  2   x
 0 , if a > 0 y =∫ f (t )dt and y = f ( x ) at arbitrary points on them are
−∞
x
Y −∫ f (t ) dt = f ( x ) ( X − x )
2 2
t dt
x x ...(i)
∫0 (a + tr )1 / p (a + xr )1 / p
−x

46. lim = lim Using L’Hospital’s rule and Y − f (x ) = f ′ (x ) (X − x ) ...(ii)


x→0 bx − sin x x → 0 b − cos x
As Eqs. (i) and (ii) intersect at the same point on the X -axis
For existence of limit, lim denominator = 0 Putting Y = 0 and equating x-coordinates, we have
x→0
∴ b −1 = 0 ⇒ b =1
Chap 02 Definite Integral 155

x
Also, f ( x ) has y = 0 as asymptotes.
x−
f (x ) ∫ f (t ) dt
= x − −∞ ∴ f ( x ) can be shown as
f ′ (x f (x )
Y
f (x ) f ′ (x )
⇒ x
= π –1
e, tan–1
∫ −∞
f (t ) dt f (x )
2 2
x
⇒ ∫ −∞
f (t ) dt = cf ( x ) ...(iii)
X
O 1
0 1
As, f ( 0 ) = 1 ⇒ ∫ f (t ) dt = c × 1 ⇒ c =
−∞ 2
π 1
tan–1
–1
x 1 – e 2
∫−∞
2
⇒ f (t ) dt =f ( x ); differentiating both the sides and
2
integrating and using boundary conditions, we get f ( x ) = e 2 x ; Clearly, f ( x ) is neither injective nor subjective, also graph for
y = 2ex is tangent to y = e 2 x . [f (x)] can be shown, as
∴ Number of solutions = 1.
Cearly, f ( x ) is increasing for all x.
−2 x
∴ lim (e 2 x )e =1 (∞ 0 form)
x→∞

Solutions (Q. Nos. 55 to 57)


x2 0 1
We have, g( x ) = g( 0 ) + xg′ ( 0 ) + g′′ ( 0 ) = − bx 2 + cx − 6
2
h( x ) = g( x ) = 4 x 4 − ax 3 + bx 2 − cx + 6 = 0 has 4 distinct real
roots. Using Descarte’s rule of signs. –1
Given biquadratic equation has 4 distinct positive roots.
Let the roots be x1 , x 2 , x 3 and x 4 . ∞ 1 ∞ 1
1 2 3 4
62. ∫ 0
[ f ( x )]dx = ∫
0
− 1 ⋅ dx + ∫ 0 ⋅ dx = −( x ) = −1
1 0
+ + +
x1 x 2 x 3 x 4 24 T
Now, ≥4 63. (A) For a = 0, I = ∫ sin 2 x dx
4 x1 x 2 x 3 x 4 0

T 1 − cos 2 x
⇒ 2 ≥2 ⇒
1
=
2
=
3
=
4
=k =∫ dx
x1 x 2 x 3 x 4
0 2
T
x 1  T 1

1 2 3 4
⋅ ⋅ ⋅ = k4 = − sin 2 x = − sin 2T
 2 4  0 2 4
x1 x 2 x 3 x 4
24 1 1 sin 2T 1
⇒ = k4 ⇒ k = 2 ∴ L= − lim =
3/2 2 T→∞ 4 T 2
1 3
∴ Roots are , 1, and 2. Also, a = 20 and c = 25
T
(B) For a = 1, ∫ 4 sin 2 x dx ⇒ L = 2
2 2 0

dy T
58. At x = 1, y = 0, = 2x ⋅ (x 4 )2 − (x 2 )2 = 1 (C) For a = − 1, ∫ 0 dx = 0 ⇒ L = 0
dx 0

∴ Equation of the tangent is y = x − 1. (D) For a ≠ 0, − 1, 1,


dy T
59. = 4 x 3 (ln x 4 ) 2 − 3 x 2 (ln x 3 ) 2 I = ∫ (sin 2 x + sin 2 ax + 2 sin x ⋅ sin ax ) dx
dx 0

= 64 x 3 (ln x ) 2 − 27 x 2 (ln x ) 2 T  1 − cos 2 x 1 − cos2ax 


=∫  + + cos(a − 1 ) x − cos(a + 1 ) x dx
dy 0  
∴ lim+ = 64 lim+ x 3 (ln x ) 2 − 27 lim+ x 2 (ln x ) 2 = 0 2 2
x → 0 dx x→0 x→0
T
x
 1 1 sin (a − 1 ) x sin (a + 1 ) x 
f ( x ) = ∫ et
2
60. We have, /2
(1 − t 2 ) dt = x − sin 2 x − sin 2ax −
1  a −1 a+1 
2
4 4a 0
∴ f ′ ( x ) = [e x /2
(1 − x 2 )]2
T 1
Now, f ′ (1 ) = e1 / 2 ⋅ 0 = 0 ∴ L = lim − lim
T→∞ T T → ∞ T
 loge x  T
61. Here, f ( x ) = tan −1  ,0 < x < ∞ 1 1 sin (a − 1 ) x sin (a + 1 ) x 
 loge x + 1  4 sin 2 x − 4a sin 2ax + a −1

a+1 
 0
1
attains minimum at x = and maximum at x = e. ⇒ L =1
e
156 Textbook of Integral Calculus

( x + sin θ ) 3 
1
(1 + sin θ ) 3 − sin 3 θ  π 1+0  π
64. Q f (θ ) =  = f′   = = − 1, g   = 0
  2  π  2
 3 0 3 1 + g3  
1  2
( x + cos θ ) 3  (1 + cos θ ) 3 − cos3 θ  π
and g(θ ) =   = ∴ f′  = −1
 3 0 3  2
1 + 3 sin θ + 3 sin 2 θ 1 + 3 cos θ + 3 cos2 θ x
∴ f (θ ) =
3
and g(θ ) =
3
(B) We have, ∫ 0
f ( x ) dx = { f ( x )} 2
Differentiating both the sides, we get
Now, f (θ ) − g(θ ) = (sin θ − cos θ ) + (sin 2 θ − cos2 θ )
1
and f (θ ) − g(θ ) = (sin θ − cos θ ) (1 + sin θ + cos θ ) f (x ) = 2 f (x ) ⋅ f ′ (x ) ⇒ f ′ (x ) =
2
Now verify all matchings. 1
Integrating both the sides, f ( x ) = x + C
65. (A) Let a = 2008, then 2
where, f ( 0 ) = 0 ⇒ C = 0
1 x
I = ∫ (1 + axa ) e x dx
a
⇒ f (x ) = ⇒ f (2 ) = 1
0 2
1
I = ∫ (e x + axa e x ) dx (note : axa = ax ⋅ xa − 1 ) (C) Maximum when a = − 1, b = 2 ⇒a + b = 1
a a

0
1 sin 2 x b
∴ I = ∫ (e x + e x ⋅ x ⋅ axa − 1 ) dx + a + 2 = 0, then
a a
(D) If lim
0 x→0 x3 x
1
= ∫ ( f ( x ) + x f ′ ( x )) dx, where f ( x ) = e x
a

sin 2 x + ax 3 + bx
0 lim =0
Hence, I = [ xe x ] 10 = e
a x→0 x3

(B) I = I1 + I 2 For limit to exist 2 + b = 0 ⇒b = − 2

Consider I 2 = ∫
1 /e
− ln x dx sin 2 x + ax 3 − 2 x
1 ∴ lim =0
x→0 x3
Put − ln x = t ⇒ − ln x = t 2 ⇒ x = e − t
2

4
Using left hand rule and solving, we get a =
dx = − 2t e − t dt
2
⇒ 3
1
∴ I2 = ∫ {
1 1 1
∴ 3a + b = 2
3) dt = [te ] 0 − ∫0 e dt = e − ∫0 e dt
te −4
t2 −t 2 1 −t 2 −t 2
t ⋅ (1
−4
22
0
sin nx sin x sin 2 x
I II
67. Let B = ∑ n = + + ...
1 1 1 1 4 4 42
I 2 = ∫ e − x dx +
− ∫ e −t dt = = e −1
2
n =1
Hence,
e 0 0 e x 2x
and A = 1 + cos + cos 2 + ...
Note that, if f ( x ) = e − x , then f −1 ( x ) = − ln x
2
4 4
1 eix ei 2 x 1
 1  1  2  2  3  3  n  n
n 2 ∴ A + iB = 1 + + 2 +…=
(C) L = lim   ⋅   ⋅   K    4 4 eix
n → ∞  n  n  n  n 1−
  4
1 1 4
⇒ ln L = lim 2 Thus, B imaginary part of =
n→ ∞ n eix 4 − cos x − i sin x
1−
  1  2  3  n  4
1 ⋅ ln  n  + 2 ⋅ ln  n  + 3 ⋅ ln  n  + K + n ⋅ ln  n   4 sin x 4 sin x
  ∴ B= ⇒ f (x ) =
r r 17 − θ cos x 17 − θ cos x
General term of ln L = 2 ln
n n π π
4 sin x 1  25
1 n r r
and ∫ 0
f ( x )dx = ∫
17 − θ cos x 2
0
= log  
9
Sum = ⋅ ∑ ln  
n r = 1 n  n  5
= log   = log  
 m
1  3 n
1  x 2 ⋅ ln x  1 1 1
ln L = ∫ x ln x dx =   −2 ∫ x2 dx ∴ m + n =8
0
 2 0 0 x π /2 cos xdx
1 68. Here, I = ∫
 − π / 2 1 + 2 [sin −1 (sin x )]
1 x2  1
= 0 − =−
 2 2  0 4 −1 cos x 0 cos x dx 1 π / 2 cos x
=∫ dx + ∫ + ∫ cos x dx + ∫ dx
−1 / 4 − π / 2 −3 −1 −1 3
∴ L =e 0 1

g′ ( x ) 1 −π / 2 −1 1 1 π /2
66. (A) We have, f ′ ( x ) = = ∫ cos xdx + ∫ cos x dx + ∫ cos x dx + ∫ cos x dx
3 −1 0 0 3 1
1 + g 3 (x )
1 π /2 1

and g′ ( x ) = (1 + sin (cos2 x )) ( − sin x ) = ∫ cost ( −dt ) + ∫ cos (t ) ⋅( −dt )


3 0 0 1 1 π /2
+ ∫ cos x dx + ∫ cos x dx
(1 + sin (cos2 x )) ( − sin x ) 0 3 1
Hence, f ′ (x ) = =0
1 + g 3 (x )
Chap 02 Definite Integral 157

1 π / 2 sin 2 (n + 1 ) θ − sin 2 nθ 1  −t 3 t 5 t 7  1  −1 1 1 
1

π ∫0
69. Here, f ( x + 1) − f ( x ) = dθ =  − + ... = − + ...
sin 2 θ 2  3 10 21  0 2  3 10 21 
1 π / 2 sin (2n + 1 ) θ ⋅ sin θ
= ∫ dθ 1 1 1 1   1 1 1 
π 0 sin 2 θ =− + + + ... = − + + + ...
2 3 10 21  2 × 3 20 42 
1 π / 2 sin (2n + 1 ) θ
= ∫ dθ  1 1  1 1  1 1 
π 0 sin 2 θ = −  −  +  −  +  −  + ...
1 sin 2n θ ⋅ cos θ   2 3   4 5   6 7  
cos 2n θdθ
π /2 π /2

π  ∫0 ∫
=  dθ+
sin θ 0   2
= loge 2 − 1 = loge  
Using, cosθ + cos3θ + cos5θ + ....+ cos (2n − 1 ) θ e
n ⋅ (2θ ) ∴ K =2
sin
= 2 ⋅ cos(θ + (n − 1 ) θ ) π /2 x 2 cos x
 2θ  72. Let I =∫ dx …(i)
sin   − π /2 1 + e x
2
Q b f ( x ) dx = bf (a + b − x ) dx 
 ∫a ∫a
1  sin 2nθ 
i.e. cosθ + cos3θ + cos5θ + ...+ cos(2n − 1 ) θ =   
2  sin θ 
∴ f (n + 1 ) − f (n ) =
1 π /2 x 2 cos ( − x )
⇒ I =∫ dx …(ii)
π −π / 2 1 + e− x
2 π / 2 cosθ + cos3θ + ...+ cos (2n − 1 ) θ ⋅ cosθ dθ + 0 
 ∫0
On adding Eqs. (i) and (ii), we get

1 π /2
= ∫ {(2 cosθ cosθ ) + (2 cos3θ cosθ ) π /2  1 1 
π 0 2I = ∫ x 2 cos x  +  dx
− π /2
 1 + e x
1 + e− x 
+ … (2 cos (2n − 1 ) ⋅ cosθ )}dθ
1 π /2 π /2 π /2
= ∫ 1 ⋅ dθ, as ∫ cos2nθ dθ = 0 =∫ x 2 cos x ⋅ (1 ) dx
π 0 0 − π /2
1
∴ f (n + 1 ) − f (n ) =  a a 
2 Q ∫−a f ( x ) dx = 2 ∫ f ( x ) dx, when f ( − x ) = f ( x ) 
1  1 2  0

If n = 1 , f (2 ) − f (1 ) =  as f (1 ) =  ⇒ f (2 ) =
2  2 2 π /2
1 ⇒ 2I = 2∫ x 2 cos x dx
If n = 2, f (3 ) − f (2 ) = 0

3 2 n Using integration by parts, we get


⇒ f (3 ) = and so on ∴ f (n ) =
2 2 2 I = 2 [ x 2 (sin x ) − (2 x ) ( − cos x ) + (2 ) ( − sin x )] π0 / 2
15 3
+
f (15 ) + f (3 ) π2 
Hence, = 2 2 =3 ⇒ 2I = 2 − 2
f (15 ) − f (9 ) 15 9
−  4 
2 2
π2
∴ I = −2
h (x )
70. Here, g( x ) = ∫ e (1 + t ) dt
2

−2 4
⇒ g′ ( x ) = h′ ( x ) ⋅ e (1 +h ( x )) x t2
73. Let f ( x ) = ∫ dt
⇒ g′ ( x ) = h′ ( x ) ⋅ e 01 + t
4

g′ (2 ) = h′ (2 ) ⋅ e (1 +h ( x ))
2

x2
⇒ e 4 = 1 ⋅ e (1 +h ( 2 )) , given g′ (2 ) = e 4 and h′ (2 ) = 1 ⇒ f ′ (x ) = > 0, for all x ∈[ 0, 1 ]
2

1 + x4
∴ (1 + h(2 )) 2 = 4
∴f ( x ) is increasing.
⇒ 1 + h(2 ) = 2, − 2
At x = 0, f ( 0 ) = 0 and at x = 1,
∴ h(2 ) = − 3, 1
∴ Absolute sum for all possible values of h(2 ) = −3 + 1 = 2 1 t2
f (1 ) = ∫ dt
01 + t
4
π /2 1 π /2
71. Let I = ∫ sin x ⋅ log (sin x )dx = ∫ sin x ⋅ log (sin 2 x )dx
0 2 0
t2 1
1 π / 2 Because, 0< <
= ∫ sin x ⋅ log (1 − cos2 x )dx 1+t 4
2
2 0
Put cos x = t ⇒ − sin x dx = dt 1 1 t2 11

1 1
⇒ ∫ 0 ⋅ dt < ∫
0 1 + t
4
dt < ∫ ⋅ dt
∴ I = ∫ log (1 − t 2 ) dt 2
0 0

2 0
1
1 1  ( −t 2 ) 2 ( −t 2 ) 3  ⇒ 0 < f (1 ) <
= ∫  −t 2 − + ... dt 2
2  0 2 2 
158 Textbook of Integral Calculus

Thus, f ( x ) can be plotted as 192  4 1  3


⇒  x −  ≤ f ( x ) − f ( 0 ) ≤ 24 x −
4

Y 12  16  2
3
⇒ 16 x − 1 ≤ f ( x ) ≤ 24 x −
4 4

1/2 2
f( x) 1
Again, integrating between the limits to 1, we get
2
1 1 1  3
∫1 / 2 (16x − 1)dx ≤ ∫1 / 2 f (x )dx ≤ ∫1 / 2 24x − 2 dx
X 4 4
O 1

∴ y = f ( x ) and y = 2 x − 1 can be shown as 16 x 5  1


1
24 x 5 3 
1

Y
⇒  5 − x  ≤ ∫1 / 2 f ( x )dx ≤  5 − 2 x 
(1, 1)  1 / 2  1 / 2
 11 2 1  33 6
y = 2x −1 ⇒  +  ≤ ∫ f ( x )dx ≤  + 
5 5 1/2  10 10
1/2
y = f( x) ⇒ 2.6 ≤ ∫
1
f ( x ) dx ≤ 3 . 9
1/2

X′
O
X 76. Let I 1 = ∫ et (sin 6 at + cos6 at )dt
1 0
π 2π
= ∫ et (sin 6 at + cos6 at ) dt + ∫ et (sin 6 at + cos6 at ) dt
0 π
3π 4π
+ ∫ et (sin 6 at + cos6 at ) dt + ∫ et (sin 6 at + cos6 at ) dt
2π 3π
(0, − 1) Y′
∴ I1 = I 2 + I 3 + I 4 + I 5 …(i)
From the graph, the total number of distinct solutions for 2π
x ∈ ( 0, 1 ] = 1. [as they intersect only at one point] Now, I3 = ∫ e (sin at + cos at ) dt
t 6 6
π

74. Here, f ( x ) = 7 tan x + 7 tan x − 3 tan x −3 tan x


8 6 4 2
Put t = π + t ⇒ dt = dt
π
 −π π  ∴ I 3 = ∫ e π +t ⋅ (sin 6 at + cos6 at ) dt
for all x ∈ ,  0
 2 2
= et ⋅ I 2 …(ii)
∴ f ( x ) = 7 tan 6 x sec 2 x − 3 tan 2 x sec 2 x 3π
Now, I 4 = ∫ e (sin at + cos at ) dt
t 6 6
= (7 tan 6 x − 3 tan 2 x ) sec 2 x 2π

π /4 π /4 Put t = 2π + t ⇒dt = dt
Now, ∫ 0
x f ( x )dx = ∫
0
x (7 tan 6 x − 3 tan 2 x ) sec 2 x dx
I II ∴
π
I 4 = ∫ et + 2 π (sin 6 at + cos6 at ) dt
0
= [ x (tan 7 x − tan 3 x )] π0 / 4
π /4
= e 2π ⋅ I 2 …(iii)
−∫ 1 (tan 7 x − tan 3 x ) dx 4π
0 and I 5 = ∫ e (sin at + cos at ) dt
t 6 6

π /4
=0−∫ tan 3 x (tan 4 x − 1 )dx Put t = 3π + t
0
π
= –∫
π /4
tan 3 x (tan 2 x − 1 ) sec 2 x dx ∴ I 5 = ∫ e 3 π +t (sin 6 at + cos6 at ) dt
0
0

Put tan x = t ⇒sec 2 x dx = dt = e 3π ⋅ I 2 …(iv)


π /4 1 From Eqs. (i), (ii), (iii) and (iv), we get
∴ ∫ 0
x f ( x )dx = − ∫ t 3 (t 2 − 1 ) dt
0 I 1 = I 2 + e π ⋅ I 2 + e 2 π ⋅ I 2 + e 3 π ⋅ I 2 = (1 + e π + e 2 π + e 3 π ) I 2
1 4π
t 4 t 5 
1
= ∫ (t 3 − t 5 )dt =  −  = − =
1 1 1
∴ L=
∫ 0
et (sin 6 at + cos6 at )dt
0
 4 5  0 4 6 12 π

π /4 π /4 ∫ 0
et (sin 6 at + cos6 at )dt
∫ f ( x ) dx = ∫ (7 tan x − 3 tan x ) sec x dx
6 2 2
Also, 1⋅ (e 4 π − 1 )
0 0
= (1 + e π + e 2 π + e 3 π ) = for a ∈ R
1 e π −1
= ∫ (7t 6 − 3t 2 )dt = [t 7 − t 3 ]10 = 0
0
77. According to the given data, F ( x ) < 0, ∀x ∈ (1, 3)
192 x 3
75. Here, f ′ (x ) = We have, f (x ) = x F (x )
2 + sin 4 πx
3
⇒ f ′ (x ) = F (x ) + x F ′ (x ) …(i)
192 x 192 x 3
∴ ≤ f ′ (x ) ≤ ⇒ f ′ (1 ) = F (1 ) + F ′ (1 ) < 0
3 2 [given F (1 ) = 0 and F ′ ( x ) < 0]
1 Also, f (2 ) = 2 F (2 ) < 0 [using F ( x ) < 0,∀x ∈(1, 3 )]
On integrating between the limits to x, we get
2 Now, f ′ (x ) = F (x ) + x F ′ (x ) < 0
x 192 x 3 x x 192 x 3

∫1 / 2 3 dx ≤ ∫1 / 2 f ′ (x )dx ≤ ∫1 / 2 2 dx ⇒ f ′ (x ) < 0
[using F ( x ) < 0, ∀ x ∈ (1,3 )]
Chap 02 Definite Integral 159

3

78. Given, ∫ 1
x 2 F ′ ( x ) dx = − 12 ⇒ loge α + 1 −
4
=9
3
⇒ [ x 2 F ( x )]13 − ∫ 2 x ⋅ F ( x ) dx = − 12 81. Plan This type of question can be done using appropriate
1
3 substitution.
⇒ 9 F (3 ) − F ( 1 ) − 2 ∫ f ( x ) dx = − 12 [Q xF ( x ) = f ( x ), given] π /2
1 Given, I = ∫ (2 cosec x )17 dx
3 π /4
⇒ − 36 − 0 − 2 ∫ f ( x ) dx = −12 π /2 217 ( cosec x )16 cosec x ( cosec x + cot x )
=∫
1
dx
3
( cosec x + cot x )
∫ f (x )dx = −12
π /4

1
3 Let cosec x + cot x = t
and ∫1
x 3 F ′′ ( x )dx = 40 ⇒ ( − cosec x ⋅ cot x − cosec 2 x ) dx = dt
3 and cosec x − cot x = 1 / t
⇒ [ x F ′ ( x )] − ∫ 3 x 2 F ′ ( x ) dx = 40
3 3
1
1 1
⇒ 2 cosec x = t +
⇒ [ x 2 ( xF ′ ( x )] 13 − 3 × ( −12 ) = 40 t
16
⇒ { x 2 ⋅ [ f ′ ( x ) − F ( x )]} 13 = 4  1
1 t +  dt
⇒ 9 [ f ′ (3 ) − F (3 )] − [ f ′ (1 ) − F (1 )] = 4 ∴ I=−∫ 2 
17 t
2 +1
⇒ 9 [ f ′ (3 ) + 4 ] − [ f ′ (1 ) − 0 ] = 4  2  t
 
⇒ 9 f ′ (3 ) − f ′ (1 ) = − 32
[ x ], x ≤ 2 Let t = e u ⇒ dt = e u du. When t = 1, e u = 1 ⇒ u = 0
79. Here, f ( x ) = 
 0, x > 2 and when t = 2 + 1, eu = 2 + 1
2 x f (x 2 ) ⇒ u = ln( 2 + 1 )
∴ I =∫ dx
−1 2 + f ( x + 1 ) 0 eu du
⇒ I =−∫ 2(eu + e − u )16
0 x f (x 2 ) 1 x f (x 2 ) ln ( 2 + 1 ) eu
=∫ dx + ∫ dx ln ( 2 + 1 )
−1 2 + f (x + 1) 0 2 + f (x + 1) =2 ∫ (eu + e − u )16 du
0
2 2
2 x f (x ) 3 x f (x )
+ ∫
1 2 + f (x + 1)
dx + ∫ 2 2 + f (x + 1)
dx 82. Plan Newton-Leibnitz’s formula
d  ψ (x ) d  d 
x f (x 2 ) ∫ f (t ) dt  = f { ψ ( x )}  ψ ( x )  − f { φ ( x )}  φ ( x ) 
dx  
2
+ ∫ dx  dx 
φ ( x )
dx
3 2 + f (x + 1) x2
0 1 2 x ⋅1 3 2 Given, F (x ) = ∫ f ( t ) dt
= ∫ 0 dx + ∫ 0 dx + ∫ dx + ∫ 0 dx + ∫ 0 dx 0
−1 0 1 2+ 0 2 3
∴ F ′ (x ) = 2x f (x )
Also, F ′ (x ) = f ′ (x )
Q − 1 < x < 0 ⇒ 0 < x 2 < 1 ⇒ [ x 2 ] = 0, ⇒ 2x f ( x ) = f ′ ( x )
0 < x < 1 ⇒ 0 < x 2 < 1 ⇒ [ x 2 ] = 0, f ′ (x )
⇒ = 2x
 1 < x2 < 2 ⇒ [x 2 ] = 1 f (x )
1 < x < 2 ⇒
2 < x + 1 < 1 + 2 ⇒ f ( x + 1 ) = 0, f ′ (x )
2 < x < 3 ⇒2 < x 2 < 3 ⇒ f ( x 2 ) = 0,
⇒ ∫ f (x )
dx = ∫ 2 x dx

+c
⇒ In f ( x ) = x 2 + c ⇒ f ( x ) = e x
2
and 3 < x < 2 ⇒3 < x 2 < 4 ⇒ f ( x 2 ) = 0

2

2 x x2  1 1
2
f (x ) = K e x [ K = ec ]
⇒ I =∫ dx =   = (2 − 1 ) =
1 2  1
4 4 4 Now, f (0) = 1
∴ 4I = 1 ⇒ 4I − 1 = 0 ∴ 1=K
2

1  12 + 9 x 2  Hence, f (x ) = e x
α = ∫ e ( 9 x + 3 tan x ) 
−1
80. Here,  dx 4
0  1 + x2  F (2 ) = ∫ et dt = [et ]40 = e 4 − 1
0
Put 9 x + 3 tan −1 x = t 1 dx
 3  83. (A) Let I = ∫
⇒ 9 +  dx = dt
−1 1 + x2
 1+ x2 Put x = tanθ
9 + 3π / 4
∴ α=∫ e dt = [e ]
t t 9 + 3π / 4
0 =e 9 + 3π / 4
−1 ⇒ dx = sec 2 θ dθ
0

3π π /4 π
⇒ loge 1 + α = 9 + ∴ I = 2∫ dθ =
4
0 2
160 Textbook of Integral Calculus


1 dx f ( x ) dx = 0
(B) Let I =∫ (D) Plan
−a
0
1−x 2
If f ( − x ) = − f ( x ), i.e. f ( x ) is an odd function.
Put x = sinθ ⇒ dx = cosθ d θ
1 + x
π /2 π Let f ( x ) = cos 2 x log  
∴ I = ∫ 1 dθ = 1 − x
0 2
1 − x
f ( − x ) = cos 2 x log   = − f (x )
3
dx 3 1  1 + x 
(C) ∫ = log   1 + x
2 1 − x2 2   1 − x   2
Hence, f ( x ) is an odd function.
1  4   3   1   2 
= log   − log    = log   
1/2

2   −2   −1   2   3   So, ∫ −1 / 2
f ( x ) dx = 0
2 dx π π (A) → (q); (B) → (r); (C) → (p); (D) → (s)
(D) ∫ = [sec −1 x ]12 = − 0 =
1
x x −1
2 3 3 85. Plan Integration by parts
d 
84. (A) Plan
∫ f (x ) g(x ) dx = f (x ) ∫ g(x ) dx − ∫  dx [ f (x )] ∫ g(x ) dx dx
(p) A polynomial satisfying the given conditions is taken.
(q) The other conditions are also applied and the number of 1 d2
Given, I = ∫ 4 x 3 (1 − x 2 ) 5 dx
polynomial is taken out. 0 I dx 2 II

Let f ( x ) = ax 2 + bx + c  d  1 d
1

= 4 x 3 (1 − x 2 ) 5 − ∫ 12 x 2 (1 − x 2 ) 5 dx
f (0) = 0 ⇒ c = 0  dx  0 0 dx

[ ]
1
1  ax 3 bx 2  1
Now, ∫0 f ( x ) dx = 1 ⇒ 
 3
+
2 0
 =1 = 4 x 3 × 5 (1 − x 2 ) 4 ( − 2 x )
0

α β − 12 [ x (1 − x 2 ) 5 ]10 − ∫ 2 x (1 − x 2 ) 5 dx 
2
1
⇒ + = 1 ⇒ 2a + 3b = 6
3 2  0 
1
As a, b are non-negative integers. = 0 − 0 − 12 ( 0 − 0 ) + 12 ∫ 2 x (1 − x 2 ) 5 dx
0
So, a = 0, b = 2 or a = 3, b = 0 1
∴ f ( x ) = 2 x or f ( x ) = 3 x 2  (1 − x 2 ) 6   1
= 12 × −  = 12 0 + 6  = 2
(B) Plan Such type of questions are converted into only sine  6 0
or cosine expression and then the number of points of maxima π /2  2  π − x 
in given interval are obtained. 86. I = ∫ x + log  π + x   cos x dx
− π /2
f ( x ) = sin ( x 2 ) + cos ( x 2 )  
a

= 2
 1
 2
cos ( x 2 ) +
1
sin ( x 2 )


As, ∫ −a
f ( x ) dx = 0, when f ( − x ) = − f ( x )
2 π /2 π /2

 π π   π ∴ I =∫ x 2 cos x dx + 0 = 2 ∫ ( x 2 cos x ) dx
= 2 cos x 2 cos + sin sin ( x 2 ) = 2 cos  x 2 −  − π /2 0
 4 4   4 π /2

π π = 2 {( x 2 sin x ) 0π / 2 − ∫ 2 x ⋅ sin x dx }
For maximum value, x 2 − = 2nπ ⇒ x 2 = 2nπ +
0

4 4 π2 π /2 
π =2  − 2 {( − x ⋅ cos x ) π0 / 2 − ∫ 1 ⋅ ( − cos x ) dx } 
⇒ x=± , for n = 0  4 0

4
π 2
 π 2
 π 2

9π =2  − 2 (sin x ) 0π / 2  = 2  − 2 =  − 4
x=± , for n = 1  4   4   2 
4
So, f ( x ) attains maximum at 4 points in [ − 13, 13 ]. 87. Put x 2 = t ⇒ x dx = dt /2
(C) Plan dt
log 3
sin t ⋅
I =∫ 2
a a

(p) ∫ −a
f ( x ) dx = ∫
−a
f ( − x ) dx log 2 sin t + sin (log 6 − t )
...(i)
a a

∫ f ( x ) dx = 2 ∫ f ( x ) dx, if f ( − x ) = f ( x ) , i.e. f is an even b b


(q)
−a 0 Using, ∫ f (x ) dx = ∫ f (a + b − x ) dx
a a
function.
1 log 3 sin (log 2 + log 3 − t )
I =∫
2 3x 2
dx and I = ∫
2 3x 2
dx
=
2 ∫ log 2 sin (log 2 + log 3 − t ) + sin
dt
−2 1 + e x −2 1 + e − x

(log 6 − (log 2 + log 3 − t ))


2  3x 2 3 x 2 (e x )
⇒ 2I = ∫  + x  dx 1 sin (log 6 − t )
log 3
−2 1 + e x
e +1 =
2∫log 2 sin (log 6 − t ) + sin (t ) dt
2 2
2I = ∫ 3 x 2 dx ⇒ 2 I = 2 ∫ 3 x 2 dx log 3 sin (log 6 − t )
−2 0 ∴ I =∫ dt …(ii)
log 2 sin (log 6 − t ) + sin t
I = [ x 3 ]20 = 8
Chap 02 Definite Integral 161

On adding Eqs. (i) and (ii), we get g(n ) r


lim , where, is replaced with x.
1 log 3 sin t + sin (log 6 − t ) n→ ∞ n n
2I = ∫ dt
2 log 2 sin (log 6 − t ) + sin t Σ is replaced with integral.
1 1 1a + 2a + K + na 1
⇒ 2 I = (t ) log
log 2 =
3
(log 3 − log 2 ) Here, lim =
n → ∞ (n + 1 )a − 1 {(na + 1 ) + (na + 2 ) + K + (na + n )} 60
2 2
1  3 n
∴ I = log  
4  2 ∑r
r =1
a

1
⇒ lim =
1 x n→ ∞  n(n + 1 )  60
88. Given, f (1) = and 6 ∫ f (t )dt = 3x f ( x ) − x 3 , ∀ x ≥ 1 (n + 1 )a − 1 ⋅ n 2a +
3 1  2 
a
r
n
Using Newton-Leibnitz formula.
2∑  
Differentiating both sides  
r =1 n 1
⇒ 6 f ( x ) ⋅ 1 − 0 = 3 f ( x ) + 3 xf ′ ( x ) − 3 x 2 ⇒ lim a −1
=
n→ ∞
 1 60
1 1 +  ⋅ (2na + n + 1 )
⇒ 3 xf ′ ( x ) − 3 f ( x ) = 3 x 2 ⇒ f ’( x ) − f ( x ) = x  n
x
1 n r 
a
xf ’( x ) − f ’( x ) d x  1 1
⇒ =1 ⇒  =1 ⇒ lim 2 ∑    ⋅ nlim =
x2 dx  x 
n→ ∞ n
 r =1  n  → ∞  1
a −1
 1  60
1 +  ⋅ 2a + 1 + 
 n  n
On integrating both sides, we get
 1 1 1 1

f (x )
= x +c Q f (1 ) = ⇒ 2 ∫ ( xa ) dx ⋅ =
 3  1 ⋅ (2a + 1 ) 60
0
x
1 2 2 2 ⋅ [ xa +1 ] 10 1
= 1 + c ⇒ c = and f ( x ) = x 2 − x ⇒ =
3 3 3 (2a + 1 ) ⋅ (a + 1 ) 60
4 8 2 1
∴ f (2 ) = 4 − = ∴ =
3 3 (2a + 1 ) (a + 1 ) 60
Note Here, f (1 ) = 2, does not satisfy given function. ⇒ (2a + 1 ) (a + 1 ) = 120
1
∴ f (1 ) = ⇒ 2a 2 + 3a + 1 − 120 = 0
3
⇒ 2a 2 + 3a − 119 = 0
2 4 8
For that f ( x ) = x − x and f (2 ) = 4 − =
2
⇒ (2a + 17 ) (a − 7 ) = 0
3 3 3
−17
1 x 4 (1 − x ) 4 ⇒ a = 7,
89. Let I = ∫ dx 2
0 1 + x2
91. Here, f ( x ) + 2x = (1 − x ) 2 ⋅ sin 2 x + x 2 + 2x …(i)
1 ( x 4 − 1 ) (1 − x ) 4 + (1 − x ) 4
=∫ dx where, P : f ( x ) + 2 x = 2 (1 + x ) 2 …(ii)
0 (1 + x 2 )
∴ 2 (1 + x 2 ) = (1 − x ) 2 sin 2 x + x 2 + 2 x
1 1 (1 + x 2 − 2 x ) 2
= ∫ ( x 2 − 1 )(1 − x ) 4 dx + ∫ dx ⇒ (1 − x ) 2 sin 2 x = x 2 − 2 x + 2
0 0 (1 + x 2 )
⇒ (1 − x ) 2 sin 2 x = (1 − x ) 2 + 1 ⇒ (1 − x ) 2 cos2 x = − 1
1 4x 2 
= ∫ ( x 2 − 1 )(1 − x ) 4 + (1 + x 2 ) − 4 x +  dx which is never possible.
0
 (1 + x 2 )  ∴ P is false.
1 4  Again, let Q : h( x ) = 2 f ( x ) + 1 − 2 x (1 + x )
= ∫ ( x 2 − 1 )(1 − x ) 4 + (1 + x 2 ) − 4 x + 4 −  dx
0 1 + x2 where, h( 0 ) = 2 f ( 0 ) + 1 − 0 = 1
1  4  h(1 ) = 2 f (1 ) + 1 − 4 = − 3, as h( 0 ) h(1 ) < 0
= ∫  x 6 − 4x 5 + 5x 4 − 4x 2 + 4 −  dx
0  1 + x2 ⇒ h( x ) must have a solution.
 x 7 4x 6 5x 5 
1 ∴ Q is true.
4x 3
= − + − + 4 x − 4 tan −1 x  92. Here, f ( x ) = (1 − x ) 2 ⋅ sin 2 x + x 2 ≥ 0, ∀ x.
7 6 5 3 0
x  2 (t − 1 ) 
1 4 5 4 π  22 and g( x ) = ∫  − log t f (t ) dt
= − + − + 4 − 4  − 0 = − π 1  t + 1 
7 6 5 3 4  7
90. Converting infinite series into definite integral 2 ( x − 1 ) 
⇒ g′( x ) =  − log x  ⋅ f ( x ) …(i)
h(n )  ( x + 1 )  1 23
i.e. lim + ve
n→ ∞ n
h (n )
For g′( x ) to be increasing or decreasing,
1 r 2(x − 1)
n→ ∞ n ∑
lim f   = ∫ f ( x ) dx let φ( x ) = − log x
r = g (n )
 n (x + 1)
162 Textbook of Integral Calculus

4 1 − (x − 1)2 In = ∫
π sin nx
φ′( x ) = − = 95. Given dx …(i)
(x + 1) 2
x x (x + 1)2 −π (1 + π x ) sin x
b b
φ′( x ) < 0, for x > 1 ⇒ φ( x ) < φ(1 ) ⇒ φ( x ) < 0 …(ii) Using ∫ a
f ( x ) dx = ∫ f (b + a − x ) dx, we get
a
From Eqs. (i) and (ii), we get
g′( x ) < 0 for x ∈ (1, ∞ )
π π x sin nx
In = ∫ dx …(ii)
∴ g( x ) is decreasing for x ∈ (1, ∞ ).
−π (1 + π x ) sin x
 x − [ x ], if [ x ] is odd. On adding Eqs. (i) and (ii), we have
93. Given, f ( x ) =  π sin nx π sin nx
1 + [ x ] − x , if [ x ] is even. 2I n = ∫ dx = 2 ∫ dx
− π sin x 0 sin x
f ( x ) and cos π x both are periodic with period 2 and both are sin nx
even. [Q f ( x ) = is an even function]
10 10
sin x
∫ f ( x ) cos π x dx = 2 ∫ f ( x ) cos π x dx π sin nx
∴ ⇒ In = ∫ dx
− 10 0 0 sin x
π sin (n + 2 ) x − sin nx
Y Now, In + 2 − In = ∫ dx
0 sin x
π 2 cos (n + 1 ) x ⋅ sin x
=∫ dx
X 0 sin x
–10 – 9 – 2 – 10 1 2 9 10 π
π sin (n + 1 ) x 
= 2 ∫ cos (n + 1 ) x dx = 2   =0
0
 (n + 1 )  0
2

= 10 ∫ f ( x ) cos π x dx ∴ In + 2 = In …(iii)
π
sin nx
0
Since, In = ∫ dx ⇒ I 1 = π and I 2 = 0
1 1 1 0 sin x
Now, ∫ f (x ) cos π x dx = ∫ (1 − x ) cos π x dx = − ∫ u cos π u du
0 0 0
From Eq. (iii) I 1 = I 3 = I 5 = .... = π and I 2 = I 4 = I 6 = ... = 0
10 10
2 2 1
⇒ ∑I = 10 π and Σ I 2m = 0
∫ f (x ) cos π x dx = ∫ (x − 1) cos πx dx = − ∫ u cos π u du
2m + 1
and m =1
m =1

∴ Correct options are (a), (b), (c).


1 1 0
10 1
40
∴ ∫− 10f (x ) cos πx dx = − 20 ∫0 u cos π u du = π 2
n
n
96. Given, Sn = ∑ 2
k = 0 n + kn + k
2

π2
10
 
10 −∫10
⇒ f ( x ) cos π x dx = 4    
n
1  1  n
1 1 
= ∑ ⋅
k k 2  n → ∞ k∑
 < lim
x x
k =0 n  =0 n
  
2

94. Given ∫ 1 − { f ′(t )} 2 dt = ∫ f (t ) dt, 0 ≤ x ≤ 1 1+ + 2 k k
1 + +   
0 0  n n   n  
n 
Differentiating both sides w.r.t. x by using Leibnitz’s rule, 1
1 1  2  2  1  
we get =∫ dx =  tan −1  x +  
0 1 + x + x2  3  3  2   0
1 − { f ′ ( x )} 2 = f ( x ) ⇒ f ′ ( x ) = ± 1 − { f ( x )} 2
f ′ (x ) 2  π π π π
=
⋅ −  = i.e. Sn <
⇒ ∫ 1 − { f ( x )} 2
dx = ± ∫ dx 
3 3 6  3 3 3 3
π
⇒ sin −1 { f ( x )} = ± x + c Similarly, Tn >
3 3
Put x = 0 ⇒ sin −1 { f ( 0 )} = c 3π / 4 dx
⇒ c = sin −1 ( 0 ) = 0 [Q f ( 0 ) = 0 ] 97. I =∫ …(i)
π / 4 1 + cos x
∴ f ( x ) = ± sin x 3π / 4 dx
but f ( x ) ≥ 0, ∀ x ∈ [ 0, 1 ] I =∫ …(ii)
π / 4 1 − cos x
∴ f ( x ) = sin x
As we know that, Adding Eqs. (i) and (ii)
3π / 4
2 3π / 4
sin x < x, ∀ x > 0 2I = ∫ 2
dx ⇒ I = ∫ cosec 2 x dx
π /4
sin x π /4
 1 1  1 1
∴ sin   < and sin   < I = − (cot x ) 3ππ/ /44 = 2
 2 2  3 3
 1 1  1 1
98. I 4 + I 6 = ∫ ( tan 4 x + tan 6 x ) dx = ∫ tan 4 x sec 2 x dx
⇒ f   < and f   <
 2 2  3 3 1 1
= tan 5 x + c ⇒ a = ,b = 0
5 5
Chap 02 Definite Integral 163

1  x − a, x ≥a
(n + 1 ) ⋅ (n + 2 ) K (3n ) n 101. Plan Use the formula, | x − a | = 
99. (b) Let l = lim  − ( x − a ), x <a
n→ ∞  
 n 2n
to break given integral in two parts and then integrate
1
separately.
(n + 1 ) ⋅(n + 2 ) ... (n + 2n ) n
= lim 2
n→ ∞   π  x π x
 n 2n
1
∫ 0
1 − 2 sin  dx = ∫ | 1 − 2 sin | dx
 2 0 2
 n + 1  n + 2  n + 2n  n π
 x π  x
= lim     K  =∫ 1 − 2 sin  dx − ∫ π 1 − 2 sin  dx
 n  
3
n→ ∞ 
 n   n  0  2  3
 2
π
Taking log on both sides, we get π
 x 3  x π
1   1  2  2n   =  x + 4 cos  −  x + 4 cos  = 4 3 − 4 −
log l = lim log 1 +  1 +  ... 1 +    2 0  2 π 3
n→ ∞ n
   n   n   n   3
π /3 dx
 2n   102. Let I =∫ …(i)
1  1  2  π /6 1 +
⇒ log l = lim log 1 + n  + log 1 + n  + ...+ log 1 + n  
tan x
n→ ∞ n
  π /3 dx π /3 dx
∴ I =∫ =∫
1 2n  r 1 + cot x
⇒ log l = lim ∑ log 1 +  π 
π /6 π /6
n→ ∞ n  n 1 + tan  − x
r =1 2 
2
⇒ log l = ∫ log (1 + x ) dx π /3 tan xdx
0 ⇒ I =∫ …(ii)
2 π /6 1 + tan x
 1 
⇒ log l = log (1 + x ) ⋅ x − ∫ ⋅ x dx  On adding Eqs. (i) and (ii), we get
 1 + x 0 π /3
2 x + 1 −1 2I = ∫ dx ⇒ 2 I = [ x ]ππ // 36 dx
log l = [log (1 + x ) ⋅ x ] − ∫
π /6
⇒ 2
dx
1+x
0
0
1 π π  π
⇒ I = − =
2  1  2  3 6  12
⇒ log l = 2 ⋅ log 3 − ∫ 1 −  dx
0  1 + x Statement I is false.
log l = 2 ⋅ log 3 − [x − log 1 + x ]
b b

2
But ∫ f ( x )dx = ∫ f (a + b − x )dx is a true statement by
0 a a

⇒ log l = 2 ⋅ log 3 − [2 − log 3 ] property of definite integrals.


⇒ log l = 3 ⋅ log 3 − 2 ⇒ log l = log 27 − 2
x
103. Given, y = ∫ | t | dt
0
27
∴ l = e log 27 − 2 = 27 ⋅ e − 2 = 2 dy
e ∴ = | x | ⋅1 − 0 = | x | [by Leibnitz’s rule]
b b
dx
100. Central Idea Apply the property ∫ f (x )dx = ∫ f (a + b − x )dx
x

a a
Q Tangent to the curve y = ∫ | t | dt, x ∈ R are parallel to the
0
and then add. Let line y = 2 x
4 log x 2 ∴ Slope of both are equal ⇒ x = ± 2
I =∫ dx ±2
2 log x 2 + log(36 − 12 x + x 2 )
Points, y =∫ | t | dt = ± 2
0
4 2 log x Equation of tangent is
=∫ dx
2 2 log x + log(6 − x ) 2 y − 2 = 2 ( x − 2 ) and y + 2 = 2 ( x + 2 )
4 2 log x dx For x intercept put y = 0, we get
=∫
2 2[log x + log(6 − x )] 0 − 2 = 2 ( x − 2 ) and 0 + 2 = 2 ( x + 2 )
log x dx
4 ⇒ x=±1
⇒ I =∫ …(i) x
[log x + log(6 − x )]
2 104. Given integral g( x ) = ∫ cos 4 t dt
0
4 log(6 − x ) To find g( x + π ) in terms of g( x ) and g( π ).
⇒ I =∫ dx …(ii)
2 log(6 − x ) + log x x
g( x ) = ∫ cos 4 t d t
Q f ( x )dx = f (a + b − x )dx 
b b 0

 ∫a ∫a t =x +π
 ⇒ g( x + π ) = ∫ cos 4 t d t
t =0
On adding Eqs. (i) and (ii), we get x x +π
4 log x + log(6 − x ) = ∫ cos 4 t d t + ∫ cos 4 t d t
2I = ∫ dx 0 x
2 log x + log(6 − x )
= g( x ) + I 1 (say)
4 x +π π
⇒ 2 I = ∫ dx = [ x ] 4
2
I1 = ∫ cos 4 t d t = ∫ cos 4t dt
2 x 0

⇒ 2I = 2 ⇒ I = 1 (definite integral property)


= g( π )
164 Textbook of Integral Calculus

⇒ g( x + π ) = g( x ) + g( π ) x  5π 
Here, f (x ) = ∫ t sin t dt, where x ∈  0, 
But the value of I 1 is zero. 0  2
π
sin 4 t   sin 4 π sin 0 f ′ ( x ) = { x sin x − 0 } …(i)
⇒ I1 = = −  =0
 4  0  4 4  (using Newton-Leibnitz formula)
⇒ g( x + π ) = g( x ) − g( π ) ∴ f ′ ( x ) = x sin x = 0
In my opinion, the examiner has made this question keeping ⇒ sin x = 0
g( x ) + g( π ) as the only answer in his/her mind. However,
∴ x = π, 2π
he/she did not realise that the value of the integral I 1 is
1
actually zero. Hence, it does not matter whether you add to or f ′′( x ) = x cos x + sin x
subtract from g( x ). 2 x
1 8 log (1 + x ) At x = π, f ′′( π ) = − π < 0
105. I =∫ dx
0 (1 + x 2 ) ∴ Local maximum at x = π. At x = 2π, f ′′(2 π ) = 2 π > 0
Put x = tan θ ⇒ dx = sec 2 θ dθ
∴ Local minimum at x = 2π.
When x = 0 ⇒ tan θ = 0
∴ θ=0 107. We have, p′( x ) = p′ (1 − x ), ∀ x ∈ [ 0, 1 ], p ( 0 ) = 1, p (1 ) = 41
When x = 1 = tanθ ⇒ p ( x ) = − p (1 − x ) + C
π ⇒ 1 = − 41 + C
⇒ θ=
4 ⇒ C = 42
π / 4 8 log [1 + tan θ ] ∴ p ( x ) + p (1 − x ) = 42
∴ I =∫ ⋅ sec 2 θ dθ
0 1 + tan 2 θ Now,
1 1
I = ∫ p ( x ) dx = ∫ p (1 − x ) dx
π /4 0 0
I =8 ∫ log (1 + tan θ ) dθ …(i) 1 1
0 ⇒ 2 I = ∫ ( p ( x ) + p (1 − x )) dx = ∫ 42 dx = 42
0 0
a a
Using ∫ 0
f ( x ) dx = ∫ f (a − x ) dx, we get
0
⇒ I = 21
π
π /4  π  108. Let I = ∫ [ cot x ] dx …(i)
I =8 ∫ log 1 + tan  − θ  dθ 0
0
 4  π π
⇒ I =∫ [cot ( π − x )] dx = ∫ [ − cot x ] dx …(ii)
π /4  1 − tan θ  0 0
=8 ∫ log 1 +  dθ
0
 1 + tan θ  On adding Eqs. (i) and (ii),
π π π
π /4  2  2I = ∫ [cot x ] dx + ∫ [ − cot x ] dx = ∫ ( − 1 ) dx
=8 ∫ log   dθ …(ii) 0 0 0
0
 1 + tan θ  [Q[ x ] + [ − x ] = − 1, if x ∉ z and 0, if x ∈ z ]
π
Adding Eqs. (i) and (ii), we get = [− x ] = − π 0
π /4   2  π
2I = 8 ∫ ∴ I =−
log {1 + tan θ } + log 
+ θ
 d θ 2
0
 1 tan 
1 sin x 1 x
π /4 109. Since, I = ∫ dx < ∫ dx ,
⇒ I =4∫ log 2 dθ = 4 ⋅ log 2(θ ) π0 / 4 0 x 0 x
0

π  because in x ∈( 0, 1 ), x > sin x.


= 4 log 2 ⋅  − 0 = π log2
4  1 2
I < ∫ x dx = [ x 3 / 2 ]10 ⇒ I <
2
0 3 3
106. If φ( x ) and ψ( x ) are defined on [a, b] and differentiable for 1
1 cos x 1 −
every x and f(t) is continuous, then and J =∫ dx < ∫ x 2dx = 2
d  ψ (x )
0 x 0

f (t )dt  = f [ ψ( x )] ⋅ ψ( x ) − f [ φ( x )] φ( x )
d d
dx ∫ φ ( x )  dx dx J <2
CHAPTER

03
Area of
Bounded Regions
Learning Part
Session 1
● Sketching of Some Common Curves

● Some More Curves which Occur Frequently in Mathematics in Standard Forms

● Asymptotes

● Areas of Curves Given by Cartesian Equations

Session 2
● Area Bounded by Two or More Curves

Practice Part
● JEE Type Examples
● Chapter Exercises

Arihant on Your Mobile !


Exercises with the #L
symbol can be practised on your mobile. See inside cover page to activate for free.
Session 1
Sketching of Some Common Curves, Some More Curves
which Occur Frequently in Mathematics in Standard Forms,
Asymptotes, Areas of Curves Given by Cartesian Equations

Sketching of Some y Example 1 Mark the region represented by


Common Curves 3x + 4 y ≤ 12 .
Sol. Converting the inequality into equation, we get
For finding the area of a given region, we require the 3x + 4y = 12.
knowledge of some standard curves. This line meets the coordinate axes at (4, 0) and (0, 3),
respectively. Join these points to obtain straight line
represented by 3x + 4y = 12.
(i) Straight Line Y
Every first degree equation in x , y represents a straight line. (0, 3)
So, the general equation of a line is ax + by + c = 0. To draw
3x
a straight line find the points, where it meets with the +4
y=
coordinate axes by putting y = 0 and x = 0 respectively in its O
12 X
(4, 0)
equation.
By joining these two points we get the sketch of the line.
Sometimes the equation of a line is given in the form
y = mx . This equation represents a line passing through
This straight line divides the plane in two parts. One part
the origin and inclined at an angle tan − 1 m with the contains the origin and the other does not contains the
positive direction of X-axis. The equation of the form origin. Clearly, (0, 0) satisfy the inequality 3x + 4y < 12. So,
x = a and y = b represents straight lines parallel to Y-axis the region represented by 3x + 4y < 12 is region containing
and X-axis, respectively. the origin as shown in the figure.

Region Represented by a Linear Inequality


(ii) Circle
The general equation of a circle is
To find the region represented by linear inequality
ax + by ≤ c and ax + by ≥ c , we proceed as follows x 2 + y 2 + 2 gx + 2 fy + c = 0
(i) Convert the inequality into equality to obtain a ∴ The second degree equation in x , y Y
linear equation in x , y . such that coeff. of x 2 = coeff. of y 2 c r
(ii) Draw the straight line represented by it. and there is no term containing xy; it (h, k)
(iii) The straight line obtained in (ii) divides the always represents a circle. To draw a
XY-plane in two parts. sketch of a circle, we write the O
X
equation instandard form
To determine the region represented by the inequality
choose some convenient points; e.g. origin or some points ( x − h ) 2 + (y − k ) 2 = r 2 , whose centre Figure 3.1

on the coordinate axes. is (h, k ) and radius is r.


If the coordinates of a point satisfy the inequality, then Remarks
region containing the points is the required region, 1. The inequality ( x − a) 2 + ( y − b) 2 < r 2 represents the interior of
otherwise the region not containing the point is required a circle.
region. 2. The inequality ( x − a) 2 + ( y − b) 2 > r 2 represents the exterior of
a circle (i.e. region lying outside the circle).
Chap 03 Area of Bounded Regions 167

(5) y 2 = − 4a ( x − h ); a, h > 0
(iii) Parabola Y
It is the locus of points such that its distance from a fixed
point is equal to its distance from a fixed straight line.
Taking the fixed straight line x = − a, a > 0 and fixed point v
X
(0, 0) (h, 0)
(a, 0 ), we get the equation of parabola y 2 = 4ax .

Steps to Sketch the Curve Figure 3.6


(i) It passes through (0, 0).
(ii) It is symmetrical about axis of X. (6) y 2 = − 4a ( x + h ); a, h > 0
(iii) No part of the curve lies on the negative side of axis Y

of X.
(iv) Curve turns at (0, 0) which is called the vertex of the v
X
(–h, 0) (0, 0)
curve.
(v) The curve extends to infinity. It is not a closed curve.
(1) y 2 = 4ax (Standard equation of parabola)
Figure 3.7
Y y2 =4ax
(7) x 2 = 4ay ; a > 0
Y
v
X
(0, 0)

X
Figure 3.2 v (0, 0)

(2) y 2 = 4a ( x − h ); where a and h are positive.


Y Figure 3.8

(8) x 2 = 4a (y + k ); a > 0, k > 0


O v
X
(0, 0) (h, 0) Y

Figure 3.3
(3) y = 4a ( x + h ); where a and h are positive.
2
X
(0, 0)
Y

v (0, –k)
v
X Figure 3.9
(–h, 0) O
(9) x = 4a (y − k ); a, k > 0
2

Y
Figure 3.4
(4) y 2 = − 4ax ; a > 0
Y

v v (0, k)
X X
(0, 0) (0, 0)

Figure 3.10
Figure 3.5
168 Textbook of Integral Calculus

(10) x 2 = − 4ay ; a > 0 Definition 2. An ellipse is the locus of a moving point


such that the sum of the its distances from two fixed
Y
points is constant. The two fixed points are the two foci of
the ellipse. To plot the ellipse, we can use the
peg-and-thread method described earlier.
v Stantard Equation
X
(0, 0) x2 y2
+ =1
a2 b2
If a > b If a < b
Vertices (a , 0 ) and ( − a , 0 ) ( 0 , b ) and ( 0 , − b )
Figure 3.11
Foci (ae , 0 ) and ( − ae , 0 ) ( 0 , be ) and ( 0 , − be )

(11) x = − 4a (y + k ); a, k > 0
2 Major axis 2a (along x-axis) 2b (along y-axis)
Minor axis 2b (along y-axis) 2a (along x-axis)
Y Directrices a a b b
x = and x = − y= and y = −
e e e e
Eccentricity e b2 a2
1− 2
1−
a b2
X Latus-rectum 2b 2 2a 2
(0, 0)
v (0,–k) a b
Focal distances of ( x , y ) a ± ex b ± ey

And lastly, if the equation of the ellipse is


Figure 3.12 (x − α )2 (y − β) 2
=1 +
a2 b2
(12) x = − 4a (y − k ); a, k > 0
2
instead of the usual standard form, we can use the
Y transformation X → x − α and Y → y − β (basically a
translation of the axes so the axes so that the origin of the
v (0, k) new system coincides with (α, β). The equation then
becomes
X2 Y2
(0, 0)
X + =1
a2 b2
We can now work on this form, use all the standard
formulae that we’d like to and obtain whatever it is that
Figure 3.13 we wish to obtain. The final result (in the x-y system) is
obtained using the reverse transformation x → X + α and
y → Y + β.
(iv) Ellipse x2 y2
(1) 2 + 2 = 1; a > 0, b > 0 (Standard equation of the
Basics of Ellipse a b
Definition 1. An ellipse is the locus of a moving point ellipse)
such that the ratio of its distance from a fixed point to its Y
distance from a fixed line is a constant less than unity. (0,b)

This constant is termed the eccentricity of the ellipse. The


fixed point is the focus while the fixed line is the directrix. X
(–a,0) (0,0) (a, 0)
The symmetrical nature of the ellipse ensures that there
will be two foci and two directrices. (0,–b)

Figure 3.14
Chap 03 Area of Bounded Regions 169

x2 y2 Latus-Rectum
(2) 2
+ 2
= 1; b > a > 0 (Conjugate ellipse)
a b The chord(s) of the hyperbola passing through the focus F
Y (or F ′) and perpendicular to the transverse axis. The
(0,b) length of the latus-rectum can be evaluated by
substituting x = ± ae in the equation for the hyperbola :
x2 y2
− =1
(–a,0) (0, 0) (a,0)
X a 2 a 2 (e 2 − 1)
y2
⇒ e2 − =1
a 2 (e 2 − 1)
(0,–b)
a2 ⋅b 4 b4
⇒ y 2 = a 2 (e 2 − 1) 2 = =
Figure 3.15 a4 a2
(x − h)2 (y − k ) 2 b2
(3) + = 1; a > 0, b > 0 and a > b ⇒ y =± .
a2 b2 a
Y 2b 2
Thus, the length of the latus-rectum is .
a
y=k b a
c (h, k) We discussed in the unit of Ellipose that an ellipse with
centre at (α, β) instead of the origin and the major and
minor axis parallel to the coordinate axes will have the
equation
X
O x=h ( x − α ) 2 (y − β) X2 Y2
+ = 1 or + =1
a2 b2 a2 b2
Figure 3.16
where X → x − α and Y → y − β.
(x − h) 2
(y − k ) 2
(4) 2
+ = 1, where a < b The same holds true for a hyperbola. Any hyperbola with
a b2 centre at (α, β) and the transverse and conjugate axis
Y
parallel to the coordinate axes will have the form
b (x − α )2 (y − β) 2
− =1
y=k a2 b2
X2 Y2
or =1 −
a2 b2
O
X
where X → x − α and Y → y − β.
Figure 3.17 We can, using the definition of a hyperbola, write the
equation of any hyperbola with an arbitrary focus and
(v) Hyperbola directrix, but we will rarely have the occassion to use it.
A hyperbola is the locus of a moving point such that the x2 y2
(1) − = 1 (Standard equation of hyperbola)
difference of its distances from two fixed points is always a2 b2
constant. The two fixed point are called the foci of the
hyperbola. Constrast this with the definition of the ellipse Y
where we had the sum of focal distances (instead of
difference) as constant. As in the case of the ellipse, we have
Focal distance of P ( x , y ) X
(–a, 0) (0, 0) (a, 0)
 a
d 1 = e ( PF ) = e  x −  = ex − a
 e
 a
d 2 = e ( PF ′ ) = e  x +  = ex + a
 e Figure 3.18
170 Textbook of Integral Calculus

y2 x2
(2)
b2

a2
= 1 (Conjugate hyperbola) Some More Curves which Occur
Y Frequently in Mathematics in
Standard Forms
1. y = –x Y y=x
X
(0, 0)

X
Figure 3.19 (0, 0)

(3) xy = c 2 (Rectangular hyperbola)


Y
Figure 3.23
(c, c) Modulus function, y = | x |
X 2. Y
(0, 0)
(–c, –c)
2

1
–2 –1
Figure 3.20 X
1 2 3
(x − h) 2
(y − k ) 2 –1
(4) 2
− 2
= 1; a > b > 0
a b –2

y=k c
a a Figure 3.24
(y, k)
Greatest integer function y =[ x ]
3. y = ex
Y
X
O
y=x
x=h

Figure 3.21 (0, 1) y = ln x

(x − h) 2
(y − k ) 2
(5) 2
− = −1 (0, 0) (1, 0)
X
a b2
Y

b Figure 3.25
y=k c
(x, h) 4. Y
b
(0, 1)
X y = e–x
2
O

X
x=h

Figure 3.22 Figure 3.26


Chap 03 Area of Bounded Regions 171

5. Y (b) Symmetry about Y -axis If all powers of x in the


equation of the given curve are even, then it is symmetric
(1, 1) about Y-axis. e.g. x 2 = 4ay is symmetric about Y-axis.
a<1
a>1 a= 1
X
(c) Symmetry in opposite quadrants If by putting − x
(0, 0) for x and − y for y, the equation of curve remains same,
then it is symmetric in opposite quadrants.
e.g. xy = c 2 , x 2 + y 2 = a 2 are symmetric in opposite
quadrants.
Figure 3.27
Y (d) Symmetric about the line y = x If the equation of a
given curve remains unaltered by interchanging x and y,
(1, 0)
1
then it is symmetric about the line y = x which passes
y= through the origin and makes an angle of 45° with
1+x2
X positive direction of X-axis.
(0, 0)

(ii) Origin and Tangents at the Origin


See whether the curve passes through origin or not. If the
Figure 3.28 point (0, 0) satisfies the equation of the curve, then it
passes through the origin and in such a case to find the
1
y= ,α >0 equations of the tangents at the origin, equate the lowest
xα degree term to zero. e.g. y 2 = 4ax passes through the
7. The Astroid origin. The lowest degree term in this equation is 4ax .
Its cartesian equation is x 2 / 3 + y 2 / 3 = a 2 / 3 Equating 4ax to zero, we get x = 0.
Its parametric equation is x = a cos 3 t , y = a sin 3 t So, x = 0 i.e. Y-axis is tangent at the origin to y 2 = 4ax .
and it could be plotted as
(iii) Points of Intersection of Curve
with the Coordinate Axes
a By putting y = 0 in the equation of the given curve, find
a
a points where the curve crosses the X-axis. Similarly, by
a putting x = 0 in the equation of the given curve we can
find points where the curve crosses the Y-axis.
e.g. To find the points where the curve
Figure 3.29 xy 2 = 4a 2 (2a − x ) meets X-axis, we put y = 0 in the
equation which gives 4a 2 (2a − x ) = 0 or x = 2a. So the
curve xy 2 = 4a 2 (2a − x ), meets X-axis at (2a, 0 ). This
Curve Sketching curve does not intersect Y-axis, because by putting x = 0
For the evaluation of area of bounded regions it is very in the equation of the given curve get an absurd result.
essential to know the rough sketch of the curves. The
following points are very useful to draw a rough sketch of
a curve. (iv) Regions where the Curve
Does Not Exist
(i) Symmetry Determine the regions in which the curve does not exist.
(a) Symmetry about X-axis If all powers of y in the For this, find the value of y in terms of x from the
equation of the given curve are even, then it is symmetric equation of the curve and find the value of x for which y
about X-axis, i.e. the shape of the curve above X-axis is is imaginary. Similarly, find the value of x in terms of y
and determine the values of y for which x is imaginary.
exactly identical to its shape below X-axis. e.g. y 2 = 4ax is
The curve does not exist for these values of x and y.
symmetric about X-axis.
172 Textbook of Integral Calculus

e.g. The values of y obtained from y 2 = 4ax are So, the origin is a point of inflexion.
imaginary for negative values of x . So, the curve does not Y y = x3
exist on the left side of Y-axis. Similarly, the curve
a 2 y 2 = x 2 (a − x ) does not exist for x > a as the values of y
are imaginary for x > a.
X' X
O

(v) Special Points


dy
Find the points at which = 0. At these points the Y'
dx
tangent to the curve is parallel to X-axis. (v) As x increases from 0 to ∞, y also increases from 0 to
dx ∞. Keeping all the above points in mind, we obtain a
Find the points at which = 0. At these points the sketch of the curve as shown in figure.
dy
tangents to the curve is parallel to Y-axis. y Example 3 Sketch the curve y = x 3 − 4 x .
Sol. We note the following points about the curve
(vi) Sign of dy/dx and Points of (i) The equation of the curve remains same, if x is
Maxima and Minima replaced by ( − x ) and y by ( − y ) , so it is symmetric in
opposite quadrants.
dy
Find the interval in which > 0. In this interval, the Consequently, the curve in the first quadrant is
dx identical to the curve in third quadrant and the curve
function is monotonically increasing, find the interval in in second quadrant is similar to the curve in fourth
dy quadrant.
which < 0. In this interval, the function is
dx (ii) The curve passes through the origin. Equating the
monotonically decreasing. lowest degree term y + 4 x to zero, we get y + 4 x = 0
dy d 2y or y = − 4 x . So, y = − 4 x is tangent to the curve at the
Put = 0 and check the sign of 2 at the points so origin.
dx dx (iii) Putting y = 0 in the equation of the curve, we obtain
obtained to find the points of maxima and minima. x 3 − 4 x = 0 ⇒ x = 0, ± 2. So, the curve meets X -axis at
Keeping the above facts in mind and plotting some points (0, 0), (2, 0), (–2, 0).
on the curve one can easily have a rough sketch of the Putting x = 0 in the equation of the curve, we get
curve. Following examples will clear the procedure. y = 0. So, the curve meets Y -axis at (0, 0) only.
dy
(iv) y = x 3 − 4 x ⇒ = 3x 2 − 4
y Example 2 Sketch the curve y = x 3 . dx
Sol. We observe the following points about the given curve dy
Now, > 0 ⇒ 3x 2 − 4 > 0
(i) The equation of the curve remains unchanged, if x dx
is replaced by − x and y by − y. So, it is symmetric  2   2 
in opposite quadrants. Consequently, the shape of ⇒  x−   x+  >0
 3   3 
the curve is similar in the first and the third
2 2
quadrants. ⇒ x<− or x > (using number line rule)
(ii) The curve passes through origin. Equating lowest 3 3
degree term y to zero, we get y = 0 i.e. X -axis is the and
dy
<0⇒−
2
<x<
2
tangent at the origin. dx 3 3
(iii) Putting y = 0 in the equation of the curve, we get So, the curve is decreasing in the interval
x = 0. Similarly, when x = 0, we get y = 0. So, the curve 2 2
meets the coordinate axes at (0, 0) only. ( − 2 / 3, 2 / 3 ) and increasing for x > or x < − ⋅
3 3
dy d 2y d 3y 2 2
(iv) y = x 3 ⇒ = 3x 2 , 2 = 6x and 3 = 6 x=− is a point of local maximum and x = is
dx dx dx 3 3
dy d 2y d 3y point of local minimum.
Clearly, = 0 = 2 at the origin but 3 ≠ 0.
dx dx dx 2 16
When x = , then y = −
3 3 3
Chap 03 Area of Bounded Regions 173

2 16 The lowest degree term is x + y . Equating it to zero,


When x=− , then y =
3 3 3 we get x + y = 0 as the equation of tangent at the
origin.
Y
) – 23 , 3163 ) (iii) Putting y = 0 in the equation of curve, we get
x 2 − x = 0 ⇒ x = 0, 1. So, the curve crosses X -axis at
(0, 0) and (1, 0).
X Putting x = 0 in the equation of the curve, we obtain
(–2, 0) O (2, 0) y = 0. So, the curve meets Y -axis at (0, 0) only.
Y
2 , – 16
) 3 3 3
)
Keeping above points in mind, we sketch the curve as
X
shown in figure. (0, 0) (1, 0)

y Example 4 Sketch the curve y = ( x − 1) ( x − 2) ( x − 3).


Sol. We note the following points about the given curve ) 12 , – 14 )
(i) The curve does not have any type of symmetry about
the coordinate axes and also in opposite quadrants. dy d 2y
(iv) y = x 2 − x ⇒ = 2x − 1 and 2 = 2
(ii) The curve does not pass through the origin. dx dx
(iii) Putting y = 0 in the equation of the curve, we get dy 1
Now, =0⇒x =
( x − 1) ( x − 2) ( x − 3) = 0 ⇒ x = 1, 2, 3. So, the curve dx 2
meets X -axis at (1, 0), (2, 0) and (3, 0). 1 d 2y
At x= , >0
Putting x = 0 in the equation of the curve, we get y = − 6. 2 dx 2
So, the curve crosses Y -axis at (0, − 6).
1
We observe that So, x = is point of local minima.
2
x <1 ⇒ y<0
dy 1
1< x < 2 ⇒ y>0 (v) > 0 ⇒ 2x − 1 > 0 ⇒ x >
dx 2
2< x < 3 ⇒ y<0
1
and x >3 ⇒ y>0 So, the curve increases for all x > and decreases for
2
Y 1
all x < . Keeping above points in mind, we sketch the
2
curve as shown in figure.

X
y Example 6 Sketch the curve y = sin 2x .
O (1, 0) (2,0) (3, 0)
Sol. We note the following points about the curve
(i) The equation of the curve remains unchanged, if x is
(0, – 6) replaced by ( − x ) and y by ( − y ), so it is symmetric in
opposite quadrants. Consequently, the shape of the
Clearly, y decreases as x decreases for all x < 1 and y curve is similar in opposite quadrants.
increases as x increases for x > 3. (ii) The curve passes through origin.
Keeping all the above points in mind, we sketch the curve (iii) Putting x = 0 in the equation of the curve, we get
as shown in figure. y = 0. So, the curve crosses the Y -axis at (0, 0) only.
Putting y = 0 in the equation of the curve, we get
y Example 5 Sketch the graph for y = x 2 − x .
sin 2x = 0 ⇒ 2x = nπ, n ∈ Z
Sol. We note the following points about the curve

(i) The curve does not have any kind of symmetry. ⇒ x= , n ∈Z
2
(ii) The curve passes through the origin and the tangent
at the origin is obtained by equating the lowest degree So, the curve cuts the X -axis at the points
term to zero. …, ( − π, 0), ( − π / 2, 0), (0, 0), ( π / 2, 0), ( π, 0), …
174 Textbook of Integral Calculus

dy d 2y dy
= 0 ⇒ sin 2x = 0 ⇒ 2x = nπ, n ∈ Z
(iv) y = sin 2x ⇒ = 2 cos 2x and = − 4 sin 2x Now,
dx dx 2 dx
dy nπ
Now, = 0 ⇒ cos 2x = 0 ⇒x = , n ∈ Z ⇒ x = ± π / 2 , ± π , ± 3 π / 2 , ± 2 π, …
dx 2
π 3π d 2y
⇒ 2x = ± , ± ,… Clearly, < 0 at x = ± π / 2 , ± 3π / 2 , ± 5π / 2, …
2 2 dx 2
π 3π
⇒ x =± ,± ,… d 2y
4 4 and > 0 at x = ± π , ± 2π , ± 3π …
dx 2
2
d y π 5π
Clearly, < 0 at x = , , … So, x = ± π / 2 , ± 3π / 2 , ± 5π / 2, … are the points of
dx 2 4 4
local maximum and local maximum value at these
3π 7π points is 1. Points x = ± π , ± 2π , ± 3π, … are points of
and atx = − ,− ,…
4 4 local minimum and the local minimum value at these
d 2y 3π 7 π points is 0.
and > 0 at x = , ,… dy
dx 2 4 4 (iv) y = sin 2 x ⇒ = sin 2x
π 5π dx
and at x =− ,− ,… dy π
4 4 Clearly, > 0 when 0 < x <
π 5π 7 π dx 2
So, the points x = , , ,… dy π
4 4 4 and < 0 when < x < π.
3π 7π dx 2
and x = − ,− , … are points of local maximum So, the given curve is increasing in the interval
4 4
and local maximum values at these points are 1. [0, π / 2] and decreasing in [ π / 2, π ].
π 5π 3π 7 π (v) sin 2 ( π + x ) = sin 2 x for all x. So, the periodicity of the
Similarly, x = − , − , … and x = , , … are
4 4 4 4 function is π. This means that the shape of the curve
points of local minimum and local minimum value at repeats at the interval of length π.
these points is ( − 1).
Y
(v) sin 2 ( x + π ) = sin 2x for all x. So, the periodicity of
the function is π. This means that the pattern of the
curve repeats at intervals of length π. (–5π/2,1) (–3π/2,1) (–π/2, 1) (π/2,1) (3π/2,1) (5π/2,1)
X
Y
(–2π,0) (–π,0) O (π,0) (2π,0) (3π,0)
(–3π/4, 1) (π/4, 1) (5π/4, 1)

X
(0, 0)
Keeping the above facts in mind, we sketch the curve as
(–π/4, –1) (3π/4, –1) (7π/4, –1) shown in figure.

Thus, keeping in mind, we sketch the curve as shown in


figure.
Asymptotes
y Example 7 Sketch the curve y = sin 2 x .
The straight line AB is called the asymptote of curve
Sol. We note the following points about the curve y = f ( x ), if the distance MK from M a point on the curve
(i) The equation of the curve remains same, if x is y = f ( x ) to the straight line AB tends to zero as M recedes
replaced by (− x). So, the curve is symmetric about infinity.
Y -axis, i.e. the curve on the left side of Y -axis is
identical to the curve on its right side. In other words, the straight line AB meets the curve
(ii) The curve meets the coordinate axes at the same points y = f ( x ) at infinity (K is a point on AB). Thus,
where y = sin x meets them. 1. If f ( x ) → ± ∞ for x → a, then the straight line x = a is
dy d 2y the asymptote of the curve y = f ( x ).
(iii) y = sin 2 x ⇒ = sin 2x and = 2 cos 2x
dx dx 2
Chap 03 Area of Bounded Regions 175

2. If in the right hand member of the equation of the Thus, its graph is as shown in figure
curve y = f ( x ) it is possible to single out a linear part Y
so that the remaining part tends to zero as x → ± ∞, y = 1 (asymptote)
i.e. if y = f ( x ) = Kx + b + g( x ) and g( x ) → 0 for
x → ± ∞, then the straight line y = Kx + b is the x2–1
y= 2
asymptote of the curve. x +1
f (x ) X
3. If there exist finite limits lim = K and (–1, 0) (1, 0)
x→ ±∞ x

lim [ f ( x ) − Kx ] = b, then the straight line (0, –1)


x→ ±∞
y = Kx + b is the asymptote of the curve. 1
y Example 9 Construct the graph for f ( x ) = x + ⋅
x
Methods to Sketch Curves
Sol. The function is defined for all x except for x = 0.
While constructing the graphs of functions, it is expedient
to follow the procedure given below It is an odd function for x ≠ 0.
It is not a periodic function.
(1) Find the domain of definition of the function.
For x → 0 + , f ( x ) → + ∞; for x → 0 −, f ( x ) → − ∞
(2) Determine the odd-even nature of the function.
For x → − ∞, f ( x ) → − ∞; for x → ∞, f ( x ) → ∞
(3) Find the period of the function if its periodic.
∴ lim ( f ( x ) − x ) = 0
(4) Find the asymptotes of the function. x→±∞

(5) Check the behaviour of the function for x → 0 ± ∴ The straight lines x = 0 and y = x are the asymptotes of
the graph of the given function.
(6) Find the values of x, if possible for which f ( x ) → 0.
Now, consider f ( x 2 ) − f ( x 1 ) (for x 2 > x 1 )
(7) The interval of increase and decrease of the function 1 1
in its range. Hence, determine the greatest and the = ( x 2 − x1 ) + −
x 2 x1
least values of the function if any.
 1 
= ( x 2 − x1 )  1 − < 0 for x 1 x 2 ∈(0, 1]
Remark
 x 1 x 2 
(5), (6) and (7) gives the points where the function cuts the
coordinate axes. and it is > 0 for x 1 x 2 ∈ [1, ∞ ).
Thus, f ( x ) increases for x ∈ [1, ∞ ) and decreases for
x2 −1 x ∈(0, 1].
y Example 8 Construct the graph for f ( x ) = ⋅
x2 +1 Thus, the least value of the function is at x = 1 which is
x2 − 1 2 f (1) = 2. Thus, its graph can be drawn as
Sol. Here, f ( x ) = =1−
x +1
2
x +1
2
Y y=x
(1) The function f ( x ) is well defined for all real x.
⇒ Domain of f ( x ) ∈ R.
(2) f ( − x ) = f ( x ), so it is an even function. (1, 2)

(3) Since, algebraic → non-periodic function.


X
f (x ) → 1 for x → ± ∞ (0, 0)
and f ( x ) → − 1 for x → 0 ±
It may be observed that f ( x ) < 1 for any x ∈ R and (–1, –2)
consequently its graph lies below the line y = 1 which
is asymptote to the graph of the given function.
2
Again, 2 decreases for (0, ∞ ) and increases for 1
x +1 y Example 10 Construct the graph for f ( x ) = ⋅
( − ∞, 0), thus f ( x ) increases for (0, ∞ ) and decreases 1 + e 1/ x
for ( − ∞, 0) in its range. Sol. The function is defined for all x except for x = 0. It is
(4) The greatest value → 1 for x → ± ∞ and the least neither even nor an odd
value is − 1 for x = 0. function. It is not a periodic function.
For x → 0 + f ( x ) → 0 ; for x → 0 −, f (x ) → 1
176 Textbook of Integral Calculus

1 1 b
For x → ∞ f ( x ) → ; for x → − ∞, f ( x ) → then ∫ f ( x ) dx will equal A1 − A2 + A 3 − A 4 and not
2 2 a
1 A1 + A2 + A 3 + A 4 .
∴ lim f ( x ) =
x→±∞ 2 If we need to evaluate A1 + A2 + A 3 + A 4 (the magnitude
1 of the bounded area), we will have to calculate
∴ The straight line y = is asymptote of the graph of the
2 x y z b
given function. ∫a f ( x ) dx + ∫x f ( x ) dx ∫y f ( x ) dx + ∫z f ( x ) dx
1
As x increases from (0, ∞ ), decreases from (0, ∞ ) and e 1/ x From this, it should also be obvious that
x
b b
decreases from (0, ∞ ). Thus, (1 + e 1/ x ) decreases from (2, ∞ ).
∫a f ( x ) dx ≤ ∫ | f ( x ) | dx
a
 1
∴ f ( x ) increases from 0,  for x ∈ (0, ∞ ). (2) The area under the curve y = f ( x ) from x = a to x = b
 2
is equal in magnitude but opposite in sign to the area
Similarly, f ( x ) increases from (1 / 2, 1) for x → ( − ∞, 0). under the same curve from x = b to x = a, i.e.
i.e. f ( x ) is an increasing function except for x = 0. b a
Thus, its graph can be drawn as shown in figure ∫a f ( x ) dx = − ∫ f ( x ) dx
b

Y This property is obvious if you consider the


Newton-Leibnitz formula. If g( x ) is the anti-derivative of
b b
(0, 1) x ( f ), then ∫ f ( x ) dx is g(b ) − g(a ) while ∫ f ( x ) dx is
Y = 1/2 (Asymptote) a a
g(a ) − g(b )
(3) The area under the curve y = f ( x ) from x = a to x = b
(0, 0)
X can be written as the sum of the area under the curve from
x = a to x = c and from x = c to x = b, that is
b c b
∫a f ( x ) dx = ∫ f ( x ) dx + ∫ f ( x ) dx
a c
Let us consider an example of this. Let c ∈(a, b )
Areas of Curves y
y= f(x)
(1) Suppose that f ( x ) < 0 on some interval [a, b]. Then,
the area under the curve y = f ( x ) from x = a to x = b will
be negative in sign, i.e.
b A2
∫a f ( x ) dx ≤ 0 A1

This is obvious once you consider how the definite x


integral was arrived at in the first place; as a limit of the a c b

sum of the n rectangles (n → ∞ ). Thus, if f ( x ) < 0 in some


interval then the area of the rectangles in that interval will It is clear that the area under the curve from x = a to x = b,
also be negative. A is A1 + A2 .
This property means that for example, if f ( x ) has the Note that c need not lie between a and b for this relation to
following form hold true. Suppose that c > b.
y y
y= f(x)

A2
A1 x y A3 b A1
z
x
a A2 A4
x
a b c
Chap 03 Area of Bounded Regions 177

b b
Observe that A = ∫ f ( x ) dx = ( A + A1 ) − A1 Observe that area (rect AXYB) < ∫ f ( x ) dx < area (rect DXYC)
a a
c
= ∫ f ( x ) dx − ∫ f ( x ) dx
c
(6) Let us consider the integral of f 1 ( x ) + f 2 ( x ) from x = a
a b to x = b. To evaluate the area under f 1 ( x ) + f 2 ( x ), we can
c b
= ∫ f ( x ) dx + ∫ f ( x ) dx separately evaluate the area under f 1 ( x ) and the area under
a a f 2 ( x ) and add the two area (algebraically).
Analytically, this relation can be proved easily using the b b b
Newton Leibnitz’s formula.
Thus, ∫a ( f 1 ( x ) + f 2 ( x )) dx = ∫a f 1 ( x ) dx + ∫ f 2 ( x ) dx
a

(4) Let f ( x ) > g( x ) on the interval [a, b]. Then, Now consider the integral of kf ( x ) from x = a to x = b. To
b b evaluate the area under kf ( x ), we can first evaluate the area
∫a f ( x ) dx > ∫ g( x ) dx
a under f ( x ) and then multiply it by k, that is :
b b
This is because the curve of f ( x ) lies above the curve of
g( x ), or equivalently, the curve of f ( x ) − g( x ) lies ∫a kf ( x ) dx = k ∫a f ( x ) dx

above the x-axis for [a, b ] (7) Consider the odd function f ( x ), i.e. f ( x ) = − f ( − x ). This
measn that the graph of f ( x ) is symmetric about the origin.
y y= f(x)
y

y= g(x) +
A2
–a
a

A1
x
a b

This is an example where f ( x ) > g( x ) > 0. a


From the figure, it should be obvious that ∫ f ( x ) dx = 0,
b −a
∫a f ( x ) dx = A1 + A2 because the area on the left side and that on the right
b algebraically add to 0.
while ∫a g( x ) dx = A1 Similarly, if f ( x ) was even, i.e. f ( x ) = f ( − x )
Similarly, if f ( x ) < g( x ) on the interval [a, b ], then y
b b
∫a f ( x ) dx < ∫ g( x ) dx
a

(5) For the interval [a, b ], suppose m < f ( x ) < M .


That is, m is a lower-bound for f ( x ) while M is an
a +a
upper bound.
b
Then m (b − a ) < ∫ f ( x ) dx < M (b − a )
a a a
This is obvious once we consider the figure below : ∫−a f ( x ) dx = 2∫0 f ( x ) dx because the graph is symmetrical
about the y-axis. If you recall the discussion in the unit on
y
functions, a function can also be even or odd about any
D C
M y = f(x)
arbitrary point x = a. Let us suppose that f ( x ) is odd about
x = a, i.e. f ( x ) = − f (2a − x )
y

m B x=a
A
X Y
x
x=a x=b
178 Textbook of Integral Calculus

The points x and 2a − x lie equidistant from x = a at either b 1 b


Thus f av (b − a ) = ∫ f ( x ) dx ⇒ f av = ∫a f ( x ) dx
sides of it. a b −a
2a
Suppose for example, that we need to calculate ∫ f ( x ) dx . This value is attained for at least one c ∈(a, b ) (under the
a
constraint that f is continuous, of course).
It is obvious that this will be 0, since we are considering
equal variation on either side of x = a, the area from x = 0 y Example 11 Sketch the graph y = | x + 1 |. Evaluate
to x = a and the area from x = a to x = 2a will add 2
algebraically to 0. ∫ −4 | x + 1| dx. What does the value of this integral
Similarly, if f ( x ) is even about x = a, i.e. f ( x ) = f (2a − x ) represents on the graph.
( x + 1), if x ≥ −1
y Sol. Here, y = | x + 1 | = 
 − ( x + 1), if x ≤ −1
which can be shown as
Y
B C

a 3 (0, 1) 3

then we have, for example A


D E X
2a a
∫0 f ( x ) dx = 2 ∫ f ( x ) dx –4 –1 O 2
0 3 3
From the discussion, you will get a general idea as to how 2 −1 2
to approach such issues regarding even/odd functions. ∴∫ | x + 1| dx = ∫ | x + 1| dx + ∫ | x + 1| dx
−4 −4 −1
(8) Let us consider a function f ( x ) on [a, b ] −1 2 2
=∫ − ( x +1)dx + ∫ | x + 1| dx + ∫ ( x + 1)dx
y −4 −1 −1
−1 2
f (b ) x 2  x 2 
= −  + x +  + x = 9
2  −4  2  −1
Representation of the value 9 of integral on graph.
2
f (a ) ∴∫ | x + 1| dx = 9 represents the area bounded by the curve
−4
x
a b y = | x + 1| . X -axis and the lines x = − 4 and x = 2, i.e. if is
equal to the sum of the ares of ∆ABD and ACE,
We want to somehow define the “average” value that f ( x ) 1 1 9 9
takes on the interval [a, b ]. What would be an appropriate i.e. ( 3) ( 3) + ( 3) ( 3) = + = 9
2 3 2 2
way to define such an average? Let f av be the average 1
(Q area of triangle = × base × height)
value that we are seeking. Let it be such that it is obtained 2
at some x = c ∈[a, b ]
y x2 y2
y Example 12 Find the area of the ellipse + = 1.
f(b) a2 b2
Sol. Using the symmetry of the figure; required area is given by
A = 4 ( area OABO )
fav=f(c) fav=f(c) a x2 y2
=4 ∫0 y dx, where
a2
+
b2
=1
f(a)

x B (0,b)
a c b

We can measure f av by saying that the area under f ( x )


O (0,0) A (a,0)
from x = a to x = b should equal the area under the
average value from x = a to x = b. This seems to be the
only logical way to define the average (and this is how it
is actually defined!).
Chap 03 Area of Bounded Regions 179

y2 x2  a2 a2 
∴ =1− = 2 a 3a 2 − log (2a + a 3 ) + log a
b2 a2  2 2 
b2  2 2  2a + a 3  
⇒ y2 = (a 2 − x 2 ) = 2 a 3 −
a
log  
a2 2  a 

b
⇒ y=± a2 − x 2 = 2 a 2 3 − a 2 log (2 + 3 ) sq units
a
In the first quadrant, y Example 14 Find the area common to the parabola
b
y= a2 − x 2 5x 2 − y = 0 and 2x 2 − y + 9 = 0.
a
a b b a Sol. Given curves are y = 5x 2 …(i)
A=4∫ a 2 − x 2 dx = 4 ∫0 a − x dx
2 2
0 a a and y = 2x 2 + 9 …(ii)
a
b x a2
x 
=4  a2 − x 2 + sin − 1  x 2=
y–9
a 2 2 a 0 Y x 2 = y /5 2

b  a 2 −1 a 
 4b a 2
=4 0 + sin  − {0 + 0} = ⋅ sin −1(1) (0, 9)
a  2 a   a 2
 π
= 2ab   X
 2  (0, 0)

A = πab sq units

y Example 13 Find the area bounded by the hyperbola


x 2 − y 2 = a 2 between the straight lines x = a and Remark
x = 2a. In such examples, figure is the most essential thing. Without
Sol. We use the symmetry of figure. figure it just becomes difficult to judge whether y1 to be
2a subtracted from y2 or otherwise.
Required area, A = 2 ∫ y dx , where x 2 − y 2 = a 2
a
Let us solve Eqs. (i) and (ii) simultaneously,
i.e. x 2 − a2 = y 2 ∴ 5x 2 = 2x 2 + 9
∴ y = ± x 2 − a2 ⇒ 3x 2 = 9 ⇒ x 2 = 3
∴ x=− 3
or x= 3
In the usual notations, the required area is given by
3

(a, 0) (2a,0)
A= ∫− 3
(y1 − y 2 ) dx

x = 2a We have to find which curve is above and which is below


w.r.t. X -axis in order to decide y1 and y 2 .
Take any point between x = − 3 and x = 3
Let us take x = 0, which lies between
In the first quadrant; y = + x 2 − a 2 x = − 3 and x = 3
2a
∴ A =2∫ x − a dx
2 2
When x = 0 from Eq. (i) y = 0
a
2a When x = 0 from Eq. (ii) y = 9
x a2 
=2 x 2 − a2 − log ( x + x 2 − a 2 ) Now, 9>0
2 2 a ∴ Parabola Eq. (ii) is above parabola Eq. (i) between
 a 2   a2  x=− 3
= 2 a 4a 2 − a 2 − log (2a + 4a 2 − a 2 ) − 0 − log a 
 2   2   and x = 3.
180 Textbook of Integral Calculus

∴y of the curve (ii) is to be taken as y1 and y of the curve (i) 1


( x 3 − 3x 2 + 2x ) dx + ∫ ( x 3 − 3x 2 + 2x ) dx 
2

is to be taken as y 2 .
= ∫0  1 
3
∴ Area ( A ) = ∫− 3
{(2x 2 + 9 ) − 5x 2 } dx  x4
= 

− x 3 + x 2 
1  x4
+  − x 3
+ x 2
 
2
 
 4 
3  4   1
= ∫− 3
(9 − 3x 2 ) dx 0  
 1    1 
3 =  − 1 + 1  + ( 4 − 8 + 4 ) −  − 1 + 1  
=2∫ (9 − 3x 2 ) dx  4    4 
0
1 1 1
= 2 [9 x − x 3 ] 0 3 = + = sq unit
4 4 2
= 2 [9 3 − 3 3 ]
Area = 12 3 sq units
y Example 16 Find the area between the curves
y = 2x 4 − x 2 , the X-axis and the ordinates of two
y Example 15 Find the area enclosed by minima of the curve.
y = x ( x − 1) ( x − 2) and X-axis.
Sol. The given curve is y = 2x 4 − x 2 .
Sol. The given curve is y = x ( x − 1) ( x − 2). It passes through 1
When y = 0, then x = 0, 0, ±
(0, 0), (1, 0) and (2, 0). 2
The sign scheme for y = x ( x − 1) ( x − 2) is as shown in The sign scheme is as shown below
figure. + +
–∞ – ∞
+ + –1 1
– 0 1 – 2 2 2

From the sign scheme it is clear that the curve is + ve when Therefore, it is clear that the curve cuts the X -axis at
0 < x < 1 or x > 2, hence in these regions the curve lies x=−
1
, 0 and
1

above X -axis while in the rest regions the curve lies below 2 2
X -axis.  1   1
The curve is –ve in  − ,0 and 0,  while positive in
 2   2
Remark dy dy
Sometimes the discussion of monotonicity of function helps the rest. Now, = 8x 3 − 2x . The sign scheme for is as
dy dx dx
us in sketching polynomials. In the present case, = 3x 2 − 6 x + 2 below
dx
dy 1 dy
When = 0 , then ; x = 1 ± . Sign scheme for is + +
dx 3 dx –∞ – 0 – ∞
–1 1
+ + 2 2

1– 1 1+ 1
3 3 i.e. The curve decreases in ( − ∞, − 1 / 2) and (0, 1 / 2) and
increases in the rest of portions. Also, the function possess
Thus, it is clear that the curve increases in  − ∞, 1 −
1 

, minimum at x = − 1 / 2 and 1/2 while maximum at x = 0.
3
Therefore, the graph of the curve is as shown below
decreases in  1 −  and again increases in
1 1
,1+ 
 3 3
 1 + 1 , ∞  . Therefore, the graph of the curve is as below Y
 
 3 

Y –1/2 1/2
X
1 O 1
1 –
1+ 2 2
3
X
O 1 1
1–
1/ 2
3
∴ Required area = 2 ∫ | 2x 4 − x 2 | dx
0 1/ 2
1/ 2  x5 x3
Hence, required area = 2∫ − (2x 4 − x 2 )dx = −2 2 − 
0
 5 3 0
7
∫ 0 x ( x − 1) ( x − 2) dx + ∫ 1 x ( x − 1) ( x − 2) dx 
1 2
= = sq unit
 120
Chap 03 Area of Bounded Regions 181

Exercise for Session 1


1. Draw a rough sketch of y = sin 2x and determine the area enclosed by the curve, X-axis and the lines x = π / 4
and x = 3π / 4.

2. Find the area under the curve y = ( x 2 + 2)2 + 2x between the ordinates x = 0 and x = 2.

3. Find the area of the region bounded by the curve y = 2x − x 2 and the X-axis.

4. Find the area bounded by the curve y 2 = 2y − x and the Y-axis.

5. Find the area bounded by the curve y = 4 − x 2 and the line y = 0 and y = 3.

6. Find the area bounded by x = at 2 and y = 2at between the ordinates corresponding to t = 1and t = 2.

7. Find the area of the parabola y 2 = 4ax and the latusrectum.

8. Find the area bounded by y = 1 + 2 sin2 x , X-axis, x = 0 and x = π.

9. Sketch the graph of y = x + 1in [0, 4] and determine the area of the region enclosed by the curve, the axis of
X and the lines x = 0, x = 4.

10. Find the area of the region bounded by the curve xy − 3x − 2y − 10 = 0 , X-axis and the lines x = 3, x = 4.
Session 2
Area Bounded by Two or More Curves
Area Bounded by Two If confusion arises in such case evaluate
or More Curves b
∫a | f ( x ) − g( x ) | dx which gives the required area.
Area bounded by the curves y = f ( x ), y = g( x ) and the
Y
lines x = a and x = b. y = f(x)
Let the curves y = f ( x ) and y = g( x ) be represented by AB
and CD, respectively. We assume that the two curves do
not intersect each other in the interval [a, b ].
Thus, shaded area = Area of curvilinear O
X
trapezoid APQB − Area of curvilinear trapezoid CPQD
b b b y = g(x)
= ∫ f ( x ) dx − ∫ g( x ) dx = ∫ { f ( x ) − g( x )} dx x=a x=b
a a a
Figure 3.34
Y
Area between two curves y = f ( x ), y = g( x ) and the lines
b
A B x = a and x = b is always given by ∫ { f ( x ) − g( x )} dx
y = f(x) a
provided f ( x ) > g( x ) in [a, b ]; the position of the graph is
immaterial. As shown in Fig. 3.34, Fig. 3.35, Fig. 3.36.
C y = g(x)
D Y
X
O P Q x=a x=b

x=a x=b
Figure 3.32
X
O y = f(x)
Now, consider the case when f ( x ) and g( x ) intersect each
other in the interval[a, b ].
First of all we should find the intersection point of
y = f ( x ) and y = g( x ). For that we solve f ( x ) = g( x ). Let
y = g(x)
the root is x = c . (We consider only one intersection point
Figure 3.35
to illustrate the phenomenon).
Thus, required (shaded) area Y y = f(x)
c b
= ∫ { f ( x ) − g( x )}dx + ∫ { g( x ) − f ( x )} dx
a c
Y
y = g(x)
X
y = g(x) O

x=b
x=a
y = f(x) Figure 3.36
X
O a c b y Example 17 Sketch the curves and identify the
region bounded by x = 1 / 2 , x = 2, y = log e x and
Figure 3.33
y = 2 x . Find the area of this region. [IIT JEE 1991]
Chap 03 Area of Bounded Regions 183

Sol. The required area is the shaded portion in the following Now, we know that
figure. C ≤ 0 denotes the region, inside the circle C = 0.
y = 2x
Y P ≤ 0 denotes the region, inside the parabola P = 0.
S ≤ 0 denotes the region, which is negative side of the line
S = 0.
∴ Required area = Area of curvilinear ∆OMRO
y = loge x + Area of trapezium MNSR – Area of curvilinear ∆ONSO
2 1
∫0
X
= 8x dx +
( MR + NS ) ⋅ MN
2
x=2 – (Area of square ONSG − Area of sector OSGO)
x = 1/2 2 1  π ⋅ 32 
=∫ 8x dx + ( 4 + 3) ⋅ 1 − 32 − 
1 0 2  4 
In the region ≤ x ≤ 2; the curve y = 2x lies above as
2  9 π 1
compared to y = loge x . = −  sq units
2
 4 6
Hence, required area = ∫1/ 2 (2x − log x ) dx

 2x 
2 y Example 19 Find the area of the region
= − ( x log x − x )  {( x , y ) : 0 ≤ y ≤ x 2 + 1, 0 ≤ y ≤ x + 1, 0 ≤ x ≤ 2}.
 log 2 1/ 2
Sol. Let R = {( x , y ) : 0 ≤ y ≤ x 2 + 1, 0 ≤ y ≤ x + 1, 0 ≤ x ≤ 2}
4 − 2 5 3
= − log 2 +  sq units = {( x , y ) : 0 ≤ y ≤ x 2 + 1} ∩ {( x , y ) : 0 ≤ y ≤ x + 1}
 log 2 2 2
∩ {( x , y ) : 0 ≤ x ≤ 2}
= R1 ∩ R 2 ∩ R 3
y Example 18 Find the area given by
where, R1 = {( x , y ) : 0 ≤ y ≤ x 2 + 1}
x + y ≤ 6, x 2 + y 2 ≤ 6 y and y 2 ≤ 8 x .
R 2 = {( x , y ) : 0 ≤ y ≤ x + 1}
Sol. Let us consider the curves
and R 3 = {( x , y ) : 0 ≤ x ≤ 2}
P ≡ y 2 − 8x = 0 …(i)
Thus, the sketch of R1, R 2 and R 3 are
C ≡ x 2 + y 2 = 6y
Y
i.e. x 2 + ( y − 3) 2 − 9 = 0 …(ii) 5
1

S ≡ x +y −6=0
+

and …(iii)
x2

4
y=

The intersection points of the curves (ii) and (iii) are given y=x+1
by 3 (2, 3)
y = x2 + 1 (1, 2)
(6 − y )2 + y 2 − 6y = 0 2

i.e. y = 3, 6 (0, 1)
Y R1∩R2∩R3

P (0, 6) R (2, 4) X
O 1 2

From the above figure,


G S (3, 3)
1 2
Required area = ∫0 ( x 2 + 1) dx + ∫1 ( x + 1) dx
1 2
X  x3   x2  23
O MN Q
=  + x  +  + x  = sq units
 3  0  2  1
6

y Example 20 The area common to the region


determined by y ≥ x and x 2 + y 2 < 2 has the value
Therefore, the points are (0, 6) and (3, 3). The intersection
points of the curves (i) and (iii) are given by (a) π sq units (b) (2π − 1) sq units
y = 8 (6 − y ), i.e. y = 4, − 12  π 1
2
(c)  −  sq units (d) None of these
 4 6
Therefore, the point of intersection in 1st quadrant is (2, 4).
184 Textbook of Integral Calculus

Sol. The region formed by y ≥ x is the outer region of the Thus, required area
parabola y 2 = x , when y ≥ 0 and x ≥ 0 and x 2 + y 2 < 2 is 3  − 3 ( 1 + x ) − ( 3 − x ) ( x − 1) 
the region inner to circle x 2 + y 2 = 2 shown as in figure.
= ∫1 
 2


 − 3 ( 1 + x ) + ( 3 − x ) ( x − 1) 
−   dx
y=x  2 
x, y > 0
=− (3 − x ) ( x − 1) dx = ∫ 12 − ( x − 2)2 dx 
3 3

 ∫1   1 
O 1
 1 3
1 −1  x − 2 
= −
 2 ( x − 2 ) − x 2
+ 4 x − 3 + sin   
 2  1  1
 
π
= sq units
Now, to find the point of intersection put y 2 = x in 2
x 2 + y 2 = 2.  {x } , x ∉ Z
y Example 22 If f ( x ) =  and g ( x ) = { x } 2
⇒ x2 + x − 2 = 0  1, x ∈ Z
⇒ ( x + 2) ( x − 1) = 0 (where, {.} denotes fractional part of x), then the
⇒ x = 1, as x ≥ 0 area bounded by f ( x ) and g ( x ) for x ∈[0, 10] is
1
∴ Required area = ∫0 ( 2 − x 2 − x ) dx 5
(a) sq units (b) 5 sq units
1
3
x 2 − x2 x 2 3/2 10
= + sin − 1 − x  (c) sq units (d) None of these
 2 2 3  3
0
1 π 2  π 1  { x } , x ∉z
= + − =  −  sq units Sol. As, f ( x ) =  and g ( x ) = { x } 2 , where both f ( x )
2 4 3  4 6  1 , x ∈z
Hence, (c) is the correct answer. and g ( x ) are periodic with period ‘1’ shown as

y Example 21 Find the area of the region enclosed by {x}

the curve 5x 2 + 6 xy + 2y 2 + 7 x + 6 y + 6 = 0. 1

Sol. Comparing ax 2 + 2hxy + by 2 + 2gx + 2 fy + c = 0, we get


a = 5, b = 2, h = 3, g = 7 / 2, f = 3 and c = 6 O 1 2 3 4 5 6 7 8 9 10
⇒ h − ab = − 1 < 0
2
1
Thus, required area = 10 ∫ [ { x } − { x } 2 ] dx
So, the above equation represents an ellipse. 0

∴ 2y 2 + 6 (1 + x ) y + (5x 2 + 7 x + 6) = 0 1
= 10 ∫ [( x )1/ 2 − x 2 ] dx
0
− 3 ( 1 + x ) ± ( 3 − x ) ( x − 1)
⇒ y= x 3/2 x 3 
1
2 = 10  − 
Clearly, the values of y are real for all x ∈[1, 3]. Thus, the 3/2 3 0
graph is as shown below
 2 1 10
= 10  −  = sq units
Y  3 3 3
O 1 2 3 Hence, (c) is the correct answer.
X

y Example 23 Find the area of the region bounded by


(1, –3) the curves y = x 2 , y = | 2 − x 2 | and y = 2, which lies to
–3
the right of the line x = 1. [IIT JEE 2002]
Sol. The region bounded by given curves on the right side of
x = 1 is shown as
–6 (3, –6) 2 2
Required area = ∫1 { x 2 − (2 − x 2 )} dx + ∫ 2
{ 4 − x 2 } dx
Chap 03 Area of Bounded Regions 185

= ∫1
2
(2x 2 − 2) dx + ∫
2
( 4 − x 2 ) dx y Example 25 Let f ( x ) = x 2 , g ( x ) = cos x and
2
2 2 α , β (α < β) be the roots of the equation
 x3   x3
= 2 ⋅ − 2x  +  4 x −  18 x 2 − 9 πx + π 2 = 0. Then, the area bounded by the
 3 1  3  2 curves y = fog ( x ), the ordinates x = α , x = β and the
Y X-axis is
1 π
2 (a) (π − 3) sq units (b) sq units
2 3
π π
(c) sq units (d) sq units
4 12
Sol. Here, y = fog ( x ) = f { g ( x )} = (cos x )2 = cos 2 x

y X Also, 18x 2 − 9 πx + π 2 = 0
–2 –1 O 1 2
⇒ ( 3x − π ) ( 6x − π ) = 0
4 2  2   8  2 2 π π
= − 2 2 −  − 2 + 8 −  − 4 2 −  ⇒ x = , (as α, β)
 3   3   3   3  6 3
 20  20 − 12 2 
= − 4 2 +  =   sq units
 3  3 

y Example 24 The area enclosed by the curve


π π
| y | = sin 2x , when x ∈[0, 2π ] is 6 3
(a) 1 sq unit (b) 2 sq units ∴ Required area of curve
(c) 3 sq units (d) 4 sq units π /3 1 π /3
Sol. As, we know y = sin 2x could be plotted as =∫ cos 2 x dx = ∫ (1 + cos 2x ) dx
π /6 2 π /6
π /3
Y 1 sin 2x  1  π π  1  2π 2π  
= x +  =   −  + sin − sin  
1 2 2  π / 6 2  3 6  2  3 6 
y=sin 2x

O π/2 π 3π X 1 π 1 3 3   π
2π =  +  −  =
–1 2 2 6 2 2 2   12
Hence, (d) is the correct answer.
Thus, | y | = sin 2x is whenever positive, y can have both
positive and negative values, i.e. the curve is symmetric y Example 26 Find the area bounded by the curves
about the axes. x 2 + y 2 = 25, 4 y = | 4 − x 2 | and x = 0 above the X-axis.
π 3π
sin 2x is positive only in 0 ≤ x ≤ and π ≤ x ≤ ⋅ Thus, Sol. The 1st curve is a circle of radius 5 with centre at (0, 0).
2 2
 π
the curve consists of two loops one in  0,  and another  4 − x2   x2 
 2  The 2nd curve is y =  = 1 − 
 3π   4   4 
in  π, .
 2  which can be traced easily by graph transformation.
y

O π/2 π 3π x
2
π /2
Thus, required area = 4 ∫0 (sin 2x ) dx (–5, 0) (–4, 0) (–2, 0) (2, 0) (4, 0) (5, 0)

π /2
 cos 2x 
= 4 −  = − 2 (cos π − cos 0)
 2 0
= − 2 ( − 1 − 1) = 4 sq units
Hence, (d) is the correct answer. When the two curves intersect each other, then
186 Textbook of Integral Calculus

 x2
2
 1
x +  1 −
2
 = 25 ⇒ x = ± 4 y Example 28 Let f ( x ) = max sin x , cos x , , then
 4   2
4   x 2  determine the area of region bounded by the curves
Hence, required area = 2 ∫  25 − x 2 − 1 −  dx y = f ( x ), X-axis, Y -axis and x = 2π .
0
  4 
 1
 4 2  x2 4 x2  Sol. We have, f ( x ) = max sin x , cos x ,  . Graphically, f ( x )
= 2 ∫ 25 − x 2 dx − ∫ 1 −  dx + ∫ 1 −  dx   2
 0 0
 4 2
 4   could be drawn as
Y
 25  4  4 8   4 
= 2 6 + sin −1   − −  = 25 sin −1   + 4 cos x sin x
 2  5  3 3    5   1
cos x
1/2
1/2
y Example 27 Find the area enclosed by | x | + | y | = 1. p X
O p/4 5p/6 5p/3 2p
Sol. From the given equation, we have
| y | = 1 − | x |0 [Q| y |0]
⇒ − 1≤ x ≤ 1 Here, the graph is plotted between 0 to 2π and between the
points of intersection the maximum portion is included,
Y thus the shaded part is required area
1 Interval Value of f ( x )
i.e. for 0≤ x < π / 4 cos x
for π / 4 ≤ x < 5π / 6 sin x
for 5π / 6 ≤ x < 5π / 3 1/2
X
–1 O 1 for 5π / 3 ≤ x < 2π cos x
Hence, required area
π /4 5π /6 5π /3 1 2π
I =∫ cos x dx + ∫ sin x dx + ∫ dx +∫ cos x dx
0 π /4 5π /6 2 5π /3
–1 π /4 5π /6 1 5π /3 2π
= (sin x )0 − (cos x ) π / 4 + ( x )5 π / 6 + (sin x )5 π / 3
Therefore, the curve exists for x ∈ [ − 1, 1] only 2
 |x |−1  1   3 1  1  5π 5π   3
and for − 1 ≤ x ≤ 1; y = ± (1 − | x | ) i.e. y =  = − 0 −  − −  +  −  + 0 + 
 2   2 2 2  3 6  2
 − (| x | − 1)
Thus, the required graph is as given in figure.  5π 
= + 2 + 3 sq units
∴ Required area = ( 2 )2 = 2 sq units  12 

Exercise for Session 2


1. The area of the region bounded by y 2 = 2x + 1 and x − y − 1 = 0 is
(a) 2/3 (b) 4/3 (c) 8/3 (d) 16/3

2. The area bounded by the curve y = 2x − x and the straight line y = x is given by
2

(a) 9/2 (b) 43/6 (c) 35/6 (d) None of these

3. The area bounded by the curve y = x x , X-axis and the ordinates x = − 1, x = 1is given by
(a) 0 (b) 1/3 (c) 2/3 (d) None of these

4. Area of the region bounded by the curves y = 2x , y = 2x − x 2, x = 0 and x = 2 is given by


3 4 3 4 4
(a) − (b) + (c) 3 log 2 − (d) None of these
log 2 3 log 2 3 3

5. The area of the figure bounded by the cuves y = e x , y = e − x and the straight line x = 1is
1 1 1
(a) e + (b) e − (c) e + −2 (d) None of these
e e e
Chap 03 Area of Bounded Regions 187

6. Area of the region bounded by the curve y 2 = 4x ,Y-axis and the line y = 3 is
(a) 2 (b) 9/4 (c) 6 3 (d) None of these

7. The area of the figure bounded by y = sin x , y = cos x is the first quadrant is
(a) 2 ( 2 − 1) (b) 3 + 1 (c) 2 ( 3 − 1) (d) None of these
−x
8. The area bounded by the curves y = xe , y = xe x
and the line x = 1is
2 2 1 1
(a) (b) 1 − (c) (d) 1 −
e e e e
9. The areas of the figure into which the curve y 2 = 6x divides the circle x 2 + y 2 = 16 are in the ratio
2 4π − 3 4π + 3
(a) (b) (c) (d) None of these
3 8π + 3 8π − 3

10. The area bounded by theY-axis, y = cos x and y = sin x , 0 ≥ x ≤ π /2 is


(a) 2( 2 − 1) (b) 2 − 1 (c) ( 2 + 1) (d) 2
3
11. The area bounded by the curve y = and y + 2 − x = 2 is
x
4 − log 27
(a) (b) 2 − log 3 (c) 2 + log 3 (d) None of these
3
12. The area bounded by the curves y = x 2 + 2 and y = 2 x − cos + x is
(a) 2/3 (b) 8/3 (c) 4/3 (d) 1/3

13. The are bounded by the curve y = 4x and the circle x + y − 2x − 3 = 0 is


2 2 2

8 8 8 8
(a) 2 π + (b) 4 π + (c) π + (d) π −
3 3 3 3
 1
14. A point P moves inside a triangle formed by A (0, 0), B 1,  , C (2, 0 ) such that min {PA, PB, PC} = 1, then the
 3
area bounded by the curve traced by P, is
3π π π 3π
(a) 3 3 − (b) 3 + (c) 3 − (d) 3 3 +
2 2 2 2
15. The graph of y 2 + 2xy + 40 | x | = 400 divides the plane into regions. The area of the bounded region is
(a) 400 (b) 800 (c) 600 (c) None of these
16. The area of the region defined by | x | − | y | ≤ 1and x + y ≤ 1 in the xy - plane is
2 2

(a) π (b) 2π (c) 3π (d) 1


17. The area of the region defined by 1 ≤ | x − 2 | + | y + 1| ≤ 2 is
(a) 2 (b) 4 (c) 6 (d) None of these
18. The area of the region enclosed by the curve | y | = − ( 1 − | x | ) + 5, is 2

8 2 2
(a) (7 + 5 5 ) sq units (b) (7 + 5 5 ) sq units (c) (5 5 − 7) sq units (d) None of these
3 3 3
π
19. The area bounded by the curve f ( x ) = | tan x + cot x | − | tan x − cot x | between the lines x = 0, x = and the
2
X-axis is
(a) log 4 (b) log 2 (c) 2 log 2 (d) 2 log 2

If f ( x ) = max sin x , cos x , , then the area of the region bounded by the curves y = f ( x ), X -axis, Y-axis and
1
20.
 2

x = is
3
5π  5π 
(a)  2 − 3+  sq units (b)  2 + 3+  sq units
 12   2
5π 
(c)  2 + 3+  sq units (d) None of these
 2
JEE Type Solved Examples :
Single Option Correct Type Questions
2
l Ex. 1 If A denotes the area bounded by Required area = 4 ∫ ( 5 − x 2 − 1 ) dx
sin x + cos x 1
f (x ) = , X-axis, x = π and x = 3π, then  2 1 
x = 10 sin −1 − sin −1  − 4
 5 5
(a) 1 < A < 2 (b) 0 < A < 2
Hence, (b) is the correct answer.
(c) 2 < A < 3 (d) None of these
2 2 2π | sin x + cos x| 2 2 l Ex. 4 The area of the region between the curves
Sol. <∫ dx <
2π π x 2π 1 + sin x 1 − sin x
 1 1 1 y= and y = and bounded by the lines
Q π < x < 2π ⇒ < < ...(i) cos x cos x
 2 π x π 
π
2 2 3π | sin x + cos x | 2 2 x = 0 and x = is
<∫ dx < ...(ii) 4 [IIT JEE 2008]
3π 2π x 2π
2 −1 t
On adding Eqs. (i) and (ii), we get (a) ∫ dt
5 2 3 2
0
(1 + t ) 1 − t 2
2
<A<
3π π 2 −1 4t
(b) ∫ dt
⇒ 0.75 < A < 1.3 0
(1 + t ) 1 − t 2
2

Hence, (b) is the correct answer. 2 +1 4t


(c) ∫ dt
0
(1 + t ) 1 − t 2
2
l Ex. 2 If f ( x ) ≥ 0, ∀x ∈(0, 2 ) and y = f ( x ) makes posi-
2 +1 t
tive intercepts of 2 and 1 unit on X and Y-axes respectively (d) ∫ dt
and encloses an area of 3/4 unit with axes, then
0
(1 + t 2 ) 1 − t 2
2
∫0 xf ′ ( x ) dx is Sol. Required area = ∫
0
π /4 


1 + sin x
cos x

1 − sin x 
cos x 
 dx
3 5 3
(a) (b) 1 (c) (d) −  1 + sin x 1 − sin x 
4 4 4 Q > > 0
2 3 3  cos x cos x 
Sol. I = xf ( x )| ∫ f ( x ) dx = 0 − = −
0−
2
0 4 4  x x 
Hence, (d) is the correct answer.  2 tan 2 tan 
 1+ 2 1− 2 
 1 + tan 2 x
1 + tan 2 x 
l Ex. 3 The area of the region included between the π /4  
=∫ 
2 − 2
 dx
regions satisfying min (| x |, | y |) 1 and x 2 + y 2 ≤ 5 is 0
 1 − tan 2 x
1 − tan 2 x

5 2 1  2 1  2 2 
(a)  sin−1 − sin−1  − 4 (b) 10  sin−1 − sin−1  − 4  1 + tan 2 x
1 + tan 2x 
2 5 5  5 5  
 2 2 
2 2 1  2 1
(c)  sin−1 − sin−1  − 4 (d) 15  sin−1 − sin−1  − 4  x x
5 5 5  5 5 π /4  1 + tan 1 − tan 
=∫  2 − 2  dx
Sol. Shaded region depicts min (| x |, |y |) ≥ 1 0  1 − tan x x 
1 + tan 
 2 2 
Y
x x x
(1,2)
π /4
1 + tan − 1 + tan π /4
2 tan
1 =∫ 2 2 dx =
∫0
2 dx
0
2x x
(2,1) 1 − tan 1 − tan 2
2 2
x
X Put tan =t
–1 1 2
2 −1 4t
∴ Area = ∫ dt
(1 + t ) 1 − t 2
0 2
–1
Hence, (b) is the correct answer.
Chap 03 Area of Bounded Regions 189

JEE Type Solved Examples :


More than One Correct Option Type Questions
l Ex. 5 Let T be the triangle with vertices (0, 0 ), (0, c 2 ) and Þ (b) is true; range is [−1,1) ⇒ into Þ (a) is false; minimum
(c , c 2 ) and let R be the region between y = cx and y = x 2 , value occurs at x = 0 and f ( 0 ) = −1 ⇒ (c) is false.
∞ x 2 − 1 ∞ dx
where c > 0, then A = 2∫ 1 − 2  dx = 4 ∫
0  x + 1 0 x2 + 1
c3 c3
(a) Area (R) = (b) Area of R = π
6 3 = [ 4 ⋅ tan −1 x ]0∞ = 4 ⋅ = 2 π ⇒ (d) is false.
2
Area (T ) Area (T ) 3
(c) lim =3 (d) lim = Hence, (a), (c) and (d) are the correct answers.
c → 0 + Area (R ) c → 0 + Area (R ) 2
 π
Sol. Area (T ) =
c ⋅c 2 c 3
=  cos x, 0 ≤ x < 2
l Ex. 7 Consider f ( x ) =
2 2  π 2
π
such that f
 − x  , ≤ x < π
y=x2 y=cx  2  2
is periodic with period p, then
(0,c2) (c,c2)
 π2 
(a) the range of f is 0, 
 4
(b) f is continuous for all real x, but not differentiable for
(0,–0) some real x
c3 c c3 c3 c3 (c) f is continuous for all real x
Area ( R ) = − ∫ x 2 dx = − = (d) the area bounded by y = f ( x ) and the X -axis from
2 0 2 3 6
 π2 
Area (T ) c3 6 x = −nπ to x = nπ is 2n 1 +  for a given n ∈ N
∴ lim = lim ⋅ 3 = 3  24 
c→ 0 + Area ( R ) c → 0 2 c
+

Hence, (a) and (c) are the correct answers.  π


cos x, 0≤x<
 2
Sol. Given, f ( x ) =  2 and f is periodic with
π π
l Ex. 6 Suppose f is defined from R → [−1, 1] as  − x , ≤x<π
 2 
2
x 2 −1
f (x ) = 2 where R is the set of real number. Then, the period π. Let us draw the graph of y = f ( x )
x +1  π 2
statement which does not hold is From the graph, the range of the function is 0,  .
(a) f is many-one onto  4
It is discontinuous at x = nπ, n ∈ I. It is not differentiable
(b) f increases for x > 0 and decreases for x < 0 nπ
(c) minimum value is not attained even though f is at x = , n ∈ I.
2
bounded Y
(d) the area included by the curve y = f ( x ) and the line 3
π2/4
y = 1 is p sq units 2
x2 −1 2
Sol. y = f ( x ) = =1− 2 1
x +1
2
x +1
X ′ –π –π/2 O π/2 π 3π/2 2π 5π/2 3π 7π/2 4π X
4x
f ′( x ) = 2 x > 0, f is increasing and x < 0 f is decreasing. –1
(x + 1)2 Y′
Y
Area bounded by y = f ( x ) and the X -axis from −nπ to nπ for
y=1 n ∈N
π  π /2 π π 
2 
= 2n ∫ f ( x ) dx = 2n ∫ cos x dx + ∫  − x dx 
X 0 0 π / 2 2 
O  
 π 3
= 2n 1 + 
 24 
(0,–1)
Hence, (a) and (d) are the correct answers.
190 Textbook of Integral Calculus

l Ex. 8 Consider the functions f ( x ) and g ( x ), both l Ex. 9 Area of the region bounded by the curve y = e x
defined from R → R and are defined as f ( x ) = 2 x − x 2 and and lines x = 0 and y = e is [IIT JEE 2009]
e
g ( x ) = x n where n ∈N . If the area between f ( x ) and g ( x ) is (a) e − 1 (b) ∫ ln (e + 1 − y ) dy
1
1/2, then n is a divisor of 1 e
(a) 12 (b) 15 (c) 20 (d) 30 (c) e − ∫ e x dx (d) ∫ lny dy
0 1
Sol. Solving, f ( x ) = 2 x − x 2 and g( x ) = xn we have
Sol. Shaded area = e −  ∫ e x dx = 1
1

2 x − x 2 = xn ⇒ x = 0 and x = 1  0 

g(x) = xn Y

f(x) = 2x–x2 e y=e

O
X
O 1
 n + 1 1
1 x3 x
A = ∫ (2 x − x 2 − xn ) dx = x 2 − −
0
 3 n + 1  0
e
=1− −
1 1 2
= −
1 Also, ∫ 1 ln(e + 1 − y ) dy
3 n+1 3 n+1
2 1 1 2 1 1 Put e + 1 −y = t
Since, − = ⇒ − =
3 n+1 2 3 2 n+1 ⇒ −dy = dt
4 −3 1 1
= ∫ ln t( −dt ) =
e

6
=
n+1
⇒ n+1 =6 0 ∫ 0 lnt dt
⇒ n =5 e
= ∫ ln y dy = 1
1
Thus, n is a divisor of 15, 20, 30.
Hence, (b), (c) and (d) are the correct answers. Hence, (b), (c) and (d) are the correct answers.

JEE Type Solved Examples :


Passage Based Questions
Passage I l Ex. 11 The function f ( x ) defined for R → R
(Q. Nos. 10 to 12) (a) is one-one onto
Consider the function f ( x ) = x 3 − 8 x 2 + 20 x − 13. (b) is many-one onto
(c) has 3 real roots
l Ex. 10 Number of positive integers x for which f ( x ) is a (d) is such that f ( x 1 ) ⋅ f ( x 2 ) < 0 where x 1 and x 2 are the
roots of f ′ ( x ) = 0
prime number, is
Sol. f ( x ) is many-one as it increases and decreases, also range of
(a) 1 (b) 2
f ( x ) ∈ R Þ many-one onto.
(c) 3 (d) 4
Hence, (b) is the correct answer.
Sol. f ( x ) = ( x − 1 )( x 2 − 7 x + 13 ) for f ( x ) to be prime atleast one of
the factors must be prime. l Ex. 12 Area enclosed by y = f ( x ) and the coordinate
Therefore, x −1 = 1
⇒ x =2
axes is
(a) 65/12 (b) 13/12
or x 2 − 7 x + 13 = 1
(c) 71/12 (d) None of these
⇒ x 2 − 7 x + 12 = 0
1 1 65
⇒ x = 3 or 4 Sol. A = ∫ 0 f (x ) dx = − ∫ ( x 3 − 8 x 2 + 20 x − 13 ) dx =
0 12
⇒ x = 2, 3, 4
Hence, (a) is the correct answer.
Hence, (c) is the correct answer.
Chap 03 Area of Bounded Regions 191

Passage II On differentiating, we get 3y 2y ′ − 3y ′ − 1 = 0


1
(Q. Nos. 13 to 15) ⇒ y′ =
3(1 − y 2 )
Let h( x ) = f ( x ) − g ( x ), where f ( x ) = sin 4 πx and
1
g ( x ) = ln x . Let x 0 , x 1 , x 2 ,..., x n +1 be the roots of f ( x ) = g ( x ) ⇒ y′ ( −10 2 ) = ...(i)
3 {1 − (2 2 ) 2 }
in increasing order. 1 1 1
⇒ y′ ( −10 2 ) = = =−
3 {1 − (2 2 ) } 3 (1 − 8 )
2
21
l Ex. 13 The absolute area enclosed by y = f ( x ) and
Again differentiating Eq. (i), we get
y = g ( x ) is given by
6yy ′ 2
n n
y ′′ =
∑ ∫x ∑ ∫x
xr +1 xr +1 r +1
(a) ( −1) h ( x ) dx
r
(b) ( −1) h ( x ) dx 3(1 − y 2 )
r r
r =0 r =0 2
n n 1
1 6 ⋅2 2 ⋅  
∑ ∫x ( −1) h ( x ) dx (d) ∑
xr + 1 xr +1
∫x
r +1
(c) 2 r
( −1) h ( x ) dx  21 4 2
r =0
r 2r =0 r y ′′( −10 2 ) = =−
3(1 − 8 ) 73 ⋅ 32
Sol. Y Hence, (b) is the correct answer.
y = f (x)
l Ex. 17 The area of the region bounded by the curve

y = f ( x ), the X-axis and the line x = a and x = b, where


1/2 −∞ < a < b < −2 is
X
O 1 3/2 x1 2 x2 5/2 x3 3 b x
y = g(x) (a) ∫ dx + bf (b ) − af (a )
a 3 [{ f ( x )} 2 − 1]
Hence, (a) is the correct answer. b x
(b) − ∫ dx − bf (b ) + af (a )
3 [{ f ( x )} 2 − 1]
a
l Ex. 14 In the above question, the value of n is b x
(c) ∫ dx − bf (b ) + af (a )
(a) 1 (b) 2 (c) 3 (d) 4 a 3 [{ f ( x )} 2 − 1]

Sol. xn+ 1 = x 3 ⇒ n = 2. b x
(d) − ∫ dx − bf (b ) + af (a )
Hence, (b) is the correct answer. a 3 [{ f ( x )} 2 − 1]

b b
l Ex. 15 The whole area bounded by y = f ( x ), y = g ( x ) Sol. Required area = ∫ f ( x ) dx = [ xf ( x )]ba − ∫ xf ′ ( x ) dx
a a
b x
and x = 0 is = bf (b ) − af (a ) + ∫ a 3[{ f (x )} 2 − 1] dx
11 8 13
(a) (b) (c) 2 (d) Hence, (a) is the correct answer.
8 3 3
1 1 11 1
Sol. Required area = ∫ sin 4 πx dx − ∫ ln x dx =
0 0 8
l Ex. 18 ∫ −1 g ′ ( x ) dx is equal to
Hence, (a) is the correct answer. (a) 2g( −1) (b) 0 (c) −2g(1) (d) 2g(1)
1
Passage III Sol. I = ∫ g′ ( x ) dx = [ g( x )]1−1 = g(1 ) − g( −1 )
−1
(Q. Nos. 16 to 18)
Since, y 3 − 3y + x = 0 ...(i)
Consider the function defined implicitly by the equation
y 3 − 3y + x = 0 on various intervals in the real line. If and y = g( x )
Since, { g( x )} 3 − 3 g( x ) + x = 0 [by Eq. (i)]
x ∈( −∞,−2 ) ∪ ( 2, ∞ ), the equation implicitly defines a
unique real-valued differentiable function y = f ( x ). If At x = 1, { g(1 )} − 3 g(1 ) + 1 = 0
3
...(ii)
x ∈( −2, 2 ), the equation implicitly defines a unique At x = −1, { g( −1 )} 3 − 3 g( −1 ) − 1 = 0 ...(iii)
real-valued differentiable function y = g ( x ) satisfying
On adding Eqs. (i) and (ii), we get
g(0 ) = 0. [IIT JEE 2008]
{ g(1 )} 3 + { g( −1 )} 3 − 3 { g(1 ) + g( −1 )} = 0
l Ex. 16 If f ( −10 2 ) = 2 2, then f ′′( −10 2 ) is equal to { g(1 ) + g( −1 )} { g(1 ) 2 + g( −1 ) 2 − g(1 ) g( −1 ) − 3 } = 0
4 2 4 2 4 2 4 2 ⇒ g(1 ) + g( −1 ) = 0, g(1 ) = − g( −1 )
(a) (b) − (c) (d) −
7 33 2 7 33 2 7 33 7 33 ⇒ I = g(1 ) − g( −1 ) = g(1 ) − ( − g(1 )) = 2 g(1 )
Sol. ∴ y 3 − 3y + x = 0 Hence, (d) is the correct answer.
192 Textbook of Integral Calculus

Subjective Type Questions


l Ex. 19 Find the total area bounded by the curves ⇒ log y = ax + log c
 π2  ⇒ y = ceax , which passes through P (1, 1 )
y = cos x − cos 2 x and y = x 2  x 2 − . −a
 4  ∴ c =e

 π 2 ⇒ Curve is y = e − a eax ⇒ y = e a ( x − 1)
Sol. Here, y = cos x − cos x and y = x  x 2 −  could be
2 2
 4 Y
drawn as in figure. 1 +1
y = x2(x2 – π )
Y 2 a
4
a P (1, 1)
e

X
O π
–π
2 y=cos x
2 X
– cos2 x O

1 1 1 + 1/a

Thus, the area = 2 ∫


π /2

  π 2  
(cos x − cos2 x ) − x 2  x 2 −   dx
∴Required area =
a ∫e −a
(log y + a ) dy + ∫
1
{(1 + a ) − ay }dy

   4  
0 1
1+
1  a  a
π /2  π2 2  = {y (log y − 1 ) + ay }1e –a +  (1 + a ) y − y 2 
=2 ∫  cos x − cos2 x − x 4 + x  dx a  2 a
0  4 
1
π /2 = {( − 1 + a ) − e − a ( − a − 1 ) − ae − a}
 x sin 2 x x 5 π 2x 3  a
= 2  sin x − − − +
2 4 5 12  0  
  a 
2
 1 1
+ (1 + a ) 1 + − 1 − 1 +  − 1 2 
 π π5   a  2  a
=2− +    
 2 120 
1 1 + a a  1 1 
= [ − 1 + a + ae − a + e − a − ae − a ] +  − 2 +  
a  a 2  a a 
l Ex. 20 A curve y = f ( x ) passes through the point

P (1, 1), the normal to the curve at P is a (y − 1) + ( x − 1) = 0. If 1 1 1  e−a 1


= ( − 1 + a + e −a ) +  + 1 − 1 −  = 1 + −  sq units
the slope of the tangent at any point on the curve is propor- a  a 2a   a 2a 
tional to the ordinate of that point, determine the equation
of the curve. Also obtain the area bounded by the Y-axis, the lEx. 21 Sketch the region bounded by the curves y = x 2 and
curve and the normal to the curve at P. 2
Sol. Here, slope of the normal at P ( x, y ). y= . Find the area.
1+ x2
1
⇒ Slope of the line a ( y − 1 ) + ( x − 1 ) = 0 is − 2
a Sol. For intersection point, x 2 =
∴ Slope of the tangent at P = a, 1 + x2
 dy  i.e. x4 + x2 − 2 = 0
  =a ...(i)
 dx  P i.e. (x 2 + 2) (x 2 − 1) = 0
It is given that the slope of the tangent at any point on the i.e. x=±1
curve y = f ( x ) is proportional to the ordinate of the point.
Y
dy dy y = x2
∴ ∝y ⇒ = λy
dx dx
 dy 
⇒   =λ ⇒ a=λ
 dx  (1, 1)
2
y=
1 + x2
dy dy X
∴ = ay ⇒ = a dx O
dx y
Chap 03 Area of Bounded Regions 193

1  2  ∴ The equation of the tangent to the curve at the point


Hence, required area = 2 ∫  − x 2  dx
0  1+x 2
  π   π
 , 1  is y − 1 = 2  x − 
1
 4   4
 x3   π 1
= 2 2 tan − 1 x −  = 2  −  π 1
3  2 3 when y = 0; x = − = OT
 0 4 2
Now, the required area = Area of curvilinear ∆OPN − Area of ∆PTN
l Ex. 22 Find the area enclosed between the curves π /4 1
=∫ (tan x ) dx − ⋅ NT ⋅ PN
1 0 2
y = log ( x + e ); x = log e   and X-axis.
y  π /4 1  π π 1 1 1
= [log (sec x )]0 −  − +  ⋅ 1 =  log 2 − 
2  4 4 2 2 2
Sol. The given curves are y = log ( x + e ) and
 1  1 Ex. 24 Find all the possible values of b > 0, so that the
x = loge   ⇒ = e x ⇒ y = e − x
l

 y  y
area of the bounded region enclosed between the parabolas
Using graph transformation we can sketch the curves. x2
x=–e y = x − bx 2 and y = is maximum.
Y b
x2
Sol. Eliminating y from y = and y = x − bx 2, we get
(0, 1) b
x 2 = bx − b 2x 2
X
(1– e, 0) O b
⇒ x = 0,
1 + b2
y

0 ∞
Hence, required area = ∫ log ( x + e ) dx + ∫0 e − x dx
1 −e
e ∞
=∫ log (t ) dt + ∫0 e − x dx
1
(putting x + e = t ) x
= [t log t − t ]1e − [e −x
] ∞
0 =1 + 1 =2
O
(1 +bb , 0 (
2

l Ex. 23 Find the area of the region bounded by the curve


c : y = tan x , tangent drawn to c at x = π /4 and the X-axis. Thus, the area enclosed between the parabolas,
Sol. The given curve is y = tan x b /(1 + b 2 )  x2
A=∫  x − bx 2 −  dx
dy 0  b
∴ = sec 2 x
dx b /1 + b 2  2 
2 1 + b 
 dy  π =∫ x − x    dx
  = sec 2 = 2 0
  b 
 dx  x = π /4 4
b /1 + b 2
 x2 x3 1 + b2 1 b2
= − ⋅  = ⋅
Y  2 3 b  0
6 (1 + b 2 ) 2
dA
P For maximum value of A, =0
1 db
dA 1 (1 + b 2 ) 2 ⋅ 2b − b 2 ⋅ 2 (1 + b 2 ) ⋅ 2b 1 b (1 − b 2 )
But = ⋅ = ⋅
db 6 (1 + b 2 ) 4 3 (1 + b 2 ) 3
dA
T N
X Hence, = 0 gives b = − 1, 0, 1 since b > 0
O db
Therefore, we consider only b = 1
dA
Sign scheme for around b = 1 is as below
db
+
0 ∞
1 –
π
Also, at x = ; y = 1 From sign scheme it is clear that A is maximum.
4
194 Textbook of Integral Calculus

l Ex. 25 Let C 1 and C 2 be the graphs of the function e log x  ∞ 


= lim  − form
x → 0+ 1/x  ∞ 
y = x 2 and y = 2 x , 0 ≤ x ≤ 1, respectively. Let C 3 be the graph
e (1 / x )
of a function y = f ( x ); 0 ≤ x ≤ 1, f (0 ) = 0. For a point P on C 1 , = lim = 0 (using L’Hospital’s rule)
x → 0+ (− 1 / x 2 )
let the lines through P parallel to the axes, meets C 2 and C 3
at Q and R respectively. If for every position of P on (C 1 ), the Thus, the first curve starts from (0, 0) but does not
include (0, 0). Now, the given curves intersect therefore
areas of the shaded region OPQ and ORP are equal, deter-
log x
mine the function f ( x ). [IIT JEE 1998] ex log x =
ex
Sol. On the curve C1, i.e. y = x 2.
i.e. (e 2x 2 − 1 ) log x = 0
Let P be (α , α ). So, ordinate of point Q on C 2 is also α .
2 2
1
i.e. x = 1, (Q x > 0)
α2 y e
Now, C 2 ( y = 2 x ) the abscissae of Q is given by x = = ⋅
2 2 Y
 α2 
∴ Q is  , α  and R on C 3 is {α , f ( x )}.
 2 
1
α2 α2  y e
Now, area of ∆OPQ = ∫ ( x1 − x 2 ) dy = ∫  y −  dy X
0 0  2 O 1
2 3 α4
= α − …(i)
3 4
α α2
Again, area of ∆ORP = ∫ (y1 − y 2 )dx = ∫ { x 2 − f ( x )} dx ...(ii)
0 0
Therefore, using the above results figure could be drawn as
1  log x 
Y ∴ Required area = ∫  − ex log x  dx
1/e  ex 
(1/2, 1)
(0, 1) (1, 1) 1 1
C2 C1 1 (log x ) 2  x2  e2 − 5
=   − e  (2 log x − 1 )  =
Q e  2 1/e 4 1/e 4e
P
l Ex. 27 Let An be the area bounded by the curve
(1, 0)
π
O
X
y = (tan x ) n and the lines x = 0, y = 0 and x = ⋅ Prove that
4
1
C3 R for n > 2; An + An − 2 = and deduce that
n −1
Thus, from Eqs. (i) and (ii), we get 1 1
< An < ⋅
2α 3 α 4 α 2n + 2 2n − 2
− = ∫ { x 2 − f ( x )} dx
3 4 0
π /4
Differentiating both the sides w.r.t. α, we get Sol. First part We have, An = ∫ (tan x )n dx
0
2α 2 − α 3 = α 2 − f (α ) π /4
Hence, An − 2 = ∫ (tan x )n − 2 dx
0
⇒ f (α ) = α 3 − α 2 ⇒ f ( x ) = x 3 − x 2
π /4
∴ An + An − 2 = ∫ (tan x )n − 2 (tan 2 x + 1 ) dx
0
l Ex. 26 Find the area of the region bounded by the π /4
log x =∫ (tan x )n − 2 ⋅ sec 2 x dx
curves y = ex log x and y = ⋅ 0

ex [IIT JEE 1990] Let tan x = t, so that sec 2 x dx = dt


Sol. Both the curves are defined for x > 0. Both are positive when 1 n−2  tn − 1 
1
1
x > 1 and negative when 0 < x < 1. ∴ An + An − 2 = ∫ t dt =   = …(i)
0  n −1  n −1
We know that, lim log x → − ∞ 0
x → 0+ Second part
log x Since, 0 ≤ x ≤ π / 4 ∴ 0 ≤ tan x ≤ 1
Therefore, lim →−∞
x → 0+ ex ⇒ tann + 2 x < tann x < tann − 2 x
Thus, Y-axis is asymptote of second curve. π /4 π /4 π /4
⇒ ∫0 tann + 2 x dx < ∫ tann x dx < ∫ tann − 2 x dx
and lim ex log x [( 0 ) ( − ∞ ) form] 0 0
+
x→ 0
⇒ An + 2 < An < An − 2 ⇒ An + An + 2 < 2 An < An + An − 2
Chap 03 Area of Bounded Regions 195

1 1 Hence, the intersection points in the first quadrant is


∴ < 2 An <
n+1 n −1 ( 2 − 1, 2 − 1 ).
1 1 ∴ Required area = 8 [Area of curvilinear ∆OLM ]
⇒ < An < [using Eq. (i)]
2 (n + 1 ) 2 (n − 1 ) 1 1/ 2 
= 8 ( 2 − 1) ( 2 − 1) + ∫ 1 − 2 x dx
2 2 −1 
l Ex. 28 Consider a square with vertices at (1, 1) (–1, 1), 1  2 (1 − 2 x ) 3/2 
1/ 2 
(–1, –1) and (1, –1). Let S be the region consisting of all = 8  (3 − 2 2 ) +   
 2  3 (− 2)  2 −1 
points inside the square which are nearer to the origin than
to any edge. Sketch the region S and find its area. 4
= (4 2 − 5)
[IIT JEE 1995] 3
Sol. For the points lying in the ∆OAB the edge AB, i.e. x = 1 is
the closest edge. Therefore, if the distance of a point P ( x, y ) l Ex. 29 Sketch the region included between the curves
(lying in the ∆OAB ) from origin is less than that of its x + y 2 = a 2 and | x | + | y | = a (a > 0 ) and find its
2
distance from the edge x = 1 it will fall in the region S.
∴ OP ≤ PQ area.
Y Sol. The graphs | x | + | y | = a and | x | 2 + | y | 2 = a 2 are as shown
y=x in figure.
D A Y
P
Q (0, a)
(x,y)
X
O

C B X
(–a, 0) O (a, 0)
y = –x

⇒ x2 + y 2 ≤ 1 − x
(0, –a)
⇒ x 2 + y 2 ≤ x 2 − 2x + 1
From the figure it can be concluded that when powers of | x |
⇒ y 2 ≤ 1 − 2x
and | y | both is reduced to half the straight lines get stretched
Similarly, for points lying in the ∆OAD the side y = 1 is the inside taking the shape as above.
closest side and therefore the region S is determined by
Thus, required area = 4 [shaded area in the first quadrant]
x 2 ≤ 1 − 2y
 πa 2 a 
Since, the edges are symmetric about the origin. Hence, by the =4 − ∫ ( a − x ) 2 dx 
above inequality and by symmetry, the required area will be the  4 0

shaded portion in the figure given below (since in 1st quadrant x, y > 0 ), hence
Y y=x | x| + |y | = a
⇒ x + y = a

M ⇒ y = ( a − x )2

L
O N (1/2,0) X (0, a)
x 2 + y 2 = a2

(a, 0)
(–a, 0) O
y = –x x + y = a

Now, when the curves y = 1 − 2 x and y = x intersect each


2

other, then x + y =a
(0, –a)
x 2 = 1 − 2x ⇒ x 2 + 2x − 1 = 0
 2
⇒ x = 2 − 1, − 2 − 1 Hence, required the area =  π −  a 2
 3
196 Textbook of Integral Calculus

l Ex. 30 Show that the area included between the parabo- Hence, required area
8 1
= ∫ {( x + x x ) − (x − x
las y 2 = 4a ( x + a ) and y 2 = 4b (b − x ) is (a + b ) ab . 0
x )} dx
3
Sol. Given parabolas are 1
= ∫ (2 x x ) dx
y = 4a ( x + a )
2
…(i) 0
1 4
and y 2 = 4b (b − x ) …(ii) = 2 ∫ x 3/2 dx = sq unit
0 5
Solving Eqs. (i) and (ii), we get
x = (b − a ) and y = ± 2 ab l Ex. 32 Prove that the areas S 0 , S1 , S 2 ,… bounded by the
∴ P and Q are (b − a, 2 ab ) and (b − a, − 2 ab ) respectively, X-axis and half-waves of the curve y = e − αx sin βx , x 0.
and the points A and A′ are ( − a, 0 ) and (b, 0 ), respectively. form a geometric progression with the common ratio
Y g = e − π α /β .
P
Sol. The curve y = e − α x sin βx intersects the positive X-axis at the
points where y = 0.
∴ e − α x sin βx = 0
X
A O M A′ Y
(–a, 0) (b, 0) +1
y = e –αx
S0
Q S2
X
Now, required area = Area APA′ QA = 2 Area APA′ A O π/β 2π/β 3π/β S3
S1
= 2 [ Area APMA + Area MPA′ M ] –αx
y = –e
b −a
=2  2 b (b − x ) dx 
b
 ∫ −a 2 a (a + x ) dx + ∫b −a 
–1

b −a nπ
b
⇒ sin βx = 0 ⇒ xn = , n = 0, 1, 2, …
=4 a ∫ −a a + x dx + 4 b ∫b −a b − x dx
β

2 
b −a
 2 
b The function y = e − α x sin βx is positive in the interval
=4 a (a + x ) 3/2 + 4 b − (b − x ) 3/2
3  −a  3  b −a
( x 2K , x 2 K + 1 ) and negative in ( x 2K + 1, x 2K + 2 ), i.e. the sign of the
function in the interval ( xn , xn + 1 ), therefore
8 8 b 8 (n + 1) π / β − α x
= a (b ) 3/2 + (a ) 3/2 = ab (a + b ) sq units Sn =  ∫ e sin βx dx 
3 3 3  n π /β 
(n + 1) π / β
  ( − 1 )n + 1 e − αx  
l Ex. 31 Determine the area of the figure bounded by two =  (α sin β x + β cos β x )  
branches of the curve (y − x ) 2 = x 3 and the straight line x =1. α +β
2 2
 n π /β 
Sol. Given curves are (y − x ) 2 = x 3 βe − n π α / β
= {1 + e − π α /β }
y −x=±x x (α 2 + β 2 )
y =x+x x …(i) βe − (n + 1) π α /β {1 + e − π α /β }
y =x−x Sn + 1 (α 2 + β 2 )
x …(ii) Hence, g = = − n π α /β
= e − π α /β
Sn βe {1 + e − π α /β }
and x =1 …(iii)
α 2 + β2
Which could be drawn as; shown in figure.
which completes the proof.
Y

x
x l Ex. 33 Let b ≠ 0 and for j = 0, 1, 2, ..., n. Let S j be the area
x+
y=
y=x–x x
of the region bounded by Y-axis and the curve x ⋅ e ay = sin by ,
jπ ( j + 1) π
X ≤y ≤ . Show that S 0 , S1 , S 2 , ..., Sn are in geomet-
O 1/2 1 2 b b
ric progression. Also, find their sum for a = −1 and b = π.
( j + 1) π /b ( j + 1) π /b − ay sin by
Sol. Here, S j =  ∫ x dy  =  ∫ e dy 
 j π /b   j π /b 
x=1
Chap 03 Area of Bounded Regions 197

( j + 1) π /b  1  e t1 + e − t1 
  e − ay    e t1 − e − t1  t1 dx 
=  2 ( − a sin by − b cos by )   =2      −∫ y dt 
a +b 2 2   2  dt 
2 1
  j π /b 
 e 2t1 − e − 2t1  t1  e t − e − t 
2 
 e − a ( j + 1) π /b e − aj π /b 
= × (− b ) (− 1) j + 1 − 2 ( −b ) ( −1 ) j  = 2   −∫   dt 
 a +b
2 2
a + b2   8  0  2  
 e − a ( j + 1) π /b e − ajπ /b  e 2t1 − e − 2t1 1 t1
=|b |  +  = − ∫0 (e 2t + e − 2t − 2 ) dt
 a +b a2 + b2 
2 2
4 2
t1
e − ajπ /b − aπ /b e 2t1 − e − 2t1 1  e 2t1 e − 2t1 
=|b | {e + 1}; { j = 0, 1, 2, ..., n} = −  − − 2t1 
a2 + b2 4 2 2 2 0
S j +1 e − a ( j + 1) π /b − aπ /b e − ajπ /b − aπ /b
Now, = |b | {e + 1} | b | 2 {e + 1} e 2t1 − e − 2t1 1  e 2t1 e − 2t1 
Sj a +b
2 2
a + b2 = −  − − 2t1  = t1
4 2 2 2 
= e − aπ /b for all j = 0, 1, 2, ..., n
Hence, S 0, S1, S 2, ..., Sn are in GP with common ratio e − aπ /b . l Ex. 35 Find the area enclosed by circle x 2 + y 2 = 4,
For a = − 1 and b = π, we have  x x 
parabola y = x 2 + x + 1, the curve y =  sin 2 + cos  and
e j π (e + 1 )  4 4 
Sj = ; j = 0, 1, 2, ..., n
π2 + 1 X-axis (where, [.] is the greatest integer function).
n n
e jπ (e + 1 ) (e + 1 ) π  en + 1 − 1   x x 
y = sin 2 + cos
∴ ∑ Sj = ∑ π2 + 1
= 
π + 1  e −1 
 Sol. Q
 4 4 
j =0 j =0
x x
Q 1 < sin 2 + cos < 2 for x ∈ ( − 2, 2 ]
4 4
l Ex. 34 For any real
 2 x x 
et + e − t et − e − t ∴ y = sin + cos =1
t, x = 2 + ,y = 2 + is a point on the hyper-  4 4 
2 2
Y
bola x 2 − y 2 − 4 x + 4y − 1 = 0. Find the area bounded by the
(0, 2)
hyperbola and the lines joining the centre to the points
corresponding to t 1 and − t 1 .
et + e − t et − e − t
Sol. The points x = 2 + ,y =2 + is on the curve
2 2
X
( x − 2 ) 2 − (y − 2 ) 2 = 1 or x 2 − y 2 − 4 x + 4y − 1 = 0 (–2, 0) – 3 –1 –1/2 O 3 (2, 0)

Y
t)
P( 1
(0, –2)

A′ Now, we have to find out the area enclosed by the circle


A
X 2
C  3   1
x 2 + y 2 = 4, parabola  y −  =  x +  , line y = 1 and
 4  2
(–t
1)
X-axis. Required area is shaded area in the figure.
Q
Hence, required area
0 2
Put ( x − 2 ) = x, (y − 2 ) = y = 3 × 1 + ( 3 − 1 ) × 1 + ∫ ( x 2+ x + 1 ) dx + 2 ∫ ( 4 − x 2 ) dx
−1 3
et + e − t et − e − t
∴ x − y = 1 and x =
2 2
,y = x3 x2  x
0 2
 x 
2 2 = (2 3 − 1 ) +  + + x + 2 4 − x 2 + 2 sin −1   
 3 2  −1  2  2 
We have to find the area of the region bounded by the curve 3
x 2 − y 2 = 1 and the lines joining the centre x = 0, y = 0 to the
  1 1    3 2π  
point (t1 ) and ( − t1 ). = (2 3 − 1 ) + 0 −  − + − 1  + 2 ( 0 + π ) −  + 
  3 2    2 3 
 et 1 + e − t1 
∴Required area = 2 Area of ∆PCN − ∫ y dx  5 2π  2π 1
2
= (2 3 − 1 ) + + − 3 = + 3 −  sq units
 1  6 3  3 6
 
198 Textbook of Integral Calculus

l Ex. 36 Let f ( x ) = max { x 2 , (1 − x ) 2 , 2 x (1 − x )} , where Y

0 ≤ x ≤ 1. Determine the area of the region bounded by the (0, 3)


curves y = f ( x ), X -axis, x = 0 and x =1. [IIT JEE 1997]
Sol. We have, f ( x ) = max { x 2, (1 − x 2 ), 2 x(1 − x )}
X
Graphically it could be shown as; O
(–2, 0)
y = –1
Y (2, 0)
(0,– 3)
y = x2

g
y = (1–x)2

ctin
x

gle
max
 12 − 3 x 2 
ma

2 6 /3
Ne Required area = ∫  − 1  dx
Ne

− 2 6 /3  4 
gle

max y = 2x (1–x)
cti
n

3 2 6 /3 2 6 /3
g

X
=
2 ∫−2 6 /3
4 − x 2 dx − ∫
− 2 6 /3
1 dx
O 1 1 2 1
Ne

3 2 3 2 2 4
gl

= 2 sin − 1 + − 6
ect

3 3 3
in g

For figure it is clear that maximum graph (i.e. above max graph  − 2, − 3 ≤ x ≤ 0
l Ex. 38 Let f (x ) =  , where
 x − 2, 0 < x ≤ 3
is considered and others are neglected).
 1 
∴ For x ∈ 0, , x 2 ≤ 2 x (1 − x ) ≤ (1 − x ) 2 g ( x ) = min { f (| x | ) + | f ( x ) |, f (| x | ) − | f ( x ) | }. Find the
 3 
area bounded by the curve g ( x ) and the X-axis between the
1 1  ordinates at x = 3 and x = − 3.
For x ∈ , , x 2 ≤ (1 − x ) 2 ≤ 2 x (1 − x )
 3 2 
− x − 2, − 3 ≤ x ≤ 0
1 2  Sol. Here, f (| x | ) = 
For x ∈ , , (1 − x ) 2 ≤ x 2 ≤ 2 x (1 − x )  x − 2, 0 < x ≤ 3
 2 3 
 2, − 3 ≤ x ≤ 0
2  
For x ∈ , 1 , (1 − x ) 2 ≤ 2 x (1 − x ) ≤ x 2 | f ( x ) | = − x + 2, 0 < x ≤ 2
 3   x − 2, 2 < x ≤ 3

Hence, f ( x ) can be written as
 − x − 4, − 3 ≤ x ≤ 0
(1 − x ) 2 , for 0 ≤ x ≤ 1 / 3 
 ∴ f (| x | ) − | f ( x ) | =  2 x − 4, 0 < x ≤ 2
f ( x ) =  2 x (1 − x ), for 1 / 3 ≤ x ≤ 2 / 3  0,
  2 < x ≤3

2
x , for 2 / 3 ≤ x ≤ 1
Hence, the area bounded by the curve y = f ( x ); X -axis and the Y
f (x)
lines x = 0 and x = 1 is given by
1/ 3 2/ 3 1 (3, 1)
=∫ (1 − x ) 2 dx + ∫ 1/ 3 2 x (1 − x ) dx + ∫ 2/ 3 ( x
2
) dx
0 X
–3 O 2 3
17
= sq unit
27
–2
l Ex. 37 Find the ratio in which the curve, Graph of f (x)
y = [ − 0.01 x 4 − 0.02 x 2 ] [where, [⋅] denotes the greatest inte- Y
y = (|x|)
ger function) divides the ellipse 3 x 2 + 4y 2 = 12.
Sol. Here, y = [ − 0.01 x 4 − 0.02 x 2 ]
X
i.e. y = − 1, when − 2 < x < 2 –2 O 2
y = − 1 cut the ellipse 3 x 2 + 4y 2 = 12
8 2 2 (0, –2)
At x 2 = or x = ±
3 3 Graph of f (|x|)
Chap 03 Area of Bounded Regions 199

Y  1 
l Ex. 40 Let O (0, 0 ), A ( 2, 0 ) and B 1,  be the vertices
 3
(3, 1)
of a triangle. Let R be the region consisting of all those
X points P inside ∆ OAB which satisfy
O 2 3
d (P , OA ) ≤ min {d (P , OB ), d (P , AB )} , when ‘d’ denotes the
distance from the point to the corresponding line. Sketch the
region R and find its area. [IIT JEE 1997]
Graph of |f (x)| Sol. Let the coordinate of P be ( x, y ).
Equation of line OA ≡ y = 0
Y
Equation of line OB ≡ 3y = x
Equation of line AB = 3y = 2 − x
–3
X d ( P , OA ) = Distance of P from line OA = y
O 2 3 | 3y − x |
(–3,–1) d ( P , OB ) = Distance of P from line OB =
2
| 3y + x − 2 |
d ( P , AB ) = Distance of P from line AB =
(0,–4) 2
Graph of g(x)
Y
Since, | f ( x ) | is always positive. 1
B
3
g( x ) = f (| x | ) − | f ( x ) |
where the graphs could be drawn as shown in above figures.
P
From the graph, required area
1 1  23 X
= (1 + 4 ) × 3 +  × 2 × 4 + 0 = sq units O (0, 0) A (2, 0)
2 2  2

l Ex. 39 Let ABC be a triangle with vertices Given, d ( P , OA ) ≤ min {d ( P , OB ), d ( P , AB )}


A ≡ (6, 2 ( 3 + 1)), B ≡ ( 4, 2 ) and C ≡ (8, 2 ). Let R be the region
 | 3y − x | | 3y + x − 2 | 
consisting of all those points P inside ∆ ABC which satisfy y ≤ min  , 
 2 2 
d (P , BC) ≥ max {d (P , AB), d (P , AC)}, where d (P , L ) denotes
the distance of the point P from the line L. Sketch the | 3y − x |
⇒ y ≤ …(i)
region R and find its area. 2
Sol. It is easy to see that ABC is an equilateral triangle with side | 3y + x − 2 |
and y ≤ …(ii)
of length 4. BD and CE are angle bisectors of angle B and C , 2
respectively. Any point inside the ∆ AEC is nearer to AC
| 3y − x |
than BC and any point inside the ∆ BDA is nearer to AB than Case I If y ≤
BC. So any point inside the quadrilateral AEGC will satisfy 2
the given condition. Hence, shaded region is the required x − 3y
region, whose area is to be found, shown as in figure ∴ y ≤ , i. e. x > 3y (Q 3y − x < 0 )
2
A (6, 2( 3 + 1)) ⇒ (2 + 3 ) y ≤ x
⇒ y ≤ (2 − 3 ) x
E
D ⇒ y ≤ x tan 15 ° …(iii)
G
(Q y = x tan 15 ° is an acute angle bisector of ∠AOB )
| 3y + x − 2 |
Case II If y ≤
B C(8, 2) 2
(4, 2) ⇒ 2y ≤ 2 − x − 3y ( i. e. 3y + x − 2 < 0 )
1
Thus, required area = 2 Area of ∆EAG = 2 × AE × EG ⇒ (2 + 3 ) y ≤ 2 − x
2
1 1 1 ⇒ y ≤ − (2 − 3 ) ( x − 2 )
= AB × CE = × 4 × 4 2 − 2 2
2 3 6 ⇒ y ≤ − (tan 15 ° ) ( x − 2 ) …(iv)
4 3
= sq units [Q y = ( x − 2 ) tan 15 ° is an acute angle bisector of CA]
3
200 Textbook of Integral Calculus

From Eqs. (iii) and (iv), P moves inside the triangle as shown in Sol. We have, x 2 + y 2 − 4 x + 2y + 1 = 0
figure. or ( x − 2 ) 2 + (y + 1 ) 2 = 4 …(i)
Y 1
B 1,( )
3
Y

Q (x–2)2 +(y+1)2 = 4
x x x x x x x
A x x x x x x x (4, 0)
x x x x x x x
X x x x x x x x X
O (0, 0) C (2, 0) O (2– 3, 0) (2+ 3, 0)
(1, 0)
3
x x
As ∠QOB = ∠OBQ = 15 °, ∆OQB is an isosceles triangle
f (x) = )100 + )= 0
35
⇒ OC = AC = 1 unit for 0 ≤ x ≤ 4
1
Area of shaded region = area of ∆OQA = (base) × (height) Now, for 0 ≤ x ≤ 4,
2
1 x3 x  x3 x 
= (2 ) (1 tan 15 ° ) = tan 15 ° 0≤ + <1 ⇒  + =0
2 100 35  100 35 
= (2 − 3 ) sq units So, we have to find out the ratio in which X-axis divides the
circle (i).
l Ex. 41 A curve y = f ( x ) passes through the origin and Now, at X-axis, y =0
lies entirely in the first quadrant. Through any point P ( x , y ) So, (x − 2)2 = 3
on the curve, lines are drawn parallel to the coordinate axes. So, it cuts the X-axis at (2 − 3, 0 ) and (2 + 3, 0 ).
If the curve divides the area formed by these lines and coor- 2+ 3
dinate axes in m : n, find f ( x ). Therefore, required area, A = ∫ ( 4 − ( x − 2 ) 2 − 1 ) dx
2− 3
x
Sol. Area of (OAPB ) = xy , Area of (OAPO ) = ∫ f (t ) dt 4π − 3 3
0 =
3
Y y = f (x) A 4π − 3 3
∴ Required ratio = =
4π − A 8π + 3 3

B P = (x, y) l Ex. 43 Area bounded by the line y = x , curve


y = f ( x ), ( f ( x ) > x , ∀ x > 1) and the lines x = 1, x = t is
X (t + 1 + t 2 ) − (1 + 2 ) for all t >1. Find f ( x ) .
A
Sol. The area bounded by y = f ( x ) and y = x between the lines
x
Therefore, area of (OBPO ) = xy − ∫
t
0
f (t ) dt x = 1 and x = t is ∫+ 1 ( f (x ) − x ) dx. But it is equal to
According to the given condition, (t + 1 + t 2 ) − (1 + 2 ).
x
xy − ∫ t

x
0
f (t ) dt
=
m So, ∫1 ( f ( x ) − x ) dx = (t + 1 + t 2 ) − (1 + 2 )

∫0 f (t ) dt n
Differentiating both the sides w.r.t. t, we get
x
⇒ n xy = (m + n ) ∫ f (t ) dt f (t ) − t = 1 +
t
⇒ f (t ) = 1 + t +
t
0
1 + t2 1 + t2
Differentiating w.r.t. x, we get
 dy  x
n x + y  = (m + n ) f ( x ) = (m + n ) y , as y = f ( x ) or f (x ) = 1 + x +
 dx  1 + x2
m dx dy m
⋅ = ⇒ ⋅ (log x ) = log y − log c, where c is a constant. l Ex. 44 The area bounded by the curve y = f ( x ), X-axis
n x y n
⇒ y = cxm /n and ordinates x =1and x = b is (b − 1) sin (3b + 4 ) , find f ( x ) .
Sol. We know that the area bounded by the curve y = f ( x ), X-axis
b
l Ex. 42 Find the ratio of the areas in which the curve and the ordinates x = 1 and x = b is ∫ f ( x ) dx.
1
 x3 x  b
y = +  divides the circle x + y − 4 x + 2y + 1 = 0
2 2 From the question; ∫ f ( x ) dx = (b − 1 ) sin (3b + 4 )
1
 100 35  Differentiating w.r.t. b, we get
(where, [.] denotes the greatest integer function). f (b ) ⋅ 1 = 3 (b − 1 ) cos (3b + 4 ) + sin (3b + 4 )
⇒ f ( x ) = 3 ( x − 1 ) cos (3 x + 4 ) + sin (3 x + 4 )
Chap 03 Area of Bounded Regions 201

l Ex. 45 Find the area of region enclosed by the curve l Ex. 46 Let f ( x ) be a function which satisfy the equation
(x − y ) 2 (x + y ) 2 f ( xy ) = f ( x ) + f (y ) for all x > 0, y > 0 such that f ′ (1) = 2.
+ = 2 (a > b), the line y = x and the posi- Find the area of the region bounded by the curves
a2 b2
tive X-axis. y = f ( x ), y = | x 3 − 6 x 2 + 11x − 6 | and x = 0.
(x − y )2 (x + y )2 Sol. Take x = y = 1 ⇒ f (1 ) = 0
Sol. The given curve + = 2 is an ellipse major 1
a b2 2
Now, y =
and minor axes are x − y = 0 and x + y = 0, respectively. x
 1   1   1 
The required area is shown with shaded region. ⇒ 0 = f  x ⋅  = f (x ) + f   ⇒ f   = − f (x )
 x   x   x 
Y  x   1 
y = –x y=x ∴ f   = f ( x ) + f   = f ( x ) − f (y ) …(i)
 y   y 
f (x + h ) − f (x )
Now, f ′ ( x ) = lim
h→ 0 h
X
 x+h 
O f 
 x 
= lim [using Eq. (i)]
h→ 0 h
 h 
f  1 +  − f (1 )
 x  f ′ (1 )
Instead of directly solving the problem we can solve equivalent = lim = [Q f (1 ) = 0 ]
h→ 0 h x
problem with equivalent ellipse whose axes are x = 0 and y = 0. ⋅x
2 x
The equivalent region is shown as (OA′ B′ O ) where the f ′ (x ) =
x2 y 2 x
equation of ellipse is 2 + 2 = 1 . ⇒ f ( x ) = 2 log x + c [since, f (1 ) = 0 ⇒ c = 0]
a b
∴ Required area = Area ( ∆OA′ A′′ + A′ A′′ B′ ) ⇒ f ( x ) = 2 log x
  Thus, f ( x ) = 2 log x and y = | x 3 − 6 x 2 + 11 x − 6 | could be
ab ab
where, A′ =  ,  plotted as
 a2 + b2 a +b 
2 2 
 y = 2 log x
Y
1 ab ab 1  a 2b 2  y=|x3– 6x2+11x–6|
Area ∆OA′ A′′ = × × =  2  …(i) 1 A
a2 + b2 2  a + b 
2
2 a2 + b2
O
a  x2  X
Area of A′ B′ A′′ = ∫ ab b  1 − 2  dx B
 a 
a2 + b2

b a

a ∫
= ⋅ ab a 2 − x 2 dx
2
a +b 2 C
a Hence, required area
b  x a 2
x 
= a2 − x2 + sin − 1 
a  2
1 0
2 a ab / = ∫ ( x 3 − 6 x 2 + 11 x − 6 ) dx + ∫−∞ e
y /2
dy
a2 + b2 0
1
b  a2 π ab  a 2b 2    x 4 6 x 3 11 x 2
= − +

− 6x  + 2 (e y /2 ) 0− ∞
= 0+ ⋅ − ⋅ a 2 − 2 
a  2 2 2 a2 + b2  a + b 2    4 3 2 
 0
a2 b   1 11 
− sin − 1  =  −2 + − 6  − ( 0 ) + 2 (e 0 − e − ∞ )
2 a 2 + b 2   4 2 
1 11 1
b  π a2 a2  b  a 3b  = + − 8 + 2 = − sq unit
=  − sin − 1   −  4 2 4
a  4 2  a 2 + b 2  2 (a 2 + b 2 ) 
   
l Ex. 47 Find the area of the region which contains all the
π ab ab   a 2b 2
= − sin − 1  b  − …(ii) points satisfying condition | x − 2y | + | x + 2y | ≤ 8 and xy ≥ 2.
4 2  a 2 + b 2  2 (a + b )
2 2
 x
Sol. The line y = ± divide the xy plane in four parts
Hence, required area = Sum of Eqs. (i) and (ii) 2
Region I 2y − x ≤ 0 and 2y + x ≥ 0
π ab ab  b 
= − sin − 1   So that, | x − 2y | + | x + 2y | ≤ 8
 
 a +b
4 2 2 2
 ⇒ ( x − 2y ) + ( x + 2y ) ≤ 8 ⇒ 0 ≤ x ≤ 4
202 Textbook of Integral Calculus

Region II 2y − x 0 and 2y + x ≥ 0  1
 {x} − , if x ∉ I
So that, | x − 2y | + | x + 2y | ≤ 8 = 2
 0, if x ∈ I
⇒ − ( x − 2y ) + ( x + 2y ) ≤ 8 ⇒ 0 ≤ y ≤ 2
Region III 2y + x ≤ 0, 2y − x ≥ 0 Thus, g( x ) = max { x 2, f ( x ), | x | }
So that, | x − 2y | + | x + 2y | ≤ 8 which could be graphically expressed as
⇒ − ( x − 2y ) − ( x + 2y ) ≤ 8 ⇒ − 4 ≤ x ≤ 0
y = x2 y = x2

max
Region II y= x y= x

x
2y –x ≥ 0, 2y+x ≥ 0

ma
y
ma x
x max ma
y =1
2
Region III Region I
x –2 –1 –1 O 1
1
2
1 2
2 2 y =– 1
2y–x ≥ 0, 2y+x ≤ 0 2y–x ≤ 0, 2y+x ≥ 0 2

 x 2, − 2 ≤ x ≤ − 1

Region IV  − x, − 1 ≤ x ≤ − 1 / 4
2y–x ≤ 0, 2y+x ≤ 0  1
Clearly, g( x ) =  x + , − 1 / 4 ≤ x ≤ 0
 2
Region IV 2y + x ≤ 0, 2y − x ≤ 0
 x, 0 ≤ x ≤ 1
So that, | x − 2y | + | x + 2y | ≤ 8 ⇒ − 2 ≤ y ≤ 0 
 x 2
, 1 ≤ x ≤2
Here, all the points lie in the rectangle.
2
Hence, required area = ∫ g( x ) dx
Y −2
−1 − 1/ 4 0  1
=∫ ( x 2 ) dx + ∫ −1 ( − x ) dx + ∫ − 1/ 4  x +  dx
−2  2
(0, 2) D 1 2
C
+ ∫ 0 x dx + ∫ 1 x 2 dx
E
−1 −1 / 4 0 1 2
X  x3  x2  x2 1   x2  x 3  275
A (1, 0) B (4, 0) =   −   +  + x  +   +   = sq
 3  −2  2  −1  2 2  −1 / 4  2 0  3  1 48
units

l Ex. 49 Find the area of the region bounded by

y = f ( x ), y = | g ( x ) | and the lines x = 0, x = 2, where f , g


Also, the hyperbola xy = 2 meets the sides of the rectangle at
the points (1, 2) and ( 4, 1 / 2 ) in the 1st quadrant graphically. are continuous functions satisfying
Hence, required area f (x + y ) = f (x ) + f (y ) − 8xy , ∀ x , y ∈R
= 2 ( Area of rectangle ABCD − Area of ABEDA ) and g (x + y ) = g (x ) + g ( y ) + 3 xy (x + y ), ∀ x , y ∈R
 4 2  Also, f ′ (0) = 8 and g ′ (0) = − 4.
=2  3 ×2 − ∫ dx  = 2 (6 − 2 log 4 ) sq units
 1 x 
Sol. Here, f ( x + y ) = f ( x ) + f (y ) − 8 xy
l Ex. 48 Consider the function Replacing x, y → 0, we get f ( 0 ) = 0
 1 f (x + h ) − f (x ) f (x + y ) − f (x )
 x − [ x ] − , if x ∉ I Now, f ′ ( x ) = lim = lim
f (x ) =  2 , where [.] denotes the great- h→ 0 h y→ 0 y
 0, if x ∈I f ( x ) + f (y ) − 8 xy − f ( x )
= lim
y→ 0 y
est integer function and I is the set of integers. If
 f (y ) 8 xy 
g ( x ) = max { x 2 , f ( x ), | x | }, − 2 ≤ x ≤ 2 , then find the area = lim  − 
y→ 0
bounded by g ( x ) when − 2 ≤ x ≤ 2.  y y 

 1  f ′ (y ) 
 x − [ x ] − , if x ∉ I = lim   − 8 x (using L’Hospital’s rule)
Sol. Here, f (x ) =  y→ 0  1 
2
 0, if x ∈ I = f ′ ( 0 ) − 8 x = 8 − 8x [given, f ′ ( 0 ) = 8 ]
Chap 03 Area of Bounded Regions 203

⇒ f ′ (x ) = 8 − 8x ⇒ sec −1 [ − sin 2 x ] = sec −1 ( − 1 ) = π


Integrating both the sides, we get Thus, to find the area bounded between
f (x ) = 8x − 4x 2 + c y = x 2 and y = π
As f (0) = 0 ⇒ c = 0
i.e. when x 2 = π or ( x = − π to x = π )
⇒ f (x ) = 8x − 4x 2 …(i)
π
g( x + y ) = g( x ) + g(y ) + 3 xy ( x + y ) π  x3 
Also, ∴ Required area = ∫ ( π − x 2 ) dx =  πx − 
Replacing x, y ⇒ 0, we get g( 0 ) = 0
− π  3 − π
g( x + y ) − g( x ) 1 4π
Now, g′ ( x ) = lim =π( π + π ) − (π π + π π ) = π
y→ 0 y 3 3

= lim
g( x ) + g(y ) + 3 x 2 y + 3 xy 2 − g( x ) l Ex. 51 Sketch the graph of cos − 1 ( 4 x 3 − 3 x ) and find
y→ 0 y the area enclosed between y = 0, y = f ( x ) and x ≥ −1 / 2.
 g(y ) y (3 x + 3 xy ) 
2
= lim  +  Sol. Here, f ( x ) = cos− 1 ( 4 x 3 − 3 x )
y→ 0
 y y 
Let x = cos θ and 0 ≤θ ≤ π
g′ ( 0 )
= + 3x = − 4 + 3x
2 2
⇒ f ( x ) = cos− 1 ( 4 cos3 θ − 3 cos θ )
1
∴ g′ ( x ) = − 4 + 3 x 2 = cos− 1 (cos 3θ ); 0 ≤ 3θ ≤ 3 π

⇒ g( x ) = x 3 − 4 x [ as g( 0 ) = 0 ] …(ii) 3θ, 0 ≤ 3θ ≤ π

Points where f ( x ) and g( x ) meets, we have ⇒ f ( x ) = 2 π − 3θ, π < 3θ ≤ 2 π
 3θ − 2 π , 2 π < 3θ ≤ 3 π
f ( x ) = g( x ) or 8 x − 4 x 2 = x 3 − 4 x 
⇒ x = 0, 2, − 6  3 cos− 1 x, 1/2 ≤ x ≤1
 −1
Y = 2 π − 3 cos x, −1/2 ≤ x <1
 −1
 3 cos x − 2 π , − 1 ≤ x < − 1 / 2
y = f(x)
− 3 / 1 − x 2 , 1/2 < x <1
y = g(x) 

X ∴ f ′ (x ) =  3 / 1 − x , − 1 / 2 < x < 1 / 2
2
O (2, 0) 
 − 3 / 1 − x , − 1 < x < − 1 / 2
2

 − 3x
 , 1/2 < x <1
 x 3 − 4 x, x ∈ [ − 2, 0 ] ∪ (2, ∞ )
(1 − x )
2 3/ 2
Now, | g( x ) | = 
 4 x − x , x ∈ [ − ∞, − 2 ] ∪ ( 0, 2 )
3  3x
and f ′′ ( x ) =  , −1/2 < x <1/2
(1 − x )
2 3/ 2
∴ Area bounded by y = f ( x ) and y = | g( x ) | between x = 0 to x = 2
2  − 3x
=∫ {(8 x − 4 x 2 ) − ( 4 x − x 3 )} dx  , −1 < x < −1/2
0  (1 − x 2 ) 3/ 2
4 Thus, the graph for f ( x ) = cos− 1 ( 4 x 3 − 3 x ) is
2
=∫ ( x 3 − 4 x 2 + 4 x ) dx = sq units
0 3
Y
l Ex. 50 Find the area of the region bounded by the curve
y = x 2 and y = sec − 1 [ − sin 2 x ], where [.] denotes the great- π
est integer function.
Sol. As we know, [ − sin 2 x ] = 0 or − 1. But sec − 1 ( 0 ) is not defined.
π/2
Y

X
–1 –1/2 O 1/2 1
y=π
1
Thus, required area = ∫ f ( x ) dx
− 1/ 2
X
– π O π =∫
1/ 2 1
(2 π − 3 cos−1 x )dx + ∫ (3 cos−1 x )dx
− 1/ 2 1/ 2
204 Textbook of Integral Calculus

 π
1/ 2  1 (iii) For λ = − a, we have y 2 = 4a ( x + a )
= 2π − 3 ∫  − sin − 1 x  dx + 3 ∫ (cos− 1 x ) dx
− 1/ 2  2  1/ 2
x 2 = 4a (y + a )
π 1 as 1/2 sin − 1 x dx = 0 
= +3∫ cos− 1 x dx
2 1/ 2  ∫ − 1/2  The point of intersection in the 1st quadrant
P = ((2 + 2 2 ) a, (2 + 2 2 ) a )
π 
 1 x 

= + 3  ( x cos− 1 x )11/2 + ∫ dx  Required area = 2 (area of ∆OPQ − area APQA )
2  1/ 2
1−x 2

1
3 3 of ∆OPQ = ⋅ (2 + 2 2 ) 2 a 2 = 2 (1 + 2 ) 2 a 2
On solving, we get = sq units 2
2
(2 + 2 2 ) a (2 + 2 2 ) a  x2 
Area of APQA = ∫ y dx = ∫  − a  dx
2a 2a  4a 
l Ex. 52 Consider two curves y 2 = 4a ( x − λ ) and
(2 + 2 2 ) a
x 2 = 4a (y − λ ), where a > 0 and λ is a parameter. Show that 1  x3  ( 2 + 2 2) a
= − a ( x ) 2a
(i) there is a single positive value of λ for which the two 4a  3  2a
curves have exactly one point of intersection in the 1st 1
quadrant find it. = [(2 + 2 2 ) 3 ⋅ a 3 − 8a 3 ] − 2 2 a 2
12a
(ii) there are infinitely many negative values of λ for which  4 2 + 12  2
the two curves have exactly one point of intersection in = a
 3 
the 1st quadrant.
(iii) if λ = − a, then find the area of the bounded by the  4 2 + 12  2
∴ Required area = 2 2 (1 + 2 ) 2 + a
two curves and the axes in the 1st quadrant.  3 
Sol. The two curves are inverse of each other. Hence, the two 4
= (15 + 8 2 ) a 2
curves always meet along the line y = x. 3
Consider, y 2 = 4a ( x − λ ) and put x = y
⇒ y 2 − 4ay + 4aλ = 0
l Ex. 53 Let f ( x ) be continuous function given by

4a ± 4 a 2 − aλ
 2x , |x|≤1
⇒ y = = 2a ± a 2 − aλ f (x ) =  2
2  x + ax + b , | x | > 1
Since, y is real ⇒ a 2 − aλ  0 or λ ≤ a
Find the area of the region in the third quadrant bounded
(i) If 0 < λ < a, then there are two distinct values of y and by the curves x = − 2 y 2 and y = f (x ) lying on the left of the
both 2 (a + a 2 − aλ ) and 2 (a − a 2 − aλ ) are positive, line 8x + 1 = 0. [IIT JEE 1999]
i.e. both points lie in the first quadrant. Sol. Given, a continuous function f ( x ), given by
If λ = a, then y = 2a only, i.e. only one point of Y
intersection (2a, 2a ).
Hence, there is exactly one point of intersection in 1st
quadrant for λ = a. It is infact the points of tangency of
the two curves.
(ii) If λ < 0, then y = 2 (a + a 2 − aλ ) > 0 and X
–2 –1 O 1
y = 2 (a − a − aλ ) < 0. i.e. the only point of intersection
2
–1
is in the first quadrant, the other in the 3rd quadrant.
Hence, there are infinitely many such values.

x2 = 4a (y + a) –2
y=x
Y
1
x = –2 x = –1 x = –
y 2 = 4 a (x + a ) 8
B P  2 x, | x | ≤1
(0, 2a) f (x ) =  2
 x + ax + b , | x | >1

 x 2 + ax + b, − ∞ < x < − 1
Q 
X i.e. f (x ) =  2 x, −1 ≤ x ≤1
O A
 2 1<x<∞
(2a, 0)  x + ax + b ,
Q f is continuous.
Chap 03 Area of Bounded Regions 205

∴ It is continuous at x = − 1 and x = 1. l Ex. 54 Let [ x ] denotes the greatest integer function.


∴ (−1) + a (− 1) + b = 2 (− 1)
2
Draw a rough sketch of the portions of the curves
and (1 ) 2 + a (1 ) + b = 2 (1 ) x 2 = 4 [ x ] y and y 2 = 4 [ y ] x that lie within the square
⇒ 1 −a + b = −2 {( x , y ) | 1 ≤ x ≤ 4, 1 ≤ y ≤ 4 }. Find the area of the part of the
and 1 + a + b =2 square that is enclosed by the two curves and the line
⇒ −a + b = −3 x + y = 3.
and a + b =1 Sol. We have, 1≤x≤4 and 1 ≤ y ≤ 4
⇒ b = − 1 and a = 2 ⇒ 1 ≤ x ≤ 2 and 1 ≤ y ≤ 2
x 2 + 2 x − 1, when − ∞ < x < − 1
∴ f (x ) =  ⇒[ x ] = 1 and [ y ] = 1 for all ( x, y ) lying with in the square.
 2 x, when − 1 ≤ x ≤ 1

Now, y = x 2 + 2x − 1 = (x + 1)2 − 2 Y

or (y + 2 ) = ( x + 1 ) 2 (1,4) (4,4)
x+y=3 4
y
We need the area of the region in third quadrant bounded by x]
4[
the curves a = − 2y 2, y = f ( x ) lying on the left of the line 2 =
3 x
8 x + 1 = 0.

x
y]
⇒ (y + 2 ) = ( x + 1 ) 2

4[
2

=
y2
Cuts the Y-axis at (0, –1) and the X-axis at
( − 1 − 2, 0 ) and ( − 1 + 2, 0 ). 1

When x = 1 and y = 2 X
O 1 2 3 4
Solving x = − 2y 2 and (y + 2 ) = ( x + 1 ) 2, we get
(y + 2 ) = ( − 2y 2 + 1 ) 2
Thus, x 2 = 4 [ x ] y and y 2 = 4 [ y ] x
⇒ 4y − 4y + 1 − y − 2 = 0
4 2
⇒ x 2 = 4y and y 2 = 4 x when 1 ≤ x, y ≤ 4 which
⇒ 4y − 4y − y − 1 = 0
4 2
could be plotted as;
For y = − 1, 4y 4 − 4y 2 − y − 1 = 0 Thus, required area
2 4  x2 
At y = − 1, x = − 2 =∫ (2 x − 3 + x ) dx + ∫2 2 x −  dx
1  4 
−1  x 
∴ Required area = ∫  − − − ( x + 1 ) + 2  dx
2
2 4
−2
 2   4 x2   4 x3 
=  x 3/ 2 − 3 x +  +  x 3/ 2 − 
− 1/ 8  x   3 2 1  3 12 
∫ −1
2
+  − − − 2 x  dx
 2   8 2   4 1
= −6 + 2  −  −3 + 
 
−1
 
−1/ 8  3   3 2
( − x / 2 ) 3/2 ( x + 1 ) 3  ( − x / 2 ) 3/2 2 x 2 
= − + 2x  +  3 −   32 64   8 2 8 
3 3 2  + −  − − 
 (1 / 2 )   (1 / 2 )  3 12   3 12 
 2 − 2  2 − 1
19
−1 − 1/ 8 = sq units
 4  x  3/ 2 ( x + 1 ) 3   4  x  3/ 2  6
=  −  − + 2x  +   −  − x 2 
 2  2
3 3 − 2  3 − 1
l Ex. 55 Find all the values of the parameter a (a ≥1) for
 4  1  3/ 2   4 (− 1)3 
=    − 0 − 2  −  (1 ) 3/2 − −4 which the area of the figure bounded by pair of straight lines
 
3 2  3 3  1
y 2 − 3y + 2 = 0 and the curves y =[a ] x 2 , y = [a ] x 2 is
 4  1  3/ 2   4  1  3/ 2 1  2
−    − (− 1)2  +    −  greatest, where [.] denotes the greatest integer function.
   
3 2   3 16 64 
1
 4 4 1 4 4 1  Sol. The curves y = [a ] x 2 and y = [a ] x 2 represent parabolas
= −2 − − + 4 − +1+ −  2
 3 ⋅ 2 2 3 3 3 ⋅ 2 2 3 (64 ) 64  which are symmetric about Y-axis.
4 1 257 The equation y 2 − 3y + 2 = 0 gives a pair of straight lines
= + = sq units
3 3(64 ) 192 y = 1, y = 2 which are parallel to X-axis.
206 Textbook of Integral Calculus

Thus, the area bounded is shown as l Ex. 56 Find the area in the first quadrant bounded by
Y [ x ] + [y ] = n, where n ∈N and y = i (where, i ∈N ∀ i ≤ n +1) ,
[ ⋅ ] denotes the greatest integer less than or equal to x.
y = [a] x2 y = 1 [a] x2
2 Sol. As we know, [ x ] + [y ] = n ⇒ [ x ] = n − [y ]
y=2 When y = 0, [ x ] = n ⇒n ≤ x < n + 1
When y = 1, [ x ] = n − 1 ⇒n − 1 ≤ x < n − 2
y=1 When y = 2, [ x ] = n − 2
⇒ n − 2 ≤ x < n − 3. ... and so on.
X When y = n, [ x ] = 0 ⇒ 0 ≤ x < 1
O
which could be shown as
From the above figure,
y=2
Required area = 2 ∫ ( x 2 − x1 ) dy
y =1 n=1
n
2  2y y 
=2 ∫  −  dy D
1  [a ] [a ]  C
A B
2 ( 2 − 1) 2 2 ( 2 − 1 ) 2 3/ 2 2
=
[a ] ∫1 y dy =
[a ] 3
(y ) 1 4
3
4 ( 2 − 1 ) 3/ 2 2
= ⋅ (2 − 1 ) 1
3 [a ]
1 2 3 4 5 n n+1
4 (5 − 2 − 2 2 ) 4 (5 − 3 2 )
= =
3 [a ] 3 [a ] From the above figure,
∴ Area is the greatest when [a ] is least, i.e. 1. = (n + 1 ) area of square ABCD = (n + 1 ) ⋅ 1
∴ Area is the greatest when [a ] = 1 Required area = (n + 1 ) sq units
Hence, a ∈[1, 2 )
#L Area of Bounded Regions Exercise 1 :
Single Option Correct Type Questions
1. A Point P( x , y ) moves such that [x + y + 1] = [x ]. (where 7. Let ‘a’ be a positive constant number. Consider two
[⋅] denotes greatest integer function) and x ∈(0, 2), then curves C 1 : y = e x , C 2 : y = e a − x . Let S be the area of the
the area represented by all the possible positions of P, is S
part surrounding by C 1 , C 2 and the Y -axis, then lim 2
(a) 2 (b) 2 2 a→ 0 a
(c) 4 2 (d) 2 equals
2. If f : [− 1, 1] → − , , f ( x ) =
1 1 x (a) 4 (b) 1/2
. The area bounded
 2 2 1+ x2 (c) 0 (d) 1/4

−1 1 1 8. 3 points O (0, 0), P (a, a 2 ), Q ( −b, b 2 )(a > 0, b > 0) are on the
by y = f ( x ), X-axis, x = , x = − is
2 2 parabola y = x 2 . Let S 1 be the area bounded by the line
1 e PQ and the parabola and let S 2 be the area of the ∆OPQ,
(a) loge (b) log  
2  2 the minimum value of S 1 S 2 is
1 e 1 e (a) 4/3 (b) 5/3
(c) log (d) log  
2 3 2  2 (c) 2 (d) 7/3
3. If the length of latusrectum of ellipse 9. Area enclosed by the graph of the function y = ln 2 x − 1
E 1 : 4( x + y − 1) 2 + 2 ( x − y + 3) 2 = 8 lying in the 4th quadrant is
2 4
x2 y2 (a) (b)
and E 2 : + 2 = 1, (0 < p < 1) are equal, then area of e e
p p  1  1
(c) 2 e +  (d) 4 e − 
ellipse E 2 , is  e  e
π π 3
(a)
2
(b)
2
10. The area bounded by y = 2 − | 2 − x | and y = is
|x|
π π
(c) (d) 4 + 3 ln 3 19
2 2 4 (a) (b) − 3 ln 2
2 8
4. The area of bounded by the curve 3
(c) + ln 3
1
(d) + ln 3
4 | x − 2017 2017 | + 5 | y − 2017 2017 | ≤ 20, is 2 2
(a) 60 (b) 50 11. Suppose g ( x ) = 2x + 1 and h( x ) = 4 x 2 + 4 x + 5 and
(c) 40 (d) 30 h( x ) = ( fog )( x ). The area enclosed by the graph of the
5. If the area bounded by the corve y = x 2 + 1, y = x and function y = f ( x ) and the pair of tangents drawn to it
the pair of lines x 2 + y 2 + 2xy − 4 x − 4y + 3 = 0 is K from the origin, is
(a) 8/3 (b) 16/3
units, then the area of the region bounded by the curve
(c) 32/3 (d) None of these
y = x 2 + 1, y = x − 1 and the pair of lines
12. The area bounded by the curves y = − − x and
( x + y − 1)( x + y − 3) = 0, is
(a) K (b) 2 K x = − −y where x , y ≤ 0
K (a) cannot be determined
(c) (d) None of these
2 (b) is 1/3
6. Suppose y = f ( x ) and y = g ( x ) are two functions whose (c) is 2/3
(d) is same as that of the figure bounded by the curves
graphs intersect at the three points (0, 4), (2, 2) and (4, 0) y = − x ; x ≤ 0 and x = −y ;y ≤ 0
with f ( x ) > g ( x ) for 0 < x < 2 and f ( x ) < g ( x ) for
2 < x < 4. If
4
13. y = f ( x ) is a function which satisfies
∫0 [ f ( x ) − g ( x )] dx = 10 (i) f ( 0 ) = 0 (ii) f ′′( x ) = f ′( x ) and (iii) f ′( 0 ) = 1
4
and ∫ [g ( x ) − f ( x )] dx = 5, then the area between two Then, the area bounded by the graph of y = f ( x ), the
2
curves for 0 < x < 2, is lines x = 0, x − 1 = 0 and y + 1 = 0, is
(a) 5 (b) 10 (a) e (b) e − 2
(c) 15 (d) 20 (c) e − 1 (d) e + 1
208 Textbook of Integral Calculus

14. Area of the region enclosed between the curves π2 π2


(a) −1 (b) −2
x = y 2 − 1 and x = | y | 1 − y 2 is 2 2
π 2
π 2
(a) 1 (b) 4/3 (c) −4 (d)
2 2
(c) 2/3 (d) 2
15. The area bounded by the curve y = x e − x ; xy = 0 and 21. If f ( x ) = x − 1 and g ( x ) = | f (| x | ) − 2|, then the area
x = c where c is the x-coordinate of the curve’s bounded by y = g ( x ) and the curve x 2 − 4y + 8 = 0 is
inflection point, is equal to
(a) 1 − 3e −2 (b) 1 − 2e −2 4 4
(a) ( 4 2 − 5 ) (b) ( 4 2 − 3 )
(c) 1 − e −2 (d) 1 3 3
8 8
16. If (a, 0); a > 0 is the point where the curve (c) ( 4 2 − 3 ) (d) ( 4 2 − 5 )
3 3
y = sin 2x − 3 sin x cuts the X -axis first, A is the area
 y(3x − 1) 
bounded by this part of the curve, the origin and the 22. Let S = ( x , y ): < 0 ,
positive X -axis, then  x (3x − 2) 
(a) 4 A + 8 cos a = 7 (b) 4 A + 8 sin a = 7 S ′ = {( x , y ) ∈ A × B : − 1 ≤ A ≤ 1,−1 ≤ B ≤ 1},
(c) 4 A − 8 sin a = 7 (d) 4 A − 8 cos a = 7 then the area of the region enclosed by all points in
17. The curve y = ax 2 + bx + c passes through the point (1, 2) S ∩ S ′ is
and its tangent at origin is the line y = x . The area (a) 1 (b) 2
bounded by the curve, the ordinate of the curve at (c) 3 (d) 4
minima and the tangent line is 23. The area of the region bounded between the curves
1 1 1 1
(a) (b) (c) (d) y = e || x | ln| x ||, x 2 + y 2 − 2(| x | + | y | ) + 1 ≥ 0 and X -axis
24 12 8 6
where | x | ≤ 1, if α is the x-coordinate of the point of
18. A function y = f ( x ) satisfies the differential equation
intersection of curves in 1st quadrant, is
dy
− y = cos x − sin x , with initial condition that y is α
(a) 4 ∫ ex ln x dx + ∫ (1 − 1 − ( x − 1 ) 2 ) dx 
1
dx
 0 α 
bounded when x → ∞. The area enclosed by
α α
y = f ( x ), y = cos x and the Y -axis in the 1st (b) 4 ∫ ex ln x dx + ∫ (1 − 1 − ( x − 1 ) 2 ) dx 
quadrant  0 1 
α
(a) 2 − 1 (c) 4 − ∫ ex ln x dx + ∫ (1 − 1 − ( x − 1 ) 2 ) dx 
1
(b) 2
(c) 1 (d) 1 / 2  0 α 
α α
19. If the area bounded between X -axis and the graph of (d) 2 ∫ ex ln x dx + ∫ (1 − 1 − ( x − 1 ) 2 ) dx 
 0 1 
y = 6x − 3x 2 between the ordinates x = 1 and x = a is 19
sq units, then ‘a’ can take the value 24. A point P lying inside the curve y = 2ax − x 2 is moving
(a) 4 or −2 such that its shortest distance from the curve at any
(b) two values are in (2, 3 ) and one in ( −1,0 ) position is greater than its distance from X -axis. The
(c) two values one in (3, 4 ) and one in ( −2, − 1 ) point P enclose a region whose area is equal to
(d) None of the above
πa 2 a2
−1 (a) (b)
20. Area bounded by y = f ( x ) and tangent and normal 2 3
drawn to it at the points with abscissae p and 2p, where 2a 2  3π − 4 2
(c) (d)  a
f ( x ) = sin x − x is 3  6 
Chap 03 Area of Bounded Regions 209

#L Area of Bounded Regions Exercise 2 :


More than One Option Correct Type Questions
25. The triangle formed by the normal to the curve (b) 1/6 sq units
f ( x ) = x 2 − ax + 2a at the point (2, 4) and the coordinate (c) 5/6 sq units
axes lies in second quadrant, if its area is 2 sq units, then (d) 9/2 sq units
a can be x2 y2
(a) 2 28. Area bounded by the ellipse + = 1 is equal to
4 9
(b) 17/4
(c) 5 (a) 6π sq units
(d) 19/4 (b) 3π sq units
(c) 12π sq units
26. Let f and g be continuous function on a ≤ x ≤ b and set
x2 y 2
p ( x ) = max { f ( x ), g ( x )} and q ( x ) = min{ f ( x), g( x)}, (d) area bounded by the ellipse + =1
9 4
then the area bounded by the curves y = p ( x ), y = q ( x )
and the ordinates x = a and x = b is given by 29. There is a curve in which the length of the perpendicular
b b from the origin to tangent at any point is equal to abscissa
(a) ∫ f ( x ) − g( x ) dx (b) ∫ p ( x ) − q ( x ) dx of that point. Then,
a a
(a) x 2 + y 2 = 2 is one such curve
b b
(c) ∫ { f ( x ) − g( x )} dx (d) ∫ { p ( x ) − q ( x )} dx (b) y 2 = 4 x is one such curve
a a
(c) x 2 + y 2 = 2cx (c parameters) are such curves
27. The area bounded by the parabola y = x 2 − 7 x + 10 and
(d) there are no such curves
X-axis equals
(a) area bounded by y = − x 2 + 7 x − 10 and X-axis

#L Area of Bounded Regions Exercise 3 :


Statement I and II Type Questions
n
Direction (Q. No. 30-34) For the following questions, 32. Statement I The area of region bounded parabola
choose the correct answers from the codes (a), (b), (c) and 32
(d) defined as follows : y 2 = 4 x and x 2 = 4y is sq units.
3
(a) Statement I is true, Statement II is also true; Statement II Statement II The area of region bounded by parabola
is the correct explanation of Statement I 16
y 2 = 4 ax and x 2 = 4 by is ab.
(b) Statement I is true, Statement II is also true; Statement II 3
is not the correct explanation of Statement I
33. Statement I The area by region | x + y | + | x − y | ≤ 2 is 8
(c) Statement I is true, Statement II is false
sq units.
(d) Statement I is false, Statement II is true
Statement II Area enclosed by region
30. Statement I The area of the curve y = sin 2 x from 0 to | x + y | + | x − y | ≤ 2 is symmetric about X -axis.
π will be more than that of the curve y = sin x from 0 to
34. Statement I Area bounded by y = x ( x − 1) and
π. 1
Statement II x 2 > x, if x > 1.
y = x (1 − x ) is .
3
31. Statement I The area bounded by the curves y = x 2 − 3 Statement II Area bounded by y = f ( x ) and y = g ( x ) is
and y = kx + 2 is least if k = 0. b
∫a ( f ( x ) − g ( x )) dx is true when f ( x ) and g ( x ) lies
Statement II The area bounded by the curves
above X -axis. (Where a and b are intersection of
y = x 2 − 3 and y = kx + 2 is k 2 + 20. y = f ( x ) and y = g ( x )).
210 Textbook of Integral Calculus

#L Area of Bounded Regions Exercise 4 :


Passage Based Questions
Passage I (c) f ( x ) is increasing on ( −1, 1 ) but has neither a local
(Q. Nos. 35 to 37) maximum nor a local minimum at x = 1
(d) f ( x ) is decreasing on ( −1, 1 ) but has neither a local
ax 2 + bx + c maximum nor a local minimum at x = 1
Let f ( x ) = such that y = −2 is an asymptote of the
x2 + 1 ex f ′(t )
curve y = f ( x ). The curve y = f ( x ) is symmetric about Y-axis 40. Let g ( x ) = ∫ dt . Which of the following is true?
0
1+t 2
and its maximum values is 4. Let h ( x ) = f ( x ) − g ( x ), where (a) g′( x ) is positive on ( − ∞, 0 ) and negative on ( 0, ∞ )
f ( x ) = sin 4 πx and g ( x ) = log e x. Let x 0 , x1 , x 2 , .... x n + 1 be the (b) g′( x ) is negative on ( − ∞, 0 ) and positive on ( 0, ∞ )
roots of f ( x ) = g ( x ) in increasing order. (c) g′( x ) change sign on both ( − ∞, 0 ) and ( 0, ∞ )
(d) g′( x ) does not change sign on ( − ∞, ∞ )
35. Then, the absolute area enclosed by y = f ( x ) and
y = g ( x ) is given by Passage III
n
xr
∑ ∫x
+1
(a) ( − 1 ) ⋅ h( x )dx
r (Q. Nos. 41 to 43)
r
r=0 Computing areas with parametrically represented boundaries :
n
xr
∑ ∫x r+1
+1
(b) (−1) ⋅ h( x )dx If the boundary of a figure is represented by parametric
r=0
r
equations i.e. x = x ( t ), y = y ( t ), then the area of the figure is
n
xr evaluated by one of the three formulas
∑ ∫x
_r + 1
(c) 2 ( − 1 )r ⋅ h( x )dx β
r=0
r
S = − ∫ y ( t ) ⋅ x ′ ( t ) dt
α
1 n xr
(d) ⋅ ∑ ∫x ( − 1 )r + 1h( x )dx
+1
β
2 r=0 r S = ∫ x ( t ) ⋅ y ′ ( t ) dt
α

36. In above inquestion the value of n, is 1 β


2 ∫α
(a) 1 (b) 2
S = ( xy ′ − yx ′ ) dt
(c) 3 (d) 4
where and are the values of the parameter ‘ t’
37. The whole area bounded by y = f ( x ), y = g ( x ) x = 0 is corresponding, respectively to the beginning and the end of the
(a) 11/8 (b) 8/3 traversal of the curve corresponding to increasing ‘ t’.
(c) 2 (d) 13/3
2/ 3 2/ 3
y 
41. The area enclosed by the asteroid  
x
Passage II +  = 1 is
a  a
(Q. Nos. 38 to 40)
3 3
Consider the function f :( − ∞ , ∞ ) → ( − ∞ , ∞ ) defined by (a) a 2π (b) πa 2
4 18
x 2 − ax + 1 3 2 3
f (x ) = 2 , 0 < a < 2. (c) πa (d) aπ
x + ax + 1 8 4
38. Which of the following is true? 42. The area of the region bounded by an arc of the cycloid
(a) (2 + a ) 2 f ′′(1 ) + (2 − a ) 2 f ′′( −1 ) = 0 x = a (t − sin t ), y = a (1 − cos t ) and the X-axis is
(b) (2 − a ) f ′′(1 ) − (2 + a ) f ′′( −1 ) = 0
2 2 (a) 6 πa 2 (b) 3 πa 2
(c) 4 πa 2 (d) None of these
(c) f ′(1 ) f ′( −1 ) = (2 − a ) 2
t t2
(d) f ′(1 ) f ′( −1 ) = − (2 + a ) 2 43. Area of the loop described as x = (6 − t ), y = (6 − t ) is
3 8
39. Which of the following is true? 27 24
(a) f ( x ) is decreasing on ( −1, 1 ) and has a local minimum at (a) (b)
5 5
x =1 27 21
(b) f ( x ) is increasing on ( −1,1 ) and has a local maximum at (c) (d)
6 5
x =1
Chap 03 Area of Bounded Regions 211

#L Area of Bounded Regions Exercise 5 :


Matching Type Questions
44. Match the statements of Column I with values of 45. Match the following :
Column II.
Column I Column II
Column I Column II (A) Area enclosed by y =| x |, | x | = 1 and (p) 2
(A) The area bounded by the curve (p) 2 y = 0 is
y = x + sin x and its inverse function (B) Area enclosed by the curve y = sin x , (q) 4
between the ordinates x = 0 to x = 2π is x = 0, x = π and y = 0 is
4s. Then, the value of s is (C) If the area of the region bounded by (r) 27
(B) The area bounded by y = x e|x| and lies (q) 1 k
x 2 ≤ y and y ≤ x + 2 is , then k is
| x | = 1, y = 0 is 4
equal to
(C) The area bounded by the curves y2 = x 3 (r)
16
(D) Area of the quadrilateral formed by (s) 18
and | y | = 2x is 5
tangents at the ends of latusrectum of
(D) The smaller are included between the 1 x 2 y2
(s) + = 1 is
curves | x | + | y | = 1and | x | + | y | = 1is 3 ellipse of ellipse
9 5

#L Area of Bounded Regions Exercise 6 :


Single Integer Answer Type Questions
46. Consider f ( x ) = x 2 − 3x + 2. The area bounded by polynomial of 2nd degree, satisfy the following
| y | = | f (| x | )|, x ≥ 1 is A, then find the value of 3A + 2 . condition :
(a) f ( 0 ) = 0
47. The value of c + 2 for which the area of the figure 1
(b) has a maximum value of at x = 1.
bounded by the curve y = 8x 2 − x 5 ; the straight lines 2
16 If A is the area bounded by y = f ( x ); y = f −1 ( x ) and the
x = 1 and x = c and X-axis is equal to , is ……… line 2x + 2y − 3 = 0 in 1st quadrant, then the value of 24A
3
is equal to ……… .
3 k − 3 ln 3
48. The area bounded by y = 2 − | 2 − x |; y = π
52. Let f ( x ) = min sin − 1 x , cos − 1 x ,  , x ∈ [0, 1]. If area
is ,
|x| 2
 6
then k is equal to ………
bounded by y = f ( x ) and X-axis, between the lines x = 0
49. The area of the ∆ABC, coordinates of whose vertices are a−X
A(2, 0), B(4, 5) and C(6, 3) is ……… and x = 1 is . Then, (a − b ) is ………… .
b( 3 + 1)
50. A point P moves in XY -plane in such a way that
53. Let f be a real valued function satisfying
[| x | ] + [| y | ] = 1, where [ ⋅ ] denotes the greatest integer
function. Area of the region representing all possible of x f (1 + x )
f   = f ( x ) − f (y ) and lim = 3. Find the area
the point P is equal to ………… . y x→ 0 x
bounded by the curve y = f ( x ), the Y -axis and the line
51. Let f : [0, 1] → 0,  be a function such that f ( x ) is a
1
 2 y = 3, where x , y ∈ R + .
212 Textbook of Integral Calculus

Area of Bounded Regions Exercise 7 :


Subjective Type Questions
54. Find a continuous function ‘ f ’, Y
( x 4 − 4 x 2 ) ≤ f ( x ) ≤ (2x 2 − x 3 ) such that the area
bounded by y = f ( x ), y = x 4 − 4 x 2 , the Y -axis and the A1 A
A2
y = sin x
line x = t , (0 ≤ t ≤ 2) is k times the area bounded by
X
y = f ( x ), y = 2 x 2 − x 3 , Y -axis and line x = t ,(0 ≤ t ≤ 2). O a p 2p

55. Let f (t ) = | t − 1 | − | t | + | t + 1 |, ∀ t ∈ R and A3


g ( x ) = max { f (t ) : x + 1≤ t ≤ x + 2}; ∀ x ∈ R. Find g ( x ) y = f(x)
and the area bounded by the curve y = g ( x ), the X-axis
and the lines x = − 3 / 2 and x = 5. between x = a and x = π; i = 2 between x = π and x = 2 π; i = 3.
−x
56. Let f ( x ) = minimum { e , 3 / 2, 1 + e
x
}, 0 ≤ x ≤ 1. Find the If A 1 = 1 − sin a + (a − 1) cos a, determine the function
area bounded by y = f ( x ), X -axis,Y -axis and the line f ( x ). Hence, determine a and A 1 . Also, calculate A 2 and A 3 .
x = 1. 62. Find the area of the region bounded by curvey = 25 x + 16
57. Find the area bounded by y = f ( x ) and the curve and the curvey = b . 5 x + 4, whose tangent at the point
2 x = 1make an angle tan − 1 ( 40 ln 5) with the X -axis.
y= , where f is a continuous function satisfying
1+ x 2
63. If the circles of the maximum area inscribed in the
the conditions f ( x ) ⋅ f (y ) = f ( xy ), ∀ x , y ∈ R region bounded by the curves y = x 2 − x − 3 and
and f ′ (1) = 2, f (1) = 1. y = 3 + 2x − x 2 , then the area of region
58. Find out the area bounded by the curve y − x 2 + 2x + 3 ≤ 0, y + x 2 − 2x − 3 ≤ 0 and s ≤ 0.
sin 2 x cos 2 x
y=∫ (sin −1 t ) dt +∫ (cos − 1 t ) dt (0 ≤ x ≤ π / 2) 64. Find limit of the ratio of the area of the triangle formed
1/ 8 1/ 8
by the origin and intersection points of the parabola
and the curve satisfying the differential equation
y = 4 x 2 and the line y = a 2 , to the area between the
y ( x + y 3 ) dx = x (y 3 − x ) dy passing through ( 4, − 2).
parabola and the line as a approaches to zero.
59. Let T be an acute triangle. Inscribe a pair R, S of 65. Find the area of curve enclosed by :
rectangles in T as shown : | x + y | + | x − y | ≤ 4, | x | ≤ 1, y ≥ x 2 − 2 x + 1.
66. Calculate the area enclosed by the curve
4 ≤ x 2 + y 2 ≤ 2 (| x | + | y | ).
S 67. Find the area enclosed by the curve [x ] + [y ] = 4 in 1st
R quadrant (where [.] denotes greatest integer function).
68. Sketch the region and find the area bounded by the
Let A ( x ) denote the area of polygon X find the curves | y + x | ≤ 1, | y − x | ≤ 1 and 2x 2 + 2y 2 = 1.
maximum value (or show that no maximum exists), of
A ( R ) + A (S ) 69. Find the area of the region bounded by the curve,
, where T ranges over all triangles and R, S 2| x | | y | + 2| x | − 1 ≤ 1, with in the square formed by the
A (T )
over all rectangles as above. lines | x | ≤ 1 / 2, | y | ≤ 1 / 2.

60. Find the maximum area of the ellipse that can be 70. Find all the values of the parameter a (a ≤ 1) for which the
inscribed in an isosceles triangles of area A and having area of the figure bounded by the pair of straight lines
1
one axis lying along the perpendicular from the vertex y 2 − 3y + 2 = 0 and the curves y = [a ] x 2 , y = [a ] x 2 is
of the triangles to its base. 2
the greatest, where [.] denotes greatest integer function.
61. In the adjacent figure the graphs of two function 71. If f ( x ) is positive for all positive values of X and
y = f ( x ) and y = sin x are given. y = sin x intersects,
y = f ( x ) at A (a, f (a )); B ( π, 0) and C (2π, 0). f ′ ( x ) < 0, f ′′ ( x ) > 0, ∀ x ∈ R + , prove that
1 n n n
A i (i = 1, 2, 3) is the area bounded by the curves y = f ( x ) f (1) + ∫ f ( x ) dx < Σ f (r ) < ∫ f ( x ) dx + f (1).
2 1 1
and y = sin x . between x = 0 and x = a; i = 1 r =1
Chap 03 Area of Bounded Regions 213

#L Area of Bounded Regions Exercise 8 :


Questions Asked in Previous 10 Years’ Exams
(i) JEE Advanced & IIT-JEE (a)
3
(b)
1
(c)
1
(d)
1
4 2 3 4
72. Area of the region {( x , y )} ∈ R : y ≥ | x + 3|, 2
79. Area of the region bounded by the curve y = e x and
5y ≤ ( x + 9 ) ≤ 15} is equal to [Single Correct Option 2016]
lines x = 0 and y = e is [More than One Option Correct 2009]
1 4 3 5 e
(a) (b) (c) (d)
6 3 2 3 (a) e − 1 (b) ∫1 ln(e + 1 − y )dy
π 1 e
73. Let F ( x ) = ∫
x2 +
6 2 cos 2 t dt for all x ∈ R and (c) e − ∫ e x dx (d) ∫1 ln y dy
0
x
 1 80. The area of the region between the curves
f : 0,  → [0, ∞ ) be a continuous function. For
 2 1 + sin x 1 − sin x
 1 y= and y = and bounded by the
a ∈ 0, , if F ′ (a ) + 2 is the area of the region bounded cos x cos x
 2 π
by x = 0 , y = 0 , y = f ( x ) and x = a, then f (0) is lines x = 0 and x = is
4 [Single Correct Option 2008]
[Integer Answer Type 2015] 2 −1 t 2 −1 4t
(a) ∫ dt (b) ∫ dt
74. The common tangents to the circle x 2 + y 2 = 2 and the 0
(1 + t ) 1 − t
2 2 0
(1 + t ) 1 − t 2
2

parabola y 2 = 8x touch the circle at the points P,Q and 2+1 4t 2+1 t
(c) ∫ dt (d) ∫ dt
the parabola at the points R, S. Then, the area 0
(1 + t 2 ) 1 − t 2 0
(1 + t 2 ) 1 − t 2
(in sq units) of the quadrilateral PQRS is
[Single Correct Option 2014] n
Directions (Q. Nos. 81 to 83) Consider the functions
(a) 3 (b) 6 (c) 9 (d) 15 defined implicity by the equation y3 − 3 y + x = 0 on
75. The area enclosed by the curves y = sin x + cos x and various intervals in the real line. If x ∈ ( −∞, − 2) ∪ (2, ∞ ),
the equation implicitly defines a unique real-valued
 π
y = | cos x − sin x | over the interval 0,  is differentiable function y = f ( x ). If x ∈ ( − 2, 2), the equation
 2 implicitly defines a unique real-valued differentiable
[Single Correct Option 2014] function y = g ( x ), satisfying g (0) = 0.
(a) 4( 2 − 1 ) (b) 2 2( 2 − 1 ) [Passage Based Questions 2008]
(c) 2 ( 2 + 1 ) (d) 2 2( 2 + 1 )
81. If f ( − 10 2 ) = 2 2, then f ′′ ( − 10 2 ) is equal to
76. If S be the area of the region enclosed by 4 2 4 2
(a) (b) −
y = e − x , y = 0, x = 0 and x = 1. Then,
2
7 33 2 7 33 2
[More than One Option Correct 2012] 4 2 4 2
1 1 (c) 3 (d) − 3
(a) S ≥ (b) S ≥ 1 − 73 73
e e
1  1  1 1  1  82. The area of the region bounded by the curve y = f ( x ),
(c) S ≤ 1 +  (d) S ≤ + 1 − 
4 e 2 e  2 the X -axis and the lines x = a and x = b, where
− ∞ < a < b < − 2, is
77. Letf : [−1, 2] → [0, ∞ ) be a continuous function such that b x
2 (a) ∫ dx + bf (b ) − af (a )
f ( x ) = f (1 − x ), ∀x ∈ [−1, 2]. If R1 = ∫ x f ( x ) dx and R 2 a 3 [{ f ( x )} 2 − 1]
−1
b x
are the area of the region bounded by y = f ( x ), (b) − ∫ dx + bf (b ) − af (a )
a 3 [{ f ( x )} 2 − 1]
x = − 1, x = 2 and the X-axis. Then,
b x
[Single Correct Option 2011] (c) ∫ dx − bf (b ) + af (a )
(a) R1 = 2 R2 (b) R1 = 3 R2 (c) 2 R1 = R2 (d) 3 R1 = R2 a 3 [{ f ( x )} 2 − 1]
78. If the straight line x = b divide the area enclosed by b x
(d) − ∫ dx − bf (b ) + af (a )
y = (1 − x ) , y = 0 and x = 0 into two parts R1 (0 ≤ x ≤ b )
2 a 3 [{ f ( x )} 2 − 1]
1 1
and R 2 (b ≤ x ≤ 1) such that R1 − R 2 = . Then, b equals
4
83. ∫− 1 g ′ ( x ) dx is equal to
to [Single Correct Option 2011] (a) 2 g( − 1 ) (b) 0 (c) − 2 g(1 ) (d) 2 g(1 )
214 Textbook of Integral Calculus

(ii) JEE Main & AIEEE


84. The area (in sq. units) of the region [2017 JEE Main] (a) 9 (b) 36
27
{( x , y ) : x ≥ 0, x + y ≤ 3, x ≤ 4y } and y ≤ 1 + x } is
2
(c) 18 (d)
4
5 59 3 7
(a) (b) (c) (d) y
2 12 2 3 90. The area bounded between the parabolas x 2 = and
4
85. The area (in sq units) of the region {( x , y ) : y 2 ≥ 2x and x 2 = 9y and the straight line y = 2 is [2012 AIEEE]
x 2 + y 2 ≤ 4 x , x ≥ 0, y ≥ 0} is [2016 JEE Main] 10 2
4 8 (a) 20 2 (b)
(a) π − (b) π − 3
3 3 20 2
4 2 π 2 2 (c) (d) 10 2
(c) π − (d) − 3
3 2 3
91. The area of the region enclosed by the curves y = x ,
86. The area (in sq units) of the region described by 1
{( x , y ) : y 2 ≤ 2x and y ≥ 4 x − 1} is [2015 JEE Main]
x = e, y = and the positive X -axis is
x [2011 AIEEE]
7 5 3
(a) (b) (a) 1 sq unit (b) sq units
32 64 2
15 9 5 1
(c) (d) (c) sq units (d) sq unit
64 32 2 2
87. The area (in sq units) of the quadrilateral formed by the 92. The area bounded by the curves y = cos x and y = sin x
tangents at the end points of the latusrectum to the 3π
2 between the ordinates x = 0 and x = is
x2 y 2
ellipse + = 1 is [2010 AIEEE]
9 5 [2015 JEE Main] (a) ( 4 2 − 2 ) sq units (b) ( 4 2 + 2 ) sq units
27 (c) ( 4 2 − 1 ) sq units (d) ( 4 2 + 1 ) sq units
(a) (b) 18
4
27 93. The area of the region bounded by the parabola
(c) (d) 27
2 (y − 2) 2 = x − 1, the tangent to the parabola at the point
88. The area of the region described by (2, 3) and the X-axis is [2009 AIEEE]
(a) 6 sq units (b) 9 sq units
A = {( x , y ) : x 2 + y 2 ≤ 1 and y 2 ≤ 1 − x } is [2014 JEE Main]
(c) 12 sq units (d) 3 sq units
π 4 π 4
(a) + (b) − 94. The area of the plane region bounded by the curves
2 3 2 3
π 2 π 2 x + 2y 2 = 0 and x + 3y 2 = 1 is equal to [2008 AIEEE]
(c) − (d) +
2 3 2 3 5 1
(a) sq units (b) sq unit
89. The area (in sq units) bounded by the curves y = x , 3 3
2 4
2y − x + 3 = 0, X-axis and lying in the first quadrant is (c) sq unit (d) sq units
[2013 JEE Main]
3 3
Answers
Exercise for Session 1  − x − 1, x≤ − 5/ 2
436 4  4 + x, − 5 / 2 < x ≤ − 2
1. 1 sq unit 2. sq units 3. sq units 
15 3 55. g (x) =  2, − 2< x≤ − 1
4 28 56 2  1 − x, − 1 < x ≤ − 1 / 2
4. sq units 5. sq units 6. a sq units 
3 3 3  1 + x, x > − 1/ 2
8 28 101
7. a2 sq units 8. 2z sq units 9. sq units and area = sq units
3 3 4
 4  1 
10. 3+16 log 2 sq units 56.  2 + log   − sq units
  3 3  e 
4
1  3π 
57.  π −  sq units
Exercise for Session 2 2
58.   sq units
1. (d) 2. (a) 3. (c) 4. (d) 5. (a)  3  8  16 
2
6. (b) 7. (a) 8. (a) 9. (c) 10. (b) 59. Required maximum ratio =
11. (d) 12. (b) 13. (a) 14. (c) 15. (b) 3
3 πA
16. (a) 17. (c) 18. (a) 19. (a) 20. (b) 60. (A )max = sq units
9
61. A1 = 1 − sin 1, A2 = π − 1 − sin 1, A3 = 3 π − 2
Chapter Exercises
 e4 
1. (d) 2. (b) 3. (b) 4. (c) 5. (b) 62. 4 log5   sq units
6. (c) 7. (d) 8. (a) 9. (b) 10. (b)  27 
4  − 4 π  sq units
11. (b) 12. (b) 13. (c) 14. (d) 15. (a) 16
63.
 3 
16. (a) 17. (a) 18. (a) 19. (c) 20. (b)
3
21. (a) 22. (b) 23. (d) 24. (c) 25. (b, c) 64. 65. 2 sq units
2
26. (a, b, d) 27. (a, d) 28. (a, d) 29. (a, c)
66. 8 sq units 67. 5 sq units
30. (d) 31. (c) 32. (d) 33. (b) 34. (c)  2 − π  sq units
68.  
35. (a) 36. (b) 37. (a) 38. (a) 39. (a)  2 
40. (b) 41. (c) 42. (b) 43. (a)  4 − 1/ 2 
44. (A) → (p); (B) → (p); (C) → (r); (D) → (s) 69.  log 2 (1 − 2 ) − 1  sq units
 
45. (A) → (p); (B) → (p); (C) → (r); (D) → (r)
70. a ∈[1, 2) 72. (c) 73. (3) 74. (d) 75. (b)
46. (7) 47. (1) 48. (4) 49. (7)
76. (b,d) 77. (c) 78. (b) 79. (b,c,d) 80. (b) 81. (b)
50. (8) 51. (5) 52. (3) 82. (a) 83. (d) 84. (a) 85. (b) 86. (d) 87. (d)
1
53. 3 e sq units 54. f (x) = [ x4 − kx3 + (2k − 4)x2 ] 88. (a) 89. (a) 90. (c) 91. (b) 92. (a) 93. (b)
k+1 94. (d)
x2 y2
∴ E2 : + =1
2 −1/ 3 4 −1/ 3

Solutions
Area of ellipse E 2, is
π
π ⋅ p ⋅ p = π p 3/ 2 =
2
4. Area of bounded region by
4 | x − 2017 2017| + 5 | y − 2017 2017| ≤ 20, is same as area of the
1. Here, [ x + y ] = [ x ] − 1 region bounded by 4 | x | + 5 | y | ≤ 20.
1
when x ∈[ 0, 1 ) ⇒[ x + y ] = − 1 ⇒ 4 × × 4 × 5 = 40
2
−1 ≤ x + y < 0 …(i) Y
when x ∈[1, 2 ) ⇒ [ x + y ] = 0 4
∴ 0 ≤ x + y <1 …(ii)
which can be shown, as
O
Y x=1 x=2 X
−5 5

−1 0 1 2 X
5. Here, y = x 2 + 1 and y = x − 1 are inverse of each other.
The shaded area is given K units
x+y=1 ⇒ Area of the region bounded by y = x 2 + 1, y = x − 1 and
( x + y − 1 ) ( x + y − 3 ) = 0, is 2K units.
x+y=0 y = x2 + 1
Y
x+y=−1 y=x
∴ Required area = 2
1/ 2
2. Required area = 2 ∫ f − 1 ( x ) dx
0
K y = √x
Let, f −1 ( x ) = t ⇒ x = f (t ) 1

dx = f ′(t )dt
1 X
∴ A = 2 ∫ (t ⋅ f ′ (t )dt 0 1 3
0 x+y=3
 
1
1  x+y=1
= 2 t ⋅ f (t ) − ∫ 1 ⋅ f (t )dt 
 0 0

 1 t  4 4
6. Given, ∫ f ( x ) dx − ∫ g ( x ) dx = 10
= 2  f (1 ) − ∫ dt  0 0
0 1 + t2
 
( A1 + A3 + A4 ) − ( A2 + A3 + A4 ) = 10
 1  
1
= 2  f (1 ) −  log (1 + t 2 )   A1 − A2 = 10 ...(i)
 2  0
 Y
 1  y = f(x)
= 2 f (1 ) − log 2
 2  (0,4)
A1
1 1  e
=2 − log 2 = 1 − log 2 = log   (2,2) y = g(x)
2 2   2 2
2 2
 x + y − 1  x − y + 3 A3 A2
   
 2   2  A4
3. Here, E1 : + =1 O 2
X
12 ( 2 )2 (4,0)
4 4
a2 2 Again, ∫ g( x ) dx − ∫ f ( x ) dx = 5
Length of latusrectum = 2 = = 2 2 2
b 2
( A2 + A4 ) − A4 = 5
2p 2
Now, = 2 ⇒ p 3/2 = 2 −1//2 A2 = 5 ...(ii)
p
Adding Eqs. (i) and (ii),
⇒ P = 2 −1/ 3 A1 = 15
Chap 03 Area of Bounded Regions 217

7. Solving, e x = ea − x , we get e 2x = ea ⇒ x =
a x > 1, y increasing and 0 < x < 1, y is decreasing
2 e
A= ∫1/e (ln x − 1 ) dx
2
x
Y y=e
e e
= ∫1/e ln x dx − ∫ dx
2
a 1/e
(0,e )
e  ln x   1
= [ x ln 2 x ]e1/e − 2 ∫   ⋅ x dx − e − 
y=e a–x 1/e  x   e
(0,1)
 1 e  ln x   1
X = e −  − 2∫   ⋅ x dx − e − 
O  e 1/e  x   e
a /2 a
S =∫ (e ⋅ e − x − e x ) dx = [ −(ea ⋅ e − x + e x )] a0/2
= − 2 [ x ln x ]e1/e − ∫ dx 
e
0
 1/e 
= (ea + 1 ) − (ea /2 + ea /2 ) = ea − 2ea /2 + 1 = (ea /2 − 1 ) 2
 1  1  4 4
2 2 = − 2 e +  − e −   = =
S  ea / 2 − 1  1  ea / 2 − 1    e   e   e e
∴ =  =  
a 2
 a  4 a /2  2 − (2 − x ), if x ≤ 2 = x x, if x ≤ 2
10. y = =
∴ lim
S
=
1  2 − ( x − 2 ), if x ≥ 2 = 4 − x 4 − x, if x ≥ 2
a→ 0 a 2 4  3
, if x > 0
a2 −b2 
8. mPQ = = a − b equation of PQ Also, y =  x
3
a +b − , if x < 0
Q(–b,b2)  x
Y

3
P(a,a2)
2
O 3/2
1
a −b 2 2
y −a2 = ( x − a ) or y − a 2 = (a − b )( x − a ) X
a +b O 1 3/2 2 3
y = a 2 + x(a − b ) − a 2 + ab y=x y = 4–x

y = (a − b ) x + ab 2  3 3 3
A=∫  x −  dx + ∫ ( 4 − x ) −  dx
3/ 2  x 2 x
a
∴ S1 = ∫ ((a − b ) x + ab − x ) dx 2
−b
2 3
(a + b ) 3 x2   x2 
which simplifies to . ...(i) =  − 3 ln x  + 4 x − − 3 ln x 
6 2  3/ 2  2 2
a a2 1  9 3   9 
1 1 1 = 2 − 3 ln 2 −  − 3 ln   + 12 − − 3 ln 3 − (8 − 2 − ln 2 )
Also, S 2 = −b b 2 1 = [ab 2 + a 2b ] = ab (a + b ) ...(ii)   8 2   
 2 
2 2 2
0 0 1 7 3 19
= − 3 ln 2 + 3 ln 3 − 3 ln 2 + − 3 ln 3 + 3 ln 2 = − 3 ln 2
S1 (a + b ) 3 (a + b ) 2 1 a b 8 2 8
2 
∴ = ⋅ = = + +2 11. Given, g( x ) = 2x + 1; h( x ) = (2x + 1) 2 + 4
S2 6 ab (a + b ) 3ab 3 b a 
S1 4 Now, h( x ) = f [ g( x ) ]
∴ = (2 x + 1 ) 2 + 4 = f (2 x + 1 )
S2 min
3
2 ln x Y
9. y = ln x − 1 ⇒ y ′ =
2
=0 ⇒ x =1
x
Y
(0,4) P(2,8)
y = mx
I X X
O I/e e O
–1
218 Textbook of Integral Calculus

Let 2x + 1 = t ⇒ f (t ) = t 2 + 4 y ′′ = −e x − [e − x − xe − x ] = e − x [ −1 − 1 + x ] = ( x − 2 )e − x
∴ f (x ) = x 2 + 4 ...(i)
Solving, y = mx and y = x + 4 2

x 2 − mx + 4 = 0 X
x=2
Put D = 0 ; m 2 = 16 ⇒ m = ±4
Tangents are y = 4 x and y = −4 x
2
A = 2 ∫ [( x 2 + 4 ) − 4 x ] dx = 2 ∫ ( x − 2 ) 2dx
2 For point of inflection y ′′ = 0 ⇒ x =2
0 0 2 −x 2 –x
2 A = ∫ xe dx = [ − xe − x ] 20 + ∫0 e = ( −2e −2 ) − (e − x ) 20
2  16 0
= ( x − 2 ) 3 = sq units
3  3 = −2e −2 − (e −2 − 1 ) = 1 − e −2 − 2e −2 = 1 − 3e −2
0

12. y = − −x ⇒ y = − x, where x and y both negative


2
16. (a, 0) lies on the given curve
x = − −y ⇒ x 2 = −y , where x and y both negative ∴ 0 = sin 2a − 3 sin a ⇒ sin a = 0 or cosa = 3 / 2
Y π
⇒ a= (as a > 0 and the first point of intersection
6
with positive X -axis)
π /6
O π /6  cos 2 x 
X and A = ∫ (sin 2 x − 3 sin x ) dx =  − + 3 cos x
x= –√–y 0  2 0
 1 3  1  7 7
=  − +  −  − + 3 = − 3 = − 2 cos a
 4 2  2  4 4
y = –√–x ⇒ 4 A + 8 cos a = 7
16 ab 1 17. x = 1 ; y = 2
Since, A= where, a = b =
3 4 2 =a +b +c ...(i)
∴ A=
1 x = 0, y = 0 ⇒ c = 0 ⇒ a + b = 2
3 Y
f ′′( x )
13. = 1.
f ′( x )
Integrating, ln f ′( x ) = x + C , f ′( 0 ) = 1 ⇒ C = 0
– 1_
Y 2 X
–1 0

O
X
dy
Now, = 2a x + b = 1
y+1=0 dx ( 0, 0 )
–1
∴ b = 1, a = 1
Hence, the curve is y = x 2 + x
f ′( x ) = e f ( x ) = e + k, f ( 0 ) = 0 ⇒ k = −1
x x
0 0 1
f (x ) = e x − 1 A = ∫ 1 ( x 2 + x − x ) dx = ∫ 1 ( x 2 ) dx = sq units
− − 24
1 2 2
Area = ∫ (e x − 1 + 1 ) dx = [e x ]10 = e − 1 −x
0 18. IF = e
1
14. A = 2∫ [y 1 − y − (y − 1)] dy = 2
2 2 ∴ ye − x = ∫ e − x (cos x − sin x ) dx. Put − x = t
0
Y = − ∫ et (cos t + sin t ) dt = − et sin t + C
1 y e − x = e − x sin x + C
x =y √1–y2
Y
X
(–1,0) O

–1 x = y2–1
X
O
15. y = xe −x
y ′ = e − x − x e − x = (1 − x ) e −x increasing for x < 1 Since, y is bounded when x → ∞ ⇒ C = 0
Chap 03 Area of Bounded Regions 219

∴ y = sin x −2+ 2 2   x 2 + 8 
π /4 ∴ Required area = 2 ∫ ( − x + 3 ) −    dx
Area = ∫ (cos x − sin x ) dx = 2 − 1 0
  4 
0
−2+ 2 2
6x 2 3x 3 2 −2+ 2 2 1 x3 
19. I = ∫ (6x − 3x ) dx = 2
− = 3 x 2 − x 3 = x 2(3 − x ) = ∫ ( 4 − 4 x − x ) dx = 4 x − 2 x 2 − 
2
2 3 4 0 2 3 0
A1 = I (2 ) − I (1 ) = 4 − 2 = 2 units 1
A2 = I (2 ) − I (3 ) = 4 − 0 = 4 units = [12 x − 6 x 2 − x 3 ]0−2+ 2 2
6
A3 = I (3 ) − I ( 4 ) = 0 − ( −16 ) = 16 units 1
= [12( − 2 + 2 2 ) − 6( − 2 + 2 2 ) 2 − ( −2 + 2 2 ) 3 ]
Y 6
1
= [ − 24 + 24 2 − 6( 4 + 8 − 8 2 ) − ( −8 + 16 2 ) + 24 2 − 48 ]
6
A1 1
3 4 X = [ − 24 + 24 2 − 72 + 48 2 + 56 − 40 2 ]
–1 0 1 2 A2 6
A3
1 8 4
= [32 2 − 40 ] = ( 4 2 − 5 ) = ( 4 2 − 5 )
O 6 6 3
22. Shaded region represents S ∩ S ′ clearly area enclosed is 2 sq
units.
⇒ One value of a will lie in (3, 4). (0,1)
Using symmetry, other will lie in (–2, –1).

20. Required area, A = ∫ ((sin x − x ) + 2 π ) dx
π

π2
O
(–1,0) 1/3 2/3 (1,0)
= − 2 sq units
2
21. g ( x ) = | f (| x | ) − 2 | = | | x | − 1 − 2 | = | | x | − 3 |
(0, –1)
 − x − 3, x < −3
 ( | x | − 3 ), x < −3 − ( − x − 3 ), − 3 ≤ x < 0 α α

= − (| x | − 3 ), − 3 ≤ x < 3 = 
23. Required area is 2 ∫ ex ln x dx + ∫ (1 − 1 − ( x − 1) 2 ) dx 
 0 1 
 | x | −3 x ≥3  − ( x − 3 ), 0 ≤ x <3
  x − 3, x ≥3
 − x − 3, x < −3
 x + 3, − 3 ≤ x < 0
=
− x + 3, 0 ≤ x < 3
 x − 3, x ≥3 1

Now, x − 4y + 8 = 0
2

–1 –1
⇒ x 2 = 4(y − 2 )
Y x2=4(y–2)
y=–x–3 y=x–3 24. y = 2ax − x 2 ⇒ ( x − a ) 2 + y 2 = a 2
3
y=x+3 Let P (h, k ) be a point, then BP > PN
2 y=–x+3 For the bounded condition BP = PN = k
1
X¢ X Now, AP = a − k = (h − a ) 2 + k 2
–3 –2 –1 1 2 3
h2
⇒ k =h−
(–2+2Ö2, 0) 2a

For point of intersection,
x 2 − 4( − x + 3 ) + 8 = 0
⇒ x 2 + 4x − 4 = 0 B
− 4 ± 16 + 16 P
⇒ x= = −2 ± 2 2
2
∴ Point of intersection is at x = − 2 + 2 2 N C(a,0)
220 Textbook of Integral Calculus

x2   dy  2   dy 
2
dy
∴ Boundary of the region is y = x − x 2 1 +    = y 2 + x 2   = 2 xy
2a  dx   dx  dx
 
a  x2 2a 2
Required area = 2 ∫  x −  dx = dy x2 −y 2
0  2a  3 ⇒ =− [homogeneous form]
dx 2 xy
25. f ′( x ) = 2x − a. At (2, 4), f ′( x ) = 4 − a x2 + y 2 
 2 −x
2
1
Equation of normal at (2, 4) is (y − 4 ) = − (x − 2) dy 1 − x x − x 
2
 y 2 − x2
(4 − a ) = = = =
dx y xy xy 2 xy
Let point of intersection with X and Y-axes be A and B
respectively, then Since, option (a) is true (equation of circle).
 4a − 18 If option (a) is true (b) can’t be (It is parabola).
A ≡ ( − 4a + 18, 0 ) B ≡  0, 
 a −4  If option (a) is true, option (c) is also true where c = 1.
9
Since, a > as 30. ∴ sin 2 x ≤ sin x , ∀ x ∈ ( 0, π )
2 Therefore, area of y = sin 2x will be lesser from area of y = sin x.
1 ( 4a − 18 )
∴ Area of triangle = ( 4a − 18 ) =2 Statement II is obviously true.
2 (a − 4 )
Hence, (d) is the correct answer.
⇒ ( 4a − 17 ) (a − 5 ) = 0
17 31. Let the line y = kx + 2 cuts y = x 2 − 3 at x = α and α = β, area
⇒ a = 5 or β β
4 bounded by the curves = ∫ (y1 − y 2 ) = ∫ {(kx + 2 ) − ( x 2 − 3 )} dx
α α
1
26. Max { f ( x ), g ( x )} = [| f ( x ) + g ( x )| + | f ( x ) − g ( x )| ] (k 2 + 20 ) 3/2
2 ⇒ f (x ) =
1 6
Min { f ( x ), g ( x )} = [| f ( x ) + g ( x )| − | f ( x ) − g ( x )| ] which clearly, shows the Statement II is false but f (k ) is least
2
b when k = 0.
∴ Area = ∫ [max { f ( x ), g( x )} − min { f ( x ), g( x )}] dx Hence, (c) is the correct answer.
a

27. Area bounded by parabola y = x 2 − 7x + 10 and X -axis is given 32. As of region bounded by parabola y 2 = 4x and x 2 = 4y is
4
by 4 x2 4 x3  32 16 16
5 5 9 ∫0 2 x −  dx =  x 3/2 −  = − = sq units
∫ 2| x − 7 x + 10| dx = ∫ | − x 2 + 7 x − 10| dx =  4
2
sq units  3 12 0 3 3 3
2 2
x2 y 2 Hence, Statement I is false.
28. Area bounded by the ellipse + = 1 will be the same as 33. As the area enclosed by | x | + | y | ≤ a is the area of square
a2 b2
x2 y 2 ( i. e., 2a 2 ).
the area bounded by the ellipse 2 + 2 = 1 and π ab
b a ∴Area enclosed by| x + y | + | x − y | ≤ 2 is area of square shown as
∴ Required area = π (2 )(3 ) = 6 π sq units y y=x
29. OP = x
dy 2 2
If slope is , then equation of tangent is
dx
dy X
Y −t = (X − x )
dx

y = –x
P(x,y)
1 
∴ Area = 4  × 2 × 2 = 8 sq units
P 2 
Also, the area enclosed by | x + y | + | x − y | ≤ 2 is symmetric
about X -axis, Y -axis, y = x and y = − x.
O ∴Both the Statements are true but Statement II is not the
correct explanation of Statement I.
Length of perpendicular from origin to this tangent is b

dy
34. ∫ a { f (x ) − g(x )} dx is true for all quadrants.
Y −x
x= dx ∴ Statement II is false.
2
 dy  The area bounded by y = x ( x − 1 ) and y = x (1 − x ).
1+  
 x
Chap 03 Area of Bounded Regions 221

Y f ′(e x ) e x
y = x(x–1)
40. Q g′( x ) =
1 + e 2x
2a ⋅ (e 2x − 1 )
Now, f ′(e x ) = 2x
0 1
X (e + ae x + 1 ) 2
It is seen from the above that f ′(e x ) and so g′( x ) is positive on
y =x (1–x) ( 0, ∞ ) and negative on ( − ∞, 0 ).
1 41. Clearly, x = a sin 3 t, y = a cos3 t, ( 0 ≤ t ≤ 2π )
1 x 2 x 3 
∴ Area enclosed = 2 ∫ x (1 − x ) dx = 2  −  S =∫

a sin 3 t ⋅ a ⋅ 3 sin 2 t ( − sin t ) dt
0
2 3 0 0
 1 1  3 − 2 1 π /2
= 2 −  = 2  = = −3a 2 × ∫ sin 4t cos2 t dt
 2 3  6  3 0

35. Since, absolute area 5 3 3 1 1


x1 x2 x3 × π× π
=∫ h( x ) dx + ∫x − h( x ) dx + ∫x h( x ) dx = −3a ×
2 2 2
= −3a 22 2 2
x0 1 2
4+2+2 2 ×3 ×2 ×1
n
xr
2
∑ ∫x
+1
= ( − 1 )r ⋅ h( x ) dx 2
r
r=0 3 3
= − πa 2 = πa 2 [absolute value]
36. Also, xn + 1 = x 3 ⇒n = 2 8 8

37. Required area = ∫ sin 4 πx dx −
1 1 11 42. S = − ∫ a (1 − cost ) a (1 − cost ) dt
0 ∫0 loge x dx =
8
0

Y = −a 2 ∫ (1 − 2 cos t + cos2 t ) dt
0
2π   1 + cos2 t  
= −a 2 ∫ 1 − 2 cos t +    dt
0  2 
a 2 2π
[3 − 4 cos t + cos2 t ]20π
2 ∫0
=−

X = − 3 πa 2 = 3 πa 2 [absolute value]
0 1x0 3/2 x1 2 x2 x3 3
1/2 2
t 1 t 1
43. x = (6 − t ) = (6t − t 2 ), y = (6 − t ) = (6t 2 − t 3 )
3 3 8 8
x 2 − ax + 1
38. f ( x ) = ∴ Area = ∫
6t 1
(6 − t ) ⋅ (12t − 13t 2 )dt
x 2 + ax + 1 03 8
For differentiation, better write f ( x ) as 6
1 6 4 1 t 5 5 

2 ax = (t + 24t 2 − 10t 3 ) dt =  + 8t 3 − t 4 
f (x ) = 1 − 2
x + ax + 1 8 0 8 5 2 0
2a ( x 2 − 1 ) 1 7776  216 27
Now, on differentiation f ( x ) = 2 ...(i) = + 1728 − 3240 = = sq units
( x + ax + 1 ) 2 8  5  5 × 8 5
f ′ (1 ) = 0 = f ′ ( −1 ) 44. (A) Required area = 4s
Then, the options (b) and (d) are eliminated. π π
s = ∫ ( x + sin x ) dx − ∫ x dx
Again, for the differentiating Eq. (i) gives 0 0

( x + ax + 1 ) ⋅ 2 x − ( x − 1 )
2 2 2 π 2
π2
= − cos π + cos 0 − = 2 sq units
2 2
2 ( x 2 + ax + 1 ) (2 x + a )
f ′′( x ) = 2a ⋅
( x 2 + ax + 1 ) 4 y=x
4a 4a
f ′′( −1 ) = − , f ′′(1 ) = − y=f(x)=x+sin x
(2 − a ) 2 (2 + a ) 2
s
f –1(x)
Combining both f ′′(1 ) (2 + a ) 2 + f ′′( −1 )(2 − a ) 2 = 0
π
(x 2 − 1) ( x − 1 )( x + 1 )
39. f ′( x ) = 2a 2 = 2a 2
( x + ax + 1 ) 2 ( x + ax + 1 ) 2
If is easily seen that f ( x ) decreases on ( −1, 1 ) and has a local
minimum at x = 1, because the derivatives changes its sign
from negative to positive.
222 Textbook of Integral Calculus

1
(B) Required area = 2 ∫ xe x dx = 2 [ xe x − e x ]10 = 2 (C) The line y = x + 2 intersects y = x 2 at x = − 1 and x = 2
0
The given region is shaded region area
Y

X
x = –1 O x =1
–1 1 2

15 2 9
2 ∫ −1
= − x 2 dx =
2
(C) y = x and | y | = 2 x both the curve are symmetric about
2 3

Y-axis (D) Here, a 2 = 9, b 2 = 5, b 2 = a 2 (1 − e 2 )


4 2
y2 = x3 ⇒ e2 = ⇒ =
9 3
 5 2x y
2x

Equation of tangent at 2,  is + =1


y=

 3 9 3

4 x 2 = x 3 ⇒ x = 0, 4 9
x-intercept = , y-intercept = 3
4 16 2
Required area = 2 ∫ (2 x − x 3/2 ) dx = 9 1
0 5 Area = 4 × × 3 × = 27 sq units
(D) x + | y | = 1 2 2

Above curve is symmetric about X-axis


46. Y

(0,1)
X
–2 –1 1 2

0 (1,0)

1 5
Area is given by A = 2 ∫ ( x 2 − 3 x + 2 ) dx =
0 3
5
⇒ 3A + 2 = 3 ⋅ + 2 = 7
| y | = 1 − x and x = 1 − | y | 3
⇒ for x > 0, y > 0, y = 1 − x
1 1
47. For c < 1; ∫ (8x − x ) dx =
2 5
c 3
1 dy 1
=− 8 1 8c 3 c 6 16
2 y dx 2 x ⇒ − − − =
3 6 3 6 6
dy x
=−  8 c  16 8 1 17
3
dx y ⇒ c 3 − +  = − + =
 3 6 3 3 6 6
dy
<0 ⇒ c = −1
dx
1 1 Again, for c ≥ 1 none of the values of c satisfy the required
Function is decreasing required area = ∫ (2 x − 2 x ) = . condition that
0 3 c 16
∫1(8x − x ) dx = 3 ⇒ c + 2 = 1
2 5
45. (A) The area = 2 unit
π
(B) Area enclosed = ∫ sin x dx = 2
0
Chap 03 Area of Bounded Regions 223

 x; x <2  3 / x; x>0 2x − x 2
48. y =  and y =  51. Clearly, f (x ) =
 4 − x ; x ≥ 2 − 3 / x x <2 2
Y
P (0,2) y=x

Q (0,3/2)
B
R S
C
√3 2 3

A
Required area = PQRSP = Area PQRP + Area PRSP O
X
2 
(0,0) M N (3/2,0) (2,0)
3 3 3
= ∫  x −  dx + ∫ ( 4 − x ) −  dx (3/4,0) (1,0)
3 x 2 x
 1 1 
4 − 3 ln 3 Since, 2 x + 2y = 3; passes through A 1,  and B  , 1
= sq units  2 2 
2 so bounded area A
5 = Area OAB = 2 [Area OCM + Area CMNA – Area ONA]
49. Equation of AB y = ( x − 2)
2 1 3 3 1  3 1 1 1 1  5
=2  × × +  +  × − ∫ 0(2x − x ) dx  =
2
3 −5 2 4 4 2  4 2 4 2  24
Equation of BC y − 5 = ( x − 4 ) ⇒y = − x + 9
6−4 ⇒ 24 A = 5
0 −3 3  −1 π
Equation of CA y − 3 = ( x − 6 ) ⇒y = ( x − 2 ) 52. f ( x ) = min sin x, cos− 1, x , x ∈ [ 0, 1]
2 −6 4  6
Required area 1/ 2 −1 1  3 1 1
∴ Area = ∫ cos− 1 x dx
2  2 2 ∫ 3 /2
sin x, dx +  −  +
5 4 6 3 6 0
= ∫ ( x − 2 ) dx + ∫ − ( x − 9 ) dx − ∫ ( x − 2 ) dx
2 2 4 4 2 3 −1
4 6 6 = ( x sin − 1 x + 1 − x 2 )10/2 +
5 (x − 2) 2  (x − 9) 2  3 (x − 2) 2  4
= − − + ( x cos−1 x − 1 − x 2 ) 3 /2
2  2  2  2  4 4  2  2
π 3 −1 3 −1
5 1 3 = (1 − 3 ) + +
= [2 2 − 0 ] − [( −3 ) 2 − ( −5 ) 2 ] − [ 4 2 − 0 ] 2 2 2
2 2 8
3 π  9−π a−π
5 1
= × 4 − [9 − 25 ] − [16 − 0 ]
3 = ( 3 − 1) −  = =
 4 12 6 ( 3 + 1 ) b( 3 + 1 )
4 2 8
1 3 ∴ a = 9, b = 6 ⇒ a − b = 3
= 5 − [ − 16 ] − × 16 = 5 + 8 − 6 = 7 sq units
2 8  x
53. Given, f   = f ( x ) − f (y ) …(i)
50. If [| x | ] = 1 and [| y | ] = 0, then 1 ≤ | x | < 2,0 ≤ | y | < 1 y 
⇒ x ∈ ( −2, − 1 ] ∪ [1, 2 ), y ∈ ( −1, 1 ) Putting, x = y = 1, f (1 ) = 0
 h
If [| x | ] = 0, [| y | ] = 1 f 1 + 
f (x + h ) − f (x )  x
Now, f ′ ( x ) = lim = lim [from Eq. (i)]
Y h→ 0 h h→ 0 h
2
 h
f 1 + 
 x
= lim
h→ 0 h
1
⋅x
3 x  f (1 + x ) 
⇒ f ′ (x ) = since, lim =3
2 x 
 x → 0 x 
–2 –1 1
X
O Y

–1 (e, 3)
D y=3
C

–2
B A
Then, x ∈ ( −1, 1 ), y ∈ ( −2, − 1 ] ∪ [1, 2 ] O (1, 0) (e, 0)
X

Area of required region = 4 (2 − 1 ) (1 − ( −1 )) = 8 sq units


224 Textbook of Integral Calculus

⇒ f ( x ) = 3 log x + c Now, required area = ∫


5
g ( x ) dx
− 3/ 2
Putting x =1 ⇒ c = 0
−1 − 1/ 2 5
⇒ f ( x ) = 3 log x = y =∫
3 3
[say]
− 3/ 2
(2 ) dx + ∫− 1 (1 − x ) dx + ∫− 1/2 (2 + x ) dx
∴ Required area = ∫ x dy = ∫ e y/3 dy = 3 [e y /3 ]−3∞
−∞ −∞  3  1 1  1  1 1  1
= 2  − 1 +  +  − −  −  −1 −  + 2 5 +  + 25 − 
= 3 (e − 0 ) = 3e sq units  2   2 8   2   2 2  4
54. According to given conditions, 101
= sq units
t t 4
∫ 0 [ f (x ) − (x − 4 x 2 )] dx = k ∫ 0 [(2x − x 3 ) − f ( x )] dx
4 2
56. It is easy to see that,
Differentiable both the sides w.r.t. t, we get  ex, 0 ≤ x < log (3 / 2 )
f (t ) − (t 4 − 4 t 2 ) = k ((2 t 2 − t 3 ) − f (t )) 
f ( x ) =  3 / 2, log (3 / 2 ) ≤ x < log (2 )
or (1 + k ) f (t ) = 2 k t 2 − kt 3 + t 4 − 4 t 2. 1 + e x , log (2 ) ≤ x ≤ 1

1
⇒ f (t ) = {t 4 − kt 3 + (2k − 4 ) t 2 } Let A be the required area. Then,
k+1 (log 3/ 2) log 2 3 1 −x
A=∫ e x dx + ∫log 3/2 dx + ∫log 2 (1 + e ) dx
Hence, required f is given by; 0 2
1 3 −x 1
f (x ) = [ x 4 − kx 3 + (2k − 4 ) x 2 ] = (e x ) log 3/ 2
+log 3/ 2 + ( x − e
( x ) log 2
) log 2
k+1 0
2
55. f (t ) = | t − 1 | = | t | + | t + 1 |  3  3 3  1 1
=  − 1  +  log 2 − log  + 1 − − log 2 + 
 2  2 2  e 2
 − t, 1 ≤ −1
2 + t, −1 <t ≤ 0   4  1 
 = 2 + log   − sq units
⇒ f (t ) =    3 3  e 
2 − t, 0 <t ≤1
 t, t >1 f (x + h ) − f (x )
57. f ′ ( x ) = lim
h→ 0 h
Y
f ( x (1 + h / x )) − f ( x )
= lim
h→ 0 h
2 f (x ) f (1 + h / x ) − f (1 ) f ( x )
= lim = ⋅ f ′ (1 )
x h→ 0 h/x x
1
2 f (x ) f ′ (x ) 2
X
∴ f ′ (x ) = or =
–1 –1/2 O 1/2 1 1½ 2
x f (x ) x
Y
f(x) = x2
1 5
Case I x+2≤− ⇒ x≤−
2 2
g ( x ) = max { f (t ) : x + 1 ≤ t ≤ x + 2 } 2
y=
= f ( x + 1 ) = − x − 1, x ≤ − 5 / 2 1+x2
1 5
Case II − < x + 2 ≤ 0 ⇒ − < x ≤ − 2 (–1, 0) O (1, 0)
X
2 2
5
g ( x ) = f ( x + 2 ) = 4 + x, − < x ≤ − 2
2 Integrating both the sides, we get f ( x ) = Cx 2, since
Case III 0 < x + 2 ≤1 ⇒ −2 < x ≤ −1 f (1 ) = 1 ⇒ C = 1.
g (x ) = 2
So, f (x ) = x 2
Case IV 1 < x + 2 ≤ 3 / 2 ⇒ − 1 < x ≤ − 1 / 2
2
g (x ) = f (x + 1) = 1 − x Now, = x2 ⇒ x4 + x2 − 2 = 0
1 + x2
Case V x + 2 > 3 / 2 ⇒ x > − 1 / 2
g (x ) = f (x + 2) = 2 + x ⇒ x2 = 1 ⇒ x=±1
 1  
− x − 1, x ≤ −5/2 Required area = 2  ∫ 
2
− x 2  dx 
 4 + x, −5/2 < x ≤ −2 
0  1 + x2  
 31
Hence, g ( x ) =  2, −2 < x ≤ −1  −1 x π 1
= 2 2 tan x −  = 2 −
 1 − x, −1 < x ≤ −1/2 3 0  2 3 
 
 2 + x, x > −1/2  2
=  π −  sq units
 3
Chap 03 Area of Bounded Regions 225

58. y ( x + y 3 ) dx = x (y 3 − x ) dy By similar triangles


x
=
y z
= ,
h b+c c
⇒ xy dx + y dx = xy dy − x dy
2 3 2

a (b + c ) x cx
+b⋅
A ( R ) + A (S ) h h
So, =
A (T ) hx / 2
y = 3π/16
2
= 2 (ab + ac + bc )
h
We need to maximize (ab + bc + ca ) subject to a + b + c = h.
One way to do this is first to fix a, so b + c = h − a.
Then, (ab + bc + ac ) = a (h − a ) + bc
and bc is maximized when b = c. We now wish to maximize
 1 y  2 ab + b 2 subject to a + 2 b = h. This is a straight forward
⇒ xd ( xy ) = x 2y 3  dy − 2 dx 
 x x  calculus problem giving a = b = c = 1 / 3. Hence, the maximum
2 ratio is 2/3 (independent of T ).
d ( xy ) y  y  1 1 y 
⇒ = ⋅d   ⇒ − =   +C 60. Consider a coordinate system with vertex P of the isosceles
( xy ) 2
x  x  xy 2  x  ∆PQR at (a, 0 ) and Q and R at ( 0, b ) and ( 0, − b ) respectively.
At, x = 4, y = − 2 1
A = a ⋅ 2 b = ab …(i)
2 2
1 1 1
So, =  −  +C ⇒ C =0 Let the centre of ellipse be (α , 0 ) and the axes be of lengths, 2α
8 2 2
and 2 β.
Hence, y 3 + 2x = 0
( x − α ) 2 (y ) 2
So, f ( x ) = ( − 2 x )1/3 So, the equation of ellipse is + 2 =1
α2 β
The second equation given is Y
sin 2 x −1 cos 2 x −1
y =∫ sin t dt + ∫1/8 cos t dt Q
(0, b)
1/ 8

⇒ y ′ = x ⋅ 2 sin x cos x + x ⋅ 2 cos x ( − sin x ) = 0


So, y is constant. X
1 (α, 0) P
Put sin x = cos x = (a, 0)
2
1/ 2 R
Hence, y = ∫ (sin − 1 t + cos− 1 t ) dt (0, –b)
1/ 8

 π 
1/ 2 π 3 3π 3π
=∫   dt = ⋅ = , and g ( x ) = Now, the line PQ is tangent to the ellipse. To apply condition
 2 
1/ 8 2 8 16 16 of tangency, let us take a new system x′ y ′ whose origin is at
3π (α , 0 ).
So, we must find the area between y = f ( x ), y =
16 Then, x = x′ + α and y = y ′.
3π 1  3π 
3 x′ 2 y ′ 2
At y = ;x =−   = P (say) So, the ellipse becomes 2 + 2 = 1
16 2  16  α β
x′ + α y ′
0  3π  and the line PQ becomes + =1
Hence, area = ∫  + (2 x )1/3  dx
P  16  a b
b  α 
 3π
0 which can be written as y ′ = − x′ + b  1 − 
x 4 /3 
4
1  3π  a  a 
= x + 21/3 ⋅  =   sq units
 16 4 / 3  P 8  16  α 
2 2
  b 
So, b2  1 −  = α2  −  + β2
A ( R ) + A (S ) ay + bz  a   a 
59. As in the figure =
A (T ) h x /2  2α 
⇒ β2 = b 2  1 −  …(ii)
where h = a + b + c, the altitude of T .  a 
Now, area of ellipse = π α β ⇒ A 2 = π 2α 2β 2
 2α 
Using Eq. (ii), A 2 = π 2α 2b 2  1 −  = f (x ) (say)
c  a 
z  6α 2 
b f ′ ( x ) = π 2 b 2  2α − 
 a 
a y
6α 2 a
x f ′ (α ) = 0 ⇒ 2α = ⇒ α = (α ≠ 0 )
a 3
226 Textbook of Integral Calculus

 12α  63. By the symmetry of the figure circle(s) of maximum area will
Since, f ′′ (α ) = π 2 b 2  2 −  = − 2 π b = ( − ve)
2 2
 a  have the end point of diameter at the vertex of the two
(at α = a / 3) parabola.
1 1
 a a2  2  π 2A 2 ⇒ Radius of circle = × AB = × 8 = 4 units
So, f   = π2 ⋅ ⋅ b2  1 −  = [using Eq. (i)] 2 2
 3  9  3 27
πA 3π A
Hence, ( A ) max = = sq units
3 3 9
a a
61. A1 = ∫ {sin x − f ( x )} dx = − [cos x ]a0 − ∫ f ( x ) dx O (1,0)
0 0
3 + 2x – x2
a
= − (cos a − 1 ) − ∫ f ( x ) dx = 1 − sin a + (a − 1 ) cos a (given)
0
a  3 1
⇒ − cos a − ∫
0
f ( x ) dx = − sin a + (a − 1 ) cos a So, the area of shaded region = 4 ×
 ∫1 (3 + 2 x − x 2 ) dx −
4
a  16 
⇒ −∫ f ( x ) dx = − sin a + a cos a area of circle = 4  − 4π  sq units
0  3 
a 3
⇒ ∫0 f ( x ) dx = sin a − a cos a  1 a
64. Area of ∆AOB = 2 ×  × × a 2  =
 2 2
 a
 2
Differentiating w.r.t. a.
y = 4x2
f (a ) = cos a − (cos a − a sin a ) = a sin a
A D B
∴ f ( x ) = x sin x
Now, y = sin x and y = f ( x ) intersects at
⇒ a sin a = sin a ⇒ (a − 1 ) sin a = 0
⇒ a =1 [as sina = 0]
C
Hence, A1 = 1 − sin 1
π
–a/2 O a/2
A2 = ∫ ( x sin x − sin x ) dx = π − 1 − sin 1
1

A3 = ∫ | x sin x − sin x | dx = 3 π − 2 a /2 a3 a3 a3
π Area = Area OCBD − ∫ 4 x 2 dx =
− =
0 2 6 3
62. For x = 1, y = b ⋅ 5 x + 4 = 5b + 4 Area of triangle
= lim
dy x → 0 Area between the line and parabola
and = b ⋅ 5 x log 5 ⇒ 5b log 5 = 40 log 5 ⇒ b = 8
dx a3 / 2 3
The two curves intersects at points where = lim =
x → 0 a3 / 3 2
8 ⋅ 5 x + 4 = 25 x + 16 1
65. Required area = × 2 × 2 = 2 sq units
⇒ 5 2x − 8 ⋅ 5 + 12 = 0 ⇒ x = log 5 2, x = log 5 6 2
Hence, the area of the given region; Y
log 5 6
=∫ {8 ⋅ 5 x + 4 − (25 x + 16 )} dx (–1, 2) (1, 2)
log 5 2
log 5 6
=∫ (8 ⋅ 5 x − 25 x − 12 ) dx
log 5 2
X
log 5 6 (–1, 0) (1, 0)
 8 ⋅ 5x 25 x 
= − 12 x −
 loge 5 loge 25  log 2
5 (–1, –2) (1, –2)
log 36 log 6
−5 5 8 ⋅5 5
= + − 12 (log 5 6 − log 5 2 )
loge 25 loge 5  π ( 2 )2 
log 4
5 5
log 2
8 ⋅5 5 66. Required area = 4 ×  − (π − 2)  = 8 sq units
+ −  2 
loge 25 loge 5
− 36 48 4 16 (0, 2)
= + − 12 [log 5 3 ] + −
2 loge 5 loge 5 2 (loge 5 ) (loge 5 )
16
= − 12 log 5 3 = 4 log 5 e 4 − 4 log 5 25
loge 5 O (2, 0)
e  4
= 4 log 5   sq units
 27 
Chap 03 Area of Bounded Regions 227

67. 5 sq units b 2
Now, curves are y = bx 2 and y = x
Y 2
5 Y y = bx2
y = bx2/2
4 y=2
3
y=1
2
X
1

X
0 1 2 3 4 5

Bounded area = 2  ∫ ( x 2 − x1 ) dy 
2
68. Area of the square ABCD = 2 sq units  1 
 2  2y y  
C =2  ∫  −  dy 

1  b b  
2
 2 y 3/ 2 1 y 3/ 2 
=2  ⋅ − ⋅
D
 b 3/2 b 3 / 2 1
B 4 4
= (( 2 − 1 ) y 3/2 ) 12 = ( 2 − 1 ) (2 2 − 1 )
3 b 3 b
A Area will be maximum when b = [a ] is least.
As a ≥ 2 ⇒ [a ]least = 1 ⇒ 1 ≤ a < 2
1 π 71. Since, f ′ ( x ) < 0 ⇒ f ( x ) is a decreasing function and also
Area of the circle = π × = sq units.
2 2 f ′′ ( x ) > 0 ⇒ f ( x ) is concave upwards.
 π  Hence, the graph of the function y = f ( x ) is as follows :
Required area =  2 −  sq units
 2 
Y
69. 2| x | ⋅ | y | + 2| x | −1 ≤ 1 …(i)
Clearly, this region is symmetrical about X and Y -axes.
Let x < 0, Eq. (i) gives,
2 x − y + 2 x −1 ≤ 1
1 − 2 x −1 1
⇒ y ≤ =2−x −
2x 2
X
Y 1 2 3 4 5 n–1 n

x = 1/2 (i)
Y
1/2 y = 1/2
C B
X
O 1/2

–1/2

Clearly, bounded region in the first quadrant is OABC. The X


required area is 4 times the area of the region OABC. 1 2 3 4 5 n–1 n
1/ 2
1/ 2  − x 1  2− x x  (ii)
Required area = 4 ∫ 2 −  dx = 4  − − 
0  2  ln 2 2  Let S L denotes the shaded area in figure (i).
0
n
 4
=

(1 − 2 − 1/2 ) − 1  sq units
⇒ S L = f (2 ) + f (3 ) + … + f (n ) = ∑ f (r ) − f (1 )
 ln 2  r=1
n
70. Pair of lines y 2 − 3y + 2 = 0 From the figure (i) it is clear that, S L < ∫ f ( x ) dx
1
⇒ Lines are y = 2, y = 1. n
n
Let [a ] = b ⇒ ∑ f (r ) − f (1 ) < ∫
1
f ( x ) dx
r =1
228 Textbook of Integral Calculus

n
n Y
⇒ ∑ f (r ) < ∫
1
f ( x ) dx + f (1 ) …(i)
(1,2)
r =1 C
1)
Let Su denotes the area of the shaded region in figure (ii). B (– 4,
 1  1 (1,0)
⇒ Su =  f (1 ) + f (2 )  + ( f (2 ) + f (3 )) + … X′ X
 2  2 –9 (– 4,0) A E (– 3,0) 0 D 6
1
+ ( f (n − 1 ) + f (n ))
2
= [( f (1 ) + f (2 ) + f (3 )) + … + f (n − 1 ) + f (n )] Y′
1
− ( f (1 ) + f (n )) ∴ Required area = Area of trapezium ABCD
n
1 1 2 − Area of ABE under parabola
= ∑ f (r ) − f (1 ) − f (n )
r =1
2 2 − Area of CDE under parabola
1 −3 1
From figure (ii) it is clear that; = (1 + 2 )(5 ) − ∫ − ( x + 3 ) dx − ∫ ( x + 3 ) dx
2 − 4 − 3
n
Su > ∫ f ( x ) dx   
−3

1
1
n
1 n 15 ( − 3 − x ) 3/2   ( x + 3 ) 3/ 2 
= −  −
⇒ ∑ f (r ) − ( f (1 ) + f (n )) > ∫ f ( x ) dx
2 1 2  −
3 3 
r =1    
n  2 − 4  2  –3
1 n
⇒ ∑ f (r ) > ∫
1 2
f ( x ) dx +
( f (1 ) + f (n )) 15 2
=
2
+ [ 0 − 1 ] − [8 − 0 ] =
15 2 16 15 18 3
− − = − =
r =1
n 1 2 3 3 2 3 3 2 3 2
> ∫ f ( x ) dx + 4 ⋅ f (1 ) …(ii)
1 2 73. Since, F ′ (a ) + 2 is the area bounded by x = 0, y = 0, y = f ( x )
From Eqs. (i) and (ii), we get and x = a .
a
∴ ∫0 f (x )dx = F ′ (a ) + 2
n
1 n n

2
f (1 ) + ∫1 f ( x ) dx < ∑ f (r ) < ∫
1
f ( x ) dx + f (1 ).
r =1 Using Newton-Leibnitz formula,
72. Here, {( x, y ) ∈ R 2 : y ≥ | x + 3|, 5y ≤ ( x + 9) ≤ 15} f (a ) = F ′′(a ) and f ( 0 ) = F ′′( 0 ) …(i)
x 2 + π /6
∴ y ≥ | x + 3| Given, F (x ) = ∫ 2
2 cos t dt
x
 x + 3 , when x ≥ − 3 On differentiating,
⇒ y ≥ 
 − x − 3 , when x ≤ − 3  π
F ′ ( x ) = 2 cos2  x 2 +  ⋅ 2 x − 2 cos2 x ⋅ 1
 6
 x + 3 , when x ≥ − 3
or y2 ≥  Again differentiating,
− 3 − x, when x ≤ − 3
  π  π  π 
Shown as F ′′( x ) = 4 cos2  x 2 +  − 2 x cos x 2 +  sin  x 2 +  2 x 
  6   6   6 
y2=–x–3 Y
y2=x+3 + { 4 cos x ⋅ sin x }
 2 2 π  π   π 
= 4 cos  x +  − 4 x 2 cos  x 2 +  sin  x 2 +  
  6   6   6
X′
0
X + 2 sin 2 x
–3  2 π 
∴ F ′′( 0 ) = 4 cos    = 3
Y′   6
Also, 5y ≤ ( x + 9 ) ≤ 15 ∴ f (0) = 3
⇒ ( x + 9 ) ≥ 5y and x ≤ 6 2
74. Let equation of tangent to parabola be y = mx +
Shown as m
Y It also touches the circle x 2 + y 2 = 2.
 2 
(0, 9/5) ∴  = 2
m 1 + m
2

X
(–9, 0) 0 ⇒ m4 + m2 = 2 ⇒ m4 + m2 − 2 = 0

x=6 ⇒ (m 2 − 1 ) (m 2 + 2 ) = 0
⇒ m = ± 1, m 2 = − 2 [rejected m 2 = − 2]
∴ {( x, y ) ∈ R : y ≥ | x + 3|, 5y ≤ ( x + 9 ) ≤ 15 }
2
So, tangents are y = x + 2, y = − x − 2.
They intersect at ( −2, 0 ).
Chap 03 Area of Bounded Regions 229

Y
R Since, x 2 ≤ x when x ∈[ 0, 1 ]
− x 2 ≥ − x or e − x ≥ e − x
2
P ⇒
T(–2,0) 1 − x2 1
X′ X ∴ ∫0 e dx ≥ ∫ e − x dx
O 0
1
Q ⇒ S ≥ − (e − x )10 = 1 − …(i)
e
Y′ 1
Also, ∫ e − x dx ≤ Area of two rectangles
2
S
0
Equation of chord PQ is −2 x = 2 ⇒ x = − 1
 1   1  1
Equation of chord RS is 0 = 4( x − 2 ) ⇒ x = 2 ≤ 1 ×  + 1 −  ×
 2  2 e
∴ Coordinates of P, Q, R, S are 1 1  1 
P ( −1, 1 ), Q ( −1, − 1 ), R (2, 4 ), S (2, − 4 ) ≤ + 1 −  …(ii)
2 e 2
(2 + 8 ) × 3
∴ Area of quadrilateral = = 15 sq units 1 1  1  1
2 ∴ + 1 −  ≥ S ≥ 1 − [from Eqs. (i) and (ii)]
2 e  2  e
75. To find the bounded area between y = f ( x ) and y = g( x ) 2
77. R1 = ∫ x f ( x ) dx …(i)
between x = a to x = b. −1
b b
Y
f (x) Using ∫ a f (x ) dx = ∫ a f (a + b − x ) dx
g (x) 2
R1 = ∫ (1 − x ) f (1 − x ) dx
g (x) −1
f (x) 2
∴ R1 = ∫ (1 − x ) f ( x ) dx …(ii)
−1
a c X
O b [f ( x ) = f (1 − x ), given]
c b Given, R2 is area bounded by f ( x ), x = − 1 and x = 2.
∴ Area bounded = ∫ [ g( x ) − f ( x )]dx + ∫ [ f ( x ) − g( x )]dx 2
a c ∴ R2 = ∫ f ( x ) dx …(iii)
b −1
= ∫ | f ( x ) − g( x )| dx On adding Eqs. (i) and (ii), we get
a
π 2
Here, f ( x ) = y = sin x + cos x, when 0 ≤ x ≤ 2 R1 = ∫ f ( x ) dx …(iv)
2 π
−1

cos x − sin x, 0≤x≤
 4 From Eqs. (iii) and (iv), we get
and g( x ) = y = | cos x − sin x | = 
π π 2 R1 = R2
sin x − cos x, ≤x≤
could be shown as  4 2 78. Here, area between 0 to b is R1 and b to 1 is R2.
b 1 1
∴ ∫0 (1 − x ) dx − ∫b (1 − x ) dx = 4
Y 2 2
f(x) y = sin x + cos x
√2
= √2 sin x + π
b 1
(1 − x ) 3  (1 − x ) 3  1
4 ⇒  −3  −  −3  = 4
1
 0  b
g(x) g(x) 1 1 1
X ⇒ − [(1 − b ) − 1 ] + [ 0 − (1 − b ) 3 ] =
3

O π/4 π/2 3 3 4
2 1 1 1
π /4 ⇒ − (1 − b ) = − + = −
3
∴ Area bounded = ∫ {(sin x + cos x ) − (cos x − sin x )} dx 3 3 4 12
0
π /2 1
⇒ (1 − b ) 3 =
+ ∫π /4 {(sin x + cos x ) − (sin x − cos x )} dx 8
π /4 π /2 1 1
=∫ ⇒ (1 − b ) = ⇒ b=
0
2 sin x dx + ∫π /4 2 cos x dx 2 2
= − 2 [cos x ]0π /4 + 2 [sin x ]ππ //24  1 x
79. Shaded area = e −  ∫ e dx = 1 
 0 
= 4 − 2 2 = 2 2( 2 − 1 ) sq units e

76. Graph for y = e −x 2 Also, ∫1 ln(e + 1 − y )dy [put e + 1 − y = t ⇒ − dy = dt]


Y Y
1 e y=e

1
— X
√e 0 1
X 1 e e
O 1 1 = ∫ ln t ( − dt ) = ∫ ln t dt = ∫ lny dy = 1
— e 1 1
√2
230 Textbook of Integral Calculus

π /4 1 − sin x 
1
1 + sin x 83. Let I = ∫ g′( x ) dx = [ g( x )]1− 1 = g (1) − g ( − 1)
80. Required area = ∫  −  dx −1
0  cos x cos x 
Since, y 3 − 3y + x = 0 …(i)
 1 + sin x 1 − sin x 
> >0 and y = g (x )
 
Q
cos x cos x
∴ { g( x )} 3 − 3 g( x ) + x = 0 [from Eq. (i)]
 x x 
 2 tan 2 tan  At x = 1, { g(1 )} − 3 g(1 ) + 1 = 0
3
…(ii)
 1+ 2 1− 2 
At x = − 1, { g( − 1 )} 3 − 3 g( − 1 ) − 1 = 0 …(iii)
 1 + tan 2
x
1 + tan 2 
x
π /4
=∫ 2 − 2  dx On adding Eqs. (i) and (ii), we get
0  x x  { g(1 )} 3 + { g( − 1 )} 3 − 3 { g (1 ) + g( − 1 )} = 0
 1 − tan 2 1 − tan 2 
 2 2  ⇒ [ g(1 ) + g( − 1 )][{ g(1 )} 2 + { g( − 1 )} 2 − g(1 ) g( − 1 ) − 3 ] = 0
 x x 
1 + tan 2 1 + tan 2 ⇒ g(1 ) + g ( − 1 ) = 0
 2 2 
⇒ g(1 ) = − g( − 1 )
 x x ∴ I = g(1 ) − g( − 1 ) = g(1 ) − { − g(1 )} = 2 g(1 )
π /4 
1 + tan 1 − tan 
=∫  2 − 2  dx 1 2 2 x2
 1 − tan x 84. Required area = ∫ (1 + x )dx + ∫ (3 − x )dx − ∫
1 + tan 
0 x dx
0 1 0 4
 2 2
Y
x x x y=1+√x
1 + tan − 1 + tan 2 tan (0, 3) (1, 2)
π /4 π /4 4y=x 2
=∫ 2 2 dx = ∫ 2 dx
0 x 0 x
1 − tan 2 1 − tan 2 (0, 1) (2, 1)
2 2 x+y=3
x 1 2 x
Put tan = t ⇒ sec dx = dt X′
(0, 0) (1, 0)(2, 0) (3, 0)
X
2 2 2
π
tan 4t dt Y′
=∫ 8
(1 + t 2 ) 1 − t 2  3/ 2  1

2 2
x2  x3 
0
x
= x +  + 3 x −  −  
2 −1 4t dt π  3 /2  0  2  1  12  0
As ∫0 (1 + t ) 1 − t
2 2
[Q tan
8
= 2 − 1]
 2  1  8 
= 1 +  + 6 − 2 − 3 +  −  
81. Given, y − 3y + x = 0
3  3  2  12
5 3 2 3 5
⇒ 3y 2
dy
−3
dy
+1=0 …(i) = + − = 1 + = sq units
dx dx 3 2 3 2 2
 d 2y  2
d 2y 85. Given equations of curves are y 2 = 2x, ...(i)
 dy 
⇒3y 2  2  + 6y   − 3 2 = 0 …(ii)
 dx   dx  dx which is a parabola with vertex (0, 0) and axis parallel to
X-axis.
At x = − 10 2, y = 2 2
And x 2 + y 2 = 4x ...(ii)
On substituting in Eq. (i) we get
which is a circle with centre (2, 0) and radius = 2
dy dy dy 1
3(2 2 ) 2 ⋅ − 3⋅ +1=0 ⇒ =− On substituting y 2 = 2 x in Eq. (ii), we get
dx dx dx 21
x 2 + 2x = 4x ⇒ x 2 = 2x
Again, substituting in Eq. (ii), we get
2 ⇒ x = 0 or x = 2
d 2y  1 d 2y
3(2 2 ) 2 + 6 (2 2 ) ⋅  −  − 3 ⋅ 2 = 0 ⇒ y = 0 or y = ± 2 [using Eq. (i)]
dx 2  21  dx
Now, the required area is the area of shaded region, i.e.
d 2y 12 2
⇒ 21 ⋅ =−
dx 2
(21 ) 2 Y

d y − 12 2 − 4 2
2
A(2,2)
⇒ = = 3 2 x2+y2=4x
dx 2 (21 ) 3 7 ⋅3
b b b X′
82. Required area = ∫ y dx = ∫ f ( x ) dx = [ f ( x ) ⋅ x ]ba − ∫ f ′( x ) x dx (0, 0) B (2,0)
X
a a a
2
b y =2x
= bf (b ) − af (a ) − ∫ f ′( x ) x dx
a
b xdx
= bf (b ) − af (a ) + ∫a 3[{ f (x )} 2 − 1] Y′

 −1 −1  Area of a circle 2
dy Required area = − ∫ 2 x dx
Q f ′ ( x ) = dx = =  4 0
 3(y 2
− 1 ) 3[{ f ( x )} 2
− 1 ] 
Chap 03 Area of Bounded Regions 231

2 x(2 ) y (5 / 3 )
π(2 ) 2 2  x 3/ 2  + =1
= − 2 ∫ x1/2dx = π − 2   9 5
4 0
 3 /2 0
Y
2 2  8 (0,3)
=π− [2 2 − 0 ] =  π −  sq units P
3  3
(–2, 5/3)
86. Given region is {( x, y ) : y 2 ≤ 2x and y ≥ 4x − 1} M L (2, 5/3)
y ≤ 2 x represents a region inside the parabola
2
Q
X′ X
y 2 = 2x …(i) (–2,0) O (2,0) (9/2, 0)
and y ≥ 4 x − 1 represents a region to the left of the line M′ L′
y = 4x − 1 …(ii) (–2, –5/3) (2, –5/3)

The point of intersection of the curve (i) and (ii) is


(4x − 1)2 = 2x Y′
or 2 x + 3y = 9 …(ii)
⇒ 16 x 2 + 1 − 8 x = 2 x 9 
Eq.(ii) intersects X and Y-axes at  , 0 and ( 0, 3 ), respectively.
⇒ 16 x 2 − 10 x + 1 = 0 2 
1 1 ∴ Area of quadrilateral = 4 × Area of ∆POQ
⇒ x= ,
2 8 1 9 
= 4 ×  × × 3 = 27 sq units
1  2 2 
∴ The points where these curves intersect, are  , 1 and
2 
 1 1 88. Given, A = {( x, y ) : x 2 + y 2 ≤ 1 and y 2 ≤ 1 − x }
 ,− .
 8 2 y = 4x – 1 Y
Y 1,
1
2 y2 = 2x
1
1 X′ X
2 (–1,0)
–1 1 (0,1)
–1 2 0 2 1
X′ X
1 , –1
–1 8 2
Y′
2 1
1 1 1  y 3
–1 Required area = πr 2 + 2 ∫ (1 − y 2 )dy = π(1 ) 2 + 2 y − 
2 0 2  3 0
Y′ π 4
1 y + 1 y 2 = +
Hence, required area = ∫  −  dy 2 3
−1/ 2  4 2 89. Given curves are y = x …(i)
1
1 y 2  1 and 2y − x + 3 = 0 …(ii)
=  + y − (y 3 )1−1/2 Y
4 2  −1/ 2 6
4
1  1   1 1  1  1 3
=  + 1 −  −   − 1 +  y = √x
4  2 8 2  6 8 1
3 =0
1 − x+
1 3 3  1 9  1 15 3 9 2y
=  +  −  = × − = X′ O X
4 2 8  6 8  4 8 16 32 −1 1 2 3 9
87. Given equation of ellipse is −2 0,− 3
−3 2
x2 y 2
+ =1 …(i) −4
9 5 Y′
∴ a 2 = 9, b 2 = 5 ⇒ a = 3, b = 5 On solving Eqs. (i) and (ii), we get 2 x − ( x ) 2 + 3 = 0
b2 5 2 ⇒ ( x )2 − 2 x − 3 = 0
Now, e= 1− = 1− =
a2 9 3 ⇒ ( x − 3) ( x + 1) = 0
b2 5 ⇒ x = 3 [Q x = − 1 is not possible]
Foci = ( ± ae, 0 ) = ( ± 2, 0 ) and =
a 3 ⇒ y =3
 5  −5 
∴ Extremities of one of latusrectum are 2,  and 2,  . 3 3
 3  3 ∴ Required area = ∫ ( line − curve) dy = ∫ {(2y + 3 ) − y 2 } dy
0 0
 5 
3
∴ Equation of tangent at 2,  is, y3
 3 = y 2 + 3y −  = 9 + 9 − 9 = 9
 3 0
232 Textbook of Integral Calculus

y
90. Given Two parabolas x 2 = and x 2 = 9y 1
4 π
To find The area bounded between the parabolas and the O
straight line y = 2. π π 3π 2π
−1 2 2
The required area is equal to the shaded region in the drawn
figure.
Y
y = 4x 2
1
y = x2
9 π 3π/2
y=2 O π/2

π /4 5π / 4
Required area = ∫ (cos x − sin x ) dx + ∫π /4 (sin x − cos x ) dx
X 0
3π / 2
+∫ (cos x − sin x ) dx
5π / 4
The area of the shaded region (which can be very easily found
= [sin x + cos x ]0π /4 + [ − cos x − sin x ]5ππ/4/4 + [sin x + cos x ]35ππ //24
by using integration) is twice the area shaded in first quadrant.
2 y 25  = ( 4 2 − 2 ) sq units
Required area = 2 ∫ 3 y −  dy = 2 ∫  y  dy
0 2  0 2  93. The equation of tangent at (2, 3) to the given parabola is
y=2
x = 2y − 4
 y 3/ 2  10 20 2 Y
=5   = (2 3/2 − 0 ) = (2, 3)
 3 /2 y = 0 3 3
1
91. Given, y = x, x = e and y = , x ≥ 0
x
Since, y = x and x ≥ 0 ⇒ y ≥0 X′ X
(– 4, 0)
∴ Area to be calculated in I quadrant shown as 2
Y ′ (y – 2) = (x – 1)
Y 3
∴ Required area = ∫ {(y − 2 ) 2 + 1 − 2y + 4 }dy
y=x 0
3
(y − 2 ) 3 
= − y 2 + 5y 
A  3 0
B 1 8
O y = 1/x =
− 9 + 15 + = 9 sq units
X′ X 3 3
D (1, 0) C (e, 0)
x=e 94. Given, equations of curves are x + 3y 2 = 1 ... (i)
Y′
∴ Area = Area of ∆ODA + Area of DABCD and x + 2y = 0
2
... (ii)
1 e 1 On solving Eqs. (i) and (ii), we get
= (1 × 1 ) + ∫ dx
2 1 x y = ± 1 and x = − 2
1 1
= + (log | x | ) e1
2
∴ Required area = ∫ −1(x1 − x 2 )dy
1 Y
= + {log | e | − log 1 } [Q log | e | = 1 ]
2 (−2, 1)
1 3
= + 1 = sq units (1, 0)
2 2 X
X′
92. Graph of y = sin x is −
1, 0
8 − 11 , 0
Y 12
(−2, −1)
1
O π 3π/2 Y′
X′ X 1 1
π/2
∫ −1(1 − 3y ∫ −1(1 − y
–2π 2π 3π = 2
+ 2y ) dy =2 2
) dy
−1
Y′ 1
1  y3  1 4
and graph of y = cos x is = 2 ∫ (1 − y 2 ) dy = 2 y −  = 2 1 −  = sq units
0
 3 0  3  3
CHAPTER

04
Differential
Equations
Learning Part
Session 1
● Solution of a Differential Equation

Session 2
● Solving of Varibale Seperable Form

● Homogeneous Differential Equation

Session 3
● Solving of Linear Differential Equations

● Bernoulli’s Equation

● Orthogonal Trajectory

Session 4
● Exact Differential Equations

Session 5
● Solving of First Order and Higher Degrees

● Application of Differential Equations

● Application of First Order Differential Equations

Practice Part
● JEE Type Examples
● Chapter Exercises

Arihant on Your Mobile !


Exercises with the #L
symbol can be practised on your mobile. See inside cover page to activate for free.
234 Textbook of Integral Calculus

A differential equation can simply be said to be dm


i.e. = - lm
an equation involving derivatives of an unknown dt
function. For example, consider the equation where m is the mass of the radioactive substance and a
dy function of t. If we know m(0 ), the initial mass, we can use
+ xy = x 2
dx this differential equation to determine the mass of the
This is a differential equation since it involves the substance remaining at any later time instant.
derivative of the funtion y ( x ) which we may wish to
determine. We must first understand why and how Scenario 3 : Population Growth
differnetial equations arise and why we need them at all. In The growth of population (of say, a biological culture) in a
general, we can say that a differential equation describes closed environment is dependent on the birth and death
the behaviour of some continuously varying quantity. rates. The birth rate will contribute to increaseing the
population while the death rate will contribute to its
Scenario 1 : A Freely Falling Body decrease. It has been found that for low populations, the
A body is release at rest from a heigh h. How do we birth rate is the dominant influence in population growth
described the motion of this body? and the growth rate is linearly dependent on the current
population. For high populations, there is a competition
The height x of the body is a function of time. Since the
among the population for the limited resources available,
d 2x
acceleration of the body is g, we have =g and thus death rate becomes dominant. Also, the death rate
dt 2 shows a quadratic dependence on the current population.
This is the differential equation describing the motion of Thus, if N (t ) represents the population at time t, the
the body. Along with the initial condition x (0 ) = h, it different equation describing the population variation is of
completely describes the motion of the body at all instants the form
after the body starts falling.
dN
= l1 N - l2 N 2
Scenario 2 : Radioactive disintegration dt
Experimental evidence shows that the rate of decay of any where l1 and l2 are constants.
ratioactive substance is proportional to the amount of the Along with the initial population N (0 ), this equation can
substance present, tell us the population at any later time instant.

Session 1
Solution of a Differential Equation
These three examples should be sufficient for you to realise variable (while treating the other independent variable y as
why and how differential equations arise and why they are ¶
a constant). A similar interpretation can be attached to .
important. ¶y
Such equations are termed partial differential
In all the three equations mentioned above, there is only
equations but we shall not be concerned with them in
independent variable (the time t in all the three cases).
this chapter.
Such equations are termed ordinary differential
equations. We might have equations involving more than Consider the ordinary differnetial equation
one independent variable : d 2y dy
+x + x2 =c
¶f ¶f 2 dx
+x = x2 dx
¶x ¶y The order of the highest derivative present in this
¶ equation is two; thus we shall call it a second order
where the notation stands for the partial derivative, i.e.
¶x differential equation (DE, for convenience).
¶f
the term would imply that we differentiate the function The order of a DE is the order of the highest derivative
¶x
f with respect to the independent variable x as the that occurs in the equation
Chap 04 Differential Equations 235

Again, consider the DE is a solution to the second-order DE


d 2y dy d 2y
+ = x 2y 2 =y
3
dx dx dx 2
The degree of the highest order derivative in this DE is but even through it (seems to) contain two arbitrary
two, so this is a DE of degree two (and order three). constants, it is not the general solution to this DE. This is
The degree of a DE is the degree of the highest order because it can be reduced to a relation involving only one
derivative that occurs in the equation, when all the arbitrary constant :
derivatives in the equation are made of free of y = ae x +b = ae x . e b = ce x (where c = a × e b )
fractional powers.
Let us summarise what we have seen till now : the most
2
æ dy ö æ d 2y ö general solution of an n th order DE will consist of n
ç ÷ -1 + x ç 2 ÷ =k orbitrary constants; conversely, from a functional relation
è dx ø è dx ø
involving n arbitrary constants, an n th order DE can be
is not of degree two. When we make this equation free of generated (we shall soon see how to do this). We are
fractional powers, by the following rearrangement, generally interested in solutions of the DE satisfying some
2 particular constraints (say, some initial values). Since the
2 ì æ d 2 y ö üï
2
æ dy ö ï most general solution of the DE involves n arbitrary
ç ÷ - 1 + í k - x ç 2÷ ý
è dx ø ïî è dx ø ï constant, we see that the maximum member of
þ independent conditions which can be imposed on a
we see that the degree of the highest order derivative will solution of the DE is n. As a first example, consider the
become four. Thus, this is a DE of degree four (and order functional relation
two). y = x 2 + c 1e 2x + c 2e 3x …(i)
Finally, an n th linear DE (degree one) is an equation of
This curve’s equation contains two arbitrary constants; as
the form we vary c 1 and c 2 , we obtain different curves; those curves
d ny d n -1 dy constitute a family of curves. All members of this family
a0 + a1 + K + an - 1 + any = b
n n -1 dx will satisfy the DE that we can generate from this general
dx dx
where the a i¢s and b are functions of x . relation; this DE will be second order since the relation
contains two arbitrary constants.
Solving an n th order DE to evaluate the unknown function
will essentially consists of doing n integrations on the DE. We now see how to generate the DE. Differentiate the
Each integration step will introduce an arbitrary constant. given relation twice to obtain
Thus, you can expect in general that the solution of an y ¢ = 2 x + 2c 1 e 2 x + 3c 2 e 3 x …(ii)
n th order DE will contain n independnet arbitrary y ¢¢ = 2 + 4c 1 e 2 x + 9c 2 e 3 x …(iii)
constants.
From Eqs. (i), (ii) and (iii), c 1 and c 2 can be eliminated to
By n independent constants, we mean to say that the most obtain
general solution of the DE cannot be expressed in fewer
that n constants. As an example, the second order DE e 2x e 3x x2 -y
dy 2e 2 x 3e 3 x 2x - y ¢
+y =0
dx 2 4e 2 x 9e 3 x 2 - y ¢¢
has its most general solution of the form 1 1 x2 -y
y = A cos x + B sin x . …(1)
Þ 2 3 2x - y ¢
(verify that this is a solution by explicit substitution).
4 9 2 - y ¢¢
Thus, two arbitrary and independent constants must be
included in the general solution. We cannot reduce (1) to a Þ 6 - 3 ¢¢ - 18 x + 9y ¢ + 8 x - 4y ¢ - 4 + 2y ¢¢ + 7 x 2 - 6y = 0
relation containing only one arbitrary constant. On the
Þ y ¢¢ - 5y ¢ + 6y = 6 x 2 - 10 x + 2 …(iv)
other hand, it can be verified that the function
y = ae x + b This is the required DE; it corresponds to the family of
curves given by Eq. (i). Differently put, the most general
solution of this DE is given by Eq. (i).
236 Textbook of Integral Calculus

As an exercise for the reader, show that the DE


corrersponding to the general equation Linear and Non-linear
y = Ae 2 x + Be x + C Differential Equation
where A, B, C are arbitrary constants, is A differential equation is a linear differential equation if it
y ¢¢¢ - 3y ¢¢ + 2y ¢ = 0 is expressible in the form
By expected, the three arbitrary constants cause the DE to d ny d n -1 y d n -2 y dy
a0 + a1 + a2 +¼ + a n -1 + any = Q
the third order. n n -1 n -2 dx
dx dx dx
where a 0 , a 1 , a 2 ,¼, a n and Q are either constants or
y Example 1 Find the order and degree (if defined) of
functions of independent variable x.
the following differential equations :
2 3
Thus, if a differential equation when expressed in the form
æ dy ö 1 æ dy ö 1 æ dy ö of a polynomial involves the derivatives and dependent
(i) y = 1 + ç ÷ + ç ÷ + ç ÷ +¼
è dx ø 2 ! è dx ø 3 ! è dx ø variable in the first power and there ae no product of
2/ 3 these, and also the coefficient of the various terms are
æ d 3y ö dy d 2y æ dy ö either constants or functions of the independent variable,
(ii) ç 3 ÷ = +2 (iii) = x ln ç ÷
è dx ø dx dx 2
è dx ø then it is said to be linear differential equation. otherwise,
it i a non-linear differential equation.
Sol. (i) The given differential equation can be rewritten as 2
dy /dx æ d 3y ö æ d 2y ö
y =e The differentiable equation ç ÷ -6 ç ÷ - 4y = 0, is
dy è dx 3 ø è dx 2 ø
Þ = ln y .
dx a non-linear differential equation, because its degree is 2,
Hence, its order is 1 and degree 1. more than one.
2
(ii) The given differential equation can be rewritten as æ d 3y ö æ dy ö
e.g. The differential equation, ç ÷ + 2 ç ÷ + 9y = x ,
æ d 3y ö
2
æ dy ö
3
è dx 3 ø è dx ø
ç 3÷ = ç + 2÷ ×
è dx ø è dx ø is non-linear differential equation, because differential
dy
Hence, its order is 3 and degree 2. coefficient has exponent 2.
(iii) Its order is obviously 2.
dx
Since, the given differential equation cannot be e.g. The differential equation ( x 2 + y 2 ) dx - 2 xydy = 0 is a
written as a polynomial in all the differential non-linear differential equation, because the exponent of
coefficients, the degree of the equation is not defined. dependent variable y is 2 and it involves the product of y
dy
y Example 2 Find the order and degree (if defined) of and . e.g. Consider the differential equation
dx
the following differential equations :
æ d 2y ö æ dy ö
d 2y d 2y ç 2 ÷ - 5 ç ÷ + 6y = sin x
dy æ dy ö è dx ø è dx ø
(i) = 3 +3 (ii) = sin ç ÷
dx 2
dx dx 2
è dx ø This is a linear differential equation of order 2 and degree.
dy
(iii) = 3x + 5
dx
Sol. (i) The given differential equation can be rewritten as
Formation of Differential Equations
3 2
If an equation in independent and dependent variables
æ d 2y ö æ dy ö involving some arbitrary constants is given, then a
ç 2 ÷ = ç + 3÷
è dx ø è dx ø differential equation is obtained as follows :
Hence, order is 2 and degree is 3. (i) Differentiate the given equation w.r.t. the
(ii) The given differential equation has the order 2. Since, independent variable (say x) as many times as the
the given differential equation cannot be written as a number of arbitrary constants in it.
polynomial in the differential coefficients, the degree (ii) Eliminate the arbitrary constants.
of the equation is not defined.
(iii) The eliminant is the required differential equation.
(iii) Its order is obviously 1 and degree 1.
i.e. If we have an equation f ( x , y , c 1 , c 2 , ¼, c n ) = 0
Chap 04 Differential Equations 237

Containing n arbitrary constants c 1 , c 2 , c 3 , ¼, c n , then by Using Eqs. (i) and (iii), we get
differentiating this n times, we shall get n-equations. dy d 2y
x = xy 2
+2
Now, among these n-equations and the given equation, in dx dx
all (n + 1) equations, if the n arbitrary constants Which is the required differential equation.
c 1 , c 2 , c 3 , ¼, c n are eliminated, we shall evidently get a
differential equation of the nth order. For there being n y Example 5 Find the differential equation whose
differentiation, the resulting equation must contain a solution represents the family c ( y + c ) 2 = x 3 .
derivative of the nth order. Sol. Differentiating c (y + c )2 = x 3 …(i)
dy
We get c [2 (y + c )] = 3x 2 but from Eq. (i), we have
Algorithm for Formation dx
2x 3 dy
of Differential Equations (y + c ) 2
(y + c )
dx
= 3x 2
Step I Write the given equation involving independent 2x 3 dy 2x dy
variable x (say), dependent variable y (say) and the Þ × = 3x 2 i.e. × =3
y + c dx y + c dx
arbitrary constant.
2x é dy ù 2x é dy ù
Step II Obtain the number of arbitrary constants in Step Þ =y +c \ c = -y
3 êë dx úû 3 êë dx úû
I. Let there be n arbitrary constants.
Substituting c in Eq. (i), we get
Step III Differentiate the relation in step I, n times with 2
respect to x. é 2x æ dy ö ù é 2x dy ù 3
ê 3 ç ÷ - yú ê ú =x
ë è dx ø û ë 3 dx û
Step IV Eliminate arbitrary consstants with the help of n
equations involving differential coefficients obtained in Which is the required differential equation.
step III and an equation in step I. y Example 6 Find the differential equation whose
The equation so obtained is the desired differential solution represents the family y = ae 3x + be x .
equation. The following examples will illustrate the above
procedure. Sol. y = ae 3 x + be x …(i)
Differentiating the given equation twice, we get
y Example 3 Form the differential equation, if dy d 2y
= 3ae 3 x + be x and = 9ae 3 x + be x
y 2 = 4a ( x + b ), where a, b are arbitrary constants. dx dx 2
Sol. Differentiating y 2 = 4a ( x + b ) w.r.t. x, From the three equations by eliminating a and b, we obtain
d 2y 4dy
2y ×
dy
= 4a i.e. y
dy
= 2a - + 3y = 0
dx dx dx 2 dx
Remark
Again, differentiating w.r.t. x, we get The order of the differential equation will be equal to number of
2 2 independent parameters and is not equal to the number of all the
d y æ dy ö
y + ç ÷ =0 parameters in the family of curves.
dx 2 è dx ø
which is the required differential equation. Thus, the y Example 7 Find the order of the family of curves
+ c4
elimination of arbitrary leads to the formation of a y = (c 1 + c 2 ) e x + c 3 e x .
differential equation. Sol. Here, the number of arbitrary parameters is 4 but the
order of the corresponding differential equation will not
y Example 4 Find the differential equation whose be 4 as it can be rewritten as, y = (c 1 + c 2 + c 3 e c 4 ) e x ,
solution represents the family xy = ae x + be - x . which is of the form y = Ae x . Hence, the corresponding
Sol. xy = ae x + be - x …(i) differential equation will be of order 1.
Differentiating Eq. (i) w.r.t. x, we get
y Example 8 The differential equation of all
dy
x + y = ae x - be - x …(ii) non-horizontal lines in a plane is given by
dx d 2y d 2x
Differentiating Eq. (ii) w.r.t. x, we get (a) 2
=0 (b) =0
dx dy 2
d 2y dy dy
x + ×1 + = ae x + be - x …(iii) d 2y d 2x
dx 2
dx dx (c) 2
= 0 and =0 (d) All of these
dx dy 2
238 Textbook of Integral Calculus

Sol. The equation of the family of all non-horizontal lines in 1 1


Þ tan a = - or sin a =
a plane is given by, y1 1 + y12
ax + by = 1 (where a ¹ 0) …(i)
y1
Differentiating w.r.t. y, we get and cos a = - …(ii)
dx 1 + y12
a +b=0 (as a ¹ 0 and b Î R)
dy From Eqs. (i) and (ii), we get
- xy1 y
Again, differentiating w.r.t. y, we get + =p
d 2x 1+ y12 1 + y12
a =0 (as a ¹ 0 and b Î R)
dy 2 Þ (y - xy1 )2 = p 2 (1 + y12 ) is required differential equations.
d 2x Hence, (c) is the correct answer.
Þ 2
=0 (as a ¹ 0)
dy
y Example 11 The differential equation of all circles of
\ Differential equation of all non-horizontal lines in a radius r, is given by
d 2x
plane is 2 = 0. Hence, (b) is the correct answer. (a) {1 + (y 1 ) 2 } 2 = r 2 y 23 (b) {1 + (y 1 ) 2 } 3 = r 2 y 23
dy
(c) {1 + (y 1 ) 2 } 3 = r 2 y 22 (d) None of these
y Example 9 The differential equation of all Sol. Equation of circle of radius r ,
non-vertical lines in a plane is given by
( x - a )2 + (y - b )2 = r 2 …(i)
d 2y 2
d x
(a) =0 (b) =0 (Here, a, b are two arbitrary constants)
dx 2 dy 2 Differentiating Eq. (i), we get
d 2x d 2y 2 ( x - a ) + 2 (y - b ) y 1 = 0 …(ii)
(c) 2
= 0 and 2
=0 (d) All of these
dy dx Again, differentiating Eq. (ii), we get

Sol. The equation of the family of all non-vertical lines in a 1 + (y - b ) y 2 + y12 = 0


plane is given by ax + by = 1, where b ¹ 0 and a Î R. æ 1 + y12 ö
Þ (y - b ) = - ç ÷ …(iii)
Differentiating both the sides w.r.t. x, we get è y2 ø
a+b
dy
=0 (as b ¹ 0 and a Î R) Putting (y - b ) in Eq. (ii), we get
dx (1 + y12 ) y1
( x - a) = …(iv)
Again, differentiating both the sides w.r.t. x, we get y2
d 2y
b =0 (as b ¹ 0 and a Î R) From Eqs. (i), (iii) and (iv), we get
dx 2
(1 + y12 )2 × y12 (1 + y12 )2
d 2y + =r2
Þ =0 (as b ¹ 0) y 22 y 22
dx 2
Þ (1 + (y1 )2 )3 = r 2y 22
\ Differential equation of all non-vertical lines in a plane.
Hence, (c) is the correct answer.
d 2y
Þ 2
=0
dx y Example 12 The differential equations of all circles
Hence, (a) is the correct answer. touching the x-axis at origin is
y Example 10 The differential equation of all æ dy ö
(a) (y 2 - x 2 ) = 2xy ç ÷
straight lines which are at a constant distance p è dx ø
from the origin, is dy
2 2
(b) (x 2 - y 2 ) = 2xy
(a) (y + xy 1 ) = p (1 + y 12 ) (b) (y - xy 12 ) 2
= p (1 + y 1 ) 2 dx
2
(c) (y - xy 1 ) = p (1 + 2
y 12 ) (d) None of these æ dy ö
(c) (x 2 - y 2 ) = 2xy ç ÷
è dx ø
Sol. As, we know x cos a + y sin a = p …(i)
Represents the family of straight lines which are at a (d) None of the above
constant distance p from origin. Differentiating Eq. (i) w.r.t. Sol. The equation of circle touches x-axis at origin.
x, we get
Þ ( x - 0) 2 + ( y - a ) 2 = a 2
dy
cos a + sin a × =0 or x 2 + y 2 - 2ay = 0 …(i)
dx
Chap 04 Differential Equations 239

Differentiating w.r.t. x, we get y Example 14 The differential equation satisfying the


2x + 2y
dy
- 2a
dy
=0 x2 y2
dx dx curve 2 + 2 = 1, where l being arbitrary
a +l b +l
dy
Þ (a - y ) = x unknown, is
dx æ dy ö
x +y ç ÷ (a) (x + yy 1 ) (xy 1 - y ) = (a 2 - b 2 ) y 1
è dx ø
Þ a= …(ii) (b) (x + yy 1 ) (x - yy 1 ) = y 1
æ dy ö
ç ÷ (c) (x - yy 1 ) (xy 1 + y ) = (a 2 - b 2 ) y 1
è dx ø
From Eqs. (i) and (ii), we get (d) None of these
é æ dy ö ù x2 y2
+ =1
ê x + y çè dx ÷ø ú
Sol. Here, 2 2
…(i)
a +l b +l
x 2 + y 2 - 2y ê ú =0
ê æ dy ö ú Differentiating both the sides, we get
ç ÷
êë è dx ø úû 2x 2y dy
+ × =0
dy a 2 + l b 2 + l dx
or (x 2 - y 2 ) = 2xy
dx Þ x (b 2 + l ) + y (a 2 + l ) y 1 = 0
Hence, (b) is the correct answer.
æ xb 2 + a 2yy1 ö
Þ l=-ç ÷
y Example 13 The differential equation of all circles in è x + yy1 ø
the first quadrant which touch the coordinate axes is
xb 2 + a 2yy1
(a) (x - y ) 2 (1 + (y ¢ ) 2 ) = (x + yy ¢ ) 2 \ a2 + l = a2 -
x + yy1
(b) (x + y ) 2 (1 + (y ¢ ) 2 ) = (x + y ¢ ) 2
(a 2 - b 2 ) x
(c) (x - y ) 2 (1 + y ¢ ) = (x + yy ¢ ) 2 Þ a2 + l = …(ii)
x + yy1
(d) None of these (a 2 - b 2 ) yy1
Also, b2 + l = …(iii)
Sol. Equation of circles touching coordinate axes is x + yy1
( x - a )2 + (y - a )2 = a 2 …(i) From Eqs. (i), (ii) and (iii), we get
Differentiating, we get
x 2 ( x + yy1 ) y 2 ( x + yy1 )
2 ( x - a ) + 2 (y - a ) y ¢ = 0 2 2
+ =1
(a - b ) x (a 2 - b 2 ) yy1
x + yy ¢ dy
Þ a= ; where y ¢ = …(ii)
1 + y¢ dx Þ ( x + yy1 ) ( xy1 - y ) = (a 2 - b 2 ) y1
y Hence, (a) is the correct answer.

y Example 15 The differential equation of all conics


whose centre lies at origin, is given by
(0, a) (a , a ) (a) (3 xy 2 + x 2 y 3 ) (y - xy 1 ) = 3 xy 2 (y - xy 1 - x 2 y 2 )
(b) (3 xy 1 + x 2 y 2 ) (y 1 - xy 3 ) = 3 xy 1 (y - xy 2 - x 2 y 3 )

O (a, 0)
x (c) (3 xy 2 + x 2 y 3 ) (y 1 - xy ) = 3 xy 1 (y - xy 1 - x 2 y 2 )
(d) None of the above
From Eqs. (i) and (ii), Sol. Equation of all conics whose centre lies at origin, is
2 2 2
æ x + yy ¢ ö æ x + yy ¢ ö æ x + yy ¢ ö ax 2 + 2hxy + by 2 = 1 …(i)
çx - ÷ + çy - ÷ =ç ÷
è 1 + y¢ ø è 1 + y¢ ø è 1 + y¢ ø Differentiating Eq. (i) w.r.t. x, we get
2 2 2 2ax + 2hxy1 + 2hy + 2byy1 = 0
æ xy ¢ - yy ¢ ö æy - x ö æ x + yy ¢ ö
Þ ç ÷ +ç ÷ =ç ÷ Þ ax + h (y + xy1 ) + byy1 = 0
è 1 + y¢ ø è1 + y ¢ ø è 1 + y¢ ø
Multiplying by x equation becomes,
Þ ( x - y )2 (y ¢ )2 + (y - x )2 = ( x + yy ¢ )2
ax 2 + h ( xy + x 2y1 ) + bxyy1 = 0 …(ii)
2 2 2
Þ ( x - y ) (1 + (y ¢ ) ) = ( x + yy ¢ ) Subtracting Eqs. (i) and (ii), we get
Hence, (a) is the correct answer. h ( xy - x 2y1 ) + b (y 2 - xyy1 ) = 1
240 Textbook of Integral Calculus

Þ (hx + by ) y - xy1 (hx + by ) = 1 y - xy1 - x 2y 2


b (y - xy1 ) =
Þ (hx + by ) (y - xy1 ) = 1 (y - xy1 )2
1
Þ hx + by = …(iii) y - xy1 - x 2y 2
y - xy1 Þ b= …(v)
(y - xy1 )3
Again, differentiating w.r.t. x, we get
Again, differentiating both the sides w.r.t. x, we get
(y - xy 2 - y1 )
h + by1 = - 1 y1 - y1 - 3xy 2 - x 2y 3 3 (y - xy1 - x 2y 2 ) xy 2
(y - xy1 )2 0= +
3
(y - xy1 ) (y - xy1 )4
xy 2
or h + by1 = …(iv)
(y - xy1 )2 Þ (3xy 2 + x 2y 3 ) (y - xy1 ) = 3xy 2 (y - xy1 - x 2y 2 )
From Eqs. (iii) and (iv), we get Hence, (a) is the correct answer.

Exercise for Session 1


1. The differential equation of all parabolas whose axis of symmetry is along X-axis is of order.
(a) 2 (b) 3 (c) 1 (d) None of these

2. The order and degree of the differential equation of all tangent lines to the parabola x 2 = 4y is
(a) 1, 2 (b) 2, 2 (c) 3, 1 (d) 4, 1
2
d y 2
æ dy ö æd y ö 2
3. The degree of the differential equation + 3 ç ÷ = x 2 log ç 2 ÷ is
è dx ø
dx 2 è dx ø
(a) 1 (b) 2 (c) 3 (d) Not defined

4. The degree of the defferential equation satisfying the relation 1+ x 2 + 1 + y 2 = l( x 1 + y 2 - y 1 + x 2 ) is


(a) 1 (b) 2 (c) 3 (d) 4
2 2
æd y ö æ dy ö
2æd y ö 2
5. The degree of the differential equation ç 2 ÷ + ç ÷ = x sin ç 2 ÷ is
è dx ø
è dx ø è dx ø
(a) 1 (b) 2 (c) 3 (d) Not defined

6. The differential equation of all circles touching the y-axis at origin, is


dy dx dy dx
(a) y 2 - x 2 = 2xy (b) y 2 - x 2 = 2xy (c) x 2 - y 2 = 2xy (d) x 2 - y 2 = 2xy
dx dy dx dy

7. The differential equation of all parabolas having their axes of symmetry coincident with the axes of x, is
(a) yy 2 + y12 = y + y1 (b) yy 2 + y12 = 0 (c) yy 2 + y12 = y1 (d) None of these

8. The differential equation of all conics whose axes coincide with the coordinate axes, is
(a) xyy 2 + xy12 - yy1 = 0 (b) yy 2 + y12 - yy1 = 0
(c) xyy 2 + (x - y ) y1 = 0 (d) None of these

9. The differential equation having y = (sin- 1 x )2 + A (cos - 1 x ) + B, where A and B are arbitrary constant, is
(a) (1 - x 2 ) y 2 - xy1 = 2 (b) (1 - x 2 ) y 2 + yy1 = 0
(c) (1 - x ) y 2 + xy1 = 0 (d) None of these

10. The differential equation of circles passing through the points of intersection of unit circle with centre at the
origin and the line bisecting the first quadrant, is
(a) y1 (x 2 + y 2 - 1) + (x + yy1 ) = 0 (b) (y1 - 1) (x 2 + y 2 - 1) + (x + yy1 ) 2 (x - y ) = 0
2 2
(c) (x + y - 1) + yy 2 = 0 (d) None of these
Session 2
Solving of Variable Seperable Form,
Homogeneous Differential Equation

Solving of Variable A differential equation of first order and first degree is of


Seperable Form the form
dy
dx
= f ( x , y ).

Solution of a Differential Equation Remark


The solution of the differential equation is a relation All the differential equations, even of first order and first degree,
between the variables of the equation not containing the cannot be solved. However, if they belong to any of the standard
derivatives, but satisfying the given differential equation forms which we are going to discuss, in the subsequent articles
they can be solved.
(i.e from which the given differential equation can be
derived).
Thus, the solution of
dy
= e x could be obtained by simply Equations in Which the
dx
integrating both the sides, i.e. y = e x + C and that of,
Variables are Separable
dy
dy x2 The equation = f ( x , y ) is said to be in variables separable
= px + q is y = p + qx + C , where C is arbitrary dx
dx 2 form, if we can express it in the form f ( x ) dx = g(y ) dy .
constant.
By integrating this, solution of the equation is obtained
(i) A general solution or an integral of a differential
which is, ò f ( x ) dx = ò g (y ) dy + C
equation is a relation between the variables (not
involving the derivatives) which contains the same
number of the arbitrary constants as the order of the y Example 16 Solve
differential equation. For example, a general solution sec 2 x tan y dx + sec 2 y tan x dy = 0.
d 2x
of the differential equation = - 4 x is Sol. Dividing the given equation by tan x tan y , we get
dt 2
x = A cos 2t + B sin 2t , where A and B are the sec 2 x sec 2 y
dx + dy = 0
arbitrary constants. tan x tan y
(ii) Particular solution or particular integral is that This is variable-separable type
solution of the differential equation obtained from the sec 2 x sec 2 y
Integrating, ò dx + ò tan y dy = C ¢
general solution by assigning particular values to the tan x
arbitrary constant in the general solution. ln | tan x | + ln | tan y | = ln C ; where C ¢ = ln C
For example, x = 10 cot 2t + 5 sin 2t is a particular or ln | tan x × tan y | = ln C ; (C > 0)
d 2x Þ | tan x × tan y | = C
solution of differential equation = - 4x .
dt 2 This is the general solution.
dy
y Example 17 Solve =ex -y + x2 e-y.
Differential Equations of the dx
First Order and First Degree Sol. Here,
dy e x x 2
= + Þ e y dy = ( x 2 + e x ) dx
dx e y e y
In this section we shall discuss the differential equations
which are of first order and first degree only. This is variable-separable form,
242 Textbook of Integral Calculus

\ Integrating both the sides, dy


\ = ln ( x + 1)
3 dx
x
òe ò ( x 2 + e x ) dx Þ e y =
y
dy = + ex + C
3 \ òdy = ò ln ( x + 1) dx (integration by parts)
Which is the general solution of the given differential x
equation, where C is an arbitrary constant. i.e. y = x ln ( x + 1) - ò x + 1 dx
dy i.e. y = x ln ( x + 1) - x + ln ( x + 1) + C
y Example 18 Solve 1 + x 2 + y 2 + x 2 y 2 + xy = 0.
dx This is the general solution.
Sol. The given differential equation can be written as To find the particular solution, put x = 0, y = 3 in the
dy general equation.
(1 + y 2 ) (1 + x 2 ) = - xy
dx \ 3=0-0+0+C
1 + x 2 dx y dy \ C =3
Þ - =
\ The required particular solution is,
x 1 + y2
y = ( x + 1) ln ( x + 1) - x + 3
This is the variable-separable form.
\ Integrating both the sides, we get
1 + x2 y
Differential Equations Reducible
- ò x
dx = ò 1 + y2
dy
to the Separable Variable Type
Sometimes differential equation of the first order cannot
é 1 æ 1 + x 2 - 1öù
Þ - ê 1 + x 2 + log ç ÷ú = 1 + y 2 + C be solved directly by variable separation but by some
êë 2 ç 2 ÷
è 1 + x + 1 ø úû substitution we can reduce it to a differential equation
with separable variable. ‘‘A differential equation of the
This is the general solution to the given differential
equation. dy
form = f (ax + by + c ) is solved by writing
dx
dy æ dy ö ax + by + c = t .’’
y Example 19 Solve y - x = a çy 2 + ÷ ×
dx è dx ø
dy
Sol. Rewriting the given equation as y Example 21 Solve = sin 2 ( x + 3y ) + 5.
dy dx
y - ay 2 = ( x + a ) 3dy dt
dx Sol. Let x + 3y = t , so that 1 + =
dx dx
dy dx
Þ = The given differential equation becomes,
y (1 - ay ) ( x + a )
1 æ dt ö 2 dt
This is the variable-separable form. ç - 1÷ = sin (t ) + 5 Þ = 3 sin 2 t + 16
3 è dx ø dx
Integrating both the sides, we get
dt
dy dx
ò y (1 - ay ) = ò ( x + a )
Þ ò 3 sin 2 t + 16 = ò dx
sec 2 t dt
Þ ò
æ1
ç +
a ö
÷ dy = ò
dx Þ ò 3 tan 2 t + 16 sec 2 t = x +C
è y 1 - ay ø x +a
sec 2 t dt
Þ
a
ln y - ln (1 - ay ) + ln C = ln (a + x )
Þ ò 19 tan 2 t + 16 = x + C
a
æ (a + x ) (1 - ay ) ö du
Þ ln ç ÷ = ln (C )
Þ ò 19u 2 + 16 = x + C ; where tan t = u
è y ø
or Cy = (a + x ) (1 - ay ) is the general solution. Þ sec 2 t dt = du
1 du 1 19 æ 19u ö
y Example 20 Solve e dy /dx = x + 1, given that when Þ ò Þ tan -1 ç ÷ +C
19 u 2 + 16 19 4 è 4 ø
x = 0, y = 3. 19
Sol. This is an Example of particular solution. 1 ì -1 æ 19 öü
Þ í tan ç tan (3y + x )÷ý + C
e (dy /dx ) = x + 1 4 19 î è 4 øþ
Chap 04 Differential Equations 243

dy x dx + y dy a2 - x 2 - y 2
y Example 22 Solve ( x + y ) 2 = a2. y Example 24 Solve = .
dx x dy - y dx x2 + y 2
dy
Sol. ( x + y )2 = a2 …(i) Sol. Let x = r cos q , y = r sin q
dx
dy dt dy æ dt ö So that x2 + y2 = r 2 …(i)
Put x + y = t Þ 1 + = or =ç - 1÷
dx dx dx è dx ø y
and tan q = …(ii)
ì dt ü x
\ Eq. (i) reduces to t 2 í - 1ý = a 2
î dx þ From Eq. (i), we have d ( x 2 + y 2 ) = d (r 2 )
2 dt 2 2
i.e. t = a + t , separating the variable and integrating. i.e. x dx + y dy = r dr …(iii)
dx
t2 æ a2 ö æ y ö
From Eq. (ii), we have d ç ÷ = d (tan q )
ò dx = ò a 2 + t 2 dt = ò çè1 - a 2 + t 2 ÷ø dt è x ø

æt ö x dy - y dx
\ x = t - a tan -1 ç ÷ + C i.e. = sec 2 q dq
èa ø x2
æx + yö i.e. x dy - y dx = x 2 sec 2 q dq = r 2 cos 2 q sec 2 q dq …(iv)
i.e. x = x + y - a tan - 1 ç ÷ +C
è a ø Using Eqs. (iii) and (iv) in the given equation, we get
æx + yö
i.e. y = a tan -1 ç ÷ - C is the required general solution. r dr a2 - r 2 dr
è a ø = i.e. = dq
2 2
r dq r a -r2
2

y Example 23 Solve æ r ö
i.e. sin -1 ç ÷ = q + C or r = a sin (q + C )
(2x + 3y - 1) dx + (4 x + 6 y - 5) dy = 0. è a ø
Sol. (2x + 3y - 1) dx + ( 4 x + 6y - 5) dy = 0 …(i) or x 2 + y 2 = a sin {C + tan - 1 (y / x )}
Substitute u = 2x + 3y - 1 It is advised to remember the results (iii) and (iv).
du 3dy dy 1 æ du ö
\ =2+ or = ç - 2÷
dx dx dx 3 è dx ø
\ Eq. (i) reduces to
1 æ du ö
Homogeneous
u + (2u - 3) ç
3 è dx
- 2÷ = 0
ø Differential Equation
2 1 du By definition, a homogeneouos function f ( x , y ) of degree
i.e. u - (2u - 3) + (2u - 3) =0
3 3 dx n satisfies the property
-u +6 1 du
i.e. + (2u - 3) =0 f ( lx , ly ) = ln f ( x , y )
3 3 dx
Writing this in the variable-separable form For example, the functions
æ 2u - 3 ö f 1 (x , y ) = x 3 + y 3
ç ÷ du = dx
è u -6 ø
f 2 ( x , y ) = x 2 + xy + y 2
2u - 3
\ ò dx = ò u - 6 du f 3 ( x , y ) = x 3 e x /y + xy 2
2 ( u - 6) + 9 are all homogeneous functions, of degrees three, two and
\ x +C = ò du
( u - 6) three respectively (verify this assertion).
\ x + C = 2u + 9 ln | u - 6 | Observe that any homogeneous function f ( x , y ) of degree
\ x + C = 2 (2x + 3y - 1) + 9 ln | 2x + 3y - 7 | n can be equivalnetly written as follows :
3x + 6y - 2 + 9 ln | 2x + 3y - 7 | = C is the general solution. æy ö æxö
f (x , y ) = x n f ç ÷ = y n f ç ÷
èxø èy ø
Remarks
Sometimes transformation to the polar coordinates facilitates For example, f (x, y ) = x 3 + y 3
separation of varibales. It is convenient to remember the
following differentials. æ æy ö3ö æ æx ö3ö
= x 1+ ç ÷ ÷ =y3
3ç ç1 + ç ÷ ÷
1. x dx + y dy = r dr 2. x dy - y dx = r 2 dq ç èxø ÷ ç èy ø ÷
è ø è ø
3. dx 2 + dy 2 = dr 2 + r 2d q2
244 Textbook of Integral Calculus

Having seen homogeneous functions we define Step III Shift v on R.H.S and seperate the variables in v
homogeneous DEs as follows : and x.
Any DE of the form M ( x , y ) dx + N ( x , y )dy = 0 Step IV Integrate both sides to obtain the solution in
dy M(x , y ) terms of v and x.
or =- is called homogeneous if M ( x , y ) and
dx N (x , y ) y
Step V Replace v by in the solution obtained in step IV
N ( x , y ) are homogeneous functions of the same degree. x
to obtain the solution in terms of x and y.
What is so special about homogeneous DEs? Well, it turns
out that they areextremely simple to solve. To see how, Following examples illustrate the procedure.
æy ö
we express both M ( x , y ) and N ( x , y ) as, say x n M ç ÷ and y Example 25 Solve y dx + (2 xy - x ) dy = 0.
èxø
æy ö Sol. y dx + (2 xy - x ) dy = 0 …(i)
x n N ç ÷ . This can be done sicne M ( x , y ) and N ( x , y ) are
èxø This is homogeneous type. Substitute y = ux
both homogeneous function of degree n. Doing this dy x du
\ =u +
reduces our DE to dx dx
æy ö æy ö \ Equation ux dx + (2 x 2 u - x ) (u dx + x du ) = 0
xnM ç ÷ Mç ÷
dy M(x , y ) èxø èxø æy ö i.e. x × { udx + (2 u - 1) u dx + x du (2 u - 1) } = 0
=- = =- =P ç ÷
dx N (x , y ) æy ö æy ö èxø
xnN ç ÷ N ç ÷ i.e. dx (2u 3 / 2 - u + u ) + x du (2 u - 1) = 0
èxø èxø
dx æ 2 u - 1 ö
- M (t ) Separating the variables, +ç ÷ du = 0
(The function P (t ) stands for ) x è 2 u 3/2 ø
N (t )
1
Now, the simple substitution y = vx reduces this DE to a Integrating both the sides ln | x | + ln | u | + =C
u
VS form
1 x æ yö
y = vx or ln | xu | + =C or ln | y | + = C çQu = ÷
u y è xø
dy dy
Þ =v + x Which is the general solution.
dx dx
dy æy ö y Example 26 Solve ( x 2 + y 2 ) dx - 2xy dy = 0.
Thus, = p ç ÷ transforms to
dx èxø dy x 2 + y 2 1 é x y ù
Sol. Here, = = ê + ú
dv dx 2xy 2ë y x û
v +x = P (v )
dx dy du
dv dx With y = ux , =u + x , so that the differential
Þ = dx dx
P (v ) - v x equation becomes
This can now be integraed directly since it is in VS form. du 1 æ 1 ö
u+x = ç + u÷
Let us see some examples of solving homogeneous DEs. dx 2 è u ø
x du 1 + u 2
Þ = -u
dx 2u
Alogorithm for Solving x du 1 - u 2
Þ =
Homogeneous Differential Equation dx 2u
2u dx
Step I Put the differential equation in the form Þ ò 1-u 2
du = ò
x
dy f ( x , y )
=
dx y ( x , y ) Þ - log | 1 - u 2 | = log | x | - log | C |
dy dv Þ x (1 - u 2 ) = C
Step II Put y = vx and =v + x in the equation in
dx dx x2 - y2 æ yö
Þ =C çQ u = ÷
step I and can out x from the right hand side. The x è xø
dv
equation reduces to the form v + x = f (v ). Hence, x 2 - y 2 = xC , is the required solution.
dx
Chap 04 Differential Equations 245

2 dy y y2 Sol. Since, xy = v , y =
v
and d ( x y ) = d v
y Example 27 Solve = + × x
dx x x2 i.e. x dy + y dx = dv
Sol. The above equation is homogeneous so that we put y = ux . æ v ö x dv - v dx
and dy = d ç ÷=
é du ù du è x ø x2
Þ 2 êu + x = u + u 2 Þ 2u + 2x = u + u2
ë dx úû dx v
i.e. x dy = dv - dx
du du dx x
Þ 2x = u2 - u Þ 2 =
dx u - u 2x v ì v ü
\ f (v ) dx + g (v ) í dv - dx ý = 0
du 1 dx
x î x þ
Þ ò u ( u - 1) = 2 ò x \
v { f (v ) - g (v )}
dx + g (v ) dv = 0
x
1 1 1 dx
Þ ò u - 1 du - ò u du = 2 ò x i.e.
dx
+
g (v ) dv
=0
x v { f (v ) - g (v )}
1
Þ log | u - 1 | - log | u | = log | x | + log | C | Which is in variables separable form.
2
u - 1½
Þ log½ = log | C x|
½ u ½ Reducible to Homogeneous Form
u -1 y-x
Þ =C x Þ =C x Type I
u y
Many a times, the DE specified may not be homogeneous
Þ y - x =C x ×y but some suibtale manipulation might reduce it to a
Which is the required solution. homogeneous form. Generally, such equations involve a
function of a rational expression whose numerator and
y Example 28 Solve denominator are linear functions of the variable, i.e., of
(1 + 2 e x / y ) dx + 2 e x / y (1 - x / y ) dy = 0. the form
dy æ ax + by + c ö
Sol. The appearance of x / y in the equation suggests the =f ç ÷ …(i)
substitution x = vy or dx = v dy + y dv. dx è dx + cy + f ø
\ The given equation is Note that the presence of the constant c and f causes this
(1 + 2 e v ) (v dy + y d v ) + 2 e v (1 - v ) dy = 0 DE to be non-homogeneous.
i.e. y (1 + 2 e v ) d v + (v + 2 e v ) dy = 0 To make it homogeneous, we use the substitutions
x ® X +h
1 + 2 ev dy
i.e. dv + =0 y ®Y + k
v + 2 ev y
and select h and k so that
1 + 2 ev dy
Integrating, ò v + 2 ev
dv + ò y
=0 ah + bk + c = 0 ü
…(ii)
ý
dh + ek + f = 0þ
log | v + 2 e v | + log | y | = log | C |
æ a dö
æ xö This can always be done ç if ¹ ÷ . The RHS of the DE in
Þ (v + 2 e v ) y = C çv = ÷ è b eø
è yø
æ a ( X + h ) + b (Y + k ) + c ö
æx x /y ö
(i) now reduces to = f ç ÷
Þ ç + 2e ÷ y = C è d ( X + h ) + e (Y + k + f ) ø
èy ø
æ aX + bY ö
Þ ( x + 2ye x / y ) = C , is required solution. =f ç ÷ (Using Eq. (ii))
è dX + eY ø
y Example 29 Show that any equation of the form This expression is clearly homogeneous! The LHS of Eq. (i)
y f ( xy ) dx + x g ( xy ) dy = 0 dy dy dY dX dy dx
is which equals × × . Since × = 1, the
dx dY dX dx dY dX
can be converted to variable separable form by substi- dy dY
LHS equals . Thus, our equation becomes
tuting xy = v . dx dX
246 Textbook of Integral Calculus

dY æ aX + bY ö Thus,
=f ç ÷ …(iii)
dX è dX + eY ø ax + by + c l(dx + ey ) + c
=
We have thus succeeded in transforming the dx + ey + f dx + ey + f
non-homogeneous DE in Eq. (i) to the homogeneous DE in This suggests the substitution dx + ey = v , which will give
Eq. (iii). This can now be solved as described earlier. dy dv
d +e =
dy 2y - x - 4 dx dx
y Example 30 Solve the DE = × dy 1 æ dv ö
dx y - 3x + 3 Þ = ç -d÷
dx e è dx ø
Sol. We substitute x ® X + h and y ® Y + k where h , k need
to be determined Thus, our DE reduces to
dy dY (2Y - X ) + (2k - h - 4 ) 1 æ dv ö lv + c
= = ç -d÷ =
dx dX (Y - 3X ) + (k - 3h + 3) e è dx ø v+f
h and k must be chosen so that dv lev + ec
Þ = +d
2k - h - 4 = 0 dx v+f
k - 3h + 3 = 0 ( le + d )v + (ec + d )
This gives h = 2 and k = 3. Thus, =
v+f
x = X +2
(v + f )
y =Y + 3 Þ dv = dx
( le + d ) v + ec + df
Our DE now reduces to
dY 2Y - X which is in VS form and hence can be solved.
=
dX Y - 3X dy x + 2y - 1
y Example 31 Solve the DE = ×
Using the substitution Y = vX , and simplifying, we have dx x + 2y + 1
(verify),
Sol. Note that h , k do not exist in this case which can reduce
v -3 -dX
2
dv = this DE to homogeneous form. Thus, we use the substitu-
v 5v + 1 X
tion
We now integrate this DE which is VS; the left-hand side x + 2y = v
can be integrated by the techniques described in the unit of dy dv
Indefinite Integration. Þ 1+2 =
dx dx
Y
Finally, we substitute v = and Thus, our DE becomes
X
1 æ dv ö v -1
X = x -2 ç - 1÷ =
2 è dx ø v +1
Y =y -3
dv 2v - 2 3v - 1
to obtain the general solution. Þ = + 1=
dx v +1 v +1
Type II v +1
Þ dv = dx
Suppose our DE is of the form 3v - 1
dy æ ax + by + c ö Þ
1æ 4 ö
=f ç ÷ ç1 + ÷ dv = dx
dx è dx + ey + f ø 3 è 3v - 1 ø
Integrating, we have
We try to find h, k so that
1æ 4 ö
ah + bk + c = 0 çv + ln (3v - 1)÷ = x + C 1
3 è 3 ø
dh + ek + f = 0
Substituting v = x + 2y , we have
What if this system does not yield a solution? Recall that
4
a d x + 2y + ln (3x + 6y - 1) = 3x + C 2
this will happen if = . How do we reduce the DE to a 3
b e
2
homogeneous one in such a case? Þ y - x + ln (3x + 6y - 1) = C
3
a b
Let = = l (say).
d e
Chap 04 Differential Equations 247

1
y Example 32 The solution of the differential ± y + y 2 + 2x 2
(a) 2 x 2 =
dy sin y + x x
equation = is
dx sin 2y - x cos y ± 2 y + y + 2 x2
2
(b) 2x =
x
x2
(a) sin 2 y = x sin y + +C ±
1
y + x 2 + 2y 2
2 (c) 2 y 2 =
x2 x
(b) sin 2 y = x sin y - +C
2 (d) None of the above
x2
(c) sin 2 y = x + sin y + +C Sol. The given differential equation can be written as
2 æ æ dy ö ö
2 2
x2 æ dy ö dy
(d) sin 2 y = x - sin y + +C y 2 ç ÷ + 4 x 2 + 4 xy × = (y 2 + 2x 2 ) çç1 + ç ÷ ÷÷
è dx ø dx è è dx ø ø
2
dy sin y + x 2
Sol. Here, = dy y 1 æy ö
dx sin 2y - x cos y Þ = ± ç ÷ +1 …(i)
dx x 2 èx ø
dy sin y + x
Þ cos y = , put sin y = t Let y = vx
dx 2 sin y - x
dv dy
dt t +x Þ v+x =
Þ = , put t = vx dx dx
dx 2 t - x
\ Eq. (i) becomes
x dv vx + x v +1
+v = = dv 1 2
dx 2vx - x 2v - 1 v+x =v ± v +1
dx 2
dv v +1 v + 1 - 2v 2 + v dv dx
\ = -v =
x
dx 2v - 1 2v - 1
or ò 1 2

x
v +1
2v 2 - v dx 2
or 2
dv =
- 2v + 2v + 1 x
Þ 2 log | v + v 2 + 2 | = log | xC |
On solving, we get
½ y + y 2 + 2x 2 ½
x2 Þ 2 log½ ½ = log | xC |,
sin 2 y = x sin y + +C ½ x ½
2
putting x = 1 and y = 0
Hence, (a) is the correct answer.
2
Þ C = ( 2)
y Example 33 The equation of curve passing through
1
(1, 0) and satisfying y + y 2 + 2x 2 ±
\Curves are given by = 2x 2
æ dy ö
2
2 ç
æ æ dy ö 2 ö x
+ = 2
+ ÷
ç y
è dx
2 x ÷
ø
( y 2 x ) ç 1 + çè dx ÷ø ÷ , is given by Hence, (a) is the correct answer.
è ø
248 Textbook of Integral Calculus

Exercise for Session 2


dy ( x + y )2
1. The solution of = , is given by
dx ( x + 2) ( y - 2)
2( y - 2)
æ 2 (y - 2) ö
(a) (x + 2)4 æç 1 +
2y ö 2y / x + 2)
÷ = ke (b) (x + 2)4 ç 1 + ÷ =ke
(x
è x ø è (x + 2) ø
2( y - 2)
æ (y - 2) ö x + 2
(c) (x + 2)3 ç 1 + 2 ÷ = ke (d) None of these
è x+ 2 ø

2. If ( y 3 - 2x 2y ) dx + (2xy 2 - x 3 ) dy = 0, then the value of xy y 2 - x 2 , is


(a) y 2 + x (b) xy 2
(c) any constant (d) None of these

3. The solution of dy / dx = cos ( x + y ) + sin ( x + y ), is given by


x + y ö½
½ 1 + tan æç
(a) log½ ÷ ½= x + C (b) log | 1+ tan (x + y ) | = x + C
½ è 2 ø½
(c) log | 1- tan (x + y ) | = x + C (d) None of these
dy x +y
4. The solution of = ( x + y - 1) + , is given by
dx log ( x + y )
(a) {1 + log (x + y ) } - log {1 + log (x + y ) } = x + C (b) {1 - log (x + y ) } - log {1 - log (x + y ) } = x + C
(c) {1 + log (x + y ) }2 - log {1 + log (x + y ) } = x + C (d) None of these

5. The solution of (2x 2 + 3y 2 - 7) xdx - (3x 2 + 2y 2 - 8) ydy = 0, is given by


(a) (x 2 + y 2 - 1) = (x 2 + y 2 - 3)5 C (b) (x 2 + y 2 - 1)2 = (x 2 + y 2 - 3)5 C
2 2 2 2 5
(c) (x + y - 3) = (x + y - 1) C (d) None of these
æ y - 2ö
( x - 1)2 + ( y - 2)2 tan-1 ç ÷
dy è x - 1ø
6. The solution of = , is equal to
dx -1 æ y - 2ö
( xy - 2x - y + 2) tan ç ÷
è x - 1ø
æ y-2ö
(a) {(x - 1)2 + (y - 1)2 } tan-1 ç 2
÷ - 2 (x - 1) (y - 2) = 2 (x - 1) log C (x - 1)
è x-1ø
æ y-2ö
(b) {(x - 1)2 + (y - 1)2 } - 2 (x - 1) (y - 2) tan- 1 ç 2
÷ = 2 (x - 1) log C
è x-1ø
æ y-2ö
(c) {(x - 1)2 + (y - 1)2 } tan- 1 ç ÷ - 2 (x - 1) (y - 2) = log C (x - 1)
è x-1ø
(d) None of the above
2
dy æ x + 2y - 3 ö
7. The solution of =ç ÷ , is
dx è 2x + y + 3 ø
(a) (x + 3)3 - (y - 3)3 = C (x - y + 6)4 (b) (x + 3)3 - (y - 3)3 = C
(c) (x + 3)4 + (y - 3)4 = C (d) None of these
dy - cos x (3 cos y - 7 sin x - 3)
8. The solution of = , is
dx sin y (3 sin x - 7 cos y + 7)
(a) (cos y - sin x - 1)2 (sin x + cos y - 1)5 = C (b) (cos y - sin x - 1)2 (sin x + cos y - 1)3 = C
(c) (cos y - sin x - 1)2 (sin x + cos y - 1)7 = C (d) None of these
Chap 04 Differential Equations 249

9. A curve C has the property that if the tangent drawn at any point P on C meets. The coordinate axes at A and B,
then P is the mid point of AB. The curve passes through the point (1, 1). Then the equation of curve is
x
(a) xy = 1 (b) =1
y
(c) 2x = xy - 1 (d) None of these
p
10. The family of curves whose tangent form an angle with the hyperbola xy = 1, is
4
(a) y = x - 2 tan-1 (x ) + K (b) y = x + 2 tan-1 (x ) + K
-1
(c) y = 2x - tan (x ) + K (d) y = 2x + tan-1 (x ) + K

11. A and B are two separate reservoires of mater capacity of reservoir are filled completely with water their inlets
are closed and then the water is released simultaneously from both the reservoirs. The rate of flow of water out
of each reservoir at any instant of time is proportional to the quantity of water in the reservoir at that time. One
1
hour after the water is released, the quantity of water in the reservoir A is 1 times the quantity of the water in
2
reservoir B. The time after which do both the reservoirs have the same quantity of water, is
(b) log3 / 4 æç ö÷
1
(a) log3 / 4 (2)
è 2ø

(c) log1/ 2 æç ö÷
1
(d) None of these
è 2ø

12. A curve passes through (2, 1) and is such that the square of the ordinate is twice the rectangle contained by the
abscissa and the intercept of the normal. Then the equation of curve is
(a) x 2 + y 2 = 9x (b) 4x 2 + y 2 = 9x
(c) 4x 2 + 2y 2 = 9x (d) None of these

x (1+ y 2 )
13. A normal at P ( x , y ) on a curve neets the X-axis at Q and N is the foot of the ordinate at P. If NQ = .
(1+ x 2 )
Then the equation of curve passing through (3, 1) is
(a) 5(1+ y 2 ) = (1+ x 2 ) (b) (1+ y 2 ) = 5(1+ x 2 )
(c) (1+ x 2 ) = (1+ y 2 ). x (d) None of these

14. The curve for which the ratio of the length of the segment intercepted by any tangent on the Y-axis to the length
of the radius vector is constant (k), is
(a) (y + x 2 - y 2 )x k -1 = c (b) (y + x 2 + y 2 )x k -1 = c
(c) (y - x 2 - y 2 )x k -1 = c (d) (y - x 2 + y 2 )x k -1 = c

15. A point P ( x , y ) nores on the curve x 2/ 3 + y 2/ 3 = a 2/ 3, a > 0 for each position ( x , y ) of p, perpendiculars are drawn
from origin upon the tangent and normal at P, the length (absolute valve) of them being P1( x ) and P2( x )
brespectively, then
dp1 dp2 dp1 dp2
(a) . <0 (b) . £0
dx dx dx dx
dp dp dp dp
(c) 1 . 2 > 0 (d) 1 . 2 ³ 0
dx dx dx dx
Session 3
Solving of Linear Differential Equations,
Bernoulli’s Equation, Orthogonal Trajectory

Solving of Linear Step II Find integrating factor (I.F.) given by I.F. = e ò .


Pdx

Differential Equations Step III Multiply both sides of equation in Step I by I.F.
Step IV Integrate both sides of the equation obtained in
First Order Linear step III. w.r.t x to obtain y (I.F.) = ò Q .(I. F. ) dx + C
Differential Equations This gives the required solution following examples
illustrate the procedure.
A differential equation is said to be linear if an unknown
variable and its derivative occur only in the first degree. dy
y Example 34 Solve + 2y = cos x .
An equation of the form dx
dy Sol. It is a linear equation of the form
+ P ( x ) × y = Q ( x ).
dx dy
+ Py = Q ( x )
Where P ( x ) and Q ( x ) are functions of x only or constant dx
is called a linear equation of the first order. where P = 2 and Q = cos x
IF = e ò = eò
Pdx 2dx
To get the general solution of the above equation we Then = e 2x
proceeds as follows. By multiplying both the sides of the Hence, the general solution is y (IF) = ò Q (IF) dx
above equation by e ò , we get
Pdx
y × e 2x = òe
2x
i.e. cos x dx + C
Pdx dy
eò × + yP × e ò = Q e ò
Pdx Pdx 2x
e
dx y × e 2x = [2 cos x + sin x ] + C
5
dy d
i.e. e ò × (e ò ) = Q e ò
Pdx Pdx Pdx
+y dy y
dx dx y Example 35 Solve + = log x .
d dx x
(y e ò ) = Q × e ò
Pdx Pdx
i.e.
dx Sol. It is a linear differential equation of the form
dy
\ Integrating, we get y e ò = ò Q e ò dx + C
Pdx Pdx
+ Py = Q ( x )
dx
Here, the term e ò which converts the left hand 1
Pdx
Here, P = , Q = log x
expression of the equation into a perfect differential is x
IF = e ò = eò
Pdx 1/ x dx
called an Integrating factor. In short it is written as IF. Then = e log x = x
Thus, we remember the solution of the above equation as Hence, the general solution is
y ( IF ) = òQ (IF) dx + C. y (IF) = ò Q (IF) dx + C

i.e. yx = ò (log x ) x dx + C
Algoritm for Solving A Linear x2 1 x2
yx = (log x ) × -ò × +C
Differential Equation i.e.
2 x 2
Step I Write the differential equation in the form x2 x2
i.e. yx = (log x ) - +C
dy / dx + Py = Q and obtain P and Q. 2 4
Chap 04 Differential Equations 251

dy y p 3 3
y Example 36 Solve = × Now, when x= ,y =
dx 2y ln y + y - x 6 8
3 3 æ 1 ö 1 3
Sol. The equation can be written as \ ç 1- ÷ = × +C Þ C =0
8 è 3 ø 2 2
dx 2y ln y + y - x x
= = (2 ln y + 1) - sin 2x
dy y y \ y=
2 (1 - tan 2 x )
dx 1
i.e. + × x = (2 ln y + 1)
dy y dy
y Example 38 Solve + y f¢ ( x ) = f( x ) × f¢ ( x ), where
1 dx
In this equation it is clear that P = and Q = (2 ln y + 1).
y f( x ) is a given function.
Which are function of y only because equation contains Sol. Here, P = f¢( x ) and Q = f( x ) f¢ ( x )

IF = e ò
derivatives of x with f¢ ( x ) dx
= e f(x )
respect to y.
\ The solution is
IF = e ò = eò
Pdy 1/ y dy
= e ln y = y
y e f (x ) = òf( x ) × f¢ ( x ) × e f (x ) dx = ò t ×e
t
dt ,
\ The solution is; x (IF) = ò(2 ln y + 1) (IF) dy
where f( x ) = t
i.e. xy = ò(2 ln y + 1) × y dy = y ln y + C 2
y e f ( x ) = e t ( t - 1) + C
C i.e. ye f (x ) = { f( x ) - 1} e f (x ) + C
i.e. x = y ln y +
y
Note In some cases a linear differential equation may be of the
form
dx
dy
+ P1 x = Q1, where P1 and Q1 are function of y alone. In Bernoulli’s Equation
such a case the integrating factor is eò 1 .
P dy
Sometimes a differential equation is not linear but it can
dy be converted into a linear differential equation by a
y Example 37 Solve cos 2 x - y tan 2x = cos 4 x , suitable substitution. An equation of the form
dx dy
p æ p ö 3 3 + Py = Q y n . (n ¹ 0, 1)
where | x | < and y ç ÷ = × dx
4 è 6 ø 8
Where P and Q are functions of x only, is known as
Sol. The given equation can be written as Bernoulli’s equation (for n = 0 the equation is linear.)
dy It is easy to reduce the above equation into linear form as
- sec 2 x × tan 2x × y = cos 2 x
dx below :
2 tan x
ò+ sec 2 x dx
2
Dividing both the sides by y n , we get
IF = e ò
- tan 2 x × sec x dx tan 2 x - 1
=e
dt
òt dy
=e , where t = tan 2 x - 1 y -n + P y1 -n = Q
dx
= e ln | t | = | t | = | tan 2 x - 1 |
dy dz
p
It is given that | x | < and for this region tan 2 x < 1. Putting y 1 - n = z and hence, (1 - n ) y - n = the
4 dx dx
dz
\ IF = (1 - tan 2 x ) equation becomes + (1 - n ) Pz = (1 - n ) Q which is linear
dx
\ The solution is
in z.
y (1 - tan 2 x ) = ò cos 2 x (1 - tan 2 x ) dx
IF = e ò
(1 - n ) P dx
Here,
= ò(cos x - sin x ) dx
2 2
\ The solution is,
zeò
(1 - n ) Pdx (1 - n ) Pdx
= ò(cos 2x ) dx =
sin 2x
+C = ò {(1 - n ) × Q × e ò } dx
2
252 Textbook of Integral Calculus

y Example 39 Solve ( y log x - 1) y dx = x dy . dy y f¢ ( x ) - y 2


y Example 41 Solve = , where f( x ) is
Sol. The given differential equation can be written as dx f( x )
dy a given function.
x + y = y 2 log x …(i)
dx Sol. The equation can be written as
Dividing by xy 2 , hence dy f¢ ( x ) y2
- y=-
1 dy 1 1 dx f( x ) f( x )
× + = log x
y 2 dx xy x dy f¢ ( x ) 1 1
i.e. - y- 2 + × =
1 1 dy dv dx f( x ) y f( x )
Let =v Þ- 2× =
y y dx dx 1 1 dy dz
Let = z . So that, - 2 =
dv 1 1 y y dx dx
So that - v = - log x
dx x x dz f¢ ( x ) 1
\ + ×z =
Which is the standard linear differential equations, with dx f( x ) f( x )
1 1 f¢ ( x )
P = - , Q = - log x ò f (x ) dx
x x IF = e = e ln f (x ) = f( x )
1
IF = e ò
- 1/ x dx -1
= e - ln x = e ln x = 1
x
\ The solution is z × f( x ) = ò f( x ) × f( x ) dx = x + C
The solution is given by f( x ) f( x )
i.e. = x + C ie, =y
1 1 æ 1 ö log x y x +C
v × = ò ç - log x ÷ dx = - ò dx
x x è x ø x2
log x 1 1 log x 1 Remark
= - ò × dx = + +C Another type of equation which is reducible to the linear form is
x x x x x dy
f ¢( y ) + P( x ) × f ( y ) = Q( x )
Þ v = 1 + log x + Cx = log (ex ) + Cx dx
1 An equation of this type can be easily reduced the linear form by
or = log (ex ) + Cx or y {log (ex ) + Cx } = 1 taking z = f ( y ).
y
dy
dy y Example 42 Solve sec 2 y + 2x tan y = x 3 .
y Example 40 Solve + xy = xy 2 . dx
dx dy dz
Sol. Let tan y = z so that sec2 y × =
Sol. Dividing by y 2 , we get dx dx
dy 1 Thus, the given equation reduces to
y- 2 + ×x = x
dx y dz
+ 2x × z = x 3
1 dx
Let =z
IF = e ò
2 x dx 2
y ` = ex
1 dy dz 2 2
So that - × = \ The solution is, z × e x = òx 3 × e x dx
y 2 dx dx
2 1 2 x2 1

\ The given equation reduces to i.e. tan y × e x = x e × (2x ) dx = ò t × e t dt ,
2
dz
- xz = - x where t = x 2
dx
2 1
IF = e ò
- x dx
= e- x
2
/2 tan y × e x = (t × e t - e t ) + C
2
\ The solution is e- x
2
- x2 2
2 2 2 i.e. tan y = Ce + × e x ( x 2 - 1)
z e- x /2
= ò - x ×e- x /2
dx = e - x /2
+C 2
2 1 2
i.e.
1 2
= 1 + Ce x / 2 tan y = Ce - x + ( x - 1)
y 2
Chap 04 Differential Equations 253

dy
y Example 43 Solve
dx
+ x ( x + y ) = x 3 ( x + y ) 3 - 1. Orthogonal Trajectory
Sol. The given equation can be written as Any curve, which cuts every member of a given family of
æ dy ö curves at right angles, is called an orthogonal trajectory of
ç + 1 ÷ + x ( x + y ) = x 3 ( x + y )3
è dx ø the family. For example, each straight line passing through
d (x + y ) the origin, ie , y = kx is an orthogonal trajectory of the
i.e. + x ( x + y ) = x 3 ( x + y )3
dx family of the circles x 2 + y 2 = a 2 .
d (x + y )
i.e. ( x + y )- 3 × + x ( x + y )- 2 = x 3
dx Procedure for Finding the
Let ( x + y )- 2 = z so that - 2 ( x + y )- 3
d ( x + y ) dz
=
Orthogonal Trajectory
dx dx (i) Let f ( x , y , c ) = 0 be the equation of the given family of
The given equation reduces to curves, where c is an arbitrary parameter.
1 dz (ii) Differentiate f = 0; w.r.t. ‘x’ and eliminate ‘c’, i.e. form
- + xz = x 3
2 dx a differential equation.
dz
i.e. - 2xz = - 2x 3 dx dy
dx (iii) Substitute - for in the above differential
dy dx
IF = e ò
- 2 x dx 2
= e- x equation. This will give the differential equation of
\ The solution is the orthogonal trajectories.
2 2 2
z × e - x = ò - 2x 3 × e - x dx = ( x 2 + 1) e - x + C (iv) By solving this differential equation, we get the
required orthogonal trajectories.
1 2
= Ce x + x 2 + 1
(x + y ) 2
y Example 45 Find the orthogonal trajectories of the
hyperbola xy = C .
dy
y Example 44 Solve sin y × = cos y (1 - x cos y ). Sol. The equation of the given family of curves is xy = c …(i)
dx
Differentiating Eq. (i) w.r.t. x, we get
Sol. The given differential equation is
x dy
dy +y =0 …(ii)
sin y = cos y (1 - x cos y ) dx
dx dx dy
dy Substitute - for in Eq. (ii), we get
or sin y - cos y = - x cos 2 y dy dx
dx
x dx
Dividing by cos 2 y, we get - +y =0 …(iii)
dy
dy
tan y × sec y ×
- sec y = - x This is the differential equation for the orthogonal
dx trajectory of given family of hyperbola. Eq. (iii) can be
dy dv rewritten as x dx = y dy , which on integration gives
Let sec y = v Þ sec y tan y × =
dx dx x 2 - y 2 = C.
dv
So that -v = - x This is the family of required orthogonal trajectories.
dx
Which is linear differential equation with P = - 1, Q = - x y Example 46 Find the orthogonal trajectories of the
curves y = cx 2 .
IF = e ò = eò
Pdx - 1 dx
= e- x Sol. Here, y = cx 2 ...(i)
The solution is given by Differentiating w.r.t. x, we get
v × e - x = ò - x × e - x dx = xe - x + e - x + C dy
= 2cx ...(ii)
dx
= e - x ( x + 1) + C
Eliminating c from Eqs. (i) and (ii),
or v = (1 + x ) + Ce x
æ 1 dy ö 2
y=ç ÷x
or sec y = (1 + x ) + Ce x è 2x dx ø
254 Textbook of Integral Calculus

Þ
dy
2y = x ...(iii) æx2 ö
Þ - dy = d ç ÷
dx èy ø
This is the differential equation of the family of curves
given in Eq. (i). Integrating both the sides, we get
dy dx x2
Now, to obtain orthogonal trajectory replace by - in -y = + C Þ x 2 + y 2 + Cy = 0
dx dy y
Eq. (iii). Represents family of orthogonal trajectory.
dx
Þ 2y = - x y Example 48 Find the orthogonal trajectory of the
dy
circles
or 2y dy = - x dx
x 2 + y 2 - ay = 0.
Integrating both the sides, we get
x2 Sol. Here, x 2 + y 2 - ay = 0 ...(i)
2
y =- + C1
2 Differentiating, we get
2 2
Þ x + 2y = C 1, is the required family of orthogonal 2x + 2yy1 - ay1 = 0
trajectory. 2 ( x + yy1 )
Þ a= ...(ii)
y1
y Example 47 Find the equation of all possible curves
Substituting ‘a’ in Eq. (i), we get
that will cut each member of the family of circles
2 ( x + yy1 )
x 2 + y 2 - 2cx = 0 at right angle. x2 + y2 - y =0
y1
Sol. Here, x 2 + y 2 - 2cx = 0 ...(i)
Þ ( x 2 - y 2 ) y1 - 2xy = 0
Differentiating w.r.t. x, we get
This is the differential equation of the family of circles
2x + 2yy1 - 2c = 0
given in Eq. (i).
Þ c = x + yy1 ...(ii)
\ The differential representing the orthogonal trajectory is
From Eqs. (i) and (ii), we eliminate c
dy dy
Þ x 2 + y 2 - 2 ( x + yy1 ) x = 0 obtained by replacing by - .
dx dx
dy
or - x 2 + y 2 - 2xy =0 i.e. - (x 2 - y 2 )
dx
- 2xy = 0
dx dy
This is the differential equation representing the given
family of circles. To find differential equation of the Þ 2xy dy - y 2 dx = - x 2 dx
dy dx x d (y 2 ) - y 2 dx
orthogonal trajectories, we replace by - . Þ = - dx
dx dy x2
dx
Þ y 2 - x 2 = - 2xy æy 2 ö
dy Þ d ç ÷ = - dx
èx ø
Þ y 2 dy = x 2 dy - 2xy dx
Integrating both the sides, we get
yd ( x 2 ) - x 2 dy
Þ - dy = y 2 + x 2 = Cx , is required family of orthogonal trajectories.
y2
Chap 04 Differential Equations 255

Exercise for Session 3


dy -1
1. The solution of (1 + x 2 ) + y = e tan x , is given by
dx
-1 -1 -1 -1
(a) 2ye tan x
= e 2 tan x
+C (b) ye tan y
= e 2 tan x
+C
-1 -1
tan y 2 tan y
(c) 2ye =e +C (d) None of these
dy x
2. The solution of + y =x y , is given by
dx 1 - x 2
3
(a) 3 y + (1 - x 2 ) = C (1 - x 2 )1/ 4 (b) y + (1 - x 2 ) = C (1 - x 2 ) 3 / 2
2
(c) 3 y - (1 - x 2 ) = C (1 - x 2 )3 / 2 (d) None of these
dy
3. The solution of + x sin 2y = x 3 cos 2 y , is
dx
2 2 2 1 2 2
(a) e x = (x 2 - 1) e x tan y + C (b) e x tan y = (x - 1) e x + C
2
2
(c) e x tan y = (x 2 - 1) tan y + C (d) None of these

4. The solution of 3x (1 - x 2 ) y 2 dy / dx + (2x 2 - 1) y 3 = ax 3 is


(a) y 3 = ax + C 1- x 2 (b) y 3 = ax + Cx 1- x 2

(c) y 2 = ax + C 1- x 2 (d) None of these

dy y y
5. The solution of + log y = 2 (log y )2, is
dx x x
1 1 1
(a) x = log y + C (b) x 2 + log y = C (c) = +C (d) None of these
2x x log y 2x 2
dy
6. The solution of + y f ¢ ( x ) - f ( x ) × f ¢ ( x ) = 0, y ¹ f ( x ) is
dx
(a) y = f (x ) + 1+ ce - f ( x ) (b) y - ce - f ( x )
-f( x )
(c) y = f (x ) - 1+ ce (d) None of these

7. The solution of ( x 2-1) dy / dx × sin y - 2x × cos y = 2x - 2x 3, is


x4 x4
(a) (x 2 - 1) cos y = - x2 + C (b) (x 2 - 1) sin y = - x2 + C
2 2
x4 x 2 x4 x 2
(c) (x 2 - 1) cos y = - +C (d) (x 2 - 1) sin y = - +C
4 2 4 2

8. The Curve possessing the property text the intercept made by the tangent at any point of the curve on the
y-axis is equal to square of the abscissa of the point of tangency, is given by
(a) y 2 = x + C (b) y = 2x 2 + cx
2
(c) y = - x + cx (d) None of these

9. The tangent at a point P of a curve meets the y-axis at A, and the line parallel to y-axis at A, and the line
parallel to y-axis through P meets the x-axis at B. If area of DOAB is constant (O being the origin), Then the
curve is
(a) cx 2 - xy + k = 0 (b) x 2 + y 2 = cx

(c) 3x 2 + 4y 2 = k (d) xy - x 2 y 2 + kx = 0

10. The value of k such that the family of parabolas y = cx 2 + k is the orthogonal trajectory of the family of ellipse
x 2 + 2y 2 - y = C, is
(a) y 2 (b) y 3 (c) y4 (d) y 5
Session 4
Exact Differential Equations
Exact Differential Equations \ The solution is,

A differential equation of the form


ò (2x log y ) dx +
y - constant ò 3y 2 dy = C

M ( x , y ) dx + N ( x , y ) dy = 0 is said to be exact (or total) if Þ x 2 log y + y 3 = C


its left hand expression is the exact differential of some
function u ( x , y ). Equations Reducible
i.e. du = M × dx + N × dy
Hence, its solution is u ( x , y ) = c (where c is an arbitrary
to the Exact Form
constant). But then there is a question that how do we Sometimes a differential equation of the form
confirm whether the above mentioned equation is exact. M dx + N dy = 0 which is not exact can be reduced to an
The answer to this question is the following theorem. exact form by multiplying by a suitable function f ( x , y )
which is not identically zero. This function f ( x , y ) which
Theorem The necessary and sufficient condition for the then multiplied to a non-exact differential equation makes
differential equation M dx + N dy = 0 to be exact is it exact is known as integrating factor.
¶M ¶N One can find integrating factors by inspection but for that
= ×
¶y ¶x some experience and practice is required.
For finding the integrating factors by inspection, the
The solution of M dx + N dy = 0 is ,
following identities must be remembered.
ò M dx + ò(terms of N not containing x ) dy = C
y - constant 1. x dy + y dx = d ( xy )
¶M ¶N 1
provided = . 2. x dx + y dy = d ( x 2 + y 2 )
¶y ¶x 2
x dy - y dx æy ö
y Example 49 Solve ( x 2 - ay ) dx + ( y 2 - ax ) dy = 0. 3. =d ç ÷
x 2 èxø
Sol. Here, we have M = x 2 - ay and N = y 2 - ax y dx - x dy æxö
¶M ¶N
4. =d ç ÷
\ = -a= y 2 èy ø
¶y ¶x
x dy - y dx dy dx é æ y öù
Thus, the equation is exact. 5. = - =d ê log çè x ÷ø ú
xy y x ë û
\ The solution is, ò ( x 2 - ay ) dx + òy
2
dy = C
y - constant y dx - x dy é æ x öù
3 3
6. = d ê log ç ÷ ú
Þ
x
- axy +
y
=C
xy ë è y øû
3 3 x dy - y dx æy ö
dç ÷
æx2 ö x dy - y dx x 2 èxø é æ y öù
7. = = = d ê tan -1 ç ÷ ú
y Example 50 Solve (2x log y ) dx + ç + 3y 2 ÷ dy = 0. 2
x +y 2
y 2 2
ë è x øû
è y ø 1+ æy ö
1 + ç ÷
x2 èxø
x2 dx + dy
Sol. Here, we have M = 2x log y and N = + 3y 2 8. = d [ln ( x + y )]
y x +y
¶M 2 x x dy + y dx
\ = 9. d (ln ( xy )) =
¶y y xy
¶N 2 x æ1 ö x dx + y dy
and = and hence the equation is exact. 10. d ç ln ( x 2 + y 2 ) ÷ =
¶x y è2 ø x2 +y2
Chap 04 Differential Equations 257

æ 1 ö x dy + y dx y Example 54 Solve
11. d ç - ÷=
è xy ø x2 y2 y + sin x cos 2 ( xy ) æ x ö
dx + ç + sin y ÷ dy = 0.
æ e y ö x e y dy - e y dx 2
cos ( xy ) 2
è cos ( xy ) ø
12. d ç ÷ =
è x ø x2 Sol. The given differential equation can be written as
æ e ö ye dx - e dy
x x x y dx + x dy
+ sin x dx + sin y dy = 0
13. d ç ÷ = cos 2 ( xy )
èy ø y2
Þ sec 2 ( xy ) d ( xy ) + sin x dx + sin y dy = 0
14. d ( x m y n ) = x m - 1 y n - 1 (my dx + nx dy )
d (tan ( xy )) + d ( - cos x ) + d ( - cos y ) = 0
d [ f ( x , y )]1 - n f ¢ (x , y ) Þ tan ( xy ) - cos x - cos y = C
15. =
1-n ( f ( x , y )) n dy
4 x+y
y Example 51 Solve ( x 2 - ay ) dx + ( y 2 - ax ) dy = 0. y Example 55 Solve dx = x 2 + 2y 2 + y ×
dy x2
Sol. The given differential equation is y -x
dx
x 2dx + y 2dy - a (y dx + x dy ) = 0
Sol. The given equation can be written as
æx3 ö æy 3 ö x dx + y dy y dx - x dy y 2
Þ d ç ÷ +d ç ÷ - ad ( xy ) = 0 = × 2
è 3 ø è 3 ø ( x 2 + y 2 )2 y2 x
x3 y3 d (x 2 + y 2 ) æ x ö
1
Integrating, we get + - a xy = k Þ ò =2ò dç ÷
3 3 2
(x + y ) 2 2 2
x /y è y ø
2

Þ x 3 + y 3 - 3axy = 3k = C Integrating both the sides, we get


1 1
æx2 ö - 2 =- +C
y Example 52 Solve (2x log y ) dx + ç + 3y 2 ÷ dy = 0. (x + y )2
( x /y)
è y ø y 1
Sol. The given differential equation is Þ - =C
2
x x2 + y2
x
(log y ) 2x dx + dy + 3y 2 dy = 0
y y Example 56 The solution of
Þ 2 2
(log y ) d ( x ) + x d (log y ) + d (y ) = 0 3 ( y 2 - 1)
x

Þ d ( x 2 log y ) + d (y 3 ) = 0
e y
{ xy 2 dy + y 3dx } + { ydx - xdy } = 0, is
(a) e xy + e x / y + C = 0 (b) e xy - e x / y + C = 0
Þ x 2 log y + y 3 = C
(c) e xy + e y / x + C = 0 (d) e xy - e y / x + C = 0
(integrating both the sides)
2
( y - 1)
x
y Example 53 Solve x dx + y dy = x dy - y dx . Sol. Here, e y
× y 2 { xdy + ydx } + {ydx - xdy } = 0
Sol. The given equation can be written as Þ e xy × y 2 × { xdy + ydx } + e x / y {ydx - xdy } = 0
1 æy ö {ydx - xdy }
d (x 2 + y 2 ) = x 2 d ç ÷
2 èx ø or e xy × { xdy + ydx } + e x / y =0
y2
æy ö
2x 2 d ç ÷ æ x ö
d (x 2 + y 2 ) èx ø or e xy × d ( xy ) + e x / y × d ç ÷ =0
Þ = è y ø
x2 + y2 x2 + y2
d (x 2 + y 2 ) 2d (y / x ) or d (e xy ) + d (e x / y ) = 0
Þ =
x2 + y2 1 + (y / x )2 Integrating both the sides, we get
æy ö e xy + e x / y + C = 0
Þ log ( x 2 + y 2 ) = 2 tan - 1 ç ÷ + C
èx ø Hence, (a) is the correct answer.
258 Textbook of Integral Calculus

y Example 57 The solution of Sol. The given equation is


x 2dy - y 2dx + xy 2 ( x - y ) dy = 0, is ( xy 2e xy ) dy + ( xe x / y ) dy = (ye x / y ) dx - (y 3e xy ) dx

½x -y½ 2
½ xy ½ 2 Þ y 2e xy ( xdy + ydx ) = e x / y (ydx - xdy )
(a) log½ ½= y + C (b) log½ ½= x + C
½ xy ½ 2 ½x -y½ 2 æ ydx - xdy ö
Þ e xy (d ( xy )) = e x / y × ç ÷
½x -y½ 2
½x -y½ è y2 ø
(c) log½ ½= x + C (d) log½ ½= x + C
½ xy ½ 2 ½ xy ½ æ x ö
Þ e xy (d ( xy )) = e x / y × d ç ÷
æ dy dx ö æ 1 1 ö è y ø
Sol. Here, x 2y 2 ç 2 - 2 ÷ + x 2y 3 ç - ÷ dy = 0
è y x ø è y x ø Þ d (e xy ) = d (e x / y )
æ 1 1 ö æ 1 1 ö Integrating both the sides, we get
Þ - d ç - ÷ + y ç - ÷ dy = 0
è y x ø è y x ø e xy = e x / y + l
æ1 1ö æ1 1ö Þ xy = log (e x / y + l )
dç - ÷ dç - ÷
èy x ø èy x ø æy 2 ö Hence, (c) is the correct answer.
Þ - + ydy = 0 or =d ç ÷
æ1 1ö æ1 1ö è 2 ø
ç - ÷ ç - ÷ y Example 60 The solution of the differential
èy x ø èy x ø
equation
Integrating both the sides, we get
½ 1 1½ y
2
( y + x xy ( x + y )) dx + ( y xy ( x + y ) - x ) dy = 0, is
log½ - ½ = +C
2 2
½x y½ 2 x +y x
(a) + 2 tan - 1 =C
Hence, (a) is the correct answer. 2 2y

y Example 58 The solution of the differential equation x2 + y 2 x


(b) + 2 tan - 1 =C
2 y
ydx - xdy + xy 2 dx = 0, is
x2 + y 2 x
x x x2 (c) + 2 tan - 1 =C
(a) + x 2 = l (b) + =l 2 y
y y 2
(d) None of these
x x2
(c) + =l (d) None of these
2y 2 4 Sol. The given equation can be written as
(ydx - xdy ) + x xy ( x + y ) dx + y xy ( x + y ) dy = 0
Sol. Given equation is, ydx - xdy + xy 2 dx = 0
Which could be converted into exact form Þ (ydx - xdy ) + ( x + y ) xy ( xdx + ydy ) = 0
ydx - xdy
i.e. + xdx = 0 ydx - xdy æ x ö x æ x2 + y2 ö
y2 Þ + ç +1 ÷× dç ÷ =0
y 2
è y ø y è 2 ø
æx ö æx2 ö
Þ d ç ÷ +d ç ÷ =0 æx ö æ x2 + y2 ö æ x ö x
èy ø è 2 ø Þ d ç ÷ +d ç ÷ ç + 1÷ =0
èy ø è 2 ø èy ø y
Integrating both the sides, we get
x x2 æx ö
+ = constant dç ÷
y 2 æ x2 + y2 ö èy ø
Þ dç ÷+ =0
x x2 è 2 ø æx ö x
or + =l ç + 1 ÷ ×
y 2 èy ø y
Hence, (b) is the correct answer. æ x2 + y2 ö æ -1 æ x ö
ö
or dç ÷ + 2d ç tan ç ÷÷ = 0
è 2 ø è è y øø
y Example 59 The solution of differential equation
xdy ( y 2e xy + e x / y ) = ydx (e x / y - y 2e xy ) , is Integrating both the sides, we get
x2 + y2 x
(a) xy = log (e x + l) (b) x 2 / y = log (e x / y + l) Þ + 2 tan - 1 =C
2 y
(c) xy = log (e x / y + l) (d) xy 2 = log (e x / y + l) Hence, (b) is the correct answer.
Chap 04 Differential Equations 259

Exercise for Session 4


1. The solution of xdy + ydx + 2x 3dx = 0, is
1 4 x2 x4
(a) xy + x4 = C (b) xy + x =C (c) + =C (d) None of these
2 y 4

2. The solution of, ydx - xdy + (1 + x 2 ) dx + x 2 sin y dy = 0, is given by


(a) x + 1 - y 2 + cos y + C = 0 (b) y + 1 - x 2 + x cos y + C = 0
x 1 y 1
(c) + - y + cos y + C = 0 (d) + - x + cos y + C = 0
y y x x

3. The solution of (1 + x x 2 + y 2 ) dx + ( - 1 + x 2 + y 2 ) ydy = 0, is


2 2 2 2
(a) 2x - y 2 + (x + y 2 ) 3 / 2 = C (b) 2x - y + (x + y 2 ) 3 / 2 = C
3 3
2 2
(c) 2y - x 2 + (x + y 2 ) 3 / 2 = C (d) None of these
3
xdy æ y ö
4. The solution of, 2 2
=ç 2 2
- 1÷ dx , is given by
x +y èx + y ø
æ x ö
(b) tan- 1 æç
y ö
(c) tan- 1 æç
y ö
(d) tan- 1 æç
y ö
(a) tan- 1 ç ÷ + x =C ÷ + x =C ÷ + xy = C
2
÷ + x =C
è y ø è x ø è x ø è x ø

5. The solution of ye x / y dx = ( xe x / y + y 2 sin y ) dy , is given by


(a) e x / y = - cos y + C (b) e x / y + 2 cos y = C (c) e x / y = x cos y + C (d) e x / y = 2 cos y e x / y + C

æ y ö ì æ y ö ü
6. The solution of x sin ç ÷ dy = í y sin ç
è x ø è
÷ - x ý dx , is given by
ø
î x þ
æx ö
(a) log x - cos æç ö÷ = log C (b) log x - sin æç ö÷ = C (c) log ç ÷ - cos æç ö÷ = log C
y y y
(d) None of these
èx ø èx ø èy ø èx ø

xdx + ydy a2 - x 2 - y 2
7. The solution of = , is given by
xdy - ydx x2 + y2

(a) sin-1 ( x 2 + y 2 ) = a tan- 1 æç ö÷ + C tan- 1 æç


y 1 y ö
(b) sin- 1 ( x 2 + y 2 ) = ÷ +C
èx ø a è x ø
æ x 2 + y 2 ö÷
= tan- 1 æç
y ö
(c) sin- 1 ç ÷ +C (d) None of the above
ç a ÷ è x ø
è ø
æ xö
8. The solution of (1 + e x / y ) dx + e x / y ç1 - ÷ dy = 0, is given by
è yø
x x/y
(a) x - ye x / y = C (b) x + ye x / y = C (c) y - e =C (d) None of these
y
x + y dy / dx x sin2 ( x 2 + y 2 )
9. The solution of = , is given by
y - x dy / dx y3
2
æ x ö
(a) - cot (x 2 + y 2 ) = ç ÷ +C (b) tan (x 2 + y 2 ) = x 2 y 2 + C
è y ø
x
(c) cot (x 2 + y 2 ) = +C (d) None of these
y
dy y 1
10. The solution of + = , is given by
dx x (1 + log x + log y )2
(a) xy (1+ log (xy )) = C (b) xy 2 (1 + log (xy )) = C (c) xy (1 + log (xy ))2 = C (d) xy (1 + (log xy )2 ) = C
Session 5
Solving of First Order and Higher Degrees, Application of
Differential Equations, Application of First Order
Differential Equations

Solving of First Order Hence, the general solution is given by


and Higher Degrees f1 ( x , y , c ), f2 ( x , y , c ), ¼, fn ( x , y , c ) = 0
Here, the arbitrary constant c 1 , c 2 , ¼, c n are replaced by a
Differential Equation of First Order single arbitrary constant c because every first order
equation has only one arbitrary constant in its solution.
and Higher Degrees
A differential equation of first order is of the form y Example 61 Solve (p - x ) (p - e x ) (p - 1 / y ) = 0;
f ( x , y , P ) where P = dy / dx . If in the equation degree of P dy
where p = ×
is greater than one, then the equation is of first order and dx
higher degree. Sol. The component linear equations are p = x , p = e x , p =
1
The differential equation of first order and higher degree y
dy x2
can be written in the form If = x , then dy = x dx Þ y = + C1
dx 2
P n + F1 ( x , y ) P n - 1 +¼ + Fn - 1 ( x , y ) P + Fn ( x , y ) = 0
dy
If = e x , then dy = e x dx Þ y = e x + C 2
The differential equations of these category can be solved dx
by one or more of the following methods : dy 1 y2
If = , then y dy = dx Þ = x + C3
(i) Equations solvable for P. dx y 2
(ii) Equations solvable for y. \ The required solution is
(iii) Equations solvable for x . æ x2 ö æy 2 ö
çy - + C ÷ (y - e x + C ) ç - x + C÷ = 0
(iv) Clairaut’s equations. è 2 ø è 2 ø
Now, we shall discuss these cases.
y Example 62 Solve x 2p 2 + xyp - 6 y 2 = 0.

(i) Equations Solvable For P Sol. The given equation is


x 2 p 2 + xyp - 6y 2 = 0
If the equation
Solving as a quadratic in p, we get
P n + F1 ( x , y ) P n - 1 +¼ + Fn - 1 ( x , y ) P + Fn ( x , y ) = 0,
( - xy ± x 2y 2 + 24 x 2y 2 ) 2y 3y
is solvable for P, then LHS expression can be resolved p= = ,-
2x 2 x x
into n linear factors and hence can be put in the form
2y dy 2y dy 2dx
( P - f 1( x , y )) ( P - f 2 ( x , y )) ¼ ( P - f n ( x , y )) = 0. If p = , then = Þ =
x dx x y x
Equating each of these factors to zero, we get n differential
y
equations of the first order and first degree. Þ log½ 2 ½ = k Þ y = C 1 x 2
dy dy dy ½x ½
= f 1 ( x , y ), = f 2 ( x , y ), ¼, = f n (x , y ) 3y dy 3y dy 3dx
dx dx dx If p = - , then =- Þ =-
x dx x y x
Let the solutions of these obtained equations are
f1 ( x , y , c 1 ) = 0, f2 ( x , y , c 2 ) = 0, ¼, fn ( x , y , c n ) = 0 Þ x 3y = C 2
respectively. \ The required solution is (y - Cx 2 ) ( x 3y - C ) = 0.
Chap 04 Differential Equations 261

y Example 63 Solve xy 2 (p 2 + 2) = 2py 3 + x 3 . (The equation p 2 - a = 0 gives us singular solution in


which we are not interested).
Sol. The given equation can be written as
The substitute p in Eq. (i), we get the required solution
( xy 2 p 2 - x 3 ) + 2 ( xy 2 - py 3 ) = 0
a
2y = Cx 2 +
Þ x (y 2 p 2 - x 2 ) + 2y 2 ( x - py ) = 0 C
Þ ( py - x ) {x ( py + x ) - 2y 2} = 0
y Example 65 Solve y = 2px - p 2 .
2 2
If py - x = 0, then y dy - x dx = 0 Þ y - x = C 1
Sol. Differentiating the given equation w.r.t. x, we get
2 2 dy 4y 2 dy dp dp
If xyp + x - 2y = 0, then 2y - = - 2x = 2p + 2x - 2p
dx x dx dx dx
dt 4 dp
Þ - t = - 2x , or 2 (x - p ) + p =0
dx x dx
4
-ò dx 1
where t = y 2 IF = e x = e - 4 ln x = 4 dx 2
or + x =2
x dp p
æ 1 ö 1
Its solution is t ç 4 ÷ = ò - 2x × 4 dx It is a linear equation in x and p.
è x ø x 2
ò dx
t 1 IF = e p
= e 2 log p = p 2
i.e. = + C 2 i.e. y 2 = x 2 + C 2 x 4
x4 x2 2 3
Hence, the required solution is
\ The solution is xp 2 = ò p 2 × 2 dp =
3
p +C

(y 2 - x 2 - C ) (y 2 - x 2 - Cx 4 ) = 0 Thus, the solution of the given equation is


2
x = p + Cp - 2 , where p is parameter.
3
(ii) Equations Solvable For y
Equation that comes under this category, can be expressed
in the form
(iii) Equations Solvable For x
y = g( x , p ) This type of equation can be put in the form
(i.e. an explicit function y in term of x and p) …(i)
x = g(y , p ) …(i)
Differentiating Eq. (i) w.r.t. x , we get
Differentiating w.r.t. y, we get
dy æ dp ö
= p = F ç x , p, ÷
dx è dx ø
1 dx æ dp ö
Which is a differential equation of the first order = = G ç x , p, ÷
p dy è dx ø
containing x and p. Let us suppose that its solution is
which is a differential equation of 1st order containing y
f ( x , p, c ) = 0 …(ii)
and p and its solution is
Then, the solution is obtained by eliminating p between
y = g( x , p ) and f ( x , p, c ) = 0. However, if eliminating of p is
f(y , p, c ) = 0
difficult express x and y as a function of the parameter p.
Then, the solution is obtained by eliminating p between
y Example 64 Solve xp 2 - 2yp + ax = 0. x = g(y , p ) and f(y , p, c ) = 0. However, if eliminating of p is
Sol. The given equation can be written as,
difficult express x and y as a function of the parameter p.

y=
xp ax
+ …(i) y Example 66 Solve y = 2px + y 2p 3 .
2 2p
Sol. Solving for x, we get
dy p x dp a ax dp
= + × + - 2× y y 2p 2
dx 2 2 dx 2p 2p dx x= - …(i)
2p 2
dp
Þ p ( p 2 - a) = x ( p 2 - a) × Differentiating Eq. (i) w.r.t. y, we get
dx
dx 1 y dp dp
dp p = - × - yp 2 - y 2 p ×
Þ = Þ p = Cx dy 2p 2p 2 dy dy
dx x
262 Textbook of Integral Calculus

æ 1 ö dp
or
1
-
p 2p
1
+ yp 2 = - y ç
è 2p 2
+ yp ÷ ×
ø dy
Application of Differential
or (1 + 2yp 3 ) p = - y (1 + 2p 3y ) ×
dp Equations
dy
dp dy C
Differential Equation of First Order
+ = 0 Þ py = C Þ p =
or
p y y But not of First Degree
Substituting this in the Eq. (i), we get 1. The most general form of a first order and higher
y 2
C 2 degree differential equation is
x= - Þ y 2 = 2Cx + C 2
2C 2 p n + P1 p n -1 + P2 p n -2 + K... + Pn = 0 where P1 , P2 ,
……, Pn are function of x, y and p = dy /dx . If a 1st
order any degree equation can be resolved into
(iv) Clairaut’s Equation differential equation (involving p) of first degree and
The differential equation y = px + f ( p ) is known as 1st order, in such case we say that the equation is
Clairaut’s equation. The solution of equation of this type solvable for p.
is given by y = cx + f (c ). Let their solution be
Which is obtained by replacing p by c in the given g 1 ( x , y , c 1 ) ´ g 2 ( x , y , c 2 ) ´ K ´ g n ( x , y , c n ) = 0,
equation. (where c 1 , c 2 , ………, c n , are arbitrary constant) we
take c 1 = c 2 = ..... = c n = c because the differential
Remark equation of 1st order 1st degree contain only one
Some equations can be reduced to Clairaut’s form by suitable arbitrary constant. So solution is
substitution.
g1 (x , y, c ) ´ g2 (x , y, c ) ´ K ´ gn (x , y, c ) = 0
p 2. The most general form of a first order and higher
y Example 67 Solve y = px + ×
1+ p 2 degree differential equation is
c p n + P1 p n -1 + P2 p n -2 + K + Pn = 0, where P1 , P2 ,
Sol. Its solution is, y = cx +
1 + c2 ……, Pn are function of x, y and p = dy /dx . If
differential equation is expressible in the form
y Example 68 Solve 1 + p 2 = tan (px - y ). y = f ( x , p ), then
dy æ dp ö
Step 1 Differentiate w.r.t. x, we get p = f ç x , p, ÷ .
Sol. The given equation is dx è dx ø
1 + p 2 = tan ( px - y )
Step 2 Solving this we obtain f ( x , p, c ) = 0.
Step 3 The solution of differential equation is
or px - y = tan - 1 ( 1 + p 2 ) obtained by eliminating p.
or y = px - tan - 1 ( 1 + p 2 )

Its solution is, y = cx - tan - 1 ( 1 + c 2 ) Application of First Order


y Example 69 Solve y 2 log y = pxy + p 2 . Differential Equations
1 dy dt
Sol. Let log y = t . Then =
y dx dx
Growth and Decay Problems
dt p Let N (t ) denotes the amount of substance (or population)
So, if = p , then =p that is either growing or decaying. If we assume that
dx y
dN /dt , the time rate of change of this amount of
Substituting these in the given equation, we have
substance, is proportional to the amount of substance
y 2t = y × p xy + p 2y 2 or t = px + p 2 present, then
dN dN
Which is in Clairaut’s form. = kN or - kN = 0 …(i)
Thus, the required solution is dt dt
t = cx + c 2 or log y = cx + c 2 Where k is the constant of proportionality. We are
assuming that N (t ) is a differentiable, hence continuous,
(c being an arbitrary constant.)
function of time.
Chap 04 Differential Equations 263

y Example 70 The population of a certain country is 1 45


45 = 50e 2k or k = ln = - 0.053
known to increase at a rate proportional to the 2 50
number of people presently living in the country. If Substituting this value into Eq. (ii), we obtain the
after two years the population has doubled and after amount of mass present at any time t as
three years the population is 20000, estimate the N = 50e -0.053 t …(iii)
number of people initially living in the country. Where t is measured in hours.
Sol. Let N denotes the number of people living in the country (b) We require N at t = 4. Substituting t = 4 into Eq. (iii)
at any time t, and let N 0 denote the number of people
initially living in the country. Then, from Eq. (i) and then solving for N , we find N = 50e (- 0.053) (4 )
dN (c) We require t when N = 50 /2 = 25. Substituting N = 25
- kN = 0
dt into Eq. (iii) and solving for t, we find 25 = 50e -0.053t
Which has the solution N = Ce kt …(i) 1
or - 0053
. t = ln or t = 13 h
At t = 0, N = N 0 ; hence, it follows from Eq. (i) that 2
N 0 = Ce k (0 ) or that C = N 0 .
y Example 72 Five mice in a stable population of 500
Thus, N = N 0e kt …(ii) are intentionally infected with a contagious disease to
At t = 2, N = 2N 0 . Substituting these values into Eq. (ii), we test a theory of epidemic spread that postulates the
have rate of change in the infected population is
1
2N 0 = N 0e 2k from which k = ln 2 = 0.347 proportional to the product of the number of mice who
2 have the disease with the number that are disease
Substituting this value into Eq. (i) gives free. Assuming the theory is correct, how long will it
N = N 0e (0.347) t …(iii) take half the population to contract the disease?
Sol. Let N (t ) denotes the number of mice with the disease at
At t = 3, N = 20000. Substituting these values into Eq. (iii),
we obtain time t. We are given that N (0) = 5, and it follows that
500 - N (t ) is the number of mice without the disease at
20000 = N 0e (0. 347 )(3 ) time t. The theory predicts that
dN
= kN (500 - N ) …(i)
y Example 71 A certain radioactive material is dt
known to decay at a rate proportional to the Where k is a constant of proportionality. This equation is
amount present. If initially there is 50 mg of the different from Eq. (i) because the rate of change is no longer
proportional to just the number of mice who have the
material present and after two hours it is observed disease. Eq. (i) has the differential form
that the material has lost 10% of its original mass, dN
find (a) and expression for the mass of the material - kdt = 0 …(ii)
N (500 - N )
remaining at any time t, (b) the mass of the material
Which is separable. Using partial fraction decomposition,
after four hours, and (c) the time at which the we have
material has decayed to one half of its initial mass. 1 1 /500 1 /500
Sol. (a) Let N denotes the amount of material present at time = +
N (500 - N ) N 500 - N
t. Then, from Eq. (i)
dN Hence, Eq. (ii) may be rewritten as
- kN = 0
dt 1 æ1 1 ö
ç + ÷ dN - kdt = 0
This differential equation is separable and linear, its 500 è N 500 - N ø
solution is 1 æ1 1 ö
500 ò è N 500 - N ø
It solution is ç + ÷ dN - ò kdt = C
N = Ce kt …(i)
At t = 0, we are given that N = 50. Therefore, from 1
or (ln | N | - ln| 500 - N | ) - kt = C
Eq. (i), 50 = Ce k (0 ) or C = 50. 500
Thus, N = 50 e kt …(ii) Which may be rewritten as
At t = 2, 10% of the original mass of 50 mg or 5 mg has N
ln = 500 (C + kt )
decayed. Hence, at t = 2, N = 50 - 5 = 45. Substituting 500 - N
these values into Eq. (ii) and solving for k, we have
264 Textbook of Integral Calculus

N dy
= e 500 (C + kt ) …(iii) y-intercept of the tangent = y 1 - x 1
500 - N dx
But e 500 (C + kt ) = e 500 e kt . Setting C 1 = e 500C , we can write Eq. (ii) The equation of normal at P is,
(iii) as 1
y - y1 = - ( x - x 1 ) x and y-intercepts of
N (dy / dx )
= C 1e 500kt …(iv)
500 - N dy dx
normal are; x 1 + y 1 and y 1 + x 1
At t = 0, N = 5. Substituting these values into Eq. (iv), we dx dy
find
4 (iii) Length of tangent = PT = | y 1 | 1 + (dx / dy ) (2x
= C 1 e 500k (0 ) = C 1 1 , y1 )
495
So, C 1 = 1 /99 and Eq. (iv) becomes
(iv) Length of normal = PN = | y 1 | 1 + (dy / dx ) (2x
1 , y1 )

N 1 500kt
= e …(v) ½ æ dx ö ½
500 - N 99 (v) Length of subtangent = ST =½ y 1 ç ÷ ½
We could solve Eq. (v) for N , but this is not necessary. We ½ è dy ø ( x1 , y 1 )½
seek a value of t when N = 250, one half the population. ½ æ dy ö ½
Substituting N = 250 into Eq. (v) and solving for t, we (vi) Length of subnormal = SN =½ y 1 ç ÷ ½
obtain ½ è dx ø ( x1 , y 1 ) ½
1 500kt
1= e ; ln99 = 500kt (vii) Length of radius vector = x 12 + y 12
99
or t = 0.0091 / k time units. Without additional
information, we cannot obtain a numerical value for the y Example 73 Find the curve for which the area of the
constant of proportionality k or be more definitive about t. triangle formed by the x-axis tangent drawn at any
point on the curve and radius vector of the point of
tangency is constant equal to a 2 .
Geometrical Applications Sol. Tangent drawn at any point ( x , y ) is
Let P ( x 1 , y 1 ) be any point on the curve y = f ( x ), then dy
Y -y = (X - x )
æ dy ö dx
slope of the tangent at P ( = tan y ) = ç ÷ and
è dx ø ( x1 , y 1 ) y

hence we find the following facts.


(x1, y1)
y
y = f(x) P(x, y)
N′

x
P (x1, y1) O (X, 0)

dx
When Y = 0, X = x - y
x dy
O T S N
Area of D = 2a 2 (given)
T′
i.e. ½ 1 × X × Y ½ = 2a 2
Fig. 4.2 ½2 ½
½
½ ½
dx ½
(i) The equation of the tangent at P is, i.e. xy - y 2 = 2a 2
dy ½ dy ½
y - y1 = ( x - x 1 ) when it cuts x-axis, y = 0. dx
dx i.e. xy - y 2 = ± 2a 2
dy
æ dx ö
\ x-intercept of the tangent = x 1 - y 1 ç ÷ dx x 2a 2
è dy ø i.e. - =± 2
dy y y
Chap 04 Differential Equations 265

1 -1
ò- dy
1 dy æ dx ö
IF = e y
= Sol. We know, =ç ÷ , differentiating both the sides
y dx è dy ø
-2
1 2a 2 1 d 2y æ dx ö d æ dx ö dy
\ The solution is x × = ò ± 2 × dy or =-ç ÷ × ç ÷×
y y y dx 2
è dy ø dy è dy ø dx
-2
x 2a 2 × y - 2 a2 æ dx ö d 2 x dy
=± + C , i.e. x = Cy + =-ç ÷ × ×
y -2 y è dy ø dy 2 dx
3
d 2y d 2 x æ dy ö
y Example 74 Find the curve for which the intercept Þ =- ×ç ÷
dx 2 dy 2 è dx ø
cut off by any tangent on y-axis is proportional to the
3
square of the ordinate of the point of tangency. d 2y 2
æ dy ö d x
or + ç ÷ × =0
Sol. The equation of tangent at any point ( x , y ) is dx 2 è dx ø dy 2
dy Hence, (d) is the correct answer.
Y -y = (X - x ) 2
dx dy dy æ dy ö
dy y Example 76 The solution of y = x + -ç ÷ ,
When X = 0; Y = y - x = y -intercept dx dx è dx ø
dx is
2
It is given Y µ y . i.e. Y = ky 2 (a) y = (x - 1) 2 (b) 4y = (x + 1) 2
(k being constant of proportionality) (c) (y - 1) 2 = 4 x (d) None of these
dy Sol. The given equation can be written as
i.e. y-x = ky 2
dx dy
2
y = xp + p - p 2 ; where p = …(i)
dy 1 ky dx
i.e. - y=-
dx x x Differentiating both the sides w.r.t. x, we get
dy 1 1 k xdp dp dp
i.e. - y- 2 + × = p=p+ + - 2p
dx x y x dx dx dx
1 1 dy dz dz z k dp
Let = z so that, - 2 × = \ + = \ ( x + 1 - 2p ) = 0
y y dx dx dx x x dx
dp
ò
1
dx \ Either = 0 i.e. p = C …(ii)
IF = e x = e log x = x dx
1
kx or x + 1 - 2p = 0 ie, p =
( x + 1) …(iii)
\ The solution is zx = ò x dx = kx + C 2
x Eliminating p between Eqs. (i) and (ii), we get
Þ = kx + C
y y = Cx + C - C 2
Þ x = kxy + Cy As the complete solution and eliminating p between Eqs. (i)
1 -C 1 and (iii)
Þ x - Cy = kxy Þ + × =1 1 1 1
ky k x y = ( x + 1) x + ( x + 1) - ( x + 1) 2
2 2 4
C1 C 2 æ 1 C ö
Þ + =1 ç where = C 2 and - = C 1 ÷ ie, 4y = ( x + 1)2 as the singular solution.
x y è k k ø
Hence, (b) is the correct answer.
y Example 75 For any differential function y = f ( x ),
3 y Example 77 The solution of
d 2y 2
æ dy ö d x 2
the value of 2 + ç ÷ × 2 is equal to æ dy ö dy
dx è dx ø dy ç ÷ + (2x + y ) + 2xy = 0, is
è dx ø dx
dy dy
(a) 2y (b) y 2 (a) (y + x 2 - C 1 ) (x + log y + y 2 + C 2 ) = 0
dx dx
2 (b) (y + x 2 - C 1 ) (x - log y - C 2 ) = 0
dy æ d 2 x ö
(c) y +ç ÷ (d) None of these (c) (y + x 2 - C 1 ) (x + log y - C 2 ) = 0
dx è dy 2 ø
(d) None of the above
266 Textbook of Integral Calculus

Sol. The given equation can be written as y Example 79 The equation of the curve passing
2
p + (2x + y ) p + 2xy = 0, where p =
dy through the points ( 3a, a ) (a > 0) in the form x = f ( y )
dx which satisfy the differential equation;
i.e. ( p + 2x ) ( p + y ) = 0 a 2 dx x y
× = + - 2, is
\ p + 2x = 0, otherwise p + y = 0 xy dy y x
dy dy
Þ + 2x = 0 or +y =0 æ 1+ ey -k ö æ 1+ ey -k ö
dx dx (a) x = y + a ç ÷ (b) x = y + a ç ÷
dy è 1 - 2e y - k ø è 1- ey -k ø
Þ òdy + 2 òx dx = C 1 or ò y + òdx = C 2 æ 1+ ey -k ö
(c) y = x + a ç ÷ (d) None of these
Þ y + x 2 = C1 or log y + x = C 2 è 1- ey -k ø
\ (y + x 2 - C 1 ) = 0 a 2 dx x y
Sol. Here, × = + -2
or ( x + log y - C 2 ) = 0 xy dy y x

Þ (y + x 2 - C 1 ) ( x + log y - C 2 ) = 0 dy 2
Þ a2 = ( x + y 2 - 2xy )
dx
is the required solution.
dy dy dv
Hence, (c) is the correct answer. Þ ( x - y )2 × = a 2 , put x - y = v \ 1 - =
dx dx dx
y Example 78 A curve y = f ( x ) passes through the æ dv ö
Þ v2 ç 1 - ÷ =a
2
è dx ø
origin. Through any point ( x , y ) on the curve, lines are
drawn parallel to the coordinate axes. If the curve dv v 2dv
Þ v 2 - a2 = v 2 Þ 2 = dx
divides the area formed by these lines and coordinate æ 2 ö
dx v - a2
a
axes in the ratio m : n. Then the equation of curve is Þ ç1 + 2 ÷ dv = dx
m /n 2 m /n è v - a2 ø
(a) y = Cx (b) my = Cx
3 m /n Integrating both the sides, we get
(c) y = Cx (d) None of these
a ½v - a½
½
Sol.
Area of OBPO m
= v+ log½ = x +C
Area of OPAO n 2 ½v + a½
y a æ x -y -a ö
Þ (x - y ) + log ç ÷ = x +C
2 è x -y +a ø
B
P (x, y) a æ x -y -a ö
Þ y +C = log ç ÷ …(i)
2 è x -y +a ø
It passes through (3a, a )
a æ 1 ö
Þ a+C = log ç ÷
2 è 3 ø
x
O A a æ 1 ö
Þ C = - a + log ç ÷
2 è 3 ø
x
æ 2 + log 3 ö
Þ
xy - ò0 y dx
=
m x
Þ nxy = (m + n ) ò y dx
Þ C = -aç
è 2
÷
ø
x 0
n
ò0 y dx
\
a
y = (2 + log 3) + log ç
æ x -y -a ö
÷
Differentiating w.r.t. x, we get 2 è x -y +a ø
æ dy ö x -y -a a
n çx + y ÷ = (m + n ) y Þ = e y - k , where k = (2 + log 3)
è dx ø x -y +a 2
y -k
x -y 1+e
Þ
dy
nx= my \ =
dx a 1 - ey -k
m dx dy æ 1 + ey -k ö a
Þ × = , integrating both the sides Þ x =y +aç y -k
÷, where k = (2 + log 3)
n x y è 1-e ø 2
y = Cx m /n Which is required equation of curve.
Hence, (a) is the correct answer. Hence, (b) is the correct answer.
Chap 04 Differential Equations 267

y Example 80 The family of curves, the subtangent dy 2y dx x + y


Þ = \ = …(i)
dx x + y dy 2xy
at any point of which is the arithmetic mean of the
coordinates of the point of tangency, is given by It is a homogeneous differential equation.
2
(a) (x - y ) = Cy (b) (y - x) = Cx 2 \ Put x = vy
2
(c) (x - y ) = Cxy (d) None of these Differentiating w.r.t. y, we get
dx dv
Sol. Let the family of curves be y = f ( x ) =v +y …(ii)
dy dy
l ( PP ¢ )
tan q = dx
l (TP ¢ ) In Eq. (i) replacing by Eq. (ii), we get
dy
y
dv vy + y 1 + v
y = f(x) v +y = =
dy 2y 2
P dv 1 + v 1 + v - 2v 1 - v
(x, f(x)) Þ y = -v = =
dy 2 2 2
2 dy
Þ dv =
1-v y
θ 2 log | 1 - v |
O T P′
x Integrating, = log | y | + log C 1 (C 1 > 0)
-1

l ( PP ¢ ) \ - 2 log | y - x | + 2 log | y | = log | y | + log C 1


tan q = Þ log | y - x | 2 = log | y | - log C 1
l (TP ¢ )
f (x ) Þ log | y - x | 2 = log | y | + log C ,
\ l (subtangent) =
f ¢( x ) where log C = - log C 1
y x +y Þ log | y - x | 2 = log | yC |
\ = (given)
y¢ 2
Þ ( x - y )2 = Cy , is the required equation of family of
2y curves.
\ y¢ =
x +y Hence, (a) is the correct answer.
268 Textbook of Integral Calculus

Exercise for Session 5


1. The equation of curve for which the normal at every point passes through a fixed point, is
(a) a circle (b) an ellipse
(c) a hyperbola (d) None of these

2. If the tangent at any point P of a curve meets the axis of x in T. Then the curve for which OP = PT , O being the
origin is
(a) x = Cy 2 (b) x = Cy 2 or x = C / y 2
(c) x = Cy or x = C / y (d) None of these

3. According to Newton’s law, the rate of cooling is proportional to the difference between the temperature of the
body and the temperature of the air. If the temperature of the air is 20°C and body cools for 20 min from 100°C
to 60°C, then the time it will take for it temperature to drop to 30°C, is
(a) 30 min (b) 40 min
(c) 60 min (d) 80 min

4. Let f ( x , y ) be a curve in the x-y plane having the property that distance from the origin of any tangent to the
curve is equal to distance of point of contact from the y-axis. If f (1, 2) = 0, then all such possible curves are
(a) x 2 + y 2 = 5x (b) x 2 - y 2 = 5x
(c) x 2 y 2 = 5x (d) All of these
x æ 1ö
5. Given the curves y = f ( x ) passing through the point (0, 1) and y = ò
f (t ) passing through the point ç 0, ÷ · The
-¥ è 2 ø
tangents drawn to both the curves at the points with equal abscissae intersect on the x-axis. Then the curve
y = f ( x ), is
x2
(a) f (x ) = x 2 + x + 1 (b) f (x ) =
ex
(c) f (x ) = e 2 x (d) f (x ) = x - e x

6. A curve passing through (1, 0) is such that the ratio of the square of the intercept cut by any tangent on the
y-axis to the Sub-normal is equal to the ratio of the product of the Coordinates of the point of tangency to the
product of square of the slope of the tangent and the subtangent at the same point, is given by
(a) x = e ±2 y/x
(b) x = e ± y/x

(c) y = e ± y/x
-1 (d) xy + e y / x - 1 = 0

7. Consider a curve y = f ( x ) in xy-plane. The curve passes through (0, 0) and has the property that a segment of
tangent drawn at any point P ( x , f ( x )) and the line y = 3 gets bisected by the line x + y = 1. then the equation of
curve, is
(a) y 2 = 9 (x - y ) (b) (y - 3)2 = 9 (1- x - y )
(c) (y + 3)2 = 9(1- x - y ) (d) (y - 3)2 - 9 (1+ x + y )

8. Consider the curved mirror y = f ( x ) passing through (0, 6) having the property that all light rays emerging from
origin, after getting reflected from the mirror becomes parallel to x-axis, then the equation of curve, is
(a) y 2 = 4 (x - y ) or y 2 = 36 (9 + x ) (b) y 2 = 4 (1- x ) or y 2 = 36 (9 - x )
(c) y 2 = 4 (1+ x ) or y 2 = 36 (9 - x ) (d) None of these
Chap 04 Differential Equations 269

JEE Type Solved Examples :


Single Option Correct Type Questions
l Ex. 1 The order of the differential equation of family of Sol. Here, ( xy 3 - x 2 )dy = ( xy + y 4 )dx
curves y = C 1 sin -1 x + C 2 cos -1 x +C 3 tan -1 x + C 4 cot -1 x Þ y 3( xdy - ydx ) - x( xdy + ydx = 0 )
(where C 1 , C 2 , C 3 and C 4 are arbitrary constants) is æy ö
Þ x 2y 3d ç ÷ - x d ( xy ) = 0
(a) 2 (b) 3 èxø
(c) 4 (d) None of these dividing by x 3y 2, we get
Sol. Here, y = C1 sin -1 x + C 2 cos-1 x + C 3 tan -1 x + C 4 cot -1 x y æy ö 1 æxö
Þ d ç ÷ - 2 2 ×d ç ÷ = 0
æp ö æp ö x è x ø xy èy ø
Þy = C1 sin -1 x + C 2 ç - sin -1 x ÷ + C 3 tan -1 x + C 4 ç - tan -1 x ÷
è2 ø è2 ø 2
æ 1 ö
1 æy ö
p Þ dç ÷ + dç ÷ = 0
= (C1 - C 2 )sin -1 x + (C 3 - C 4 ) tan -1 x + (C 3 - C 4 ) 2 èxø è xy ø
2
Now integrating, we get
There are only two independent arbitrary constant order of the 2
differential equation is 2. 1 æy ö 1
ç ÷ + =c
Hence, (c) is the correct answer. è
2 x ø xy
It passes through ( 4, - 2 )
l Ex. 2 The solution of the differential equation 1 1
Þ - =c Þ C = 0
dy 1 8 8
= is
dx xy ( x 2 sin y 2 + 1) \ y 3 = - 2 x is required curve.
2
(a) x 2 (cos y 2 - siny 2 - 2ce - y ) = 2 l Ex. 4 Spherical rain drop evaporates at a rate propor-

2 2 2 -y 2 tional to its surface area. The differential equation corre-


(b) y (sin x - cos y - 2ce )=2
2 sponding to the rate of change of the radius of the rain drop,
(c) x 2 (cos y 2 - siny 2 - e - y ) = 4c if the constant of proportionality is K > 0, is
(d) None of the above dr dr
(a) +K =0 (b) -K =0
dx dt dt
Sol. Here, = xy ( x 2 sin y 2 + 1 )
dy dr
(c) = Kr (d) None of these
1 dx 1 dt
Þ - y = y sin y 2
x 3 dy x 2 Sol.
dV
= - k 4 pr 2 …(i)
1 2 dx dt dt
Let, - 2 =t Þ 3 = 4
x x dy dy But V = pr 3
+ y2
3
dt
Þ + 2t × y = y × sin y 2, I . F . = e ò + 2y dy = e dV dr
dy Þ = 4 pr 2 …(ii)
dt dt
So, required solution is dr
2 2 1 2 Therefore, =-K
t × e y = ò 2y sin y 2 ´ e y dy = e y (sin y 2 - cosy 2 ) + C dt
2 Hence, (a) is the correct answer.
2
Þ 2t = (sin y 2 - cosy 2 ) + 2C e - y
Þ 2 = - x 2(sin y 2 - cosy 2 + 2ce - y )
2
l Ex. 5 A function y = f ( x ) satisfies the differential equa-
Þ
2
x 2(cosy 2 - sin y 2 - 2ce - y ) = 2 tion f ( x ) × sin 2 x - cos x + (1 + sin 2 x ) f ¢ ( x ) = 0 with initial
condition y (0 ) = 0. The value of f ( p /6 ) is equal to
l Ex. 3 The curve satisfying the differential equation (a) 1/5 (b) 3/5
dy y ( x +y 3 ) (c) 4/5 (d) 2/5
= and passing through ( 4, - 2 ) is
dx x (y 3 - x ) dy
Sol. y sin 2 x - cos x + (1 + sin 2 x ) = 0 where y = f ( x )
dx
(a) y 2 = - 2x (b) y = - 2x
dy æ sin 2 x ö cos x
3 +ç ÷y =
(c) y = - 2x (d) None of these dx è 1 + sin 2 x ø 1 + sin 2 x
270 Textbook of Integral Calculus

sin 2x
ò 1 + sin 2 x dx ò
dt
2
l Ex. 8 Number of straight lines which satisfy the differ-
IF = e =e t = e ln (1 + sin x)
2
dy æ dy ö
ential equation + x ç ÷ - y = 0 is
= 1 + sin 2 x (by putting 1 + sin 2 x = t) dx è dx ø
y (1 + sin 2 x ) = ò cos x dx (a) 1 (b) 2 (c) 3 (d) 4
dy
y (1 + sin 2 x ) = sin x + C ; y ( 0 ) = 0 Þ C = 0 Sol. y = kx + b; =k
dx
sin x æpö 2 Þ kx + b º k + xk 2 Þ k = k 2 and b = k
Therefore, y = ;y ç ÷ =
1 + sin 2 x è 6 ø 5
k = 0 or k = 1
Hence, (d) is the correct answer.
Hence, (b) is the correct answer.
l Ex. 6 The general solution of the differential equation l Ex. 9 Consider the two statements :
dy 1 - x
= is a family of curves which looks most like which Statement I y = sin kt satisfy the differential equation
dx y y ¢¢ + 9y = 0.
of the following? Statement II y = e kt satisfy the differential equation
y ¢¢ + y ¢ - 6y = 0.
(a) (b) (c) (d) The value of k for which both the statements are correct is
(a) -3 (b) 0 (c) 2 (d) 3
Sol. Statement I y = sin kt, y ¢ = k cos kt; y ¢¢ = - k 2 sin kt
Sol. ò ydy = ò (1 - x ) dx \ - k 2 sin kt + 9 sin kt = 0
y2 x2
=x- +C sin kt [9 - k 2 ] = 0 Þ k = 0, k = 3, k = - 3
2 2
x 2 + y 2 - 2x = C Statement II y = ekt , y ¢ = kekt ; y ¢¢ = k 2ekt
(x - 1)2 + y 2 = C + 1 = C \ k 2ekt + kekt - 6ekt = 0
Hence, (b) is the correct answer.
ekt [k 2 + k - 6 ] = 0

Remark (k + 3 ) (k - 2 ) = 0
Family of concentric circles with (1, 0) as the centre and k = - 3 or 2
variable radius. Common value is k = - 3.
Hence, (a) is the correct answer.
l Ex. 7 Water is drained from a vertical cylindrical tank
by opening a valve at the base of the tank. It is known that x
l Ex. 10 If y = (where c is an arbitrary constant) is
the rate at which the water level drops is proportional to the ln | cx |
square root of water depth y, where the constant of propor-
the general solution of the differential equation
tionality k > 0 depends on the acceleration due to gravity and
the geometry of the hole. If t is measured in minutes and dy y æxö æxö
= + f ç ÷ , then the function f ç ÷ is
1 dx x èy ø èy ø
k = , then the time to drain the tank, if the water is 4 m
15 x2 x2 y2 y2
deep to start with is (a) (b) - (c) (d) -
y2 y2 x2 x2
(a) 30 min (b) 45 min (c) 60 min (d) 80 min
dy x
Sol. = - k y ; when t = 0; y = 4 Sol. ln c + ln | x| =
dt y
0 dy t 1 y - xy1
ò4 y = - k ò0dt Differentiating w.r.t. x,
x
=
y2
t y2 dy
[2 y ]04 = - kt = - =y - x
15 x dx
t
0-4=- dy y y 2 æxö y2
15 = - 2 Þ fç ÷=- 2
dx x x èy ø x
Þ t = 60 min
Hence, (c) is the correct answer. Hence, (d) is the correct answer.
Chap 04 Differential Equations 271

x a2 a2
l Ex. 11 If ò ty (t ) dt = x 2 + y ( x ), then y as a function of Hence, 2
-a =2 + Ce ; Ce 2
2 = - (2 + a 2 )
a
x is
x 2 - a2 x 2 - a2 a2 x2 - a2
-
(a) y = 2 - ( 2 + a 2
)e 2 (b) y = 1 - ( 2 + a 2
)e 2 C = - (2 + a 2 ) e 2 ;y = 2 - (2 + a 2 ) e 2

x 2
-a 2 Hence, (a) is the correct answer.
2
(c) y = 2 - (1 + a ) e 2 (d) None of these
dy 1-y 2
Sol. Differentiating both the sides, we get l Ex. 12 The differential equation = deter-
xy ( x ) = 2 x - y ¢( x ) dx y
dy é dy ù mines a family of circles with [IIT JEE 2007]
Hence, - xy = - 2 x êëy ¢( x ) = dx ; y ( x ) = y úû
dx (a) variable radii and a fixed centre at (0, 1)
-x 2 (b) variable radii and fixed centre at (0, -1)
IF =e ò - x dx
=e 2 (c) fixed radius 1 and variable centres along the x-axis
-x 2 -x 2 (d) fixed radius 1 and variable centres along the y-axis
ye 2 =ò -2 xe 2 dx
dy 1 -y2
x 2
x2 Sol. Q =
- - dx y
Let e 2 =t Þ - xe 2 dx = dt y 1
I = ò 2dt
Þ ò 1 -y2
dy = ò dx Þ -
2
×2 1 -y2 = x + C

-
x2
-
x2 Þ - 1 - y 2 = x + C Þ 1 - y 2 = (x + C )2
ye 2 = 2e 2 +C
Þ (x + C )2 + y 2 = 1
x2
y =2+ Ce 2 Therefore, the differential equation represents a circle of fixed
radius 1 and variable centres along the x-axis. Hence, (c) is the
If x = a Þ a 2 + y = 0 Þ y = - a 2 (from the given equation) correct answer.

JEE Type Solved Examples :


More than One Correct Option Type Questions
l Ex. 13 A curve y = f ( x ) has the property that the dy y 2 - x 2
= Þ homogeneous
perpendicular distance of the origin from the normal at any dx 2 xy
point P of the curve is equal to the distance of the point P dy
Also, x × 2y × - x2 = y 2
from the x-axis. Then the differential equation of the curve dx
dy dt dt
(a) is homogeneous Put y 2 = t; 2y = ; x× + x2 = t
dx dx dx
(b) can be converted into linear differential equation with dt 1
some suitable substitution - t = - x which is linear differential equation.
dx x
(c) is the family of circles touching the x-axis at the origin
Hence, (a), (b) and (d) are the correct answers.
(d) the family of circles touching the y-axis at the origin
Sol. Equation of normal l Ex. 14 A differentiable function satisfies
1 x
Y - y = - ( X - x ) Þ - my + my = X - x f ( x ) = ò { f (t ) cos t - cos (t - x )} dt . Which is of the follow-
m 0
X + my - ( x + my ) = 0 ing hold good?
(a) f ( x ) has a minimum value 1- e
P(x, y) (b) f ( x ) has a maximum value 1 - e -1
æpö
(c) f ¢¢ ç ÷ = e (d) f ¢ (0) = 1
è 2ø
O x
Sol. f ( x ) = ò { f (t ) cos t - cos (t - x )} dx
½ x + my ½ 2 2
0
Perpendicular from ( 0, 0 ) = ½ ½ = y Þ x + 2 xym = y
= ò f (t ) cos t dt - ò cos( - t ) dt é ò f ( x ) dx = ò f (a - x ) dx ù
x x a a
2
1 + m
½ ½ 0 0 ëê 0 0 ûú
272 Textbook of Integral Calculus

x x+1
f ( x ) = ò f (t ) cos t dt - sin x \ lim = lim (e -2 + Ce - x )
0 x®2 ex x®2
Differentiating both the sides, we get 3e -2 = e -2 + Ce -2 Þ C = 2
f ¢( x ) = f ( x ) cos x - cos x
\ for x > 2
dy
Let f ( x ) = y ; f ¢( x ) = y = e -2 + 2e - x
dx
dy Hence, y (3 ) = 2e -3 + e -2 = e -2(2e -1 + 1 )
- y cos x = - cos x (L.D.E.)
dx y ¢ = - 2e - x
IF = e ò
- cos x dx
= e - sin x y ¢(3 ) = - 2e -3
Therefore, y × e - sin x = - ò e - sin x cos x dx; Hence, (a), (b) and (d) are the correct answers.
- sin x - sin x sin x
y ×e =C + e ; y = Ce +1 l Ex. 16 A curve y = f ( x ) passes through (1, 1) and
If x = 0; y =0 (from the given relation)
tangent at P ( x , y ) cuts the x-axis and y-axis at A and B
Þ C = -1
respectively such that BP : AP = 3 : 1, then [IIT JEE 2006]
Therefore, f ( x ) = 1 - e sin x
(a) equation of curve is xy ¢ - 3y = 0
Now, minimum value = 1 - e (when x = p /2)
(b) normal at (1, 1) is x + 3y = 4
Maximum value = 1 - e -1 (when x = - p / 2 )
æ 1ö
(c) curve passes through ç 2, ÷
f ¢( x ) = - e sin x cos x è 8ø
Therefore, f ¢( 0 ) = - 1 (d) equation of curve is xy ¢ + 3y = 0
f ¢¢( x ) = - [cos2 x × e sin x - e sin x × sin x ] Sol. Equation of the tangent to the curve y = f ( x ) at ( x, y ) is
æpö Y -y =
dy
(X - x )
f ¢¢ç ÷ = e
è2ø dx
y
Hence, (a), (b) and (c) are the correct answers.
B
dy y=f(x)
l Ex. 15 Let + y = f ( x ) where y is a continuous func- 3
dx
ì e - x , if 0 £ x £ 2 P(x, y)
tion of x with y(0 ) = 1 and f ( x ) = í -2 . Which
îe , if x > 2 1
is of the following hold(s) good? x
O A
-1 -1
(a) y (1) = 2e (b) y ¢ (1) = - e
Q Thus, cuts the x-axis at A and y-axis at B.
(c) y (3 ) = - 2e -3 (d) y ¢ (3 ) = - 2e -3 æ dy ö
çx -y ÷
dy dx æ dy ö
Sol. + y = f ( x ) Þ IF = e x \ Aç , 0 ÷ and B ç 0, - x + y÷
dx ç dy ÷ è dx ø
è dx ø
ye x = ò e x f ( x ) dx + C
Q BP : PA = 3 : 1
Now, if 0 £ x £ 2, then ye x = ò e x e - x dx + C æ dy ö
3 çx -y÷
è dx ø
Þ ye x = x + C +1´0
(dy / dx )
x = 0, y( 0 ) = 1, C = 1 Þ x=
4
\ ye x = x + 1 …(i) dy
x x Þ x + 3y = 0
x+1 2 e - (x + 1)e dx
y = ; y (1 ) = Þ y¢ =
ex e e 2x dy dx
e - 2e -e 1
Þ ò y = ò -3 x
y ¢(1 ) = = 2 =-
e2 e e Þ log y = - 3 log x + log C
x-2 C
If x > 2, ye = ò e
x
dx Þ y = 3
x
ye x = e x - 2 + C Q Curve passes through (1, 1) \ C = 1
y = e -2 + Ce - x æ 1ö
\ Curve is x 3y = 1 which also passes through ç2, ÷ .
è 8ø
As y is continuous. Hence, (c) and (d) are the correct answers.
Chap 04 Differential Equations 273

JEE Type Solved Examples :


Statement I and II Type Questions
l Ex. 17 Let a solution y = y ( x ) of the differential equa- Integration yields, ò
dx

dy
2 x x2 - 1 y y2 -1
tion x x 2 - 1 dy - y y 2 - 1 dx = 0 satisfy y( 2 ) = .
3 Þ sec -1 x = sec -1 y + C
[IIT JEE 2008]
æ pö Þ sec -1(2 ) = sec -1 (2 / 3 ) + C
Statement I y ( x ) = sec ç sec -1 x - ÷ .
è 6ø p p p
Þ = +C Þ C =
1 2 3 1 3 6 6
Statement II y ( x ) is given by = - 1- . p
-1 -1
y x x2 Thus, sec x = sec y +
6
(a) Statement I is true, Statement II is also true; Statement
æ -1 pö
II is the correct explanation of Statement I. Þ y = sec çsec x - ÷
è 6ø
(b) Statement I is true, Statement II is also true; Statement 1 1
II is not the correct explanation of Statement I. = =
æ -1 1 p ö 1 3 1 1
(c) Statement I is true, Statement II is false. cos ç cos - ÷ × + 1- 2×
è x 6ø x 2 x 2
(d) Statement I is false, Statement II is true. 1
1- 2
Sol. Q x x 2 - 1 dy - y y 2 - 1 dx = 0 1 3 x
Þ = +
dx dy y 2x 2
Which can be rewritten as =
x x2 - 1 y y2 -1 Hence, (c) is the correct answer.

JEE Type Solved Examples :


Passage Based Questions
Passage Sol. (Q. Nos. 18 to 20)
(Q. Nos. 18 to 20) dy æ 2 x ö 4x 2
+ç ÷y =
A curve y = f ( x ) satisfies the differential equation dx è 1 + x ø 2
1 + x2
dy 2x
(1 + x 2 ) + 2 yx = 4x 2 and passes through the origin. ò 1 + x 2 dx
dx IF = e
18 The function y = f ( x ) = e ln (1 + x
2
)
= (1 + x 2 )
(a) is strictly increasing, " x Î R 4x 3
(b) is such that it has a minima but no maxima \ y (1 + x 2 ) = ò 4 x 2 dx = +C
3
(c) is such that it has a maxima but no minima Passing through (0, 0) ÞC = 0
(d) has no inflection point
4x 3
19 The area enclosed by y = f -1
( x ), the x-axis and the \ y =
3(1 + x 2 )
ordinate at x = 2 /3 is dy 4 é (1 + x 2 ) 3 x 2 - x 3 × 2 x ù
4 2 1 = ê ú
(a) 2 ln 2 (b) ln 2 (c) ln 2 (d) ln 2 dx 3 ë (1 + x 2 ) 2 û
3 3 3
4 é 3 x 2 + x 4 ù 4 x 2(3 + x 2 )
20 For the function y = f ( x ) which one of the following = ê ú=
3 ë (1 + x 2 ) 2 û 3(1 + x 2 ) 2
does not hold good?
dy dy
(a) f ( x ) is a rational function Hence, > 0, " x ¹ 0; = 0 at x = 0
dx dx
(b) f ( x ) has the same domain and same rage
and it does not change sign Þ x = 0 is the point of inflection
(c) f ( x ) is a transcendental function
y = f ( x ) is increasing for all x Î R.
(d) y = f ( x ) is a bijective mapping
274 Textbook of Integral Calculus

x ® ¥; y ® ¥ ; x ® - ¥; y ® - ¥ 2 4 1 x3
A= -
3 3 ò0 1 + x 2 dx
y
Put 1 + x2 = t Þ 2x dx = dt
2/3 2 2 2 (t - 1 )
A= - ò dt
x 3 3 1 t
O 2 2 2æ 1ö
= - ò ç1 - ÷ dt
3 3 1 è tø
2 2 2 2
= - [t - ln t ]12 = - [(2 - ln 2 ) - 1 ]
3 3 3 3
2
Area enclosed by y = f -1( x ), x-axis and ordinate at x = 2 2 2
3 = - [1 - ln 2 ] = ln 2
3 3 3

JEE Type Solved Examples :


Single Integer Answer Type Questions
l Ex. 21 Let y = f ( x ) be a curve passing through (4, 3) 2
y= x –1
4 y= 5 – x
such that slope of normal at any point lying in the first
quadrant is negative and the normal and tangent at any
point P cuts the Y -axis at A and B respectively such that the
mid-point of AB is origin, then the number of solutions of –2 0 2
y = f ( x ) and y = |5 - | x ||, is
Sol. Equation of tangent at any point ( x1, y1 ) of curve y = f ( x ) is
(y - y1 ) = f ¢( x1 )( x - x1 ), so B ( 0, y1 - x1 f ¢( x1 ))
1 \ number of solutions for y = f ( x ) and y = | 5 - | x ||
Equation of normal at ( x1, y1 ) is (y , y1 ) = - ( x, x1 ) so, \ number of solutions are 2.
f ¢( x1 )
æ x1 ö æ pö
A = ç 0, y1 + ÷ mid point of AB is origin, so l Ex. 22 A real valued function, f ( x ), f : ç 0, ÷ ® R +
è f ¢( x1 ) ø è 2ø
æ 1 ö satisfies the differential equation xf ¢ ( x ) =1 + f ( x ){ x 2 f ( x ) -1 }
2y1 - x1 ç f ¢( x1 ) - ÷=0 æpö 4
è f ¢( x1 ) ø and f ç ÷ = , then lim f ( x ) , is
è 4ø p x ®0
Thus differential equation of curve y = f ( x ), is
2
Sol. Here, xf ¢( x ) = 1 + x 2 f 2( x ) - f ( x )
æ dy ö dy
x f ¢( x ) + f ( x )
x ç ÷ - 2y -x=0
è dx ø dx Þ =1
1 + x 2 f 2( x )
2 2
dy y ± x + y Integrating both sides
Thus, =
dx x ( x f ¢( x ) + f ( x ))dx
In first quadrant, x > 0, y > 0,
dy
> 0,
ò 1 + ( x f ( x )) 2
=x+C
dx
æpö 4
Þ tan -1( xf ( x ) = x + C, as f ç ÷ = .
dy y + x2 + y 2 è 4ø p
So = , put y = vx
dx x p
Þ tan -1 1 = + C
2 4
dv vx + x 1 + v
Þ v+x = Þ C =0
dx x
dv dx \ x f ( x ) = tan x
Þ ò 1 + v2 = ò x tan x
Þ f (x ) =
x
x2
on solving we get y = -1 tan x
4 and lim f ( x ) = lim =1
x ®0 x ®0 x
Chap 04 Differential Equations 275

-x 1
1 3 dy x 1 ò 2 dx log |1 - x | 2
l Ex. 23 If the area bounded by y = f ( x ), x = , x = and Þ - 2
× y = 2 , I.F. = e 1- x = e 2 = 1 - x2
2 2 dx 1 - x x
2 2 4
the X-axis is A sq units where f ( x ) = x + x 3 + × x 5 1
3 3 5 \ y × 1 - x2 = ò × 1 - x2 + C
2 4 6 1 - x2
+ × × x 7 +¼ ¥, | x | < 1 ,Then the value of[ 4A ] is (where[× ]
3 5 7 Þ y 1 - x 2 = sin -1 x + C , as f ( 0 ) = 0 Þ C = 0
is G . I . F )
f ¢( x ) = 1 + 2 x 2 +
2 2 4
× 4x 4 + × × 6x 6 + ¼ ¥ sin -1 x
Sol. Here, Þ y =
3 3 5 1 - x2 p
æd ö
= 1 + x ç ( xf ( x )) ÷ 3 /2 sin x -1 æt2 ö 2 1 é p2 p2 ù
p /3
è dx ø Þ A=ò dx = ò t dt = ç ÷ = ê - ú
1/ 2
1 - x2 p /6 è 2 øp 2ë 4 36 û
Þ f ¢( x ) = 1 + x xf ¢( x ) + f ( x ) 6
Þ (1 - x 2 ) f ¢( x ) = 1 + xf ( x ) \ [ 4A ] = 1

Subjective Type Questions


l Ex. 24 For a certain curve y = f ( x ) satisfying l Ex. 25 If f( x ) is a differentiable real-valued function
1
2
d y satisfying f¢ ( x ) + 2 f ( x ) £ 1, prove that f ( x ) - is a
= 6 x - 4; f ( x ) has a local minimum value 5 when x =1. 2
dx 2 non-increasing function of x.
Find the equation of the curve and also the global maximum Sol. f¢( x ) + 2 f( x ) £ 1
and global minimum values of f ( x ) given that 0 £ x £ 2. 2x
Þ e f¢( x ) + 2 f( x ) e 2x £ e 2x
2
d y dy
Sol. Integrating, = 6 x - 4, we get = 3x 2 - 4x + C d æ 2x 1 2x ö
dx 2 dx Þ ç e f (x ) - e ÷ £ 0
dx è 2 ø
dy
when x = 1, = 0. So that C = 1 æ 1 ö
dx Þ e 2x ç f( x ) - ÷ is a non-increasing function of x.
è 2ø
dy 1
Hence, = 3x 2 - 4x + 1 …(i) Þ f( x ) - is a non-increasing function of x.
dx 2
Integrating, we get
y = x 3 - 2 x 2 + x + C1, when x = 1, y = 5, l Ex. 26 Determine all curve for which the ratios of the
so that C1 = 5 length of the segment intercepted by any tangent on the
Thus, we have y = x 3 - 2x 2 + x + 5 y-axis to the length of the radius vector is a constant.
1 Sol. Let y = f ( x ) be the equation of the required curve.
Form Eq. (i), we get the critical points x = , x =1
3 ½ y - x dy ½
½
½
1 d 2y ½ dx ½
At the critical point x = , 2 is (–ve). Given that = k (a constant)
3 dx
x2 + y 2
1 2
Therefore, at x = , y has a local maximum. dy y æ y ö
3 Þ = ±k 1+ç ÷
dx x è x ø
d 2y
At x = 1, 2 is ( + ve). dv
dx Let y = vx, then v + x = v ± k 1 + v2
dx
Therefore, at x = 1, y has a local minimum.
dv dx
Also, f (1 ) = 5 Þ =±k , integrating we get
1 + v2 x
æ1ö 139
Þ f ç ÷ = Þ log v + 1 + v 2 = ± k ln x + C
è3ø 27

f ( 0 ) = 5, f (2 ) = 7 xy + x2 + y 2
Þ log = ± k ln x + C
Hence, the global maximum value = 7, the global minimum x
value = 5.
Which are the equations of the required curves.
276 Textbook of Integral Calculus

2
l Ex. 27 Let u( x ) and v ( x ) satisfy the differential equa- Þ
æ dy ö
y2 + y2 ç 2 dy
= ± k2 - y 2
÷ = k or y
du dv è dx ø dx
tions + p( x ) u = f ( x ) and + p( x ) v = g ( x ), where
dx dx y dy
Þ = ± dx
p( x ), f ( x ) and g ( x ) are continuous functions. k2 - y 2
If u ( x 1 ) > v ( x 1 ) for some x 1 and f ( x ) > g ( x ), for all x > x 1 , Integrating both the sides, we get
prove that any point ( x , y ), where x > x 1 , does not satisfy the
- k 2 - y 2 = ± x + C , since it passes through ( 0, k ) ® C = 0.
equation y = u( x ) and y = v ( x ). [IIT JEE 1997]
\ - k2 - y 2 = ± x
Sol. Given that
du dv or k2 - y 2 = x2
+ p( x ) × u = f ( x ) and + p( x ) × v = g ( x )
dx dx Þ x 2 + y 2 = k 2, is required equation of the curve.
Subtracting, we get
d (u - v )
+ p( x ) × (u - v ) = f ( x ) - g( x ) l Ex. 29 A curve passing through the point (1, 1) has the
dx
property that the perpendicular distance of the normal at
Multiplying by e ò
p ( x ) dx
, we get any point P on the curve from the origin is equal to the
d (u - v ) distance of P from x-axis. Determine the equation of the
eò + (u - v ) × p( x ) × e ò
p ( x ) dx p ( x ) dx
×
dx curve. [IIT JEE 1999]
= { f ( x ) - g( x )} × e ò
p ( x ) dx
Sol. Let P ( x, y ) be any point on the curve y = f ( x ). Then, the
d equation of the normal at P is,
{(u - v ) e ò } = { f ( x ) - g( x )} e ò
pdx p ( x ) dx
i.e.
dx 1
Y -y = - (X - x )
Since, exponential function takes only positive values and (dy / dx )
f ( x ) > g( x ) for all x > x1, RHS is + ve; x > x1 dy æ dy ö
or X +Y -çy +x ÷=0 ...(i)
\
d
{(u - v ) × e ò
p ( x ) dx
}=0 dx è dx ø
dx It is given that distance of Eq. (i) from origin = Distance from
(u - v ) × e ò
p ( x ) dx x-axis (i. e. y )
ie, is increasing function.
½ æ dy ö ½
Hence, if e ò
p ( x ) dx
= f( x ), then for x > x1 ½ 0 - çè y dx + x ÷ø ½
We have, {u( x ) - v( x )} f ( x ) > {u( x1 ) - v( x1 )} f ( x1 ) i.e. ½ ½= y
{u( x1 ) - v( x1 )} × f( x1 ) ½ æ dy ö
2 ½
i.e. u( x ) - v( x ) > > 0 [Q u( x1 ) > v( x1 )] ½ 1+ç ÷ ½
f( x ) ½ è dx ø ½
Thus, u( x ) > v( x ), " x > x1 æ dy ö
2 é æ dy ö ù
2
Þ çy + x ÷ = y 2 ê1 + ç ÷ ú
i.e. u( x ) ¹ v( x ), " x > x1 è dx ø êë è dx ø ú
û
Hence, no point ( x, y ) such that x > x1 can satisfy the equations dy
y = u ( x ) and y = v( x ). Þ x 2 + 2 xy =y2
dx
2 2
dy y - x
or =
l Ex. 28 A normal is drawn at a point P ( x , y ) of a curve. It dx 2 xy
meets the x-axis at Q. If PQ is of constant length k, then which is homogeneous differential equation and we can solve
show that the differential equation describing such curves is, by homogeneous or by total differential.
Here, using total differential,
dy
y = ± k 2 - y 2 and the equation of such a curve pass- 2 xy dy - y 2 dx = - x 2 dx
dx
ing through (0, k ). [IIT JEE 1994] x d (y 2 ) - y 2 dx
Þ = - dx
x2
Sol. Let y = f ( x ) be the curve such that the normal at P ( x, y ) to this
æ y2 ö
curve meets x-axis at Q. Then, Þ d ç ÷ = - dx
PQ = length of the normal at P è x ø
2 Integrating both the sides, we get
æ dy ö
=y 1+ç ÷ y2
è dx ø Þ =-x+C ...(ii)
x
But PQ = k
It passes through (1, 1) Þ C = 2
2
æ dy ö y2
\ y 1+ç ÷ =k \ = - x + 2 or y 2 = - x 2 + 2 x
è dx ø x
Þ x 2 + y 2 - 2 x = 0, is required equation of curve.
Chap 04 Differential Equations 277

l Ex. 30 A country has a food deficit of 10%. Its population l Ex. 31 A right circular cone with radius R and height H
grows continuously at a rate of 3% per year. Its annual food contains a liquid which evaporates at a rate proportional to
production every year is 4% more than that of the last year. its surface area in contact with air (proportionality constant
Assuming that the average food requirement per person = k > 0), find the time after which the cone is empty.
remains constant, prove that the country will become [IIT JEE 2003]
self-sufficient in food after n years, where n is the smallest Sol. Let the semi-vertical angle of the cone be q and let the height
log e 10 - log e 9 of the liquid at time ‘t’ be ‘h’ from the vertex V and radius of
integer bigger than or equal to .[IIT JEE 2000] the liquid cone be r. Let V be the volume at time t. Then,
(log e 1.04) - 0.03
O R B
Sol. Let P0 be the initial population, Q0 be its initial food produc- A
tion.
Let P be the population of the country in year t and Q be its
food production in year t.
dP 3 P dP 3 O' r
H
Þ = or = dt
dt 100 P 100
Integrating, we get h
θ
3
log P = t+C
100
At t = 0, we have P = P0 1 2
V = pr h
Þ C = log P0 3
Þ P = P0 e 0.03 t ...(i) 1 æ rö
Þ V = pr 3 cot q çQ tan q = ÷
3 è hø
It is given that the annual food production every year is 4%
more than that of last year. Let S be the surface area of the liquid in contact with air at time t.
æ 4 ö
t Then, S = pr 2
Þ Q = Q0 ç 1 + ÷
è 100 ø dV
Þ µS
Let the average consumption per person be k units. dt
æ 90 ö dV
Þ Q0 = kP0 ç ÷ = 0.9 kP0 Þ = - kS , k is constant of proportionality.
è 100 ø dt
d æ 1 3 ö 2
\ Q = 0.9 kP0 (1.04)t ...(ii) Þ ç pr cot q ÷ = - kpr
dt è 3 ø
This gives quantity of food available in year t. The population
dr
in year t is, Þ pr 2 cot q = - kpr 2 Þ cot q dr = - k dt
dt
P = P0 e 0.03 t [from Eq. (i)] 0 T
On integrating, we get cot q ò dr = - k ò0 dt
\ Consumption in year, t = kP0e 0.03 t ...(iii) R

The country will be self sufficient, if Þ R cot q = + kT , where T is required time.


Q³P Þ T = H /k (as tan q = R / H )
Þ 0.9k P0 (1.04)t ³ kP0e 0.03 t
9
l Ex. 32 Solve the equation
Þ (1.04)t ³ e 0.03 t x x
10 x ò y (t ) dt = ( x + 1) ò t y (t ) dt , x > 0.
0 0
10
Þ (1.04)t e - 0.03 t ³ Sol. Differentiating the equation w.r.t. x, we get
9 x x
xy ( x ) + 1 × ò y (t ) dt = ( x + 1 ) xy ( x ) + 1 × ò t y (t ) dt
æ 10 ö 0 0
Þ t log (1.04) - 0.03 t ³ log ç ÷
è9ø x x
i.e. ò0 y (t ) dt = x 2 y ( x ) + ò0 t y (t ) dt
æ 10 ö
Þ t {log (1.04) - 0.03 } ³ log ç ÷ Again, differentiating w.r.t. x, we get
è9ø
y ( x ) = x 2 y ¢( x ) + 2 xy ( x ) + xy ( x )
log 10 - log 9
Þ t³
log (1.04) - 0.03 x 2 dy ( x )
i.e. (1 - 3 x ) y ( x ) =
Thus, the least number of year in which country becomes self dx
sufficient. (1 - 3 x ) dx dy ( x )
i.e. =
log 10 - log 9 x2 y (x )
Þ t=
log (1.04) - 0.03 C - 1/x
Integrating, we get y = 3 e
x
278 Textbook of Integral Calculus

l Ex. 33 If (y 1 , y 2 ) are two solutions of the differential Þ - x f ¢1 ( x ) + f1( x ) = - x f ¢2 ( x ) + f 2( x )


dy Þ f1( x ) - f 2( x ) = x ( f ¢1 ( x ) - f ¢2 ( x ))
equation + P(x ) × y = Q (x ) Integrating both the sides, we get
dx
Þ ln | f1( x ) - f 2( x ) | = ln | x | + C
Then prove that y = y 1 + C (y 1 - y 2 ) is the general solution
Þ f1( x ) - f 2( x ) = ± C1 x …(i)
of the equation where C is any constant. For what relation
Now, equations of normal with equal abscissa x, are
between the constant a, b will the linear combination 1
ay 1 + by 2 also be a solution. y - f1( x ) = - (X - x )
f ¢1 ( x )
Sol. As y1, y 2 are the solutions of the differential equation; 1
dy and (y - f 2( x )) = - (X - x )
+ P ( x ) × y = Q( x ) …(i) f ¢2 ( x )
dx
As these normal intersect on the x-axis,
dy1
\ + P ( x ) × y1 = Q( x ) …(ii) x + f1( x ) × f ¢1 ( x ) = x + f 2( x ) × f ¢2 ( x )
dx
dy 2 Þ f1( x ) × f1 ¢( x ) = f 2( x ) × f 2 ¢( x ) Integrating
and + P ( x ) × y 2 = Q( x ) …(iii) Þ f ¢1 ( x ) - f 2 2( x ) = C 2
2
dx
æ dy dy1 ö C2 C l
From Eqs. (i) and (ii), ç - ÷ + P ( x ) (y - y1 ) = 0 Þ f1( x ) + f 2( x ) = =± 2 =± 2
è dx dx ø f1( x ) - f 2( x ) C1 x x
d [using Eq. (i)] …(ii)
\ (y - y1 ) + P ( x ) × (y - y1 ) = 0 …(iv)
dx From Eqs. (i) and (ii), we get
d æ l ö æ l ö
From Eqs. (ii) and (iii), (y1 - y 2 ) + P ( x ) (y1 - y 2 ) = 0 …(v) 2 f1( x ) = ± ç 2 + C1 x ÷ , 2 f 2( x ) = ± ç 2 - C1 x ÷
dx è x ø è x ø
d
(y - y1 ) We have, f1(1 ) = 1 and f 2(2 ) = 3
y - y1 2 2
From Eqs. (iv) and (v), dx = Þ f1( x ) = - x and f 2( x ) = + x
1 - y2
d y x x
(y1 - y 2 )
dx
d
(y - y1 )
d
(y1 - y 2 )
l Ex. 35 Given two curves y = f ( x ) passing through (0, 1)
Þ dx = dx x
y - y1 y1 - y 2 and y = ò f (t ) dt passing through (0, 1 / n ). The tangents

Integrating both the sides, we get drawn to both the curves at the points with equal abscissae
log (y - y1 ) = log (y1 - y 2 )
intersect on the x-axis find the curve y = f ( x ).
\ y = y1 + C (y1 - y 2 )
Sol. Equation of the tangent to the curve; y = f ( x ) is
Now, y = ay1 + by 2 will be a solutions, if
d (Y - y ) = f ¢( x ) ( X - x )
( ay1 + by 2 ) + P ( x ) ( ay1 + by 2 ) = Q( x ) x
dx Equation of tangent to the curve g( x ) = y1 = ò f (t ) dt is

æ dy ö æ dy ö
or a ç 1 + P ( x ) y1 ÷ + b ç 2 + P ( x ) y 2 ÷ = Q( x ) (Y - y1 ) = g ¢( x ) ( X - x ) = f ( x ) ( X - x )
è dx ø è dx ø
Given that tangent with equal abscissas intersect on the x-axis.
or a Q( x ) + b Q( x ) = Q( x ) [using Eqs. (ii) and (iii)]
y y
\ ( a + b ) Q( x ) = Q( x ) Þ x- =x- 1
f ¢( x ) f (x )
Hence, a + b =1
f (x ) y
Þ = 1 [Qy = f ( x )]
l Ex. 34 Find a pair of curves such that f ¢( x ) f ( x )
(a) the tangents drawn at points with equal abscissae f ( x ) f ¢( x ) g ¢( x ) f ¢( x )
Þ = Þ =
intersect on the y-axis. y1 f (x ) g( x ) f (x )
(b) the normal drawn at points with equal abscissae g ¢( x )
Þ =k Þ g( x ) = Cekx
intersect on x-axis. g( x )
(c) one curve passes through (1, 1) and other passes Þ g ¢( x ) = k Cekx Þ f ( x ) = k Cekx
through (2, 3).
y = f ( x ) passes through ( 0, 1 ) Þ kC = 1
Sol. Let the curve be y = f1( x ) and y = f 2( x ) equation of tangents
with equal abscissa, x are y1 = g( x ) passes through
(y - f1( x )) = f ¢1 ( x ) ( X - x ) 1
( 0, 1 / n ) Þ C = Þk =n
and Y - f 2 ( x ) = f ¢2 ( x ) ( X - x ) n
These tangent intersect at y-axis, Þ f ( x ) = enx
Chap 04 Differential Equations 279

l Ex. 36 A normal is drawn at a point P ( x , y ) of a curve. It dy y + x


or = , put y = vx
dx y - x
meets the x-axis and the y-axis in point A and B, respec-
1 1 dv 1 + 2v - v 2
tively, such that + = 1, where O is the origin, find the Þ x =
OA OB dx v -1
equation of such a curve passing through (5, 4). (1 - v ) dv dx
Þ + =0
Sol. The equation of the normal at ( x, y ) is 1 + 2v - v 2 x

( X - x ) + (Y - y )
dy
=0 Þ log (1 + 2v - v 2 ) + log x = C1
dx
X Y Þ x 2 + 2 xy - y 2 = C
Þ + =1 where C1 = log C
dy ( x + y dy / dx )
x+y
dx dy / dx Since, curve passes through (1, 1) ®C = 2
æ dy ö \ Required curve, x 2 - y 2 + 2 xy = 2
çx + y ÷
dy è dx ø
Þ OA = x + y , OB =
dx dy l Ex. 38 The tangent and a normal to a curve at any point
dx P meet the x and y axes at A, B, C and D respectively. Find
1 1 dy dy
Also, + =1Þ 1 + =x+y the equation of the curve passing through (1, 0) if the centre
OA OB dx dx of circle through O, C , P and B lies on the line y = x (where O
dy
Þ (y - 1 ) + (x - 1) = 0 is origin).
dx
Integrating, we get Sol. Let P ( x, y ) be a point on the curve.
(y - 1 ) 2 + ( x - 1 ) 2 = C æ dy ö
Þ C ºç x+y ,0 ÷
è dx ø
Since, the curve passes through (5, 4), C = 25.
Hence, the curve is ( x - 1 ) 2 + (y - 1 ) 2 = 25. æ dy ö
B º ç 0, y - x ÷
è dx ø
l Ex. 37 A line is drawn from a point P ( x , y ) on curve Circle passing through O, C , P and B has its centre at
mid-point of BC.
y = f ( x ), making an angle with the x-axis which is supple-
mentary to the one made by the tangent to the curve at B
P ( x , y ). The line meets the x-axis at A. Another line perpen-
dicular to the first, is drawn from P ( x , y ) meeting the y-axis P
at B. If OA = OB, where O is origin, find all curve which
passes through (1, 1). A
O C
Sol. The equation of the line through P ( x, y ) making an angle
with the x-axis which is supplementary to the angle made by
the tangent at P ( x, y ) is
dy
Y -y = - (X - x ) ...(i) D
dx
where it meets the x-axis.
Let the centre of the circle be ( a, b ).
y dx
Y = 0, X = x + Þ OA = x + y ...(ii) Þ 2a = x + y
dy
dy / dx dy dx
The line through P ( x, y ) and perpendicular to Eq. (i) is dy
and 2b = y - x
dx dx
Y -y = (X - x )
dy dy dy
and since b = a, y - x =x+y
where it meets the y-axis. dx dx
dx dx dy y - x
X = 0, Y = y - x Þ OB = y - x ...(iii) Þ =
dy dy dx y + x
Since, OA = OB
dv (1 + v 2 )
dx dx Let y = vx Þ x =-
Þ x+y =y - x dx 1+v
dy dy
1+v dx
dx Þ dv = -
or (y - x ) = (y + x ) v2 + 1 x
dy
280 Textbook of Integral Calculus

Integrating both the sides, we get 1


lim f ( x ) = 3 1/ 4 . Also, f ¢ ( x ) ³ f 3 ( x ) + , then
1+v dx x ®b - f (x )
ò v 2 + 1 dv = - ò x (a) b - a ³ p / 4 (b) b - a £ p / 4
1 2v dv dx (c) b - a £ p / 24 (d) None of these
2 ò v2 + 1
Þ dv + ò 2 =-ò
v +1 x Sol. Since,
1
Þ
1
log | v 2 + 1 | + tan - 1 | v | = - log x + C f ¢( x ) ³ f 3 ( x ) +
2 f (x )

Þ log {( v 2 + 1 ) x } + tan - 1 v = C Þ f ¢( x ) f ( x ) ³ 1 + f 4 ( x )

y f ( x ) f ¢( x )
Þ log x 2 + y 2 + tan - 1 =C Þ ³1
x 1 + f 4 (x )
As x = 1 and y = 0, On integrating w.r.t. ‘x’ from x = a to x = b.
-1
log 1 + tan 0 =C ÞC = 0 1
(tan - 1 ( f 2( x )))ba ³ (b - a )
æ y ö 2
\ Required curve, (log x 2 + y 2 ) + tan - 1 ç ÷=0
è x ø 1ì -1 2 -1 2 ü
or (b - a ) £ í lim (tan ( f ( x ))) - lim+ (tan ( f ( x )))ý
2 îx ® b- x ®a þ
l Ex. 39 If f ( x ) be a positive, continuous and differentia- or (b - a ) £ p / 24
ble on the interval (a , b ). If lim f ( x ) =1 and Hence, (c) is the correct answer.
x ®a+
#L Differential Equations Exercise 1 :
Single Option Correct Type Questions
1. If the differential equation of the family of curve given 7. The real value of m for which the substitution,
by y = Ax + Be 2 x , where A and B are arbitrary y = u m will transform the differential equation,
constants, is of the form dy
d æ dy 2x 4 y + y 4 = 4 x 6 into a homogeneous equation is
ö æ dy ö dx
(1 - 2x ) ç + ly ÷ + k ç + ly ÷ = 0, then the ordered
dx è dx ø è dx ø (a) m = 0 (b) m = 1
pair (k , l ) is (c) m = 3 / 2 (d) No value of m
(a) (2, - 2 ) (b) ( -2, 2 ) 8. The solution of the differential equation,
(c) (2, 2) (d) ( -2, - 2 )
dy 1 1
p x2 × cos - y sin = - 1,
2. A curve passes through the point æç1, ö÷ and its slope at dx x x
è 4ø
where y ® -1 as x ® ¥ is
y æy ö
any point is given by - cos 2 ç ÷ . Then, the curve has 1 1 x+1
x èx ø (a) y = sin - cos (b) y =
1
x x x sin
the equation x
æ eö x+1
(a) y = x tan -1 ç ln ÷ (b) y = x tan -1 (ln 2 ) (c) y = sin
1
+ cos
1
(d) y =
è xø 1
x x x cos
1 æ eö
(c) y = tan -1 ç ln ÷ (d) None of these x
x è xø
9. A wet porous substance in the open air loses its
3. The x-intercept of the tangent to a curve is equal to the moisture at a rate proportional to the moisture content.
ordinate of the point of contact. The equation of the If a sheet hung in the wind loses half its moisture during
curve through the point (1, 1) is the first hour, then the time when it would have lost
x x 99.9% of its moisture is (weather conditions remaining
y y
(a) ye = e (b) xe = e same)
y y (a) more than 100 h
(c) xe x =e (d) ye x = e (b) more than 10 h
(c) approximately 10 h
4. A function y = f ( x ) satisfies the condition (d) approximately 9 h
f ¢ ( x )sin x + f ( x ) cos x = 1, f ( x ) being bounded when
p/ 2 10. A curve C passes through origin and has the
x ® 0. If I = ò f ( x ) dx , then
0 property that at each point ( x , y ) on it, the normal line at
p p2 p p2 that point passes through (1, 0). The equation of a
(a) < I < (b) < I <
2 4 4 2 common tangent to the curve C and the parabola
p y 2 = 4 x is
(c) 1 < I < (d) 0 < I < 1
2 (a) x = 0 (b) y = 0
5. A curve is such that the area of the region bounded by (c) y = x + 1 (d) x + y + 1 = 0
the coordinate axes, the curve and the ordinate of any 11. A function y = f ( x ) satisfies
point on it is equal to the cube of that ordinate. The 2
2 ex
curve represents ( x + 1) × f ¢ ( x ) - 2( x + x ) f ( x ) = , " x > - 1.
(a) a pair of straight lines (b) a circle ( x + 1)
(c) a parabola (d) an ellipse If f (0) = 5, then f ( x ) is

6. The value of the constant ‘m’ and ‘c’ for which æ 3x + 5 ö x 2 æ 6x + 5 ö x 2


(a) ç ÷ ×e (b) ç ÷ ×e
y = mx + c is a solution of the differential equation è x+1ø è x+1ø
D 2 y - 3Dy - 4y = - 4 x . æ 6x + 5 ö x 2 æ 5 - 6x ö x 2
(a) is m = - 1; c = 3 / 4 (b) is m = 1; c = - 3 / 4 (c) ç ÷ ×e (d) ç ÷ ×e
è (x + 1)2 ø èx+1ø
(c) no such real m, c (d) is m = 1; c = 3 / 4
282 Textbook of Differential Calculus

12. The curve, with the property that the projection of the 16. The substitution y = z a transforms the differential
ordinate on the normal is constant and has a length equation ( x 2 y 2 - 1) dy + 2xy 3 dx = 0 into a homogeneous
equal to ‘a’, is
differential equation for
(a) x - a ln ( y 2 - a 2 + y ) = C (a) a = - 1 (b) 0
2 2 (c) a = 1 (d) No value of a
(b) x + a - y = C
17. A curve passing through (2, 3) and satisfying the
(c) (y - a ) 2 = Cx x
differential equation ò ty (t ) dt = x 2 y( x ), ( x > 0) is
(d) ay = tan -1 ( x + C ) 0
9
(a) x 2 + y 2 = 13 (b) y 2 = x
13. The differential equation corresponding to the family of 2
curves y = e x (ax + b ) is x2 y 2
(c) + =1 (d) xy = 6
d 2y dy d 2y dy 8 18
(a) 2 + 2 -y = 0 (b) 2 - 2 +y =0
dx dx dx dx 18. Which one of the following curves represents the
2
d y dy 2
d y dy solution of the initial value problem Dy = 100 - y, where
(c) +2 +y =0 (d) -2 -y = 0 y (0) = 50?
dx 2 dx dx 2 dx
y y
14. The equation to the orthogonal trajectories of the
2
system of parabolas y = ax is 100 100
2 2 50
x y (a) (b)
(a) + y2 =C (b) x 2 + =C 50
2 2
x x
x2 y2 O O
(c) -y2 =C (d) x 2 - =C
2 2
1 y y
15. A function f ( x ) satisfying ò f (tx ) dt = n f ( x ), where
0 100 100
x > 0, is
1-n n (c) 50 (d) 50
(a) f ( x ) = C × x n (b) f ( x ) = C × x n - 1
1 x x
O O
(c) f ( x ) = C × x n (d) f ( x ) = C × x (1-n )

#L Differential Equations Exercise 2 :


More than One Option Correct Type Questions
dy 3 21. Which of the following pair(s) is/are orthogonal?
19. The differential equation x + =y2
dx dy (a) 16 x 2 + y 2 = C and y 16 = kx
dx (b) y = x + Ce - x and x + 2 = y + ke - y
(a) is of order 1 (b) is of degree 2 (c) y = Cx 2 and x 2 + 2y 2 = k
(c) is linear (d) is non-linear
(d) x 2 - y 2 = C and xy = k
20. The function f ( x ) satisfying the equation
22. Family of curves whose tangent at a point with its
f 2 ( x ) + 4 f ¢ ( x ) × f ( x ) + [ f ¢ ( x )]2 = 0. p
( 2- 3 ) x
intersection with the curve xy = c 2 form an angle of is
(a) f ( x ) = C × e 4
(a) y 2 - 2 xy - x 2 = k
(b) f ( x ) = C × e ( 2 + 3 )x
(b) y 2 + 2 xy - x 2 = k
3 - 2) x
(c) f ( x ) = C × e ( æxö
(c) y = x - 2c tan -1 ç ÷ + k
(d) f ( x ) = C × e - ( 2 + 3 )x èc ø
where C is an arbitrary constant. ½c + x½
(d) y = c ln ½ ½- x + k
½c - x½
Chap 04 Differential Equations 283

23. The general solution of the differential equation, 27. The graph of the function y = f ( x ) passing through the
æ dy ö æy ö point (0, - 1) and satisfying the differential equation
x ç ÷ = y × log ç ÷ is
è dx ø èx ø dy
+ y cos x = cos x is such that
(a) y = xe1 - Cx dx
(b) y = xe1 + Cx (a) it is a constant function
(b) it is periodic
(c) y = ex × eCx
(c) it is neither an even nor an odd function
(d) y = xeCx (d) it is continuous and differentiable for all x
where C is an arbitrary constant.
28. A function y = f ( x ) satisfying the differential equation
24. Which of the following equation(s) is/are linear? dy sin 2 x
dy y × sin x - y cos x + =0
(a) + = ln x dx x2
dx x
æ dy ö is such that, y ® 0 as x ® ¥, then the statement which is
(b) y ç ÷ + 4 x = 0 correct?
è dx ø
p/2 p
(c) dx + dy = 0 (a) lim f ( x ) = 1 (b) ò f ( x ) dx is less than
x®0 0 2
d 2y
(d) 2 = cos x p/2
dx (c) ò f ( x ) dx is greater than unity
0
25. The equation of the curve passing through (3, 4) and (d) f ( x ) is an odd function
satisfying the differential equation,
2
29. Identify the statement(s) which is/are true?
æ dy ö dy d 2y
yç ÷ + (x - y) - x =0 (a) The order of differential equation 1 + = x is 1.
è dx ø dx dx 2
can be (b) Solution of the differential equation
(a) x - y + 1 = 0
xdy - ydx = x 2 + y 2 dx is y + x 2 + y 2 = Cx 2.
(b) x 2 + y 2 = 25
d 2y æ dy ö
(c) x 2 + y 2 - 5 x - 10 = 0 (c) =2 ç - y ÷ is differential equation of family of
dx 2 è dx ø
(d) x + y - 7 = 0 x
curves y = e ( A cos x + B sin x ).
26. Identify the statement(s) which is/are true?
y (d) The solution of differential equation
(a) f ( x, y ) = e y /x + tan is homogeneous of degree zero. -1 dy -1 -1
x (1 + y 2 ) + ( x - 2e tan y
) = 0 is xe tan y
= e 2 tan y
+ k.
2 dx
y y y
(b) x × log dx + sin -1 dy = 0 is homogeneous
x x x 30. Let y = ( A + Bx ) e 3 x is a solution of the differential
differential equation. d 2y dy
2 equation +m + ny = 0, m, n Î I , then
(c) f ( x, y ) = x + sin x × cosy is not homogeneous. dx 2 dx
2 2 2 3
(d) ( x + y ) dx - ( xy - y ) dy = 0 is a homogeneous (a) m = - 6 (b) n = - 6
(c) m = 9 (d) n = 9
differential equation.
284 Textbook of Differential Calculus

#L Differential Equations Exercise 3 :


Statement I and II Type Questions

Directions 35. Statement I The order of the differential equation


(Q. Nos. 31 to 40) whose general solution is
For the following questions, choose the correct answers from y = c1 cos2 x + c 2 sin 2 x + c 4e 2x + c 5e 2x + c 6 is 3.
the codes (a), (b), (c) and (d) defined as follows : Statement II Total number of arbitrary parameters in
(a) Statement I is true, Statement II is true and Statement II is the given general solution in the Statement I is 6.
the correct explanation for Statement I.
(b) Statement I is true, Statement II is true and Statement II is d 2y dy
not the correct explanation for Statement I. 36. Consider differential equation ( x 2 + 1) × 2
= 2x ×
dx dx
(c) Statement I is true, Statement II is false.
(d) Statement I is false, Statement II is true. Statement I For any member of this family y ® ¥ as
x ® ¥.
31. A curve C has the property that its initial ordinate of
Statement II Any solution of this differential equation
any tangent drawn is less than the abscissa of the point
is a polynomial of odd degree with positive coefficient of
of tangency by unity.
maximum power.
Statement I Differential equation satisfying the curve
is linear. 37. Statement I Order of differential equation of family of
Statement II Degree of differential equation is one. parallel whose axis is parallel to Y -axis and latusrectum
is fixed is 2.
32. Statement I Differential equation corresponding to all Statement II Order of first equation is same as actual
lines, ax + by + c = 0 has the order 3. number of arbitrary constant present in differential
Statement II General solution of a differential equation.
equation of nth order contains n independent arbitrary
constants. 38. Statement I The differential equation of all
d 2x
33. Statement I Integral curves denoted by the first order non-vertical lines in a plane is = 0.
dy 1 dy 2
linear differential equation - y = - x are family of
dx x Statement II The general equation of all non-vertical
parabolas passing through the origin. lines in a plane is ax + by = 1, where b ¹ 0.
Statement II Every differential equation geometrically 39. Statement I The order of differential equation of all
represents a family of curve having some common conics whose centre lies at origin is, 2.
property.
Statement II The order of differential equation is same
æx ö as number of arbitrary unknowns in the given curve.
34. Statement I The solution of (y dx - x dy ) cot ç ÷
èy ø
2 æx ö nx 40. Statement I y = a sin x + b cos x is general solution of
= ny dx is sin ç ÷ = Ce
èy ø y ¢¢ + y = 0.
Statement II Such type of differential equations can Statement II y = a sin x + b cos x is a trigonometric
only be solved by the substitution x = vy. function.
Chap 04 Differential Equations 285

#L Differential Equations Exercise 4 :


Passage Based Questions

Passage I Passage III


(Q. Nos. 41 to 43) (Q. Nos. 47 to 49)
Let y = f ( x ) satisfies the equation If any differential equation in the form
-x x
f (x ) = (e + e ) cos x - 2x - ò ( x - t ) f ¢ ( t )dt.
x f ( f1 ( x, y )) d ( f1 ( x, y )) + f( f 2 ( x, y )) d ( f 2 ( x, y )) + K = 0
0
then each term can be integrated separately.
41. y satisfies the differential equation For example,
dy
(a) + y = e x (cos x - sin x ) - e - x (cos x + sin x ) æ xö æ xö 1 æ xö
2
dx
dy
ò sin xy d ( xy ) + ò ç ÷ d ç ÷ = - cos xy + ç ÷ + C
è yø è yø 2 è yø
(b) - y = e x (cos x - sin x ) + e - x (cos x + sin x )
dx
dy 47. The solution of the differential equation
(c) + y = e x (cos x + sin x ) - e - x (cos x - sin x )
dx xdy - y dx = x 2 - y 2 dx is
dy
- y = e x (cos x - sin x ) + e - x (cos x - sin x )
y y
(d) sin -1 sin -1
dx (a) Cx = e x (b) xe x =C
y
sin -1
42. The value of f ¢ (0) + f ¢¢(0) equals to (c) x + e x =C (d) None of these
(a) -1 (b) 2
(c) 1 (d) 0 48. The solution of the differential equation
( xy 4 + y ) dx - xdy = 0 is
43. f ( x ) as a function of x equals to 2 3
x3 1 æ x ö x4 1 æ x ö
-x ex 2 (a) + ç ÷ =C (b) + ç ÷ =C
(a) e (cos x - sin x ) + (3 cos x + sin x ) + e - x 4 2 èy ø 4 3 èy ø
5 5
3 2
ex 2 x4 1 æ x ö x3 1 æ x ö
(b) e - x (cos x + sin x ) + (3 cos x - sin x ) - e - x (c) - ç ÷ =C (d) - ç ÷ =C
5 5 4 2 èy ø 4 2 èy ø
ex 2
(c) e - x (cos x - sin x ) +(3 cos x - sin x ) + e - x 49. Solution of differential equation
5 5
(2x cos y + y 2 cos x ) dx + (2y sin x - x 2 sin y ) dy = 0 is
-x ex 2
(d) e (cos x + sin x ) + (3 cos x - sin x ) - e - x (a) x 2 cosy + y 2 sin x = C
5 5
(b) x cosy - y sin x = C
Passage II (c) x 2 cos2 y + y 2 sin 2 x = C
(Q. Nos. 44 to 46) (d) None of the above
2
d y Passage IV
For certain curves y = f ( x ) satisfying = 6x - 4, f ( x ) has
dx 2 (Q. Nos. 50 to 52)
local minimum value 5 when x = 1.
dy
44. Number of critical point for y = f ( x ) for x Î[0, 2] Differential equation = f ( x ) g ( x ) can be solved by
dx
(a) 0 (b) 1 dy
(c) 2 (d) 3
separating variable = f ( x ) dx.
g( y)
45. Global minimum value of y = f ( x ) for x Î[0, 2] is
50. The equation of the curve to the point (1, 0) which
(a) 5 (b) 7
(c) 8 (d) 9
satisfies the differential equation (1 + y 2 ) dx - xy dy = 0
is
46. Global maximum value of y = f ( x ) for x Î[0, 2] is (a) x 2 + y 2 = 1 (b) x 2 - y 2 = 1
(a) 5 (b) 7 2 2
(c) x + y = 2 (d) x 2 - y 2 = 2
(c) 8 (d) 9
286 Textbook of Differential Calculus

dy 1+y2 53. If C 1 , C 2 Î C
51. Solution of the differential equation + = 0 is
dx 1- x2 C 1 : Curve is passing through (1, 0)
(a) tan -1 y + sin -1 x = C C 2 : Curve is passing through ( -1, 0)
(b) tan -1 x + sin -1 y = C The number of common tangents for C 1 and C 2 is
-1 -1
(c) tan y × sin x =C (a) 1 (b) 2
(d) tan -1
y - sin -1
x =C (c) 3 (d) None of these

dy 54. If C 3 Î C
52. If = 1 + x + y + xy and y ( -1) = 0, then y is equal to x y
dx C 3 : is passing through (2, 4). If + = 1. is tangent to
(1 - x ) 2 (1 + x ) 2 a b
(a) e 2 (b) e 2 -1 C 3 , then
(c) ln (1 + x ) - 1 (d) 1 + x (a) 25a + 10b 2 - ab 2 = 0 (b) 25a + 10b - 13ab = 0
(c) 13a + 25b - 16ab = 0 (d) 29a + b + 13ab = 0
Passage V 55. If common tangents of C 1 and C 2 form an equilateral
(Q. Nos. 53 to 55) triangle, where C 1 , C 2 Î C and C 1 : Curve passes through
Let C be the set of curves having the property that the point of (2, 0), then C 2 may passes through
intersection of tangent with y-axis is equidistant from the point (a) ( -1 / 3,1 / 3 ) (b) ( -1 / 3,1 )
of tangency and origin (0, 0) (c) ( -2 / 3, 4 ) (d) ( -2 / 3, 2 )

#L Differential Equations Exercise 5 :


Matching Type Questions
56. Match the following :

Column I Column II

xdx + ydy a2 - x 2 - y2 (p) C -1/ x


(A) = y= e
xdy - ydx x 2 + y2 x3

(B) dy p æpö 3 3 (q) ì æ y öü


Solution of cos2 x - tan 2x × y = cos4 x , where | x | < and y ç ÷ = x 2 + y2 = a sin íC + tan -1 ç ÷ý
dx 4 è 6ø 8 î è x øþ

(C) The equation of all possible curves that will cut each member of the family (r) x 2 + y2 + Cy = 0
of circles x 2 + y2 - 2cx = 0 at right angle
x x sin 2x
(D) Solution of the equation x ò0 y(t ) dt = (x + 1) ò0 ty (t ) dt, x > 0 is (s) y=
2 (1 - tan 2 x )

57. Match the following :

Column I Column II

(A) Circular plate is expanded by heat from radius 5 cm to 5.06 cm. Approximate increase in (p) 4
area is

(B) Side of cube increasing by 1%, then percentage increase in volume is (q) 0.6p
2
(C) If the rate of decrease of x - 2x + 5 is twice the rate of decrease of x, then x is equal to (r) 3
2

(D) Rate of increase in area of equilateral triangle of side 15 cm, when each side is increasing (s) 3 3
at the rate of 0.1 cm/s; is 4
Chap 04 Differential Equations 287

58. Match the following :


Column I Column II
(A) The differential equation of the family of curves y = e ( A cos x + B sin x ), where A, B
x
(p) 2, 1
are arbitrary constants, has the degree n and order m. Then, the values of n and m are,
respectively
(B) The degree and order of the differential equation of the family of all parabolas whose (q) 1, 1
axis is the x-axis, are respectively
(C) The order and degree of the differential equations of the family of circles touching the (r) 2, 2
x-axis at the origin, are respectively
(D) The degree and order of the differential equation of the family of ellipse having the same (s) 1, 2
foci, are respectively.

#L Differential Equations Exercise 6 :


Single Integer Answer Type Questions
59. Find the constant of integration by the general solution 62. If f( x ) is a differential real-valued function satisfying
of the differential equation (2x 2 y - 2y 4 ) dx f¢ ( x ) + 2f ( x ) £ 1, then the value of 2f( x ) is always less
3 3
+ (2x + 3xy ) dy = 0 if curve passes through (1, 1). than or equal to ……… .

60. A tank initially contains 50 gallons of fresh water. Brine 63. The degree of the differential equation satisfied by the
contains 2 pounds per gallon of salt, flows into the tank curves 1 + x - a 1 + y = 1, is ……… .
at the rate of 2 gallons per minutes and the mixture kept 64. Let f ( x ) be a twice differentiable bounded function
uniform by stirring, runs out at the same rate. If it will
take for the quantity of salt in the tank to increase from satisfayi 2 f 5 ( x ) × f ¢ ( x ) + 2 ( f ¢ ( x )) 3 × f 5 ( x ) = f ¢ ¢ ( x ). If
40 to 80 pounds (in seconds) is 206l, then find l. f ( x ) is bounded in between y = k , and y = k 2 , Then the
number of integers between k 1 and k 2 is/are (where
61. If f : R - {-1} ® and f is differentiable function which f (0) = f ¢ (0) = 0).
satisfies :
f {x + f (y ) + xf (y )) = y + f ( x )+yf ( x ) " x , y ÎR - {-1}, 65. Let y( x ) be a function satisfying d 2 y / dx 2 - dy / dx
+e 2 x = 0, y(0) = 2 and y ¢ (0) = 1. If maximum value of y( x )
then find the value of 2010 [1 + f (2009 )].
is y(a ), Then Integral part of (2a) is …… .

Differential Equations Exercise 7 :


Subjective Type Questions
66. Find the time required for a cylindrical tank of radius r 67. The hemispherical tank of radius 2 m is initially full of
and height H to empty through a round hole of area ‘a’ water and has an outlet of 12 cm 2 cross-sectional area at
at the bottom. The flow through a hole is according to the bottom. The outlet is opened at some instant. The
flow through the outlet is according to the law
the law U (t ) = u 2gh(t ) where v (t ) and h(t ) are
v (t ) = 0 × 6 2gh(t ), where v (t ) and h(t ) are respectively
respectively the velocity of flow through the hole and velocity of the flow through the outlet and the height of
the height of the water level above the hole at time t and water level above the outlet at time t and g is the
g is the acceleration due to gravity. acceleration due to gravity. Find the time it takes to
empty the tank.
288 Textbook of Differential Calculus

68. Let f : R + ® R satisfies the functional equation where P and Q are function of x alone and y 2 = y 1 z,
Q
xy - x - y y x + -ò dx
f ( xy ) = e (e f ( x ) + e f (y )) " x , y Î R . If then prove that z = 1 + c × e y1
,
f ¢ (1) = e , determine f ( x ). where c is an arbitrary constant.
69. Let y = f ( x ) be curve passing through (1, 3 ) such that 71. Let y = f ( x ) be a differentiable function " x Î R and
tangent at any point P on the curve lying in the first satisfies :
quadrant has positive slope and the tangent and the 1 1
f ( x ) = x + ò x 2 z f (z ) dz + ò x z 2 f (z ) dz.
normal at the point P cut the x-axis at A and B 0 0
respectively so that the mid-point of AB is origin. Find Determine the function.
the differential equation of the curve and hence
determine f ( x ). 72. If f : R - {- 1} ® R and f is differentiable function
satisfies :
70. If y 1 and y 2 are the solution of differential equation
f (( x ) + f (y ) + x f (y )) = y + f ( x ) + yf ( x ) " x ,
dy / dx + Py = Q, y Î R - {- 1} Find f ( x ).

#L Differential Equations Exercise 8 :


Questions Asked in Previous 10 Years’ Exams

(i) JEE Advanced & IIT-JEE


73. A solution curve of the differential equation 76. Consider the family of all circles whose centres lie on the
( x 2 + xy + 4 x + 2y + 4 )
dy
- y 2 = 0, x > 0, passes straight line y = x . If this family of circles is represented by
dx the differential equation Py ¢ ¢+ Qy ¢ + 1 = 0, where P, Q are
through the point (1, 3). Then, the solution curve dy d 2y
the functions of x, y and y ¢ (here, y ¢ = , y ¢¢ = 2 ),
[More than One Correct 2016] dx dx
(a) intersects y = x + 2 exactly at one point then which of the following statement(s) is/are true?
(b) intersects y = x + 2 exactly at two points [More than One correct 2015]
(c) intersects y = ( x + 2 ) 2 (a) P = y + x
(d) does not intersect y = ( x + 3 ) 2 (b) P = y - x
(c) P + Q = 1 - x + y + y ¢+ (y ¢ ) 2
74. Let f : (0, ¥ ) ® R be a differentiable function such that
f (x ) (d) P – Q = x + y – y ¢ – (y ¢ ) 2
f ¢( x ) = 2 - for all x Î (0, ¥ ) and f (1) ¹ 1. Then
x [More than One Correct 2016] 77. The function y = f ( x ) is the solution of the differential
æ1ö dy xy x 4 + 2x
(a) lim f ¢ç ÷ = 1 equation + 2 = in ( -1, 1) satisfying
x ® 0+ èxø dx x - 1 1- x 2
æ1ö 3 /2
(b) lim x f ç ÷ = 2 f (0) = 0. Then, ò f ( x ) dx is [Only One correct 2014]
x ® 0+ èxø
3
-
(c) lim x 2 f ¢( x ) = 0 2
x ® 0+ p 3 p 3 p 3 p 3
(d) | f ( x )| £ 2 for all x Î( 0, 2 ) (a) - (b) - (c) - (d) -
3 2 3 4 6 4 6 2
75. Let y( x ) be a solution of the differential equation 78. Let f : [1/2, 1] ® R (the set of all real numbers) be a
(1 + e x )y ¢ + ye x = 1. If y(0) = 2, then which of the
positive, non-constant and differentiable function such
following statement(s) is/are true?
[More than One Correct 2015] that f ¢ ( x ) < 2 f ( x ) and f (1 / 2) = 1 . Then, the value of
1
(a) y ( -4 ) = 0 ò f ( x ) dx lies in the interval [Only One Correct 2013]
1/ 2
(b) y ( -2 ) = 0
(a) (2e - 1, 2e ) (b) (e - 1, 2e - 1 )
(c) y ( x ) has a critical point in the interval ( -1 , 0 )
æe - 1 ö æ e - 1ö
(d) y ( x ) has no critical point in the interval ( -1 , 0 ) (c) ç , e - 1÷ (d) ç 0, ÷
è 2 ø è 2 ø
Chap 04 Differential Equations 289

p
79. A curve passes through the point æç1, ö÷. Let the slope of (c) I is false and II is true (d) Both I and II are false
6ø è
y æy ö 84. If y( x ) satisfies the differential equation
the curve at each point ( x , y ) be + sec ç ÷, x > 0. Then,
x èx ø y ¢ - y tan x = 2 x sec x and y(0), then
the equation of the curve is [Only One Correct 2013] [More than One Correct 2016]
æy ö 1 æy ö æpö p2 æpö p
2
(a) sin ç ÷ = log x + (b) cosec ç ÷ = log x + 2 (a) y ç ÷ = (b) y ¢ ç ÷ =
èxø 2 èxø è 4ø 8 2 è 4 ø 18
2 2
æ 2y ö æ 2y ö 1 æpö p æ p ö 4p 2p
(c) sec ç ÷ = log x + 2 (d) cos ç ÷ = log x + (c) y ç ÷ = (d) y ¢ ç ÷ = +
èxø èxø 2 è3ø 9 è3ø 3 3 3

n Directions (Q. Nos. 80 to 83) Let f : [0, 1] ® R (the set of 85. Let y ¢ ( x ) + y( x ) g ¢ ( x ) = g ( x ) g ¢ ( x ), y(0) = 0, x Î R, where
d f (x )
all real numbers) be a function. Suppose the function f is f ¢ ( x ) denotes and g ( x ) is a given non-constant
twice differentiable, f (0) = f (1) = 0 and satisfies dx
f ¢¢( x ) - 2f ¢ ( x ) + f ( x ) ³ e x , x Î [0, 1] differentiable function on R with g (0) = g (2) = 0. Then,
[Passage Based Questions 2013] the value of y(2) is …… [Integer Type 2011]
-x
80. If the function e f ( x ) assumes its minimum in interval 86. Let f : R ® R be a continuous function, which satisfies
[0, 1] at x = 1 / 4, then which of the following is true? x
1 3 1 f (x ) = ò f (t ) dt . Then, the value of f (ln 5) is … .
0
(a) f ¢( x ) < f ( x ), < x < (b) f ¢( x ) > f ( x ), 0 < x < [Integer Type 2009]
4 4 4
1 3 Direction For the following question, choose the correct
(c) f ¢( x ) < f ( x ), 0 < x < (d) f ¢( x ) < f ( x ), < x < 1
n

4 4 answer from the codes (a), (b), (c) and (d) defined as
81. Which of the following is true? follows
1 1 (a) Statement I is true, Statement II is also true; Statement II is
(a) 0 < f ( x ) < ¥ (b) - < f (x ) < the correct explanation of Statement I.
2 2
1 (b) Statement I is true, Statement II is also true; Statement II is
(c) - < f (x ) < 1 (d) - ¥ < f ( x ) < 0 not the correct explanation of Statement I.
4
(c) Statement I is true; Statement II is false.
82. Which of the following is true?
(d) Statement I is false; Statement II is true.
(a) g is increasing on (1, ¥ )
(b) g is decreasing on (1, ¥ ) 87. Let a solution y = y( x ) of the differential equation
(c) g is increasing on (1, 2 ) and decreasing on (2, ¥ ) 2
(d) g is decreasing on (1, 2 ) and increasing on (2, ¥ ) x x 2 - 1 dy - y y 2 - 1 dx = 0 satisfy y(2) =
3
æ pö
83. Consider the statements. Statement I y( x ) = sec çsec -1 x - ÷ and
I. There exists some x Î R such that, è 6ø
f ( x ) + 2x = 2(1 + x 2 ) 1 2 3 1
Statement II y( x ) is given by = - 1- 2
y x x
II. There exists some x Î R such that,
[Statement Based Questions 2008]
2 f ( x ) + 1 = 2x (1 + x )
(a) Both I and II are true (b) I is true and II is false

(ii) JEE Main & AIEEE


88. If a curve y = f ( x ) passes through the point (1, - 1) and 90. Let the population of rabbits surviving at a time t be
satisfies the differential equation, y(1 + xy )dx = x dy, governed by the differential equation
æ 1ö dp (t ) 1
then f ç - ÷ is equal to [2016 JEE Main] = p (t ) - 200. If p(0) = 100, then p (t ) is equal to
è 2ø dt 2 [2014 JEE Main]
2 4 2 4 t
(a) - (b) - (c) (d)
5 5 5 5 (a) 400 - 300 e 2
t
-
89. Let y( x ) be the solution of the differential equation (b) 300 - 200 e 2

dy t
( x log x ) + y = 2x log x , ( x ³ 1). Then, y(e ) is equal to (c) 600 - 500 e 2
dx [2015 JEE Main] t
-
(a) e (b) 0 (c) 2 (d) 2e (d) 400 - 300 e 2
290 Textbook of Differential Calculus

91. At present, a firm is manufacturing 2000 items. It is the total life in years of the equipment. Then, the scrap
estimated that the rate of change of production P with value V (T ) of the equipment is [2010 AIEEE]
respect to additional number of workers x is given by kT 2 k (T - t ) 2
dP (a) I - (b) I -
= 100 - 12 x . If the firm employees 25 more workers, 2 2
dx 1
(c) e -kT 2
(d) T -
then the new level of production of items is k
[2013 JEE Main]
(a) 2500 (b) 3000 95. Solution of the differential equation
(c) 3500 (d) 4500 p
cos x dy = y (sin x - y ) dx , 0 < x < , is [2010 AIEEE]
92. The population p (t ) at time t of a certain mouse species 2
dp (t ) (a) sec x = (tan x + C ) y
satisfies the differential equation = 0.5(t ) - 450. If (b) y sec x = tan x + C
dt
(c) y tan x = sec x + C
p(0) = 850, then the time at which the population (d) tan x = (sec x + C ) y
becomes zero is [2012 AIEEE]
(a) 2 log 18 (b) log 9 96. The differential equation which represents the family of
1
(c) log 18 (d) log 18
curves y = c 1e c 2x , where c 1 and c 2 are arbitrary
2 constants, is [2009 AIEEE]
dy (a) y ¢ = y 2 (b) y ¢¢ = y ¢y
93. If = y + 3 > 0 and y (0) = 2, then y (log 2) is equal to
dx [2011 JEE Main] (c) yy ¢¢ = y ¢ (d) yy ¢¢ = (y ¢ ) 2
(a) 5 (b) 13 97. The differential equation of the family of circles with
(c) -2 (d) 7
fixed radius 5 units and centre on the line y = 2 is
94. Let I be the purchase value of an equipment and V (t ) be [AIEEE 2008]
2 2
the value after it has been used for t years. The value (a) ( x - 2 )y ¢ = 25 - ( y - 2 )
V (t ) depreciates at a rate given by differential equation (b) ( y - 2 )y ¢2 = 25 - ( y - 2 ) 2
dV (t ) (c) ( y - 2 ) 2y ¢2 = 25 - ( y - 2 ) 2
= - k (T - t ), where k > 0 is a constant and T is
dt (d) ( x - 2 ) 2y ¢2 = 25 - ( y - 2 ) 2
Answers
Exercise for Session 1 26. (a,b,c) 27. (a,b,d) 28. (a,b,c)
1. (a) 2. (a) 3. (d) 4. (a) 5. (d) 29. (b,c,d) 30. (a,d) 31. (b)
6. (a) 7. (b) 8. (a) 9. (a) 10. (b) 32. (d) 33. (d) 34. (c) 35. (c) 36. (a)
37. (b) 38. (d) 39. (d) 40. (b) 41. (a)
Exercise for Session 2 42. (d) 43. (c) 44. (c) 45. (a) 46. (b)
47. (a) 48. (b) 49. (a) 50. (b) 51. (a)
1. (b) 2. (c) 3. (a) 4. (a) 5. (c)
52. (b) 53. (c) 54. (a) 55. (a)
6. (a) 7. (a) 8. (a) 9. (a) 10. (a)
56. (A) ® (q), (B) ® (s), (C) ® (r), (D) ® (p)
11. (b) 12. (c) 13. (a) 14. (b) 15. (b)
57. (A) ® (q), (B) ® (r), (C) ® (p), (D) ® (s)
Exercise for Session 3 58. (A) ® (s), (B) ® (s), (C) ® (q), (D) ® (p)
59. (1) 60. (8) 61. (1) 62. (1) 63. (1)
1. (a) 2. (a) 3. (b) 4. (b) 5. (c)
64. (3) 65. (1)
6. (c) 7. (a) 8. (c) 9. (a) 10. (c)
pr 2 2 H
66. t =
Exercise for Session 4 ma g
1. (b) 2. (d) 3. (a) 4. (b) 5. (a) 7p ´ 105
67. t =
6. (a) 7. (c) 8. (b) 9. (a) 10. (d) 135 g
68. f (x) = ex log x
Exercise for Session 5
69. x + f (x) f ¢ (x) = x2 + f 2 (x) and f 2 (x) = 1 + 2x
1. (a) 2. (c) 3. (c) 4. (a) 5. (c)
20x
6. (a) 7. (b) 8. (c) 71. f (x) = (4 + 9x)
119
-x
Chapter Exercises 72. f (x) =
1+ x
1. (a) 2. (a) 3. (a) 4. (a) 5. (c)
73. (a, d) 74. (a,d) 75. (a, c) 76. (b, c) 77. (b)
6. (b) 7. (c) 8. (a) 9. (c) 10. (a)
78. (d) 79. (a) 80. (c) 81. (d) 82. (b)
11. (b) 12. (a) 13. (b) 14. (a) 15. (a)
83. (c) 84. (d) 85. (0) 86. (0) 87. (c)
16. (a) 17. (d) 18. (b) 19. (a,b,d)
88. (d) 89. (c) 90. (a) 91. (c) 92. (a)
20. (c,d) 21. (a,b,c,d) 22. (b,c,d)
93. (d) 94. (a) 95. (a) 96. (d) 97. (c)
23. (a,b,c) 24. (a,c,d) 25. (a,b)
v - v + v2

Solutions
dv v
x= -v =
dx 1 - v 1 -v
1 -v dx
ò v 2 dv = ò x
1
- - ln v = ln x + C
v
1. y × e -2x = Ax e -2x + B x y x
- - ln = ln x + C Þ - = ln y + C
e -2x × y1 - 2ye -2x = A(e -2x - 2 x e -2x ) y x y
Cancelling e -2x throughout x = 1, y = 1 Þ C = - 1
y1 - 2y = A(1 - 2 x ) …(i) x
1 - = ln y Þ y = e × e - x /y
Differentiating again y 2 - 2y1 = - 2 A y
2y - y 2 e
Þ A= 1 e -x /y = Þ ye x /y = e
2 y
On substituting A in Eq. (i) dy
2(y 2 - 2y ) = (2y1 - y 2 ) (1 - 2 x ) 4. sin x + y cos x = 1
dx
2y1 - 4y = 2y1(1 - 2 x ) - (1 - 2 x )y 2
d æ dy ö æ dy ö
(1 - 2 x ) ç - 2y ÷ + 2 ç - 2y ÷ = 0 π/2
dx è dx ø è dx ø
Hence, k = 2 and l = - 2
1
Þ Ordered pair (k, l ) º (2, - 2 )
dy y y
2. = - cos2 y = vx
dx x x
dv π/2
v+x = v - cos2 v O
dx
dv dx dy
ò cos2 v + ò x = C Þ tanv + ln x = C dx
+ y cot x = cosec x

IF = e ò
y cotx dx
tan + ln x = C = e ln (sin x ) = sin x
x
y sin x = ò cosec x × sin x dx
p y e
If x = 1, y = Þ C = 1 Þ tan = 1 - ln x = ln
4 x x y sin x = x + C
æ e ö If x = 0, y is finite \ C = 0
y = x tan -1 ç ln ÷
è xø x
y = x (cosec x ) =
3. Y - y = m( X - x ). For X -intercept Y = 0 sin x
y p2 p
X =x- Now, I < and I >
m 4 2
y
Therefore, x - =y p p2
m Hence, <I <
y 2 4
x dy
ò0 f (x ) dx = y
3
5. . Differentiating f ( x ) = 3y 2 ×
dx

P(x,y)
(x,y)

(x,y)
x
O
dy
y = 3y 2 Þ y =0 (rejected)
dy y dx
or =
dx x - y or 3y dy = dx
dv v
Put y = vx v + x = 3y 2
dx 1 - v = x + C Þ Parabola
2
Chap 04 Differential Equations 293

dy d 2y dy 1 - x
6. y = mx + c; = m; 2 = 0 \ =
dx dx dx y

d 2y dy y2 x2
Substituting in -3 - 4y = - 4 x =x- +C …(ii)
dx 2
dx 2 2
Eq. (ii) passes through (0, 0).
0 - 3m - 4 (mx + c ) = - 4 x
Thus, C = 0
- 3m - 4c - 4mx = - 4 x x 2 + y 2 - 2x = 0
- (3m + 4c ) = 4 x(m - 1 ) …(i) Now, tangent to y 2 = 4 x
Eq. (i) is true for all real x, if m = + 1 and c = - 3 / 4. 1
dy dy y = mx + …(iii)
7. y = um Þ = mum -1 m
dx dx If it touches the circle
du
Since, 2 x 4 × um × mum -1 × + u 4m = 4 x 6 y
dx
du 4 x 6 - u 4m
=
dx 2m x 4 u 2m -1
x
O (1,0)
3
Þ 4m = 6 Þ m =
2
3
and 2m - 1 = 2 Þ m =
2
x 2 + y 2 - 2x = 0
dy y 1 1 1
8. - tan = - sec × 2 ½m + (1 / m )½
dx x 2 x x x Then, ½ 2
½= 1
½ 1+m ½
1 1
-ò tan dx 1 1
IF = e x2 x
= sec Þ y ×sec
x x Þ 1 + m2 = m2
2 æ1ö 1 1 Þ m®¥
= - ò sec ç ÷ 2 dx = tan + C
èxø x x Hence, tangent is y-axis ie, x = 0.
If y ® - 1, then x ® ¥ 2 x( x + 1 ) ex
2

1 1 11. f ¢( x ) - f (x ) =
Þ C = - 1 Þ y = sin - cos x+1 (x + 1)2
x x
IF = e ò
- 2x dx 2
dM = e -x
9. = - KM Þ M = Ce -kt when t = 0; M = M0
dt 2 dx
\ f (x ) × e -x = ò
Þ C = M0 (x + 1)2
Þ M = M0e -kt 2 1
M Þ f (x ) × e -x = - +C
when t = 1, M = 0 x+1
2
At x = 0, f ( 0 ) = 5 Þ C = 6
Þ k = ln2
æ 6x + 5 ö x 2
Therefore, M = M0e -t ln 2 \ f (x ) = ç ÷ ×e
è x+1ø
M
when M = 0 , then t = log 2 1000 = 9.98 12. Ordinate = PM. Let P º ( x, y )
1000
= 10 h approximately Projection of ordinate on normal = PN
y
10. Slope of the normal = y
x -1
y

P (x,y)
P (x,y)
a
N θ
O (1,0) π−θ
θ x
O T M
294 Textbook of Integral Calculus

\ PN = PN cos q = a (given) ln y + ln x = ln C
\
y
=a xy = C
1 + tan 2 q At point (2, 3 ),
2 ´3 =C ÞC =6
Þ y = a 1 + (y1 ) 2
\ xy = 6
dy y 2 - a2 18. ò
dy
100 - y ò
Þ = = dx
dx a
a dy - ln (100 - y ) = x + C
Þ ò y 2 - a2
= ò dx
ln (100 - y ) = - x + C
Also x = 0, y = 50
Þ a ln | y + y 2 - a 2 | = x + C
\ C = ln 50
13. y = e x (ax + b ) …(i) x = ln 50 - ln (100 - y )
Differentiating w.r.t. x, we get 50
Þ ln =x
dy dy 100 - y
= e x (ax + b ) + e x × a or = y + ae x …(ii)
dx dx 50
Þ = ex
Again, differentiating both the sides 100 - y
d 2y dy Þ 100 - y = 50e - x
= + ae x …(iii)
dx 2 dx Þ y = 100 - 50e - x
From Eq. (iii) – Eq. (ii) 2
d 2y 2dy æ dy ö 2 æ dy ö
- +y =0
19. Here, x ç ÷ - 3y ç ÷+3=0
è dx ø è dx ø
dx 2 dx
is required differential equation. has order 1, degree 2 and non-linear.
dy y dy 2y 20. Here, ( f ¢( x )) 2 + 4 f ¢( x ) × f ( x ) + ( f ( x )) 2 = 0
14. = 2ax = 2 x × 2 ; = ; 2
dx x dx x æ f ¢( x ) ö æ f ¢( x ) ö
dy x dy x Þ ç ÷ +4ç ÷+1=0
Now, m = -1 Þ m = - Þ =- è f (x ) ø è f (x ) ø
dx 2y dx 2y
f ¢( x ) -4 ± 16 - 4
x2 Þ =
y2 = - +C f (x ) 2
2
1 f ¢( x )
15. ò0 f (tx ) dt = n × f (x ) Þ
f (x )
= -2 ± 3
1
Put, x = y Þ dt = dy Integrating both the sides
x
1 x log | f ( x )| = ( -2 ± 3 ) x + C1
\
x ò0 f (y ) dy = nf (x ) Þ f ( x ) = e ( -2 ± 3 )x
×C
x
\ ò0 f (y ) dy = x × n × f ( x )
21. (a) 32x + 2y
dy
=0
Differentiating, f ( x ) = n [ f ( x ) + xf ¢( x )] dx
f ( x ) (1 - n ) = nx f ¢( x ) dy 16 x
Þ = m1 = -
f ¢( x ) 1 - n dx y
\ =
f (x ) nx dy
and 16y 15 =k
1 -n dx
æ1 - nö
Integrating, ln f ( x ) = ç ÷ ln Cx = ln (Cx ) n ; dy k
è n ø Þ = m2 =
1 -n
dx 16y 15
\ f ( x ) = Cx n 16 x k x
m1m2 = - × = - 16 × k
16. ( x z 2 2a
- 1 ) az a -1
dz + 2 x z 3a
dx = 0 y 16y 15 y

or a( x z 2 3a - 1
-z a -1
) dz + 2 x z 3a dx = 0 x y 16
=- × = -1
y 16 x
for homogeneous every term must be of the same degree,
dy
3a + 1 = a - 1 Þ a = - 1 (b) =1 -ce - x = 1 - (y - x ) = - (y - x - 1 ) [using ce - x = y - x ]
dx
17. Differentiate, xy ( x ) = x 2y ¢( x ) + 2 xy ( x ) dy dy - y
and -k× e =1
or xy ( x ) + x 2 y ¢( x ) = 0 dx dx
dy dy
x +y =0 [1 - ke - y ] = 1
dx dx
Chap 04 Differential Equations 295

dy dv dx dt dx
or [1 - ( x + 2 - y )] =1 [using ke - y = x - y + 2] Þ ò v (log v - 1) = ò Þ òt -1 = ò ,
dx x x
dy 1 let log v = t
= m2 =
dx y - x -1 Þ log (t - 1 ) = log ( x ) + log C
Þ m1m2 = - 1 Þ log (t - 1 ) = log ( xC )
dy y 2y Þ t = 1 + xC
(c) = 2cx = 2 x × 2 = = m1
dx x x y
dy dy x Þ log = 1 + xC
Also, 2 x + 4y =0 Þ =- = m2 x
dx dx 2y
Þ y = x × e1 + Cx
Hence, m1m2 = - 1
or y = x × e1 - Cx
(d) x2 - y 2 = c
y
dy dy x and log = log e + Cx
2 x - 2y =0 Þ = = m1 x
dx dx y
Þ y = ex × eCx
xy = k
dy
dy dy y 24. Clearly, (a) and (c) are of the form + Py = Q, which is
x +y =0 Þ = - = m2 dx
dx dx x linear in y.
\ m1m2 = - 1 d 2y dy
Hence, (a), (b), (c) and (d) are all correct. Also, (d) is = cos x, on integrating = sin x + C
dx 2 dx
dy
22. Let m = be the slope of tangent ( x, y ) to the required curve. which is also linear in y.
dx
m1 = slope of the tangent at xy = c 2 2
dy (y - x ) ± ( x - y ) + 4 xy
25. =
c2 y dx 2y
=- =-
x2 x dy dy x
Þ = 1 or =-
c2 y dx dx y
m+ 2 m+
Hence, x = ± 1 or x =±1 …(i) Þ x - y + 1 = 0 and x 2 + y 2 = 25
c2 y
1- m
1 - 2m
x x 26. (a) f ( x, tx ) = et + tan -1(t ), independent of x.
Consider y 2 - 2 xy - x 2 = k Þ Homogeneous differential equation.
dy æ dy ö æy ö y2 y
Þ 2y - 2 çy + x ÷ - 2x = 0 (b) log ç ÷ dx + 2 sin -1 × dy = 0
dx è dx ø èxø x x
dy æy ö
Þ (y - x ) = x + y log ç ÷
dx dy èxø
dy x + y Þ = 2
Þ = =m (say) dx y æy ö
dx y - x sin -1 ç ÷
x2 èxø
From Eq. (i)
x+y y æy ö
log ç ÷
+ èxø
y -x x f ( x, y ) =
LHS = y2 æy ö
y æx + yö sin -1 ç ÷
1- ç ÷ x2 èxø
x èy - x ø
log (t )
x2 + y 2 \ f ( x, tx ) = , independent of x
= = 1 RHS t sin -1 (t )
2
x2 + y 2
Þ Homogeneous differential equation.
Similarly option, (b), (c) and (d) safisfy
(c) f ( x, y ) = x 2 + sin x × cosy
dy æ y ö dy y æy ö f ( x, tx ) = x 2 + sin x × cos (tx ), not independent of x.
23. x = y log ç ÷ = log ç ÷ , put y = vx
dx è x ø dx x èxø Þ Not homogeneous differential equation.
dy dv x2 + y 2
Þ =v + x (d) f ( x, y ) = 2
dx dx xy - y 3
dv dv
\ v+x = v log v Þ x = v (log v - 1 ) Þ f ( x, tx ) is not independent of x.
dx dx Þ Not homogeneous differential equation.
296 Textbook of Integral Calculus

27. Integrating Factor = e ò cos x dx


= e sin x
(d) (1 + y 2 )
dy -1
+ x = 2e tan y
or y × e sin x = ò e + sin x cos x = - e + sin x + C dx
-1
dx 1 e tan y
At point (0, –1), Þ + x =
dy 1 + y 2 1 + y2
- 1 e0 = -e0 + C Þ 0
1
\ ye sin x = - e sin x ò 1 + y2 -1
IF = e = e tan y

y = -1 1
-1 -1 e tan y
28.
dy sin x
- y cot x = - 2 Þ xe tan y
= 2 ò e tan y
× dy
dx x 1 + y2
-1 -1
1
Integrating Factor = e ò - cot x dx = e - log sin x = Þ xe tan y = e 2 tan y + k
sin x dy
30. = 3 ( A + Bx ) e 3x = Be 3x
1 sin x 1 dx
sin x ò x 2 sin x
\ Required solution is y × = - × dx + C
dy
Þ + my = (3 + m ) ( A + Bx ) e 3x + Be 3x
1 1 dx
y× = +C
sin x x d 2y dy
Þ +m + ny = (9 + 3m + n ) ( A + Bx ) e 3x
As x ® ¥, y ® 0 ÞC = 0 dx 2
dx
sin x + B (6 + m ) e 3x = 0
\ y = Þ lim f ( x ) = 1 …(i)
x x®0 Þ 3m + n + 9 = 0 and m + 6 = 0
p/ 2 sin x Þ m = - 6 and n = 9
\ I =ò dx
0 x 31. Equation of tangent
Since,
sin x
is decreasing, when x > 0 Y - y = m (X - x )
x Put X = 0, Y = y - mx
Þ f (x ) < f (0)
p/2 p p
Þ ò0 f ( x ) < and x <
2 2
æpö p/2
Þ f ( x ) > ò ç ÷ Þ ò f ( x ) dx > 1 P(x,y)
è2ø 0

29. (a) Order of the differential equation is 2.


xdy - ydx O
xdy - ydx x2 dx
(b) = dx Þ = Since, initial ordinate is
x2 + y 2 æ y ö
2 x
y - mx = x - 1
ç1 + 2 ÷
è x ø Þ mx - y = 1 - x
½y dy 1 1-x
y 2½ - y = which is a linear differential equation.
\ ln ½ + 1 + 2½ = ln | Cx | dx x x
½x x ½
Hence, Statement I is correct and its degree is 1.
y x2 + y 2 Þ Statement II is also correct. Since, every 1st degree
\ + = Cx differential equation need not be linear, hence Statement II is
x x
not the correct explanation of Statement I.
i.e. y + x 2 + y 2 = Cx 2
32. Statement I The order of differential equation is 2.
(c) y = e x ( A cos x + B sin x ) \ Statement I is false.
dy
= e x ( A cos x + B sin x ) + e x ( - A sin x + B cos x ) 33. Integral curves are y = cx - x 2
dx The differential equation does not represent all the parabolas
= y + e x ( - A sin x + B cos x ) passing through origin but it represents all parabolas through
d 2y dy origin with axis of symmetry parallel to y-axis and coefficient
\ = + e x ( - A sin x + B cos x ) of x 2 as -1, hence Statement I is false.
dx 2 dx
Statement II is universally true.
ydx - xdy x
+ e x ( - A cos x - B sin x ) 34. × cot = x dx
y2 y
d 2y dy
\ = + e x ( - A sin x + B cos x ) - y x æxö
dx 2 dx or ò cot y × d çè y ÷ø = ò x dx
dy dy æ dy ö
+ -y -y =2 ç -y÷
dx dx è dx ø
Chap 04 Differential Equations 297

41. f ( 0) = 2
or ò cot t dx = nx + C
é x x ù
x f ( x ) = (e x + e - x ) cos x - 2 x - ê x ò f ¢(t ) dt - ò { ¢(t ) dt ú
ln (sin t ) = nx + C ; sin = Cenx 0 0
t f{
y ëê I II úû
35. y = c1 cos2 x + c 2 sin 2 x + c 3 cos2 x + c 4e 2x + c 5 x
f ( x ) = (e x + e - x ) cos x - 2 x - [ x( f ( x ) - f ( 0 )) - {t × f (t ) 0 -ò f (t )dt }]
x
0
é 1 - cos 2 x ù é cos 2 x + 1 ù
e 2x + c 6 = c1 cos2 x + c 2 ê úû + c 3 êë úû ) cos x - 2 x - xf ( x ) + 2 x + é x f ( x ) - ò f (t ) dt ù
x -x x
ë 2 2 f ( x ) = (e + e
êë 0 úû
+ c 4e + c 5e 2x × ec 6
2x
x
æ c c ö æc c ö f ( x ) = (e x + e - x ) cos x - ò f (t ) dt …(i)
= çc1 - 2 + 3 ÷ cos2 x + ç 2 + 3 ÷ + (c 4 + c ¢5 ) e 2x 0
è 2 2ø è2 2ø
2x
On differentiating Eq. (i)
= l1 cos2 x + l 2e + l 3
f ¢( x ) + f ( x ) = cos x (e x - e - x ) - (e x + e - x ) sin x …(ii)
Þ Total number of independent parameters in the given
general solution is 3. dy
Hence, + y = e x (cos x - sin x ) - e x (cos x + sin x )
36. The given differential equation is dx
æ dy ö 42. f ¢( 0) + f ( 0) = 0 - 2 × 0 = 0
d ç ÷
è dx ø 2x 43. IF of Eq. (i) is e x
= 2 dx
dy x +1 y × e x = ò e 2x (cos x - sin x ) dx - ò ( cos x + sin x ) dx
dx

Þ
æ dy ö
ln ç ÷ = ln ( x 2 + 1 ) + ln C , C > 0 y × e x = ò e 2x (cos x - sin x ) dx - (sin x - cos x ) + C
è dx ø
dy Let I = ò e 2x (cos x - sin x ) dx = e 2x ( A cos x + B sin x )
Þ = C (x 2 + 1)
dx
æ x3 ö Solving, A = 3 / 5 and B = - 1 / 5 and C = 2 / 5
Þ y =C ç + x ÷ + C ¢, C ¢ Î R
è 3 ø æ3 1 ö 2
\ y = e ç cos x - sin x ÷ - (sin x - cos x ) e - x + e -x
x
è5 5 ø 5
Obviously y ® ¥, as x ® ¥; as C > 0
37. ( x - h ) 2 = 4b (y - k ) here b is constant and h, k are parameters. Solutions (Q. Nos. 44 to 46)
d 2y
38. The general equation of all non-vertical lines in a plane is Integrating, = 6 x - 4,
ax + by = 1, where b ¹ 0. dx 2
ax + by = 1 dy
Now, we get = 3x 2 - 4x + A
dy dx
Þ a+b =0 (differentiating w.r.t. x) dy
dx When x = 1, = 0 so that A = 1.
dx
d 2y
Þ b 2 =0 (differentiating w.r.t. x) dy
dx Hence, = 3x 2 - 4x + 1 …(i)
dx
d 2y
Þ =0 (as b ¹ 0) Integrating, we get y = x 3 - 2 x 2 + x + B.
dx 2
When x = 1, y = 5, so that B = 5
d 2y
Hence, the differential equation is 2 = 0. Thus, we have y = x 3 - 2 x 2 + x + 5
dx
2 2 1
39. The equation ax + 2hxy + by = 1 represents the family of all From Eq. (i), we get the critical points x = , x = 1
3
conics whose centre lies at the origin for different values of a,
2
h, b. 1 d y
At the critical point x = , 2 is negative.
\ Order is 3. 3 dx
Thus, Statement I is false and Statement II is true. 1
Therefore, at x = , y has a local maximum.
Hence, option (d) is the correct answer. 3
40. y = a sin x + b cos x d 2y
At x = 1, 2 is positive. Therefore, at x = 1, y has a local
dy dx
= a cos x - b sin x minimum.
dx
d 2y æ 1 ö 157
Þ = - a cos x - b sin x = - y Also, f (1 ) = 5 , f ç ÷ = , f ( 0 ) = 5, f (2 ) = 7
dx 2 è 3 ø 27
d 2y Hence, the global maximum value = 7.
Þ +y =0
dx 2 and the global minimum value = 5.
But Statement II is not the correct explanation of the Statement I.
298 Textbook of Integral Calculus

47. xdy - ydx = x 2 - y 2 dx 54. C 3 :( x - 5) 2 + y 2 = 5 2


1 - (y / x ) 2 Tangent, bx + ay - ab = 0, length of perpendicular to tangent
æy ö d (y / x ) dx
Þ d ç ÷=
èxø x
dx Þ ò 1 - (y /x ) 2

x
from centre = radius.
Þ 5b - ab = 5 a 2 + b 2
-1 y -1
Þ sin = ln x = ln C or Cx = e sin y /x Þ a 2b 2 - 10ab 2 = 25a 2
x
Þ ab 2 - 10b 2 = 25a
48. ( xy 4 + y ) dx - x dy = 0
55. C1 : x 2 + y 2 - 2x = 0
Þ xy 4dx + y dx - x dy = 0
2 C 2 : x 2 + y 2 - Cx = 0
æxö æxö
ò x dx - ò çè y ÷ø d çè y ÷ø = 0
3
Þ
P
3 Q
x4 1 æ x ö
Þ + ç ÷ =C
4 3 èy ø
R O (1, 0)
M
49. 2x cos y dx - x 2 sin y dy + y 2 cos x dx + 2y sin x dy = 0

ò d (x cosy ) + ò d (y 2 sin x ) = 0
2
Þ

Þ x 2 cosy + y 2 sin x = C ÐQRM = 30°


dx ydy 1
50. = sin30° = Þ OR = 1
x 1 + y2 1 + OR
1 QM
Þ ln x = × ln (1 + y 2 ) + C sin 30° = Þ QM = 1 / 3
2 1 - QM
From the given condition, C = 0 2
radius of C 2 = 1 / 3 Þ C = -2 / 3,C 2: x 2 + y 2 + x = 0
\ x2 - y 2 = 1 3
dy 1 + y2 Point ( -1 / 3, 1 / 3 ) will satisfy C 2.
51. + =0
dx 1 - x2 56. (A) Let x = r cos q, y = r sin q
dy dx \ x 2 + y 2 = r 2 (sin 2 q + cos2 q) = r 2 …(i)
Þ + =0
1 + y2 1-x 2
y
and tan q = …(ii)
Þ tan -1 y + sin -1 x = C x
(1 + x ) 2 \ d ( x 2 + y 2 ) = d (r 2 )
dy
52. = (1 + x ) × (1 + y ) gives y = e 2 - 1
dx From Eq. (i), x dx + y dy = rdr …(iii)
53. Tangent at point P ( x,y ), is y - y = f ¢( x )( x - x ) From Eq. (ii),
æy ö
Q :( 0, y - x f ¢( x )) d ç ÷ = d (tan q)
èxø
Then, PQ = OQ
xdy - ydx
Þ x 2 + x 2( f ¢( x )) 2 = y 2 + x 2( f ¢( x )) 2 - 2 xy ( f ¢( x )) Þ = sec 2 q dq
x2
Þ x 2 = y 2 - 2 xy × f ¢( x )
\ x dy - y dx = x 2 sec 2 q dq
y 2 - x2
or dy / dx = , put y = tx = r 2 cos2 q sec 2 q dq = r 2dq …(iv)
2 xy
From Eqs. (iii) and (iv)
dt dy
Þ t+x =
dx dx rdr a2 - r 2
=
dt t 2 - 1 r 2dq r2
\ t+x =
dx 2t dr
or = dq
dt - (1 + t 2 ) 2t dx a - r2
2
Þ x = Þ dt = -
dx 2t 1 + t2 x
ær ö
c y2 sin -1 ç ÷ = q + C
Þ = 1 + 2 Þ x 2 + y 2 - cx = 0 èa ø
x x
\ C1 : x 2 + y 2 - x = 0 Þ r = a sin ( q + C )

C2 : x 2 + y 2 + x = 0 ì æ y öü
Þ x 2 + y 2 = a sin íC + tan -1 ç ÷ý
î è x øþ
C1 and C 2 touch externally Þ number of common tangents = 3.
Chap 04 Differential Equations 299

(B)
dy
- sec 2 x tan 2 xy = cos2 x x 2dy ( x )
or (1 - 3 x ) y ( x ) =
dx dx
2 tan x
ò ´ sec 2 x dx (1 - 3 x ) dx dy ( x ) C
tan 2x sec 2 x dx = or y = 3 e -1/x
IF = e ò tan 2 x - 1 or
=e x2 y (x ) x
dt
ò 57. (A) r = 5 cm, dr = 0.06, A = pr 2
=e t , where t = tan 2 x - 1
dA = 2 pr dr = 10p ´ 0.06 = 0.6 p
= e ln | t | = | t | = | tan 2 x - 1 |
(B) v = x 3
p
Given that, | x | <
4 dv = 3 x 2 dx
\ tan 2 x < 1 dv dx
´ 100 = 3 ´ 100 = 3 ´ 1 = 3
\ IF = 1 - tan 2 x v x
dx dx
Solution is y (1 - tan 2 x ) = ò cos2x (1 - tan 2 x ) dx (C) ( x - 2 ) =2 Þ x=4
dt dt
= ò ( cos2 x - sin 2 x ) dx 3 2
(D) A= x
sin 2 x 4
= ò cos 2 x dx = +C dA 3 dx 3 1 3 3
2 = x = × 15 × =
p 3 3 dt 2 dt 2 10 4
when x = , y = dy
6 8 58. (A) = e x ( A cos x + B sin x ) + e x ( - A sin x + B cos x )
dx
3 3 æ 1ö 1 3
\ ç1 - ÷ = ´ +C
8 è 3ø 2 2 = y + e x ( - A sin x + B cos x )
sin 2 x d 2y dy
\ C =0 Þ y = \ = + e x ( - A sin x + B cos x ) + e x ( - A cos x - B sin x )
2(1 - tan 2 x ) dx 2 dx
(C) x 2 + y 2 - 2cx = 0 …(i) dy æ dy ö æ dy ö
= +ç -y÷ -y =2 ç -y÷
\ 2 x + 2y
dy
- 2c = 0 dx è dx ø è dx ø
dx
So, degree = 1 and order = 2
ydy
or C =x+ …(ii) (B) The equation of the family is y 2 = 4a ( x - b )
dx
where a, b are arbitrary constants.
æ ydy ö
From Eqs. (i) and (ii), x 2 + y 2 - 2 ç x + ÷x=0
è dx ø dy æ dy ö d 2y
2
\ 2y = 4a or ç ÷ + y 2 = 0
dy dx è dx ø dx
or - x 2 + y 2 - 2 xy = 0 is the differential equation
dx
So, degree = 1 and order = 2
representing the given family of circles. To find the orthogonal
trajectories. (C) The equation of the family is
dx (x - c )2 + y 2 = c 2
y 2 - x 2 = - 2 xy
dy
x2 + y 2
or y 2 dy = x 2 dy - 2 xy dx or x 2 + y 2 - 2cx = 0 or = 2c
x
yd ( x 2 ) - x 2 dy
or - dy = æ dy ö 2 2
y2 ç2 x + 2y ÷ x - (x + y ) × 1
è dx ø
æ x2 ö Þ =0
or - dy = d ç ÷ x2
èy ø So, degree = 1 and order = 1
x2 x2 y2
or -y =
+C (D) The equation of the family is 2 + 2 = 1 because
y a +l b +l
Þ x 2 + y 2 + Cy = 0
they have the same foci ( ± a 2 - b 2 , 0 ).
Þ Orthogonal trajectory.
2x 2y dy
(D) Differentiating the equation w.r.t. x, we get On differentiating, 2
+ 2 × =0
x x d + l b + l dx
xy ( x ) + 1 ò y (t ) dt = ( x + 1 ) xy ( x ) + 1 ò ty (t ) dt
0 0 x yp æ dy ö
or + =0 ç Let p = ÷
Again, differentiating, w.r.t. x, we get a2 + l b2 + l è dx ø
y ( x ) = x 2y ¢( x ) + 2 xy ( x ) + xy ( x ) or x(b 2 + l ) + yp(a 2 + l ) = 0
300 Textbook of Integral Calculus

-b 2x - a 2yp æ 5ö
Þ l= \ The required time (t 2 - t1 ) = 25 ç ln 5 - ln ÷ = 25 ln 3
x + yp è 3ø
\The differential equation is = 25 ´ 1.0986 = 26 min 28 s
= 1648 s = 206 ´ 8 = 206 ´ l
x2 y2
2 2
+ =1 \ l =8
2 - b x - a yp 2 - b 2x - a 2yp
a + b +
x + yp x + yp 61. f ( x + f (y ) + xf (y )) = y + f ( x ) + yf ( x ) …(i)
x( x + yp ) y ( x + yp ) Differentiating w.r.t. x and y is constant
or + 2 =1
a2 - b2 (b - a 2 ) p f ¢( x + f (y ) + xf (y )) (1 + f (y )) = f ¢( x ) + yf ¢( x ) …(ii)
From Eq. (i) again differentiating w.r.t. y as x is constant
So, order = 1 and degree = 2
f ¢( x + f (y ) + xf (y )) (1 + x ) f ¢(y ) = 1 + f ( x ) …(iii)
2 4 3 3
59. 2x y dx - 2y dx + 2x dy + 3xy dy = 0 From Eqs. (ii) and (iii)
Dividing throughout by x 3y , we get 1 + f (y ) (1 + y ) f ¢( x )
=
dx 2y 3 dy 3y 2 (1 + x ) f ¢(y ) 1 + f (x )
2 - 3 dx + 2 × + 2 dy = 0
x x y x (1 + y ) f ¢(y ) 1 + f (x )
= =l
dx dy 3y 2 x 4 dy - 2y 3x 3dx 1 + f (y ) (1 + x ) f ¢( x )
Þ 2 +2 + =0 1 + f (y ) 1 + f (x )
x y x6 \ f ¢(y ) = l and f ¢( x ) =
1+y l(1 + x )
dx dy 3y 2x 2 dy - 2y 3x dx
Þ 2 +2 + =0 1 f ¢( x ) 1
x y x4 \ l= Þ l=±1 \ =±
l 1 + f (x ) 1+x
æy 3 ö
Þ 2d (ln x ) + 2d (ln y ) + d ç 2 ÷ = 0 Integrating both the sides f ( x ) = C (1 + x ) ±1 - 1
èx ø
From Eq. (i) put x = y = 0
y3
Þ 2 ln | x| + 2 ln | y | + =C f ( f ( 0 )) = f ( 0 )
x2
From Eq. (ii), f ( 0 ) = C - 1
when x = 1, y = 1. So, C = 1
So, f (C - 1 ) = C - 1
60. Let the salt content at time ‘t’ be u lb, so that its rate of change
is du / dt. \ C = 0, 1 (taking +ve sign)
= 2 gal ´ 2 lb = 4 lb / min So, f ( x ) = - 1 and f ( x ) = (1 + x ) - 1 = x and C = 1
-x
If c be the concentration of the brine at time t, the rate at which \ f ( x ) = (1 + x ) -1 - 1 =
the salt content decreases due to the out flow 1+x
= 2 gal ´ c lb / min 1 + f (x ) =
1
= 2c lb / min 1+x
du 1
\ = 4 - 2c …(i) \ 1 + f (2009 ) = \ (2010 ) (1 + f (2009 )) = 1
dt 2010
Also, since there is no increase in the volume of the liquid, the 62. f¢( x ) + 2 f( x ) £ 1
u
concentrations c= e 2x f¢( x ) + 2 f ( x ) e 2x £ e 2x
50
du 2u d æ 2x 1 2x ö
\ Eq. (i) becomes =4- çe f( x ) - e ÷ £ 0
dt 50 dx è 2 ø
Separating the variables and integrating, we have æ 1ö
\ çe 2x f ( x ) - ÷ is a non-increasing function of x.
du è 2ø
ò dt = 25 ò 100 - u 1
Þ f (x ) - is a non-increasing function of x.
or t = - 25 ln (100 - u ) + K …(ii) 2
Initially when t = 0, u = 0 \ 2f ( x ) is always less than or equal to 1.
0 = - 25 ln 100 + K …(iii) 1 a dy
63. (1 + x ) -1/2 - (1 + y ) -1/2 =0
Eliminating ‘K ’ from Eqs. (ii) and (iii), we get 2 2 dx
æ 100 ö 1 a dy 1+y 1
t = 25 ln ç ÷ Þ = × Þ a= ×
è 100 - u ø 1+x 1 + y dx 1 + x dy / dx
Taking t = t1 when u = 40 and t = t 2 when u = 80, we have Putting this value in the given equation,
æ 100 ö æ 100 ö dy 1+y dy
t1 = 25 ln ç ÷ and t 2 = 25 ln ç ÷ 1+x - =
è 60 ø è 20 ø dx 1 + x dx
Chap 04 Differential Equations 301

Þ
dy
(1 + x )
=1 + y + 1 + x
dy Now, when t = 0, h = H and when t = t, h = 0
dx dx t - p r 2 0 - 1/ 2
Þ (1 + x - 1 + x )
dy
=1 + y
Thus, ò0 dt = ma 2g òH h dh
dx
- p r2
\ Degree of the given equation is 1. Þ t= × 2 ( h ) H0
ma 2 g
64. Here, 2 f 5( x ) × f ¢( x ) × 1 + ( f ¢( x )) 2 = f ¢¢( x ) p r2 2H
Þ t= ×
f ¢¢( x ) ma g
Þ f 4 ( x )d ( f 2( x )) =
1 + ( f ¢( x )) 2 67. Let at time t the depth of water is h; the radius of water surface
Þ f 4 ( x )d ( f 2( x )) = ò d (tan -1( f ¢( x ))) is r.
Then, r 2 = R 2 - (R - h )2
f 6( x )
Þ + C = tan -1( f ¢( x )), as f ( 0 ) = f ¢( 0 ) = 0 Þ c = 0 Þ r 2 = 2 Rh - h 2
3
f 6( x ) R
Þ = tan -1( f ¢( x ))
3
R–h
p f 6( x ) p
Þ - < < dh
2 3 2
1/ 6 1/ 6
æ 3p ö æ 3p ö
\ -ç ÷ < f (x ) < ç ÷
è 2 ø è 2 ø
Þ Number of integers between k1 and k2 are 3. Now, if in time dt the decrease in water level
is dh, then
65. Put, dy / dx = t
-x - p r 2 dh = 0.6 2 gh × a dt
\ dt / dx - t + e 2x = 0, I.F. = e ò -dx =e
(a is cross-sectional area of the outlet)
\ Solution is, t × e -x = ò -e 2x × e - x dx + C -p dh
Þ (2 Rh - h 2 ) = dt
Þ te - x = -e x + C y ¢( 0 ) = 1 Þ C = 2 ( 0.6 ) a 2 g h
2 -ex p 0 t
\ dy / dx =
e -x ò
dy = ò (2e x - e 2x )dx Þ
( 0.6 ) a 2 g òR (h 3/2 - 2 R h1/2 ) dh = ò dt
0

e 2x p é 2 5/ 2 4 ù
0
Þ y = 2ex - + C 1, y ( 0 ) = 2 Þ C 1 = 1 / 2 Þ h - Rh 3/2 ú = t
2 ( 0.6 ) a 2 g êë 5 3 ûR
e 2x 1 5 p 5
\ y ( x ) = 2e x - + Þ y ( x ) £ for x = log2 é 5/ 2 æ 2 4 ö ù 7 p ´ 10
2 2 2 Þ t= ê0 - R ç - ÷ú =
( 0.6 ) a 2 g ë è 5 3 ø û 135 g
\ [2 x ] = [2 log 2 ] = [log 4 ] = 1
68. We have been given,
66. Let in time dt the decrease in water level in the tank is dh, then
amount of water flown out in time dt = pr 2 × dh f ( xy ) = e xy - x - y {e y f ( x ) + e x f (y )}, " x, y Î R + …(i)
Now, through the hole the amount of water flown Replacing x = 1, y = 1 in Eq. (i), we get
= (Volume of cylinder of cross sectional area ‘a’ f (1 ) = e1 - 1 - 1 {e f (1 ) + e f (1 )} Þ f (1 ) = 0
and length vdt ). f (x + h ) - f (x )
Now, f ¢ ( x ) = lim
r h®0 h
æ æ h öö
f ç x ç1 + ÷ ÷ - f (x )
dh è è x øø
= lim
h®0 h
H h
x + h - x -1 - x
= lim e
x (e1 + h / x × f ( x ) + e × f (1 + h/x )) - f ( x )
h®0 h
h
h -1 - + x æ hö
O eh f ( x ) + e x × f ç1 + ÷ - f (x )
è xø
= lim
h®0 h
= a × v × dt = a m 2 gh dt
h
h -1 - + x æ æ hö ö
Hence, ma 2 gh dt = - p r 2dh f ( x ) (eh - 1 ) + e x × ç f ç1 + ÷ - f (1 ) ÷
è è xø ø
- p r2 = lim
Þ dt = × h - 1/2 dh h®0 h
ma 2 g
302 Textbook of Integral Calculus

h -1 -
h
æ æ
+ x hö ö dz æ dy ö
Þ y1 × + z ç 1 + py1 ÷ = Q
e x
ç f çè1 + ÷ø - f (1 ) ÷ dx è dx ø
h
f ( x ) (e - 1 ) è x ø
= lim + lim dz
h®0 h h ® 0 h Þ y1 × + z Q =Q
×x
x dx
x -1
e × f ¢ (1 ) ex Þ
dz
= Q (1 - z )
= f (x ) + = f (x ) + y1
x x dx
ex dz Q
Þ f ¢ (x ) - f (x ) = Þ = dx
x 1 -z y1
e x f ¢ (x ) - f (x ) e x 1 Q
Þ = Þ ln | z - 1 | = - ò dx + l, ‘ l’ being constant of integration
e 2x x y1
Q
d æ f (x ) ö 1 ò - y1 dx
Þ ç ÷= Þ z =1 + c e
dx è e x ø x
1 1
Integrating both the sides, we get 71. We have, f ( x ) = x + x 2 ò0 z f (z ) dz + x ò0 z 2 f (z ) dz
f (x )
= ln | x | + C f ( x ) = x + x 2 l1 + x l 2 (say)
ex 1 1
Since, f (1 ) = 0 Þ C = 0 Now, l1 = ò z f (z ) dz = ò ((1 + l 2 ) z + z 2 l1 ) z dz
0 0
Thus, f ( x ) = e x × ln ( x ) 1 + l 2 l1
= +
69. Let P ( x, f ( x )) be a point lying on the curve in first quadrant. 3 4
Equation of normal and tangent at P are Þ 9 l1 - 4 l 2 = 4 …(i)
1 1
(Y - f ( x )) = - ( X - x ) and Also, l 2 = ò z f (z ) dz 2

f ¢ (x ) 0
1
(Y - f ( x )) = f ¢ ( x ) ( X - x ) respectively. =ò ((1 + l 2 ) z 3 + z 4 l1 ) dz
0
æ f (x ) ö (1 + l 2 ) l1
Þ Aºç x- , 0 ÷ , B º ( x + f ( x ) f ¢ ( x ), 0 ) = +
è f ¢ (x ) ø 4 5
Since, the mid-point of segment AB is origin. Þ 15 l 2 - 4 l1 = 5 …(ii)
f (x ) From Eqs. (i) and (ii),
Þ 2x - + f (x ) × f ¢ (x ) = 0
f ¢ (x ) 80 61
l1 = and l 2 =
Þ f ( x ) ( f ¢ ( x )) 2 + 2 x f ¢ ( x ) - f ( x ) = 0 119 119
80 2 61 20 x
Þ f (x ) = x + x + x= (4 + 9x )
- 2 x ± 4 x 2 + 4 f 2( x ) - x + x 2 + f 2( x ) 119 119 119
Þ f ¢( x ) = =
2 f (x ) f (x ) 72. f ( x + f (y ) + x f (y )) = y + f ( x ) + y f ( x )
Negative sign been neglected as f ¢ ( x ) > 0 Keep y constant and differentiating the expression w.r.t. ‘x’, we
Thus, we have, x + f ( x ) × f ¢ ( x ) = x 2 + f 2 ( x ) get
f ¢ ( x + f (y ) + x (y )) (1 + f (y )) = f ¢ ( x ) (1 + y ) …(i)
d
Þ ( x 2 + f 2 ( x )) = x 2 + f 2
(x ) Similarly, differentiate the expression w.r.t. ‘y’ and keep x
2dx
constant, we get
d ( x 2 + f 2 ( x ))
Þ = dx f ¢ ( x + f (y ) + x f (y )) ( f ¢ (y ) (1 + x )) = (1 + f ( x )) …(ii)
2 x 2 + f 2 (x ) Dividing Eq. (i) by Eq. (ii), we get
Integrating both the sides, we get x 2 + f 2 ( x ) = x + l 1 + f (y ) f ¢ ( x ) (1 + y )
=
f ¢ (y ) (1 + x ) (1 + f ( x ))
It passes through (1, 3 ).
1 + f (y ) f ¢ ( x ) (1 + x )
Hence, l + 1 = 2 Þ l = 1 Þ = =C
f ¢ (y ) (1 + y ) (1 + f ( x ))
\ Curve is x 2 + f 2 ( x ) = ( x + 1 ) 2 or y 2 = 1 + 2 x
1 é 1 + f (y ) ù 1 + f (x )
70. We have been given, Þ f ¢ (y ) = ê ú and f ¢ ( x ) = C
C ë 1+y û (1 + x )
dy1 dy
+ Py1 = Q, 2 + Py 2 = Q 1
dx dx Þ C= ÞC = ± 1
dy dz dy C
Now, y 2 = y1 z Þ 2 = y1 +z 1
dx dx dx f ¢ (x ) 1
\ =±
dz dy1 1 + f (x ) 1+x
Þ y1 +z + Py1 z = Q
dx dx
Þ f ( x ) = l1 (1 + x ) ± 1 - 1
Chap 04 Differential Equations 303

Replacing x, y ® 0 in the Eq. (i), we get Thus, (a) is the correct answer.
f ( f ( 0 )) = f ( 0 ) To check option (c), we have y = (x + 2)2
Now, f ( x ) = l1 (1 + x ) ± 1 - 1 Þ f ( 0 ) = l1 - 1 æ |y | ö y
and log ç ÷ + =0
and f ( f ( 0 )) = f ( l1 - 1 ) = l1 l1± 1 - 1 è 3e ø x + 2

Since, f ( f ( 0 )) = f ( 0 ), l1 l1± 1 - 1 = l1 - 1 é | x + 2| 2 ù (x + 2 )2
Þ log ê ú+ =0
Þ l11 ± 1 = l1 ë 3e û x+2

By taking +ve sign, we get l1 = 0, 1 é | x + 2|2 ù


Þ log ê ú = - (x + 2)
Þ f ( x ) = - 1 or f ( x ) = x ë 3e û
By taking - ve sign, we get l1 = 1 (x + 2)2
1 x Þ = e - ( x + 2) or ( x + 2 ) 2 × e x + 2 = 3e
Þ f (x ) = -1 = - 3e
1+x 1+x 3e
Þ ex+ 2 =
dy (x + 2)2
73. Given, ( x 2 + xy + 4x + 2y + 4) -y 2 = 0
dx Y
dy e x +2
Þ [( x 2 + 4 x + 4 ) + y ( x + 2 )] -y 2 = 0
dx
dy e2
Þ [( x + 2 ) 2 + y ( x + 2 )] - y 2 = 0
dx 3e /4
3e /( x + 2)2
Put x + 2 = X and y = Y , then
X
dY O
( X 2 + XY ) -Y 2 = 0
dX Clearly, they have no solution.
Þ X 2dY + XYdY - Y 2dX = 0 To check option (d), y = ( x + 3 ) 2
Þ X 2dY + Y ( XdY - YdX ) = 0 é | x + 3| 2 ù ( x + 3 ) 2
dY XdY - YdX i.e. log ê ú+ =0
Þ - = ë 3e û ( x + 2 )
Y X2
To check the number of solutions.
æY ö
Þ - d (log | Y | ) = d ç ÷ (x + 3)2
èX ø Let g ( x ) = 2 log ( x + 3 ) + - log (3e )
(x + 2)
On integrating both sides, we get
Y 2 æ (x + 2) × 2 (x + 3) - (x + 3)2 × 1 ö
- log | Y | = + C, where x + 2 = X and y = Y \ g ¢( x ) = +ç ÷-0
X x+ 3 è (x + 2)2 ø
y 2 ( x + 3 )( x + 1 )
Þ - log | y | = +C …(i) = +
x+ 2 x +3 (x + 2)2
Since, it passes through the point (1, 3). Clearly, when x > 0, then, g ¢( x ) > 0
\ - log 3 = 1 + C \ g( x ) is increasing, when x > 0.
Þ C = - 1 - log 3 = - (log e + log 3 ) Thus, when x > 0, then g( x ) > g( 0 )
= - log 3e æ3ö 9
g( x ) > log ç ÷ + > 0
\ Eq. (i) becomes èe ø 4
y Hence, there is no solution.
log | y | + - log (3e ) = 0
x +2 Thus, option (d) is true.
æ |y | ö y f (x )
Þ log ç ÷ + =0 …(ii) 74. Here, f ¢( x ) = 2 -
è 3e ø x + 2 x
dy y
Now, to check option (a), y = x + 2 intersects the curve. or + =2 [i.e. linear differential equation in y]
dx x
æ | x + 2| ö x + 2
Þ log ç ÷+ =0 1
ò x dx
è 3e ø x + 2 Integrating Factor, IF = e = e log x = x
æ | x + 2| ö \ Required solution is y × (IF) = ò Q (IF)dx + C
Þ log ç ÷ = -1
è 3e ø
| x + 2| 1 Þ y ( x ) = ò 2( x ) dx + C
Þ = e -1 =
3e e Þ yx = x 2 + C
Þ | x + 2 | = 3 or x + 2 = ± 3 C
\ y =x+ [QC ¹ 0, as f (1 ) ¹ 1]
\ x = 1, - 5 (rejected), as x > 0 [given] x
\ x = 1 only one solution.
304 Textbook of Integral Calculus

æ1ö 76. Since, centre lies on y = x.


(a) lim f ¢ ç ÷ = lim (1 - Cx 2 ) = 1
x®0 + è x ø x ® 0+ \ Equation of circle is x 2 + y 2 - 2ax - 2ay + c = 0
\ Option (a) is correct. On differentiating, we get
æ1ö 2 x + 2yy ¢ - 2a - 2ay ¢ = 0
(b) lim x f ç ÷ = lim (1 + Cx 2 ) = 1
x®0 + è x ø x ® 0+ Þ x + yy ¢ - a - ay ¢ = 0
\ Option (b) is incorrect. x + yy ¢
Þ a=
(c) lim x 2 f ¢( x ) = lim ( x 2 - C ) = - C ¹ 0 1 + y¢
x ® 0+ x ® 0+
Again differentiating, we get
\ Option (c) is incorrect. (1 + y ¢ )[1 + yy ¢¢+ (y ¢ ) 2 ] - ( x + yy ¢ ) × (y ¢¢ )
C 0=
(d) f ( x ) = x + , C ¹ 0 (1 + y ¢ ) 2
x
Þ (1 + y ¢ ) [1 + (y ¢ ) 2 + yy ¢¢ ] - ( x + yy ¢ ) (y ¢¢) = 0
For C > 0, lim f ( x ) = ¥
+
x®0 Þ 1 + y ¢ [(y ¢ ) 2 + y ¢ + 1 ] + y ¢¢(y - x ) = 0
\ Function is not bounded in (0, 2). On comparing with Py ¢¢ + Qy ¢ + 1 = 0, we get
\ Option (d) is incorrect. P =y - x
75. Here, (1 + e x )y ¢ + y e x = 1 and Q = (y ¢ ) 2 + y ¢ + 1
dy dy dy
Þ + ex × + ye x = 1 77. (i) Solution of the differential equation + Py = Q is
dx dx dx
Þ dy + e xdy + ye x dx = dx
y × (IF) = ò Q × (IF) dx + c
Þ dy + d (e xy ) = dx
IF = e ò
P dx
On integrating both sides, we get where,
a a
y + e xy = x + C (ii) ò-a f ( x ) dx = 2 ò f ( x ) dx, if f ( - x ) = f ( x )
0
Given, y (0) = 2
Given differential equation
Þ 2 + e0 ×2 = 0 + C dy x x 4 + 2x
+ 2 y =
Þ C =4 dx x - 1 1 - x2
\ y (1 + e x ) = x + 4
This is a linear differential equation.
x+4 x
Þ y = ò dx 1
ln | x 2 - 1 |
1 + ex x2 - 1
IF = e = e2 = 1 - x2
-4 + 4
Now at x = - 4, y = =0 Þ Solution is
1 + e -4
x( x 3 + 2 )
\ y( -4 ) = 0 …(i) y 1 - x2 = ò × 1 - x 2 dx
1 - x2
dy
For critical points, =0
dx x5
or y 1 - x 2 = ò ( x 4 + 2 x ) dx = + x2 + c
dy (1 + e x ) × 1 - ( x + 4 )e x 5
i.e. = =0
dx (1 + e x ) 2 f (0) = 0 ⇒ c = 0
x
Þ e (x + 3) - 1 = 0 x5
-x ⇒ f (x ) 1 - x 2 = + x2
or e = (x + 3) 5
3 /2 3 /2 x2
Y Now, ò- 3 /2
f ( x ) dx = ò
- 3 /2
1 - x2
dx [using property]
y = e–x y=x+3
(–1, e) 3 /2 x2
=2 ò dx
(–1, 2)
0
1 - x2
X′ X p / 3 sin 2 q
O
–1 =2 ò cos q dq [taking x = sin q]
0 cos q
p /3 p /3
=2 ò sin 2 q dq = ò (1 - cos 2 q) dq
0 0
Y′ p/3
æ sin2 q ö p sin 2 p / 3 p 3
= ç q- ÷ = - = -
Clearly, the intersection point lies between ( - 1, 0 ). è 2 ø0 3 2 3 4
\ y ( x ) has a critical point in the interval ( - 1, 0 ).
Chap 04 Differential Equations 305

78. Whenever we have linear differential equation containing æ1 ö


and f¢ ( x ) > 0, x Î ç , 1 ÷
inequality, we should always check for increasing or è4 ø
decreasing,
æ 1ö
dy
+ Py < 0 Þ
dy
+ Py > 0 Þe - x f ¢( x ) - e - x f ( x ) < 0, x Î ç 0, ÷
i.e. for è 4ø
dx dx
1
Multiply by integrating factor, i.e. e ò Þ f ¢( x ) < f ( x ) ,0 < x <
Pdx
and convert into total
differential equation. 4
Here, f ¢( x ) < 2 f ( x ), multiplying by e - ò 2dx 81. Here, f ¢¢( x ) - 2 f ¢( x ) + f ( x ) ³ e x
d Þ f ¢¢( x )e - x - f ¢( x )e - x - f ¢( x )e - x + f ( x )e - x ³ 0
f ¢( x ) × e -2x - 2e -2x f ( x ) < 0 Þ ( f ( x ) × e -2x ) < 0
dx d d
Þ { f ¢( x )e - x } - { f ( x )e - x } ³ 1
é1 ù dx dx
\ f( x ) = f ( x )e -2x is decreasing for x Îê , 1 ú
ë2 û d
Þ { f ¢( x )e - x - f ( x )e - x } ³ 1
1 dx
Thus, when x >
2 d 2 -x
Þ {e f ( x )} ³ 1, "x Î [ 0, 1 ]
æ1ö æ1ö dx 2
f( x ) < f ç ÷ Þ e -2x f ( x ) < e -1 × f ç ÷
è2ø è2ø \ f ( x ) = e - x f ( x ) is concave function.
æ1ö f ( 0 ) = f (1 ) = 0
Þ f ( x ) < e 2x -1 × 1, given f ç ÷ = 1
è2ø Þ f( 0 ) = 0 = f (1 )
Þ
1 1
0 < ò f ( x ) dx < ò e 2x - 1 dx Þ f( x ) < 0
-x
1/ 2 1/ 2
Þ e f (x ) < 0
1
1 æ e 2x - 1 ö \ f (x ) < 0
Þ 0<ò f ( x ) dx < ç ÷
1/ 2 è 2 ø 1/ 2 82. Here, f ( x ) = (1 - x ) × sin x + x 2 ³ 0, " x
2 2

1 e -1 x æ 2(t - 1 ) ö
Þ 0 < ò f ( x ) dx < and g( x ) = ò ç - log t ÷ f (t )dt
1/ 2 2 1 è t + 1 ø
79. To solve homogeneous differential equation, ì 2( x - 1 ) ü
i.e. substitute
y
=v Þ g ¢ (x ) = í - log xý × f{
(x ) …(i)
x î ( x + 1 ) þ + ve
dy dv For g ¢( x ) to be increasing or decreasing.
\ y = vx Þ =v + x
dx dx 2( x - 1 )
Let f( x ) = - log x
Here, slope of the curve at ( x, y ) is x+1
dy y æy ö 4 1 - (x - 1)
= + sec ç ÷ f¢ ( x ) = - =
dx x èxø (x + 1)2 x x (x + 1)2
y
Put =v f¢ ( x ) < 0, " x > 1
x Þ f ( x ) < f (1 ) Þ f( x ) < 0 …(ii)
dv
\ v+x = v + sec(v ) From Eqs. (i) and (ii), g ¢( x ) < 0, x Î (1, ¥ )
dx
\g( x ) is decreasing on x Î (1, ¥ ).
dv dv dx
= sec(v ) Þ ò
sec v ò x
Þ x = 83. Here, f ( x ) + 2x = (1 - x ) 2 × sin 2 x + x 2 + 2x …(i)
dx
dx where, I : f ( x ) + 2 x = 2(1 + x ) 2 …(ii)
Þ ò cosv dv = ò x Þ sinv = log x + log c \ 2(1 + x 2 ) = (1 - x ) 2 sin 2 x + x 2 + 2 x
æy ö Þ (1 - x ) 2 sin 2 x = x 2 - 2 x + 2
Þ sin ç ÷ = log(cx )
èxø
Þ (1 - x ) 2 sin 2 x = (1 - x ) 2 + 1
æ pö æpö
As it passes through ç1, ÷ Þ sin ç ÷ = log c Þ (1 - x ) 2 cos2 x = - 1
è 6ø è6ø
1 which is never possible.
Þ logc =
2 \I is false.
æy ö 1 Again, let h( x ) = 2 f ( x ) + 1 - 2 x(1 + x )
\ sin ç ÷ = log x +
èxø 2 where, h( 0 ) = 2 f ( 0 ) + 1 - 0 = 1
80. Let f( x ) = e -x f ( x ) h(1 ) = 2(1 ) + 1 - 4 = - 3 as [h( 0 )h(1 ) < 0 ]
Þ h( x ) must have a solution.
æ 1ö
Here, f¢ ( x ) < 0, x Î ç 0, ÷ \II is true.
è 4ø
306 Textbook of Integral Calculus

84. Linear differential equation under one variable. Alter


dy x
+ Py = Q; IF = e ò
P dx
Given, f ( x ) = ò f (t ) dt
dx 0

\ Solution is, y (IF) = ò Q × (IF)dx + C Þ f ( 0 ) = 0 and f ¢( x ) = f ( x )


If f ( x ) ¹ 0
y ¢- y tan x = 2 x sec x and y ( 0 ) = 0 f ¢( x )
Þ = 1 Þ ln f ( x ) = x + c
dy f (x )
Þ - y tan x = 2 x sec x
dx Þ f ( x ) = ec × e x
- tan x
\ IF = ò e dx = e log| cos x|
= cos x
Q f (0) = 0
Solution is y × cos x = ò 2 x sec x × cos x dx + C Þ ec = 0, a contradiction

Þ y × cos x = x 2 + C \ f ( x ) = 0, " x Î R Þ f (ln 5 ) = 0


2
As y( 0 ) = 0 dy y y - 1
87. Given, =
Þ C =0 dx x x 2 - 1
\ y = x 2 sec x dy dx
2 òy 2
y -1

x x2 - 1
æpö p
Now, yç ÷ =
è 4ø 8 2 Þ sec -1 y = sec -1 x + c
2 2
æpö p p æ p ö 2p 2 p p p
Þ y ¢ç ÷ = + Þ y ç ÷= At x = 2 , y = ; = +c Þ c=-
è 4ø 2 8 2 è3ø 9 3 6 3 6
2
æ p ö 4p 2p æ pö é 1 3ù
Þ y ¢ç ÷ = + Now, y = sec çsec-1 x - ÷ = cos ê cos-1 - cos-1 ú
è3ø 3 3 3 è 6ø ë x 2 û
dy é
85. + y × g ¢( x ) = g ( x ) g ¢( x ) æ 3 1 3 öù
dx = cos ê cos-1 ç + 1- 2 1- ÷ú
ë è 2 x x 4 øû
IF = e ò
g ¢ ( x ) dx
= e g (x )
3 1 1
y = + 1- 2
\ Solution is y (e g ( x ) ) = ò g( x ) × g ¢( x ) × e g ( x ) dx + C 2x 2 x
Put g( x ) = t, g ¢( x ) dx = dt 88. Given differential equation is
y (e g (x )
) = ò t × e dt + C = t × e - ò 1 × e dt + C
t t t y (1 + xy ) dx = x dy
Þ y dx + xy 2 dx = x dy
= t × et - et + C
x dy - y dx
y e g ( x ) = ( g( x ) - 1 ) e g ( x ) + C …(i) Þ = x dx
y2
Given, y ( 0 ) = 0, g( 0 ) = g(2 ) = 0 æxö
(y dx - x dy )
\ Eq. (i) becomes, Þ - = x dx Þ - d ç ÷ = x dx
y 2
èy ø
y ( 0 ) × e g ( 0) = ( g ( 0 ) - 1 ) × e g ( 0) + C
On integrating both sides, we get
Þ 0 = (- 1) × 1 + C Þ C = 1 x x2
\ y ( x ) × e g ( x ) = ( g( x ) - 1 ) e g ( x ) + 1 - = +C …(i)
y 2
Þ y (2 ) × e g ( 2) = ( g(2 ) - 1 ) e g ( 2) + 1, where g(2 ) = 0 Q It passes through (1, - 1 ).
Þ y(2 ) × 1 = ( - 1 ) × 1 + 1 1 1
\ +C ÞC =
1=
y(2 ) = 0 2 2
2
86. From given integral equation, f ( 0) = 0 . x x 1
Now, from Eq. (i) - = +
y 2 2
Also, differentiating the given integral equation w.r.t. x
2 2x 2x
f ¢( x ) = f ( x ) Þ x +1=- Þy = - 2
y x +1
If f (x ) ¹ 0 æ 1ö 4
\ f ç- ÷ =
f ¢( x ) è 2ø 5
Þ = 1 Þ log f ( x ) = x + c
f (x ) 89. Given differential equation is
Þ f (x ) = e e c x dy
( x log x ) + y = 2 x log x, (x ³ 1)
c
dx
Q f ( 0 ) = 0 Þ e = 0, a contradiction dy y
Þ + =2
∴ f ( x ) = 0, " x Î R Þ f (ln 5 ) = 0 dx x log x
Chap 04 Differential Equations 307

This is a linear differential equation. To find The time at which the population of the mouse will
1 become zero, i.e. to find the value of ‘t’ at which p(t ) = 0.
ò x log x dx
\ IF = e = e log(log x ) = log x Let’s solve the differential equation first
dp (t )
Now, the solution of given differential equation is given by p ¢(t ) = = 0.5 p (t ) - 450
dt
y × log x = ò log x × 2dx 2dp (t )
Þ = dt
Þ y × log x = 2 ò log x dx p (t ) - 900
2dp (t )
Þ y × log x = 2[ x log x - x ] + c
Þ ò p (t ) - 900 = ò dt
At x = 1, c = 2 Þ 2 log | p (t ) - 900 | = t + C , where C is the constant of
Þ y × log x = 2[ x log x - x ] + 2 integration.
At x = e, To find the value of ‘C ’, let’s substitute t = 0.
Þ 2 log | p( 0 ) - 900 | = 0 + C
y = 2(e - e ) + 2
Þ C = 2 log | 850 - 900 |
Þ y =2
Þ C = 2 log 50
dp 1
90. Given differential equation - p(t ) = -200 is a linear Now, substituting the value of C back in the solution, we get
dt 2
differential equation. 2 log | p(t ) - 900 | = t + 2 log 50
-1 Here, since we want to find the value of t at which p(t ) = 0,
Here, p (t ) = , Q(t ) = -200 hence substituting p(t ) = 0, we get
2
æ1 ö t 2 log | 0 - 900 | = t + 2 log 50
ò - çè 2 ÷ødt -
IF = e =e 2 900
Þ t = 2 log
Hence, solution is 50
p (t ). IF = ò Q(t ) IF dt Þ t = 2 log 18
t t dy
- - 93. Here, = y + 3 > 0 and y( 0 ) = 2
p (t ) × e 2 =ò -200 × e 2dt dx
t t
- - dy
p (t ) × e 2 = 400 e 2 +K Þ ò y + 3 = ò dx
Þ p (t ) = 400 + ke -1/2
Þ log | y + 3 | = x + C
If p( 0 ) = 100, then k = - 300
t Since, y( 0 ) = 2
Þ p (t ) = 400 - 300 e 2 Þ loge | 2 + 3 | = 0 + C
dP
91. Given, = (100 - 12 x ) \ C = loge 5
dx
Þ loge | y + 3 | = x + loge 5
Þ dP = (100 - 12 x ) dx
When x = loge 2
On integrating both sides, we get
Þ loge | y + 3 | = loge 2 + loge 5 = loge 10
ò dP = ò (100 - 12 x ) dx
Þ y + 3 = 10
P = 100 x - 8 x 3/2 + C
Þ y =7
When x = 0, then P = 2000
d {V (t )}
Þ C = 2000 94. Given, = - k (T - t )
dt
Now, when x = 25, then
\ d {V (t )} = - k (T - t ) dt
P = 100 ´ 25 - 8 ´ (25 ) 3/2 + 2000
On integrating both sides, we get
= 2500 - 8 ´ 125 + 2000 (T - t ) 2
V (t ) = -k +C
= 4500 - 1000 = 3500 2( -1 )
92. Given k
(i) The population of mouse at time ‘t’ satisfies the Þ V (t ) = (T - t ) 2 + C
2
dp(t )
differential equation p ¢(t ) = = 0.5 p(t ) - 450 Q At t = 0, V (t ) = I
dt
k
(ii) Population of mouse at time t = 0 is \ I = (T - 0 ) 2 + C
2
p( 0 ) = 850
308 Textbook of Integral Calculus

k 96. Given, y = c1ec 2 x


Þ C = I - T2
2 Þ y ¢ = c 2c1ec 2 x
k k
\ V (t ) = (T - t ) 2 + I - T 2 Þ y ¢ = c 2y …(i)
2 2
k 2 Þ y ¢¢ = c 2y ¢ …(ii)
Now, V (T ) = I - T
2 (y ¢ ) 2 é y ¢ù
Þ y ¢¢ = ê from Eq. (i), c 2 = ú
95. Since, cos x dy = y sin x dx - y 2 dx y ë yû

1 dy 1 Þ yy ¢¢ = (y ¢ ) 2
Þ - tan x = - sec x
y 2 dx y 97. Equation of circle having centre (h, k) and radius a is
1 ( x - h ) 2 + (y - k ) 2 = a 2.
Put - = z
y The equation of family of circles with centre on y = 2 and of
1 dy dz radius 5 is
Þ = (x - a)2 + (y - 2)2 = 52 …(i)
y 2 dx dx
dz Þ x 2 + a 2 - 2 ax + y 2 + 4 - 4y = 25
Þ + (tan x ) z = - sec x
dx On differentiating w.r.t. x, we get
This is a linear differential equation. dy dy
2 x - 2 a + 2y -4 =0
Therefore, dx dx
IF = e ò tan x dx = e log sec x = sec x dy
⇒ a=x+ (y - 2)
Hence, the solution is dx
On putting the value of a in Eq. (i), we get
z × (sec x ) = ò - sec x × sec x dx + C1 2
é dy ù 2 2
1 êë x - x - dx ( y - 2 ) úû + ( y - 2 ) = 5
Þ - sec x = - tan x + C1
y 2
æ dy ö
⇒ 2
ç ÷ ( y - 2 ) = 25 - ( y - 2 )
2
Þ sec x = y (tan x + C ) è dx ø
⇒ y ¢2( y - 2 ) 2 = 25 - ( y - 2 ) 2

You might also like